Vous êtes sur la page 1sur 458

les maths

en tête

ANALYSE
3e
édition

Xavier GOURDON
les maths
en tête

ANALYSE
les maths
en tête

ANALYSE
3e
édition

Xavier GOURDON
Ancien élève de l’école polytechnique
Du même auteur chez le même éditeur

Algèbre, Les maths en tête, 2e édition, 304 pages.

ISBN 9782340-038561
© Ellipses Édition Marketing S.A., 2020
32, rue Bargue 75740 Paris cedex 15
Avant-propos de la troisième édition

Près de ving-cinq ans après la première édition et douze ans après la seconde édition,
cette troisième édition contient de nouveaux exercices et problèmes, ainsi que des ex-
tensions d’exercices. Ces ajouts sont toujours réalisés au travers de thèmes riches pour
l’étudiant, soit parce que les résultats démontrés sont classiques, soit parce que l’esprit
de leur résolution est intéressant par la méthode qu’il permet d’acquérir. On trouve aussi
quelques corrections de coquilles, qui restaient présentes malgré le soin porté aux retouches
de la deuxième édition.
xavier.gourdon livres@yahoo.fr
Avant-propos de la deuxième édition

Plus de dix ans après la sortie de la première édition, cet ouvrage est encore large-
ment utilisé pour la préparation des concours (grandes écoles, agrégation). Les retours de
mes lecteurs (grâce à internet notamment) sont nombreux et m’ont encouragé à réaliser
cette deuxième édition. Cette dernière contient des corrections de coquilles, quelques re-
touches et améliorations de preuves et solutions, ainsi que quelques exercices et problèmes
supplémentaires dans l’esprit de l’édition précédente. Par ailleurs, le chapitre intégration a
été largement revu pour être plus conforme au programme des classes préparatoires scien-
tifiques. Enfin, une nouvelle annexe (annexe C) propose une démonstration du théorème
des nombres premiers accessible à partir du programme des classes préparatoires.
J’espère que les améliorations de cette deuxième édition seront utiles à mes lecteurs
pour la préparation de leur concours. Je tiens à remercier chaleureusement ceux qui m’ont
aidé, directement ou indirectement, et en particulier les internautes dont les commentaires
ont été nombreux.
Avant-propos de la première édition

Cet ouvrage propose aux étudiants des classes de mathématiques spéciales (programme
M’) des rappels et des compléments de cours complets, ainsi que des exercices et des
problèmes corrigés. Il pourra également intéresser les élèves préparant l’agrégation.
L’ouvrage est orienté dans le même esprit que le tome Algèbre : le lien étroit qui existe
entre le cours et les exercices est mis en avant. En effet, une bonne compréhension du cours
passe nécessairement par la résolution d’exercices, et réciproquement, il est illusoire de
s’attaquer à des exercices difficiles sans avoir une compréhension profonde du cours. Dans
cet esprit, de multiples remarques ponctuent les parties de cours, mettant en avant ses
subtilités, et faisant le lien avec les exercices qui suivent.
Les parties de cours ne sont pas un substitut au cours du professeur, mais plutôt un
résumé exhaustif qui l’éclaire d’une façon différente. Les compléments sont des résultats
très classiques qui ne figurent pas au programme mais dont la connaissance est utile
et parfois indispensable pour mener à bien un exercice ou un problème. Les résultats
présentés sont démontrés lorsqu’ils sont à la limite du programme ou lorsqu’ils constituent
un point important dont la démonstration met en place des techniques instructives que
l’étudiant doit connaı̂tre et savoir maı̂triser.
À la fin de chaque section, on trouve une liste d’exercices de difficultés progressives,
classiques ou parfois originaux, qui constituent une illustration du cours qui les précède.
Je me suis efforcé à chaque fois de passer en revue tous les problèmes qui tournent autour
du thème de l’exercice. Les nombreuses références au cours sont là pour inviter le lecteur
à s’y reporter, le but étant de savoir et de comprendre précisément les résultats que l’on
utilise.
Une liste de problèmes ponctue la fin de chaque chapitre, ces problèmes étant des
exercices plus longs, plus difficiles ou plus originaux que les précédents et faisant appel à
l’ensemble du cours du chapitre. À la fin de certains chapitres, on trouve des sujets d’étude
introduisant des théories élégantes dans le thème du chapitre. Deux annexes présentent
des curiosités mathématiques liées au programme d’analyse.
Les résultats du cours ou les exercices les plus importants sont indiqués par une flèche
dans la marge de gauche.
Je tiens à remercier toutes les personnes qui m’ont aidé, Georges Papadopoulo, Ber-
trand Saint-Aubin et Alexia Stéfanou pour la relecture de certains chapitres, Bénédicte
Herbinet à qui je dois une élégante solution, Philippe Flajolet pour sa contribution aux
énoncés de quelques problèmes. Ce travail a pu se concrétiser grâce au projet Algo-
rithmes qui m’a permis de donner à l’ouvrage sa version typographique actuelle et à la
collection ellipses qui l’a accueilli, et je les en remercie.
Je serais reconnaissant à ceux de mes lecteurs qui me feront parvenir leurs remarques
sur cette première édition.
Xavier Gourdon
Table des matières

Avant-propos de la troisième édition .............................................................. 1

Avant-propos de la deuxième édition ............................................................. 2

Avant-propos de la première édition ............................................................... 3

Chapitre 1. Topologie sur les espaces métriques et les espaces vectoriels


normés ........................................................................................................... 7
1. Généralités ............................................................................................ 7
2. Suites dans un espace métrique .......................................................... 19
3. Espaces compacts ............................................................................... 27
4. Espaces connexes ................................................................................ 38
5. Espaces vectoriels normes (e.v.n) ........................................................ 47
6. Problèmes ........................................................................................... 57

Chapitre 2. Fonctions d’une variable réelle ................................................. 71


1. Fonctions dérivables ............................................................................ 71
2. Développements limités et développements asymptotiques ................ 87
3. Fonctions convexes, fonctions réglées ................................................. 95
4. Problèmes ......................................................................................... 103

Chapitre 3. Intégration ............................................................................... 123


1. Intégrale sur un segment de R .......................................................... 123
2. Calcul de primitives .......................................................................... 136
3. Intégrale sur un intervalle quelconque ...............................................147
4. Intégrales dépendant d’un paramètre, équivalents d’intégrales ..........161
5. Problèmes ..........................................................................................176

Chapitre 4. Suites et séries ......................................................................... 199


1. Suites numériques ............................................................................. 199
2. Séries numériques ............................................................................. 208
3. Suites et séries de fonctions ...............................................................231
4. Séries entières ................................................................................... 247
5. Séries de Fourier ............................................................................... 267
6. Problèmes ......................................................................................... 281
7. Sujets d’étude .....................................................................................315

Chapitre 5. Fonctions de plusieurs variables .............................................. 323


1. Différentielle, dérivees partielles ...................................................... 323
2. Extremums relatifs .............................................................................335
3. Inversion locale, fonctions implicites .................................................341
4. Intégrales multiples, intégrales curvilignes ........................................351
5. Problèmes ......................................................................................... 363

Chapitre 6. Equations différentielles ......................................................... 373


1. Généralités ........................................................................................ 373
2. Equations différentielles linéaires .................................................... 377
3. Equations différentielles non linéaires .............................................. 390
4. Quelques compléments ..................................................................... 394
5. Problèmes ......................................................................................... 407

Annexe A. Théorème de Baire et applications ...........................................417


Le théorème de Baire ............................................................................417
Applications ........................................................................................... 419

Annexe B. Espaces de Hilbert ................................................................... 427


1. Résultats généraux sur les espaces de Hilbert ................................... 427
2. Quelques propriétés des espaces de Hilbert ...................................... 430

Annexe C. Théorème des nombres premiers ............................................. 437


1. Préliminaires .................................................................................... 437
2. Preuve du Théorème des nombres premiers ..................................... 443
3. Histoire du Théorème des nombres premiers ................................... 445
Index des notations ............................................................................... 447

Index........................................................................................................... 449
CHAPITRE 1

Topologie sur les espaces métriques


et les espaces vectoriels normés

Lanombres
naissance de la topologie est directement liée à l’étude des ensembles de
réels. Un premier signe fut certainement la définition de la notion de
point d’accumulation par Weierstrass vers 1860 (qui démontra que tout en-
semble de nombres réels infini borné admet au moins un point d’accumulation,
résultat admis auparavant).
Ce point de vue un peu étroit tomba ensuite en désuétude. Ce n’est qu’en
1906, à force d’étudier des ensembles de plus en plus abstraits, qu’apparut
la notion de distance, introduite par Fréchet. La notion d’espace topologique
général ne naquit qu’en 1914 grâce à Hausdorff qui définit la notion de voisi-
nage.
Le développement des espaces vectoriels normés (en particulier de dimen-
sions infinies) est d’abord dû à Hilbert ; Banach compléta largement cette
théorie dans les années 1930.
La notion d’ensemble compact, en germe dès 1900, se développa avec Borel
et Lebesgue grâce aux considérations liées à la théorie de la mesure.

La théorie générale des espaces topologiques n’étant pas au programme des classes de
mathématiques spéciales, nous nous limiterons à l’étude des espaces métriques. Les
curieux trouveront cependant leur compte dans les différentes remarques des parties de
cours.
En annexe, sont présentés sous forme de problèmes et d’exercices :
— Le théorème de Baire et quelques applications (annexe A).
— Quelques résultats sur les espaces de Hilbert (annexe B).
Ces curiosités ne sont pas au programme de mathématiques spéciales, mais elles consti-
tuent de forts jolies théories qui nous sont accessibles.

1. Généralités
1.1. Normes et Distances
Normes.
Définition 1. Soit E un K-espace vectoriel (où K = R ou C). Une norme sur E est une
application E → R + x → x telle que
(i) On a x = 0 si et seulement si x = 0.
(ii) Pour tout λ ∈ K, pour tout x ∈ E , λx = |λ| · x.
(iii) Pour tout (x, y ) ∈ E 2 , x + y ≤ x + y  (inégalité triangulaire).
Muni d’une norme, E est appelé un K-espace vectoriel normé (en abrégé e.v.n).
Exemple 1. — x → |x| est une norme sur R, z → |z | est une norme sur C.
8 1. TOPOLOGIE SUR LES ESPACES M ÉTRIQUES ET LES ESPACES VECTORIELS NORMÉS

— Dans R n, en notant x = (x 1, . . . , xn), on a les normes classiques suivantes



 
x1 = |xi |, x2 = x2i , x∞ = sup |x i |.
i
i i

Plus généralement, pour tout α ≥ 1, x α = ( i |xi |α )1/α est une norme sur Rn
(voir la conséquence de l’inégalité de Minkowsky, page 98).
— L’ensemble B(X, E ) des applications bornées d’un ensemble X dans un e.v.n E est
un espace vectoriel normé muni de la norme f  ∞ = supx∈X |f (x)| (cette norme
s’appelle norme de la convergence uniforme.)
Remarque 1. Lorsque seules les propriétés (ii) et (iii) de la définition sont vérifiées, on dit
que  ·  est une semi-norme.
Distances.
Définition 2. Soit E un ensemble. On appelle distance sur E toute application d : E ×
E → R+ telle que :
(i) d(x, y) = 0 si et seulement si x = y .
(ii) Pour tout x, y ∈ E , d(x, y ) = d(y, x) (symétrie).
(iii) Pour tout x, y , z ∈ E , d(x, z) ≤ d(x, y) + d(y, z) (inégalité triangulaire).
Muni d’une distance, E est appelé espace métrique.
Exemple 2. — Si E est un e.v.n, d(x, y ) = x − y  définit une distance sur E , qui fait
de l’e.v.n E un espace métrique. Sauf mention contraire, c’est cette distance que
l’on choisit toujours dans un e.v.n.
— Sur tout ensemble E , la distance d définie par
d(x, y ) = 0 si x = y, d(x, y) = 1 si x = y
est appelée distance discrète sur E. L’espace métrique (E, d) est alors appelé espace
discret.
Diamètre d’une partie, distance entre deux parties.
Définition 3. Soit (E, d) un espace métrique. Si A ⊂ E , A = ∅, on appelle diamètre de
A l’élément de [0, +∞] défini par
δ (A) = sup d(x, y ).
(x,y)∈A 2

On dit que A est bornée si A = ∅ ou si δ (A) < +∞.


Définition 4. Soient A et B deux parties non vides d’un espace métrique (E, d). On
appelle distance de A à B le réel
d(A, B ) = inf d(x, y ).
x∈A
y∈B

Lorsque x est un élément de E, on appelle distance de x à A le réel


d(x, A) = d({x}, A) = inf d(x, y ).
y∈A

Remarque 2. Attention ! Avec cette définition, l’application d : (P (E ) {∅})2 (A, B ) →


d(A, B ) n’est pas une distance sur P (E )  {∅} (on peut avoir d(A, B) = 0 avec A = B ).
1. G ÉNÉRALIT ÉS 9

Boules et sphères.
Définition 5. Soit (E, d) un espace métrique. Pour tout x ∈ E et pour tout ρ > 0, on
appelle
— boule ouverte de centre x de rayon ρ l’ensemble B(x, ρ) = {y ∈ E | d(x, y) < ρ},
— boule fermée de centre x de rayon ρ l’ensemble Bf (x, ρ) = {y ∈ E | d(x, y ) ≤ ρ},
— sphère de centre x de rayon ρ l’ensemble S(x, ρ) = {y ∈ E | d(x, y) = ρ}.
Lorsque E est un espace vectoriel normé (muni de la distance issue de la norme) et que
x = 0, ρ = 1, on parle de boule unité ouverte, boule unité fermée et de sphère unité.
Proposition 1. Soit (E, d) un espace métrique, A une partie de E , et x ∈ E . L’ensemble
A est borné si et seulement s’il existe r > 0 tel que A ⊂ B(x, r ).
1.2. Topologie d’un espace métrique
Sauf mention contraire, dans toute cette sous
sous-suite
partie, (E, d) désigne un espace métrique.
Ouverts.
Définition 6. Une partie Ω de E est dite ouverte (ou Ω un ouvert ) si Ω = ∅ ou si
∀x ∈ Ω, ∃ρ > 0 tel que B(x, ρ) ⊂ Ω.
L’ensemble des parties ouvertes de E s’appelle topologie de E.
Exemple 3. Une boule ouverte est un ouvert. En particulier, dans R (muni de la distance
usuelle d(x, y) = |x − y |), les intervalles ouverts ]α, β [ sont des ouverts.
Proposition 2. (i) Les parties ∅ et E sont des ouverts.
(ii) Une réunion d’ouverts est un ouvert.
(iii) Une intersection finie d’ouverts est un ouvert.
Remarque 3. Attention, une intersection infinie d’ouverts peut ne pas être ouverte. Par
exemple, dans R, ∩n∈N∗ ]−1/n, 1/n[ = {0} n’est pas ouvert.
Fermés.
Définition 7. Une partie F de E est dite fermée (ou F un fermé ) si E  F est ouvert.
Exemple 4. Une boule fermée est un fermé. En particulier, un singleton {x} = B f (x, 0)
est un fermé. Dans R, les intervalles fermés [α, β] sont des fermés.
Proposition 3. (i) Les parties ∅ et E sont des fermés.
(ii) Une intersection de fermés est un fermé.
(iii) Une réunion finie de fermés est un fermé.
Remarque 4. Attention, une réunion infinie de fermés peut ne pas être fermée. Par
exemple, dans R, ∪n∈N∗ [1/n, 1 − 1/n] = ]0, 1[ n’est pas un fermé.
Voisinages.
Définition 8. On appelle voisinage d’un élément x de E toute partie V de E contenant
un ouvert contenant x. L’ensemble des voisinages de x est noté V (x).
Exemple 5. Un ouvert contenant x est un voisinage de x, une boule fermée de centre x
de rayon ρ > 0 est un voisinage de x.
Remarque 5. Une réunion (resp. une intersection finie) de voisinages de x est un voisinage
de x.
10 1. TOPOLOGIE SUR LES ESPACES MÉTRIQUES ET LES ESPACES VECTORIELS NORM ÉS

Commentaire sur les espaces topologiques généraux.


Un espace topologique général E est défini comme étant un ensemble muni d’une partie
de P (E ) dont les éléments sont appelés des ouverts et vérifient les axiomes (i), (ii) et (iii)
de la proposition 2. On définit alors les fermés comme à la définition 7 et les voisinages
comme à la définition 8.
Toutes les notions de cette partie 1.2 peuvent être étendues aux espaces topologiques.
Il existe pour les espaces topologiques généraux une notion importante appelée la
séparation. Un espace topologique E est dit séparé si pour tous éléments x, y ∈ E , x = y ,
il existe V ∈ V(x) et W ∈ V(y) tels que V ∩ W = ∅. On voit facilement que tout espace
métrique est séparé.
Adhérence.
Définition 9. L’adhérence d’une partie A de E , notée A, est le plus petit ensemble fermé
contenant A.
Remarque 6. — L’ensemble A existe, c’est l’intersection des fermés contenant A.
— Une partie A est fermée si et seulement si A = A.
Proposition 4. Soit A une partie de E . Un élément x de E est dans A si et seulement
si l’une des assertions suivantes est vérifiée :
(i) ∀ε > 0, ∃a ∈ A, d(a, x) < ε.
(ii) Pour tout voisinage V de x, V ∩ A = ∅.
(iii) d(x, A) = 0.
Exemple 6. — Dans R, l’adhérence de tout intervalle ouvert borné ]α, β[ est [α, β ].
— Dans un e.v.n, on a B(0, 1) = B f (0, 1), propriété fausse dans un espace métrique
général (voir l’exercice 1).
— Si A est fermé, on a
x ∈ A ⇐⇒ x ∈ A ⇐⇒ d(x, A) = 0.
Définition 10. Une partie A de E est dite dense dans E si A = E .
Exemple 7. En utilisant la proposition précédente, on voit facilement qu’une partie A de
R est dense dans R (R étant muni de la distance usuelle) si et seulement si
(∀(a, b) ∈ R2 , a < b), ]a, b[ ∩ A = ∅.
Par exemple, Q et R  Q sont denses dans R.
Intérieur.

Définition 11. L’intérieur d’une partie A de E , noté A, est le plus grand ouvert contenu
dans A.
Remarque 7. — L’intérieur de A existe : c’est la réunion des ouverts contenus dans
A. ◦
— Une partie A de E est ouverte si et seulement si A = A.

◦   
— Pour toute partie A de E, A = E (E A) et A = E( E A).

Proposition 5. Soit A une partie de E et x un élément de A. On a x ∈ A si et seulement
si l’une des assertions suivantes est vérifiée.
(i) A est un voisinage de x.
(ii) Il existe ε > 0 tel que B(x, ε) ⊂ A.
Exemple 8. Dans R, l’intérieur de [α, β] est ]α, β [ ; l’intérieur de Q, de RQ, est ∅.
1. G ÉNÉRALIT ÉS 11

Frontière.

Définition 12. La frontière d’une partie A de E est l’ensemble A A. On la note Fr(A)
(ou encore ∂A).
Point d’accumulation, point isolé.
Définition 13. Soit A une partie de E .
— On dit que a ∈ E est un point d’accumulation de A si pour tout voisinage V de a,
V ∩ A = ∅ et V ∩ A =
 {a}, ce qui s’écrit encore
(∀ε > 0), B(a, ε) ∩ A = ∅ et = {a}.
— On dit que a ∈ A est un point isolé de A s’il existe un voisinage V de a tel que
V ∩ A = {a}, ce qui s’écrit encore
(∃ε > 0), B(a, ε) ∩ A = {a}.
Remarque 8. Si a est un point d’accumulation de A, alors a ∈ A et de plus pour tout
ε > 0, B(a, ε) contient une infinité de points de A.
Exemple 9. Dans R, 0 est point d’accumulation de l’ensemble {1/n, n ∈ N ∗}.
Topologie induite dans un espace métrique. Soit (E, d) un espace métrique et
A ⊂ E . Une manière bien naturelle de faire de A un espace métrique est de le munir de
la restriction de la distance d de E à A × A. Ainsi, (A, d) est un espace métrique dont
la topologie est appelée topologie induite par (E, d). La proposition suivante permet de
caractériser les ouverts, fermés et voisinages de A par rapport à ceux de E.
Proposition 6. Soit A une partie de E .
— Les ouverts de A sont les ensembles de la forme Ω ∩ A, Ω étant un ouvert de E .
— Les fermés de A sont les ensembles de la forme F ∩ A, où F est un fermé de E .
— Si a ∈ A, les voisinages de a dans A sont les ensembles de la forme V ∩ A, V étant
un voisinage de a dans E .
Exemple 10. L’ensemble [0, 1[ est un ouvert de A = [0, 2] (on peut écrire par exemple
[0, 1[ = ] − 1, 1[ ∩A).
1.3. Continuité
Applications continues.
Définition 14. Soient (E, d) et (E , d ) deux espaces métriques, et f : E → E une
application. On dit que f est continue en a ∈ E si pour tout voisinage W de f (a), il
existe un voisinage V de a tel que f (V ) ⊂ W . Lorsque f est continue en tout point de E,
on dit que f est continue sur E.
Proposition 7. Soient (E, d) et (E , d ) deux espaces métriques, et soit f : E → E  une
application. Alors f est continue en a ∈ E si et seulement si
(∀ε > 0, ∃α > 0, ∀x ∈ E ), (d(a, x) < α =⇒ d(f (a), f (x)) < ε).
Proposition 8. Soient (E, d), (E  , d ), (E  , d ) trois espaces métriques, et deux appli-
cations f : E → E et g : E  → E . Si f est continue en a ∈ E et g continue en f (a),
alors l’application g ◦ f : E → E  est continue en a.

Proposition 9. Soit f : (E, d) → (E , d  ) une application. Les trois assertions suivantes


sont équivalentes.
(i) f est continue sur E .
12 1. TOPOLOGIE SUR LES ESPACES MÉTRIQUES ET LES ESPACES VECTORIELS NORM ÉS

(ii) L’image réciproque par f de tout ouvert de E est un ouvert de E .


(iii) L’image réciproque par f de tout fermé de E est un fermé de E .
Remarque 9. Lorsque l’image de tout ouvert par f est un ouvert, on dit que f est une
application ouverte. Une application continue n’est pas forcément ouverte (considérer par
exemple une fonction constante sur R). De même, l’image d’un fermé par une application
continue n’est pas forcément fermée. Par exemple,
x
f : R→R x →
1 + |x|
est continue et f (R) = ] − 1, 1[ n’est pas fermé.
Homéomorphismes.
Définition 15. Soit une application F : (E, d) → (E , d ). On dit que f est un
homéomorphisme si f est bijective, continue, et si f −1 est continue.
Remarque 10. Une application peut être continue et bijective sans que l’application
réciproque ne soit continue. Par exemple, l’application identité f de (R, ddis ) dans (R, d)
(ddis désignant la distance discrète sur R, d la distance usuelle) est continue mais f −1 n’est
pas continue. (Cependant, on sait que si f : (R, d) → (R, d) est continue et bijective,
alors f −1 est continue. Sous certaines hypothèses de compacité, il est également possible
de conclure à la continuité de l’application réciproque — voir la proposition 14 page 31.)
Définition 16. Deux distances d et d sur E sont dites topologiquement équivalentes si
elles définissent la même topologie (i. e. si les ouverts de (E, d) sont des ouverts de (E, d)
et réciproquement).
Proposition 10. Deux distances d et d sur E sont topologiquement équivalentes si et
seulement si l’application identité de (E, d) sur (E, d ) est un homéomorphisme.
Remarque 11. Si d et d  sont topologiquement équivalentes, les espaces métriques (E, d)
et (E, d ) possèdent les mêmes propriétés topologiques (en effet, les ouverts de ces deux
espaces métriques coı̈ncident, et il en est donc de même pour les fermés et les voisinages).
Normes et distances équivalentes.
Définition 17. — Deux normes N1 et N2 sur un même e.v E sont dites équivalentes
s’il existe a > 0 et b > 0 tels que pour tout x ∈ E , a N 1(x) ≤ N 2(x) ≤ b N 1 (x).
— Deux distances d1 et d 2 sur E sont dites équivalentes s’il existe a > 0 et b > 0 tels
que pour tout x, y ∈ E , a d1 (x, y ) ≤ d 2 (x, y ) ≤ b d 1(x, y).
Remarque 12. — Deux normes équivalentes induisent deux distances équivalentes.
— Deux distances équivalentes sont topologiquement équivalentes. Ainsi, les résultats
de nature topologiques sont indépendant du choix de l’une ou l’autre des distances.
— On verra plus loin (voir le théorème 3 page 50) que sur un e.v de dimension finie,
toutes les normes sont équivalentes.
Applications uniformément continues.
Définition 18. Une application f : (E, d) → (E , d ) est dite uniformément continue
sur E si
(∀ε > 0, ∃α > 0, ∀x, y ∈ E ), (d(x, y ) < α =⇒ d(f (x), f (y)) < ε).
Remarque 13. — Une fonction uniformément continue est continue ; la nuance entre
ces deux notions est qu’une fonction uniformément continue vérifie d (f (x), f (y)) <
ε pour tous les couples (x, y ) tels que d(x, y ) < α, α étant indépendant de x, alors
que pour une fonction continue, α dépend de x. L’uniformité de cet α > 0 pour une
1. G ÉNÉRALIT ÉS 13

fonction uniformément continue f en fait une fonction souple d’emploi. Du coup,


certains théorèmes sont vrais pour les fonctions uniformément continues mais pas
pour les fonctions continues. Nous verrons cependant que toute fonction continue
sur un compact y est uniformément continue (voir le théorème 2 page 31).
— Attention ! L’uniforme continuité n’est pas une notion topologique. Autrement dit,
la seule définition de la topologie de E et E  ne suffit pas à définir l’uniforme conti-
nuité. En particulier, une fonction uniformément continue vis-à-vis d’une certaine
distance ne l’est pas forcément vis-à-vis d’une distance topologiquement équivalente.
Par contre, une fonction uniformément continue de (E, d1) dans (E  , d 1), lorsqu’elle
est regardée comme une fonction de (E, d2) dans (E  , d2 ), reste uniformément conti-
nue lorsque les distances d 1, d 2 et d 1, d 2 sont équivalentes, ou lorsqu’elles sont uni-
formément équivalentes (voir la définition qui suit).
Exemple 11. — Une fonction f : (E, d) → (E , d  ) lipschitzienne, c’est-à-dire vérifiant
(∃k > 0, ∀x, y ∈ E ), d (f (x), f (y)) ≤ k d(x, y ),
est uniformément continue.
— La fonction f : ]0, 1] → R x → 1/x est continue mais n’est pas uniformément
continue.
La fin de la remarque précédente motive la définition suivante.
Définition 19. Deux distances d et d  sur E sont dites uniformément équivalentes si
l’application identité est uniformément continue de (E, d) dans (E, d ) et de (E, d ) dans
(E, d).
Remarque 14. Deux distances équivalentes sont uniformément équivalentes. Deux dis-
tances uniformément équivalentes sont topologiquement équivalentes.
1.4. Produit d’espaces métriques
On se donne un nombre fini n d’espaces métriques (E 1 , d1 ), . . . , (E n , dn) et on pose
E = E1 × · · · × En . On veut faire de E un espace métrique. Un moyen naturel est de
construire une distance d sur E à partir des distances di . Par exemple si x = (x1 , . . . , xn)
et y = (y 1, . . . , y n ) ∈ E, d(x, y) = max 1≤i≤n d i(x i, yi ) définit une distance sur E. Cette
distance est appelée distance produit sur E. Sauf mention contraire, c’est cette distance
que nous utiliserons sur un produit d’espaces métriques.
Remarque 15. — En posant

n  n
 
d(x, y) = d i(x i, y i ) et d (x, y) =  d i (xi , yi )2 ,
i=1 i=1

on a également affaire à des distances sur E . Ces distances sont équivalentes à la


distance produit d, car
∀x, y ∈ E, d(x, y) ≤ d (x, y ) ≤ d (x, y ) ≤ n d(x, y ).
Il est donc indifférent de travailler avec l’une ou l’autre de ces distances. C’est parce
que la distance produit est plus souple d’utilisation que nous l’avons choisie.
— Au sens de la distance produit d, la boule ouverte de centre a = (a1 , . . . , an ) de
rayon r > 0 vérifie
B(a, r ) = B(a1 , r) × · · · × B(an , r).
Proposition 11. Si O 1, . . . , On sont des ouverts de E1, . . . , E n , le produit O1 × · · · × On
est un ouvert de E appelé ouvert élémentaire.
14 1. TOPOLOGIE SUR LES ESPACES MÉTRIQUES ET LES ESPACES VECTORIELS NORM ÉS

Un ouvert de E n’est en général pas un ouvert élémentaire.


Proposition 12. La projection d’indice i, définie par
Pri : E = E 1 × · · · × En → Ei (x 1, . . . , x n ) → xi
est une application continue et ouverte (une application est dite ouverte si l’image de tout
ouvert par cette application est un ouvert).
Proposition 13. Une application
f : (F, δ) → E1 × · · · × E n x → f (x) = (f1(x), · · · , f n(x)).
est continue en a ∈ F si et seulement si pour tout i, fi = Pr i ◦f est continue en a.
Proposition 14. Soit une application f : E = E1×· · ·×E n → F et a = (a1 , . . . , an ) ∈ E.
Pour tout i, on note
fi : E i → F x → f (a1, . . . , a i−1, x, a i+1, . . . , an )
(fi est appelée application partielle d’indice i au point a). Si f est continue en a, alors
pour tout i, l’application partielle fi est continue en ai .
Remarque 16. Attention ! La réciproque de ce dernier résultat est fausse. En d’autres
termes, il se peut que tous les fi soient continues en ai sans que f soit continue en a. Par
exemple, considérons l’application f : R2 → R définie par
xy
f (0, 0) = 0 et ∀(x, y ) = (0, 0), f (x, y) = 2 .
x + y2
Les applications partielles en (0, 0) sont nulles, donc continues, et pourtant f n’est pas
continue en (0, 0) (sinon, l’application ϕ : x → f (x, x) serait continue — composée
d’applications continues — ce qui est faux puisque ϕ(0) = 0 et ϕ(x) = 1/2 dès que
x = 0).
Continuité de la distance.
Proposition 15. Soit (E, d) un espace métrique. Alors l’application distance d : E ×
E → R est lipschitzienne de rapport 2, en particulier continue.
Conséquence : Soit a ∈ E et r > 0. L’application ϕ : E → R x → d(a, x) est
continue d’après les deux dernières propositions. On en déduit que Bf (a, r) = ϕ −1([0, r])
et S(a, r) = ϕ −1 ({r }) (sphère de centre a de rayon r ), images réciproques de fermés par
une application continue, sont des fermés de E . On retrouve de même qu’une boule ouverte
est un ouvert.
Continuité des opérations dans un e.v.n.
Proposition 16. Soit E un e.v.n sur K (avec K = R ou C). Les applications
E×E →E (x, y ) → x + y et K × E → E (λ, x) → λ · x
sont continues.
Algèbre normée.
Définition 20. On dit qu’une norme  ·  sur une K-algèbre A (avec K = R ou C) est
une norme d’algèbre si xy ≤ x · y  pour tout (x, y ) ∈ A 2. Munie d’une telle norme,
A est appelée algèbre normée. L’application A × A → A (x, y ) → xy est alors continue.
Proposition 17. Soit A un K-e.v.n (K = R ou C) et f, g : (E, d) → A deux applications
continues en a ∈ E . Alors les applications f + g , λf (pour tout λ ∈ K fixé) sont continues
en a. Si A est une algèbre normée, l’application f g est continue en a.
1. G ÉNÉRALIT ÉS 15

1.5. Limites
Définition 21. Soient (E, d) et (F, δ) deux espaces métriques et une application f : D ⊂
E → F . Soient A ⊂ D et a ∈ A,  ∈ F . On dit que f (x) tend vers  quand x tend vers a
selon A, et on note lim
x→a
f (x) = , si pour tout voisinage W de , il existe un voisinage V
x∈A
de a tel que f (A ∩ V ) ⊂ W . Ceci s’écrit aussi
∀ε > 0, ∃α > 0 tel que ∀x ∈ A vérifiant d(a, x) < α, on a δ (f (x), ) < ε.
Remarque 17. Si  existe,  est unique et on a  ∈ f (A) ;  est alors appelée la limite de
f en a selon A.
Exemple 12. — Limite usuelle. Si a est point d’accumulation de D, si A = D  {a},
lim
x→a
est encore noté lim
x→a
.
x∈A x=a

— Limite à gauche. Soit f une fonction de la variable réelle x ∈ I , et a ∈ I. Lorsque


A = ]−∞, a[ ∩ I , la limite lim
x→a
f (x), si elle existe, est encore notée lim
x→a
f (x) (ou
x∈A x<a
limx→a − f (x), ou encore f (a−)) et appelée limite à gauche de f en a. On définit de
même xlim
→a
f (x) (encore notée limx→a + f (x) ou f (a+)) la limite à droite de f en a.
x>a

— Limite en +∞. On note R l’ensemble R ∪ {−∞, +∞} (cet ensemble est appelé R
achevé). Sur R, on définit
x
ϕ(x) = si x ∈ R, ϕ(+∞) = 1, ϕ(−∞) = −1.
1 + |x|
On vérifie facilement que d(x, y) = |ϕ(x) − ϕ(y )| définit une distance sur R. Cette
distance fait de R un espace métrique et nous autorise à parler de limite en +∞
ou en −∞. Lorsqu’une fonction de la variable réelle f est définie sur ]c, +∞[, il est
évidemment très lourd de caractériser la limite de f en +∞ grâce à la distance d
sur R. On montre facilement que f (x) tend vers  en +∞ si et seulement si
(∀ε > 0, ∃C > c, ∀x > C), d(f (x), ) < ε,
et on note alors limx→+∞ f (x) = . On caractériserait de même la limite en −∞.
Noter que l’on peut de la même manière caractériser la limite d’une suite N →
(E, d) en +∞.
— Limite infinie. Lorsqu’une fonction f est à valeurs réelles, il est possible de ca-
ractériser simplement les limites infinies de f . Par exemple, f tend vers +∞ lorsque
x tend vers a selon A si et seulement si
(∀C > 0, ∃α > 0, ∀x ∈ A), (d(a, x) < α =⇒ f (x) > C).
On note alors lim xx→
∈A
a f (x) = +∞.

— Le lecteur pourra de même caractériser simplement une limite infinie prise à l’infini.
Composition des limites.
Proposition 18. Soient f : D1 ⊂ (E, d) → (F, d ) et g : D 2 ⊂ (F, d  ) → (G, d) telles
que f (D1 ) ⊂ D2 . Soient A ⊂ D1, a ∈ A ; B ⊂ D 2 et b ∈ B . On suppose que
f (A) ⊂ B, lim
x→a
f (x) = b et lim g (y ) = c.
y→b
x∈A y∈B

Alors l’application composée h = g ◦ f vérifie lim xx→


∈A
a h(x) = c.

Proposition 19. Soient f, g : D ⊂ (E, d) → F , où F est un K e.v.n (K = R ou C).


Soient A ⊂ D et a ∈ A. On suppose lim xx→∈Aa f (x) =  et lim xx→ a g (x) =  . Alors, lorsque
∈A
x tend vers a selon A,
16 1. TOPOLOGIE SUR LES ESPACES MÉTRIQUES ET LES ESPACES VECTORIELS NORM ÉS

(i) (f + g)(x) tend vers  +  ,


(ii) pour tout λ ∈ K, (λf )(x) tend vers λ,
(iii) f (x) tend vers ,
(iv) si F est une algèbre normée, (f g)(x) tend vers  ,
(v) si F = R et si f (x) ≤ g (x) sur un voisinage de a dans A,  ≤  ,
(vi) si F = R, si pour une fonction h : D → R on a f (x) ≤ h(x) ≤ g (x) dans un
voisinage de a dans A et si  =  , alors h(x) tend vers .
Limite et continuité en un point.
Proposition 20. Soient f : D ⊂ (E, d) → (F, d ) et a ∈ D un point d’accumulation de
D. L’application f est continue en a si et seulement si xlim
→a
f (x) = f (a).
x=a

Remarque 18. Lorsque f n’est pas définie en a et lorsque limxx→ a f (x) = , la fonction g
=a
définie sur D ∪ {a} par g (x) = f (x) sur D et g (a) =  est continue en a et est appelée
prolongement par continuité de f en a.

Définition 22. Soit f : D ⊂ R → (E, d) et a ∈ D. On dit que
— f est continue à droite en a si lim xx>a
→a f (x) = f (a),

— f est continue à gauche en a si lim xx<a


→a f (x) = f (a).

On dit que f présente une discontinuité de première espèce en a si g = lim xx<a


→a f (x) et

d = lim xx>a
→a f (x) existent, et si  =  .
g d

1.6. Exercices
Exercice 1. Soit (E, d) un espace métrique.
a) Soit a ∈ E et r > 0. Montrer que B(a, r ) ⊂ Bf (a, r). Peut-on affirmer, dans le cas
général, que B(a, r) = Bf (a, r) ?
b) Si E est un R-e.v.n, montrer que B(0, 1) = Bf (0, 1).

Solution. a) On a B(a, r) ⊂ B f (a, r ). Comme B f (a, r ) est fermé et que B(a, r) est le plus petit
fermé contenant B(a, r), on en déduit B(a, r ) ⊂ B f (a, r).
L’égalité n’a pas lieu dans le cas général, comme nous allons le vérifier sur un contre-exemple.
Si E est muni de sa distance discrète d (définie par d(x, y) = 0 si x = y , = 1 si x = y), on a
B(a, 1) = {a}, fermé, et Bf (a, 1) = E . Ainsi, dès que E possède plus d’un élément, B(a, 1) =
{a} = E = Bf (a, 1).
b) D’après la question précédente, il suffit de montrer l’inclusion Bf (0, 1) ⊂ B(0, 1). Soit x ∈
Bf (0, 1). Si x < 1, alors x ∈ B(0, 1) ⊂ B(0, 1) et c’est terminé. Sinon x = 1, et alors pour
tout ε > 0, il existe y ∈ B(0, 1) tel que x − y  < ε (prendre par exemple y = (1 − ε/2)x). En
d’autres termes, x ∈ B(0, 1) d’après la proposition 4, d’où le résultat.

Exercice 2. Soit A ⊂ R tel que tout point de A est isolé dans A. Montrer que A est au
plus dénombrable.
Solution. Nous allons associer à chaque élément de A un rationnel de manière injective. Tout
élément a ∈ A est isolé dans A, donc
∃ra > 0, ]a − ra, a + r a [ ∩ A = {a}. (∗)
Comme Q est dense dans R, il existe qa ∈ Q tel que qa ∈ ]a − ra /2, a + ra/2[.
1. G ÉNÉRALIT ÉS 17

Montrons maintenant que l’application ϕ : A → Q a → qa est injective. Si q a = qb = q,


avec a, b ∈ A, alors |q − a| < ra /2 (car q = q a est choisi dans ]a − r a/2, a + r a/2[), de même,
|q − b| < rb /2, donc
ra + r b
|a − b| = |(q − b) − (q − a)| ≤ |q − b| + |q − a| < . (∗∗)
2
L’un des réels ra, rb est plus grand que l’autre, par exemple ra ≥ rb . L’inégalité (**) entraı̂ne
|a − b| < ra , ce qui d’après (*) implique b = a car b ∈ A.
Finalement, nous avons construit une application injective ϕ : A → Q, et comme Q est
dénombrable, on en déduit que A est au plus dénombrable.
Remarque. Le raisonnement effectué dans la démonstration reste valable pour A ⊂ R n ,
en utilisant le fait que Qn est dénombrable.

Exercice 3. Soit f : R + → R une fonction uniformément continue sur R+ . Montrer


∃α, β > 0, ∀x ∈ R + , |f (x)| ≤ αx + β.

Solution. L’application f étant uniformément continue, on a l’existence de η > 0 tel que


∀(x, y ) ∈ (R + ) 2, |x − y| ≤ η, |f (x) − f (y)| ≤ 1. (∗)
+
Maintenant, fixons x ∈ R et notons n = [x/η] (partie entière de x/η ). On a nη ≤ x < (n + 1)η ,
donc
 
 n−1
  
 
|f (x) − f (0)| = f (x) − f (nη ) + f ((k + 1)η) − f (kη) 
 
k=0
n−1
 n−1

≤ |f (x) − f (nη)| + |f ((k + 1)η) − f (kη)| ≤ 1 + 1= 1+n
k=0 k=0
ce qui entraı̂ne
x
|f (x)| ≤ |f (0)| + 1 + n ≤ |f (0)| + 1 + = β + αx
η
avec β = |f (0)| + 1 et α = 1/η. Les réels α et β sont indépendants du choix de x, donc ceci est
vrai pour tout x ∈ R +, d’où le résultat.

Exercice 4. Soit (E, d) un espace métrique.


1/ Soit A ⊂ E . Montrer que l’application E → R x → d(x, A) est continue sur E (on
rappelle que d(x, A) = inf y∈A d(x, y )).
2/ Soient A et B deux fermés disjoints de E.
a) Montrer qu’il existe f : E → R continue, 0 ≤ f ≤ 1, telle que A = f −1 ({0}) et
B = f −1 ({1}).
b) En déduire l’existence de deux ouverts U et V de E , disjoints, tels que A ⊂ U et
B ⊂V.
Solution. 1/ On va montrer que l’application x → d(x, A) est 1-lipschitzienne. Fixons (x 1 , x2 ) ∈
E2 . D’après l’inégalité triangulaire,
∀y ∈ A, d(x1 , A) ≤ d(x1 , y) ≤ d(x1, x2 ) + d(x2, y).
L’inégalité d(x1, A) ≤ d(x 1 , x 2) + d(x 2, y) étant vrai pour tout y ∈ A, on en déduit d(x 1, A) ≤
d(x1 , x2 ) + d(x2 , A). Autrement dit, d(x1 , A) − d(x 2 , A) ≤ d(x 1, x2). On montrerait de même
d(x2 , A) − d(x1 , A) ≤ d(x1, x2 ), ce qui prouve |d(x 1, A) − d(x2 , A)| ≤ d(x 1, x2 ).
18 1. TOPOLOGIE SUR LES ESPACES MÉTRIQUES ET LES ESPACES VECTORIELS NORM ÉS

L’application x → d(x, A) étant lipschitzienne, elle est uniformément continue sur E (voir
l’exemple 11), donc continue sur E.
2/a) Rappelons que lorsque F est fermé, on a d(x, F ) = 0 si et seulement si x ∈ F .
d(x, A)
Ceci étant, définissons f : E → R par f (x) = .
d(x, A) + d(x, B)
— L’application f est bien définie, car si d(x, A) + d(x, B ) = 0, alors d(x, A) = d(x, B ) = 0,
donc x ∈ A et x ∈ B (A et B sont fermés), ce qui est impossible à réaliser puisque
A ∩ B = ∅.
— On a f (x) = 0 si et seulement si d(x, A) = 0, i. e. x ∈ A.
— On a f (x) = 1 si et seulement si d(x, A) = d(x, A) + d(x, B ), i. e. d(x, B ) = 0 ou encore
x ∈ B.
— L’application f est continue d’après a) et comme composée d’applications continues.
— Enfin, il est clair que 0 ≤ f (x) ≤ 1 pour tout x ∈ E .
b) Si f est la fonction exhibée à la question précédente, on a

A ⊂ U = f −1 ( ] − ∞, 1/2[ ) et B ⊂ V = f −1 ( ]1/2, +∞[ ),

et U et V sont ouverts par continuité de f (voir la proposition 9), disjoints par construction.
Remarque. Il est possible de répondre à la question 2/b) sans utiliser le résultat de la
question 2/a).

Exercice 5. Soit (E, d) un espace métrique.


a) Soit A un ouvert de E et B ⊂ E . Montrer que A ∩ B ⊂ A ∩ B . A-t-on l’égalité dans
le cas général ?
b) Soit A ∈ E . On dit que A est localement fermé si

∀x ∈ A, ∃V voisinage de x tel que V ∩ A est un fermé de V.

Montrer que A est localement fermé si et seulement si A est l’intersection d’un ouvert de
E et d’un fermé de E.

Solution. a) Soit x ∈ A ∩ B . On a x ∈ A. Comme est ouvert, A est un voisinage de x. On


en déduit que pour tout voisinage V de x, V ∩ A est un voisinage de x. De plus x ∈ B, donc
(V ∩ A) ∩ B = V ∩ (A ∩ B ) = 0, et ceci pour tout voisinage V de x. On en déduit x ∈ A ∩ B ,
donc A ∩ B ⊂ A ∩ B .
On n’a pas toujours l’égalité. Par exemple, dans R, si A est l’ouvert ]0, 2[ et si B = [1, 3], on
a A ∩ B = [1, 2[ et A ∩ B = [1, 2].
b) La condition suffisante parait plus simple. Commençons donc par cela.
Condition suffisante. Supposons A = Ω ∩ F , où Ω est un ouvert de E et F un fermé de E . Soit
x ∈ A. On a x ∈ Ω, ouvert, donc Ω est un voisinage de x. Il vérifie Ω ∩ A = Ω ∩ (Ω ∩ F ) = Ω ∩ F ,
c’est donc un fermé de Ω. Finalement, A est localement fermé.
Condition nécessaire. Soit A localement fermé. Pour tout x ∈ A, nous allons montrer qu’il existe
un voisinage ouvert Vx de x tel que Vx ∩ A est un fermé de Vx . Par hypothèse sur A, il existe un
voisinage Ux de x tel que Ux ∩A est un fermé de U x . On peut donc écrire Ux ∩A = U x ∩F où F est

un fermé de E . Soit Vx = U x. C’est un ouvert contenant x et on a V x ∩ A = V x ∩ U x ∩ A = Vx ∩ F ,
donc Vx ∩ A est un fermé de Vx .
Posons maintenant Ω = ∪ x∈A Vx, ouvert de E (réunion d’ouverts). Nous allons prouver que
A = Ω ∩ A, ce qui montrera le résultat.
– On a A ⊂ Ω et A ⊂ A, donc A ⊂ Ω ∩ A.
2. SUITES DANS UN ESPACE MÉTRIQUE 19

– Il reste à montrer l’inclusion réciproque. Soit x ∈ Ω ∩ A. Comme x ∈ Ω, il existe y ∈ A tel que


x ∈ Vy . On a aussi x ∈ A, donc
x ∈ Vy ∩ A ⊂ V y ∩ A d’après a). (∗)
Supposons x ∈ A. Alors x ∈ W = Vy  (Vy ∩ A), ouvert de V y (car Vy ∩ A est un fermé de
Vy ). Comme Vy est ouvert, W est même un ouvert de E. Or (Vy ∩ A) ∩ W = ∅, c’est-à-dire
Vy ∩ A ⊂ E  W , et comme E  W est un fermé de E, Vy ∩ A ⊂ E  W . Autrement dit,
Vy ∩ A ∩ W = ∅, donc x ∈ V y ∩ A, ce qui est contradictoire avec (*). Finalement, x ∈ A et le
résultat.
Remarque. Nous n’avons pas utilisé la métrique de E ; ce résultat reste donc vrai, avec
la même démonstration, pour un espace topologique E.

2. Suites dans un espace métrique


2.1. Généralités
Définition 1. Soit (E, d) un espace métrique et (xn) n∈N une suite de points de E . On
dit que (x n) converge vers  ∈ E si pour tout voisinage V de  dans E , il existe N ∈ N
tel que pour tout n ≥ N , xn ∈ V , ou de manière équivalente si
(∀ε > 0, ∃N ∈ N), (n ≥ N =⇒ d(x n, ) < ε).
Si  existe,  est unique est s’appelle limite de (xn ). On note alors  = limn→+∞ x n.
Remarque 1. — Toute suite convergente est bornée.
— On rappelle qu’une suite extraite (ou sous-suite) d’une suite (xn )n∈N est une suite
de la forme (yn ) n∈N où yn = x ϕ(n) , avec ϕ : N → N strictement croissante. Si (xn)
converge vers , toute sous-suite de (xn ) converge vers .
— La notion de convergence d’une suite est topologique, autrement dit, elle ne dépend
que de la topologie de E.
Définition 2. Soit (xn ) une suite d’un espace métrique (E, d). On dit que a ∈ E est
valeur d’adhérence de (x n) si
(∀ε > 0, ∀p ∈ N, ∃n ≥ p), d(xn , a) < ε.
Proposition 1. Soit (x n) une suite de (E, d), soit a ∈ E . Les assertions suivantes sont
équivalentes.
(i) a est valeur d’adhérence de (xn ).
(ii) Il existe une sous-suite de (xn ) qui converge vers a.
(iii) Pour tout p ∈ N, a ∈ Ap où Ap = {xn , n ≥ p}.
(iv) a est point d’accumulation de A = {xn , n ∈ N} ou a est point de répétition de (x n)
(i. e. l’ensemble {n ∈ N | x n = a} est infini).
L’ensemble des valeurs d’adhérences de (xn ) est donc égal à ∩p∈N Ap , donc fermé.
Remarque 2. Si limn→∞ xn = ,  est l’unique valeur d’adhérence de (xn ).
2.2. Caractérisation de l’adhérence, des fermés, de la continuité
Proposition 2. Soient (E, d) un espace métrique et A une partie de E . Un élément
x ∈ E est dans A si et seulement s’il existe une suite de points de A qui converge vers x.
Proposition 3. Une partie F d’un espace métrique (E, d) est fermée si et seulement si
toute suite de points de F convergente converge vers un élément de F .
20 1. TOPOLOGIE SUR LES ESPACES MÉTRIQUES ET LES ESPACES VECTORIELS NORM ÉS

Proposition 4. Une application f : (E, d) → (E , d ) est continue en a ∈ E si et


seulement si pour toute suite (x n ) de E tendant vers a, la suite (f (xn)) converge vers
f (a).
Proposition 5. Soient une application f : D ⊂ (E, d) → (E  , d ), A une partie de D et
a ∈ A. On a lim xx→
∈A
a f (x) =  si et seulement si pour toute suite (x ) de points de A qui
n
converge vers a, la suite (f (xn )) converge vers .
Conséquence : Les propriétés de la proposition 19 de la partie 1.5 (page 15) restent vraies
pour des suites convergentes d’un K-e.v.n E (K = R ou C).
2.3. Suites de Cauchy et espaces complets
Définition 3. Soient (E, d) un espace métrique et (xn) une suite de points de E . On dit
que (xn ) est une suite de Cauchy (ou que la suite (xn ) vérifie le critère de Cauchy) si
(∀ε > 0, ∃N ∈ N), ∀p > N, ∀q > N, d(xp , x q ) < ε.
Remarque 3. — Une suite convergente est de Cauchy.
— Une suite de Cauchy est bornée.
— Attention ! La notion de suite de Cauchy n’est pas topologique (i. e. elle ne peut
pas être définie à partir des ouverts de E ). Cependant, on a le résultat suivant.
Proposition 6. Soient d et d deux distances uniformément équivalentes de E . Une suite
(x n) est de Cauchy dans (E, d) si et seulement si c’est une suite de Cauchy dans (E, d ).
Le résultat reste vrai en particulier lorsque d et d sont équivalentes.
Définition 4. On dit qu’un espace métrique (E, d) est complet si toute suite de Cauchy
de E converge.
Un espace vectoriel normé complet s’appelle un espace de Banach.
Exemple 1. — Les espaces R, Rn (n ∈ N∗ ) sont complets.
— L’ensemble Q n’est pas complet. Par exemple, si α ∈ RQ, une suite de points de
Q tendant vers α est de Cauchy mais ne converge pas dans Q. On verra cependant
(voir l’exercice 6) que tout espace métrique peut se plonger dans un espace complet.
Propriétés des espaces complets.
Proposition 7. — Toute partie complète d’un espace métrique est fermée.
— Toute partie fermée d’un espace complet est complète.
Proposition 8. Soient (E1 , d 1), . . . , (E n , d n) des espaces métriques. L’espace métrique
E1 × · · · × E n est complet (au sens de la distance produit) si et seulement si pour tout i,
l’espace métrique (Ei , di ) est complet.
Proposition 9. Soit (E, d) un espace complet et (Fn ) une suite décroissante de fermés
non vides de E , telle que limn→∞ δ (Fn ) = 0 ( où δ (Fn ) désigne le diamètre de Fn ). Alors
il existe x ∈ E tel que ∩n∈NFn = {x}.
Démonstration. Notons Γ = ∩n∈N F n.
L’ensemble Γ est non vide. En effet, choisissons pour tout n ∈ N un point xn dans Fn. Le
fait que limn→∞ δ (F n ) = 0 entraı̂ne que la suite (x n) est de Cauchy (si ε > 0, si N est choisi tel
que δ (FN ) < ε, alors pour tout p, q > N , d(xp , xq ) < ε car xp , x q ∈ F N ), donc converge dans E
puisque E est complet. Comme les Fp sont fermés et que xn ∈ Fp lorsque n ≥ p, la limite  de
(xn) appartient à F p pour tout p, donc à Γ. Ainsi, Γ = ∅.
Le fait que (δ (Fn )) tende vers 0 montre que Γ a au plus un élément, d’où le résultat. 
2. SUITES DANS UN ESPACE MÉTRIQUE 21

 Théorème 0 (du point fixe). Soit (E, d) un espace métrique complet, et une appli-
cation f : E → E telle que
∃k ∈ ]0, 1[ , ∀(x, y ) ∈ E 2, d(f (x), f (y)) ≤ k d(x, y )
(on dit alors que f est k -contractante). Alors f admet un unique point fixe, i. e. il existe
un unique x ∈ E tel que f (x) = x.
Démonstration. Existence. Fixons x0 ∈ E. On définit la suite (xn) par x n+1 = f (x n). Une
récurrence immédiate donne d(x n+1, xn ) ≤ k n d(x1, x0 ) pour tout entier naturel n. Ainsi, lorsque
p<q
kp
d(xp, x q) ≤ d(x p , xp+1 ) + · · · + d(x q−1, x q ) ≤ (k p + · · · + k q−1 ) d(x1 , x0 ) ≤ d(x1, x 0 ),
1−k
ce qui prouve que la suite (xn ) est de Cauchy. Comme E est complet, (x n) converge. Notons x
sa limite. Par continuité de f (f est continue car k-lipschitzienne), on a
f (x) = lim f (xn) = lim xn+1 = x.
n→∞ n→∞
Le point x est donc un point fixe de f .
Unicité. Supposons f (x) = x et f (y ) = y . Alors
0 ≤ d(x, y) = d(f (x), f(y)) ≤ k d(x, y ),
et comme k < 1, ceci n’est possible que si d( x, y) = 0, i. e. x = y . 

Remarque 4. Attention Le théorème est faux si l’on suppose seulement d(f (x), f (y)) <
d(x, y ) pour tout x = y . Cependant, dans un compact, une telle condition suffit à montrer
l’existence et l’unicité d’un point fixe (voir l’exercice 4 page 34).
Critère de Cauchy pour les fonctions. à l’aide de la proposition 5, on montre
facilement le résultat qui suit.
Proposition 10. Soit (E, d) un espace métrique et (F, δ) un espace métrique complet.
Soit une application f : D ⊂ E → F , soient A ⊂ E et a ∈ A. La fonction f admet une
limite lorsque x tend vers a selon A si et seulement si
(∀ε > 0, ∃α > 0, ∀(x, y ) ∈ A2 ), (d(a, x) < α et d(a, y ) < α) =⇒ δ(f (x), f (y)) < ε.
2.4. Exercices
 Exercice 1. Soit X un ensemble. On note B(X, R) le R-e.v des fonctions bornées de X
dans R. On norme B(X, R) en posant
∀f ∈ B(X, R), f  = sup |f (x)|.
x∈X

Muni de cette norme, montrer que B(X, R) est un espace de Banach.


Solution. Rappelons qu’un espace de Banach est un e.v.n complet. La preuve de la complétude
d’un espace métrique est hyper-classique. On procède comme suit :
(i) On considère une suite de Cauchy, et on construit sa limite éventuelle,
(ii) on vérifie qu’elle appartient à l’ensemble de départ,
(iii) on montre que la suite de Cauchy converge bien vers cette limite éventuelle.
(i) Soit (fn ) une suite de Cauchy de B(X, R). Fixons x ∈ X . Pour tous p, q ∈ N, l’inégalité
|fp (x) − fq(x)| ≤ f p − fq  implique que la suite (f n(x)) est de Cauchy dans R. Comme R est
complet, (fp (x)) converge. Notons f (x) sa limite.
(ii) L’application f : X → R ainsi construite vérifie f (x) = limn→∞ f n(x) pour tout x ∈ X .
Montrons que f est bornée. La suite (f n) étant de Cauchy, elle est bornée. Ainsi, il existe M > 0
tel que fn  ≤ M pour tout n. Si x ∈ X , on a donc |fn (x)| ≤ M pour tout n, donc en passant
à la limite |f (x)| ≤ M . Ceci étant vrai pour tout x, f est bien bornée, i. e. f ∈ B(X, R).
22 1. TOPOLOGIE SUR LES ESPACES MÉTRIQUES ET LES ESPACES VECTORIELS NORM ÉS

(iii) Montrons maintenant que (fn ) tend vers f dans B(X, R). Soit ε > 0. Il existe N > 0 tel
que pour tout p, q ≥ N , f p − f q ≤ ε. Ainsi, si on fixe un élément x quelconque de X , on a
∀p ≥ N, ∀q ≥ N, |f p (x) − f q (x)| ≤ fp − fq < ε.
En fixant p dans l’assertion précédente et en faisant tendre q vers l’infini, on en déduit l’inégalité
|fp (x) − f (x)| ≤ ε. Ceci étant vrai pour tout x ∈ X , on a f p − f ≤ ε. Ceci est vrai pour tout
p ≥ N , donc (fp ) converge vers f .
Finalement, toute suite de Cauchy (fn) de B(X, R) converge, donc B(X, R) est complet.

 
1 1
Exercice 2. On munit l’espace ]0, +∞[ de la distance δ (x, y ) =  − .
x y
a) Montrer que δ est bien une distance sur ]0, +∞[.
b) Montrer que cette distance définit sur ]0, +∞[ la même topologie que la topologie
usuelle.
c) Montrer que l’espace métrique ( ]0, +∞[ , δ) n’est pas complet.
d) On restreint la distance δ à l’espace ]0, 1]. Montrer que ( ]0, 1], δ) est complet.

Solution. a) C’est bien une distance car :


(i) δ(x, y) = 0 si et seulement si 1/x = 1/y , i. e. x = y .
(ii) ∀(x, y) ∈ ]0, +∞[2, δ (x, y) = δ (y, x).
(iii) ∀(x, y, z) ∈ ]0, +∞[ 3,
     
 1 1   1 1  1 1 
δ(x, z) =  −  ≤  −  +  −  = δ (x, y) + δ (y, z ).
    
x z x y y z

b) Notons d la distance usuelle sur les nombres réels (d(x, y) = |x − y |). Rappelons le fait suivant :
dire que les deux espaces métriques ( ]0, +∞[, δ) et ( ]0, +∞[, d) définissent la même topologie,
c’est dire qu’ils ont les mêmes ouverts. La proposition 10 de la page 12 affirme que ceci équivaut
au fait que l’application identité Id]0,+∞[ : ( ]0, +∞[, d) → ( ]0, +∞[, δ) est un homéomorphisme.
Pour prouver ce dernier point, nous allons utiliser le fait que cette application est la composée
de
1 1
ϕ : ( ]0, +∞[, d) → ( ]0, +∞[, d) x → et ψ : ( ]0, +∞[, d) → ( ]0, +∞[, δ) x → ,
x x
et que ϕ et ψ sont des homéomorphismes.
L’application ϕ est continue (c’est classique puisque d est la distance usuelle), elle est bijec-
tive, et ϕ−1 = ϕ est aussi continue. On a donc bien affaire à un homéomorphisme.
L’application ψ vérifie
 
 1 1 
2
∀(x, y ) ∈ ]0, +∞[ , 
δ(ψ (x), ψ(y)) =  − = |x − y | = d(x, y ),
ψ (x) ψ (y ) 
c’est donc une isométrie, donc un homéomorphisme. D’où le résultat.
Remarque. On aurait pu également prouver le résultat “à la main”, en montrant qu’un ouvert
de ( ]0, +∞[, d) est un ouvert de ( ]0, +∞[, δ) et réciproquement.
c) Dans ( ]0, +∞[, δ), la suite (un) n∈N∗ définie par un = n est de Cauchy car δ (u p, u q ) = |1/p −
1/q| tend vers 0 lorsque p et q tendent vers +∞. Il est clair que cette suite ne converge pas
dans ( ]0, +∞[, d) (elle n’est pas bornée), donc elle ne converge pas dans ( ]0, +∞[, δ) puisque ces
deux espaces métriques définissent la même topologie (la notion de convergence est topologique).
Ainsi, ( ]0, +∞[, δ) n’est pas complet.
d) On applique la méthode utilisée dans la solution de l’exercice précédent pour prouver la
complétude souhaitée. Considérons une suite (un) de Cauchy dans ( ]0, 1], δ). Pour tout p, q, on
2. SUITES DANS UN ESPACE MÉTRIQUE 23

a    
 1 1   up − uq 
δ (up , uq ) =  −  =   ≥ |up − u q|,
up uq upu q 
ce qui montre que (un ) est de Cauchy dans (R, d). Ce dernier étant complet, (un ) converge dans
(R, d). Soit u sa limite dans cet espace métrique.
La limite u est forcément dans l’adhérence de ]0, 1] dans (R, d), qui est [0, 1]. Si u = 0, alors
δ(1, un ) = |1 − 1/un | tend vers +∞, donc la suite (u n) n’est pas bornée dans ( ]0, 1], δ), ce qui
est impossible car c’est une suite de Cauchy de cet espace. Ainsi, u = 0, donc u ∈ ]0, 1]. Comme
( ]0, 1], d) et ( ]0, 1], δ) possèdent la même topologie (c’est la topologie induite par ( ]0, +∞[, d)
ou ( ]0, +∞[, δ) — qui possèdent la même topologie — sur ]0, 1]), on en déduit que (un ) converge
vers u dans ( ]0, 1], δ).
On a prouvé que toute suite de Cauchy de ( ]0, 1], δ) converge, donc ( ]0, 1], δ) est complet.

Exercice 3 (Deux résultats de point fixe). 1/ Soit (E, d) un espace métrique


complet et une application f : E → E . On suppose l’existence d’un entier naturel non
nul r tel que l’application f r (composée r fois de f ) est k -contractante (0 < k < 1).
Montrer que f admet un unique point fixe.
2/ (Point fixe à paramètre). Soient (X, δ) et (E, d) deux espaces métriques, (E, d) étant
complet. On considère une application
F : X×E → E (λ, x) → F (λ, x),
continue, et k -contractante en la seconde variable, i. e.
∃k ∈ ]0, 1[, ∀λ ∈ X, ∀(x, y ) ∈ E 2, d(F (λ, x), F (λ, y )) ≤ k d(x, y ).
Montrer que pour tout λ ∈ X , l’application F (λ, ·) : x → F (λ, x) admet un unique point
fixe, que l’on note xλ . Montrer ensuite que l’application X → E λ → x λ est continue.

Solution. 1/ Comme E est complet et que f r : E → E est k -contractante, f r a un unique point


fixe x ∈ E (voir le théorème du point fixe). De l’égalité f r (x) = x on tire f (f r(x)) = f (x) =
f r(f (x)), ce qui montre que f (x) est un point fixe de f r. Comme x est l’unique point fixe de
f r, on a forcément f (x) = x.
Maintenant, x est le seul point fixe de f car l’égalité f (y) = y entraı̂ne f r (y) = y donc y = x
car x est le seul point fixe de f r .
2/ Pour tout λ ∈ X , l’application F (λ, ·) : E → E est k -contractante, donc admet un unique
point fixe xλ car E est complet.
Montrons que l’application λ → xλ est continue. Si λ, λ  ∈ X, on a

d(xλ, xλ  ) = d(F (λ, x λ), F (λ  , xλ  )) ≤ d(F (λ, xλ ), F (λ, x λ )) + d(F (λ, x λ ), F (λ , xλ ))
≤ k d(x λ , xλ ) + d(F (λ, x λ), F (λ , x λ ))
donc
1
d(xλ , xλ ) ≤ d(F (λ, x λ), F (λ , x λ )).
1−k
La continuité de F au point (λ , xλ  ) permet maintenant d’affirmer limλ→λ  xλ = xλ , d’où le
résultat.

Exercice 4 (Deux résultats de prolongement). Soient (E, d) et (F, δ) deux es-


paces métriques, A une partie de E dense dans E.
24 1. TOPOLOGIE SUR LES ESPACES MÉTRIQUES ET LES ESPACES VECTORIELS NORM ÉS

a) Si une application f : (A, d) → (F, δ) est continue et si


∀x ∈ E A, lim f (y ) existe,
y→x
y∈A

montrer qu’il existe une unique fonction g : E → F , continue, telle que la restriction g|A
de g à A soit égale à f .
b) On suppose cette fois ci que (F, δ) est complet. Soit f : (A, d) → (F, δ) une appli-
cation uniformément continue. Montrer l’existence d’une unique fonction g : E → F
uniformément continue, telle que g|A = f .

Solution. a) Définissons g : E → F de la manière suivante :


∀x ∈ A, g (x) = f (x) et ∀x ∈ E A, g (x) = lim
y→x
f (y ).
y∈A

Montrons que g est continue sur E. Soit x ∈ E et (xn) n∈N∗ une suite de points de E tendant
vers x. Pour tout n ∈ N∗ , on a limy→x n f (y ) = g (xn ). On en déduit facilement
y∈A

1 1
(∀n ∈ N∗ , ∃y n ∈ A), d(xn , yn ) ≤ et δ (g (x n), f(yn )) < .
n n
1
La relation d(x, yn ) ≤ d(x, xn ) + d(xn , yn ) ≤ n + d(x, xn ) montre que
lim yn = x, et donc lim f (yn) = g (x). (∗)
n→∞ n→∞
Maintenant, les inégalités
1
δ (g (xn ), g(x)) ≤ δ (g (xn ), f(y n )) + δ (f (yn ), g(x)) ≤
+ δ (f (yn), g(x))
n
montrent avec (*) que limn→∞ g (x n ) = g (x). Ceci étant vrai pour tout suite (xn) de E tendant
vers x, on en conclut que g est continue en x, et ceci pour tout x ∈ E .
Unicité. Soient g et h : E → F deux applications continues telles que g|A = h|A .
— Par hypothèse, g (x) = h(x) pour tout x ∈ A.
— Soit x ∈ E A. Comme A est dense dans E , il existe une suite (x n ) de points de A
tendant vers x. Comme g et h sont continues, on a
g (x) = lim g (x n ) = lim f (xn ), de même h(x) = lim f (xn),
n→∞ n→∞ n→∞
ce qui suffit pour conclure g (x) = h(x).
b) L’idée est de se ramener au cas précédent puis de prouver que la fonction g obtenue est bien
uniformément continue.
Soit x0 ∈ E A. Montrons que lim y→x 0 f (y ) existe. Soit ε > 0. Comme f est uniformément
y∈A
continue sur A,
(∃α > 0, ∀(x, y ) ∈ A2 ), d(x, y ) < α =⇒ δ (f (x), f (y)) < ε.
En particulier, si x, y ∈ A vérifient d(x, x 0) < α/2 et d(y, x0 ) < α/2, on a d(x, y ) < α donc
δ (f (x), f(y)) < ε. Comme (F, δ) est complet, d’après le critère de Cauchy pour les fonctions
(voir la proposition 10), on en déduit que lim y→x0 f (y ) existe.
y∈A
D’après le résultat de la question précédente, la fonction g définie sur E par
∀x ∈ A, g (x) = f (x) et ∀x ∈ E A, g (x) = lim
y→x
f (y )
y∈A

est continue sur E. Nous allons prouver qu’elle est uniformément continue sur E . Fixons ε > 0.
Par hypothèse, f est uniformément continue sur A, donc
(∃α > 0, ∀(x, y ) ∈ A2 ), d(x, y ) < α =⇒ δ (f (x), f (y)) < ε.
Donnons nous (x, y) ∈ E 2 avec d(x, y) < α. Comme A est dense dans E, il existe deux suites
(xn) et (y n) de points de A tendant respectivement vers x et y . La distance étant continue, on a
limn→∞ d(x n, yn ) = d(x, y) < α, ce qui montre l’existence de N ∈ N tel que d(xn, y n) < α pour
tout n ≥ N . Ainsi, pour tout n ≥ N , δ (f (xn), f(y n)) < ε et en faisant tendre n vers l’infini,
2. SUITES DANS UN ESPACE MÉTRIQUE 25

on obtient δ (g (x), g(y)) ≤ ε. Ceci est vrai pour tout couple (x, y ) ∈ E2 tel que d(x, y) < α, la
fonction g est donc uniformément continue sur E.
L’unicité découle de l’unicité de la question précédente car une fonction uniformément conti-
nue est continue.

Exercice 5. Soient (E, d) et (F, d) deux espaces métriques. On suppose que (E, d) est
complet. Soient f : E → F continue et (E n)n∈N une suite décroissante de fermés non
vides dont le diamètre δ (En ) tend vers 0. Montrer que
 
 
f En = f (En ).
n∈N n∈N

Solution. Comme E est complet, il existe x0 ∈ E tel que ∩n∈NE n = {x0 } (voir la proposition 9).
On en déduit f (∩ n∈NEn ) = {f (x0 )}. Il nous faut donc prouver ∩n∈Nf (E n) = {f (x0 )}.
Pour tout n ∈ N, x0 ∈ E n donc {f (x0 )} ⊂ ∩n∈Nf (En ).
Montrons l’inclusion réciproque. L’application f est continue en x0 , donc pour tout voisinage
V de f (x0), il existe un voisinage U de x 0 tel que f (U ) ⊂ V . Or pour tout n, En ⊂ Bf (x0 , δ(En ))
(car x0 ∈ En ), et comme δ (E n ) tend vers 0,
∃n ∈ N, En ⊂ Bf (x 0, δ(E n )) ⊂ U, donc f (En ) ⊂ f (U ) ⊂ V.
Ainsi, ∩n∈N f (E n ) ⊂ V , et ceci pour tout voisinage V de f (x0). Ceci suffit pour conclure à
l’inclusion ∩ n∈Nf (En ) ⊂ {f (x 0)}, d’où le résultat.
Remarque. Rappelons qu’en général l’égalité f (∩ i∈IX i) = ∩i∈I f (Xi ) est fausse. Par
contre, l’inclusion f (∩i∈I Xi ) ⊂ ∩i∈I f (X i) a toujours lieu.

Exercice 6 (Complété d’un espace métrique). Soit (E, d) un espace métrique.


On note C l’ensemble des suites de Cauchy U = (un) n∈N de E . Le but de l’exercice est de
plonger (E, d) dans un espace complet dont la distance prolonge celle de E.
1/ a) Soient U = (un ) et V = (v n) ∈ C. Montrer que la suite (d(u n, vn )) converge. On
note δ(U, V ) sa limite.
b) Montrer que δ est symétrique et vérifie l’inégalité triangulaire.
2/ On considère la relation d’équivalence sur C définie par (U ∼ V ) ⇐⇒ (δ(U, V ) = 0).
On note Ê l’espace quotient C/∼ et Û la classe d’équivalence dans Ê de U ∈ C.
a) Quelle est la classe d’une suite convergente dans E ?
b) Montrer que si U ∼ U  et V ∼ V  , alors δ(U, V ) = δ (U , V  ). Lorsque Uˆ, V̂ ∈ Ê, le réel
δ(U, V ) est donc indépendant du choix des représentants U et V de Û et V̂ . On le note
δ(Û , V̂ ).
c) Ainsi définie, montrer que δ est une distance sur Ê.
d) Montrer qu’il existe une injection naturelle i : E → Ê, isométrique, et que i(E ) est
dense dans Ê.
3/ Montrer que Ê est complet.
4/ Soient (E 1, d1 ), (E 2, d2 ) deux espaces métriques complets tels qu’il existe une isométrie
i1 (resp. i2 ) de E dans E1 (resp. dans E 2), avec i 1 (E) (resp. i2 (E)) dense dans E1 (resp.
dans E 2). Montrer l’existence d’une unique isométrie ϕ de E1 dans E2 , bijective, et
vérifiant ϕ(i1 (x)) = i 2(x) pour tout x ∈ E .
26 1. TOPOLOGIE SUR LES ESPACES MÉTRIQUES ET LES ESPACES VECTORIELS NORM ÉS

Solution. 1/ a) Comme R est complet, il suffit de montrer que la suite (d(un , vn )) n∈N est de
Cauchy. Pour tous p, q ∈ N,
d(up , vp ) ≤ d(u p, uq ) + d(uq , vq ) + d(v q, v p)
donc
d(up , vp ) − d(uq , vq ) ≤ d(u p , uq) + d(vp , v q ).
On obtient de même d(u q , vq ) − d(up , v p ) ≤ d(up , uq ) + d(v p , v q ), d’où
|d(uq , vq ) − d(up , vp )| ≤ d(up , uq ) + d(vp, v q).
Les suites U et V étant de Cauchy, on en déduit que la suite (d(un, v n)) est de Cauchy.
b) C’est immédiat par passage à la limite, les propriétés de symétrie et d’inégalité triangulaires
étant vraies pour d.
2/ a) Soit U = (u n) une suite de E convergeant vers α ∈ E . Toute suite convergente est de
Cauchy, donc U ∈ C. Soit V = (vn ) ∈ C. On a
U ∼ V ⇐⇒ δ (U, V ) = 0 ⇐⇒ lim d(u n , vn ) = 0.
n→∞
Les inégalités
d(α, vn ) ≤ d(α, un ) + d(un , vn ) et d(u n, vn ) ≤ d(u n, α) + d(α, vn )
montrent que U ∼ V si et seulement si limn→∞ d(α, v n) = 0. Finalement, la classe de U est
l’ensemble des suites qui convergent vers α.
b) Si U ∼ U  et V ∼ V , comme δ satisfait l’inégalité triangulaire, on a
δ (U, V ) ≤ δ (U, U ) + δ (U , V ) + δ (V , V ) = δ (U  , V  ),
de même δ (U , V ) ≤ δ (U, V ). Donc δ (U, V ) = δ (U  , V  ).
c) Après le résultat de la question 1/b), il reste à prouver δ(Û , V̂ ) = 0 si et seulement si Û = V̂ .
Ceci est vrai par construction de la relation d’équivalence ∼.
d) Pour tout α ∈ E , on note (α) ∈ C la suite constante égale à α. Soit i : E → Ê α → (α). 
On a
δ (i(α), i(β)) = δ((α), (β)) = d(α, β),
c’est-à-dire i est isométrique, et c’est donc une injection.
Montrons que i(E ) est dense dans Ê. Soit Û ∈ Ê , avec U = (un ) n∈N ∈ C. Nous allons
prouver que Û est la limite de la suite (i(un )) n∈N. Soit ε > 0. La suite (u n ) est de Cauchy, de
sorte qu’il existe N ∈ N tel que pour tous p, q ≥ N , d(up , uq) < ε. En fixant p ≥ N , on en déduit
δ(Û , i(up )) = δ (U, (up )) = lim d(un , up ) ≤ ε.
n→∞

Ceci étant vrai pour tout p ≥ N , on en déduit limp→∞ i(up ) = Û . Ainsi, on a montré que tout
élément Û de Ê est limite de points de i(E ), d’où le résultat.
3/ Soit (αn )n∈N une suite de Cauchy de Ê . Comme i(E ) est dense dans Ê, il existe pour tout
n ∈ N∗ un point xn de E tel que δ (αn, i(xn )) < 1/n. L’inégalité
1 1
d(x p , xq ) = δ (i(xp), i(xq)) ≤ δ (i(x p), α p) + δ (αp, α q ) + δ (αq , i(xq )) ≤ δ (α p , αq) + +
p q
montre que la suite (xn ) est de Cauchy dans E . Notons α la suite ( x n)n∈N de Ê.
Montrons que (α n) converge vers α. Comme δ (α n, α) ≤ δ (αn, i(x n)) + δ (i(x n ), α) ≤ 1n +
δ (i(xn ), α), il suffit de prouver que limn→∞ δ (i(x n ), α) = 0. Soit ε > 0. La suite (xn ) étant de
Cauchy dans E ,
∃N ∈ N, ∀p ≥ N, ∀q ≥ N, d(xp, x q ) < ε,
donc si on fixe n ≥ N ,
δ (i(xn ), α) = lim d(x n, xp ) ≤ ε,
p→∞
et ceci pour tout n ≥ N , d’où le résultat.
3. ESPACES COMPACTS 27

4/ Définissons ϕ sur i 1(E ) en posant ϕ(i1 (x)) = i 2(x) pour tout x ∈ E . L’application ϕ restreinte
à i1(E ) est isométrique car
∀(x, y ) ∈ E 2 , d2(ϕ(i1 (x)), ϕ(i1 (y))) = d2 (i2(x), i 2 (y)) = d(x, y) = d1 (i 1(x), i 1 (y)).
Ainsi, ϕ est uniformément continue sur i1 (E ). Comme i1 (E ) est dense dans E1 et que E2
est complet, il existe (voir l’exercice 4) un prolongement de ϕ sur E1 , encore noté ϕ, qui est
uniformément continu sur E 1 . De plus ϕ est isométrique sur i1 (E ), et comme i1(E ) est dense
dans E1 et que ϕ est continue, ϕ est isométrique sur E 1 tout entier. En particulier, ϕ est injective.
Il nous reste à montrer que ϕ est surjective. Soit β ∈ E2 . Comme i2(E ) est dense dans E2 ,
il existe une suite (β n ) = (i2 (xn )) de i2(E ) qui converge vers β. De plus pour tous p, q ∈ N,
d1(i1 (xp), i 1 (xq )) = d(xp , xq ) = d2 (i 2(xp ), i2(x q)) = d2 (β p , βq )
la suite (i1(x n)) est donc de Cauchy dans E 1 . Comme E1 est complet, elle converge. Soit α sa
limite. Comme ϕ est continue,
ϕ(α) = lim ϕ(i 1(x n)) = lim i 2(xn) = lim β n = β,
n→∞ n→∞ n→∞

d’où la surjectivité.
L’application ϕ est unique d’après l’unicité du prolongement continue sur E1 (voir l’exer-
cice 4).

Remarque. Par abus, on identifie souvent E et i(E ) dans Ê. Ainsi, Ê est un espace
complet dans lequel E est plongé, et sa métrique prolonge celle de E.
– La partie 4/ montre que Ê est unique à une isométrie bijective près. On l’appelle le
complété de E.
– On procède de manière similaire pour définir R à partir de Q (R est le complété de Q). La
différence est que, R “n’existant pas encore”, on ne peut pas définir δ(U, V ) pour U, V ∈ C.
On peut par contre définir la classe d’équivalence U ∼ V ⇐⇒ limn→∞ u n − vn = 0. La
notion de limite peut en effet être définie si l’on reste uniquement sur Q (il suffit de
prendre les ε et les α dans Q).

3. Espaces compacts
3.1. Propriété de Borel-Lebesgue
Définition 1. Un espace métrique (E, d) est dit compact si de tout recouvrement de E
par des ouverts de E , on peut en extraire un sous-recouvrement fini. Autrement dit, si
E = ∪i∈I Oi avec O i ouvert pour tout i, il existe J ⊂ I , J fini, tel que E = ∪ i∈J O i .
Exemple 1. — Tout espace métrique fini est compact.
— L’ensemble R des nombres réels n’est pas compact (on ne peut pas extraire un
sous-recouvrement fini du recouvrement R = ∪ n∈N∗ ] − n, n[).
Remarque 1. La notion de compacité peut être définie dans un espace topologique général.
Si E est un espace topologique, E est dit compact s’il est séparé (voir la page 10) et s’il
satisfait les propriétés de la définition 1. Les propositions 2, 3, 4 et 5 de cette partie 3.1
restent vraies pour les compacts d’un espace topologique. Mais attention ! La propriété
de Bolzano-Weierstrass (voir la partie 3.2) n’est vraie que dans les espaces métriques.
Proposition 1. Un espace métrique compact est borné.
Démonstration. C’est immédiat car si E est compact, si x0 ∈ E, en extrayant du recouvrement
E = ∪n∈N∗ B(x0, n) un sous-recouvrement fini, on s’aperçoit que E est borné. 
28 1. TOPOLOGIE SUR LES ESPACES MÉTRIQUES ET LES ESPACES VECTORIELS NORM ÉS

Aspect dual de la propriété de Borel-Lebesgue. En passant au complémentaire de


la définition 1, on obtient facilement le résultat qui suit.
Proposition 2. Un espace métrique (E, d) est compact si et seulement si de toute in-
tersection vide de fermés de E , on peut en extraire une sous-suite
sous famille finie d’intersection
vide.
Une conséquence immédiate est la suivante.
Proposition 3. Si (Fn) n∈N est une suite décroissante de fermés non vides dans un
compact E , alors ∩ n∈NFn = ∅.
Remarque 2. Nous avons vu plus haut (voir la proposition 9 page 20) que ce dernier
résultat reste vrai dans un espace complet pourvu que limn→∞ δ (Fn ) = 0. Si cette dernière
condition n’est pas vérifiée, le résultat est faux dans un espace complet (par exemple,
∩n∈N [n, +∞[ = ∅).
Parties compactes. Les propriétés des ouverts d’une topologie induite entraı̂nent une
caractérisation simple des parties compactes.
Proposition 4. Soit (E, d) un espace métrique. Une partie A de E est compacte si
et seulement si de tout recouvrement de A par des ouverts de E (A ⊂ ∪i∈IO i avec O i
ouvert de E pour tout i), il en existe un sous-recouvrement fini ( ∃J ⊂ I , J fini, tel que
A ⊂ ∪i∈J O i ).
On en déduit facilement :
Proposition 5. Une réunion finie de parties compactes est compacte.
Proposition 6. Une intersection de compacts est compacte.
3.2. Propriété de Bolzano-Weierstrass
La compacité d’un espace métrique peut être caractérisée par la propriété dite de
Bolzano-Weierstrass, a priori totalement différente, mais parfois beaucoup plus souple
d’utilisation.
 Théorème 1 (de Bolzano-Weierstrass). Un espace métrique (E, d) est compact si
et seulement si de toute suite de points de E , on peut en extraire une sous-suite convergente
dans E .
Démonstration. La condition nécessaire se montre facilement, la condition suffisante est plus
délicate.
Condition nécessaire. (Borel-Lebesgue =⇒ Bolzano-Weierstrass). Nous allons en donner deux
preuves.
Première preuve. Soit (xn ) n∈N une suite de E . Pour tout p ∈ N, on note A p = {xn, n ≥ p}.
La suite (A p )p∈N est une suite décroissante de fermés non vides, donc (voir la proposition 3)
∩p∈NAp = ∅. Choisissons x ∈ ∩ p∈NA p. On construit une sous-suite (x ϕ(n) ) de (xn ) comme suit.
— On choisit xϕ(0) ∈ A 0 .
— L’élément x ϕ(n) étant construit, on prend xϕ(n+1) ∈ An+1 tel que ϕ(n + 1) > ϕ(n) et
d(xϕ(n+1) , x) < 1/2 n+1 (c’est possible car x ∈ A n+1).
Ainsi construite, (xϕ(n) ) est une sous-suite de (x n) qui converge vers x.
Seconde preuve. Soit (xn) une suite de E . Si cette suite ne prend qu’un nombre de valeurs fini,
on peut en extraire une sous-suite constante donc convergente. Sinon, A = {xn, n ∈ N} est
infini. Nous allons prouver que A a au moins un point d’accumulation. Supposons le contraire,
de sorte que pour tout x ∈ E , il existe rx > 0 tel que B(x, rx ) ∩ A est fini. Du recouvrement
∪x∈E B(x, r x) de E, on peut en extraire un sous-recouvrement fini. En termes mathématiques,
ceci s’écrit
∃J ⊂ E, J fini, avec E = ∪x∈J B(x, rx).
3. ESPACES COMPACTS 29

Ainsi, A = ∪ x∈J (B(x, rx ) ∩ A), réunion finie d’ensembles finis, est fini, ce qui est contradictoire.
L’ensemble A admet donc au moins un point d’accumulation x ∈ E . Ainsi, x est valeur
d’adhérence de la suite (xn ), donc il existe une sous-suite de (xn ) qui converge vers x.
Condition suffisante. (Bolzano-Weierstrass =⇒ Borel-Lebesgue). Nous montrons deux lemmes.
Précompacité (ou ε-recouvrement). Un espace métrique E est dit précompact si pour tout
ε > 0, il existe un recouvrement fini de E par des boules (ouvertes) de rayon ε.
Lemme 1. Tout espace métrique (E, d) vérifiant la propriété de Bolzano-Weierstrass est pré-
compact.
En effet. Raisonnons par l’absurde en supposant l’existence de ε > 0 tel que l’on ne puisse
pas trouver un sous-recouvrement fini de E par des boules de rayon ε.
— Soit x0 ∈ E . Alors B(x0, ε) = E .
— Donc il existe x1 ∈ E tel que d(x 0 , x1 ) ≥ ε.
— De même, comme B(x0 , ε) ∪ B(x 1 , ε) =  E, il existe x2 ∈ E tel que d(x0, x1 ) ≥ ε et
d(x0 , x2 ) ≥ ε.
— On recommence . . . x 0, x1 , . . . , xn étant construits tels que ∀i < j ≤ n, d(xi, xj ) ≥ ε, on
sait que ∪ 0≤i≤nB(x i, ε) = E, donc il existe x n+1 ∈ E tel que pour tout i, 0 ≤ i ≤ n,
d(xi , xn+1 ) ≥ ε.
On construit ainsi une suite (xn) de E telle que d(xi , x j) ≥ ε dès que i = j . La suite (xn ) n’admet
donc aucune sous
sous-suite
suite convergente car aucune sous-suite n’est de Cauchy, d’où la contradiction
voulue, d’où notre premier lemme.
Lemme 2. Soit (E, d) un espace métrique vérifiant la propriété de Bolzano-Weierstrass, et soit
(Oi)i∈I un recouvrement de E par des ouverts de E. Alors :
(∃α > 0, ∀x ∈ E, ∃i ∈ I ), B(x, α) ⊂ Oi .
En effet. Raisonnons par l’absurde, en supposant que
(∀α > 0, ∃x ∈ E, ∀i ∈ I ), B(x, α) ⊂ Oi .
En particulier,
 
∗ 1
(∀n ∈ N , ∃x n ∈ E, ∀i ∈ I ), B xn ,  Oi .

n
Soit (xϕ(n)) une sous-suite de (xn ) qui converge. Notons x sa limite. Il existe i ∈ I tel que x ∈ Oi .
Comme Oi est ouvert, il existe r > 0 tel que B(x, 2r ) ⊂ O i. Comme (x ϕ(n) ) converge vers x,
1
∃N ∈ N, ∀n ≥ N, (d(x ϕ(n) , x) < r et ϕ(n) > ).
r
Alors
 
1
∀n ≥ N, ∀y ∈ B x ϕ(n) , , d(x, y) ≤ d(x, x ϕ(n)) + d(xϕ(n) , y) < r + r = 2r,
ϕ(n)
donc pour tout n ≥ N , B(xϕ(n) , 1/ϕ(n)) ⊂ Oi , ce qui est absurde. D’où le lemme 2.
Achevons notre raisonnement. Soit (E, d) vérifiant la propriété de Bolzano-Weierstrass, soit
(Oi)i∈I un recouvrement de E par des ouverts de E. D’après le lemme 2,
∃α > 0, ∀x ∈ E, ∃i ∈ I, B(x, α) ⊂ Oi .
D’après le lemme 1, on peut recouvrir E par un nombre fini de boules de rayon α, ce qui s’écrit
∃n ∈ N∗ , ∃x 1, . . . , xn ∈ E, E = ∪ ni=1B(x i, α).
Or pour tout j, 1 ≤ j ≤ n, il existe i j ∈ I tel que B(x j, α) ⊂ O ij . On en déduit E = ∪ nj=1O ij ,
d’où le résultat. 
Il existe d’autres formulations de ce théorème qui sont les suivantes.
30 1. TOPOLOGIE SUR LES ESPACES MÉTRIQUES ET LES ESPACES VECTORIELS NORM ÉS

 Corollaire 1. Un espace métrique (E, d) est compact si et seulement si l’une des as-
sertions suivantes est vérifiée
— Toute suite de E admet au moins une valeur d’adhérence dans E .
— Toute partie infinie de E admet au moins un point d’accumulation dans E .

3.3. Propriétés générales des compacts


Proposition 7. Soit (E, d) un espace métrique.
— Si E est compact et si A est une partie fermée de E , A est compacte.
— Si A est une partie compacte de E , A est fermée et bornée.
Proposition 8. Un espace compact est complet.
Proposition 9. Soit (E, d) un espace métrique compact et (xn) une suite de E admettant
une et une seule valeur d’adhérence x. Alors (x n) converge vers x.
Démonstration. Supposons le contraire. Alors
∃ε > 0, ∀N, ∃n ≥ N, d(xn , x) ≥ ε.
On peut donc construire une sous-suite (xϕ(n) ) de (x n) telle que pour tout n, d(xϕ(n) , x) ≥ ε.
Comme E est compact, on peut extraire de la sous-suite (x ϕ(n) ) une nouvelle sous-suite (xϕ◦ψ(n) )
convergente. Si y est sa limite, on a d(x, y ) ≥ ε, donc x = y . Ceci est absurde car y est une
valeur d’adhérence de (xn ), d’où le résultat. 

Remarque 3. Ce résultat peut s’avérer parfois bien pratique.


Proposition 10. Soient E1, . . . , En un nombre fini d’espaces métriques. L’ensemble E =
E1 × · · · × En est compact si et seulement si E i est compact pour tout i.
Remarque 4. Étant données deux suites (x n) et (y n) à valeurs dans un même compact E,
cette proposition montre l’existence d’une injection croissante ϕ de N dans N telle que les
deux suites (xϕ(n) ) et (yϕ(n) ) convergent (ceci car (xn , yn) est une suite dans le compact
E × E ).
 Proposition 11. Les parties compactes de R n (muni de la distance produit usuelle, avec
n ∈ N ∗) sont les fermés bornés de Rn .

Remarque 5. Comme nous le verrons à la partie 5.3, toutes les normes d’un R-e.v.n de
dimension finie sont équivalentes. Ceci nous permettra d’affirmer que le résultat de cette
proposition reste vrai dans tout R-espace vectoriel normé de dimension finie. Par contre,
il est faux en dimension infinie (voir le théorème de Riesz à l’exercice 9 de la page 56).
 Proposition 12. Soit (x n )n∈N une suite convergente d’un espace métrique (E, d),  sa
limite. Alors l’ensemble Γ = {x n, n ∈ N} ∪ {} est compact.
Démonstration. Nous sommes dans un des rares cas ou il est plus facile de montrer la compacité
de Γ en prouvant qu’il vérifie la propriété de Borel-Lebesgue (la caractérisation par la propriété
de Bolzano-Weierstrass donnerait ici une preuve bancale et peu satisfaisante). Cette preuve
faisant appel uniquement à la topologie de E montrera qu’en fait le résultat reste vrai dans un
espace topologique séparé général.
Soit (Oi )i∈I un recouvrement de Γ par des ouverts de E . Comme  ∈ Γ, il existe i 0 ∈ I tel
que  ∈ Oi0 , et comme O i0 est ouvert et que (x n ) tend vers ,
∃N ∈ N, ∀n > N, x n ∈ O i0 .
Pour tout n ≤ N , il existe jn ∈ I tel que x n ∈ Oj n. En notant J = {j n, n ≤ N } ∪ {i0 }, on
s’aperçoit que Γ ⊂ ∪j ∈J O j , et comme J est fini, le résultat est prouvé. 
3. ESPACES COMPACTS 31

3.4. Compacts et applications continues


Proposition 13. Soient (E, d) un espace métrique compact, (F, δ) un espace métrique
et une application continue f : E → F . Alors f (E ) est compact.
Remarque 6. Cette proposition entraı̂ne que l’image par f de tout fermé de E ( où E est
compact) est un fermé (une application vérifiant cette propriété est dite fermée). Ceci est
faux en général. En corollaire (et grâce à la proposition 9), on a le résultat qui suit.
Proposition 14. Soit f : (E, d) → (F, δ) une application continue et bijective. Si
(E, d) est compact, alors f −1 : F → E est continue (en d’autres termes, f est un
homéomorphisme).
Remarque 7. Pour toute fonction continue et bijective f : I → J , où I et J sont des
intervalles de R, l’application réciproque f −1 est continue. Il suffit en effet d’appliquer la
proposition précédente à la restriction de f à tout segment (donc compact) de I.
 Proposition 15. Soit f : (E, d) → R une application continue, où (E, d) est compact.
Alors f est bornée et atteint ses bornes. En d’autres termes, il existe c, d ∈ E tels que
f (c) = inf f (x) et f (d) = sup f (x).
x∈E x∈E

Remarque 8. On verra (voir le théorème 3 page 41) que si f : R → R est une application
continue, l’image par f d’un intervalle est un intervalle (c’est le théorème des valeurs
intermédiaires). Avec cette dernière proposition, on en déduit que l’image par f d’un
intervalle fermé borné est un intervalle fermé borné.
 Théorème 2 (de Heine). Soient (E, d) et (F, δ) deux espaces métriques, E étant com-
pact, et f : E → F une application continue. Alors f est une application uniformément
continue.

3.5. Exercices
Exercice 1. 1/ a) Soient (E, d) et (F, δ) deux espaces métriques et f : E → F une
application continue telle que pour tout compact K de F , f −1(K) est compact. Montrer
que f est une application fermée (rappel : une application f est dite fermée si l’image de
tout fermé par f est un fermé).
b) Existe-t-il des applications continues qui ne sont pas fermées ?
2/ (Application). On fixe un entier naturel non nul n et on note
Rn [X] = {P ∈ R[X ] | deg(P ) ≤ n}.
Montrer que l’ensemble Γn des polynômes unitaires de degré n de Rn [X] dont toutes les
racines sont réelles est un fermé du R-e.v.n Rn [X].

Solution. 1/ a) Soit Γ un fermé de E . Soit (yn ) = (f (xn ))n∈N une suite de f (Γ) qui converge
vers un point y ∈ F . Il s’agit de montrer que y ∈ f (Γ).
L’ensemble K = {yn , n ∈ N} ∪ {y } est un compact de F (voir la proposition 12), l’ensemble
f −1 (K ) est donc compact. La suite (xn) de Γ prenant ses valeurs dans f −1 (K ), on peut en
extraire une sous-suite convergente (x ϕ(n))n∈N . Notons x sa limite. Comme Γ est fermé, x =
limn→∞ xϕ(n) appartient à Γ, et par continuité de f ,
f (x) = lim f (xϕ(n)) = lim yϕ(n) = y,
n→∞ n→∞
donc y = f (x) ∈ f (Γ). L’ensemble f (Γ) est donc bien fermé.
b) Il existe des applications continues non fermées. Par exemple, f : R → R x → e x est
continue et pourtant, f (R) = ]0, +∞[ n’est pas fermé.
32 1. TOPOLOGIE SUR LES ESPACES MÉTRIQUES ET LES ESPACES VECTORIELS NORM ÉS

2/ Le résultat sera prouvé si on montre que l’application continue


f : Rn → Rn [X] (λ1 , . . . , λ n ) → (X − λ1 ) · · · (X − λn )
est fermée, puisque Γ n = f (Rn ). En vertu du résultat de la question 1/ a), il suffit pour cela de
prouver que pour tout compact K de Rn[X], f −1 (K ) est un compact.
Donnons nous un compact K de Rn [X]. L’ensemble f−1 (K) est déjà un fermé puisque K
est fermé et que f est continue. Montrons qu’il est borné. Comme K est compact, K est borné,
donc il existe M > 0 tel que
 n
∀P = ak X k ∈ K, P  = sup |a k| ≤ M.
k=0 k

Soit (λ 1, . . . , λn ) ∈ f −1 (K), de sorte que P = f (λ 1 , . . . , λn ) = ni=1(X − λ i) ∈ K . Écrivons
P = Xn + a 1 X n−1 + · · · + an . Si λ est une racine de P , nous allons prouver que |λ| ≤ 1 + P .
Si |λ| ≤ 1, c’est terminé, sinon l’égalité P (λ) = 0 entraı̂ne
+∞ 
   1
 a2 an−1 an  1
|λ| =  a1 + + · · · + n−2 + n−1  ≤ P  k
= P  ,
λ λ λ |λ| 1 − 1/|λ|
k=0
d’où on tire facilement |λ| ≤ 1 + P  ≤ 1 + M .
Ainsi, nous avons prouvé que si (λ1 , . . . , λ n) ∈ f −1 (K), alors pour tout i, |λ i| ≤ 1 + M . En
d’autres termes, f −1 (K ) est borné. L’ensemble f −1(K), fermé borné de Rn , est donc compact,
d’où le résultat.

 Exercice 2 (Un pr écompact complet est compact). On rappelle qu’un espace


métrique est précompact si pour tout ε > 0, il existe un recouvrement fini de cet espace
par des boules (ouvertes) de rayon ε.
Montrer qu’un espace métrique précompact et complet est compact.
Solution. Soit (E, d) un tel espace métrique. Soit (x n ) une suite de E. Il s’agit de montrer que
l’on peut extraire de (xn) une sous
sous-suite
suite convergente.
Comme E est précompact, on peut recouvrir E par un nombre fini de boules de rayon 1. Il
existe donc une de ces boules, B(a0 , 1), qui contient la valeur x n pour une infinité d’indices n. On
peut donc construire une sous-suite (xϕ 0(n) )n∈N de (xn ) telle que pour tout n, xϕ 0(n) ∈ B(a 0, 1).
Comme B(a 0 , 1) ⊂ E , B(a0, 1) est précompact. On peut donc recouvrir B(a 0 , 1) par un
nombre fini de boules de rayon 1/2. Il existe donc une de ces boules, B (a1, 1/2), telle que
B(a 0, 1) ∩ B(a1 , 1/2) contienne la valeur xϕ0 (n) pour une infinité d’indices n. On peut donc
construire une sous-suite (xϕ0 ◦ϕ1(n) )n∈N de (xϕ0 (n) ) telle que
∀n ∈ N, xϕ 0◦ϕ1 (n) ∈ B(a 0 , 1) ∩ B(a1 , 1/2).
En procédant par récurrence, on peut ainsi construire, pour tout entier naturel p, une sous-
suite (xϕ0 ◦···◦ϕp (n) )n∈N et une boule B(a p , 1/2p ) telles que

∀n ∈ N, xϕ0 ◦···◦ϕp (n) ∈ B(ak , 1/2 k).
0≤k≤p

Simplifions les notations. En notant ψp = ϕ0 ◦ · · · ◦ ϕ p pour tout p, on a construit, pour tout


p, une sous-suite (xψ p (n)) de (x ψp−1 (n)) telle que
∀n ∈ N, x ψp(n) ∈ B(ap, 1/2 p ).
On va maintenant construire une sous-suite de (xn ) par la méthode dite du procédé diagonal
(à retenir).
— On choisit xψ(0) ∈ {xψ 0(n) , n ∈ N}.
— On choisit ensuite x ψ(1) ∈ {xψ1 (n) , n ∈ N} avec ψ(1) > ψ(0).
— ...
— L’indice ψ(p) étant construit, on choisit xψ(p+1) ∈ {xψp+1 (n) , n ∈ N} avec ψ (p +1) > ψ(p).
3. ESPACES COMPACTS 33

— ...
Ainsi construite, la suite (xψ(p) )p∈N est une sous-suite de (xn ). Si maintenant p ∈ N et q ≥ p, on
a
xψ(q ) ∈ {xψq(n) , n ∈ N} ⊂ {x ψp(n) , n ∈ N} ⊂ B(ap , 1/2p ),
ce qui prouve que
1
∀p, q ∈ N (p ≤ q ), d(xψ(p),ψ (q )) ≤
.
2p−1
La suite (x ψ(p) ) est donc de Cauchy, et comme E est complet, elle converge. D’où le résultat.
Remarque. La réciproque est immédiate : un compact est précompact et complet.

 Exercice 3 (Distance entre deux parties). 1/ Soit (E, d) un espace métrique.


a) Soient K 1 et K 2 deux compacts de E . Montrer l’existence de x 1 ∈ K1 et x2 ∈ K2 tels
que d(x1 , x 2) = d(K 1, K 2 ) (= inf x∈K1 d(x, y )).
y∈K2
b) Soient K un compact de E et F un fermé de E. Si K ∩F = ∅, montrer que d(K, F ) = 0.
Ce résultat subsiste t-il si K est seulement supposé fermé ?
2/ Ici, E = Rn (n ∈ N∗ ), muni de la métrique usuelle.
a) Soit F un fermé non borné de E et f : F → R une application continue telle que
lim f (x) = +∞.
x→∞
x∈F

Montrer qu’il existe x ∈ F tel que f (x) = inf y∈F f (y ).


b) Soient K un compact de E et F un fermé de E . Montrer
(∃x ∈ K, ∃y ∈ F ), d(x, y) = d(K, F ).
Ceci reste t-il vrai si E est un R-e.v.n de dimension infinie ?
Solution. 1/ a) L’application d : K 1 × K 2 → R (x, y ) → d(x, y) est continue sur le compact
K 1 × K 2, donc elle atteint son minimum sur K1 × K2 , c’est-à-dire qu’il existe (x1 , x 2) ∈ K 1 × K2
tel que d(x 1, x 2 ) = min (x,y)∈K 1×K 2 d(x, y) = d(K1 , K2).
Remarque. En remplaçant K1 par {x}, on voit qu’en particulier, si K2 est compact,
∃y ∈ K2 , d(x, K2 ) = d(x, y ).

b) L’application x → d(x, F ) est continue (voir l’exercice 4 page 17) sur le compact K, donc
∃x0 ∈ K, d(x0 , F ) = inf d(x, F ) = d(K, F ).
x∈K

Si d(K, F ) = 0, alors d(x0, F ) = 0 donc x 0 ∈ F car F est fermé. Ceci est absurde car K ∩ F = ∅.
Donc d(K, F ) = 0.
Lorsque K est seulement supposé fermé, le résultat est faux. Par exemple, dans R2, les
ensembles K = {(x, y ) ∈ R 2 | y ≤ 0} et F = {(x, y ) ∈ R 2 | y ≥ e x } sont fermés, disjoints, et
pourtant d(K, F ) = 0.
2/ a) C’est classique. Fixons a ∈ F et considérons l’ensemble Γ = {x ∈ F | f (x) ≤ f (a)}. Comme
f est continue, Γ = f −1(]−∞, f(a)]) est un fermé de F , donc de E. Comme lim x→∞ f (x) = +∞,
x∈F
Γ est borné. Finalement, Γ est compact (fermé borné de Rn ). De plus, Γ = ∅ puisque a ∈ Γ.
Donc il existe x ∈ Γ tel que f (x) = infy∈Γ f (y ). Or, par construction de Γ,
inf f (y ) = inf f (y ),
y∈Γ y∈F

donc f (x) = infy∈F f (y ) avec x ∈ F .


34 1. TOPOLOGIE SUR LES ESPACES MÉTRIQUES ET LES ESPACES VECTORIELS NORM ÉS

b) L’application x → d(x, F ) étant continue sur le compact K ,


∃x ∈ K, d(x, F ) = inf d(y, F ) = d(K, F ).
y∈K

De même l’application F → R y → d(x, y) est continue. Si F est bornée, F est compact


(fermé borné de R n) et le résultat résulte de 1/ a). Sinon F n’est pas borné, et on a alors
lim y→∞ d(x, y) = +∞ ce qui en vertu de la question précédente permet d’affirmer
y∈F

∃y ∈ F, d(x, y ) = inf d(x, z) = d(x, F ) = d(K, F ),


z ∈F
d’où le résultat.
Ceci est faux lorsque E est de dimension infinie. Prenons par exemple pour E l’e.v des suites
bornées (un ) de R, normé par (u n ) = supn∈N |u n|. Désignons pour tout n par X n la suite
dont tous les termes sont nuls sauf le n-ième qui vaut 1 + 1/2n. L’ensemble F = {X n , n ∈ N}
est fermé (si p = q on a Xp − Xq ≥ 1, donc toute suite convergente donc de Cauchy dans F
devient stationnaire à partir d’un certain rang, donc converge dans F ). Si K = {(0)} (ou (0)
désigne la suite nulle) il est clair que d(K, F ) = 1 et pourtant on a d(K, Xn ) = 1 + 1/2 n > 1
pour tout n.

 Exercice 4. Soit (E, d) un espace métrique compact et f : E → E une application


vérifiant
∀(x, y ) ∈ E2 , x = y, d(f (x), f (y)) < d(x, y ). (∗)

a) Montrer que f admet un unique point fixe, que nous noterons α.


b) Soit x0 un point quelconque de E . On définit la suite (xn ) par récurrence grâce à la
relation xn+1 = f (x n ). Montrer que (xn) converge vers α.
c) Ces résultats restent-ils vrais si l’on suppose seulement E complet ?
Solution. C’est classique ! C’est une généralisation du théorème du point fixe dans un compact.
a) L’application E → R x → d(x, f (x)) est continue (composée d’applications continues) sur
le compact E, donc
∃α ∈ E, d(α, f (α)) = inf d(x, f (x)).
x∈E
Supposons α = f (α). Posons β = f (α). D’après (*),
d(β, f(β )) = d(f (α), f (β)) < d(α, β) = d(α, f (α)),
ce qui contredit la définition de α. On doit donc avoir α = f (α).
Montrons maintenant que α est l’unique point fixe de f . Supposons β = f (β ) avec β = α .
On a d(f (α), f(β)) = d(α, β), ce qui est absurde d’après (*) appliqué au couple (α, β ).
b) Posons un = d(α, x n). S’il existe n0 ∈ N tel que un 0 = α, alors u n = un0 = α pour tout
n ≥ n0 et le résultat est évident. Sinon,
∀n ∈ N, u n+1 = d(f (α), f(x n )) < d(α, xn ) = un,
c’est-à-dire que la suite (u n) décroı̂t strictement. Comme elle est minorée par 0, elle converge.
Notons  sa limite. Il s’agit de montrer que  = 0.
Supposons  > 0. Comme (un ) décroı̂t, on a un ≥  pour tout n ∈ N. De plus, (x n) est une
suite du compact E , on peut donc en extraire une sous-suite convergente (xϕ(n) ). Notons β la
limite de cette dernière. Alors limn→∞ d(α, xϕ(n) ) = d(α, β) = . De plus, f étant continue, on a
lim d(α, f (xϕ(n) )) = d(α, f (β)).
n→∞
Cette dernière assertion est une absurdité puisque
d(α, f (β)) = d(f (α), f (β)) < d(α, β) =  et ∀n, d(α, f (xϕ(n) )) = d(α, xϕ(n)+1 ) ≥ .
On doit donc avoir  = 0, d’où le résultat.
3. ESPACES COMPACTS 35

c) Le résultat est faux si E est seulement supposé complet. Par exemple, la fonction
1
f : R → R f (x) = 1 si x < 0, f(x) = x + si x ≥ 0,
1+x
(voir la figure ci contre) est continue, sans point fixe et vérifie l’hypothèse (*) (immédiat par
l’inégalité des accroissements finis).

y
y=x

0 x

Figure 1. Le graphe de l’application sans point fixe f

 Exercice 5 (Isom étries d’un compact). Soit (E, d) un espace métrique compact et
f : E → E une application continue, vérifiant
∀(x, y ) ∈ E 2, d(f (x), f (y)) ≥ d(x, y ). (∗)

a) Montrer que f est une isométrie, c’est-à-dire


∀(x, y ) ∈ E 2 , d(f (x), f (y)) = d(x, y ).

b) Montrer que f est bijective.


c) Si f est bijective, montrer que f est encore une isométrie si elle vérifie
∀(x, y ) ∈ E 2, d(f (x), f (y)) ≤ d(x, y ),

Solution. a) Soient x, y ∈ E. On définit deux suites (xn)n∈N et (yn ) n∈N par x n = f n (x) et
yn = f n (y ) pour tout n ∈ N ( où fn désigne la composée n fois de l’application f avec elle
même). Nous allons construire deux sous-suites (xψ(n) ) et (yψ(n) ) qui convergent respectivement
vers x et y.
La suite (xn , y n) n∈N prend ses valeurs dans le compact E × E . On peut donc en extraire une
sous-suite convergente (xϕ(n) , yϕ(n) ). Quitte à extraire encore une sous-suite de cette dernière,
on peut même supposer
∀n ∈ N, ϕ(n + 2) − ϕ(n + 1) > ϕ(n + 1) − ϕ(n). (∗∗)
De l’inégalité (*), on tire
∀n ∈ N, d(x, xϕ(n+1)−ϕ(n) ) ≤ d(f ϕ(n)(x), f ϕ(n) (xϕ(n+1)−ϕ(n))) = d(x ϕ(n), xϕ(n+1) )
et comme (xϕ(n)) converge, on en déduit lim n→∞ xϕ(n+1)−ϕ(n) = x. Ainsi, en posant ψ (n) =
ϕ(n + 1) − ϕ(n), (xψ(n) ) est une sous-suite de (xn ) d’après (**), et elle converge vers x. Pour les
mêmes raisons, (yψ(n) ) est une sous-suite de (yn) qui converge vers y .
Ceci étant, pour n ≥ 1, on a d’après (*)
d(x, y) ≤ d(f (x), f(y)) ≤ d(x ψ(n) , yψ(n) ),
36 1. TOPOLOGIE SUR LES ESPACES MÉTRIQUES ET LES ESPACES VECTORIELS NORM ÉS

et en faisant tendre n vers +∞, on en déduit d(f (x), f(y)) = d(x, y).
b) L’injectivité résulte de l’hypothèse (*). Pour montrer la surjectivité de f , nous donnons deux
méthodes.
Première méthode. Cette méthode utilise la solution que nous avons donnée à la question a).
Fixons x ∈ E . Si xn = f n (x), nous avons vu plus haut qu’il existe une sous-suite (x ψ(n)) de
(xn) qui converge vers x. Autrement dit, lim n→∞ f (xψ(n)−1 ) = x, donc x ∈ f (E ). Comme E est
compact et que f est continue, f (E ) est compact donc fermé. Donc x ∈ f (E ) = f (E ), et ceci
étant vrai pour tout x ∈ E , f est surjective.
Seconde méthode. Raisonnons par l’absurde et supposons f (E ) =  E . Comme f (E ) est fermé (car
compact), E f (E) est ouvert. De plus E f (E ) =  ∅ donc il existe une boule ouverte B(x, ρ)
incluse dans E f (E ). Ainsi, pour tout y ∈ f (E ), d(x, y) ≥ ρ. Considérons la suite (f n(x))n∈N .
Pour tout p, q ∈ N, p > q, on a
d(fp (x), f q (x)) ≥ d(f p−q (x), x) ≥ ρ,
donc (f n (x)) n’a aucune sous-suite convergente, ce qui est absurde. On a donc f (E) = E .
c) L’application f est bijective et continue (car 1-lipschitzienne) sur un compact, donc f −1 : E →
E est continue (voir le théorème 14 page 31) et vérifie d’après les hypothèses
∀(x, y ) ∈ E 2, d(f −1 (x), f −1 (y)) ≥ d(f (f−1 (x)), f(f −1 (y))) = d(x, y ).
L’application f −1 vérifie donc les hypothèses de la question a), c’est donc une isométrie. On en
déduit que f est une isométrie.

Exercice 6. Soient (E, d) et (F, δ) deux espaces métriques et f : E → F une application


injective. Prouver que f est continue si et seulement si pour tout compact K de E , f (K )
est compact. Le résultat subsiste-t-il si on ne suppose pas f injective ?
Solution. La condition nécessaire est immédiate, c’est du cours !
Passons à la condition suffisante. Soit x ∈ E et (xn)n∈N une suite de E qui converge vers
x. Il s’agit de montrer que lim n→∞ f (xn) = f (x). L’ensemble K = {x n, n ∈ N} ∪ {x} est
un compact de E (voir la proposition 12 page 30), donc K  = f (K ) est compact. Notons g
la restriction de f à K . Comme f est injective, g est une bijection de K sur K . L’application
g−1 : K  → K est continue car pour tout fermé F de K , F est compact donc (g −1)−1 (F ) = g (F )
est compact, donc fermé. L’ensemble K  étant compact, (g −1) −1 = g est continue. En particulier,
limn→∞ g (x n) = g (x), ce qui s’écrit aussi limn→∞ f (xn) = f (x), d’où le résultat.
Si f n’est pas injective, le résultat est faux. Par exemple, l’application
f : R→R x → 0 si x<0 x → 1 si x ≥ 0
vérifie f (K ) est compact pour tout compact K de R, et pourtant f n’est pas continue.

Exercice 7. Soit [a, b] un segment de R (avec a < b), (E, d) un espace métrique et
f : [a, b] → E une application. On suppose que pour tout x ∈ ]a, b[, les limites à gauche
f (x−) et à droite f (x+) de f en x existent, et que f (a+) et f (b−) existent.
a) Si x ∈ ]a, b[, on note ω(f, x) = max{d(f (x−), f (x)), d(f (x), f (x+))}. Soit ε > 0 et
A ε = {x ∈ ]a, b[ | ω(f, x) ≥ ε}.
Montrer que l’ensemble Aε est fini.
b) Montrer que l’ensemble des points de discontinuité de f est au plus dénombrable.
c) Soit ε > 0 et x ∈ ]a, b[ tel que ω (f, x) < ε. Montrer
∃α > 0, ∀y, z ∈ ]x − α, x + α[ d(f (y ), f (z)) < 2ε.
3. ESPACES COMPACTS 37

d) Soit ε > 0 et ]c, d[ ⊂ [a, b] tel que ω (f, x) < ε sur ]c, d[. Montrer
∃α > 0, ∀x, y ∈ ]c, d[ , d(x, y ) < α, d(f (x), f (y)) < 2ε.

Solution. a) Raisonnons par l’absurde et supposons Aε infini. Comme Aε est inclus dans le
compact I, il en existe un point d’accumulation x ∈ I . Ainsi, il existe une suite (x n) de points
distincts de Aε qui converge vers x.
S’il existe une infinité de valeurs de n pour lesquelles xn < x, on peut, quitte à extraire une
sous-suite de (xn ), supposer xn < x pour tout n. On a x ∈ ]a, b] et d’après les hypothèses, f
admet une limite à gauche en x. Il existe donc α > 0 tel que
∀y ∈ ]x − α, x[, d(f (y), f (x−)) < ε/3. (∗)
Comme (x n ) converge vers x par valeurs inférieures, il existe un entier naturel n, désormais fixé,
tel que x − α < xn < x. D’après (*), on a l’inégalité
∀y ∈ ]x − α, x n [, d(f (y), f (xn )) ≤ d(f (y ), f(x−)) + d(f (x−), f (xn )) < 2ε/3,
et en faisant tendre y vers xn , on en déduit d(f (xn −), f(xn )) ≤ 2ε/3. De même l’inégalité
∀y ∈ ]xn , x[, d(f (xn ), f(y)) ≤ d(f (x n ), f(x−)) + d(f (x−), f (y)) < 2ε/3,
conduit par passage à la limite à d(f (xn), f(x n+)) ≤ 2ε/3. On a donc ω(f, xn ) ≤ 2ε/3. Donc
xn ∈ A ε , ce qui est absurde.
Dans le cas où il existe une infinité de valeurs de n pour lesquelles xn > x, on aboutirait de
la même manière à une absurdité en utilisant la continuité à droite de f en x.
L’ensemble Aε est donc fini.
b) La fonction f est discontinue en x ∈ ]a, b[ si et seulement si ω (f, x) > 0. L’ensemble D des
points de ]a, b[ où f est discontinue est donc égal à ∪n∈N ∗A1/n , réunion dénombrable d’ensembles
finis, donc D est au plus dénombrable. En ajoutant éventuellement les points a et b, on en déduit
que l’ensemble des points de discontinuité de f sur [a, b] est au plus dénombrable.
c) Posons η = ε − ω(f, x) > 0. Comme f est continue à gauche et à droite en x, il existe α > 0
tel que d(f (y), f (x−)) < η pour y ∈ ]x − α, x[ et d(f (y), f (x+)) < η pour y ∈ ]x, x + α[. On a
alors
∀y ∈ ]x − α, x + α[, d(f (y ), f (x)) < ε. (∗∗)
En effet, si x − α < y < x il suffit d’écrire d(f (y), f (x)) ≤ d(f (y ), f(x−)) + d(f (x−), f(x)) <
η + ω (f, x) = ε, et le cas x < y < x + α se traite de la même manière. Lorsque y = x, le résultat
(**) est évident.
On conclue facilement car d’après (**)
∀y, z ∈ ]x − α, x + α[, d(f (y ), f(z)) ≤ d(f (y ), f(x)) + d(f (x), f(z)) < 2ε.

d) Supposons le résultat faux. Alors il existe une suite (x n, yn) de ]c, d[ telle que (x n − yn ) tende
vers 0 et telle que d(f (xn ), f(yn )) ≥ 2ε pour tout n. La suite (xn ) est à valeur dans le compact
[c, d], donc on peut en extraire une sous-suite
sous suite convergente (xϕ(n) ) donc la limite x appartient à
[c, d]. Comme (xn − y n) tend vers 0, la suite (y ϕ(n)) converge également vers x.
Plusieurs cas se présentent, selon que la limite x est dans l’intérieur de [c, d] ou au bord.
(i) Le cas x ∈ ]c, d[ est absurde car (xϕ(n) ) et (y ϕ(n) ) tendent vers x et d(f (xϕ(n)), f(yϕ(n) )) ≥
2ε pour tout n, ce qui est absurde d’après le résultat de la question précédente.
(ii) Si x = c, alors comme f est continue à droite en c il existe α > 0 tel que d(f (y ), f (c+)) < ε
pour tout y ∈ ]c, c + α[. Comme (xϕ(n)) et (y ϕ(n) ) convergent vers c à droite, il existe n tel
que xϕ(n) et y ϕ(n) appartiennent à ]c, c + α[, donc d(f (x ϕ(n) ), f(yϕ(n) )) ≤ d(f (x ϕ(n)), f(c+)) +
d(f (c+), f(y ϕ(n) )) < 2ε, ce qui est absurde car d(f (xϕ(n)), f(y ϕ(n) )) ≥ 2ε.
(iii) Pour le cas x = d, on aboutit également à une absurdité en procédant de la même
manière, en utilisant la continuité à gauche de f en d.
Ainsi, on a aboutit à une absurdité dans tous les cas, donc le résultat est bien vérifié.
38 1. TOPOLOGIE SUR LES ESPACES MÉTRIQUES ET LES ESPACES VECTORIELS NORM ÉS

Remarque. Le résultat de cet exercice permet de montrer qu’une fonction f à valeurs


dans un e.v.n complet E et vérifiant les hypothèses de l’exercice est une fonction réglée
(voir le théorème 4 page 99).

Exercice 8. Soit X un espace métrique compact. On note C(X, R) l’algèbre des appli-
cations continues de X dans R.
a) Soit I =
 C (X, R) un idéal de C(X, R). Montrer
∃s ∈ X, ∀f ∈ I, f (s) = 0.

b) Caractériser les idéaux maximaux de C(X, R) (i. e. les idéaux I =


 C (X, R) tels que les
seuls idéaux contenant I sont I et C (X, R)).
c) Déterminer la forme des morphismes d’algèbre de C(X, R) dans R.
Solution. a) Raisonnons par l’absurde. Supposons que
∀s ∈ X, ∃f s ∈ I, f s (s) = 0.
Pour tout s ∈ X , comme fs est continue, il existe un voisinage ouvert Ωs de s tel que ∀x ∈
Ωs, f s (x) = 0.
Les (Ωs )s∈X recouvrent le compact X . On peut donc en extraire un sous-recouvrement fini
(Ωsi )1≤i≤n. Comme I est un idéal, pour tout i, f 2si ∈ I. Or fs2i prend des valeurs strictement

positives sur Ω si . Les (Ωs i )1≤i≤n recouvrant X , la fonction f = ni=1 f 2si ne s’annule pas sur
X. Comme I est un idéal, on a f ∈ I, et 1 = (1/f ) · f ∈ I donc I = C (X, R). Ceci contredit
l’hypothèse I =  C (X, R). Il existe donc s ∈ X tel que pour tout f ∈ I, f (s) = 0.
b) Remarquons déjà que pour tout s ∈ X , l’ensemble J s = {f ∈ C(X, R) | f (s) = 0} est un
idéal de C (X, R). Nous allons montrer que les Js sont les idéaux maximaux de C (X, R).
Pour tout s ∈ X , l’ensemble J s est un idéal maximal de C (X, R). En effet, J s =  C (X, R) et

si I est un idéal contenant J s et différent de C(X, R), alors d’après a), il existe s ∈ X tel que
I ⊂ Js . On a alors J s ⊂ Js  , ce qui n’est possible que si s = s , et donc I = Js .
Soit I un idéal maximal de C (X, R). On a I = C (X, R) donc d’après a), il existe s ∈ X tel
que I ⊂ Js. Comme I est maximal et que Js =  C (X, R), on a I = Js .
c) Soit ϕ un morphisme d’algèbre de C(X, R). Supposons ϕ non nulle. L’ensemble Ker ϕ est un
idéal de C (X, R), différent de C (X, R) car ϕ = 0. D’après a), il existe s ∈ X tel que Ker ϕ ⊂ J s . La
forme linéaire f → f (s) s’annule donc sur l’hyperplan Ker ϕ, et l’ensemble des formes linéaires
s’annulant sur un hyperplan formant une droite du dual (voir le tome d’algèbre), il existe λ ∈ R
tel que f (s) = λϕ(f ) pour tout f ∈ C(X, R). Soit f ∈ C(X, R) tel que f (s) = 0. On a
λ(f2 )(s) = λ 2ϕ(f 2) = λ 2 ϕ(f ) 2 = f 2 (s),
donc λ = 1. Finalement ϕ(f ) = f(s) pour tout f .
En résumé, les morphismes d’algèbre de C(X, R) dans R sont l’application nulle et les mor-
phismes de la forme f → f (s) , où s ∈ X .

4. Espaces connexes
4.1. Définitions
Proposition 1. Soit (E, d) un espace métrique. Les assertions suivantes sont équiva-
lentes.
(i) Il n’existe pas de partition de E en deux ouverts disjoints non vides.
(ii) Il n’existe pas de partition de E en deux fermés disjoints non vides.
4. ESPACES CONNEXES 39

(iii) Les seules parties ouvertes et fermées de E sont ∅ et E .


Démonstration. (i) =⇒ (ii). Supposons E = F1 ∪F 2 où F1 et F2 sont deux fermés de E vérifiant
F1 ∩ F2 = ∅. Les ouverts O1 = E F 1 et O2 = E F2 vérifient O 1 ∪ O2 = ∅ et O 1 ∩ O 2 = ∅, donc
d’après (i), O1 = ∅ ou O 2 = ∅, donc F1 = E ou F 2 = E , et comme F1 ∪ F 2 est une partition de
E , F1 = ∅ ou F 2 = ∅.
(ii) =⇒ (iii). Soit A une partie ouverte et fermée de E . Alors l’ensemble B = E A est
ouvert et fermé, et comme E = A ∪ B avec A ∩ B = ∅, on en déduit d’après (ii) que A = ∅ ou
B = ∅, c’est-à-dire A = ∅ ou A = E .
(iii) =⇒ (i). Supposons E = O1 ∪ O 2 où O1 et O2 sont deux ouverts disjoints de E.
L’ensemble O1 est fermé car O1 = E O 2, donc O1, ouvert et fermé de E , vérifie O 1 = ∅ ou
O1 = E, d’où (i). 

Définition 1. Un espace métrique vérifiant l’une des assertions de la proposition précé-


dente est dit connexe.
Remarque 1. La notion de connexité est une notion topologique. Tous les résultats de cette
partie restent vrais dans un espace topologique général (à l’exception du théorème 5).
Parties connexes. Soient (E, d) un espace métrique et A une partie de E . On munit
A de la distance d induite sur A. On veut savoir à quelle condition A est connexe. Les
propriétés de la topologie induite sur A entraı̂nent facilement le résultat qui suit.
Proposition 2. La partie A de E est connexe si et seulement si l’une des deux conditions
suivantes est réalisée.
(i) Si A ⊂ O1 ∪ O 2 où O 1 et O2 sont deux ouverts de E vérifiant A ∩ O1 ∩ O2 = ∅, alors
(A ∩ O 1 = ∅ et A ⊂ O 2) ou (A ∩ O2 = ∅ et A ⊂ O1 ).
(ii) Si A ⊂ F1 ∪ F 2 où F 1 et F 2 sont deux fermés de E vérifiant A ∩ F 1 ∩ F2 = ∅, alors
(A ∩ F1 = ∅ et A ⊂ F 2) ou (A ∩ F2 = ∅ et A ⊂ F 1).
Exemple 1. L’ensemble Q des rationnels n’est pas un connexe de R car si on se donne
a ∈ RQ, on a Q ⊂ ] − ∞, a[ ∪ ]a, +∞[.
4.2. Propriétés
Théorème 1. Soit f : (E, d) → (E  , d ) une application continue. Si E est connexe,
alors f (E ) est connexe.
Démonstration. Soit B une partie ouverte et fermée de f (E ). Il existe un ouvert O et un fermé F
de E tels que B = O ∩ f (E ) = F ∩ f (E ). On a alors f −1(B) = f −1(O) = f −1 (F ), donc f −1(B )
est fermé et ouvert dans E , et E étant connexe, f −1(B) = ∅ ou f −1(B) = E , c’est-à-dire B = ∅
ou B = f (E). 

Caractérisation des connexes. On note D = {0, 1} muni de la distance discrète δ


(δ(0, 0) = δ(1, 1) = 0 et δ(0, 1) = δ(1, 0) = 1). L’espace métrique (D, δ) n’est pas connexe
puisque D = {0} ∪ {1} est réunion de deux fermés disjoints. Grâce à cet espace métrique,
nous allons donner une caractérisation commode des connexes.
 Proposition 3. Un espace métrique (E, d) est connexe si et seulement si toute applica-
tion continue f : E → D est constante.
Démonstration. Condition nécessaire. Si E est connexe, alors f (E ) est connexe, donc constante
car D n’est pas connexe et a deux éléments.
Condition suffisante. Si E n’est pas connexe, on peut écrire E = O1 ∪ O2 où O1 et O2 sont deux
ouverts non vides disjoints de E . Soit f : E → D une application définie par f (x) = 0 si x ∈ O1 ,
40 1. TOPOLOGIE SUR LES ESPACES MÉTRIQUES ET LES ESPACES VECTORIELS NORM ÉS

f (x) = 1 si x ∈ O 2. Elle est continue (l’image réciproque de tout ouvert est un ouvert), ce qui
est contraire aux hypothèses puisqu’elle est non constante. Finalement, E est connexe. 

Proposition 4. Soit A une partie connexe d’un espace métrique (E, d). Si une partie B
de E vérifie A ⊂ B ⊂ A, alors B est connexe.
Démonstration. Soit f : B → D = {0, 1} une application continue. Comme A est connexe, f|A
est constante, par exemple f |A = 0. Soit x 0 ∈ B . Il existe un voisinage V de x 0 dans B tel que
1
∀x ∈ V, δ(f (x), f(x0 )) < ,
2
ce qui montre que pour tout x ∈ V , f (x) = f (x0). Or B ⊂ A, donc V ∩ A = ∅. Si on choisit
x1 ∈ V ∩ A, on a f (x 0) = f (x1) = 0. Ainsi, f = 0, d’où le résultat d’après la proposition 3. 
Dans le cas général, une réunion de connexes n’est pas connexe (par exemple, {0} et
{1} sont connexes dans R, mais {0, 1} = {0} ∪ {1} n’est pas connexe). On a cependant le
résultat qui suit.
Proposition 5. Soit (Ci) i∈I une famille de connexes d’un espace métrique (E, d) telle
que
∃i0 ∈ I, ∀i ∈ I, Ci ∩ Ci0 =
 ∅.
Alors ∪i∈I Ci est connexe.
Démonstration. Soit f : ∪i∈IC i → D = {0, 1} une application continue. Pour tout i, Ci est
connexe donc f|C i est constante. En particulier, f|C i est connexe, par exemple f|Ci = 0. Soit
0 0
x ∈ ∪i∈IC i et soit i ∈ I tel que x ∈ C i. Comme C i ∩ C i0 = ∅, on peut trouver x0 ∈ C i ∩ C i0 .
On a alors f (x) = f (x0) = 0 car f |Ci est constante. Ainsi, f est constante sur ∪ i∈I C i, d’où le
résultat d’après la proposition 3. 

Remarque 2. Si (C i )i∈I est une famille de connexes telle que ∩ i∈IC i =


 ∅, alors ∪ i∈I Ci est
connexe (si x ∈ ∩ i∈I Ci , tous ces connexes ont une intersection non vide avec le connexe
{x}).
Dans le cas dénombrable, on a également le résultat qui suit.
Proposition 6. Soit (Ci) i∈I une famille au plus dénombrable de connexes (avec I =
{0, 1, . . . , p} ou I = N) telle que pour tout i ∈ I , i = 0, Ci−1 ∩ C i = ∅. Alors ∪i∈I Ci est
connexe.
Démonstration. Soit f : ∪i∈I C i → D = {0, 1} une application continue. Pour tout i, f|Ci est
constante, et comme Ci−1 ∩ Ci =  ∅ pour i = 0, on a f|Ci−1 = f|Ci pour tout i = 0. En procédant
par récurrence sur i, on en déduit que f|C i = f |C0 pour tout i. Ainsi, f est constante, d’où le
résultat. 

Proposition 7. Soient (E1 , d 1), . . . , (E n , dn) des espaces métriques (en nombre fini).
L’espace produit E = E1 × · · · × E n est connexe si et seulement si E i est connexe pour
tout i.
Démonstration. Condition nécessaire. Soit i ∈ {1, . . . , n} et soit f : Ei → D = {0, 1} une
application continue. La projection pi de E sur Ei étant continue, l’application f ◦ pi : E → D
est continue, et comme E est connexe, f ◦ pi est constante. Donc f est constante et Ei est
connexe.
Condition suffisante. Soient (x1, . . . , x n ) et (y 1, . . . , yn ) ∈ E et une application continue f : E →
D . L’application f1 : E1 → D x → f (x, x2 , . . . , xn ) est continue, donc constante car E1 est
connexe. En particulier, f (x1 , x2 , . . . , xn ) = f (y1 , x2, . . . , xn ). En itérant le procédé sur chacun
des connexes E2, . . . , E n , on obtient f (x1 , . . . , xn ) = f (y1 , . . . , yn ). L’application f est donc
constante, donc E est connexe d’après la proposition 3. 
4. ESPACES CONNEXES 41

Composantes connexes. Soit (E, d) un espace métrique. On considère sur (E, d) la


relation
(x R y ) ⇐⇒ (∃C connexe de E tel que x ∈ C et y ∈ C ).
On a affaire à une relation d’équivalence car
— Si x R y , y R x (immédiat).
— Pour tout x, x R x car {x} est un connexe.
— Si x R y et y R z , il existe deux connexes C et C  tels que x ∈ C , y ∈ C et y ∈ C ,
z ∈ C . On a C ∩ C  = ∅ donc d’après la remarque 2, C ∪ C  est connexe, et comme
x ∈ C ∪ C  et z ∈ C ∪ C  , x R z .
Si x ∈ E , sa classe d’équivalence, notée x,
˙ est la réunion des connexes contenant x. C’est
donc un connexe d’après la proposition 5 (en effet, tous les connexes contenant x ont tous
une intersection non vide avec le connexe {x}). L’ensemble x˙ s’appelle une composante
connexe de E.
Les composantes connexes de E forment donc une partition de E . L’espace métrique
E est connexe si et seulement s’il n’a qu’une seule composante connexe.
Les composantes connexes de E sont des fermés de E . Considérons en effet une com-
posante connexe ẋ de E . La proposition 4 entraı̂ne le fait que ẋ est connexe, c’est donc
un connexe contenant x, donc x˙ ⊂ ẋ et ẋ = ẋ.
Si les composantes connexes de E sont en nombre fini, elles sont ouvertes comme
complémentaires d’une réunion finie de fermés de E.
Connexes de R.
 Théorème 2. Les parties connexes de R sont les intervalles de R.
Démonstration. Un connexe de R est un intervalle. En effet, si C ⊂ R n’est pas un intervalle, il
existe (a, b) ∈ C 2 et x ∈ R tels que a < x < b et x ∈ C . Mais alors C ⊂ ]−∞, x[ ∪ ]x, +∞[, donc
C n’est pas connexe.
Réciproquement, montrons qu’un intervalle I de R est connexe. C’est un peu plus délicat.
Si I est un singleton, c’est immédiat.
Si I = ]a, b[ avec −∞ ≤ a < b ≤ +∞, considérons une application continue f : I → D =
{0, 1}. Si f n’est pas constante, il existe x, y ∈ I vérifiant a < x < y < b tels que f (x) =
 f (y ),
par exemple f (x) = 0 et f (y ) = 1. Considérons l’ensemble
Γ = {z ∈ I | z ≥ x et ∀t ∈ [x, z ], f (t) = 0}.
L’ensemble Γ est non vide car x ∈ Γ. De plus, Γ est majoré car pour tout z ∈ Γ, z ≤ y . Soit
c = sup Γ. Comme f est continue, f (c) = 0. De même, f étant continue en c,
1
∃ε > 0, ∀t ∈ [c, c + ε], δ(f (t), f(c)) < ,
2
donc pour tout t ∈ [c, c + ε], f (t) = 0, ce qui montre que c + ε ∈ Γ. Ceci contredit le fait que
c = sup Γ. Finalement, l’application f est constante et I est connexe d’après la proposition 3.
Tout intervalle I de R non réduit à un singleton étant compris entre un intervalle ouvert et
son adhérence, on en conclut avec la proposition 4 que tout intervalle de R est connexe. 

 Théorème 3 (des valeurs intermédiaires). Soit I un intervalle de R et f : I → R


une application continue. Alors f (I ) est un intervalle.
Démonstration. Le théorème précédent assure la connexité de I , donc f (I ) est connexe d’après
le théorème 1, c’est donc un intervalle. 

Remarque 3. — Une autre manière d’écrire le résultat est la suivante. Si f (a) ≤ f (b)
(resp. f (b) ≥ f (a)) avec a < b, alors pour tout γ tel que f (a) ≤ γ ≤ f (b) (resp.
f (b) ≤ γ ≤ f (a)), il existe c ∈ [a, b] tel que f (c) = γ .
— Avec la proposition 15, on en conclut que l’image d’un segment de R par f est un
segment de R.
42 1. TOPOLOGIE SUR LES ESPACES MÉTRIQUES ET LES ESPACES VECTORIELS NORM ÉS

4.3. Connexité par arcs


Définition 2. Soit (E, d) un espace métrique. On appelle chemin de E toute application
γ : [0, 1] → (E, d), continue. L’image γ ([0, 1]) du chemin s’appelle un arc, γ (0) l’origine
de l’arc, γ (1) son extrémité.
Définition 3. Soit (E, d) un espace métrique. On dit que E est connexe par arcs si pour
tout (a, b) ∈ E2 , il existe un arc inclus dans E d’origine a et d’extrémité b.

 Théorème 4. Un espace connexe par arcs est connexe.


Démonstration. Soient E un espace connexe par arcs et f : E → D = {0, 1} une application
continue. Soit (a, b) ∈ E2. Il existe une application continue γ : [0, 1] → E telle que γ (0) = a
et γ (1) = b. L’application f ◦ γ : [0, 1] → D est continue, donc constante car [0, 1] est connexe.
Donc (f ◦ γ )(0) = (f ◦ γ )(1), c’est-à-dire f (a) = f (b). L’application f est donc constante, donc
E est connexe d’après la proposition 3 

Remarque 4. — La connexité par arcs est surtout une notion pratique pour montrer
qu’un espace est connexe. En termes intuitifs, un espace est connexe par arcs si
on peut toujours relier deux de ses points par une courbe continue, ce qui fait
du concept de connexité par arcs une notion moins abstraite que la notion de
connexité.
— La réciproque du théorème 4 est fausse (voir l’exercice 5 page 44). Elle est cependant
vraie dans un ouvert d’un R-e.v.n (voir le théorème 5).
— La connexité par arcs est, comme la connexité, une notion topologique.
Connexité par lignes brisées dans un espace vectoriel normé. Dans tout ce
paragraphe, E désigne un R-espace vectoriel normé.
Définition 4. Soient (a, b) ∈ E 2 . On appelle segment d’extrémité a et b l’ensemble
{λa + (1 − λ)b, λ ∈ [0, 1]}, et on le note [a, b].
Définition 5. On appelle ligne brisée (ou ligne polygonale) de E joignant deux points a
et b de E tout ensemble de la forme ∪1≤i≤n [x i−1, x i ] où n ∈ N∗ , x0 = a et xn = b et pour
tout i, xi ∈ E.
Définition 6. Une partie A de E est dite connexe par lignes brisées si pour tout (a, b) ∈
A2 , il existe une ligne brisée incluse dans A joignant a et b.
Remarque 5. Il est clair qu’une ligne brisée est un arc. Une partie de E connexe par lignes
brisées est donc connexe par arcs.
Exemple 2. Un R-e.v.n E est connexe par lignes brisées (donc connexe par arcs et connexe)
car pour tout (a, b) ∈ E2 , [a, b] ⊂ E .
Théorème 5. Soit E un R-e.v.n. Une partie ouverte Ω de E est connexe si et seulement
si elle est connexe par lignes brisées.
Démonstration. La condition suffisante découle du théorème 4, une partie connexe par ligne
brisée étant connexe par arcs.
Montrons la condition nécessaire. Soit Ω un ouvert connexe non vide de E . Soit x0 ∈ Ω et
notons T x0 l’ensemble des points de Ω que l’on peut joindre à x 0 par une ligne brisée contenue
dans Ω.
— On a T x0 = ∅ car x 0 ∈ Tx0 .
— L’ensemble T x0 est ouvert. En effet, si x ∈ Tx 0, on a x ∈ Ω donc il existe ρ > 0 tel que
la boule B(x, ρ) soit incluse dans Ω. Ainsi, pour tout y ∈ B(x, ρ), [x, y] ⊂ Ω et comme
x ∈ Tx0 , on en déduit y ∈ T x0 .
4. ESPACES CONNEXES 43

— L’ensemble T x 0 est fermé dans Ω. En effet, si x ∈ T x0 ∩ Ω, il existe une boule B(x, ρ)


incluse dans Ω telle que B(x, ρ) ∩ Tx0 = ∅. Si on choisit y ∈ B(x, ρ) ∩ Tx 0, on a alors
[y, x ] ⊂ Ω, donc x ∈ Tx0 .
Finalement, T x0 = ∅ est ouvert et fermé dans le connexe Ω, donc Tx 0 = Ω, d’où le résultat. 

Remarque 6. On en déduit avec la remarque 5 que tout ouvert connexe d’un e.v.n est
connexe par arcs. Attention, ce dernier résultat n’est plus vrai si on remplace E par un
espace métrique quelconque.
4.4. Exercices
Exercice 1. Soient (E, d) un espace métrique, B une partie connexe de E et A une

◦   
partie de E telle que B ∩ A = ∅ et B ∩ (E A) = ∅. Montrer que B ∩ Fr(A) =
 ∅ ( où
Fr(A) désigne la frontière de A).

Solution. On note X = E X le complémentaire de X dans E . Remarquons déjà que



◦ ◦ ◦ ◦    ◦
E Fr(A) = (AA) = (A ∩ A) = (A) ∪ A = A ∪ (E A) = A ∪ (E A).

  
◦ ◦
Ceci étant, supposons B ∩ Fr(A) = ∅. Alors B ⊂ E Fr(A) = A ∪ ( E A). Les ensembles A

   ◦
et E A étant deux ouverts disjoints de E , on en déduit, B étant connexe, que B ∩ A = ∅ ou

  
B ∩ (E A) = ∅, ce qui absurde par hypothèse. Donc B ∩ Fr(A) = ∅.

Exercice 2. Soient A et B deux espaces métriques connexes. Soit X ⊂ A, X = A et


Y ⊂ B , Y = B . Montrer que C = (A × B )(X × Y ) est un connexe de A × B .

Solution. D’après les hypothèses, il existe a0 ∈ A tel que a 0 ∈ X et b0 ∈ B tel que b0 ∈ Y .


Soit f : C → {0, 1} une application continue. Il s’agit de montrer que f est constante. Soit
(a, b) un point de C . Comme (a, b) ∈ X × Y , on a a ∈ X ou b ∈ Y , par exemple a ∈ X . Ainsi,
pour tout y ∈ B , (a, y) ∈ C. La restriction de f à {a} × B est continue. Comme {a} et B sont
connexes, la restriction de f à {a} × B est constante. En particulier, f (a, b) = f (a, b0). Comme
b 0 ∈ Y , on montrerait de même f (a, b 0) = f (a0 , b0 ). Donc f (a, b) = f (a0 , b 0), et ceci pour tout
(a, b) ∈ C, donc f est constante. D’après la proposition 3 page 39, C est donc connexe.

Exercice 3. Soit (E, d) un espace métrique connexe et F un fermé de E . On suppose


que Fr(F ) est connexe. Montrer que F est connexe. Le résultat subsiste-t-il si F n’est pas
supposé fermé ?
Solution. En vertu de la proposition 3 page 39, il suffit de montrer que toute application
continue f : F → D = {0, 1} est constante. Soit f une telle application. La restriction f|Fr(F ) de
f à Fr ( F ) ⊂ F est continue, et comme Fr(F ) est connexe par hypothèse, f|Fr(F ) est constante.
Supposons par exemple que f (x) = 0 pour tout x ∈ Fr(F ). On définit une extension g de f sur
E par
g (x) = f (x) si x ∈ F, g(x) = 0 si x ∈ E F.
On va montrer que g est continue, il suffit donc de montrer que l’image réciproque de tout fermé
de D est fermée dans E . Les fermés de D sont ∅, D , {0} et {1}. On a
g −1(∅) = ∅, g−1 (D) = E, g−1({1}) = f −1 ({1})
44 1. TOPOLOGIE SUR LES ESPACES MÉTRIQUES ET LES ESPACES VECTORIELS NORM ÉS

donc ces images réciproques sont fermées (f −1 ({1}) est fermé dans F car f est continue, donc
dans E car F est fermé dans E ). Il reste à monter que g −1 ({0}) est fermé dans E. Partant de

g−1({0}) = (E F ) ∪ f −1 ({0}) = (EF ) ∪ f−1 ({0}),

(cette dernière égalité est vraie car EF = (E F ) ∪ Fr(F ) et Fr(F ) ⊂ f −1 ({0})), on voit que
g−1 ({0}), union de deux fermés dans E, est fermé dans E .
Finalement on a démontré que g : E → D est continue. Or E est connexe, donc g est
constante, donc f = g|F est constante. Ceci conclut à la connexité de F .
Le résultat est faux si F n’est pas supposé fermé. Par exemple, si E = R et F = R∗ ,
Fr(F ) = {0} est connexe et pourtant F = ] − ∞, 0[ ∪ ]0, +∞[ n’est pas connexe.

Exercice 4. On note U = {z ∈ C, |z | = 1} et on considère f : U → R une application


continue. Montrer qu’il existe deux points diamétralement opposés du cercle unité U ayant
même image par f .
Solution. Il s’agit de prouver l’existence de z ∈ U tel que f (z ) = f (−z ). Considérons l’application
g: U→R z → f (z ) − f (−z ).
La continuité de g et le caractère connexe de U (U est connexe comme image du connexe [0, 2π ]
par l’application continue θ → e iθ) entraı̂ne que g(U) est connexe dans R, c’est donc un intervalle
de R. Or g (1) = f (1) − f (−1) = −g (−1). L’intervalle g(U) contient donc deux valeurs opposées,
donc 0 ∈ g(U). En d’autres termes, il existe z ∈ U tel que g (z ) = 0 = f (z ) − f (−z ), d’où le
résultat.

sous ensemble de R2 défini par


Exercice 5. Soit Γ le sous-suite
   
 
Γ= ({x} × R+ ) ∪ ({x}×] − ∞, 0[ ) .
x∈Q x∈RQ

a) Montrer que Γ est un connexe de R2.


b) Montrer que Γ n’est pas connexe par arcs.
Solution. a) Pour prouver la connexité de Γ, il suffit de montrer que toute fonction continue
f : Γ → {0, 1} est constante.
Pour tout x ∈ Q, l’ensemble {x} × R + est une demi droite, donc connexe par arc, donc
connexe, donc :

∀x ∈ Q, f est constante sur {x} × R+ , de même
(∗)
∀x ∈ RQ, f est constante sur {x}×] − ∞, 0[
On peut donc définir une application g : R → {0, 1} comme suit :
— Si x ∈ Q, g (x) est la valeur prise par f sur {x} × R + ,
— si x ∈ RQ, g(x) est la valeur prise par f sur {x}×] − ∞, 0[.
Si on montre que g est constante. on aura prouvé, en vertu de (*), que f est constante et donc
que Γ est connexe.
L’application g est localement constante. En effet.
– Soit x0 ∈ Q. On a (x 0, 0) ∈ Γ et comme f est continue en ce point et que {f (x 0 , 0)} est un
ouvert de {0, 1}, f −1 ({f (x0 , 0)}) est un ouvert de Γ, de sorte que
∃α > 0, ∀(x, y ) ∈ ( ]x 0 − α, x 0 + α [×] − α, α[ ) ∩ Γ, f(x, y) = f (x0 , 0).
Donc si x ∈ ]x0 − α, x 0 , x 0 + α[, g (x) = g (x0 ) (en effet, si x est rationnel, g (x) = f (x, 0) =
f (x0 , 0) = g (x0 ) et si x est irrationnel, g (x) = f (x, −α/2) = f (x0, 0) = g (x 0)).
4. ESPACES CONNEXES 45

– Soit x0 ∈ RQ. Fixons y0 < 0. L’application f étant continue en (x0 , y0) ∈ Γ, on en déduit
comme précédemment que
∃α > 0, ∀x ∈ ]x0 − α, x 0 + α [ ∩(RQ), f (x, y0 ) = f (x0 , y 0 ) = g (x0).
Grâce à (*), on en déduit
∀x ∈ ]x0 − α, x0 + α [ ∩(RQ), f|{x}×]−∞,0[ = g (x0 ).
On en tire g (x) = g (x0) pour tout x ∈ ]x 0 − α, x 0 + α[. (En effet, si x est irrationnel, g(x) =
f|{x}×]−∞,0[ = g (x 0 ). Si x est rationnel, la densité de RQ dans Q permet d’affirmer l’existence
d’une suite (xn ) de ]x 0 − α, x 0 + α [ ∩(RQ) qui converge vers x, et
g (x) = f (x, 0) = lim f (x n , −1/n) = g (x0 ).)
n→∞

– Ainsi, g est localement constante autour des rationnels et des irrationnels, donc sur R, donc
continue sur R. Comme g est à valeurs dans {0, 1} et que R est connexe, on en déduit que g est
constante sur R et le résultat.
b) Raisonnons par l’absurde et supposons Γ connexe par arcs. En particulier, il existe un arc
contenu dans Γ qui relie (0, 0) et un point (x0 , y0) ∈ (RQ)×] − ∞, 0[. En d’autres termes, il
existe une application continue
γ : [0, 1] → R2 t → (γ 1(t), γ 2(t))
telle que γ(0) = (x0 , y0 ) ∈ (RQ)×] − ∞, 0[, γ (1) = (0, 0) et ∀t ∈ [0, 1], γ (t) ∈ Γ.
L’application γ 1 étant continue, l’ensemble γ−1
1 ({x0 }) est un fermé, non vide car il contient
−1
x0. Donc α = sup γ 1 ({x0 }) existe et γ1 (α) = x 0. On a α < 1 car γ 1(1) = 0. En résumé,
γ 1 (α) = x0 et ∀t ∈ ]α, 1], γ1 (t) = x0 . (∗∗)
Comme γ1 (α) = x 0 et que γ (α) ∈ Γ, on a γ2(α) < 0. La continuité de γ 2 assure l’existence
d’un ε > 0 tel que γ 2(t) < 0 pour tout t ∈ [α, α + ε], et comme γ (t) ∈ Γ, on en déduit
∀t ∈ [α, α + ε], γ 1(t) ∈ Q. (∗∗∗)
D’après le théorème des valeurs intermédiaires, γ1 ([α, α + ε]) est un intervalle, non réduit à
un singleton d’après (**). Ceci est absurde d’après (***), d’où le résultat.

Exercice 6 (Espaces bien enchaı̂nés). Soient (E, d) un espace métrique. Soit ε > 0.
On dit que E est ε-enchaı̂né si pour tout (a, b) ∈ E2 , il existe n ∈ N ∗ et des points
x0 , . . . , xn de E tels que x0 = a, xn = b et d(xi, xi−1 ) < ε pour tout i ∈ {1, . . . , n}. On dit
que E est bien enchaı̂né si il est ε-enchaı̂né pour tout ε > 0.
a) Si E est connexe, montrer que E est bien enchaı̂né.
b) Si E est compact et si E est bien enchaı̂né, montrer que E est connexe. Ce résultat
reste-t-il vrai si E n’est pas supposé compact ?
Solution. a) Soit ε > 0. On définit la relation d’équivalence Rε sur E par : x Rε y si et seulement
s’il existe n ∈ N∗ et x0, . . . , x n ∈ E tels que x 0 = x, x n = y et d(xi , x i−1 ) < ε pour tout
i ∈ {1, . . . , n} (on vérifie facilement que l’on a bien affaire à une relation d’équivalence). Soit
x ∈ E . Nous montrons que la classe ẋ de x est ouverte et fermée.
La classe ẋ est ouverte. En effet, soit y ∈ ẋ. Pour tout z ∈ B(y, ε), on a y Rε z donc z ∈ ẏ = ẋ.
Ainsi, B(y, ε ) ⊂ x˙ , et ceci pour tout y ∈ ẋ, donc ẋ est ouvert.
La classe ẋ est également fermée car c’est le complémentaire de l’ouvert ∪y∈ẋ ẏ.
Ainsi, ẋ, ouverte et fermée dans le connexe E, est égale à E . Ceci montre que E est ε-enchaı̂né
pour tout ε > 0, d’où le résultat.
b) Raisonnons par l’absurde en supposant E non connexe. On peut écrire E = F 1 ∪ F2 où F1
et F2 sont deux fermés non vides disjoints. Un fermé dans un compact est compact, donc F1
et F 2 sont compacts. Il existe donc a1 ∈ F1 et a2 ∈ F2 tels que d(a1 , a2 ) = d(F1, F2 ) (voir
46 1. TOPOLOGIE SUR LES ESPACES MÉTRIQUES ET LES ESPACES VECTORIELS NORM ÉS

l’exercice 3 page 33). Comme F1 ∩ F 2 = ∅, a1 = a 2 donc ε = d(a 1 , a2 ) > 0, de sorte que


d(x, y) ≥ ε pour tout (x, y) ∈ F1 × F2 . Comme E est bien enchaı̂né, on peut trouver une chaı̂ne
(x0 , . . . , xn) ∈ E n+1 telle que x0 ∈ F 1 , xn = y0 ∈ F 2, et d(x i , xi−1 ) < ε pour tout i ∈ {1, . . . , n}.
Il existe un indice i ∈ {1, . . . , n} tel que xi−1 ∈ F1 et xi ∈ F 2 (ceci car x0 ∈ F1 et xn ∈ F 2), et
alors d(xi−1 , x i) ≥ d(F1, F 2 ) = ε, ce qui est absurde. L’espace métrique E est donc connexe.
Si E n’est pas supposé compact, le résultat est faux. Par exemple Q (muni de la métrique
d(x, y) = |x − y|) est bien enchaı̂né (car dense dans R) mais non connexe.

Exercice 7. Soit (E, d) un espace métrique compact et (un) n∈N une suite de E telle
que limn→∞ d(un , un+1 ) = 0. Montrer que l’ensemble des valeurs d’adhérence de (u n) est
connexe.
Solution. Pour tout p ∈ N, on note Ap = {u n , n ≥ p}. On sait que l’ensemble Γ des valeurs
d’adhérence de (un ) est égal à ∩p∈NAp . C’est donc un fermé, et comme E est compact, Γ est
compact.
Supposons Γ non connexe, de sorte que l’on peut écrire Γ = A ∪ B où A et B sont deux
fermés non vides disjoints de Γ. Comme Γ et compact, A et B sont même compacts et donc
(voir l’exercice 3 page 33) α = d(A, B ) > 0 puisque A et B sont disjoints. Notons
 α   α  α
A  = x ∈ E | d(x, A) < = B x, et B  = x ∈ E | d(x, B) < .
3 3 3
x∈A

Les ensembles A et B sont ouverts donc K = E(A ∪ B ) est fermé dans le compact E, donc
 

compact. Nous allons montrer que (un ) admet au moins une valeur d’adhérence dans K , ce qui
sera une absurdité car Γ ∩ K = ∅.
Par hypothèse, limn→∞ d(un , un+1 ) = 0 donc
α
∃N0 ∈ N, ∀n ≥ N0 , d(un , un+1 ) < . (∗)
3

α/3
α/3

A α B
E
A

B
K = E (A  ∪ B )

Figure 2. Les compacts A, B , E et K = E (A ∪ B )

Soit N un entier quelconque supérieur à N0 . Donnons nous x0 ∈ A. Le point x 0 est valeur


d’adhérence de (un ) donc il existe n1 > N tel que d(x0 , u n1 ) < α/3, donc u n1 ∈ A . Si y 0 ∈ B,
y0 est aussi valeur d’adhérence de (u n) donc il existe n2 > n1 tel que d(y0, u n2 ) < α/3, de sorte
que un2 ∈ B . Notons maintenant n0 le premier entier supérieur à n1 tel que u n0 ∈ A (un tel
entier existe car un2 ∈ A  ). On a un 0 −1 ∈ A, donc d’après (*)
α
d(un0 , B) ≥ d(un 0 −1, B) − d(un0 −1 , u n0 ) ≥ d(A, B ) − d(un 0−1, A) − d(un 0−1 , un 0 ) > ,
3
ce qui prouve que un 0 ∈ B . Comme de plus un0 ∈ A, on a un 0 ∈ K.
Résumons. Nous venons de montrer que pour tout N ≥ N 0, il existe n0 ≥ N tel que un0 ∈ K.
On peut donc construire une sous-suite (u ϕ(n) ) de (un) qui prend ses valeurs dans K. Comme
5. ESPACES VECTORIELS NORMÉS (E.V.N) 47

K est compact, (u ϕ(n)) admet au moins une valeur d’adhérence dans K , donc (un ) admet au
moins une valeur d’adhérence dans K . Ceci est impossible car Γ ∩ K = ∅. L’ensemble Γ est
donc connexe.

Exercice 8 (R et R2 ne sont pas hom éomorphes). Il est bien connu que R et R2


sont deux ensembles équipotents, autrement dit il existe une bijection f de R dans R2 .
Montrer qu’une telle bijection ne peut pas être un homéomorphisme.

Solution. Supposons l’existence d’un homéomorphisme f de R sur R2 . L’ensemble R 2{0} est


connexe (car trivialement connexe par arcs), et f −1 étant continue, f −1 (R2 {0}) est un connexe
de R. Or f étant une bijection,
f −1 (R 2{0}) = R{f −1(0)} = ] − ∞, f −1 (0)[ ∪ ]f −1 (0), +∞[
n’est pas connexe. Ceci est absurde, d’où le résultat.

Exercice 9 (Théorème de Darboux, preuve topologique). Soit f : R → R


une application dérivable sur R (mais pas forcément de classe C1 ). Soit I un intervalle
ouvert non vide de R. En considérant l’ensemble
 
f (x) − f (y ) 2
Γ= , (x, y ) ∈ I et x < y ,
x−y
montrer que f (I) est un intervalle de R.

Solution. Notons A l’ensemble {(x, y) ∈ I 2 , x < y}. Cet ensemble est connexe (il est même
convexe). L’application
f (x) − f (y)
F : A → R (x, y) →
x−y
est continue, donc f (A) = Γ, image continue d’un connexe, est connexe.
On a f  (I ) ⊂ Γ. En effet, si x ∈ I et si ( xn) est une suite de I qui tend vers x telle que
xn > x pour tout n, on a f (x) = lim n→∞ F (x, xn ).
D’après le théorème des accroissements finis, on a Γ ⊂ f (I). Finalement, Γ ⊂ f (I ) ⊂ Γ,
donc f (I ) est connexe d’après la proposition 4. Les connexes de R étant les intervalles, on en
déduit le résultat.
Remarque. Ce résultat est prouvé avec des moyens différents à l’exercice 4 page 80.

5. Espaces vectoriels normés (e.v.n)


Cette section propose une étude générale des espaces vectoriels normés. L’étude parti-
culière de la topologie des espaces de matrices est traitée à la partie 4.3 du tome Algèbre.
5.1. Rappels
On rappelle qu’un K-espace vectoriel E (avec K = R ou C) est normé s’il est muni
d’une norme  . . En notant d(x, y) = x − y , on fait de E un espace métrique. Sauf
mention contraire, c’est toujours cette distance qui est utilisée dans un e.v.n. On dit qu’un
e.v.n est un espace de Banach s’il est complet.
Deux normes  . 1 et  . 2 sur E sont dite équivalentes si
∃a > 0, ∃b > 0, ∀x ∈ E, a x 1 ≤ x2 ≤ b x1 .
48 1. TOPOLOGIE SUR LES ESPACES MÉTRIQUES ET LES ESPACES VECTORIELS NORM ÉS

Deux normes équivalentes définissent des distances équivalentes. Sur un plan topologique,
et lorsque l’on travaille avec des suites de Cauchy, il est indifférent de prendre l’une ou
l’autre de ces normes.
L’application x → |x| est une norme sur R, et z → |z | une norme sur C. Plus
généralement, pour tout α ≥ 1, si x = (x1 , . . . , xn ) ∈ Kn (avec K = R ou C),
 n 1/α

x  α = |x i |α
i=1
n
définit une norme sur K (voir la conséquence de l’inégalité de Minkowsky, page 98), de
même que x∞ = supi |x i|.
Dans un e.v.n, les opérations (x, y ) → x + y et (λ, x) → λx sont continues. La norme
est également une fonction continue.
Si V est un s.e.v. de l’e.v.n E , alors son adhérence V est un s.e.v de E . En particulier,
un hyperplan de E est soit fermé, soit dense dans E.
5.2. Continuité des applications linéaires
Dans toute cette sous-suite
sous partie, E et F désignent deux K-e.v.n (avec K = R ou C).
Théorème 1. Soit f ∈ L(E, F ) une application linéaire de E dans F . Les assertions qui
suivent sont équivalentes.
(i) f est continue sur E ;
(ii) f est continue en 0 ;
(iii) f est bornée sur la boule unité fermée Bf (0, 1) de E ;
(iv) f est bornée sur la sphère unité S(0, 1) de E ;
(v) il existe M > 0 tel que f (x) ≤ M x pour tout x ∈ E ;
(vi) f est lipschitzienne ;
(vii) f est uniformément continue sur E .
Remarque 1. — Dans la pratique, on utilise souvent les assertions (iv) et (v).
— De même, on montre qu’une application multilinéaire de E1 × · · · × En dans F est
continue si et seulement si
∃M > 0, ∀x = (x1 , . . . , xn) ∈ E1 × · · · × En , f (x) ≤ M x1  · · · xn.
Définition 1. L’ensemble des applications linéaires continues de E dans F est noté
Lc(E, F ). On norme Lc (E, F ) en posant
∀f ∈ Lc(E, F ), |||f ||| = sup f (x) = sup f (x),
x=1 x≤1

ce qui fait de L c(E, F ) un e.v normé.


Remarque 2. Le réel |||f ||| est le plus petit réel positif M tel que f (x) ≤ M x pour
tout x ∈ E . En particulier, pour tout x ∈ E , f (x) ≤ |||f ||| · x. Sauf mention contraire,
c’est la norme x  → |||f ||| qui est choisie sur Lc(E, F ).
Proposition 1. Soient E , F et G trois e.v.n, soient f ∈ Lc (E, F ) et g ∈ L c(F, G). Alors
g ◦ f ∈ Lc(E, G) et |||g ◦ f ||| ≤ |||g ||| · |||f |||.
En particulier, la norme ||| . ||| sur Lc (E) = Lc (E, E ) en fait une algèbre normée (voir
plus bas).

 Théorème 2. Si F est un espace de Banach, l’e.v.n Lc (E, F ) est un espace de Banach.


5. ESPACES VECTORIELS NORMÉS (E.V.N) 49

Démonstration. Soit (fn ) une suite de Cauchy de L c (E, F ).


On construit la limite éventuelle de (fn ). Fixons un élément quelconque x de E . On a
∀(p, q ) ∈ N2, f p (x) − fq (x) ≤ |||fp − fq || · x,
donc la suite (fn (x)) est de Cauchy dans F , et comme F est complet, elle converge. On note
f (x) sa limite. On définit ainsi une application f de E dans F .
On vérifie que f ∈ Lc (E, F ). L’application f est linéaire car si x, y ∈ E et λ, µ ∈ K,
f (λx + µy) = lim fn (λx + µy) = lim [λf n (x) + µfn (y)] = λf (x) + µf (y ).
n→∞ n→∞
Elle est continue. En effet, (fn ) étant de Cauchy, elle est bornée donc il existe M > 0 tel que
|||fn ||| ≤ M pour tout n. Si x ∈ E est fixé, on a donc fn (x) ≤ M x pour tout n, donc à la
limite lorsque n → ∞, on obtient f (x) ≤ M x. Ainsi, f est bien continue.
Il nous reste à prouver que (fn ) converge vers f au sens de la norme ||| . |||. Soit ε > 0. Il existe
N ∈ N tel que pour tout p, q ≥ N , |||fp − fq ||| ≤ ε. En fixant x ∈ E , on a donc
∀p, q ≥ N, f p(x) − f q (x) ≤ |||fp − fq || · x ≤ εx.
En fixant p ≥ N et en faisant tendre q vers l’infini, on obtient fp (x) − f (x) ≤ εx. Ceci est
vrai pour tout x ∈ E , on a donc |||f p − f ||| ≤ ε, et ceci pour tout p ≥ N , d’où le résultat. 

Remarque 3. Il faut savoir refaire cette démonstration. Retenez en particulier les trois
étapes principales (faites l’exercice 1 page 21 qui est du même type).
Formes linéaires continues. Si E est un K-e.v.n (avec K = R ou C), l’e.v.n L c(E, K)
(ensemble des formes linéaires continues sur E ) est un s.e.v du dual E ∗ de E . On le note
souvent E  et on l’appelle dual topologique de E . Comme K est complet, E  est un espace
de Banach d’après le théorème 2. Dans un espace de Hilbert, E  et E sont isomorphes
(voir l’annexe B, question 3/ a) du problème 1 page 427).
Une forme linéaire f sur E est continue si et seulement si son noyau Ker f est un fermé
de E (voir l’exercice 7 page 55).
L’algèbre normée Lc (E ). On rappelle qu’une algèbre normée A est une algèbre (i. e.
un e.v muni d’une loi de produit interne, distributive par rapport à l’addition, vérifiant
(λu)v = λ(uv) = u(λv) pour tout u, v ∈ A et λ ∈ K) munie d’une norme  .  vérifiant
uv ≤ u·v  pour tout u, v ∈ A. Ceci assure la continuité de l’application (u, v ) → uv .
D’après la proposition 1, l’algèbre Lc (E) = Lc (E, E), munie de la norme ||| . |||, est une
algèbre normée. Si de plus E est un espace de Banach, Lc(E) est un espace de  Banach
d’après le théorème 2. Une conséquence importante
 est la suivante. Toute série un de
Lc(E) absolument convergente (i. e. telle que |||un ||| converge) est convergente (en effet,
la suite associée est de Cauchy).
Une application importante.
Proposition 2. Soient E un espace  de Banach et u ∈ Lc (E ) tel que |||u||| < 1. Alors
Id −u est inversible, son inverse est +∞
n=0 un
∈ L c E ).
(

Démonstration. La série u n converge absolument car |||un ||| ≤ |||u||| n et |||u||| < 1. Or
 +∞  +∞ +∞
  
(Id −u) un = un − un = Id,
n=0 n=0 n=1
+∞
de même ( n=0 un)(Id −u) = Id, d’où le résultat. 

Remarque 4. — On note G c(E) l’ensemble des endomorphismes inversibles u continus


et tels que u−1 est continu (en fait, si E est un espace de Banach et si u est
inversible et continu, u −1 est toujours continu — c’est le théorème de Banach, voir
l’annexe A, exercice 6 page 423). Si E est un espace de Banach, la proposition
50 1. TOPOLOGIE SUR LES ESPACES MÉTRIQUES ET LES ESPACES VECTORIELS NORM ÉS

précédente permet de montrer que Gc (E) est un ouvert de Lc (E) (voir l’exercice 4
page 52). 
— De manière générale, pour toute série entière n an z n (avec a n ∈ R ou a n ∈ C)
de
 rayon de convergence R > 0, on peut définir la série entière d’endomorphisme
n
a
n n u dès que |||u||| < R pour u ∈ Lc (E) avec E un espace de Banach. Par
exemple, l’exponentielle
∞ n d’un endomorphisme continu u ∈ Lc (E), est défini par la
série exp(u) = n=0 u /n! (voir le tome Algèbre, partie 4.3). On peut aussi utiliser
les séries entières d’endomorphisme pour démontrer des résultats intéressant. Par
exemple, on prouve l’existence, dès que |||u||| < 1, d’un
 endomorphisme continu
v ∈ Lc (E) tel que v2 = 1 + u ;√il suffit de choisir v = n an un où l es (na) sont les

coefficients de la série entière 1 + z = n a nz n.
5.3. Espaces vectoriels normés de dimension finie
Comme nous allons le voir, en dimension finie, “tout est continu”. Les notions de la
sous-suite
sous partie précédente présentent donc moins d’intérêt en dimension finie.
 Théorème 3. Dans un e.v.n de dimension finie, toutes les normes sont équivalentes.
Démonstration.Soit E un K-e.v (K = R ou C) de dimension finie, (e1, . . . , en ) une base de E.
Pour tout x = i xi ei , N0 (x) = sup i |xi| définit une norme sur E.
Montrons que toutes les normes sur E sont équivalentes
 à N0. Soit N une norme sur E. En
désignant par a le réel i N (ei ), on a pour tout x = i xi ei ∈ E
n
 n

N (x) ≤ N (xi e i ) = |xi| N (e i) ≤ a N 0 (x). (∗)
i=1 i=1
Munissons K n de la norme produit (x 1 , . . . , xn )∞ = sup i |xi |. L’application ϕ : (Kn ,  . ∞ ) →
(E, N0) (x 1 , . . . , xn ) → i xi e i est une isométrie, donc S = {x ∈ E | N 0(x) = 1} est un
compact de (E, N0 ) (image de la sphère unité de K n — qui est compacte car fermée bornée dans
Kn — par ϕ qui est continue car isométrique). D’après (*), on a |N (x) − N (y )| ≤ N (x − y) ≤
a N 0 (x − y), donc N : (E, N0) → R est continue. Comme S est un compact de (E, N0), on en
déduit b = infN 0 (x)=1 N (x) = 0. Ainsi,
 
x
∀x ∈ E, x = 0, N (x) = N0 (x) · N ≥ b N0 (x).
N0 (x)
Avec (*), on en déduit le théorème. 
Ce théorème est important. Il permet de choisir la norme que l’on veut sur un e.v.n
de dimension finie. Voici des corollaires.
Corollaire 1. Toute application linéaire d’un e.v.n de dimension finie dans un e.v.n
(quelconque) est continue.
Corollaire 2. Tout e.v.n de dimension finie est complet.
Corollaire 3. Tout s.e.v de dimension finie d’un e.v.n est fermé.
Corollaire 4. Les parties compactes d’un e.v.n de dimension finie sont les parties
fermées bornées.
Remarque 5. Tous ces corollaires sont faux en dimension infinie. Par exemple:
— Munissons l’espace vectoriel des polynômes réels R[X ] de la norme  i ai X i =
supi |ai|. L’application linéaire f : R[X ] → R[X ] P → P  n’est pas continue
(en effet, f (X n ) = n et X n  = 1 donc l’assertion (iii) du théorème 1 n’est pas
vérifiée).
— Tout e.v.n à base dénombrable n’est pas complet (voir l’exercice 8).
— La boule unité fermée d’un e.v.n de dimension infinie n’est pas compacte (théorème
de Riesz, voir l’exercice 9).
5. ESPACES VECTORIELS NORMÉS (E.V.N) 51

5.4. Convexes
Les parties convexes jouent un rôle important dans les e.v.n, en particulier dans les
espaces de Hilbert (voir l’annexe B, problème 1). Dans toute cette sous-suite
sous partie, E désigne
un K-e.v (avec K = R ou C).

Définition 2. Soient A, B ∈ E . On appelle segment d’extrémités A et B et on note


[A, B] l’ensemble {λ A + (1 − λ) B, λ ∈ [0, 1]}. Dans un e.v.n, un segment est fermé.

Définition 3. Soit C une partie de E . On dit que C est convexe si pour tout (A, B ) ∈ C2 ,
[A, B ] ⊂ C .

Remarque 6. — Un s.e.v est convexe.


— Une partie convexe est connexe par lignes brisées donc connexe. La même remarque
vaut pour les parties étoilées définies plus bas.
— Plus généralement, un convexe peut être défini dans un espace affine.

Définition 4. Une partie A de E est dite étoilée s’il existe P ∈ A tel que [P, M ] ⊂ A
pour tout M ∈ A (on dit alors que A est étoilée par rapport à P ). Un tel point P s’appelle
un centre de A.

Enveloppe convexe.

Définition 5. Soit A une partie de E . Il existe une plus petite partie convexe de E
contenant A. On l’appelle enveloppe convexe de A et on la note Conv(A). L’ensemble
Conv(A) est aussi l’ensemble des barycentres des points de A affectés de coefficients
positifs, i. e. Conv(A) est l’ensemble des x tels que
n
 n

+
∃x1 , . . . , xn ∈ A, ∃λ 1 , . . . , λn ∈ R , λ i = 1, x= λi xi .
i=1 i=1

5.5. Exercices
Exercice 1. Soient E un R-e.v.n et A, B deux parties de A. On note A + B = {a +
b, (a, b) ∈ A × B }.
a) Si A est ouvert (et B quelconque), montrer que A + B est ouvert.
b) Si A est compact et B fermé, montrer que A + B est fermé. Ce résultat subsiste-t-il si
A est seulement supposé fermé ?

Solution. a) On a A + B = ∪b∈B (A + {b}). Pour tout b ∈ B, il est clair que A + {b} est un
ouvert de E (si B(x, ρ) ⊂ A, B(x + b, ρ) ⊂ A + {b}). Donc A + B , réunion d’ouverts, est un
ouvert.
b) Soit (zn) = (x n + yn)n∈N une suite de A + B , convergente dans E vers z, où ( xn ) est une
suite de A et (yn ) une suite de B. La compacité de A entraı̂ne l’existence d’une sous-suite
(xϕ(n) ) de (xn ) qui converge dans A. Notons x ∈ A sa limite. Comme (zϕ(n) ) converge vers
z, (yϕ(n)) = (z ϕ(n) − xϕ(n) ) converge vers z − x. Comme B est fermé, y = z − x ∈ B . Ainsi,
z = x + y ∈ A + B , d’où le résultat.
Si A est seulement supposé fermé, le résultat est faux. Par exemple, dans le plan, les en-
sembles A = {(x, e x), x ∈ R} et B = R × {0} sont fermés et pourtant A + B = R×]0, +∞[ n’est
pas fermé. On peut donner un autre contre exemple dans R, en considérant les ensembles Z et
xZ (avec x ∈ RQ). Ces ensembles sont fermés, et Z + xZ est dense dans R (voir l’exercice 5
page 205). Si ce dernier était fermé, il serait égal à R tout entier, ce qui est impossible puisque
Z + xZ est dénombrable et que R ne l’est pas.
52 1. TOPOLOGIE SUR LES ESPACES MÉTRIQUES ET LES ESPACES VECTORIELS NORM ÉS

Exercice 2. Soit K un compact convexe d’un e.v.n et f : K → K une application


continue telle que
∀(x, y ) ∈ K2 , f (x) − f (y ) ≤ x − y .
Montrer que f admet au moins un point fixe.
Solution. Si f était k -contractante (avec k < 1), le résultat serait vrai (théorème du point fixe,
page 21). Fixons a ∈ K . La convexité de K nous invite à poser, pour tout entier naturel non
nul n  
1 1
fn : K → K x → a + 1 − f (x).
n n
L’application fn est bien à valeurs dans K car K est convexe. D’autre part,
   
∗ 2 1 1
∀n ∈ N , ∀(x, y) ∈ K , fn (x) − f n(y ) = 1 − f (x) − f (y) ≤ 1 − x − y ,
n n
donc fn est (1 − 1/n)-contractante. Comme K est compact, K est complet, et le théorème du
point fixe nous assure l’existence de xn ∈ K tel que fn (xn ) = xn. La suite (x n ) prend ses valeurs
dans le compact K , on peut donc en extraire une sous-suite convergente (xϕ(n) ), dont la limite
x appartient à K . Grâce à l’inégalité
1
f (x) − f n(xn ) ≤ f (x) − f n(x) + fn (x) − fn (xn) ≤ (a + f (x)) + x − x n
n
on voit que (fϕ(n) (xϕ(n) )) converge vers f (x). En passant à la limite dans l’égalité f ϕ(n) (xϕ(n)) =
xϕ(n), on en déduit f (x) = x, d’où le résultat.
Remarque. Plus généralement, on peut montrer que toute application continue d’un
convexe compact dans lui même admet au moins un point fixe (théorème de Brouwer).


Exercice 3. Montrer qu’un e.v.n E  est complet si et seulement si toute série un
absolument convergente (i. e. telle que un  converge) est convergente.
 
 n nécessaire. Si
Solution. Condition u n converge, alors la suite (Sn ) associée à la série un
définie par Sn = k=0 uk est de Cauchy car
 q 
    q
 
∀p < q, S p − S q  =  uk  ≤ u k .
 
k=p+1 k=p+1

Comme E est complet, (Sn) converge donc, c’est-à-dire un converge.
Condition suffisante. Soit (S n) une suite de Cauchy de E . D’après le critère de Cauchy,
— ∃ϕ(0) ∈ N, ∀n ≥ ϕ(0), S n − S ϕ(0)  ≤ 1, de même
— ∃ϕ(1) > ϕ(0), ∀n ≥ ϕ(1), S n − Sϕ(1) ≤ 12 ,
— on construit ainsi par récurrence ϕ(k ) > ϕ(k − 1) tel que ∀n ≥ ϕ(k ), S n − Sϕ(k)  ≤ 21k .
−k

Pour tout k ∈ N, on pose uk = Sϕ(k+1) − Sϕ( k) . Par construction, u k  ≤ 2 donc uk est
n
absolument convergente, donc convergente. Or k=0 uk = S ϕ(n+1) −Sϕ(0), donc (S ϕ(n)) converge.
Une suite de Cauchy admettant une sous-suite convergente converge, d’où le résultat.

Exercice 4. 1/ Soit A une R-algèbre normée (i. e. une algèbre munie d’une norme  . 
vérifiant xy ≤ x · y  pour tout (x, y ) ∈ A 2) unitaire et complète.
a) Si x ∈ A et si x < 1, si 1 désigne l’élément unité de A, montrer que 1 − x est
inversible dans A.
b) Montrer que l’ensemble des inversibles de A est un ouvert de A.
c) Soit ϕ : A → R un morphisme d’algèbre. Montrer que ϕ est continue.
5. ESPACES VECTORIELS NORMÉS (E.V.N) 53

2/ Soit E un R-espace de Banach et u ∈ L c(E). On appelle spectre de u l’ensemble des


réels λ tels que u − λ Id ∈ G c(E) , où Gc(E) désigne l’ensemble des v ∈ Lc (E) tels que v
est inversible et v −1 est continu. Montrer que le spectre de u est compact.

Solution. 1/ a) On ne fait que réécrire la ndémonstration de la proposition 2 dans le cas plus
général d’une algèbre normée. La série x converge absolument car x  ≤ xn et x < 1,
n

et comme A est complet, elle converge. Notons y sa somme. Alors

+∞
 +∞

n
(1 − x)y = y − xy = x − xn = 1,
n=0 n=1

+∞ n
de même y(1 − x) = 1. Donc (1 − x) est inversible, son inverse est y = n=0 x .
b) Soit x 0 ∈ A un élément inversible. Si h ∈ A, h < x −1 −1 −1 −1
0  , on a x0 h ≤ x 0  · h < 1.
−1 −1
D’après la question précédente, 1 + x 0 h = 1 − (−x0 h) est inversible. On en déduit que
x0 + h = x 0 (1 + x−1 −1 −1 −1
0 h) est inversible (son inverse est (1 + x0 h) x 0 ). La boule de centre x0
−1
de rayon 1/x0  est donc incluse dans l’ensemble des inversibles, d’où le résultat.
c) Si ϕ est nulle, c’est terminé. Sinon, il existe x ∈ A tel que ϕ(x) = 0. On a alors ϕ(x) =
ϕ(1 · x) = ϕ(1)ϕ(x), donc ϕ(1) = 1. Montrons maintenant que si x = 1, alors |ϕ(x)| ≤ 1, ce
qui montrera la continuité de ϕ (un morphisme d’algèbre est linéaire). Raisonnons par l’absurde
et supposons l’existence de x ∈ A, x = 1, tel que λ = ϕ(x) vérifie |λ| > 1. Alors  1λ x < 1,
donc 1 − λ1 x est inversible, donc λ · 1 − x est inversible. Désignons par y son inverse. On a
 
ϕ (λ · 1 − x) y = ϕ(1) = 1 = ϕ(λ · 1 − x) ϕ(y ),

ce qui est absurde car ϕ(λ · 1 − x) = λϕ(1) − ϕ(x) = 0.


2/ Si A = Lc (E), A est une algèbre normée complète. D’après 1/b), l’ensemble Gc (E ) des
inversibles de A est un ouvert, et l’application ϕ : λ → u − λ Id étant continue, on en déduit
que le spectre S de u est fermé car S = ϕ −1 (AG c (E)).
Le spectre S est également borné. En effet, S ⊂ [−|||u|||, |||u|||] car si λ > |||u|||, on a ||| 1λu||| < 1
donc Id − 1λu est dans G c (E), donc u − λ Id est dans G c(E).
Finalement, S , fermé borné de R, est compact.
Remarque. - Si E est un espace de Banach et si u ∈ Lc (E) est inversible, alors u−1
est forcément continu. Ce résultat est appelé théorème de Banach, il est démontré à
l’annexe A, exercice 6 page 423.
– La notion de spectre de u ∈ Lc (E) généralise celle des valeurs propres d’un endomor-
phisme en dimension finie.


Exercice 5. On note 1le R-e.v des suites réelles (un ) telles que |un | converge, muni
∞ ∞
de la norme (un )1 = n=0 |un |. On note  le R-e.v des suites réelles (u n) bornées,
muni de la norme (un )∞ = sup n |u n|.
Montrer que le dual topologique (1 ) de 1 (i. e. l’e.v des formes linéaires continues
sur 1 ) s’identifie à  ∞ à une isométrie bijective près.

Solution. Pour toute suite k = (k n ) de ∞ , on définit l’application



1
Φ k :  → R (un ) → k n un
n=0
54 1. TOPOLOGIE SUR LES ESPACES MÉTRIQUES ET LES ESPACES VECTORIELS NORM ÉS

 
(la série kn un converge absolument car (kn ) est bornée et un converge absolument). Il est
clair que Φk est une forme linéaire de 1 . Elle est même continue car


|Φk [(un )]| ≤ |k n| |un | ≤ (k n )∞ · (u n )1 ,
n=0

et de plus cette inégalité montre que |||Φk ||| ≤ k ∞.


On a même |||Φk ||| = k  ∞. En effet, pour n ∈ N, désignons par e n la suite dont tous les
éléments sont nuls sauf le n-ième qui vaut 1. On a |Φk (en )| = |k n| et en 1 = 1, donc |||Φ k||| ≥ |k n |
pour tout entier naturel n, d’où on tire |||Φ k||| ≥ k∞ , et donc |||Φ k ||| = k  ∞ .
L’application linéaire Φ : ∞ → (1 ) k = (kn ) → Φk est donc une isométrie, en particulier
injective.
Il nous reste à montrer que Φ est surjective. Soit ϕ ∈ (1)  . Comme ϕ est continue,
∃M > 0, ∀(u n) ∈  1, |ϕ[(un )]| ≤ M · (un )1 .
La suite k = (k n) définie par k n = ϕ(e n ) est donc une suite de  ∞ et
N  N N
N 
   
∀N ∈ N, ϕ un e n = un ϕ(e n) = kn un = Φk un en .
n=0 n=0 n=0 n=0

Or la suite ( N 1 1
n=0 un en )N∈N de  converge vers (u n) dans  , et ϕ et Φ k étant continues, on en
1
déduit ϕ[(un )] = Φ k [(un )]. Ceci est vrai pour tout (un ) ∈  , donc ϕ = Φ k , d’où le résultat.

Exercice 6 (Convexes, théor ème de Carathéodory et application). 1/ Soit



E un e.v.n et C un convexe de E . Montrer que C et C sont convexes.
2/a) (Théorème de Carathéodory.) Soit E un R-e.v de dimension n ∈ N∗ , et soit x ∈ E le
barycentre
+
 1, . . . , x p ∈ E affectés de coefficients positifs (i. e. ∃λ 1 , . . . , λp ∈
de p vecteurs x
R , i λi = 1 avec x = i λ i xi ). Montrer qu’il existe I ⊂ {1, . . . , p} tel que Card I ≤
n + 1 et tel que x soit barycentre des (x i)i∈I affectés de coefficients positifs.
b) (Application.) Soit E un R-e.v.n de dimension finie et A une partie compacte de E .
Montrer que Conv(A), l’enveloppe convexe de A, est compacte.

Solution. 1/ L’adhérence C de C est convexe. En effet. Donnons nous deux points x et y de C


et un réel λ ∈ [0, 1]. Il existe deux suites (x n) et (yn) de C qui convergent respectivement vers x
et y. Pour tout n ∈ N, on a λ xn + (1 − λ)y n ∈ C , et en passant à la limite lorsque n tend vers
l’infini, on en déduit que λ x + (1 − λ)y , limite de points de C , appartient à C.
◦ ◦
Montrons que l’intérieur C de C est convexe. Soient x et y ∈ C. Il existe ρ > 0 tel que
B(x, ρ) ⊂ C et B(y, ρ ) ⊂ C. Soit λ ∈ [0, 1]. Pour tout h ∈ E tel que h < ρ, on a [λ x + (1 −
λ)y ] + h = λ (x + h) + (1 − λ)(y + h) ∈ C , donc B(λ x + (1 − λ)y, ρ) ⊂ C. Ceci prouve que

λ x + (1 − λ)y ∈ C, d’où le résultat.
2/ a) Soit Γ l’ensemble des parties J de {1, . . . , p} telles que x est barycentre des (xi) i∈J affectés
de coefficients positifs. Notons q = inf{Card J, J ∈ Γ}. Il s’agit de montrer q ≤ n + 1.
Supposons q ≥ n + 2. Par construction de q, il existe une partie J de {1, . . . , p} de cardinal
q telle que x soit barycentre des (xi )i∈J affectés de coefficients positifs :
 
x= λ i xi avec λi = 1 et ∀i ∈ J, λi ≥ 0.
i∈J i∈J

Comme E est de dimension n, les q ≥ n + 2 éléments (xi )i∈J sont affinements liés, c’est-à-dire
qu’il existe une famille (µi )i∈J d’éléments non tous nuls tels que
 
µi xi = 0 et µi = 0
i∈J i∈J
5. ESPACES VECTORIELS NORMÉS (E.V.N) 55

(si i0 ∈ J, écrire par exemple que les q − 1 ≥ n + 1 vecteurs (x i − xi 0 )i∈J {i 0} sont liés). Soit
α = inf µi >0 λi /µi , de sorte que λi − αµ i ≥ 0 pour tout i ∈ J (remarquons qu’il existe bien i ∈ J
tel que µi > 0 car les µi sont non tous nuls et leur somme est nulle). On a
  
x= (λi − αµ i)xi , avec ∀i ∈ J, λ i − αµ i ≥ 0 et (λi − αµi ) = λi = 1,
i∈J i∈J i∈J

et par définition de α , il existe i 0 ∈ J tel que λ i 0 − αµ i 0 = 0. Ainsi, x est barycentre des points
(xi )i∈J {i0} affectés de coefficients positifs. Ceci est absurde par définition de q, d’où le résultat.
n+1
b) Notons ∆ = {(λ 1, . . . , λ n+1 ) ∈ Rn+1 | i=1 λ i = 1 et ∀i, λ i ≥ 0}. Cet ensemble, fermé du
n+1
compact [0, 1] est compact. On considère l’application
n+1

n+1
ϕ : ∆×A →E ((λ1 , . . . , λ n+1), x 1, . . . , x n+1) → λ i x i.
i=1

Comme Conv(A) est l’ensemble des barycentres des points de A affectés de coefficients positifs,
on a d’après le théorème de Carathéodory Conv(A) = ϕ(∆ × An+1 ). Or ∆ × An+1 est compact
(produit de compacts) et ϕ est continue, donc Conv(A) est compact.

Exercice 7. Soit E un R-e.v.n et ϕ : E → R une forme linéaire.


1/ Montrer que ϕ est continue si et seulement si Ker ϕ est un fermé de E .
2/ a) Soit F un s.e.v de E . Montrer que l’application
N : E/F → R ẋ → inf y 
y∈ẋ

est une semi-norme sur l’espace quotient E/F . Que dire si F est fermé ?
b) En utilisant la question précédente, retrouver le résultat de la question 1/.

Solution. 1/ Si ϕ est continue, Ker ϕ est l’image réciproque par ϕ du fermé {0} donc fermé.
Réciproquement, supposons Ker ϕ fermé. Si ϕ n’est pas continue, ϕ n’est pas bornée sur la
sphère unité. Il existe donc une suite (x n) de E telle que
(i) ∀n ∈ N, xn  = 1 (ii) lim |ϕ(x n )| = +∞.
n→∞
xn
Fixons u ∈ E tel que ϕ(u) = 1. Pour tout n, posons un = u − . On a ϕ(u n ) = ϕ(u) −
ϕ(xn )
1
ϕ(xn ) = 0, donc un ∈ Ker ϕ. D’après (i) et (ii), (un ) converge vers u, et Ker ϕ étant fermé,
ϕ(xn )
u ∈ Ker ϕ. Ceci est absurde car ϕ(u) = 1. Ainsi, ϕ est continue.
·

2/ a) Soit x ∈ E et λ ∈ R. Rappelons que la classe ẋ de x est ẋ = x + F . De l’égalité λx = λ ẋ,
on tire N (λ ẋ) = |λ|N (ẋ).
Il nous reste à montrer que N vérifie l’inégalité triangulaire. Soient x, y ∈ E . On a
·
  
∀u, v ∈ F, N( x + y ) ≤ (x + y) + (u + v ) ≤ x + u + y + v,
·
  
ce qui en passant aux inf à droite donne N (x + y ) ≤ N (ẋ) + N (ẏ).
Ainsi, N est une semi-norme. Comme ẋ = x + F pour tout x, N (ẋ) = inf y∈F x − y  est
la distance de x à F . Si F est fermé, on a donc N (ẋ) = 0 si et seulement si x ∈ F , c’est-à-dire
ẋ = 0. Finalement, si F est fermé, N est une norme sur E F .
b) Soit ϕ : E → R une forme linéaire telle que Ker ϕ est fermé. Si ϕ est nulle, il est clair que ϕ
est continue. Sinon ϕ(E) = R. Considérons la factorisation canonique de ϕ suivante
56 1. TOPOLOGIE SUR LES ESPACES MÉTRIQUES ET LES ESPACES VECTORIELS NORM ÉS

ϕ
E −→ R

s ψ
E/ Ker ϕ
On a ϕ = ψ ◦ s où s est la surjection canonique de E sur E/ Ker ϕ et ψ : E/ Ker ϕ → R est
linéaire.
— L’application ψ est continue (c’est une application linéaire sur des espaces de dimension
finie).
— La surjection s de (E,  . ) dans (E/ Ker ϕ, N ) est continue.
(où N est la norme définie
  
plus haut avec F = Ker ϕ, fermé), car N (ẋ − ẏ ) = N (x − y ) ≤ x − y .
On en déduit que ϕ = ψ ◦ s est continue.
Remarque. La méthode utilisée au 2/ b) se généralise aisément pour montrer qu’une
application linéaire de rang fini est continue si et seulement si son noyau est fermé.

Exercice 8. Démontrer qu’un espace vectoriel normé E qui admet une base dénombrable
n’est jamais complet.
Solution. Soit (en )n∈N ∗ une base de E . Quitte à normaliser
 les en , on peut supposer en = 1

pour tout n ∈ N . Nous allons construire une série  λn en absolument convergente à partir
d’une suite (λn ) particulière, et nous allons prouver que λn en ne converge pas (intuitivement,
si une telle série convergeait, sa somme serait combinaison linéaire infinie de (en ), ce qui est
impossible dans un espace vectoriel par définition d’une base).
Notons F0 = {0} et F n = Vect(e 1 , . . . , en ) pour tout n ∈ N∗ . A partir de λ1 = 1/3, on définit
(λn ) par
1 1
λ n+1 = d(λn e n, F n−1) = inf λn e n − x.
3 3 x∈F n−1
Comme Fn est fermé (s.e.v de dimension finie), on sait que d(x, Fn ) = 0 si et seulement si
x ∈ F n. Par récurrence, on en déduit λn > 0 pour tout n ∈ N∗ . Enfin, l’inégalité
1 1 λn
λn+1 = d(λ n en , Fn−1 ) ≤ λn en = ,
3 3 3
assure la majoration
 λn ≤ 1/3 n pour tout n.
La série λn en converge donc absolument. Si E est supposé complet, elle converge donc.

Notons
 ∞x sa limite. Comme n(en) n∈N∗ est une base de E, il existe n ∈ N tel que x ∈ F n. Ainsi,
y = k=n+1 λk ek = x − k=1 λ k ek ∈ Fn, donc

 ∞
 1 3
3λn+2 = d(λ n+1 en+1 , F n ) ≤ λn+1 e n+1 − y ≤ λk ≤ λ n+2 = λn+2,
3k 2
k=n+2 k=0

ce qui est absurde car λ n+2 = 0. L’espace métrique E n’est donc pas complet.
Remarque. Ce résultat est aussi une conséquence immédiate du théorème de Baire (voir
l’annexe A, exercice 1).

Exercice 9 (Théorème de Riesz). Soit E un R-e.v.n de dimension infinie. Montrer


que la boule unité fermée de E ne peut pas être incluse dans une réunion finie de boules
ouvertes de rayon 1. Qu’en conclure ?
Solution. Raisonnons par l’absurde en supposant l’existence de n ∈ N∗ et de x1, . . . , xn ∈ E tels
que Bf (0, 1) ⊂ ∪ 1≤i≤n B(xi, 1). Notons F = Vect(x 1 , . . . , xn). Comme E est de dimension infinie,
6. PROBLÈMES 57

il existe x ∈ E tel que x ∈ F . Comme F est un s.e.v de dimension finie, il existe y ∈ F tel que
x − y  = d(x, F ) (voir l’exercice 3 page 33). Soit x0 = xx−−yy . On a d(x0 , F ) ≤ x 0 = 1 et

1   d(x, F )
 
∀z ∈ F, x0 − z  = x − (y + x − y  z )  ≥ = 1,
x − y  x − y 
donc d(x0, F ) = 1.
Or x0 ∈ B f (0, 1), donc il existe i tel que x0 ∈ B(xi , 1), de sorte que d(x0 , xi ) < 1, ce qui est
absurde car 1 = d(x 0, F ) ≤ d(x 0, xi ).
Finalement, nous avons démontré que Bf (0, 1) n’est pas précompact, en particulier non
compact.

6. Problèmes
Problème 1. Soient E et F deux espaces métriques, K un espace métrique compact.
Soit f : E × K → F (λ, x) → f (λ, x) une application continue. Pour tout y ∈ F , on
note Ey = {λ ∈ E | ∃x ∈ K, f (λ, x) = y }.
a) Montrer que Ey est un fermé de E .
b) Fixons y ∈ E . On suppose que
∀λ ∈ Ey, ∃!x ∈ K, f(λ, x) = y,
et on note x = ϕ(λ). Montrer que l’application ϕ : E y → K ainsi définie est continue.

Solution. a) L’ensemble Fy = f −1({y }) est fermé par continuité de f , et on remarque que


Ey = {λ ∈ E | ∃x ∈ K, (λ, x) ∈ F y }. Considérons une suite (λn ) de Ey qui converge vers λ ∈ E .
Il s’agit de montrer que λ ∈ E y . Pour tout n, il existe x n ∈ K tel que (λ n, xn ) ∈ F y. Comme K
est compact, on peut extraire de (xn ) une sous-suite (xϕ(n) ) qui converge vers un point x ∈ K .
La suite (λϕ(n), x ϕ(n) ) converge vers (λ, x) et comme F y est fermé on en déduit (λ, x) ∈ Fy , donc
λ ∈ Ey .
b) Soit (λn ) une suite de Ey qui tend vers λ ∈ Ey . Il s’agit de montrer que la suite (ϕ(λn ))
converge vers ϕ(λ).
Notons x n = ϕ(λ n), et considérons une valeur d’adhérence a ∈ K de (x n), limite d’une sous-
suite (xψ(n) ) de (x n). Par continuité de f , f (λ, a) = lim n→∞ f (λψ(n) , xψ(n) ) = y donc a = ϕ(λ).
Ceci prouve que la seule valeur d’adhérence de (xn ) est ϕ(λ). Comme (xn ) prend ses valeurs
dans un compact, on en déduit que (xn) converge vers ϕ(λ) (voir la proposition 9 page 30),
c’est-à-dire limn→∞ ϕ(λn ) = ϕ(λ).

Problème 2. Soit (E, d) un espace métrique et f : E → R une application continue. On


suppose qu’il existe un compact K de E tel que la restriction de f à K , f|K , soit injective
et tel que
∀x ∈ K, ∃ε > 0, f|B(x,ε) est injective.
Montrer qu’il existe un ouvert Ω contenant K tel que f|Ω est injective.

Solution. Raisonnons par l’absurde en supposant qu’un tel ouvert n’existe pas. Pour tout n ∈ N∗ ,
    1
1
On = x ∈ E | d(x, K) < = B x,
n n
x∈K

est un ouvert contenant K, donc


∃(xn , yn ) ∈ On2, (x n = yn et f (xn ) = f (yn)).
58 1. TOPOLOGIE SUR LES ESPACES MÉTRIQUES ET LES ESPACES VECTORIELS NORM ÉS

Pour tout n, xn ∈ On donc il existe x n ∈ K tel que d(xn, x n ) < 1/n. De même, il existe pour
tout n un point yn de K tel que d(yn , yn ) < 1/n.
La suite [(xn , yn )] n∈N∗ prend ses valeurs dans le compact K2 . On peut donc en extraire une
sous-suite convergente [(xϕ(n), y ϕ(n))] dont nous noterons (x, y) ∈ K 2 la limite. Les inégalités
1 1
d(xn , x) ≤ d(xn , xn) + d(x n, x) < + d(xn , x) et d(yn , y) < + d(y n, y)
n n
montrent que les suites (xϕ(n) ) et (y ϕ(n) ) tendent respectivement vers x et y . Comme f est conti-
nue et que pour tout n, f (xϕ(n) ) = f (yϕ(n) ), on a f (x) = f (y) et comme f|K est injective, x = y .
De plus, il existe par hypothèse un ε > 0 tel que f |B(x,ε) soit injective. Ceci est contradictoire car
il existe N ∈ N∗ tel que x ϕ(N ) et yϕ(N ) appartiennent à B(x, ε), et de plus f (xϕ(N ) ) = f (yϕ(N ) )
avec xϕ(N ) = yϕ(n) par construction.
Il existe donc un ouvert Ω contenant K tel que f|Ω soit injective.

Problème 3. Soit (E, d) un espace métrique.


a) On suppose que toute application continue de E dans E admet un point fixe. Montrer
que E est connexe.
b) On suppose E connexe et compact. Peut-on affirmer que toute application continue
de E dans E admet un point fixe ?
Solution. a) Raisonnons par l’absurde en supposant E non connexe. On peut trouver deux
ouverts disjoints non vides A et B tels que E = A ∪ B . Fixons a ∈ A et b ∈ B . Définissons
f : E → E par
f (x) = b si x ∈ A, f (x) = a si x ∈ B.
L’application f est continue puisque l’image réciproque de tout ouvert est un ouvert (si X ⊂ E ,
f −1 (X ) est égal soit à ∅, E , A ou B ), sans point fixe par construction. Ceci est contraire aux
hypothèses, donc E est nécessairement connexe.
b) Non ! Par exemple, la partie du plan E = {z ∈ C | |z| = 1} est compacte, connexe (car
connexe par arcs), et l’application f : E → E x →
 −x est continue et sans point fixe.

Problème 4. Soit K un fermé de [0, 1]2. On suppose que


∀x ∈ [0, 1], K x = {y ∈ [0, 1] | (x, y ) ∈ K } est un intervalle non vide.
a) Montrer que K est connexe.
b) Montrer qu’il existe x ∈ [0, 1] tel que x ∈ K x.

Solution. a) En vertu de la proposition 3 de la page 39, il suffit de montrer que toute application
continue de K dans {0, 1} est constante. Soit f une telle application.
Pour tout x ∈ [0, 1], l’application f(x, ·) : Kx → {0, 1} y → f (x, y ) est continue, et K x
étant connexe (c’est un intervalle), on en déduit que f (x, ·) est constante. Notons g (x) cette
constante. Si on montre que l’application ainsi définie g : [0, 1] → {0, 1} est constante, on aura
prouvé que f est constante.
Comme [0, 1] est connexe, il suffit de prouver que g est localement constante pour montrer
qu’elle est constante. Supposons que ce ne soit pas le cas, de sorte qu’il existe x ∈ [0, 1] et
une suite (x n) de [0, 1] qui tend vers x telle que g (x n) = g (x) pour tout n ∈ N. Supposons
par exemple g(x) = 0 et g (x n ) = 1 pour tout n. Pour tout n, Kxn est non vide donc il existe
yn ∈ K xn . L’ensemble K , fermé du compact [0, 1]2 est compact. On peut donc extraire de la
suite [(xn , y n)]n∈N de K une sous-suite convergente [(xϕ(n), yϕ(n) )]. On a x = lim n→∞ xϕ(n), et
notons y la limite de (yϕ(n) ). Comme f est continue sur K ,
g (x) = f (x, y) = lim f (xϕ(n) , y ϕ(n) ) = lim g (x ϕ(n)).
n→∞ n→∞
6. PROBLÈMES 59

Ceci est impossible puisque g (x) = 0 et g (xϕ(n) ) = 1 pour tout n.


L’application est donc localement constante sur [0, 1], donc continue sur [0, 1]. Comme [0, 1]
est connexe, g est constante, donc f est constante et le résultat est prouvé.
b) Soit ϕ : K → R (x, y) → x − y . Comme K est connexe et que ϕ est continue, ϕ(K ) est
un connexe de R, donc un intervalle. L’ensemble K0 est non vide, il existe donc y ∈ [0, 1] tel
que (0, y) ∈ K, donc ϕ(0, y) = −y ≤ 0. De même, il existe y ∈ [0, 1] tel que (1, y) ∈ K, donc
ϕ(1, y) = y ≥ 0. Comme ϕ(K ) est un intervalle, on en déduit l’existence de (x, y) ∈ K tel que
ϕ(x, y) = 0, c’est-à-dire x = y , ou encore x ∈ Kx .

Problème 5. Soit f : Rn → R n une application continue. Un point x 0 de R n étant


donné, on définit la suite (xn) par x n+1 = f (xn ). On suppose que (xn ) admet une et une
seule valeur d’adhérence. Montrer que la suite (x n ) converge.

Solution. Il suffit de montrer que la suite (xn) est bornée, car elle prendra alors ses valeurs dans
un compact (les compacts de Rn sont les fermés bornés) et on conclura grâce à la proposition 9
de la page 30.
Raisonnons par l’absurde et supposons (xn) non bornée. Notons a l’unique valeur d’adhérence
de (xn ), et notons K le compact B f (a, 1). Soit N ∈ N. Comme a est valeur d’adhérence de (x n),
il existe n0 ≥ N tel que xn0 ∈ K . De plus, (xn ) n’est pas bornée donc il existe n > n0 tel que
xn ∈ K. Ceci prouve qu’il existe un entier m ≥ n 0 tel que xm ∈ K et x m+1 ∈ K. On a aussi
xm+1 = f (xm) ∈ f (K), donc xm+1 ∈ f (K )K . En résumé, nous venons de montrer
∀N ∈ N, ∃m ≥ N, xm ∈ f (K )K.
Ceci montre qu’il existe une sous-suite (xϕ(n) ) de (xn ) prenant ses valeurs dans f (K )K , en
particulier dans f (K ). L’ensemble f (K ) est compact (image d’un compact par une application
continue), on peut donc extraire de (xϕ(n) ) une sous-suite (xϕ◦ψ(n) ) qui converge. Notons b sa
limite. Pour tout n, xϕ(n) ∈ K donc d(xϕ(n) , a) ≥ 1, on a donc d(b, a) ≥ 1. Ceci est contraire
aux hypothèses car b est une valeur d’adhérence de (xn ) différente de a. La suite (x n) converge
donc vers a.

Problème 6. Soit G un groupe. On suppose que G est muni d’une distance, et que l’ap-
plication ϕ : G × G → G (x, y ) → xy −1 est continue (on parle de groupe topologique).
a) On désigne par e l’élément neutre de G et par C la composante connexe de G contenant
{e}. Montrer que C est un sous-suite
sous groupe de G.
b) On suppose G connexe. Soit H un sous-suite
sous groupe ouvert de G. Montrer que H = G.

Solution. a) En vertu d’un résultat classique d’algèbre, il suffit de montrer que ϕ(C × C )
est inclus dans C . Comme C est connexe, C × C est connexe. De plus, ϕ est continue, donc
ϕ(C × C ) est connexe. Or e ∈ ϕ(C × C ) (par exemple e = ϕ(e, e)). L’ensemble ϕ(C × C ) est
donc un connexe contenant e. Par définition de C , on a donc ϕ(C × C ) ⊂ C , d’où le résultat.
b) Considérons la relation d’équivalence x R y ⇐⇒ xy −1 ∈ H. Les classes d’équivalence sont
de la forme Hx avec x ∈ G. Pour y ∈ G, l’application ψ : G → G x → xy −1 étant continue,
Hy = ψ −1(H ) est ouvert. Ainsi, GH = ∪ y∈H Hy, réunion d’ouverts est ouvert, et donc H est
fermé. Ainsi, H est ouvert et fermé dans le connexe G, donc H = G.

Problème 7 (Points fixes de fonctions réelles à variable réelle).


a) Soit f : [0, 1] → [0, 1] une application croissante. Montrer que f a au moins un point
fixe.
60 1. TOPOLOGIE SUR LES ESPACES MÉTRIQUES ET LES ESPACES VECTORIELS NORM ÉS

b) Montrer qu’une application continue f : [0, 1] → [0, 1] a au moins un point fixe.


c) Soit f et g : [0, 1] → [0, 1] deux applications continues, telles que f ◦ g = g ◦ f . Montrer
qu’il existe a ∈ [0, 1] tel que f (a) = g (a).
d) Soit f : R → R une application continue telle que f ◦ f admette un point fixe. Montrer
que f admet un point fixe. Généraliser.

Solution. a) Soit A = {x ∈ [0, 1], x ≤ f (x)}. L’ensemble A est non vide car 0 ∈ A. Soit
a = sup A. Pour tout x ∈ A on a x ≤ f (x) ≤ f (a) car f est croissante, donc f (a) est un
majorant de A donc a ≤ f (a). Comme f est croissante, ceci entraı̂ne f (a) ≤ f (f (a)) donc
f (a) ∈ A par définition de A, donc f (a) ≤ a par définition de a. Donc f (a) = a.
b) Il suffit de considérer l’application g : x → f (x) − x. On a g (0) = f (0) ≥ 0 et g (1) =
f (1) − 1 ≤ 0, donc g change de signe, et comme g est continue elle s’annule au moins en un
point a. On a alors f (a) = a.
c) L’ensemble E des points fixes de f est non vide d’après la question précédente. Si x ∈ E
alors f (g (x)) = g (f (x)) = g (x) donc g (x) ∈ E. Autrement dit, g (E ) ⊂ E . Donc si m = inf E
et M = sup E , on a g (m) ≥ m = f (m) et g (M ) ≤ M = f (M ), donc la fonction continue g − f
change de signe sur [0, 1] donc elle s’annule en un moins un point a, qui vérifie f (a) = g (a).
d) Si f n’admet pas de point fixe, alors la fonction continue x → f (x) − x ne s’annule jamais,
elle garde donc un signe constant, par exemple f (x) − x > 0 sur R. Ainsi pour tout réel x on a
f (x) > x donc f (f (x)) > f (x) > x, ce qui n’est pas compatible avec l’hypothèse de point fixe
de f ◦ f . On généralise aisément : si la composée n fois de f avec elle même admet un point fixe,
alors f admet un point fixe.

Problème 8. Soit f : [0, 1] → R une application continue vérifiant f (0) = f (1).


a) Montrer qu’il existe x ∈ [0, 1/2] tel que f (x) = f (x + 1/2).
b) Pour tout n ∈ N, n ≥ 2, montrer qu’il existe x ∈ [0, 1 − 1/n] tel que f (x) = f (x + 1/n).
c) Montrer qu’il existe α > 0 tel que ∀β ∈ ]0, α], ∃x ∈ [0, 1 − α], f (x) = f (x + β).
d) Montrer qu’il existe une fonction continue f : [0, 1] → R telle que f (0) = f (1) et
vérifiant f (x) =
 f (x + 2/5) pour tout x ∈ [0, 3/5].

Solution. a) Il suffit de considérer l’application g : [0, 1/2] → R x → f (x) − f (x + 1/2).


On a g (0) = −g (1/2) donc g change de signe, et comme elle est continue, le théorème des
valeurs intermédiaires nous assure de l’existence de x ∈ [0, 1/2] tel que g (x) = 0, ce qui entraı̂ne
f (x) = f (x + 1/2).
b) Raisonnons par l’absurde et supposons f (x) = f (x + 1/n) pour tout x ∈ [0, 1 − 1/n]. Ceci
signifie que l’application continue g (x) = f (x) − f (x + 1/n) ne s’annule jamais, donc g garde
un signe constant non nul, par exemple g > 0. On en déduit f (x) > f(x + 1/n) pour tout
x ∈ [0, 1 − 1/n] donc f (0) > f (1/n) > f (2/n) > · · · > f (n/n) = f (1), ce qui est absurde.
c) Si f est constante, le résultat est évident, sinon l’intuition nous suggère de nous placer
autour d’un extremum de f . Comme f n’est pas constante, il existe x tel que f (x) =  f (0), par
exemple f (x) > f (0). Comme f est continue sur le compact [0, 1] elle atteint son maximum
donc il existe c ∈ ]0, 1[ tel que f (c) = max t∈[0,1] f (t). Choisissons α = min(c, 1 − c). Considérons
β ∈ ]0, α] et construisons la fonction g (x) = f (x) − f (x + β ), définie sur [0, 1 − β ]. Si α = c, on
a g(α − β ) = f (c − β ) − f (c) ≤ 0 et g(α) = f (c) − f (c + β ) ≥ 0, donc g change de signe et
comme elle est continue, elle s’annule au moins en un point, ce qui entraı̂ne bien l’existence de
x ∈ [0, 1 − β ] tel que f (x) = f (x + β ). Le raisonnement est similaire si α = 1 − c à partir des
inégalités g(1 − α − β) = f (c − β ) − f (c) ≤ 0 et g(1 − α) = f (c) − f (c + β ) ≥ 0.
d). Soit f une telle fonction. La fonction continue f (x) − f (x + 2/5) ne s’annule pas donc garde
un signe constant non nul, par exemple positif. Donc on a toujours f (x) > f (x + 2/5). La
fonction f doit donc nécessairement vérifier f (0) > f (2/5) > f (4/5) et f (1/5) > f (3/5) > f (1).
Ceci suggère de construire f telle que f (0) = 0, f (2/5) = −1, f(4/5) = −2 et f (1/5) = 2,
6. PROBLÈMES 61

0 0,2 0,4 0,6 0,8 1

–1

–2

Figure 3. Le graphe de l’application f telle que f (x + 2/5) = f (x), et en


pointillé, le graphe de l’application x → f (x + 2/5)

f (3/5) = 1 et f (1) = 0, puis on interpole linéairement f sur chaque intervalle [i/5, (i + 1)/5].
Ainsi construite, on vérifie facilement que f (x) > f (x + 2/5) pour tout x ∈ [0, 3/5] (voir la figure
ci-contre).

Problème 9 (Distance de Hausdorff). Soit (E, d) un espace métrique. On note F


l’ensemble des parties fermées bornées non vides de E . Pour A, B ∈ F, on pose
λ(A, B ) = sup d(x, B) et ∆(A, B) = sup{λ(A, B ), λ(B, A)}.
x∈A

a) Montrer que ∆ est une distance sur F .


b) Pour tout n ∈ N∗ , on note Fn l’ensemble des éléments de F qui contiennent au moins
n éléments de E. Pour tout n ∈ N ∗, montrer que F n est un ouvert de F .
c) On suppose E compact. Montrer qu’une suite de Cauchy (Yn)n∈N de (F , ∆) telle que
Yn+1 ⊂ Yn pour tout n converge vers Y = ∩n∈N Yn . En déduire que (F , ∆) est complet.
d) Si E est compact, montrer que F est compact (on pourra utiliser le résultat de l’exer-
cice 2 de la page 32).

Solution. a) Il est clair que ∆ est symétrique.


Si λ(A, B ) = 0 avec A, B ∈ F , alors pour tout x ∈ A, d(x, B) = 0 et B étant fermé, x ∈ B .
On en déduit A ⊂ B . Maintenant, si ∆(A, B ) = 0, alors λ(A, B) = λ(B, A) = 0 donc A ⊂ B et
B ⊂ A donc A = B . Réciproquement si A = B on a bien sûr ∆(A, B) = 0.
Il nous reste à montrer que ∆ vérifie l’inégalité triangulaire. Donnons nous A, B et C ∈ F .
— On a ∀(x, y, z) ∈ A × B × C, d(x, C) ≤ d(x, z) ≤ d(x, y) + d(y, z).
— En ne retenant que le premier et le dernier terme dans les inégalités précédentes, on
obtient, en passant à la borne inférieure sur les z ∈ C d(x, C) ≤ d(x, y) + d(y, C).
— Comme d(y, C ) ≤ λ(B, C), on a d(x, C ) ≤ d(x, y) + λ(B, C).
— En prenant la borne inférieure sur les y ∈ B , on obtient d(x, C) ≤ d(x, B) + λ(B, C ).
— Comme d(x, B ) ≤ λ(A, B), d(x, C ) ≤ λ(A, B) + λ(B, C).
— Il ne reste plus qu’à prendre la borne supérieure sur les x ∈ A, ce qui donne λ(A, C ) ≤
λ(A, B ) + λ(B, C).
— On a de même λ(C, A) ≤ λ(C, B) + λ(B, A).
— Ceci suffit pour conclure ∆(A, C ) ≤ ∆(A, B) + ∆(B, C).
b) Soit F ∈ Fn . Il existe n éléments distincts x1 , . . . , xn dans F . Soit ε = inf i=j d(xi , xj ) > 0. Soit
G ∈ F tel que ∆(F, G) < ε/2. Alors λ(F, G) < ε/2, donc pour tout i, 1 ≤ i ≤ n, d(x i, G) < ε/2,
de sorte qu’il existe yi ∈ G tel que d(xi, y i) < ε/2. Les points y 1 , . . . , y n sont distincts (si y i = y j ,
alors d(xi, xj ) ≤ d(xi , y i) + d(yj , xj ) < ε, donc i = j par définition de ε), donc G ∈ Fn . La boule
ouverte de centre F de rayon ε/2 est donc incluse dans Fn , donc F n est ouvert.
62 1. TOPOLOGIE SUR LES ESPACES MÉTRIQUES ET LES ESPACES VECTORIELS NORM ÉS

c) Les Yn sont fermés dans le compact E donc compact, et l’intersection d’une suite décroissante
de compacts non vides étant non vide, on en déduit Y = ∩n Yn ∈ F .
Donnons nous maintenant ε > 0. Par hypothèse, la suite (Yn ) est de Cauchy dans (F , ∆)
donc il existe N ∈ N tel que pour tout p, q ≥ N , ∆(Y p, Yq ) < ε. Nous allons montrer que
∆(Y, Yn ) ≤ ε pour tout n ≥ N .
Fixons n ≥ N . Pour tout p ≥ n, λ(Y n, Yp ) ≤ ∆(Yn , Yp ) < ε. Donnons nous x ∈ Y n. On a
d(x, Yp ) < ε, donc
∀p ≥ n, ∃x p ∈ Yp , d(x, x p ) < ε.
La suite (xp ) prend ses valeurs dans le compact E, on peut donc en extraire une sous-suite
convergente (x ϕ(p) ), dont nous noterons  la limite. Pour tout p ≥ n,  ∈ Yp car pour tout
m ≥ p, xm ∈ Yp et Yp est fermé. Ainsi,  ∈ ∩p≥n Y p = Y . Par ailleurs, pour tout p, d(x, xϕ(p) ) < ε
donc d(x, ) ≤ ε. Comme  ∈ Y , on en déduit d(x, Y ) ≤ ε. Ceci étant vrai pour tout x ∈ Yn , on
en déduit λ(Yn , Y ) ≤ ε. Or Y ⊂ Y n , donc λ(Y, Yn) = 0, donc ∆(Y n, Y ) ≤ ε. Ceci étant vrai pour
tout n ≥ N , on en déduit que (Yn ) converge vers Y .
Montrons maintenant que l’espace métrique (F , ∆) est complet. Soit (Y n ) une suite de Cau-
chy de cet espace. Pour tout n ∈ N, on pose Xn = ∪ p≥n Yp. La suite (X n) ainsi définie est une
suite décroissante de (F, ∆).
Soit ε > 0 et N ∈ N tel que pour tous p, q ≥ N , ∆(Yp , Yq ) < ε. Soit n ≥ N . Si x ∈ ∪ p≥nYp ,
il existe p ≥ n tel que x ∈ Y p , donc d(x, Yn ) ≤ λ(Yp , Yn ) < ε. On en déduit que pour tout x ∈
∪p≥n Yp , d(x, Yn ) ≤ ε, donc λ(X n, Yn ) ≤ ε. Comme X n ⊂ Yn, on a λ(Y n , Xn ) = 0. Finalement,
nous venons de montrer que
∀ε > 0, ∃N ∈ N, ∀n ≥ N, ∆(Xn , Yn ) < ε. (∗)
On en déduit maintenant aisément que la suite (X n) vérifie le critère de Cauchy. C’est de plus
une suite décroissante de fermés non vides, donc d’après ce que l’on a vu précédemment, (Xn )
converge vers X = ∩ n∈NXn . La relation (*) montre alors que (Yn) converge vers X .
d) Nous venons de voir que (F , ∆) est complet. En vertu du résultat de l’exercice 2 de la page 32,
il suffit, pour prouver la compacité de cet espace métrique, de montrer qu’il est précompact (i. e.
pour tout ε > 0, il existe un nombre fini de boules de rayon ε qui recouvrent F ).
Soit ε > 0. L’ensemble E est compact donc précompact, donc il existe une famille finie de
boules (B(xi , ε)) 1≤i≤n qui recouvre F . Notons Γ l’ensemble des parties non vides de {x1 , . . . , xn }.
Nous allons montrer que F ⊂ ∪F ∈Γ B∆ (F, ε), où pour tout F ∈ Γ, B∆ (F, ε) désigne la boule
fermée de centre F de rayon ε dans (F , ∆).
Soit A ∈ F. Notons F = {xi | 1 ≤ i ≤ n, d(xi , A) < ε}. Comme la famille (B(xi , ε))1≤i≤n
recouvre E , F est non vide donc F ∈ Γ. Pour tout x ∈ F , d(x, A) < ε donc λ(F, A) < ε.
Soit x ∈ A. La famille (B(xi , ε)) 1≤i≤n recouvrant E, il existe xi (1 ≤ i ≤ n) tel que d(x, xi ) <
ε. On a donc x i ∈ F et d(x, F ) ≤ d(x, xi) < ε, et ceci pour tout x ∈ A, donc λ(A, F ) ≤ ε. On
en déduit ∆(A, F ) ≤ ε, donc A ∈ B ∆ (F, ε).
Nous venons de montrer que F ⊂ ∪F ∈ΓB∆ (F, ε), et comme Γ est fini, on en déduit le résultat.

Problème 10 (Oscillation d’une fonction). Soient (E, d) et (F, δ ) deux espaces


métriques et une application f : E → F . Pour tout x 0 ∈ E, on note
 
ω(f, x0 ) = inf sup δ (f (x), f (y)) ∈ R + ∪ {+∞}
ρ>0 x,y∈B(x0 ,ρ)

(oscillation de f en x0), où B(x 0, ρ) désigne la boule ouverte dans (E, d) de centre x0 de
rayon ρ.
a) Montrer que f est continue en x0 ∈ E si et seulement si ω (f, x0 ) = 0.
b) Pour tout ε ≥ 0, montrer que l’ensemble
Aε = {x ∈ E | ω (f, x) ≥ ε}
6. PROBLÈMES 63

est un fermé de E. L’application x → ω (f, x) est elle continue ?


c) Montrer que l’ensemble des points où f est continue est le complémentaire de l’ensemble
∪n∈N∗ A1/n.
d) On suppose ici E compact et qu’il existe ε > 0 tel que ω (f, x) < ε pour tout x ∈ E .
Montrer
∃α > 0, ∀(x, y ) ∈ E 2 , d(x, y ) < α, δ(f (x), f (y)) < ε.

Solution. a) Supposons f continue en x0 ∈ E. Soit ε > 0. Alors


∃ρ > 0, ∀y ∈ B(x0 , ρ), δ(f (x0 ), f(y)) < ε,
donc
∀(x, y ) ∈ B(x0 , ρ) 2 , δ(f (x), f(y)) ≤ δ (f (x), f(x0)) + δ (f (x 0 ) + f (y )) < 2ε,
d’où
ω(f, x 0) ≤ sup δ (f (x), f(y)) ≤ 2ε.
x,y∈B(x 0,ρ)
Ceci étant vrai pour tout ε > 0, on en déduit ω (f, x0 ) = 0.
Réciproquement, si ω (f, x0) = 0, alors
∀ε > 0, ∃ρ > 0, sup δ (f (x), f(y)) < ε,
x,y∈B(x 0,ρ)

en particulier
∀x ∈ E, d(x 0 , x) < ρ, δ (f (x), f(x 0)) < ε,
ce qui montre la continuité de f en x0 .
b) Montrons que Bε = E A ε = {x ∈ E | ω (f, x) < ε} est ouvert. Soit x0 ∈ Bε . On
a ω (f, x0 ) < ε donc il existe ρ > 0 tel que sup x,y∈B(x0,ρ) δ (f (x), f(y)) < ε. Considérons
maintenant x1 ∈ B(x0 , ρ), et r = ρ − d(x0, x 1). On a B(x1 , r) ⊂ B(x0 , ρ) donc ω(f, x 1) ≤
sup x,y∈B(x1,r) δ (f (x), f(y)) < ε, donc x 1 ∈ Bε . Ainsi, Bε est ouvert.
L’application x0 → ω(f, x0 ) n’est pas forcément continue. Par exemple, la fonction f de R
dans R nulle partout sauf en 0 où elle vaut 1, vérifie ω(f, x) = f (x).
c) C’est immédiat car
1
(f est continue en x 0) ⇐⇒ (ω (f, x0) = 0) ⇐⇒ (∀n ∈ N ∗ , ω(f, x 0) < )
n
⇐⇒ (∀n ∈ N∗ , x0 ∈ A 1/n ) ⇐⇒ (x0 ∈ ∪n∈N ∗ A1/n ).

d) Supposons le contraire. Alors


1
∀n ∈ N ∗, ∃(x n , y n) ∈ E 2, d(x n, yn ) <
et δ (f (xn ), f(y n )) ≥ ε.
n
La suite (xn , y n) prend ses valeurs dans le compact E 2, on peut donc en extraire une sous-suite
convergente (xϕ(n), y ϕ(n)) dont nous noterons (x, y ) la limite. On a x = y car pour tout n,
d(xϕ(n), y ϕ(n) ) < 1/ϕ(n). Or ω(f, x) < ε donc
∃ρ > 0, ∀(z, z ) ∈ B(x, ρ)2 , δ(f (z ), f(z  )) < ε. (∗)
Comme (xϕ(n) ) et (yϕ(n) ) convergent vers x, il existe n ∈ N∗ tel que (xϕ(n) , y ϕ(n) ) ∈ B(x, ρ)2 .
Ceci est absurde d’après (*) car par hypothèse, δ (f (xϕ(n) ), f(yϕ(n) )) ≥ ε.
Remarque. Le résultat de la partie d) généralise le théorème de Heine, et est de nature
proche du résultat de la question d) de l’exercice 7 page 36.

Problème 11 (Parties négligeables de R, ensemble triadique de Cantor).


1/ Pour tout intervalle de R ouvert borné I = ]a, b[, on note (I ) = b − a. Une partie
64 1. TOPOLOGIE SUR LES ESPACES MÉTRIQUES ET LES ESPACES VECTORIELS NORM ÉS

A de R est dite négligeable si pour tout ε > 0, il existe une famille au plus dénombrable
(In)n∈J d’intervalles ouverts bornés de R telle que
 
(i) A ⊂ ∪ n∈J In . (ii) (I n ) converge et (In ) ≤ ε.
n ∈J

a) Montrer qu’une réunion au plus dénombrable de parties négligeables est négligeable.


b) Montrer que Q est une partie négligeable de R.
c) Si A est une partie négligeable de R, montrer que A est d’intérieur vide.
d) L’ensemble RQ est-il négligeable ?
2/ (Ensemble triadique de Cantor.) Soit I = [0, 1]. On construit une suite de parties
(Kn )n∈N de I par récurrence, comme suit.
— On prend K 0 = I .
— Kn étant une réunion finie de segments disjoints : Kn = ∪k [ak , bk ], on construit
  b k − ak
 
bk − ak

Kn+1 = ak, ak + ∪ ak + 2 , bk .
k
3 3

On pose alors K = ∩n∈NKn (ensemble triadique de Cantor).


a) Montrer que K est négligeable.
b) Soit x ∈ [0, 1[. Donner une condition nécessaire et suffisante sur le développement en
base 3 de x pour que x ∈ K .
c) Montrer que K est un fermé, d’intérieur vide, sans point isolé, et qu’il a la puissance
du continu.
Solution. 1/ Remarquons tout d’abord que la convergence et la somme d’une série à termes
positifsne dépend pas de l’ordre de sommation (voir le théorème 8 page 216), ce qui donne un
sens à n∈J (I n ).
a) Soit (Aj )j ∈J une famille au plus dénombrable de parties négligeables (avec J ⊂ N). Soit
ε > 0. Pour tout j ∈ J , on peut écrire A j ⊂ ∪n∈Kj I n,j où l es nI,j sont des intervalles de R
ouverts bornés, Kj est au plus dénombrable et où

(I n,j) < ε/2j+1 .
n∈Kj

Notons Γ = ∪j ∈J {j} × Kj . L’ensemble Γ est au


 plus dénombrable
 (réunion au plus dénombrable
d’ensembles au plus dénombrables), et on a j ∈J Aj ⊂ (j,n)∈Γ I n,j avec
 
    ε +∞
 ε
(In,j ) =  
(In,j ) ≤ ≤ = ε.
j ∈J n ∈K j ∈J
2j+1 j=0
2j+1
(j,n)∈Γ j

Ainsi, A = ∪ j ∈JAj est négligeable.


b) Un singleton {x0 } est négligeable car pour tout ε > 0, {x0} ⊂ ]x 0 − ε/2, x0 + ε/2[. Donc Q,
réunion dénombrable de singletons, est négligeable d’après a).
c) Soit A une partie négligeable de R. Raisonnons par l’absurde en supposant que A n’est pas
d’intérieur vide. Alors il existe un segment I = [a, b] inclus dans A tel que a < b. Comme A est
négligeable, il existe
une famille au plus dénombrable (In)n∈J d’intervalles ouverts bornés qui
recouvre A, tel que n∈J (In ) < b − a. Cette famille recouvre aussi le compact [a, b], dont on
peut extraire un sous-recouvrement fini (In i) 1≤i≤p qui vérifie
p
 
(I ni ) ≤ (In) < b − a.
i=1 n ∈J
6. PROBLÈMES 65

p
Ceci est impossible, car comme [a, b] ⊂ ∪ 1≤i≤nI ni , on a χ [a,b] ≤ i=1 χ In i ( où χP désigne la
fonction caractéristique de P ), donc
  
p p

χ[a,b] = b − a ≤ χ In i = (Ini ).
R R i=1 i=1

d) Non ! S’il était négligeable, alors R = (RQ) ∪ Q serait négligeable (d’après a) et b)), ce qui
est absurde d’après c) car l’intérieur de R est non vide.
2/ a) On voit facilement (par récurrence sur n) que Kn est la réunion de 2 n intervalles fermés
de longueur 3−n .
Soit ε > 0 et soit N ∈ N∗ tel que (2/3)N < ε/2. Comme on l’a vu, on peut écrire KN =
ε ε
∪1≤n≤2N [a n , bn ] avec bn − an = 3−N pour tout n. En posant In = an − N
, bn + ,
4·2 4 · 2N
on a K ⊂ KN ⊂ ∪ 1≤n≤2N In, et
N N
2   N
ε 
2
  2 ε
(In ) = (bn − an ) + = + < ε,
n=1 n=1
2 · 2N 3 2

d’où le résultat.
b) Une récurrence facile donne
 n n

  εk 1  εk
∀n ∈ N, K n = , + . (∗)
3 k 3n 3k
ε1 ∈{0,2},··· ,ε n∈{0,2} k=1 k=1

Ceci étant, soit x ∈ [0, 1[. Considérons son développement en base 3 : x = ∞ k
k=1 αk /3 , avec
αk ∈ {0, 1, 2}, la suite (αk ) n’étant pas stationnaire à 2 à partir d’un certain rang (propriété des
développements tri-adiques). Si les εk sont dans {0, 2}, on a l’équivalence
n
 n
εk 1  εk
≤x< n + ⇐⇒ ∀k ∈ {1, . . . , n}, αk = εk ,
3k 3 3k
k=1 k=1
n
et compte tenu du fait que les réels de [0, 1[ de la forme 1/3n + k=1 εk /3
k
(avec εk ∈ {0, 2})
sont aussi ceux de la forme
p
 βk
avec p ≤ n, βk ∈ {0, 2} pour 1 ≤ k ≤ p − 1, et βp = 1,
3k
k=1

on en déduit gr âce à (*) que


p
 εk 1
(x ∈ K ) ⇐⇒ (∀n, x ∈ K n) ⇐⇒ (∀k, α k ∈ {0, 2} ou ∃p ∈ N, x = + , ε k ∈ {0, 2}).
3k 3p+1
k=1

En d’autres termes, x ∈ K si et seulement si tous les termes (αk ) du développement de x en


base 3 sont dans {0, 2} ou s’il existe un entier naturel p tel que αk ∈ {0, 2} pour k ≤ p, α p+1 = 1
et αk = 0 pour k > p + 1.
c) L’ensemble K est une intersection de fermés, c’est donc un fermé.
Il est d’intérieur vide car négligeable. On pouvait aussi remarquer, grâce à la question
précédente, que pour tout x ∈ K , pour tout N ∈ N, il existe n ≥ N tel que x + 1/3n ∈ K.
Aucun point de K n’est isolé dans K . En effet, deux cas se présentent.

— Si le développement en base 3 de x ∈ K est de la forme x = ∞ k
k=1 αk /3 avec α k ∈ {0, 2}
pour tout k , alors la suite (αk ) n’étant jamais stationnaire à 2 à partir d’un certain rang,
on s’aperçoit que
2
∀N ∈ N, ∃n ≥ N, x + n ∈ K.
3
66 1. TOPOLOGIE SUR LES ESPACES MÉTRIQUES ET LES ESPACES VECTORIELS NORM ÉS

p
— Si x est de la forme k=1 ε k /3k + 1/3p+1 avec ε k ∈ {0, 2}, alors pour tout n ≥ p + 1,
p n−1
1  εk  2
x− n
= + ∈ K.
3 3k 3k
k=1 k=p+1

L’ensemble K a la puissance du continu. En effet, l’application ϕ de [0, 1[dans K qui à


∞ k ∞ k
tout x dont le développement di-adique est x = k=1 εk /2 associe ϕ(x) = 2 k=1 εk /3 est
injective.
Remarque. A la question 2/ b), on aurait pu montrer que x ∈ K si et seulement s’il existe
N∗ ∞
une suite (αk ) ∈ {0, 2} telle que x = k=1 αk/3 k . Mais les écritures de cette forme ne
sont pas forcément un développement en base 3, car dans un tel développement, la suite
des termes du développement ne peut pas être stationnaire à 2 à partir d’un certain rang
(cette contrainte permet d’avoir l’unicité de l’écriture triadique).
– L’ensemble de Cantor possède des propriétés intéressantes qui sont des contre-exemples
à beaucoup de fausses intuitions.
– Dans la théorie de la mesure de Lebesgue, les ensembles négligeables sont les ensembles
de mesure nulle.
– Dans la même veine, signalons l’escalier de Cantor-Lebesgue, introduite par Lebesgue,
qui est le graphe d’une fonction continue ψ : [0, 1] → [0, 1], vérifiant ψ (0) = 0 et ψ (1) = 1,
dérivable presque partout (c’est-à-dire sur le complémentaire d’une partie négligeable) et
de dérivée presque partout nulle. On l’appelle aussi parfois escalier du diable car il est
continu, a presque partout une tangente horizontale, et pourtant il monte. Cettefonction
peut être définie sur l’ensemble triadique de Cantor K par ψ( k 2βk /3k ) = k βk /2k
(avec βk ∈ {0, 1}), et elle est localement constante sur le complémentaire de K .

Problème 12 (Th éorème de prolongement de Tietze-Urysohn). Soient (E, d)


un espace métrique, A un fermé de E , et f : A → R une application continue et bornée.
Montrer l’existence d’une fonction g : E → R, continue, telle que g |A = f et
sup g (x) = sup f (y ) et inf g (x) = inf f (y ).
x∈E y∈A x∈E y∈A

(Indication. Se ramener au cas où inf y∈A f (y ) = 1, sup y∈A f (y ) = 2, et prendre g = f sur
1
A et g (x) = inf [f (y)d(x, y )] si x ∈ A.)
d(x, A) y∈A

Solution. Si f est constante, le résultat est évident, sinon en remplaçant f par α f + β avec
α = 0 et β des réels bien choisis, on peut supposer infy∈A f (y ) = 1 et supy∈A f (y ) = 2. Suivons
l’indication et construisons la fonction g : E → R définie par g (x) = f (x) si x ∈ A et
h(x)
∀x ∈ E A, g ( x) = , avec h(x) = inf (f (y)d(x, y )) ( ∗)
d(x, A) y∈A

(cette dernière expression est bien définie, car A étant fermé, on a d(x, A) = 0 si et seulement si
x ∈ A).
Les inégalités 1 ≤ f (x) ≤ 2 pour tout x ∈ A montrent que 1 ≤ g (x) ≤ 2 pour tout x ∈ E .
Comme g|A = f , on a même inf x∈E g (x) = 1 et supx∈E g (x) = 2.
Il nous reste à montrer la continuité de g en tout point x0 de E. Nous allons traiter les cas

x0 ∈ A, puis x 0 ∈ E A, puis x 0 ∈ Fr(A).

(i) Si x0 ∈ A, comme g |A = f et que f est continue sur A, g est continue en x0 .
6. PROBLÈMES 67

(ii) Supposons x0 ∈ E A. Sur l’ouvert E A, l’application g prend la forme (*), et comme
x → d(x, A) est continue (voir l’exercice 3, page 33), il nous suffit de montrer que h est continue
en x 0. Soit r > 0 tel que B(x 0, r) ⊂ E A. Pour tout x ∈ B(x0, r), pour tout y ∈ A,
h(x 0 ) ≤ f (y) d(x0, y) ≤ f (y)(d(x 0, x) + d(x, y)) ≤ 2d(x 0 , x) + f (y) d(x, y),
et ceci étant vrai pour tout y ∈ A, on en déduit, en ne considérant que les termes des extrémités
et en prenant la borne inférieure sur les y ∈ A, que h(x0 ) ≤ h(x) + 2d(x0 , x). De même,
h(x) ≤ h(x0 ) + 2d(x, x 0), donc finalement, |h(x 0) − h(x)| ≤ 2d(x 0 , x), ce qui prouve la continuité
de h en x0 ∈ E A.
(iii) Il reste le cas où x 0 ∈ Fr(A). Soit ε > 0. Comme f est continue en x 0 ∈ A,
∃r > 0, ∀y ∈ A ∩ B(x 0 , r), |f (y ) − f (x0)| ≤ ε.
Si x ∈ E A et d(x0 , x) ≤ r/4, on a, en notant C = A ∩ B(x0 , r) :
3r
∀y ∈ AC, d(x, y) ≥ d(x0 , y) − d(x0, x) ≥ ,
4
donc inf y∈AC f (y )d(x, y) ≥ 3r/4. D’autre part, f (x 0)d(x0 , x) ≤ 2d(x0 , x) ≤ r/2, donc
inf f (y )d(x, y ) = inf f (y )d(x, y),
y∈A y∈C

et comme f (x0 ) − ε ≤ f (y ) ≤ f (x0) + ε pour tout y ∈ C , on en déduit


(f (x0 ) − ε)d(x, A) ≤ inf (f (y)d(x, y )) ≤ (f (x0) + ε)d(x, A),
y∈A

donc g (x0) − ε ≤ g (x) ≤ g (x 0) + ε pour tout x ∈ E A tel que d(x0 , x) ≤ r/4. Ceci reste vrai
si x ∈ A et d(x, x0 ) ≤ r/4 car c’est vrai pour f. Ainsi, g est continue en x 0 ∈ Fr(A), d’où le
résultat.

Problème 13 (Continuité des racines de polyn ômes). On norme C[X ] en posant


a0 + a1 X + · · · + an X  = ni=0 |a i|.
n

1/ Soit P ∈ C[X ] un polynôme unitaire et λ ∈ C une racine de P . Montrer que |λ| ≤ P .


2/ Soit n ∈ N∗ et P = (X − λ1 ) · · · (X − λn ) ∈ C[X ]. Soit (Pm)m∈N une suite de polynômes
unitaires de degré n qui tend vers P .
a) Montrer que pour toute racine λ de P , il existe une suite de nombres complexes (α m)
telle que pour tout m, αm est racine de P m et telle que limm→∞ αm = λ.
b) Montrer que l’on peut écrire
∀m ∈ N, Pm = (X − λ1,m ) · · · (X − λn,m )
avec pour tout i ∈ {1, . . . , n}, lim m→∞ λ i,m = λi .

Solution. 1/ Écrivons P = a 0 + a1 X + · · · + an−1 X n−1 + X n, où n = deg(P ). On a P  =


|a0 | + · · · + |a n−1 | + 1 ≥ 1, donc si |λ| ≤ 1, c’est terminé. Sinon |λ| > 1, et l’égalité P (λ) = 0
entraı̂ne
a n−2 a1 a0
λ = −an−1 − − · · · − n−2 − n−1
λ λ λ
de sorte que |λ| ≤ |a n−1| + · · · + |a 1 | + |a0 | < P , d’où le résultat.
2/ a) Pour tout m ∈ N, notons (αi,m) 1≤i≤n les racines de Pm (sans tenir compte pour l’instant
de la numérotation). Il s’agit de montrer que min1≤i≤n |α i,m − λ| tend vers 0, c’est-à-dire
∀ε > 0, ∃N, ∀m ≥ N, ∃i ∈ {1, . . . , n}, |αi,m − λ| < ε.
Supposons le résultat faux, de sorte que
∃ε > 0, ∀N, ∃m ≥ N, ∀i ∈ {1, . . . , n}, |αi,m − λ| ≥ ε.
68 1. TOPOLOGIE SUR LES ESPACES MÉTRIQUES ET LES ESPACES VECTORIELS NORM ÉS

On peut alors extraire de la suite (αm) m∈N = (α 1,m, . . . , α n,m )m∈N une sous-suite (α ϕ(m)) telle
que
∀m ∈ N, ∀i, |αi,ϕ(m) − λ| ≥ ε. (∗∗)
Comme la suite (Pm) converge, elle est bornée. Notons M un majorant de (P m ). La suite
(α ϕ(m))m∈N est à valeur dans le compact K n , où K = {z ∈ C | |z | ≤ M } (d’après 1/). On peut
donc en extraire une sous-suite convergente (α ϕ◦ψ(m)) dont nous noterons α = (α1 , . . . , αn ) la
limite. Alors
 n  n
  
P = lim P ϕ◦ψ(m) = lim X − αi,ϕ◦ψ (m) = (X − αi ),
m→∞ m→∞
i=1 i=1
donc les αi sont les racines de P , donc il existe k tel que αk = λ. Or d’après (**), pour tout m,
|αk − α k,ϕ◦ψ (m) | ≥ ε. Ceci est contradictoire car (αk,ϕ◦ψ (m)) tend vers α k. D’où le résultat.
b) Nous allons procéder par récurrence sur n ∈ N∗ . Pour n = 1, c’est évident. Supposons le
résultat vrai au rang n − 1 et montrons le au rang n. Considérons la racine λn de P . D’après
la question précédente, on peut écrire P m = (X − αm )Qm pour tout m, où (αm ) converge vers
λn. Notons Q le polynôme de degré n − 1 tel que P = (X − λn )Q. En écrivant chacun des
polynômes Qm comme le quotient de la division euclidienne de Pm par (X − αm ), on s’aperçoit
que les coefficients de Qm s’expriment comme un polynôme en les coefficients de Pm et en α m.
Comme (αm ) tend vers λ et que (Pm ) tend vers P , on en déduit que (Qm) tend vers Q. D’après
l’hypothèse de récurrence, on peut donc écrire Q m = (X − λ 1,m) · · · (X − λ n−1,m ) pour tout m,
avec pour tout i limm→∞ λi,m = λi. Finalement, on a P m = (X − λ 1,m) · · · (X − λ n−1,m )(X − αm)
pour tout m et
∀i ∈ {1, . . . , n − 1}, lim λi,m = λ i et lim α m = λn ,
m→∞ m→∞
ce qui est précisément ce que l’on voulait montrer.
Remarque. On aurait pu répondre à la question 2/ a) sans utiliser la question 1/, en
utilisant le résultat suivant.
Lemme 1. Soit F ∈ C[X ] un polynôme unitaire de degré n et γ ∈ C.
Alors il existe une racine α de F telle que |α − γ| ≤ |F (γ )|1/n .
Le lemme se démontre en notant α 1, . . . , α n les racines de F et en lessupposant ordonnées
de sorte que |γ − α1 | ≤ |γ − αi| pour tout i. Alors on a |γ − α 1 |n ≤ ni=1 |γ − α i| = |F (γ )|
d’où le résultat.
Le fait que (Pm) tende vers P entraı̂ne que Pm(λ) tend vers P (λ) = 0. Le lemme nous
assure pour tout m l’existence d’une racine αm de P m telle que |λ − α m | ≤ |Pm(λ)|1/n .
Comme (Pm(λ)) tend vers 0, on en déduit que (α m ) tend vers λ, ce qui répond à la
question 2/ a).

Problème 14 (Théorème d’Ascoli). Soit (X, d) et (Y, δ) deux espaces métriques


compacts. On note C(X, Y ) l’ensemble des fonctions continues de X dans Y . Muni de la
distance de la convergence uniforme ∆(f, g) = supx∈X δ (f (x), g(x)), l’ensemble C(X, Y )
est un espace métrique.
Soit A une partie de C(X, Y ). On dit que A est équicontinue si
∀x ∈ X, ∀ε > 0, ∃η > 0; ∀f ∈ F, ∀y ∈ X, d(x, y ) < η ⇒ δ (f (x), f (y)) < ε.

1/ Montrer que si A est équicontinue, alors A est uniformément équicontinue, c’est-à-dire


∀ε > 0, ∃η > 0; ∀f ∈ A, ∀x, y ∈ X, d(x, y ) < η ⇒ δ (f (x), f (y)) < ε.

2/ (Théorème d’Ascoli) Démontrer que les deux propositions suivantes sont équivalentes :
(i) A est relativement compacte (i.e son adhérence est compacte) dans C(X, Y ).
6. PROBLÈMES 69

(ii) A est équicontinue.


(Pour montrer (ii) ⇒ (i), on pourra utiliser le résultat de l’exercice 2 page 32 qui dit que
tout précompact complet est compact).

Solution. 1/ Soit ε > 0. Comme A est équicontinue, pour tout x ∈ X il existe η x > 0 tel que
∀f ∈ F, ∀y ∈ X ; d(x, y) < 2η x ⇒ δ (f (x), f(y)) < ε.
On peut extraire du recouvrement d’ouverts ∪x∈X B(x, ηx ) du compact X, un recouvrement fini.
Autrement dit, il existe n ∈ N ∗ et des éléments x1 , . . . , xn ∈ X tels que X ⊂ ∪ 1≤i≤nB(xi, η x i).
Soit η = min1≤i≤n ηx i . On a η > 0. Soit x, y ∈ X tels que d(x, y ) < η. Soit i tel que d(x, xi ) < ηxi .
On a
d(x, x i) < ηxi < 2η xi donc δ (f (x), f(xi )) < ε
d(y, xi ) ≤ d(y, x) + d(x, xi ) < η + η xi ≤ 2η xi donc δ (f (y ), f(xi )) < ε.
On en déduit δ (f (x), f(y)) ≤ δ (f (x), f(xi ))+δ (f (xi ), f(y)) < 2ε. Ceci est vrai pour tout x, y ∈ X
tels que d(x, y) < η, donc A est bien uniformément équicontinue.
2/ Montrons (i) ⇒ (ii). Soit x ∈ X et ε > 0. Comme A est relativement compact, on peut le
recouvrir par un nombre fini de boules centrées dans A de rayon ε, c’est-à-dire
∃n ∈ N ∗, ∃f 1, . . . , fn ∈ A, A ⊂ ∪1≤i≤n B(fi , ε). (∗)
Chaque fonction fi est continue en x donc
∀i, ∃η i > 0; ∀y ∈ K, d(x, y) < η i ⇒ δ (f i(x), f i (y)) < ε.
Choisissons η = inf 1≤i≤n η i. Soit f ∈ A. D’après (*) , il existe i tel que f ∈ B(fi , ε). Avec ce
choix de η > 0, on a démontré (ii) car pour tous y ∈ K tel que d(x, y ) < η, on a l’inégalité
δ (f (x), f(y)) ≤ δ (f (x), fi (x)) + δ (f i (x), fi(y)) + δ (fi (y ), f(y)) < ε + ε + ε = 3ε.
- Montrons maintenant (ii) ⇒ (i). Suivons l’indication et démontrons que A est précompact
et complet. Montrons d’abord que A est précompact. Pour cela il suffit de montrer que A est
précompact. Soit ε > 0. On va montrer qu’on peut recouvrir A par un recouvrement fini de
boules de rayon 4ε. Pour tout x ∈ X , il existe ηx > 0 tel que
∀f ∈ F, ∀y ∈ X, d(x, y ) < ηx ⇒ δ (f (x), f(y)) < ε.
Du recouvrement d’ouverts ∪x∈XB(x, η x) de X on peut en extraire un sous-recouvrement fini
∪1≤i≤n B(xi , ηx i ), avec les x i ∈ X . Comme Y est compact on peut le recouvrir par un nombre
fini de boules de rayon ε > 0, autrement dit il existe m ∈ N ∗ et z1 , . . . , zm ∈ Y tels que
Y ⊂ ∪1≤j ≤m B(zj , ε). Pour tout n-uplet J = (j 1 , . . . , jn) de {1, . . . , m}n , on note
B J = {f ∈ C(X, Y ) | ∀i = 1, . . . , n, ∀x ∈ B (xi , ηx i ), δ(f (x), z ji ) < 2ε}.
Pour tout J = (j 1, . . . , jn ), BJ est inclus dans une boule de rayon 4ε. En effet, choisissons
arbitrairement fJ ∈ BJ . Soit f ∈ BJ . Pour tout x ∈ X , il existe i tel que x ∈ B(xi, ηxi ) donc
δ (f (x), zji ) < 2ε et δ (fJ (x), zji ) < 2ε. Donc δ (f (x), f J (x)) ≤ δ (f (z ), zji ) + δ (zji , f J(z)) < 4ε.
Ainsi, ∆(f, fJ ) < 4ε (le sup d’une fonction continue est atteint sur un compact donc l’inégalité
reste stricte) donc BJ ⊂ B(f J , 4ε). Par ailleurs, les BJ recouvrent bien A. En effet, soit f ∈ A.
Pour tout i, soit ji tel que f (xi ) ∈ B(zj i , ε). On a
∀x ∈ B (x i, η xi ), δ(f (x), zj i) ≤ δ (f (x), f (x i)) + δ (f (x i), z ji ) < 2ε.
En notant J = (j1 , . . . , jn ), on en déduit que f ∈ BJ .
Il nous reste à montrer que A est complet. Tout fermé dans un complet est complet, il nous
suffit donc de prouver que C(X, Y ) est complet. Pour cela, on considère une suite de Cauchy
(f n)n∈N de C(X, Y ). Pour tout p, q ∈ N et pour tout x ∈ X on a δ (fp (x), f q (x)) ≤ ∆(fp , fq ),
donc la suite (fn(x)) est de Cauchy. Cette suite est à valeur dans le compact Y , donc complet,
donc elle converge. En désignant par f (x) sa limite, on défini une fonction f de X dans Y .
Comme f est la limite uniforme des fonctions continues fn , elle est donc continue. Ainsi (fn )
converge vers f ∈ C(X, Y ).
70 1. TOPOLOGIE SUR LES ESPACES MÉTRIQUES ET LES ESPACES VECTORIELS NORM ÉS

Remarque. - Le résultat de la question 1/ peut être vu comme une généralisation du


théorème de Heine. Nous ne l’avons pas utilisé pour la solution de la question 2/.
– Le résultat de la question 2/ est une version faible du théorème d’Ascoli. Dans sa
version plus générale, le compact Y est remplacé par un espace métrique quelconque et
la condition d’équicontinuité est complétée par une autre condition (dite de ponctuelle
relative compacité).
– On peut aussi prouver le résultat de la question sans faire appel à la propriété ”tout
précompact complet est compact” , en démontrant que de toute suite (fn ) de A, on peut
en extraire une sous-suite convergente, en construisant cette sous-suite par un procédé
diagonal (comme pour l’exercice 2 page 32).
– La
Le théorème d’Ascoli a de multiple conséquences, comme la compacité de certains
opérateurs ou l’existence de solutions dans certaines équations différentielles.
CHAPITRE 2

Fonctions d’une variable réelle

Jusqu’au dix-septième siècle, la notion de fonction n’était pas dégagée de


façon explicite. Le mot fonction semble avoir été utilisé pour la première
fois par Leibniz en 1692, pour désigner les grandeurs géométriques dépendant
d’autres grandeurs géométriques. Pour Euler, une fonction était une expres-
sion construite au moyen des opérations algébriques élémentaires, des opé-
rations transcendantes (exponentielle, logarithmes, fonctions circulaires), et
d’opérations telles que formation de séries, de produits infinis, de suites. C’est
finalement Dirichlet qui en introduisant une fonction discontinue partout (la
fonction caractéristique des irrationnels), définit explicitement la notion de
fonction comme nous la connaissons aujourd’hui.
Le concept de dérivée, quant à lui, déjà en germe au début du dix-septième
siècle dans divers domaines, fut unifié par Newton et Leibniz. Alors que Leib-
niz utilisait la notation ( dydx ), Lagrange évacuait les infiniments petits et in-

troduisit la notation f . C’est enfin Cauchy qui définit la dérivée à partir du
concept de limite.
Rolle énonça en 1691 le théorème auquel son nom est resté attaché, dans
le cas des polynômes, en s’appuyant sur l’intuition géométrique. Lagrange,
à la fin du dix-huitième siècle, prit un point de vue totalement différent,
fondé sur des considérations purement analytiques. Il montra qu’une fonc-
tion dont la dérivée est positive est croissante, et en déduisit la formule des
accroissements finis. Lagrange souligna par ailleurs que pour appliquer cor-
rectement la formule de Taylor (qui était jusque là utilisée abusivement), il
convient de considérer une somme partielle et d’évaluer le reste, et donne
le premier une version correcte de cette formule. On pensait alors que toute
fonction était égale à sa série de Taylor, et Cauchy exhiba le contre-exemple
x → exp(−1/x2 ).

1. Fonctions dérivables
1.1. Dérivabilité
Définition 1. Soient E un R-e.v.n, I un intervalle de R, f : I → E une application et
a ∈ I . On dit que f est dérivable en a si
f (t) − f (a)
lim
t→a
t∈I {a}
t−a

existe. Lorsqu’elle existe, cette limite est notée f  (a) (ou df


dx
(a)).
On dit que f est dérivable à gauche (resp. à droite) en a si la limite
 
f (t) − f (a) resp. lim f (t) − f (a)
lim
t→a
t<a t−a t→a
t>a t−a
t∈I t∈I

existe. On la note alors fg (a) (resp. f d (a)).


72 2. FONCTIONS D’UNE VARIABLE RÉELLE


Remarque 1. — Si a ∈ I , f est dérivable en a si et seulement si f est dérivable à
gauche, dérivable à droite en a et fg(a) = f d (a).
— Si f est dérivable en a, f est continue en a.
— Sur l’ensemble D des points où f est dérivable, on peut définir l’application a →
f (a) appelée application dérivée de f et notée f .
— L’application f est dérivable en a si et seulement si
∃ ∈ E, f (x) = f (a) + (x − a) + o(x − a) (x → a).
On a alors  = f  (a).
— Une fonction dérivée n’est pas forcément continue. Considérons par exemple
f : R → R x → x2 sin(1/x) si x = 0, 0 → 0.

Si x = 0, f est dérivable en x et on a f (x) = 2x sin(1/x) − cos(1/x). En 0, comme
−x2 ≤ f (x) ≤ x 2 pour tout x, on a
f (x) − f (0) f (x) − f (0)
−x ≤ ≤ x donc lim = 0,
x−0 x→0
x=0
x
en d’autres termes, f  (0) existe et vaut 0. Cependant, on voit que f (x) n’admet
pas de limite lorsque x tend vers 0.
— Nous verrons cependant qu’une fonction dérivée vérifie toujours la propriété des va-
leurs intermédiaires (théorème de Darboux, voir l’exercice 4 page 80) et est continue
sur un ensemble dense (voir l’exercice 2 page 419). Par contre, il existe des fonctions
dérivées discontinues sur un ensemble dense (voir l’exercice 9 page 244).
— Une fonction dérivée, même bornée, n’est pas nécessairement Riemann-intégrable.
Par récurrence, on peut définir la fonction dérivée n-ième (lorsqu’elle existe) par
f  = (f ) , f = (f  ) , . . . , f (n) = (f (n−1)), . . . .
Une application f : I → E est dite de classe C n si f (n) existe sur I et y est continue. On
note parfois Cn (I, E) l’ensemble des applications de classe C n de I dans E . Lorsque f est
de classe Cn pour tout n, on dit que f est de classe C ∞ .
Proposition 1. Soient I un intervalle de R, f et g deux applications de I dans E ( où
E est un R-e.v.n), dérivables en a ∈ I . Alors
(i) Pour tout λ, µ ∈ R, λf + µg est dérivable en a et (λf + µg )(a) = λ f (a) + µ g (a).
(ii) Si E est une R-algèbre normée, le produit f g est dérivable en a et
(f g) (a) = f (a)g (a) + f (a)g (a).
(iii) Si E = R ou C et si g (a) = 0, le rapport f /g est dérivable en a et
 
f f (a)g (a) − f (a)g  (a)
(a) = .
g g (a)2
Conséquence : En procédant par récurrence, on en déduit que la somme, le produit, le
rapport de deux fonctions de classe Cn est de classe C n .
Proposition 2 (Formule de Leibniz). Soient I un intervalle de R, f et g deux
applications de I dans une R-algèbre normée E et a ∈ I tel que f (n)(a) et g (n)(a) existent.
Alors le produit f g est n fois dérivable en a et
n
(n)
(f g) (a) = Cnk f (k) (a)g(n−k) (a),
k=0

où par convention f (0) = f et g (0) = g.


1. FONCTIONS DÉRIVABLES 73

Proposition 3. Soient I et J deux intervalles de R, E un R-e.v.n, f : J → E et


g : I → J deux applications, et a ∈ I tel que f est dérivable en a et g dérivable en f (a).
L’application composée f ◦ g est dérivable en a et on a
(f ◦ g ) (a) = g  (a) · (f ◦ g )(a).
Conséquence : On en déduit que la composée de deux applications de classe Cn est de
classe Cn (procéder par récurrence sur n).
Homéomorphismes dérivables. Les lettres I et J désignent deux intervalles de R.
Définition 2. Soient f : I → J une bijection et n ∈ N∗ . On dit que f est un Cn -
difféomorphisme si f et f −1 sont de classe C n .
Proposition 4. Soit f une bijection de I dans J , dérivable en a ∈ I . L’application f−1
est dérivable en b = f (a) si et seulement si f (a) = 0, et on a
1 1
(f −1) (b) =  =  −1 .
f (a) f (f (b))
Conséquence : Une fonction surjective f de I dans J est un C n -difféomorphisme si et
seulement si f et de classe C n et vérifie f (a) = 0 pour tout a ∈ I (l’injectivité découle du
théorème de Rolle).
1.2. Dérivées des fonctions usuelles
Le tableau qui suit donne la dérivée des fonctions usuelles.

f (x) f  (x) f (x) f (x)


xr (r ∈ R) rxr−1 ex ex
1
xs (s ∈ C) sx s−1 arctan x 1+x2
1
log x x
ch x sh x
sin x cos x sh x ch x
cos x − sin x th x 1 − th 2 x = 1
ch 2 x
1 √ 1
tan x 1 + tan 2 x = cos2 x argch x x 2 −1
arcsin x √ 1 argsh x √ 1
1−x 2 x 2 +1
1 1
arccos x − √1−x 2 argth x 1−x2

1.3. Résultats relatifs à la dérivabilité pour les fonctions à valeurs réelles


Dans toute cette partie, [a, b] désigne un segment de R non réduit à un singleton.
Proposition 5. Soit I un intervalle de R et f : I → R une application. Si f admet un

extremum relatif en c ∈ I et si f  (c) existe, alors f (c) = 0.
 Théorème 1 (de Rolle). Soit f : [a, b] → R une application vérifiant :
(i) f est continue sur [a, b].
(ii) f est dérivable sur ]a, b[.
(iii) f (a) = f (b).
74 2. FONCTIONS D’UNE VARIABLE RÉELLE

f (a) = f (b)
x
0 a c b

Figure 1. Une illustration du théorème de Rolle.

Alors il existe c ∈ ]a, b[ tel que f  (c) = 0.


Démonstration. Si f est constante sur [a, b], le résultat est évident. Sinon, il existe x0 ∈ ]a, b[ tel
que f (x0) = f (a), par exemple f (x 0 ) > f(a). Comme [a, b] est un compact de R et que f est
continue, il existe c ∈ [a, b] tel que f (c) = supx∈[a,b] f (x). Or f (c) ≥ f (x0 ) > f (a) = f (b), donc
c ∈ ]a, b[, donc f (c) étant un extremum de f , on a f  (c) = 0 d’après la proposition 5. 

 Théorème 2 (des accroissements finis). Soit f : [a, b] → R une application continue


sur [a, b] et dérivable sur ]a, b[. Alors
f (b) − f (a)
∃c ∈]a, b[, f  (c) = .
b−a

f (b)

f (a)
x
0 a c b

Figure 2. Une illustration du théorème des accroissements finis.


f (b) − f (a)
Démonstration. Posons A = et ϕ : [a, b] → f (x) −A(x − a). On a ϕ(a) = f (a) = ϕ(b)
b−a
donc d’après le théorème de Rolle, il existe c ∈ ]a, b[ tel que ϕ (c) = 0. On en déduit f (c) = A,
d’où le résultat. 
Conséquence : Une fonction continue sur [a, b], dérivable sur ]a, b[ est croissante si et
seulement si f (x) ≥ 0 pour tout x ∈ ]a, b[. Elle est constante si et seulement si f  (x) = 0
pour tout x ∈ ]a, b[.
Remarque 2. Les théorèmes de Rolle et des accroissements finis sont faux lorsque f est
à valeurs dans un R-e.v.n. Par exemple, l’application f : [0, 2π ] → C t → eit vérifie
f (0) = f (2π ) et pourtant pour tout t ∈ [0, 2π ], f  (t) = ieit n’est jamais nul.

Théorème 3 (des accroissements finis généralisés). Soient f , g deux applications


de [a, b] dans R, continues sur [a, b] et dérivables sur ]a, b[. Alors
   
∃c ∈ ]a, b[, f (b) − f (a) g  (c) = g (b) − g (a) f  (c).
f (b) − f (a) f (c)
Si g (c) = 0 et g(a) = g (b), cette égalité s’écrit aussi =  .
g (b) − g (a) g (c)
1. FONCTIONS DÉRIVABLES 75

Démonstration. Il suffit d’appliquer le théorème de Rolle à l’application


[a, b] → R x → [f (b) − f (a)] [g (x) − g(a)] − [g(b) − g(a)] [f (x) − f (a)],
qui est continue sur [a, b], dérivable sur ]a, b[ et s’annule en a et en b. 
 
Conséquence : (Règle de L’Hospital) Si f (a) = g (a) = 0 et si  = lim xx→=aa f (x)/g (x)
existe, alors lim xx→ a f (x)/g(x) = .
=a

Remarque 3. La réciproque de la règle de L’Hospital est fausse. Autrement dit, on peut


avoir f (a) = g (a) = 0 et f (x)/g(x) peut converger lorsque x → a (x = a), sans que
f  (x)/g(x) converge lorsque x → a (x = a).
 Théorème 4 (Formule de Taylor-Lagrange). Soit f : [a, b] → R une application
de classe C n sur [a, b], telle que f (n+1) existe sur ]a, b[. Alors
(b − a)n (n) (b − a) n+1 (n+1)
∃c ∈ ]a, b[, f (b) = f (a) + (b − a)f  (a) + · · · + f (a) + f (c) .
n! (n + 1)!
  
reste de Lagrange

Démonstration. Considérons l’application


(b − x) n (n) (b − x)n+1
ϕ : [a, b] → R x → f (b) − f (x) − (b − x)f (x) − · · · − f (x) − A ,
n! (n + 1)!
la constante A étant choisie telle que ϕ(a) = ϕ(b) = 0. Cette application est continue sur [a, b],
dérivable sur ]a, b[ et
(b − x) n (n+1) (b − x) n
∀x ∈ ]a, b[, ϕ (x) = − f (x) + A ,
n! n!
donc d’après le théorème de Rolle, il existe c ∈ ]a, b[ tel que ϕ(c) = 0, ce qui s’écrit A = f (n+1) (c),
d’où le résultat car ϕ(a) = 0. 

Remarque 4. — Avec n = 0, on retrouve le théorème des accroissements finis.


— Lorsque 0 appartient à l’intervalle de définition I de f , on a, sous les mêmes hy-
pothèses,
xn (n) xn+1
∀x ∈ I, ∃θ ∈ ]0, 1[, f (x) = f (0) + xf  (0) + · · · + f (0) + f (n+1) (θx).
n! (n + 1)!
Cette relation s’appelle formule de Maclaurin (avec reste de Lagrange).
1.4. Résultats relatifs à la dérivabilité pour les fonctions à valeurs dans un
e.v.n
Dans toute cette partie, E désigne un R-e.v normé.
Lorsque les applications sont à valeurs dans un R-e.v.n, les formules de Taylor du
paragraphe précédent ne sont plus vraies (voir par exemple la remarque 2). Par contre,
il existe des résultats analogues faisant intervenir des inégalités. Le théorème qui suit est
une inégalité fondamentale dont nous nous servirons beaucoup.
Théorème 5. Soient F : [a, b] → E et g : [a, b] → R deux applications continues sur
[a, b], dérivables sur ]a, b[. Si pour tout t ∈ ]a, b[, on a F  (t) ≤ g (t), alors F (b) −
F (a) ≤ g (b) − g (a).
Démonstration. Supposons dans un premier  temps que F (t) < g  (t) pour tout t ∈]a, b[. Pour
 F (t)−F (x)  g(t)−g (x)
tout x ∈ ]a, b[, on a lim t→x  t−x  − t−x < 0, donc
t>x
 
 F (t) − F (x)  g (t) − g (x)
∀x ∈ ]a, b[, ∃y > x, ∀t ∈ ]x, y [,  <
 t−x  t−x
76 2. FONCTIONS D’UNE VARIABLE RÉELLE

de sorte que
∀x ∈ ]a, b[, ∃y > x, ∀t ∈ [x, y ], F (t) − F (x) ≤ g (t) − g(x). (∗)
 
Soit [α, β] ⊂ ]a, b[ et montrons
F (β ) − F (α) ≤ g (β ) − g (α). (∗∗)
Soit Γ = {θ ∈ ]α, β ], ∀t ∈ [α, θ ], F (t) − F (α ) ≤ g (t) − g (α)}. D’après (*), Γ est non vide. Soit
γ = sup Γ. Montrons γ = β , ce qui prouvera (**). Par continuité de F et g , on a F (γ )− F (α ) ≤
g (γ ) − g(α). Si γ < β, d’après (*),
∃δ ∈ ]γ, β ], ∀t ∈ [γ, δ ], F (t) − F (γ ) ≤ g (t) − g (γ ),
donc
∀t ∈ [γ, δ ], F (t) − F (α) ≤ F (t) − F (γ ) + F (γ ) − F (α) ≤ g (t) − g(α).
Ceci montre que δ ∈ Γ, ce qui est absurde car δ > γ = sup Γ.
L’assertion (**) est donc prouvée. En faisant tendre α vers a puis β vers b, on en déduit, en
vertu de la continuité de F et g , l’inégalité F (b) − F (a) ≤ g (b) − g(a).
– Ramenons nous au cas général. Pour tout ε > 0, on définit gε(t) = g (t) + εt. Pour tout ε > 0,
on a F  (t) < g ε (t) sur ]a, b[ , donc d’après ce que l’on vient de prouver, F (b) − F (a) ≤
gε (b) − gε (a), d’où le résultat en faisant tendre ε vers 0. 

Remarque 5. Lorsque F et g sont de classe C1 sur [a, b], le résultat s’obtient facilement
b b b
par intégration, en écrivant F (b) − F (a) =  a F   ≤ a F   ≤ a g  = g (b) − g (a).

Théorème 6 (In égalité des accroissements finis). Soit F : [a, b] → E une


application continue sur [a, b], dérivable sur ]a, b[. S’il existe M > 0 tel que F  (t) ≤ M
pour tout t ∈ ]a, b[, alors F (b) − F (a) ≤ M (b − a).
Démonstration. Il suffit d’appliquer le résultat précédent avec g (t) = M t. 

Remarque 6. Dans le cas d’une fonction à valeurs réelles, cette inégalité s’obtient direc-
tement à partir de l’égalité des accroissements, sans utiliser le théorème précédent.
Une conséquence importante de l’inégalité des accroissements finis est la suivante.
 Proposition 6. Soit F : [a, b[→ E une application continue, dérivable sur ]a, b[ et telle
 
→a F (t) existe. Alors F est dérivable en a et F (a) = .
que  = lim xx>a
Démonstration. Quitte à changer F en F (t) − t  on peut supposer  = 0. Soit ε > 0. Par
hypothèse, il existe c ∈ ]a, b[ tel que F (t) < ε pour tout t ∈ ]a, c[. L’inégalité des accroissements
finis entraı̂ne  
 F (t) − F (a) 
∀t ∈ ]a, c[,   ≤ ε.
 t−a 
Ceci étant possible pour tout ε > 0, on en déduit que [F (t) − F (a)]/(t − a) converge vers 0
lorsque t tend vers a, d’où le résultat. 

Remarque 7. En plus de l’existence de F (a), la proposition montre que la fonction dérivée


F  est continue en a.

Théorème 7 (Inégalité de Taylor-Lagrange). Soit F : [a, b] → E une application


de classe Cn sur [a, b], n + 1 fois dérivable sur ]a, b[. On suppose qu’il existe M > 0 tel
que ∀t ∈ ]a, b[, F (n+1)(t) ≤ M . Alors
 
 n  n+1
F (b) − F (a) − (b − a)F  (a) − · · · − (b − a) F (n) (a) ≤ M (b − a) .
 n!  (n + 1)!
1. FONCTIONS DÉRIVABLES 77

Démonstration. Il suffit d’appliquer le théorème 5 aux fonctions


(b − x)n (n) (b − x)n+1
G(x) =F (b) − F (x) − (b − x)F  (x) − · · · −  g (x) = −M
F (x) et .
n! (n + 1)!

 Théorème 8 (Formule de Taylor-Young). Soient n ∈ N et F une fonction définie


sur un intervalle I de R, à valeurs dans E , de classe C n sur I . Soit a ∈ I tel que F (n+1)(a)
existe. Alors, lorsque h → 0 on a
h n (n) hn+1
F (a + h) = F (a) + h F  (a) + · · · + F (a) + F (n+1)(a) + o(hn+1 ).
n! (n + 1)!
Démonstration. On procède par récurrence sur n ∈ N. Pour n = 0, la formule F (a + h) =
F (a) + h F (a) + o(h) résulte de la définition de F (a). Supposons le résultat vrai au rang n − 1,
ce qui entraı̂ne (en l’appliquant à F  )
h n (n+1)
G(h) = F  (a + h) − F  (a) − hF (a) − · · · −F (a) = o(hn ),
n!
et montrons le au rang n. La fonction G est la dérivée de l’application
hn+1
H (h) = F (a + h) − F (a) − hF (a) − · · · − F (n+1) (a),
(n + 1)!
et il s’agit de montrer que H (h) = o(h n+1 ). Soit ε > 0. Il existe α > 0 tel que pour |h| < α,
|G(h)| < ε|h| n. Si g (h) = εhn+1 /(n + 1), on a donc
∀h, 0 < h < α, H (h) < g(h).
En appliquant le théorème 5 entre les points 0 et h, on en déduit
h n+1
∀h, 0 ≤ h < α, H (h) = H (h) − H (0) ≤ g(x) − g (0) = ε .
n+1
On procéderait de même pour −α < h ≤ 0, d’où le résultat. 
Citons enfin une dernière formule de Taylor, qui rend parfois de précieux services car
elle donne beaucoup d’informations sur le terme de reste. L’e.v.n E doit être ici un espace
de Banach pour assurer l’existence de l’intégrale (voir la définition 4 page 124).
 Théorème 9 (Formule de Taylor avec reste int égral). Soient n ∈ N et une
application F : [a, b] → E de classe C n+1 sur [a, b], où E est un R-espace de Banach.
Alors
 b
 (b − a)n (n) (b − t)n (n+1)
F (b) = F (a) + (b − a)F (a) + · · · + F (a) + F (t) dt.
n! a n!
La preuve est immédiate par récurrence sur n.
1.5. Exercices
Exercice 1. Démontrer les inégalités suivantes.
x2 x2 x3
a) ∀x ≥ 0, x− ≤ log(1 + x) ≤ x − + .
2 2 3
3 3 5
x x x
b) ∀x ≥ 0, x− ≤ sin x ≤ x − + .
6 6 120
x2 x 2 x4
c) ∀x ∈ R, 1− ≤ cos x ≤ 1 − + .
2 2 24
78 2. FONCTIONS D’UNE VARIABLE RÉELLE

Solution. a) D’après la formule de Taylor-Lagrange appliquée à la fonction f : x → log(1 + x) :


x2  x3 
∀x ≥ 0, ∃θ ∈ ]0, 1[, log(1 + x) = f (x) = f (0) + xf  (0) + f (0) + f (θx). (∗)
2 6
Comme
1 1 2
f (x) = log(1 + x), f (x) = , f (x) = − , f  (x) = ,
1+x (1 + x) 2 (1 + x)3
on a f (0) = 0, f (0) = 1, f (0) = 2 et 0 ≤ f  (θx) ≤ 2, ce qui en remplaçant dans (*) donne le
résultat demandé.
b) On procède de la même manière : on considère le développement de Taylor-Lagrange de
x → sin x à l’ordre 3 pour prouver l’inégalité de gauche, à l’ordre 5 pour celle de droite.
c) Pareil en développant à l’ordre 2 puis à l’ordre 4.

Exercice 2. Soit f : R → R une application dérivable en 0 et nulle en 0. Soit  ∈ N∗ .


Pour tout n ∈ N∗, on pose
n  
k
Sn = f .
k=0
n2
Montrer que la suite (Sn ) n∈N ∗ converge et calculer sa limite.

Solution. Le principe est le suivant. Lorsque n est grand, chacun des termes k/n 2 pour 0 ≤
k ≤ n est petit, donc au premier ordre, on a l’approximation f ( nk2 ) ≈ f  (0)nk2 , de sorte que
  n
Sn ≈ fn(0)
2
2 
k=0 k ≈  /2f (0). Nous allons mettre rigoureusement en forme cette idée.
L’application f est dérivable en 0, donc f (x) = f (0) + x f  (0) + o(x) = x f (0) + o(x). Ainsi,
si on se donne ε > 0,
∃η > 0, ∀x ∈ [0, η[, |f (x) − xf  (0)| ≤ εx.
Soit N ∈ N∗ tel que /N < η . Pour tout n ≥ N et pour tout entier k vérifiant 0 ≤ k ≤ n, on a
k n  
0≤ 2
≤ 2 = ≤ < η,
n n n N
donc    
 k k 
∀n ≥ N, ∀k, 0 ≤ k ≤ n, f − f 
(0)  ≤ε k,
 n2 n2  n2
de sorte que
 n    n    
 k
n
 k    k k 
n
 k
    
∀n ≥ N,  f 2
− f (0) 2 ≤ f
 2
− f (0) 2  ≤ ε . (∗)
 n n  n n n2
k=0 k=0 k=0 k=0
Or
n
 k 1 n(n + 1) 2 
2
= 2
= + ,
n n 2 2 2n
k=0
donc (*) s’écrit aussi
   2 
 2 2 2
∀n ≥ N, Sn − f (0)  − f (0)   ≤ ε  +  ≤  2ε.
 2 2n  2 2n
On a donc
     
 2  2 2   2  2
 Sn − f (0)   ≤ Sn − f  (0) − f (0)   +  f (0)   ≤ 2 ε + |f (0)| .
 2   2 2n   2n  2n
Si on choisit N1 ≥ N tel que |f  (0)|/(2N1 ) < ε, on a donc |Sn − f (0)2 /2| < 2 2ε pour tout
n ≥ N1 , ce qui prouve que (Sn ) converge et tend vers f (0)2 /2.
1. FONCTIONS DÉRIVABLES 79

Exercice 3. a) Soit l’application f : R → R définie par f (x) = 0 si x ≤ 0, et par


f (x) = e−1/x si x > 0. Démontrer que f est de classe C∞ sur R.
b) Démontrer qu’il existe une fonction ϕ de classe C∞ sur R telle que
∀x ∈ [−1, 1], ϕ(x) = 1 et ∀x ∈ R, |x| ≥ 2, ϕ(x) = 0.

(ϕ)

x
−2 −1 0 1 2

Figure 3. Le graphe de la fonction ϕ.

Solution. a) Il est clair que f est de classe C ∞ sur chacun des intervalles ] − ∞, 0[ et ]0, +∞[.
Il nous reste donc à prouver l’existence de f (n) (0) pour tout n ∈ N ∗ .
Commençons par montrer par récurrence sur n que
 
(n) −1/x 1
∀n ∈ N, ∃Pn ∈ R[X ], ∀x > 0, f (x) = e Pn . (∗)
x
Pour n = 0 c’est vrai avec P0 = 1. Supposons le résultat vrai au rang n − 1 et montrons le au
rang n. Il suffit d’écrire
     
(n)
 (n−1)  1 −1/x 1 1 −1/x  1 −1/x 1
∀x > 0, f (x) = f (x) = 2 e Pn−1 − 2e Pn−1 =e Pn
x x x x x

avec Pn (X) = X 2 Pn−1 (X ) − X 2Pn−1 (X).
On déduit de (*) que lim x→0 f (x) = 0 pour tout n ∈ N∗ . Or pour x < 0, f (n) (x) = 0,
(n )
x>0
donc
∀n ∈ N∗ , lim f (n)(x) = 0.
x→0
x=0

En utilisant maintenant la proposition 6 page 76 par récurrence sur n, on en déduit que f (n) (0)
existe pour tout n et f(n) (0) = 0. Finalement, f est bien de classe C∞ sur R.
b) Nous allons donner deux manières de construire une telle fonction.
Première méthode. Considérons l’application
f (f (1) − f (x))
g: R→R x → .
f (f (1))

y
(g)

x
0 1

Figure 4. Le graphe de la fonction g.

L’application g, composée de fonctions de classe C∞, est de classe C ∞ sur R. Par ailleurs,
f (f (1) − 0)
∀x ≤ 0, g(x) = =1
f (f (1))
∀x ≥ 1, f (1) − f (x) ≤ 0 =⇒ f (f (1) − f (x)) = 0 =⇒ g (x) = 0.
80 2. FONCTIONS D’UNE VARIABLE RÉELLE

On voit maintenant facilement que l’application ϕ : R → R x → g (x − 1)g (−x − 1) convient.


Seconde méthode. L’application g : R → R x → f (x + 1)f (−x + 1) est de classe C ∞ sur R et
∀x ∈ ] − 1, 1[, g(x) > 0, ∀x ∈ R, |x| ≥ 1, g(x) = 0.

y
(g)
x
−1 0 1

Figure 5. Le graphe de la fonction g (seconde méthode).


x  +∞
On pose maintenant h : R → R x → 1K −∞ g (t) dt, où K = −∞ g (t) dt. Cette application
est de classe C∞ et vérifie h(x) = 0 pour x ≤ −1 et h(x) = 1 pour x ≥ 1. On termine en
choisissant pour ϕ l’application
ϕ : R → R x → h(2x + 3)h(−2x + 3).

Exercice 4 (Théorème de Darboux). Par commodité de notation, on désigne par


(t, u) l’intervalle [t, u] si t ≤ u, l’intervalle [u, t] si u < t.
Soit I un intervalle de R et f : I → R une application dérivable sur I . On veut
montrer que f  vérifie la propriété des valeurs intermédiaires, c’est-à-dire que f  (I ) est un
intervalle.
On se donne (a, b) ∈ I2 tel que a < b et y ∈ (f (a), f  (b)). Il s’agit donc de montrer
qu’il existe c ∈ [a, b] tel que f  (c) = y. Nous proposons deux méthodes pour obtenir ce
résultat.
1/ (Première méthode.) On considère les deux applications
f (x) − f (a)
ϕ : [a, b] → R ϕ(x) = si x = a, ϕ(a) = f (a)
x−a
f (b) − f (x)
ψ : [a, b] → R ψ (x) = si x = b, ψ (b) = f  (b).
b−x
Montrer que y ∈ ϕ([a, b]) ∪ ψ ([a, b]). En déduire l’existence de c ∈ [a, b] tel que y = f  (c).
2/ (Seconde méthode.) a) Soit g : I → R une application vérifiant g  (a) ≥ 0 et g (b) ≤ 0.
Montrer l’existence de c ∈ [a, b] tel que g (c) = 0.
b) En déduire qu’il existe c ∈ [a, b] tel que f (c) = y .

Solution. 1/ L’application ϕ est clairement continue sur ]a, b], et elle est continue en a par
définition de f (a). Elle est donc continue sur [a, b], ce qui avec le théorème des valeurs in-
termédiaires entraı̂ne
 
 f (b) − f (a)
(ϕ(a), ϕ(b)) = f (a), ⊂ ϕ([a, b]).
b−a
De même,  
f (b) − f (a) 
(ψ (a), ψ(b)) = , f (b) ⊂ ψ ([a, b]).
b−a
On en déduit
   
   f (b) − f (a) f (b) − f (a) 
(f (a), f (b)) ⊂ f (a), ∪ , f (b) ⊂ ϕ([a, b]) ∪ ψ ([a, b]).
b−a b−a
Or y ∈ (f (a), f  (b)), donc y ∈ ϕ([a, b]) ∪ ψ([a, b]).
Si y ∈ ϕ([a, b]), deux cas se présentent.
1. FONCTIONS DÉRIVABLES 81

(i) Soit y = f  (a), et alors c’est terminé avec c = a.


(ii) Soit ∃x ∈ ]a, b], y = ϕ(x) = f (xx)− f (a)
−a et d’après le théorème des accroissements finis, il

existe c ∈ ]a, x[⊂ [a, b] tel que y = f (c).
On procéderait de même si y ∈ ψ ([a, b]), d’où le résultat.
2/ a) Si g (a) = 0 ou g (b) = 0, c’est évident avec c = a ou c = b. Sinon g  (a) > 0 et g (b) < 0.
Considérons le développement de Taylor-Young de g à l’ordre 1 au voisinage de a
g (x) = g (a) + (x − a)g(a) + o(x − a) = g (a) + (x − a)(g (a) + ε(x − a)), ε(x − a) = o(1).
Il existe η > 0 tel que ε(x − a) < g (a)/2 sur [a, a + η], donc g (x) ≥ g (a) + (x − a)g  (a)/2 > g(a)

sur ]a, a + η]. On démontrerait de même l’existence d’un α > 0 tel que g (x) > g(b) sur [b − α, b[.
L’application g est continue sur le compact [a, b], donc il existe c ∈ [a, b] vérifiant g(c) =
sup x∈[a,b] g (x). D’après ce que l’on vient de voir, on a c = a et c = b. Donc c ∈ ]a, b[, et on en
déduit que g (c) = 0, d’où le résultat.
b) Considérons l’application g : I → R x → yx − f (x). Quitte à changer f en −f , on peut
supposer f (b) ≥ f (a). Comme y ∈ [f (a), f (b)], on a g (a) = y − f (a) ≥ 0 et g (b) = y − f (b) ≤
0. Le résultat de la question précédente entraı̂ne alors l’existence de c ∈ [a, b] tel que g  (c) = 0.
On a donc y − f (c) = 0, donc y = f  (c).
Remarque. Si f est de classe C 1 , ce théorème résulte tout simplement du théorème des
valeurs intermédiaires appliqué à la fonction dérivée f . Mais une fonction dérivée n’est
pas forcément continue (voir la remarque 1 page 71).
– Une démonstration de nature plus topologique de ce théorème fait l’objet de l’exercice 9
page 47.
– La réciproque du théorème de Darboux est fausse : une fonction vérifiant la propriété
des valeurs intermédiaires n’est pas forcément une fonction dérivée.

Exercice 5 (Formule de Simpson). a) Soit α > 0 et g : [−α, α] → R une application


impaire 5 fois dérivable sur [−α, α]. Montrer
α  α5 (5)
∃θ ∈ ]0, α[, g(α) = (g (α) + 2g (0)) − g (θ ).
3 180

b) Soient a, b ∈ R, a < b, et f : [a, b] → R une application de classe C 5 sur [a, b]. Montrer
  
b−a    a+b (b − a)5 (5)
∃θ ∈ ]a, b[, f (b) − f (a) = f (a) + f (b) + 4f − f (θ ).
6 2 2880

c) (Application.) Soit f : [a, b] → R une application de classe C 4 . Soit n ∈ N∗ . On


considère la subdivision a = x0 < x 1 < · · · < x2n = b telle que x i = a + i b2n
−a
pour tout i.
(4)
Si M = sup |f (x)|, montrer
x∈[a,b]
 b 

 f (t) dt − b − a f (a) + 4f (x 1 ) + 2f (x 2) + 4f (x3 ) + · · ·
 6n
a

  (b − a)5 M

· · · + 2f (x2n−2 ) + 4f (x2n−1 ) + f (b)  ≤ · .
 n4 2880

Solution. a) On considère l’application


t  A t5
ϕ : [−α, α] → R t → g (t) − (g (t) + 2g (0)) + ,
3 180
82 2. FONCTIONS D’UNE VARIABLE RÉELLE

la constante A étant choisie telle que ϕ(α ) = 0. L’application ϕ est trois fois dérivable et on a
2  2 t t4
ϕ(t) = g (t) − g  (0) − g (t) + A,
3 3 3 36
3
1 t t
ϕ (t) = g(t) − g (t) + A,
3 3 9
2
t t
ϕ(t) = − g (4) (t) + A.
3 3
Comme ϕ(0) = ϕ(α) = 0 le théorème de Rolle nous assure l’existence de α 1 ∈ ]0, α[ tel que
ϕ (α 1) = 0.
De même, l’égalité ϕ (0) = ϕ (α1) = 0 entraı̂ne l’existence de α2 ∈ ]0, α 1 [ tel que ϕ (α2 ) = 0.
Comme g est impaire, g est impaire donc g (0) = 0, donc ϕ(0) = ϕ (α 2 ) = 0, de sorte
qu’il existe α3 ∈ ]0, α 2[ tel que ϕ (α3 ) = 0.
Finalement, on a trouvé α3 ∈ ]0, α[ tel que
1 (4) g (4) (α3) − g (4) (0)
A= g (α 3) =
α3 α3 − 0
(on a g (4)(0) = 0 car g est impaire), et d’après le théorème des accroissements finis appliqué
à g (4) , on en déduit l’existence de θ ∈ ]0, α3 [ ⊂ ]0, α[ tel que A = g(5) (θ ). Le résultat est ainsi
prouvé car ϕ(α) = 0.
b) On pose α = b−2 a et g : [−α, α] → R x → f (x + a+ b a+b
2 ) − f (−x + 2 ). Ainsi définie, g est
impaire, et après avoir appliqué à g le résultat de la question précédente, on obtient
    
b−a b−a    a+b
∃θ ∈ 0, , f(b) − f (a) = f (a) + f (b) + 4f
2 6 2
5
    
(b − a) (5) a+b (5) a+b
− f θ+ +f −θ + . (∗)
180 · 25 2 2
L’application f étant de classe C 5, f (5) est continue donc vérifie la propriété des valeurs in-
termédiaires, donc f (5)(]a, b[) est un intervalle. Comme
   
(5) a+b (5) (5) a+b
f θ+ ∈ f (]a, b[) et f −θ + ∈ f (5) (]a, b[),
2 2
on en déduit     
1 (5) a+b (5) a+b
f θ+ +f −θ + ∈ f (5) (]a, b[),
2 2 2
donc     
 1 (5) a+b a+b
∃θ ∈ ]a, b[, f θ+ +f (5)
−θ + = f (5) (θ ),
2 2 2
d’où le résultat avec (*).
c) En appliquant le résultat de la question précédente à une primitive F de f sur [x 2i−2, x 2i]
pour tout i ∈ {1, . . . , n}, on obtient l’existence de θi ∈ ]x2i−2 , x2i [ tel que
 x2i
b−a (x2i − x 2i−2 )5 (5)
f (t) dt − (f (x 2i−2 ) + 4f (x 2i−1) + f (x2i )) = f (θi),
x2i−2 6n 2880
donc  
 x 2i b−a  (b − a)5
 
 f (t) dt − (f (x2i−2) + 4f (x2i−1) + f (x2i )) ≤ M.
 x2i−2 6n  2880n 5
Il ne reste plus qu’à sommer cette relation sur i et à utiliser l’inégalité triangulaire pour en
déduire le résultat.
Remarque. Le résultat de la question b) reste vrai si f est seulement supposée 5 fois
dérivable. En effet, le caractère continue de f (5) a été utilisé pour montrer que f (5) vérifie
la propriété des valeurs intermédiaires et on sait d’après le théorème de Darboux (voir
l’exercice 4) que ceci est vérifié pour toute fonction dérivée.
1. FONCTIONS DÉRIVABLES 83

Exercice 6. Soient m ≥ 2 un entier, f : R → R une application de classe C m et n ∈ N∗ ,


n < m. On suppose qu’il existe un entier k , n < k ≤ m, tel que f (k)(0) = 0. D’après
l’égalité de Taylor-Lagrange,
x n−1 (n−1) xn
∀x ∈ R, ∃θx ∈ ]0, 1[, f (x) = f (0) + xf (0) + · · · + f (0) + f (n)(θx x).
(n − 1)! n!
Montrer l’existence et donner la valeur de lim x→0 θx .
x=0

Solution. Les hypothèses vérifiées par f entraı̂nent l’existence d’un plus petit entier p > n tel
que f (p) (0) = 0. La formule de Taylor-Young appliquée à f à l’ordre p donne, lorsque x → 0,
xn (n) xp (p)
f (x) = f (0) + xf  (0) + · · · + f (0) + f (0) + o(xp),
n! p!
donc
xn (n) x n (n) xp
f (θ xx) = f (0) + f (p) (0) + o(xp ),
n! n! p!
donc lorsque x → 0, x =
 0,
n! (p) n!
f (n) (θx x) − f (n)(0) = x p−n f (0) + o(xp−n ) ∼ x p−n f (p)(0). (∗)
p! p!
L’égalité de Taylor-Young appliquée à f (n) donne, lorsque x → 0, x = 0,
(θx x)p−n (p) (θ x) p−n (p)
f (n) (θx x) − f (n) (0) = f (0) + o(xp−n ) ∼ x f (0).
(p − n)! (p − n)!
Avec (*), on en déduit
 1/(p−n)  −1/(p−n)
(θx x)p−n x p−n n! (p − n)!n!
∼ donc lim θx = = C np .
(p − n)! x→0
x=0
p! x→0
x=0
p!

 Exercice 7. Soient a, b ∈ R, a < b, et f : [a, b] → R une fonction de classe C n avec


n ≥ 2. On suppose qu’il existe des réels (xi )1≤i≤n , a ≤ x1 < x2 < · · · < xn ≤ b, tels que
f (xi ) = 0 pour tout i ∈ {1, . . . , n}.
a) Montrer
n
f(n) (u) 
∀x ∈ [a, b], ∃u ∈ [a, b], f (x) = (x − xi ).
n! i=1

b) Soit M = supt∈[a,b] |f (n)(t)|. Montrer que


n
M
∀x ∈ [a, b], |f (x)| ≤ |x − xi |.
n! i=1

c) Montrer que
M 
∀i ∈ {1, . . . , n}, |f (xi )| ≤ |xi − xj |.
n! j =i

d) Plus généralement, montrer que


n
 
M  
∀x ∈ [a, b], |f  (x)| ≤ |x − xj | .
n! i=1 j =i

Que dire si n = 2, a = x1 et b = x 2 ?
84 2. FONCTIONS D’UNE VARIABLE RÉELLE

Solution. a) S’il existe i tel que x = x i, c’est terminé. Sinon, on considère l’application
n

ϕx : [a, b] → R t → f (t) − A (t − xi ),
i=1
où la constante A est choisie telle que ϕx(x) = 0. Cette application est de classe C n et s’annule
(au moins) en n + 1 points distincts (x et les (xi )1≤i≤n ). En appliquant le théorème de Rolle, on
en déduit que ϕx s’annule en n points distincts. En l’appliquant ensuite à ϕ x, on en déduit en
(n)
ϕx s’annule en n − 1 points distincts. En poursuivant ainsi, on s’aperçoit que ϕx s’annule en
(n)
au moins un point u. On en déduit ϕx (u) = 0 = f (n) (u) − An!, donc A = f (n) (u)/n!, et comme
ϕx (x) = 0, on en déduit le résultat.
b) C’est une conséquence immédiate du résultat de la question précédente.
c) D’après la question précédente, pour tout x = x i on a
   
 f (x) − f (xi)   f (x)  M 
 = 
 x − xi  x − x i  ≤ n! |x − xj |,
j =i

d’où le résultat en faisant tendre x vers xi.


d) S’il existe i tel que x = x i, c’est terminé d’après la question précédente. Sinon, nous allons
construire une application qui s’annule en n points dont x et lui appliquer le résultat de la
question c). Posons
 t − xi 
n−1
ψx : [a, b] → R t → f (t) − f (x) .
i=1
x − xi
Cette application s’annule aux n points x 1, . . . , xn−1 et x, donc en lui appliquant c) au point x,
on en déduit
n−1
M 
|ψ x(x)| ≤ |x − x i |
n! i=1
(n) (n)
(car ψx = f (n) , donc |ψx (x)| ≤ M pour tout x). Ceci s’écrit aussi
  
 
 n−1
  
  f (x)  M 
f (x) −  n−1  (x − xj )  ≤ |x − xi |
 i=1 (x − x i) i=1
 n!
 j =i
j =n
 i=n

ce qui entraı̂ne
  
n−1 n−1
 |f (x)|     M 
|f (x)| ≤ n−1   |x − xj|  + |x − xi |
i=1 |x − xi| i=1 j =i
n! i=1
j =n
   
n−1
  n−1
 n 
M   M M 
≤ |x − xn |  |x − xj |  + |x − x i | = |x − xj | .
n! i=1
n! i=1
n! i=1
j =i j =i
j =n

Lorsque n = 2, x1 = a et x2 = b, ceci s’écrit


M b−a
|f (x)| ≤ [(x − a) + (b − x)] = M .
2 2

Exercice 8. Soient n ≥ 2 un entier et f : R → R une application de classe Cn. Pour


tout entier k, 0 ≤ k ≤ n, on note Mk = supt∈R |f (k)(t)| ∈ R + ∪ {+∞}. On suppose que
M0 et M n ont des valeurs finies.
a) Montrer que pour tout entier k, 0 < k < n, Mk a une valeur finie.
1. FONCTIONS DÉRIVABLES 85

b) Montrer que si limt→+∞ f (t) = limt→+∞ f (n)(t) = 0, alors lim t→+∞ f (k)(t) = 0 pour
tout entier k tel que 0 < k < n.
c) Montrer que pour tout entier m, 1 ≤ m ≤ n, et pour tout entier k, 0 ≤ k ≤ m,

M k ≤ 2 k(m−k )/2 M01−k/m Mm


k/m
.

(Indication. On pourra commencer par montrer que M1 ≤ 2M0 M2 .)

Solution. a) Soit x ∈ R. Pour tout i ∈ {1, . . . , n − 1}, il existe d’après le théorème de Taylor-
Lagrange θi,x ∈ ]0, i[ tel que
n−1
 f (k) (x) f (n)
(x + θ i,x)
f (x + i) = f (x) + ik + in ,
k! n!
k=1

ce qui, en posant
in (n) f (k)(x)
Xi (x) = f (x + i) − f (x) − f (x + θi,x ) et Y k (x) =
n! k!

s’écrit Xi (x) = n−1 k
k=1 i Y k (x). En notant X (x) (resp. Y (x)) le vecteur colonne de R
n−1 dont les

composantes sont les Xi (x) (resp. Yi(x)), ceci s’écrit matriciellement X (x) = M Y (x) avec
 
1 1 ··· 1
 2 22 ··· 2n−1 
 
 3 3 2 · · · 3n−1 
M = .
 .. .. .. 
 . . . 
(n − 1) (n − 1)2 · · · (n − 1) n−1

On a det(M ) = Vandermonde(1, 2, . . . , n − 1) =
 0, donc M est inversible. L’application linéaire
Rn−1 → Rn−1 X → M −1X est continue (on est en dimension finie) donc il existe A > 0 tel
que M −1 X  ≤ AX  pour tout X ∈ R n−1 . Les hypothèses sur f entraı̂nent que x → X (x) est
bornée, et comme Y (x) = M −1 X (x) ≤ AX (x), on en déduit que x → Y (x) est bornée,
ce qu’il fallait démontrer.
b) Comme limx→+∞ X (x) = 0, et que Y (x) ≤ AX (x), on en déduit limx→+∞ Y (x) = 0.
c) Comme indiqué, nous commençons par montrer

M 1 ≤ 2M0M 2 . (∗∗)
Soit x ∈ R. Pour tout h > 0,

 2
 ∃θ1 ∈ ]0, 1[ f (x + h) = f (x) + hf  (x) + h f  (x + θ1 h)

2
 2
 h
 ∃θ2 ∈ ]0, 1[ f (x − h) = f (x) − hf  (x) + f  (x − θ2 h)
2
Ceci entraı̂ne
f (x + h) − f (x − h) h 
f  (x) = + [f (x − θ 2 h) − f (x + θ1 h)],
2h 4
donc
2M 0 h M0 h
|f  (x)| ≤ + 2M 2 = + M2 .
2h 4 h 2
Ceci est vrai pour tout h > 0. On va donc choisir h tel que Mh0 + h2 M2 soit minimal. Une rapide
étude de la fonction ψ : R+ → R, h → Mh0 + h2 M2 montre qu’elle atteint son minimum pour
 √
h = 2M M2 , point en lequel elle vaut
0
2M0 M2. Cette majoration étant vraie pour tout x, on en
déduit (**).
86 2. FONCTIONS D’UNE VARIABLE RÉELLE

Montrons maintenant le résultat demandé. Nous procédons par récurrence sur m. Si m = 1,


c’est évident, et le cas m = 2 est une conséquence de (**). Supposons le résultat vrai jus-
qu’au rang m et montrons le au rang m + 1. En appliquant (**) à f(m−1) , on obtient Mm ≤
√ 1/m (m−1)/m
2Mm−1 Mm+1 . Or M m−1 ≤ 2(m−1)/2M0 Mm , donc

M 2m ≤ 2Mm−1M m+1 ≤ 2 (m+1)/2 M01/m Mm


(m−1)/m
Mm+1 .
Si Mm = 0, alors f est polynomiale de degré < m et la propriété est trivialement vérifiée au
rang m + 1. Sinon la dernière inégalité entraı̂ne
1+1/m 1/m
Mm ≤ 2(m+1)/2 M0 Mm+1 . (∗∗∗)
Soit k, 0 ≤ k ≤ m + 1. Si k = m + 1, le résultat est évident, sinon d’après l’hypothèse de
récurrence,
1−k/m
M k ≤ 2 k(m−k )/2M0 k/m
Mm ,
et en élevant (***) à la puissance k/ (m + 1) puis en remplaçant dans cette dernière inégalité,
on obtient
Mk ≤ 2k(m+1−k )/2 M 01−k/(m+1)M m+1
k/m+1
,
d’où le résultat.

Exercice 9 (Une fonction continue, nulle part d érivable). On note ∆ la


fonction définie sur R, 1-périodique, dont la restriction à [−1/2, 1/2] vérifie ∆(x) = |x|.
Montrer que la fonction
+∞
 1
f : R → R x → p
∆(2p x)
p=0
2

est continue mais n’est dérivable en aucun point de R.


 1 p
Solution. Pour tout x ∈ R, on a |∆(x)| ≤ 1/2 donc la série de fonctions 2 p ∆(2 x) converge
normalement sur R. Ainsi, f est bien définie sur R, et comme ∆ est continue, f est aussi continue.
Montrons maintenant que f n’est dérivable en aucun point de R. Comme f est 1-périodique,
il suffit de montrer que f n’est dérivable
+∞ ε k en aucun point de [0, 1[. Soit x0 ∈ [0, 1[. On∗considère
l’écriture diadique de x0 : x 0 = k=1 2k , où εk ∈ {0, 1} pour tout k. Pour tout n ∈ N , on pose
n
 εk 1
x n = et x n = x n + .
2k 2n
k=1

Les suites (xn ) et (xn ) tendent vers x0 . Lorsque p ≥ n, les nombres 2p x n et 2pxn sont des entiers,
donc ∆(2p x n) = ∆(2 pxn) = 0. Maintenant, si p < n, on a
n
 p

εk
2 p xn =N+ où N = 2 p−k ε k est un entier,
2k−p
k=p+1 k=1

donc
   
n
 n

εk  εk 1 
∆(2p xn ) = ∆  , et de même ∆(2p xn ) = ∆  + . (∗)
2k−p 2k−p 2n−p
k=p+1 k=p+1

Si ε p+1 = 0, l’encadrement
n
 n

εk 1 1 1 1
0≤ + ≤ + =
2 k−p 2 n−p 2k−p 2n−p 2
k=p+1 k=p+2
2. DÉVELOPPEMENTS LIMIT ÉS ET D ÉVELOPPEMENTS ASYMPTOTIQUES 87

montre que dans (*), les valeurs de ∆ sont prises sur un intervalle où ∆ croit, et étant donnée
la forme de ∆, on en déduit finalement
1
si ε p+1 = 0, ∆(2p xn ) − ∆(2p xn) = 2p (x n − xn ) = n−p.
2
On montrerait de même
1
si εp+1 = 1, ∆(2p xn ) − ∆(2 pxn) = − n−p .
2
En résumé, on a ∆(2p xn) − ∆(2 pxn ) = (−1)ε p+1 /2 n−p pour 0 ≤ p < n, donc finalement
n−1
 n−1
(−1)ε p+1 f (xn ) − f (xn ) 
f (xn ) − f (xn ) = ou encore = (−1) εp+1 ,
p=0
2n xn − x n p=0
f (x  
n)−f (x n)
ce qui montre que la suite (yn ) définie par y n = x  ne converge pas. Si maintenant f
n −x n
est dérivable en x0, on a
f (xn) − f (x 0) = (x n − x 0 )[f (x0) + εn ] et f (xn ) − f (x0) = (x n − x 0)[f  (x0) + ε n]
où les suites (εn ) et (ε n ) tendent vers 0. Par différence, on a
f (xn ) − f (xn) = (xn − xn)f  (x0) + (x 0 − xn)εn + (xn − x 0 )εn,
et comme x n ≤ x0 ≤ xn , ceci entraı̂ne
|f (xn ) − f (x n) − (xn − x n)f (x0 )| ≤ (x n − x n)(εn + εn ) donc |yn − f (x 0)| ≤ εn + ε n,
donc (yn ) converge vers f (x0 ), ce qui est contradictoire. Donc f n’est pas dérivable en x0 , d’où
le résultat.
Remarque. Une méthode non constructive de la preuve de l’existence d’une fonction
continue jamais dérivable est donnée à l’exercice 4 page 421.

2. Développements limités et développements


asymptotiques
Dans toute cette partie, E désigne un R-e.v.n.
2.1. Relations de comparaison
Soit X un espace métrique. On considère deux applications f, g de D ⊂ X dans E et
x0 un point d’accumulation de D.
— On dit que f est dominée par g au voisinage de x0 si
∃C > 0, ∃V ∈ V(x0 ), ∀x ∈ V ∩ D, f (x) ≤ C g (x)
( où V (x0 ) désigne l’ensemble des voisinages de x0 ). On note alors f (x) = O (g (x))
x→x 0
(lire “grand o”, notation de Landau) ou encore f (x)  g (x) (notation de Hardy).
x→x0
— On dit que f est négligeable devant g au voisinage de x0 si
∀ε > 0, ∃V ∈ V(x 0 ), ∀x ∈ V ∩ D, f (x) ≤ εg (x).
On note alors f (x) = o(g (x)) (lire “petit o”, notation de Landau) ou encore
x→x0
f (x) ≺≺ g (x) (notation de Hardy).
x→x 0
— On dit que f et g sont équivalentes au voisinage de x 0 si f (x) − g (x) = o(g (x)),
x→x 0
et on écrit alors f (x) ∼ g (x).
x→x0
88 2. FONCTIONS D’UNE VARIABLE RÉELLE

Remarque 8. — Il faut prendre garde au fait que les notations de Landau sont des
abus. Lorsqu’on écrit f (x) = O (g (x)) par exemple, il n’y a pas à proprement parler
d’égalité (si f1(x) = O (g (x)) et f2 (x) = O (g (x)), on ne peut pas dire f1(x) = f 2(x))
— une notation correcte serait f (x) ∈ O (g (x)). Cependant, l’usage est bien établi
et cette notation est très commode, c’est pourquoi on l’utilise beaucoup.
— Dans la pratique, on utilisera souvent cette notation pour des fonctions R → C,
au voisinage d’un point de R ou de l’infini, ou pour des suites réelles ou complexes
(un) lorsque n → +∞.
— Attention ! La relation ∼ se manie avec précaution. En particulier, elle n’est pas
compatible avec l’addition, par exemple
x+2 ∼ x + 1, −x ∼ −x et pourtant 2 ∼ 1.
x→+∞ x→+∞ x→+∞

Par contre, l’équivalence est compatible avec le produit et la puissance (si f1 ∼ f2


et g1 ∼ g 2, f 1g1 ∼ g 1g2 ; si f ∼ g , f α ∼ g α pour tout α ∈ R).
2.2. Développements asymptotiques
Échelle de comparaison.
Définition 3. Soit X un espace métrique et x 0 ∈ X. On appelle échelle de comparaison
un ensemble E de fonctions définies au voisinage de x0 dans X sauf éventuellement en
x0 , et vérifiant la propriété suivante : si f, g ∈ E , alors f = g ou bien f = o(g ) ou bien
g = o(f ).
Exemple 1. Au voisinage de +∞ pour les fonctions de la variable réelle, les échelles de
comparaison les plus courantes sont les suivantes :
— celles constituées des fonctions du type xα (α ∈ R),
— plus généralement celles constituées des fonctions du type xα(log x) β (α, β ∈ R)
γ
— plus généralement celles constituées des fonctions du type xα (log x) β ecx (α, β, c ∈
R, γ > 0).
De même, au voisinage de 0, une échelle de comparaison courante est celle contenant les
fonctions du type xα log β x (α, β ∈ R).
Développements asymptotiques.
Définition 4. Soit X un espace métrique. Soient f : D ⊂ X → E une application,
x0 un point d’accumulation de D dans X et k un entier naturel non nul. On appelle
développement asymptotique à k termes de f par rapport à une échelle de comparaison E
au voisinage de x0 toute expression de la forme
c1 f1 + c2 f2 + · · · + ck fk
vérifiant
(i) c1 , . . . , ck ∈ E sont des constantes multiplicatives,
(ii) f1 , . . . , fk ∈ E avec pour tout i, fi+1 (x) = o(f i(x)),
x→x0
(iii) f (x) = c1 f 1 (x) + c2 f2(x) + · · · + c kfk (x) + o(fk (x)).
x→x0
Lorsque qu’un tel développement par rapport à E existe pour f , il est unique. On a en
particulier f (x) ∼ c1f1 (x), et on dit que c1 f1 est la partie principale (ou l’équivalent) de
f au voisinage de x0.
L’unicité découle facilement des propriétés vérifiées par une échelle de comparaison.
Remarque 9. Lorsque l’on ne précise pas l’échelle de comparaison par rapport à laquelle
on veut un développement asymptotique, c’est en général une des échelles usuelles décrites
dans l’exemple 1.
2. DÉVELOPPEMENTS LIMIT ÉS ET D ÉVELOPPEMENTS ASYMPTOTIQUES 89

Exemple 2. On peut démontrer, pour tout k ∈ N∗ , le développement asymptotique sui-


vant, lorsque x → +∞ :
x  
dt x 1! x (k − 1)! x x
= + + ··· + +o .
2 log t log x log 2 x log k x log k x
(voir l’exercice 8 page 173).
2.3. Développements limités
Dans la suite de cette partie, I désigne un intervalle de R non réduit à un singleton.
Définition 5. Soit f : I → E une application, et supposons 0 ∈ I . Si n ∈ N∗ , on dit que
f admet un développement limité d’ordre n au voisinage de 0 s’il existe a 0, a1 , . . . , an ∈ E
tels que, au voisinage de 0,
f (x) = a0 + a1 x + · · · + a n xn + o(x n ).
Remarque 10. — Une définition équivalente est f (x) = Pn (x) + o(xn) où Pn est une
fonction polynomiale de degré ≤ n.
— On pourrait de même définir les développements limités au voisinage d’un point
quelconque a de I (en développant avec des termes de la forme α (x − a)k — ou
1/xk, k ∈ N, lorsque a = ±∞). Nous avons choisi de nous limiter à a = 0 pour
alléger les notations.
— Un développement limité est aussi un développement asymptotique par rapport à
l’échelle de comparaison constituée des fonctions de la forme xn (n ∈ N).
Il découle de l’unicité d’un développement asymptotique le résultat suivant.
Proposition 7. Si f admet un développement limité d’ordre n ∈ N∗ , il est unique.
Proposition 8. Si f admet un développement limité d’ordre n ≥ 1 au voisinage de 0
f (x) = a0 + a1 x + · · · + a n xn + o(x n ),
alors f (0) = a 0, f est dérivable en 0 et f  (0) = a1 .
Remarque 11. Attention ! Même lorsque n ≥ 2, l’existence d’un développement limité
d’ordre n n’assure pas l’existence de f  (0). Par exemple, la fonction f définie par
 
2 3 1
f (x) = 1 + x + x + x sin si x = 0, f (0) = 1,
x2
n’est pas deux fois dérivable en 0, et pourtant elle vérifie f (x) = 1 + x + x2 + o(x2 ) au
voisinage de 0.
Proposition 9. Soit a > 0 et f : ] − a, a[ → E une application admettant au voisinage
de 0 un développement limité d’ordre n ∈ N∗ :
f (x) = a0 + a1 x + · · · + a n xn + o(x n ).
— Si f est paire, tous les termes ak d’indices k impairs sont nuls.
— Si f est impaire, tous les termes ak d’indices k pairs sont nuls.
Condition suffisante de l’existence d’un développement limité. La formule de
Taylor-Young entraı̂ne immédiatement le résultat suivant.
 Proposition 10. Si f : I → E est une application n fois dérivable en 0, alors f admet
au voisinage de 0 le développement limité d’ordre n suivant :
f (n)(0) n
f (x) = f (0) + f (0) x + · · · + x + o(xn ).
n!
90 2. FONCTIONS D’UNE VARIABLE RÉELLE

2.4. Opérations sur les développements limités


Intégration terme à terme.
Proposition 11. Soit f : I → E (avec 0 ∈ I ) une application dérivable sur I telle qu’au
voisinage de 0
f (x) = a0 + a1 x + · · · + a n xn + o(xn ).
Alors l’application f admet au voisinage de 0 le développement limité d’ordre n +1 suivant
a1 2 an n+1
f (x) = f (0) + a 0 x + x + ··· + x + o(x n+1 ).
2 n+1
Dérivation d’un développement de Taylor.
Proposition 12. Soit f : I → E (avec 0 ∈ I ) une application n ≥ 2 fois dérivable en
0. Si au voisinage de 0, on a
f (x) = a0 + a1 x + · · · + a n xn + o(x n ),
alors au voisinage de 0,
f  (x) = a 1 + 2a 2 x + · · · + na nxn−1 + o(xn−1 ).
Remarque 12. Sans l’hypothèse d’existence de f (n)(0), le résultat est faux. Considérez par
exemple l’application f introduite à la remarque 11 : f admet un développement limité
d’ordre 2 et pourtant, f (x) qui existe sur un voisinage de 0, n’admet pas de développement
limité au voisinage de 0 (f  n’a pas de limite en 0).
Somme, produit, quotient de développements limités.
Proposition 13. Soient f et g : I → E (avec 0 ∈ I ) deux applications admettant au
voisinage de 0 un développement limité d’ordre n :
f (x) = Pn (x) + o(xn ), g(x) = Qn(x) + o(x n),
où Pn et Q n sont deux fonctions polynomiales de degré ≤ n. Alors
— la somme f + g admet un développement limité d’ordre n donné par (f + g)(x) =
(Pn + Q n )(x) + o(xn ),
— Si E est une algèbre normée, le produit f g admet un développement limité d’ordre
n donné par (f g)(x) = Rn (x) + o(x n), où Rn est le reste de la division euclidienne
de PnQ n par X n+1 : P n Qn = R n + Xn+1S n avec deg(R n) ≤ n,
— Si E = R ou E = C, et si g (0) = Q(0) = 0, le quotient f /g admet un développement
limité d’ordre n donné par (f /g)(x) = Rn (x) + o(xn), où Rn est le quotient de la
division selon les puissances croissantes de Pn par Q n à l’ordre n.
Développement limité d’une fonction composée.
Proposition 14. Soient g : I → R (avec 0 ∈ I ) une application admettant un
développement limité d’ordre n au voisinage de 0, et f : J → E ( où J est un inter-
valle de R tel que g (I ) ⊂ J ) une application admettant un développement limité d’ordre
n au voisinage de g0 = g(0). On écrit, au voisinage de 0,
g (x) = g 0 + Pn (x) + o(x n ) et f (g0 + t) = Q n(t) + o(tn ),
où Pn et Qn sont deux polynômes de degré ≤ n avec Pn(0) = 0. Alors la fonction composée
f ◦ g admet au voisinage de 0 un développement limité d’ordre n : f ◦ g(x) = Rn (x)+o(xn ),
où Rn est le reste de la division euclidienne de P n ◦ Qn par X n+1 (P n ◦ Q n = R n + X n+1 Sn ,
deg(Rn ) ≤ n).
2. DÉVELOPPEMENTS LIMIT ÉS ET D ÉVELOPPEMENTS ASYMPTOTIQUES 91

2.5. Développements limités usuels au voisinage de 0


La formule de Taylor-Young permet d’obtenir facilement les développements limités
suivants, lorsque x → 0.

x2 xn
ex = 1 + x + + ··· + + o(x n)
2! n!
x 3 x5 p x
2p+1
sin x = x− + + · · · + (−1) + o(x2p+2)
3! 5! (2p + 1)!
2 4
x x x2p
cos x = 1− + + · · · + (−1)p + o(x 2p+1)
2! 4! (2p)!
x 3 x5 x2p+1
sinh x = x+ + + ··· + + o(x2p+2)
3! 5! (2p + 1)!
x2 x4 x2p
cosh x = 1+ + + ··· + + o(x2p+1)
2! 4! (2p)!
α α(α − 1) 2 α(α − 1) · · · (α − n + 1) n
∀α ∈ R, (1 + x)α = 1+ x+ x + ··· + x + o(xn )
1! 2! n!
En particulier
1
= 1 − x + x 2 − · · · + (−1)n xn + o(xn )
1+x
√ x 1 1·3 3 1 · 3 · · · (2n − 3) n
1+x = 1+ − x2 + x + · · · + (−1)n−1 x + o(xn)
2 2·4 2·4·6 2 · 4 · · · (2n)
√ x 1 1·3 3 1 · 3 · · · (2n − 3) n
1−x = 1− − x2 − x − ···− x + o(xn)
2 2·4 2·4·6 2 · 4 · · · (2n)
1 1 1·3 2 1 · 3 · · · (2n − 1) n
√ = 1− x+ x + · · · + (−1)n x + o(x n)
1+x 2 2·4 2 · 4 · · · (2n)
1 1 1·3 2 1 · 3 · · · (2n − 1) n
√ = 1+ x+ x + ··· + x + o(xn)
1−x 2 2·4 2 · 4 · · · (2n)
1 1 1 1
En intégrant respectivement les développements limités de 1+x , 1+x2
, 1−x 2
, √1−x 2,

√ 1 (qui sont connus grâce aux formules précédentes), on obtient


1+x 2

x2 x 3 n−1 x
n
log(1 + x) = x − + + · · · + (−1) + o(x n)
2 3 n
x3 x 5 x 2n+1
arctan x = x − + + · · · + (−1)n + o(x2n+2 )
3 5 2n + 1
x3 x 5 x2n+1
argth x = x + + + ··· + + o(x2n+2 )
3 5 2n + 1
1 3 1·3 5 1 · 3 · · · (2n − 1)
arcsin x = x + x + x + ··· + x2n+1 + o(x 2n+2)
2·3 2·4·5 2 · 4 · · · (2n) · (2n + 1)
1 3 1·3 5 (−1)n · 1 · 3 · · · (2n − 1) 2n+1
argsh x = x − x + x + ··· + x + o(x 2n+2)
2·3 2·4·5 2 · 4 · · · (2n) · (2n + 1)
Enfin, le développement limité de la fonction x → tan x peut être effectué grâce à la
proposition 13. Les calculs sont un peu lourds dès que l’on veut dépasser l’ordre 4, c’est
pourquoi il est bon de connaı̂tre ses premiers termes :
x3 2 5 17 7
tan x = x + + x + x + o(x 8).
3 15 315
92 2. FONCTIONS D’UNE VARIABLE RÉELLE

Remarque 13. Celui de tan x mis à part, il n’est pas nécessaire d’apprendre par cœur ces
développement limités. Il faut par contre savoir les retrouver rapidement.
2.6. Formes indéterminées
Beaucoup d’exercices exigent de connaı̂tre les développements limités pour trouver la
limite d’une forme indéterminée. Par exemple, nous allons montrer l’existence et donner
tan x − x
la valeur de lim
x→0 sin x − x
. Les développements limités usuels donnent
x=0

x3 x3 x3 x3
+ o(x 3) ∼ −
sin x − x = − et tan x − x = + o(x3 ) ∼ ,
6 6 3 3
tan x − x
et on en déduit lim
x→0 sin x − x
= −2.
x=0

2.7. Exercices
Exercice 1. Donner le développement limité au voisinage de 0 à l’ordre 4 des fonctions
suivantes  1/2  
√ sin x
a) x → 1 + 1 + x 2 b) x → log
x
1 1
c) x → (1 + 2x)1/(1+x) d) x → 2

  x sinπx sinh2 x
 
e) x → esinh x − log tan + .
2 4

√ 2
Solution. a) Le développement limité 1 + u = 1 + u2 − u8 + o(u2 ) entraı̂ne
  1/2  
x2 x 4
1/2


x2 x 4
1/2
2 4 4
1+ 1+x = 1+ 1+ − + o( x ) = 2 1+ − + o( x ) ,
2 8 4 16
ce qui, par composition des développements limités est égal à
    2   
√ 1 x2 x4 1 x2 4
√ 1 2 5 4 4
2 1+ − − + o( x ) = 2 1 + x − x + o( x ) .
2 4 16 8 4 8 158

b)
       2
sin x x2 x4 4 x2 x4 1 x2
log = log 1 − + + o( x ) = − + − + o(x4 )
x 6 120 6 120 2 6
x2 x4
=− − + o(x 4).
6 180
c) En procédant de même, on trouve
 
1/(1+x) log(1 + 2x) 10 4
(1 + 2x) = exp = 1 + 2x − 2x2 + x + o(x 4 ).
1+x 3

d) Cette fois, on commence par développer sin x et sinh x à l’ordre 7, ce qui entraı̂ne
 −2   −2 
1 1 1 x2 x4 x6 6 x2 x4 x6 6
− = 2 1− + − + o( x ) − 1+ + + + o( x ) .
sin 2 x sinh2 x x 6 120 7! 6 120 7!
En utilisant le développement limité (1 + u) −2 = 1 − 2u + 3u 2 − 4u3 + 5u 4 + o(u4) on trouve,
après calculs
1 1 2 4 4
2
− 2
= + x + o(x 4 ).
sin x sinh x 3 189
2. DÉVELOPPEMENTS LIMIT ÉS ET D ÉVELOPPEMENTS ASYMPTOTIQUES 93

 
e) On commence par calculer le développement de ϕ(x) = log  tan(x2 + π4 ) . Le moyen le plus
simple est certainement de remarquer que
1 1 x2
ϕ  (x) = = 2 = 1 + + o(x3 )
cos x 1 − x2 + o(x3 ) 2
puis d’intégrer ce développement limité, ce qui donne
x3 x3
ϕ(x) = ϕ(0) + x + + o(x4 ) = x + + o(x 4 ).
6 6
Maintenant, après calculs
x2 x3 5
esinh x = 1 + x + + + x4 + o(x 4 ),
2 3 24
et finalement
  x π 
sinh x   x2 x3 5
e − log tan +  =1+ + + x4 + o(x 4).
2 4 2 6 24

Exercice 2. Calculer le développement limité


1
a) à l’ordre 4, au voisinage de 1, de l’application x → x −1+log x ,
b) à l’ordre 4, au voisinage de +∞, de l’application x → (x3 + x)1/3 − (x3 − x) 1/3,
c) à l’ordre 3, au voisinage de π/6, de x → log(2 sin x).

Solution. a) Posons t = x − 1. La relation


   
1 log(1 + t) 1
x −1+log x = exp = exp 1 + ,
−1 + log(1 + t) −1 + log(1 + t)
permet, à partir du développement limité de log(1 + t) et par composition de développements
limités, de montrer
1 (x − 1)4
x −1+log x = 1 − (x − 1) + + o[(x − 1)4 ].
12
b) On écrit  1/3   
3 1/3 3 1/3 1 1 1/3
(x + x) − (x − x) =x 1+ 2 − 1− 2 ,
x x
et le développement limité de (1 ± x−2 ) 1/3 en 1/x donne, après calculs
 
3 1/3 3 1/3 2 1 1
(x + x) − (x − x) = +o .
3 x x4

c) On commence par écrire


π  √
π π 1 3
sin + t = sin cos t + sin t cos = sin t + cos t,
6 6 6 2 2
puis on calcule le développement limité de sin t et cos t en t = 0, et après composition de
développements limités, on trouve
√   
√  π  π 2 3 π 3 π 3
log(2 sin x) = 3 x − −2 x− − x− +o x− .
6 6 3 6 6

Exercice 3. Donner la limite, lorsque x tend vers 0+ , des expressions suivantes


x
esin x − e tan x xx log x
a) b)
sin x − tan x xx − 1
94 2. FONCTIONS D’UNE VARIABLE RÉELLE

(1 + x)log x/x − x 2 arccos(1 − x)


c) d) (cos x) cot x e) √ .
x (xx − 1) x

Solution. Tout le problème, dans ce type d’exercices, est de sentir à l’avance jusqu’à quel ordre
on va devoir développer pour obtenir le résultat.
a) On développe jusqu’à l’ordre 3
x3 x3 x3 x3
esin x − etan x = − + o(x3 ) ∼ − , sin x − tan x = − + o( x 3 ) ∼ − ,
2 2 2 2
donc l’expression proposée est équivalente à 1, donc a pour limite 1 lorsque x → 0+ .
b) Comme x log x = o(1), on a
x
x x = xexp(x log x) = x 1+x log x+o(x log x) = x · exp (log x(x log x + o(x log x))) ∼ x
et
xx − 1 = ex log x − 1 ∼ x log x,
et on en déduit facilement que la limite recherchée est 1.
c) On écrit
   
log x/x log x x2 2 log x
(1 + x) = exp log(1 + x) = exp (x − + o(x ))
x 2 x
 x   x 
= exp log x − log x + o(x log x) = x · exp − log x + o(x log x)
2 2
 x  x2
= x · 1 − log x + o(x log x) = x − log x + o(x 2 log x)
2 2
ce qui montre que le numérateur de l’expression est équivalent à −x 2 log x/2 lorsque x → 0 +.
Le dénominateur vérifie
x(x x − 1) = x(ex log x − 1) ∼ x · x log x = x2 log x,
et finalement, la limite recherchée est égale à −1/2.
d) Lorsque u → 0+ , on a
 
1 1 1 1 1 1
cot u = = = · = +o
tan u u + o(u) u 1 + o(1) u u
donc
    
cot x2 1 1 x2 2
(cos x) = exp +o log 1 − + o( x )
x2 x2 2
    2   
1 1 x 2 1
= exp +o − + o( x ) = exp − + o(1) ,
x2 x2 2 2

ce qui montre que la limite recherchée est 1/ e.
e) Il y a une astuce à connaı̂tre.
 Posons y = arccos(1 −x). On a 1−x = cos y donc x = 1−cos y =
2
2 sin (y/2), d’où sin y/2 = x/2 et finalement y = 2 arcsin x/2. Cette relation permet de
calculer un nombre de termes quelconque du développement asymptotique de arccos(1 − x)
lorsque x → 0+ (un autre moyen de faire est de calculer un développement asymptotique de la
dérivée de arccos(1 − x) — qui a une forme suffisamment explicite — puis de l’intégrer). En
particulier,

x √
arccos(1 − x) ∼ 2 = 2 x,
2

et on en déduit que la limite recherchée est 2.
3. FONCTIONS CONVEXES, FONCTIONS RÉGL ÉES 95

Exercice 4. Donner la limite, lorsque x → +∞, des expressions suivantes :


 x    x  1/x
log(x + 1) 1
a) − 1 log x b) e− 1+ .
log x x

Solution. a) On écrit
 
log(x + 1) log x + log(1 + 1x ) 1 1
= =1+ +o ,
log x log x x log x x log x
donc
        
log(x + 1) x 1 1
− 1 log x = exp x log 1 + +o − 1 log x
log x x log x x log x
        
x 1 1 1
= exp +o − 1 log x = 1 + +o − 1 log x = 1 + o(1),
x log x log x log x log x
et la limite recherchée est 1.
b) Le développement limité
        
1 x 1 1 1 1
1+ = exp x log 1 + = exp x − 2 +o
x x x 2x x2
     
1 1 1 1
= exp 1 − +o = e 1− +o
2x x 2x x
montre que    
1 x e 1
e− 1+ = +o ,
x 2x x
donc l’expression proposée est égale à
  1/x    
e 1 1 e 1
+o = exp log +o = eo(1)
2x x x 2x x
et sa limite est donc 1 lorsque x → +∞.

3. Fonctions convexes, fonctions réglées


3.1. Fonctions convexes
Dans toute cette sous-partie, I désigne un intervalle de R non réduit à un singleton.
Définition 1. Une application f : I → R est dite convexe si
∀(a, b) ∈ I 2 , ∀λ ∈ [0, 1], f [(1 − λ)a + λb] ≤ (1 − λ)f (a) + λf (b). (∗)
Elle est dite concave si −f est convexe.
Remarque 1. — Lorsque l’inégalité (*) est stricte pour tout λ ∈ ]0, 1[ (lorsque a = b)
on dit que f est strictement convexe.
— La fonction f est convexe si et seulement si l’ensemble {(x, y ) ∈ I × R | y ≥ f (x)}
est convexe.
— L’inégalité (*) exprime le fait que tous les points du segment [(a, f (a)), (b, f (b))]
sont au dessus du graphe de f .
Proposition 1. Une application f : I → R est convexe si et seulement si pour tout
x0 ∈ I, l’application
f (x) − f (x 0)
gx0 : I {x 0} → R x → g x0 (x) =
x − x0
96 2. FONCTIONS D’UNE VARIABLE RÉELLE

y
(f )
f (b)
f (a)
x
0 a b
Figure 6. Entre a et b, les points du graphe de la fonction convexe f se
trouvent en dessous de la corde reliant les points (a, f (a)) et (b, f (b)).

est croissante.
Conséquence : Si f : I → R est convexe et si a, b, c ∈ I , avec a < b < c, on a (en
appliquant la proposition précédente à ga puis à g c) l’inégalité suivante entre les taux de
variation (voir aussi la figure ci-contre)
f (b) − f (a) f (c) − f (a) f (c) − f (b)
≤ ≤ .
b−a c−a c−b

y (f )

x
0 a b c
Figure 7. La propriété de croissance de la pente des cordes pour une
fonction convexe f .

La proposition précédente entraı̂ne aussi le résultat qui suit.



Proposition 2. Une fonction convexe f : I → R possède en tout point de I une dérivée

à droite et une dérivée à gauche. Elle est donc continue sur I (pas forcément aux bornes

de I ). De plus, les applications f g et fd sont croissantes sur I et f g(x) ≤ f d(x) pour tout

x ∈ I.
Théorème 1. Soit f : I → R une application dérivable sur I . Les assertions suivantes
sont équivalentes
(i) f est convexe.
(ii) f  est croissante.
(iii) La courbe représentative de f est au dessus de ses tangentes.
Corollaire 1. Une application f : I → R deux fois dérivable est convexe si et seulement
si f (x) ≥ 0 pour tout x ∈ I .
Proposition 3. Soit une application f : I → R convexe. Alors
 
α1 x1 + · · · + α nx n α1 f (x1 ) + · · · + αn f (xn )
∀x 1 , . . . , xn ∈ I, ∀α1 , . . . , αn > 0, f ≤ .
α 1 + · · · + αn α 1 + · · · + αn
3. FONCTIONS CONVEXES, FONCTIONS RÉGL ÉES 97

Inégalités classiques. Th éorème 1 (Inégalité arithm ético-géométrique). Soient


x1 , . . . , xn des nombres réels positifs. On a
x1 + · · · + xn
(x 1 · · · xn) 1/n ≤ .
n
Démonstration. Cela provient de la concavité de la fonction logarithme. Soit f : R +∗ → R x →
− log x. Cette application est deux fois dérivable et f (x) = 1/x2 > 0 pour tout x > 0. C’est
donc une fonction convexe, ce qui entraı̂ne
 
x 1 + · · · + xn f (x1) + · · · + f (xn )
f ≤
n n
d’après la proposition 3, c’est-à-dire
   
x 1 + · · · + xn
log ≥ log (x1 · · · x n )1/n ,
n
et la fonction logarithme étant croissante, on en déduit le théorème. 

Théorème 2 (In égalité de H ölder). Soient deux nombres réels p, q > 0 tels que
1
p
+ 1q = 1. Pour tous nombres réels positifs a 1 , . . . , an et b 1, . . . , bn on a
n
 n 1/p  n  1/q
  p  q
a ibi ≤ ai bi .
i=1 i=1 i=1

Démonstration. La concavité de la fonction logarithme entraı̂ne


 
log x log y x y x y
∀x, y > 0, + ≤ log + donc x1/p y1/q ≤ + ,
p q p q p q
inégalité qui reste vraie lorsque x ou y est nul. En choisissant
ap bq
x= i p et y =  i q ,
j aj j bj

(on peut supposer qu’au moins l’un des ai et l’un des bj sont non nuls, sinon l’inégalité de Hölder
est immédiate) on tire
a b ap bq
 ip 1/p ·  iq 1/q ≤  i p + i q ,
( j aj ) ( j bj ) p( j aj ) q( j bj )
puis en sommant sur i n
 p 1/p i=1 a b 1 1
i i q 1/q ≤ + = 1,
( j a j ) ( j bj ) p q
d’où l’inégalité désirée. 

Remarque 2. Lorsque p = q = 2, on retrouve l’inégalité de Schwarz.

Théorème 3 (In égalité de Minkowsky). Soient p ≥ 1 un nombre réel et x1 , . . . , xn ,


y1 , . . . , y n des nombres réels positifs. Alors
 n 1/p  n 1/p  n 1/p
  p  p
(x i + yi ) p ≤ xi + yi .
i=1 i=i i=i
p 1 1
Démonstration. Si p = 1, c’est évident. Sinon, on pose q = p−1 , de sorte que p+ q = 1. D’après
l’inégalité de Hölder
 n   n 1/q
 n  p 1/p 
p−1 q(p−1)
xi (xi + y i ) ≤ xi (xi + y i)
i=1 i=1 i=1
98 2. FONCTIONS D’UNE VARIABLE RÉELLE

et n 1/p  1/q
n
  n

yi(xi + yi ) p−1 ≤ y pi (xi + yi ) q(p−1) ,
i=1 i=1 i=1
ce qui par sommation entraı̂ne (sachant que q (p − 1) = p)
 1/p  n 1/p   n  1/q
n
 n  
p p
(x i + y i) ≤ 
p
xi + yi  (xi + yi) p

i=1 i=1 i=1 i=1

donc n 1−1/q  n   n 
  p 1/p  p 1/p
(xi + yi )p ≤ xi + yi ,
i=1 i=1 i=1
(on peut supposer que les xi et yj sont non tous nuls — sinon l’inégalité est évidente — ce qui
autorise à simplifier) d’où le résultat car 1 − 1/q = 1/p. 
Conséquence : Pour tout p ≥ 1, considérons la fonction de Rn dans R définie par
 n  1/p

N (x1 , . . . , x n ) = |x i| p .
i=1
L’inégalité de Minkowsky entraı̂ne que N vérifie l’inégalité triangulaire. On en déduit
facilement que N est une norme sur Rn. On la note souvent  .  p . Remarquons aussi que
sup |xi | = (x1 , . . . , xn)∞ = lim (x 1, . . . , x n)p .
i p→∞

3.2. Fonctions réglées


Dans cette partie, E désigne un R-espace de Banach, et [a, b] désigne un segment de
R non réduit à un singleton.
Définition 2. Une application ϕ : [a, b] → E est dite en escalier s’il existe une subdi-
vision a = x0 < x 1 < · · · < x n = b de [a, b] telle que pour tout i, ϕ soit constante sur
]xi−1, x i[.
Définition 3. Une application f : [a, b] → E est dite réglée si elle est limite uniforme
sur [a, b] de fonctions en escalier. En d’autres termes, f est réglée si pour tout ε > 0, il
existe une fonction ϕ est escalier telle que f (x) − ϕ(x) < ε pour tout x ∈ [a, b].
Proposition 4. Une fonction réglée sur [a, b] est bornée.
Démonstration. Soit f : [a, b] → E une fonction réglée. Il existe une fonction en escalier ϕ telle
que f (x) − ϕ(x) < 1 pour tout x ∈ [a, b]. Comme ϕ est en escalier, ϕ est bornée. Soit M un
majorant de ϕ sur [a, b]. Pour tout x ∈ [a, b], f (x) < 1 + ϕ(x) ≤ 1 + M , ce qui prouve le
résultat. 

Définition 4. Une application f : [a, b] → E est dite continue par morceaux s’il existe
une subdivision a = x 0 < x 1 < · · · < x n = b de [a, b] telle que la restriction de f à chaque
intervalle ouvert ]x i−1, x i[ soit prolongeable en une fonction continue sur l’intervalle fermé
[xi−1, x i].
Remarque 3. — La condition “prolongeable en une fonction continue sur [xi−1, x i]”
est importante ; elle équivaut à dire que la limite à droite (resp. à gauche) de f (x)
lorsque x tend vers xi−1 (resp. vers x i ) existe.
— L’ensemble C m([a, b], E) des fonctions à valeurs dans E et continues par morceaux
sur [a, b] est un espace vectoriel. Si E est une algèbre, Cm([a, b], E) est une algèbre.
3. FONCTIONS CONVEXES, FONCTIONS RÉGL ÉES 99

Proposition 5. Toute fonction continue par morceaux sur [a, b] est réglée.
Démonstration. On montre d’abord le résultat pour une fonction continue f : [a, b] → E .
Comme [a, b] est compact, f est uniformément continue sur [a, b] d’après le théorème de Heine.
Ainsi, si on se donne ε > 0,
∃α > 0, ∀x, y ∈ [a, b], |x − y | < α, f (x) − f (y) < ε. (∗)
Considérons une subdivision a = x0 < x1 < · · · < x n = b de [a, b] telle que xi − xi−1 < α pour
tout i ∈ {1, . . . , n} et construisons une application ϕ : [a, b] → E de la manière suivante :
 
xi−1 + x i
∀i, ϕ(xi ) = f (xi ) et ∀i, ∀x ∈ ]xi−1, x i[, ϕ(x) = f .
2
Cette fonction est en escalier et elle vérifie f (x) − ϕ(x) < ε pour tout x ∈ [a, b] d’après (*).
La fonction f est donc réglée.
Considérons maintenant le cas d’une fonction continue par morceaux pour une subdivision
a = x0 < x 1 < · · · < xn = b. Soit ε > 0. La restriction fi de f sur ]xi−1 , x i [ se prolonge en
une fonction continue fi sur [x i−1 , xi], donc on peut trouver une fonction en escalier ϕ i vérifiant
fi −ϕ i < ε sur ]x i−1, xi [. La fonction ϕ construite sur [a, b] par ϕ(x) = ϕ i(x) pour x ∈ ]x i−1 , xi [
et ϕ(xi) = f (x i) est en escalier et par construction, f − ϕ < ε sur [a, b] tout entier. Ainsi f
est une fonction réglée. 

Définition 5. Soient I un intervalle de R et une application f : I → E . On dit que f



présente une discontinuité de première espèce en un point x 0 ∈ I si f n’est pas continue
en x0 et si f (x 0−) (limite à gauche de f en x 0) et f (x0 +) (limite à droite de f en x0)
existent. La définition s’étend aux bornes de I lorsque I y est fermé en ne considérant
que l’existence de f (x0 +) ou f (x0 −) selon le cas.
Remarque 4. Il est équivalent de dire que f (x 0+) et f (x 0−) existent et que les valeurs
f (x0−), f (x0+), f (x 0 ) ne sont pas toutes identiques.
Théorème 4. Une application f : [a, b] → E est réglée si et seulement si tout point de
discontinuité de f est de première espèce. L’ensemble des points de discontinuité de f est
alors au plus dénombrable.
Démonstration. Condition nécessaire. Soit x0 < b un point de discontinuité de f . Soit ε > 0 et
soit ϕ une fonction en escalier telle que f (x) − ϕ(x) < ε pour tout x ∈ [a, b]. Il existe α > 0
tel que ϕ soit constante sur ]x 0, x 0 + α[, ce qui entraı̂ne
∀x, y ∈ ]x0 , x0 + α [, f (x) − f (y ) ≤ f (x) − ϕ(x) + ϕ(x) − ϕ(y) + ϕ(y) − f (y ) < 2ε.
L’espace vectoriel E étant complet, on en déduit d’après le critère de Cauchy pour les fonctions
que f (x0 +) = lim x→x 0 f (x) existe. On montrerait de même que f (x 0 −) existe pour x0 > a.
x>x0
Condition suffisante. Soit ε > 0. Pour x ∈ ]a, b[ on pose ω (f, x) = max(f (x−) − f (x), f (x) −
f (x+)). D’après le résultat de la question a) de l’exercice 7 page 36, l’ensemble Aε = {x ∈
]a, b[, ω(f, x) ≥ ε} est fini. Il existe donc une subdivision a = x0 < x1 < · · · < xn = b de [a, b]
telle que ω(f, x) < ε sur chaque ]xi , xi+1 [, et d’après le résultat de la question d) du même
exercice, on peut donc trouver pour tout i un αi > 0 tel que
∀x, y ∈ ]xi, x i+1[, |x − y| < αi , f (x) − f (y ) < 2ε. (∗)
Pour tout i, considérons une subdivision xi = yi,0 < yi,1 < · · · < yi,n i = xi+1 de [xi , xi+1 ] telle
que yi,j+1 − yi,j < αi pour tout j . On construit la fonction en escalier ϕ i sur [x i, xi+1 ] en posant
 
yi,j + yi,j+1
∀j, ϕi (yi,j ) = f (y i,j), ∀j, ∀x ∈ ]y i,j , yi,j+1 [, ϕi (x) = f .
2
D’après (*), on a f − ϕi  < 2ε sur [xi , x i+1]. La fonction ϕ construite sur [a, b] par ϕ(x) = ϕ i (x)
pour x ∈ ]xi−1 , xi [ et ϕ(xi) = f (x i) est en escalier et vérifie f − ϕ < 2ε sur [a, b] tout entier.
Ainsi f est une fonction réglée.
100 2. FONCTIONS D’UNE VARIABLE RÉELLE

Ensemble des points de discontinuité de f . L’ensemble des points de discontinuité de f est


au plus dénombrable d’après le résultat de la question b) de l’exercice 7 page 36. 
Conséquence : Une application f : [a, b] → R monotone est réglée. En effet, supposons
par exemple f croissante. Si x 0 ∈ ]a, b], alors f (x) ≤ f (x 0 ) pour tout x < x0 , et comme f
est croissante, f (x0 −) = lim xx<x
→x 0 f (x) existe. De même, f (x +) existe pour tout x ∈ [a, b[.
0 0
0
Ainsi, tout point de discontinuité de f est forcément de première espèce, donc f est réglée.
3.3. Exercices
Exercice 1. Soit f : R → R une fonction convexe et majorée sur R. Montrer que f
est constante. Le résultat reste t-il valable si on suppose seulement f définie, convexe et
majorée sur R + ?

Solution. Supposons f non constante, de sorte qu’il existe x, y ∈ R, x < y, avec f (x) =
 f (y ).
— Si f (x) < f (y), la convexité de f entraı̂ne
f (z ) − f (y ) f (y ) − f (x)
∀z > y, ≥ = a ou encore f (z ) ≥ f (y) + a(z − y ).
z−y y−x
Comme f (y ) > f(x), on a a > 0 et donc limz→+∞ f (z ) = +∞ ce qui est contraire aux
hypothèses.
— Si f (x) > f (y), on montrerait de même que lim z→−∞ f (z ) = +∞, d’où l’absurdité.
L’application f est donc constante.
Le résultat est faux lorsque les hypothèses ne sont vraies que sur R+ , comme le montre
l’exemple de la fonction f : R + → R x → 1+x 1
.

Exercice 2. Soit f : R + → R une fonction convexe.


f (x)
a) Montrer que  = lim existe.
x→+∞ x
b) On suppose que  ∈ R. Montrer que limx→+∞ f (x) − x existe.

f (x) − f (0)
Solution. a) Comme f est convexe, la fonction R+∗ → R x → est croissante.
x−0
f (x) − f (0)
Donc lim existe, et cette limite n’est autre que . On a d’ailleurs  ∈ R ∪ {+∞}
x→+∞ x
(on peut avoir  = +∞ comme le montre l’exemple de la fonction x → x2 ).
b) L’application g : R + → R x → f (x) − x est convexe (somme de deux fonctions convexes),
et elle vérifie limx→+∞ g(xx) = 0.
Montrons que g est décroissante. Soient a, b ∈ R+, a < b. Comme g est convexe, la fonction
g (x) − g (a) g (x) − g (a) g (x)
x → est croissante. Or lim = lim = 0. Donc pour tout x = a,
x−a x→+∞ x−a x→+∞ x
g (x) − g (a)
≤ 0. En appliquant ceci à x = b, on en déduit g (b) ≤ g (a).
x−a
Une fonction décroissante sur R+ admet toujours une limite en +∞, donc limx→+∞ g (x) =
limx→+∞ f (x) − x existe. Cette limite appartient à R ∪ {−∞} (elle peut valoir −∞ comme le
montre l’exemple de la fonction x → − log(1 + x)).

Exercice 3 (Crit ère pratique de convexité d’une fonction). Soient I un in-


tervalle de R et f : I → R une application. Montrer que f est convexe si et seulement
si pour tout segment [a, b] inclus dans I et pour tout µ ∈ R, l’application ϕ : [a, b] →
R x → f (x) + µx est bornée sur [a, b] et atteint sa borne supérieure en a ou en b.
3. FONCTIONS CONVEXES, FONCTIONS RÉGL ÉES 101

Solution. Condition nécessaire. Soit [a, b] un segment inclus dans I et µ ∈ R. L’application


ϕ : [a, b] → R x → f (x) + µx est convexe sur [a, b] car c’est la somme de deux fonctions
convexes. L’un des réels ϕ(a), ϕ(b) est donc la borne supérieure de ϕ car si M = sup{ϕ(a), ϕ(b)},
∀λ ∈ [0, 1], ϕ(λa + (1 − λ)b) ≤ +λϕ(a) + (1 − λ)ϕ(b) ≤ λM + (1 − λ)M = M.

Condition suffisante. Soit [a, b] un sous-intervalle fermé de I . On choisit µ de sorte que ϕ(a) =
ϕ(b), c’est-à-dire que l’on considère l’application
f (b) − f (a)
ϕ : [a, b] → R x → f (x) − x.
b−a
Ainsi on a bien ϕ(a) = ϕ(b). Par hypothèse on a
∀λ ∈ [0, 1], ϕ(λa + (1 − λ)b) ≤ ϕ(a) = ϕ(b)
et en remplaçant ϕ par son expression en fonction de f , on vérifie facilement que ceci s’écrit
aussi
∀λ ∈ [0, 1], f (λa + (1 − λ)b) ≤ λf (a) + (1 − λ)f (b).
Ceci étant vrai pour tout [a, b] ⊂ I, on en déduit que f est convexe.
Remarque. Ce critère rend parfois des services pour démontrer la convexité d’une fonction
(voir les deux exercices qui suivent).

Exercice 4. Soit I un intervalle de R et f : I → R une application continue. Montrer


que f est convexe si et seulement si
 x+h
2
∀x ∈ I, ∀h > 0 vérifiant (x − h, x + h) ∈ I , 2h f (x) ≤ f (t) dt.
x−h

Solution. Condition nécessaire. Soit x ∈ I et h > 0 tel que (x − h, x + h) ∈ I 2 . On a


 
(x − t) + (x + t) 1
∀t ∈ [0, h], f (x) = f ≤ (f (x − t) + f (x + t)) ,
2 2
et en intégrant cette inégalité pour t ∈ [0, h] on obtient
 x+h  x  x+h 
1
f (x) dt ≤ f (t) dt + f (t) dt
x 2 x−h x

d’où le résultat.
Condition suffisante. Utilisons le critère de l’exercice 3. Si f n’était pas convexe, il existerait un
sous-intervalle fermé [a, b] de I et µ ∈ R tels que l’application ϕ : [a, b] → R x → f (x) + µx
n’admet pas son maximum en a ou en b. L’application ϕ étant continue, il existe c ∈ [a, b]
tel que ϕ(c) = supx∈[a,b] ϕ(x), et par construction, on a ϕ(c) > ϕ(a) et ϕ(c) > ϕ(b). Soit
h = inf{b − c, c − a}. La continuité de ϕ entraı̂ne
 c+h  c+h
ϕ(t) dt < ϕ(c) dt = 2hϕ(c),
c−h c−h

et en remplaçant ϕ par son expression en fonction de f , on obtient


 c+h
f (t) dt < 2hf (c).
c−h

Ceci est contraire aux hypothèses. La fonction f est donc convexe.


102 2. FONCTIONS D’UNE VARIABLE RÉELLE

Exercice 5. Soit I un intervalle de R et f : I → R une application continue.


a) Montrer que f est convexe si et seulement si
◦ 1
∀x ∈ I, ∀ε > 0, ∃h ∈ ]0, ε[, f (x) ≤ [f (x + h) + f (x − h)].
2
(on pourra utiliser le résultat de l’exercice 3).
b) Montrer que f est convexe si
◦ f (x + h) + f (x − h) − 2f (x)
∀x ∈ I, lim sup ≥0
→0
>0 |h|≤ h2

c) Montrer que f est affine si


◦ f (x + h) + f (x − h) − 2f (x)
∀x ∈ I, lim 2
= 0,
h→0
h>0
h

Solution. a) La condition nécessaire est évidente. Pour montrer la condition suffisante, nous
allons utiliser le critère donné dans l’exercice 3.
Soit [a, b] ⊂ I et µ ∈ R. On vérifie immédiatement que l’application ϕ : [a, b] → R x →
f (x) + µx vérifie les mêmes hypothèses que f . Il s’agit de montrer que ϕ atteint son maximum
sur [a, b] en a ou en b. Comme ϕ est continue sur le compact [a, b], ϕ est bornée et atteint ses
bornes. Si on désigne par M son maximum, l’ensemble Γ = ϕ −1 ({M }) est donc non vide. Soit
c = inf Γ. La continuité de ϕ entraı̂ne que Γ est fermé, donc c ∈ Γ, donc ϕ(c) = M . Si a < c < b,
alors par hypothèse
1
∃h > 0, (a < c − h < c < c + h < b et
ϕ(c) ≤ [ϕ(c − h) + ϕ(c + h)]).
2
Comme ϕ(c) = supx∈[a,b] ϕ(x), cette dernière inégalité entraı̂ne ϕ(c − h) = ϕ(c + h) = ϕ(c) = M ,
ce qui est contradictoire avec la définition de c. Donc c = a ou c = b et le résultat est prouvé.
b) Pour tout α > 0, on définit l’application f α : I → R x → f (x) + αx2 . On a
fα(x + h) + f α(x − h) − 2fα (x) f (x + h) + f (x − h) − 2f (x)
∀x ∈ I, ∀h > 0, = + 2α,
h2 h2
et donc en vertu des hypothèses vérifiées par f ,
◦ fα(x + h) + f α (x − h) − 2fα (x)
∀ε > 0, ∀x ∈ I, ∃h ∈ ]0, ε[, ≥ α > 0,
h2
ce qui entraı̂ne
◦ 1
∀ε > 0, ∀x ∈ I, ∃h ∈ ]0, ε[, f α (x) ≤
[fα(x + h) + f α (x − h)],
2
ce qui prouve la convexité de f α d’après le résultat de la question précédente.
Ainsi l’application f , limite simple de fonctions convexes (les applications fα lorsque α → 0+)
est convexe.
c) L’application f vérifie les hypothèses de la question précédente donc est convexe. Ceci est vrai
également pour −f donc f est concave. Une fonction convexe et concave est forcément affine,
d’où le résultat.
Remarque. Un corollaire du résultat de la question a) est le suivant.
 
x+y 1
Si f est continue sur I et vérifie f ≤ [f (x) + f (y)] pour tout
2 2
2
(x, y ) ∈ I , alors f est convexe.
Ceci peut être démontré directement, en procédant comme suit :
— On se donne x et y ∈ I .
4. PROBLÈMES 103

— On montre npar récurrence


 sur n que pour tout entier k compris entre 0 et 2n, on a
kx + (2 − k )y kf(x) + (2n − k )f (y )
f ≤ .
2n 2n
— En utilisant la continuité de f et la densité des nombres de la forme k/2 n (k ∈ Z,
n ∈ N), on prouve que f (λx + (1 − λ)y ) ≤ λf (x) + (1 − λ)f (y ) pour tout λ ∈ [0, 1].

Exercice 6 (Fonctions logarithmiquement convexes). On dit qu’une fonction


f à valeurs dans ]0, +∞[ est logarithmiquement convexe si la fonction log f est convexe.
Soit I un intervalle de R et f : I → R+∗ une application.
a) Montrer que si f est logarithmiquement convexe, alors f est convexe.
b) Montrer que f est logarithmiquement convexe si et seulement si l’application I →
R+∗ x → f (x) cx est convexe pour tout c > 0.
c) Montrer que si f et g : I → R+∗ sont logarithmiquement convexes, alors f + g aussi.

Solution. a) Soient a, b ∈ I et λ ∈ [0, 1]. On a


log[f (λa + (1 − λ)b)] ≤ λ log f (a) + (1 − λ) log f (b) ≤ log[(1 − λ)f (a) + λf (b)]
(la première inégalité résulte des hypothèses et la seconde résulte de la concavité de l’application
logarithme). En ne considérant maintenant les membres extrêmes de ces inégalités, on en déduit,
en vertu du caractère croissant de la fonction logarithme f (λa + (1 − λ)b) ≤ λf (a) + (1 − λ)f (b),
d’où le résultat.
b) Condition nécessaire. Pour tout c > 0, l’application x → log f (x)+ x log c est convexe (somme
de deux fonctions convexes), autrement dit x → log(f (x)cx ) est convexe. D’après la question
précédente, ceci entraı̂ne la convexité de x → f (x)cx .
Condition suffisante. Soient (a, b) ∈ I 2 et λ ∈ [0, 1]. D’après les hypothèses, on a
∀c > 0, f(λa + (1 − λ)b) c λa+(1−λ)b ≤ λf (a) c a + (1 − λ)f (b) cb .
Ceci s’écrit aussi
∀c > 0, f(λa+(1−λ)b) ≤ λf (a) c(1−λ)(a−b) +(1−λ)f (b) c λ(b−a) = λf (a) α(1−λ) +(1−λ)f (b) α−λ ,
(∗)
a−b
où on a posé α = c . L’idée est maintenant de minimiser le terme de droite de cette dernière
équation pour avoir la majoration la plus fine possible du membre de gauche. Les paramètres
a, b, λ étant fixés, une étude de la fonction α → λf (a)α (1−λ) + (1 − λ)f (b)α−λ montre qu’elle
atteint son minimum sur R +∗ en α = f (b)/f (a), valeur en laquelle elle vaut précisément
f (a) λf (b)1−λ . En remplaçant dans (*) avec cette valeur de α, on en déduit
f (λa + (1 − λ)b) ≤ f (a) λf (b)1−λ .
En prenant le logarithme, on en déduit alors la convexité de log f .
c) Il suffit d’utiliser le résultat de la question précédente (sans laquelle le résultat serait difficile
à prouver), en montrant que pour tout c > 0, l’application x → (f (x) + g (x))cx est convexe. Ceci
est immédiat car toujours d’après le résultat de la question précédente, les fonctions x → f (x) cx
et x → g (x) cx sont convexes pour tout c > 0.

4. Problèmes
Problème 1. Soit f : R+ → R une application continue en 0 vérifiant
f (x) − f (kx)
∃k ∈ ]0, 1[,  = x→0
lim existe.
x>0
x
104 2. FONCTIONS D’UNE VARIABLE RÉELLE

Montrer que f est dérivable en 0 et exprimer f (0) en fonction de  et de k .


Solution. Soit ε > 0. D’après les hypothèses,
 
 f (kt) − f (t) 
∃η > 0, ∀t ∈ ]0, η],  −   < ε.
 t
Ainsi, si x ∈ ]0, η], on a
   
 f (k nx) − f (k (k nx))   f (k nx) − f (kn+1 x) 
∀n ∈ N,   −   ≤ ε ou encore  − k n 
 n
kn x   x ≤k ε
donc
 n−1   n−1    n−1 
 f (x) − f (knx)     f (ki x) − f (k i+1 x)  
 i   i  ≤ε
∀n ∈ N,  − k  ≤  − k   ki ,
 x  x
i=0 i=0 i=0
ce qui en faisant tendre n vers +∞ entraı̂ne, en vertu de la continuité de f en 0
 
 f (x) − f (0)   ε
 − ≤ .
 x 1−k  1−k

Ceci étant vrai pour tout x ∈ ]0, η], on en déduit la dérivabilité de f en 0 et f (0) = .
1−k

Problème 2. Soit f : [0, 1] → R une application dérivable telle que f (0) = f (0) = 0 et
f  (1) = 0. Montrer
f (c)
∃c ∈ ]0, 1], f  (c) = .
c

Solution. Commençons par donner l’idée de la preuve. L’égalité f (c) = f (c)/c exprime le

f (c)

x
0 c 1

Figure 8. à l’abscisse c, la tangente du graphe de f passe par l’origine.

fait que la droite passant par l’origine et le point (c, f (c)) est tangente au graphe de f (voir la
figure ci-contre). En regardant la figure, on s’aperçoit d’ailleurs que le point correspondant est
un extremum relatif de la fonction “pente” x → f (x)/x.
Cette remarque nous invite à considérer l’application
f (x)
g : [0, 1] → R x → si x = 0, g(0) = f  (0) = 0,
x
qui est continue sur [0, 1] et dérivable sur ]0, 1], avec
xf (x) − f (x)
∀x ∈ ]0, 1], g (x) = . (∗)
x2
Si f (1) = 0 le résultat est évident en prenant c = 1. Sinon, nous allons prouver que g admet un
extremum en point intérieur à [0, 1]. Quitte à considérer −g, on peut supposer g(1) > 0. Soit
4. PROBLÈMES 105

c ∈ [0, 1] tel que g(c) = supx∈[0,1] g (x) (un tel point c existe car g est continue sur le compact
g (1) − g (x)
[0, 1]). On a c = 1 sinon on aurait g (1) = lim ≥ 0, ce qui est absurde vu que
x→1 1−x
g(1) = −f (1) < 0. On a aussi c = 0 car g (c) ≥ g (1) > 0 = g(0). Ainsi, c est un point intérieur à
[0, 1] où g atteint son maximum, donc g (c) = 0, ce qui donne f (c) = f (c)/c grâce à l’expression
(*).

Problème 3 (e n’est pas algébrique d’ordre 2). Montrer que e n’est pas algébrique
d’ordre 2, c’est-à-dire qu’on ne peut pas trouver trois entiers a, b, c non tous nuls tels que
ae2 + be + c = 0. (Indication : raisonnez par l’absurde en considérant le développement
de Taylor de la fonction f (x) = aex + ce −x).

Solution. La preuve est en quelque sorte une généralisation de la démonstration de l’irrationalité


de e à partir de son expression en terme d’une série. Supposons ae2 + be + c = 0, où a, b, c sont
trois entiers non tous nuls. En désignant par f la fonction f : x → aex + ce−x , ceci entraı̂ne que
f (1) est entier. Nous allons prouver que cette dernière assertion est absurde.
L’égalité de Taylor-Lagrange appliquée à f entraı̂ne
f (0) f (n−1)(0) f (n) (θn )
∀n ∈ N∗ , ∃θ n ∈ ]0, 1[, f(1) = f (0) + +··· + + .
1! (n − 1)! n!
Or pour tout entier naturel k , f (k)(x) = aex + (−1)k ce−x, en particulier f (k) (0) = a + (−1)kc
est entier. On en déduit
f (n)(θn ) (n − 1)!  (n − 1)! (n−1)
∀n ∈ N∗ , = (n −1)!f (1) − (n − 1)!f (0)− f (0) −· · ·− f (0) ∈ Z.
n 1! (n − 1)!
(∗)
(n ) θ
La majoration |f (θn )| = |ae + (−1) ce
n n −θ n (n )
| ≤ |a|e + |c| montre que f (θn ) est bornée, donc
f (n)(θ n)/n tend vers 0 lorsque n tend vers +∞. D’après (*), ce terme est toujours entier, il est
donc nul à partir d’un certain rang, c’est-à-dire
∃N ∈ N∗, ∀n ≥ N, f (n) (θn ) = aeθ n + (−1)n ce−θ n = 0.
On en conclut que pour tout n ≥ N , a et (−1)n c sont de signe opposés, ce qui n’est possible
que si a = c = 0, et donc b = 0. Ceci est en contradiction avec les hypothèses de l’énoncé, d’où
le résultat.
Remarque. On en déduit en particulier que e est irrationnel. En fait, e est même un
nombre transcendant (voir le tome Algèbre).

Problème 4 (Zéros d’un polyn ôme lacunaire). Soit P = Xn + an−1 X n−1 + · · · +


a1 X + a0 un polynôme à coefficients réels.
a) On suppose qu’il existe p ∈ N∗ , p < n, tel que a n−1 = . . . = a n−p = 0 et an−p−1 = 0
(on dit que P présente une lacune de longueur p). Si p est pair, montrer que, comptées
avec leur ordre de multiplicité, P admet au plus n − p racines réelles ; si p est impair et
an−p−1 > 0, montrer que P admet au plus n − p − 1 racines réelles ; si p est impair et
an−p−1 < 0, montrer que P admet au plus n − p + 1 racines réelles.
b) Étudier le cas plus général où p coefficients consécutifs ai de P sont nuls, c’est-à-dire
am = a m+1 = . . . = a m+p−1 = 0 et am−1 = 0, am+p = 0.

Solution. a) Tout repose sur le principe suivant : si un polynôme Q ∈ R[X ] a r racines réelles
(comptées avec leur ordre de multiplicité), alors son polynôme dérivé Q  a au moins r − 1
racines réelles. En effet, si u1 < · · · < u k sont les racines réelles d’un polynôme Q ∈ R[X ]
106 2. FONCTIONS D’UNE VARIABLE RÉELLE

d’ordre de multiplicité respectifs α1, . . . , α k, alors pour tout i ∈ {1 . . . , k − 1}, Q  a au moins


une racine réelle v i dans l’intervalle ]ui , ui+1 [ d’après le théorème de Rolle. De plus, si αi ≥ 2,
alors ui est 
 une racine de Q d’ordre de multiplicité αi − 1. Finalement, nous avons exhibé
(k − 1) + i (α i − 1) = ( i αi) − 1 racines de Q  , ce qui prouve notre remarque.
Maintenant, notons r le nombre de racines réelles de P (comptées avec leur ordre de multi-
plicité). Nous venons de voir que P  a au moins r − 1 racines réelles, et en itérant le procédé,
on s’aperçoit que P (n−p−1) a au moins r − (n − p − 1) = r − n + p + 1 racines réelles. Comme
an−1 = . . . = a n−p = 0, on a

P (n−p−1) = n(n − 1) . . . (p + 2)X p+1 + (n − p − 1)! an−p−1 . (∗)


Si p est pair, la forme de ce dernier polynôme montre qu’il a exactement une racine réelle, et
comme nous avons montré qu’il en a au moins r − n + p + 1, on a r − n + p + 1 ≤ 1, c’est-
à-dire r ≤ n − p. Si p est impair et ap+1 > 0, le polynôme (*) n’a aucune racine réelle, donc
r − n + p + 1 ≤ 0, c’est-à-dire r ≤ n − p − 1. Si p est impair et a p+1 < 0, (*) a exactement deux
racines réelles donc r − n + p + 1 ≤ 2, c’est-à-dire r ≤ n − p + 1.
b) Remarquons tout d’abord que pour tout polynôme Q de degré d, dont le terme constant est
non nul, le polynôme Q∗ = X d Q(1/X ) (appelé polynôme réciproque de Q) a le même nombre

de racines réelles que Q : en effet, si Q = ki=1 (X − u i )α i est la factorisation de Q dans C[X ],

on a Q∗ = ki=1 (1 − u i X)α i (les racines de Q∗ sont les inverses des racines de Q, avec le même
ordre de multiplicité).
Ceci étant, notons Q = P (m−1) . On a
Q = n(n − 1) . . . (n − m − 2) Xn−m+1 + · · · + (m + p) · · · (p + 2)a m+p X p+1 + (m − 1)! am−1 ,
le polynôme réciproque de Q est
Q∗ = (m − 1)!am−1 X n−m+1 + (m + p) · · · (p + 2)am+p X n−m−p + · · · + n(n − 1) · · · (n − m − 2).
D’après la question précédente, Q∗ a au plus (n − m + 1) − p racines réelles si p est pair, au
plus (n − m + 1) − p − 1 si p est impair et am+pa m−1 > 0, au plus (n − m + 1) − p + 1 si p
est impair et a m+pam−1 < 0. Le nombre de racines réelles de Q est égal au nombre de racines
réelles de Q∗ , donc ceci vaut pour P (m−1). On en déduit, d’après le principe général énoncé plus
haut, que P a au plus m − 1 + [(n − m + 1) − p] = n − p racines réelles si p est pair, au plus
m − 1 + [(n − m + 1) − p] − 1 = n − p − 1 si p est impair et am+pa m−1 > 0, au plus n − p + 1 si
p est impair et am+p am−1 < 0.

Problème 5. Soit f : R+ → R une application de classe C ∞ telle que


f (0) = 0 et lim f (x) = 0.
x→+∞

Démontrer l’existence d’une suite (xn ) strictement croissante à valeurs positives telle que
f (n) (xn ) = 0 pour tout n ∈ N∗.

Solution. Nous allons construire cette suite par récurrence. Commençons par n = 1. La fonction
f étant nulle à l’origine et tendant vers 0 en +∞, elle n’est pas strictement monotone. Ainsi,
la fonction dérivée f  prend des valeurs positives et négatives, ce qui entraı̂ne qu’elle s’annule
en au moins un point x 1 ∈ R+ d’après le théorème des valeurs intermédiaires et en vertu de la
continuité de f . Ainsi, nous avons construit x 1 ≥ 0 tel que f (x 1) = 0.
Pour passer du rang n ≥ 1 au rang n + 1, on généralise la technique utilisée pour n = 1.
Supposons xn construit et montrons l’existence de x n+1 > x n tel que f (n+1)(xn+1) = 0. Pour
cela, raisonnons par l’absurde en supposant f (n+1) (x) = 0 pour tout x > xn . La continuité de
f (n+1) entraı̂ne que f (n+1) garde un signe constant sur ]x n , +∞[, par exemple strictement positif
(quitte à changer f en −f ). Ainsi, f (n) est une fonction strictement croissante sur [xn , +∞].
4. PROBLÈMES 107

Fixons un réel a > xn . D’après l’égalité de Taylor-Lagrange,


(x − a)n−1 (n−1) (x − a)n (n)
∀x ≥ a, ∃y ∈ [a, x], f(x) = f (a) + (x − a)f  (a) + · · · + f (a) + f (y ),
(n − 1)! n!
ce qui entraı̂ne, en vertu du caractère croissant de f (n) sur [xn , +∞[ et du fait que α = f (n)(a) >
f (n)(x n ) = 0
(x − a) n−1 (n−1) (x − a)n
∀x ≥ a, f (x) ≥ f (a) + (x − a)f (a) + · · · + f (a) + α .
(n − 1)! n!
Le terme de droite de cette dernière expression diverge vers +∞ lorsque x → +∞, il en est donc
de même pour f (x), ce qui est contraire aux hypothèses. Ainsi, il existe bien un réel x n+1 > xn
tel que f (n+1) (xn+1 ) = 0 et le résultat est prouvé.

Problème 6. Soit une application continue g : R → R telle que


∀x ∈ R, g 2 (x) = g ◦ g (x) = 2g (x) − x.
a) Montrer que g est une bijection croissante de R sur R.
b) Déterminer la forme de g . (Indication : on pourra exprimer g n = g ◦ g ◦ · · · en fonction
de g pour tout entier naturel n).

Solution. a) L’injectivité de g est immédiate puisque


g (x) = g (y ) =⇒ g 2(x) = g 2 (y) =⇒ x = 2g (x) − g2(x) = 2g (y ) − g 2 (y) = y.
Montrons la surjectivité de g. Une application injective et continue sur R est strictement
monotone, donc g est strictement monotone (c’est classique par le théorème des valeurs in-
termédiaires). L’application g ◦ g est strictement croissante (composée de deux fonctions de
même monotonie) donc g = (g 2(x) + x)/2 est strictement croissante. Le caractère croissant de
g2 entraı̂ne d’ailleurs
g 2(x) + x g2 (0) + x
∀x ≥ 0, g(x) = ≥ donc lim g (x) = +∞.
2 2 x→+∞

On montrerait de même que limx→−∞ g (x) = −∞. Tout ceci permet de conclure que g est une
bijection de R sur R.
b) Par récurrence sur n on obtient
∀n ∈ N ∗, ∀x ∈ R, g n(x) = ng (x) − (n − 1)x. (∗)
En effet, la relation est vraie pour n = 1 ; pour passer du rang n au rang n + 1 on part de la
relation (*) dans laquelle on remplace x par g (x), ce qui donne
∀x ∈ R, g n+1(x) = ng 2(x) − (n − 1)g (x) = n(2g (x) − x) − (n − 1)g(x) = (n + 1)g(x) − nx.
On peut récrire la relation (*) sous la forme
g n(x) − g n (0) x
∀x ∈ R, ∀n ∈ N∗ , = g (x) − g (0) − x + .
n n
Comme g est croissante, gn également donc pour tout n ∈ N ∗
x x
∀x ≥ 0, g(x) − g (0) − x + ≥ 0 et ∀x ≤ 0, g(x) − g(0) − x + ≤ 0.
n n
En fixant x puis en faisant tendre n vers l’infini, on obtient
∀x ≥ 0, g(x) − g (0) − x ≥ 0 et ∀x ≤ 0, g(x) − g (0) − x ≤ 0. (∗∗)
Nous avons démontré à la question précédente que g est une bijection de R sur R. Sa bijection
réciproque g−1 est continue et vérifie
∀x ∈ R, g 2 [g −2(x)] = 2g [g−2 (x)] − g−2 (x) ou encore g −2(x) = 2g −1 (x) − x.
108 2. FONCTIONS D’UNE VARIABLE RÉELLE

Autrement dit, g−1 vérifie les mêmes hypothèses que g . On en conclut que (**) est vrai pour
g−1 , ce qui s’écrit
∀x ≥ 0, g−1 (x) − g−1 (0) − x ≥ 0 et ∀x ≤ 0, g−1 (x) − g −1 (0) − x ≤ 0.
En utilisant la relation g−1(x) = 2x − g (x) (qui découle de g 2 = 2g − Id après composition à
droite par g −1), on en déduit
∀x ≥ 0, g(x) − g (0) − x ≤ 0 et ∀x ≤ 0, g(x) − g (0) − x ≥ 0. (∗∗∗)
Avec (**), on en conclut que g (x) = x + g(0) pour tout x ∈ R.
Réciproquement, on vérifie facilement que toute fonction de la forme g : x → x + K vérifie
les hypothèses de l’énoncé.

Problème 7 (Dérivée selon Schwarz). Soit I un intervalle ouvert non vide de R


et f : I → R une fonction continue, telle que pour tout x ∈ I , la limite
f (x + h) − f (x − h)
f S (x) = lim
h→0
h=0
2h
existe (on dit que f est dérivable selon Schwarz, ou encore pseudo-dérivable, et f S (x) est
appelé la dérivée symétrique de f en x).
a) Montrer que si f S ≥ 0, alors f est croissante (on commencera par le cas f S ≥ α > 0).
b) Montrer que si fS est continue en a ∈ I , alors f est dérivable en a et f  (a) = f S(a).

Solution. a) Suivons l’indication et supposons d’abord f S ≥ α > 0 sur I. On raisonne par


l’absurde en supposant que f n’est pas croissante, ce qui implique l’existence de a, b ∈ I , a < b,
tels que f (a) > f (b). L’idée est de déterminer une abscisse c tel que f (c + h) ≤ f (c − h) pour
des valeurs de h tendant vers 0.
On choisit y tel que f (b) < y < f (a) et on considère l’ouvert O = {x ∈ ]a, b[ | f (x) > y}.
Comme f (a) > y et que f est continue, O est non vide. Donc c = sup O est bien défini et c > a.
On a c < b, puisque sur un voisinage de b, f (x) < y. Par définition de c, on a f (x) ≤ y dès que
x ∈ [c, b[. Comme c = sup O , il existe une suite (hn) de valeurs > 0 tendant vers 0 telle que les
c − hn ∈ O pour tout n, et donc telle que f (c − hn) > y. Lorsque n est suffisamment grand on
a c + hn < b donc f (c + hn ) ≤ y . Ainsi, on a (f (c + hn) − f (c − hn ))/(2hn) ≤ 0, et par passage
à la limite lorsque n → ∞ on en déduit f S(c) ≤ 0, ce qui est contradictoire. On a donc bien
démontré que f est croissante sur I .
Passons maintenant au cas où on a simplement fS ≥ 0. Pour tout α > 0, la fonction
fα : x → f (x) + αx est dérivable selon Schwarz sur I et on a fαS (x) = fS (x) + α ≥ α, donc
d’après ce qu’on vient de montrer, fα est croissante. Ceci implique, pour tout a, b ∈ I tels que
a < b, l’inégalité fα (a) ≤ fα (b) , et ceci étant vrai pour tout α > 0 on obtient f (a) ≤ f (b). Donc
f est bien croissante.
b) Nous allons commencer par montrer un résultat intermédiaire, qui est l’équivalent de l’inégalité
des accroissements finis pour les fonctions dérivables selon Schwarz. Supposons qu’il existe un
intervalle ouvert J ⊂ I et M1 , M2 ∈ R tels que ∀x ∈ J ; M1 ≤ f S (x) ≤ M 2 . Alors
∀α, β ∈ J ; α < β, (β − α )M1 ≤ f (β ) − f (α) ≤ (β − α)M 2 . (∗)
On montre la première inégalité en appliquant le résultat de la question précédente à la fonction
g (x) = f (x) − M 1x. La fonction g est continue et dérivable selon Schwarz sur J , et g S =
f S − M1 ≥ 0 sur J donc g est croissante sur J , ce qui implique g (β ) ≥ g (α), prouvant ainsi la
première inégalité. On montre la seconde inégalité de la même manière en considérant la fonction
h(x) = M 2x − f (x).
Prouvons maintenant la dérivabilité de f en a. Soit ε > 0. La continuité de f S en a implique
l’existence de η > 0 tel que
∀x ∈ ]a − η, a + η [ , f S(a) − ε ≤ f S (x) ≤ f S (a) + ε.
4. PROBLÈMES 109

On en déduit, avec le résultat (*), que


f (x) − f (a)
∀x ∈ ]a − η, a + η [ , x =
 a, f S (a) − ε ≤ ≤ fS (a) + ε.
x−a
L’existence d’un tel η > 0 est possible pour tout ε > 0, ce qui prouve que f est bien dérivable
en a, et on a f  (a) = f S (a).

Problème 8. On note E l’ensemble des fonctions f de classe C 1 bijectives de ]0, +∞[


dans ]0, +∞[ telles que f  = f −1 .
a) Trouver un élément f de E de la forme x → αxβ , avec α, β ∈ R.
b) Si f ∈ E , déterminer la limite en 0 de f et de f −1 .
c) Montrer que si f ∈ E , alors f est un C ∞ difféomorphisme de ]0, +∞[ dans ]0, +∞[.
d) Montrer que toute fonction f ∈ E admet un unique point fixe.
e) Soit f et g deux élément de E. Montrer que f et g admettent le même point fixe.

Solution. a) Si f (x) = αxβ , alors f  (x) = αβxβ−1 et f −1 (x) = (x/α)1/β = (1/α)1/β x1/β .
Remarquons que f > 0 donc α = f (1) > 0. On a également f  = f −1 > 0, donc f (1) = αβ > 0
ce qui entraı̂ne β > 0.
On aura f  = f −1 si αβ = (1/α √
)1/β et si β − 1 = 1/β. La dernière équation s’écrit aussi
1+ 5
β2 − β − 1 = 0, dont β = ϕ = 2 est la seule solution positive. L’autre équation admet la
solution α = ϕ−ϕ/(ϕ+1) (comme ϕ + 1 = ϕ 2 on a l’expression plus simple α = ϕ −1/ϕ). Ainsi
choisis, α et β répondent bien au problème.
b) Si f ∈ E alors f −1 > 0 donc f  = f −1 est positive. Donc f est croissante, et donc f (x)
converge forcément vers  = inf x>0 f (x) lorsque x → 0 (x > 0). Comme f est positive, on a
 ≥ 0. Par ailleurs, f (x) ≥  pour tout x > 0, et comme f est une bijection dans ]0, +∞[ on
en déduit nécessairement  = 0. On montrerait de même que lim x→0,x>0 f−1 (x) = 0 (on peut
également obtenir ce dernier résultat en prolongeant f en une fonction f˜ sur R+ avec f˜(0) = 0.
L’application f˜ est continue et bijective de R+ dans R+ ; sa fonction réciproque est continue,
en particulier en x = 0 ou elle vaut f˜−1(0) = 0).
c) Montrons par récurrence sur n ∈ N ∗ que f est un C n difféomorphisme. Pour n = 1, on sait
par hypothèse que f est C1 . Or f −1 > 0, donc f  = f −1 ne s’annule pas. On en déduit, d’après
la proposition 4 page 73 sur les homéomorphismes dérivables, que f −1 est également de classe
C 1 . Supposons maintenant n ≥ 1 et que f est un Cn difféomorphisme (hypothèse de récurrence).
Alors f  = f −1 est de classe C n , donc f est de classe C n+1 , et là encore, la proposition sur les
homéomorphismes dérivables implique que f −1 est également de classe C n+1 .
d) Comme f  = f −1 , on a lim x→0+ f (x) = limx→0+ f −1(x) = 0. Ainsi, le prolongement f˜ par
continuité de f en 0, défini en 0 par f̃ (0) = 0, est dérivable en 0 et f˜(0) = 0 (voir la proposition 6
page 76). Ceci montre que f (x) = o(x) lorsque x → 0 (x > 0). En particulier, il existe a > 0 tel
que f (a) < a/2, donc f (a) − a < 0.
Par ailleurs, f −1 étant croissante on a f  (x) = f −1 (x) ≥ f −1 (α) = 2 pour x ≥ α = f (2).
Lorsque x ≥ α, l’égalité des accroissements finis entraı̂ne f (x) − f (α) ≥ 2(x − α), d’où on déduit
f (x) − x ≥ f (α) + x − 2α. Ainsi, b = 2α vérifie f (b) − b > 0.
Ainsi, nous avons montré que la fonction continue f (x) − x change de signe sur [a, b], elle
s’annule donc en un point c de cet intervalle, qui est un point fixe de f .
Montrons que le point fixe de f est unique. Un dessin nous suggère d’utiliser la convexité
de f . Si f admet deux points fixes c et d (avec 0 < c < d), alors f étant convexe (f = f−1
est croissante), elle vérifie f (x) ≥ f (c) + (x − c)f  (c) = c + (x − c)f (c) pour tout x > 0. En
faisant tendre x vers 0 on en déduit 0 ≥ c(1 − f (c)), donc f (c) ≥ 1. On a même f (c) > 1 car
si f (c) = 1, alors f (x) ≥ c + (x − c)f (c) = x, ce qui est incompatible avec f (x) = o(x) lorsque
x → 0. Ceci implique f (d) ≥ c + (d − c)f (c) > d donc d ne peut pas être un deuxième point
fixe de f .
110 2. FONCTIONS D’UNE VARIABLE RÉELLE

e) Nous utiliserons le lemme suivant :

Lemme 1. Soit f ∈ E et λ > 0 le point fixe de f . Alors on a f (x) < x pour 0 < x < λ et
f (x) > x pour x > λ.

En effet, f n’a que le seul point fixe λ, donc f (x) − x garde un signe constant non nul sur
]0, λ[, et comme f (x) = o(x) lorsque x → 0, f (x) − x est forcément négatif sur cet intervalle.
De même, f (x) − x garde un signe constant non nul pour x > λ. Nous avons vu plus haut qu’il
existe µ > 0 tel que f (µ) − µ > 0. On a donc forcément µ > λ, et finalement le signe de f (x) − x
est strictement positif sur ]λ, +∞[.

f g
g −1

f −1

0 c a b

Figure 9. Le graphe des applications f et g et leurs fonctions inverses

Raisonnons maintenant par l’absurde et supposons l’existence de f, g ∈ E tels que f (a) = a


et g (b) = b avec a = b, par exemple a < b (voir la figure ci-contre). D’après le lemme, on a
g (x) < x sur ]0, b[, en particulier g(a) < a = f (a). Toujours d’après le lemme, on a x ≤ f (x)
pour x ∈ [a, b], et finalement g (x) < x ≤ f (x) pour x ∈ [a, b[. Soit c le plus petit réel positif tel
que g (x) < f (x) sur l’intervalle ]c, b[ (on a 0 ≤ c < a).
Montrons maintenant que g−1 > f −1 sur ]c, b[ (on le voit sur la figure). Tout d’abord,
on remarque que ]c, b[ ⊂ f (]c, b[) car d’après le lemme, f (c) < c et f (b) > b. Ceci entraı̂ne
f −1 (]c, b[) ⊂ ]c, b[. On montre aussi facilement que g −1 (]c, b[) ⊂ ]c, b[. Considérons maintenant
c < y < b. Comme x = f −1(y ) ∈ ]c, b[, on a g (x) < f (x). Ceci s’écrit aussi g (f −1 (y)) < y,
c’est-à-dire g (f −1(y)) < g(g−1 (y )) et comme g est strictement croissante, on a nécessairement
f −1 (y ) < g−1 (y).
Terminons la démonstration. Comme g = g −1 et f  = f −1 , ce que nous venons de mon-
trer s’écrit g  > f  sur ]c, b[, donc g − f est strictement croissante sur ]c, b[. En particulier,
lim x→c g (x) − f (x) < g(b) − f (b) ≤ 0. Ceci entraı̂ne c > 0 (car g (x) − f (x) tend vers 0 lorsque
x>c
x → 0), donc g (c) − f (c) < 0, ce qui est incompatible avec la définition de c.
Ainsi, toutes les fonctions f de E ont le même point fixe λ. C’est donc celui de la fonction
F (x) = ϕ−1/ϕ xϕ que nous avons construite dans la question a). On calcule ce point fixe en
résolvant l’équation F (λ) = λ :

F (λ) = λ ⇐⇒ ϕ−1/ϕ λϕ−1 = 1 ⇐⇒ λ = ϕ1/(ϕ(ϕ−1))

et comme ϕ√2 − ϕ = 1, on trouve que le point fixe des applications de E est le nombre d’or
λ = ϕ = 1+2 5 .
4. PROBLÈMES 111

Problème 9 (Théor ème de Sarkowski). Soit I un segment de R et f : I → I une


fonction continue.
a) Pour tout segment K inclus dans f (I ), montrer qu’il existe un segment J inclus dans
I tel que K = f (J ).
b) Si S1 et S2 sont deux segments de I tels que S2 ⊂ f (S1 ) on note S1 → S 2. Supposons
qu’il existe n ∈ N∗ segments I0 , . . . , In−1 de I tels que I0 → I1 → . . . I n−1 → I0 . Montrer
que la fonction f n = f ◦ f ◦ · · · ◦ f admet un point fixe x0 tel que f k (x0) ∈ I k pour
k = 0, 1, . . . , n − 1.
c) Pour n ∈ N∗ , si x ∈ I vérifie f n (x) = x et f k(x) = x pour 1 ≤ k ≤ n − 1, on dit que
x est un point n-périodique. S’il existe un point 3-périodique pour f , montrer qu’il existe
des points n-périodiques pour tout n ∈ N ∗ .

Solution. a) Soit α, β ∈ R tels que K = [α, β]. L’hypothèse K ⊂ f (I) implique l’existence de
a, b ∈ I tels que α = f (a) et β = f (b). Si α = β le résultat est immédiat, en choissant J = {a}.
Dans le cas α < β, on a a = b.
Commençons par le cas a < b. L’idée est de déterminer des antécédents de α et β, de
sorte qu’entre les deux, il n’y ait pas d’autres antécédents de α et β . On considère l’ensemble
A = {x ∈ [a, b] | f (x) = β }, fermé non vide (il contient b) et minoré par a. On peut donc définir
v = inf A, et on a f (v ) = β . L’ensemble B = {x ∈ [a, v] | f (x) = α} est aussi un fermé non vide
(il contient a) et majoré par v. On peut donc définir u = sup B , et on a f (u) = α et u < v.
Le segment J = [u, v] vérifie bien f (J ) = K : comme f (u) = α et f (v) = β, le théorème des
valeurs intermédiaires assure que K ⊂ f (J ). On a bien l’égalité sinon il existerait w ∈ J tel
que f (w) ∈ K . Si f (w ) > β, alors par continuité de f il existerait x ∈ [u, w [ (donc x < v) tel
que f (x) = β ce qui est en contradiction avec la définition de v ; si f (w ) < α alors il existerait
x ∈ ]w, v] (donc x > u) tel que f (x) = α ce qui est en contradiction avec la définition de u.
On traite le cas a > b de la même manière en prenant cette fois u = sup{x ∈ [b, a] | f (x) = β}
puis v = inf {x ∈ [u, a] | f (x) = α}.
b) On commence par le cas n = 1.Supposons I0 ⊂ f (I0 ) et notons I0 = [α, β ]. Il existe a, b ∈ [α, β ]
tel que f (a) ≤ α et f (b) ≥ β . Ainsi la fonction g (x) = f (x) − x vérifie g(a) ≤ 0 et g(b) ≥ 0.
Cette fonction est continue et change de signe sur I0, il existe donc x0 ∈ I 0 tel que g (x 0 ) = 0,
c’est-à-dire f (x 0) = x0 .
Traitons maintenant le cas n = 2, en supposant que I 0 → I 1 → I0 . On a I 1 ⊂ f (I 0) donc
le résultat de la question précédente assure l’existence d’un segment J ⊂ I0 tel que I1 = f (J ).
Comme I0 ⊂ f (I1 ), on en déduit J ⊂ I0 ⊂ f 2 (J ). Le cas n = 1 appliqué à la fonction f 2 montre
l’existence d’un point fixe x0 ∈ J de f 2 . On a bien x0 ∈ I0 et f (x0 ) ∈ f (J) = I 1, nous avons
donc démontré le cas n = 2.
Voyons maintenant le cas général n ≥ 3. Comme I 1 ⊂ f (I0 ) il existe un segment J1 ⊂ I0
tel que f (J1 ) = I1 . On a alors I2 ⊂ f (I 1) = f 2(J 1 ), donc il existe un segment J2 ⊂ J 1 tel que
f 2 (J2 ) = I2 . On construit ainsi des segments J 3, . . . , J n−1 tels que J n−1 ⊂ . . . ⊂ J 2 ⊂ J1 ⊂ I0 et
f k (Jk ) = Ik pour k = 1, . . . , n− 1. Comme I 0 ⊂ In−1 = f n(Jn−1 ), il existe un segment Jn ⊂ Jn−1
tel que f n (Jn ) = I 0. On a J n ⊂ I 0 = f n (Jn ), et donc f n admet un point fixe x0 ∈ Jn ⊂ I0 . Par
construction on a f k (x0) ∈ I k pour k = 1, . . . , n − 1.
c) Notons a un point 3-périodique de f , et b = f (a), c = f 2 (a). Tous les points a, b et c sont
3-periodiques donc quitte à remplacer a par b ou c, on peut supposer a < b et a < c.
- Supposons b < c, de sorte que a < b < c. Soit I 0 = [a, b] et I1 = [b, c]. Comme b = f (a)
et c = f (b), le théorème des valeurs intermédiaires assure que I1 ⊂ f (I 0), c’est-à-dire I 0 → I1 .
De même, f (b) = c et f (c) = a donc [a, c] ⊂ f (I1) ce qui implique I 1 → I 0 et I 1 → I 1. La
propriété I1 → I1 entraı̂ne l’existence d’un point fixe de f dans I1 . On a I0 → I 1 → I0 et ceci
entraı̂ne l’existence d’un point fixe x 2 ∈ I 0 de f 2 tel que f (x2 ) ∈ I1 . On a bien x2 = f (x2 ) (car
x2 ∈ I0 et f (x2) ∈ I 1, donc x 2 ≤ b ≤ f (x2 ) donc si x2 = f (x 2 ) on aurait x2 = b ce qui est
impossible puisque b est un point 3-périodique de f ). Ainsi, x 2 est un point 2-périodique de f .
Pour n ≥ 4, le cycle I0 → I1 → I 1 → . . . → I 1 → I 0, où l’intervalle I1 figure n − 1 fois, montre
l’existence d’un point fixe xn ∈ I 0 de f n tel que f k (xn ) ∈ I1 pour k = 1, . . . , n − 1. Ici aussi, on
112 2. FONCTIONS D’UNE VARIABLE RÉELLE

a xn ≤ b ≤ fk (xn ) pour k = 1, . . . , n − 1 et on ne peut pas avoir x n = b (sinon f 2(xn ) = a ce


qui est impossible car f 2 (xn) ≥ b) donc x n est bien un point n-périodique de f .
- Le cas a < c < b se traite de la même manière en posant I0 = [a, c] et I 1 = [c, b]. On a ici
I 1 → I 0 , I0 → I 1 et I0 → I0. En échangeant le rôle de I 0 et I1 dans le raisonnement précédent
on démontre également l’existence d’un point n-périodique de f pour tout n ∈ N∗ .

Problème 10 (Courbe de Péano — remplissant un carr é). On se donne deux


fonctions f, g : R → R continues, à valeurs dans [0, 1], 1-périodiques et vérifiant :
 
 ∀t ∈  1 , 2  , f (t) = g (t) = 0  ∀t ∈  5 , 6  , f (t) = 1, g(t) = 0
10 10 10 10
 
 ∀t ∈ 3 , 4 , f (t) = 0, g(t) = 1  ∀t ∈ 7 , 8  , f (t) = g (t) = 1

10 10 10 10

(voir la figure ci-dessous).


y y
 
1 1

x x
 
0 0 1 1
x → f (x) x → g (x)

De telles fonctions f et g existent bien, il suffit par exemple de relier les extrémités
des segments en gras sur la figure par des segments de droite.
On considère ensuite les fonctions
+∞
 +∞

f (10n−1 t) g(10n−1 t)
α : R → R t → , β : R → R t →
n=1
2n n=1
2n

et
F : [0, 1] → R 2 t → (α(t), β(t)).

a) Montrer l’existence et la continuité de F .


b) Montrer que F est une surjection de [0, 1] sur [0, 1]2 (autrement dit, le graphe de F
remplit totalement le carré [0, 1]2 ).
c) Existe-t-il une fonction F : [0, 1] → R 2 de classe C1 et vérifiant la propriété de la
question b) ?
d) Existe-t-il une fonction F : [0, 1] → R2 continue et bijective de [0, 1] sur [0, 1] 2 ?

Solution. a) La fonction f est à valeurs dans [0, 1], donc


 
f (10n−1 t) 

∀n ∈ N , ∀t ∈ R,  ≤ 1.
 2n  2n

Ainsi, la série de fonction n≥1 f (10n−1 t)/2n converge normalement sur R, d’où l’existence et
la continuité de α. On procéderait de même pour β .
b) Commençons par considérer t ∈ [0, 1[, et écrivons sa décomposition décimale :
+∞
 tn
t= , (tn ∈ {0, 1, . . . , 9} ∀n ∈ N∗ )
10n
n=1
4. PROBLÈMES 113

de sorte que
+∞
  
∗ n−1 tn ti 1
∀n ∈ N , 10 t = Nn + + Rn avec Nn ∈ N et Rn = ∈ 0, ,
10 10 i+1−n 10
i=n+1

(cette dernière assertion est vraie car les t n ne sont jamais stationnaires à 9 à partir d’un certain
rang) donc, les fonctions f et g étant 1-périodiques,

 2−n f (10n−1 t) = 2−n f  tn + R  
1

∗ 10 n
∀n ∈ N , avec Rn ∈ 0, . (∗)
 2−n g(10n−1 t) = 2−n g  tn + R n  10
10

Ceci étant, donnons nous (x, y ) ∈ [0, 1]2 et écrivons la décomposition diadique de x et y :
+∞
 +∞

xk yk
x= , y= , (x k , yk ∈ {0, 1}).
2k 2k
k=1 k=1

Si x = 1 on choisit xk = 1 pour tout k, de même pour y. Nous allons construire t ∈ [0, 1[ à partir
de sa décomposition décimale pour avoir f (10k−1t) = x k et g (10 k−1t) = y k pour tout k , ce qui
montrera que F (t) = (x, y). Soit k ∈ N ∗ ,
— si (x k , yk) = (0, 0), on choisit t k = 1 ;
— si (x k , yk) = (0, 1), on choisit t k = 3 ;
— si (x k , yk) = (1, 0), on choisit t k = 5 ;
1 — si (x k , yk) = (1, 1), on choisit t k = 7.

L’écriture t = +∞ k
n=1 t k/10 est bien un développement décimal, et t vérifie la propriété requise
d’après (*) et d’après la forme des fonctions f et g . Nous venons donc de prouver que [0, 1] 2 ⊂
F ([0, 1]). L’inclusion réciproque est immédiate, donc finalement, F ([0, 1]) = [0, 1] 2.
c) Non ! Raisonnons par l’absurde en supposant qu’une telle fonction existe. Nous allons re-
couvrir F ([0, 1]) par une surface dont l’aire est inférieure à celle du carré [0, 1] 2, d’où découlera
l’absurdité désirée.
On note M = supt∈[0,1] F  (t) ( où  .  désigne la norme du sup : (x, y) = sup{|x|, |y|}).
On considère un entier naturel non nul quelconque n et la subdivision 0 < n1 < · · · < n−1 n
<1
de [0, 1]. Pour tout entier k, 0 ≤ k ≤ n − 1, on a d’après l’inégalité des accroissements finis
      
k k+1  k + 12   k + 12  M
  
∀t ∈ , ,  F (t) − F  ≤ M t − ≤ .
n n  n   n  2n
 
1
k +
Ainsi, F ([nk , k+1
n
]) est inclus dans le carré de centre F 2
de demi-côté M 2n
, que nous
n
noterons Cn,k . On a donc
  k k + 1  n−1
n−1 
F ([0, 1]) = F , ⊂ C n,k = Sn .
n n
k=0 k=0

L’aire A(Sn ) de Sn est inférieure à la somme des aires des carrés C n,k , ce qui s’écrit
n−1
  M  2 M 2
n−1
A(Sn ) ≤ A(C n,k ) = = .
n n
k=0 k=0

On choisit maintenant n tel que n > M 2. Alors A(S n ) < 1, donc [0, 1]2 ⊂ Sn , et comme
F ([0, 1]) ⊂ Sn on en déduit [0, 1] 2 ⊂ F ([0, 1]). Ceci est impossible, d’où le résultat.
d) C’est dans un cadre légèrement différent la même chose que l’exercice 8 page 47. On raisonne
par l’absurde en supposant qu’une telle bijection F : [0, 1] → [0, 1] 2 existe. L’application F est
continue sur un compact, son application réciproque F −1 : [0, 1] 2 → [0, 1] est donc continue
(voir la proposition 14 page 31). Donnons nous un point quelconque c dans ]0, 1[. L’ensemble
114 2. FONCTIONS D’UNE VARIABLE RÉELLE

[0, 1]2 {F (c)}est connexe (car −1


 connexe par arcs comme on le vérifie facilement), donc F étant
−1 2
continue, F [0, 1] {F (c)} est connexe. On a affaire à une bijection, donc
F −1([0, 1] 2{F (c)}) = F −1 ([0, 1]2 ){c} = [0, 1]{c} = [0, c[ ∪ ]c, 1],
qui n’est pas connexe ! Ceci est absurde, d’où le résultat.

Problème 11 (Fonction de Weierstrass). On définit une application f : [0, 1] → R


par
1 p
f (x) = si x = ∈ Q (p ∈ N, q ∈ N ∗, p ∧ q = 1), f (x) = 0 si x ∈ RQ
q q
(fonction introduite par Weierstrass).
a) Déterminer l’ensemble des points où f est continue.
b) Montrer que f est une fonction réglée.

Solution. a) Nous allons montrer que cette étrange fonction est discontinue en tout point
rationnel et continue en tout point irrationnel.
Soit x ∈ [0, 1] un rationnel et supposons (on raisonne par l’absurde) que f soit continue en
x. Comme les irrationnels sont denses dans R, il existe une suite (x n) de points irrationnels de
[0, 1] qui converge vers x. On a alors f (x) = limn→∞ f (xn ), ce qui est impossible car f (x) = 0
et f (xn ) = 0 pour tout n. Ainsi, f est discontinue en tout point rationnel de [0, 1].
Soit x ∈ [0, 1] un irrationnel. Nous allons montrer la continuité de f en x. Donnons nous
ε > 0 et fixons un entier naturel non nul N tel que 1/N < ε. L’ensemble Γ des rationnels de
[0, 1] qui peuvent s’écrire sous la forme p/q avec p ∈ N, q ∈ N∗ et q < N est fini. Comme de plus
x ∈ Γ, le réel α = infy∈Γ |x − y | est non nul. Considérons maintenant un élément quelconque
y ∈ [0, 1] tel que |x − y| < α. Si y est irrationnel, on a f (y ) = 0 donc |f (x) − f (y)| < ε. Si y = p/q
est rationnel, on a q ≥ N car y ∈ Γ par construction de α, donc |f (x) − f (y )| = 1/q < ε. Ainsi,
∀y ∈ [0, 1], |x − y | < α, |f (x) − f (y )| < ε,
ce qui prouve bien la continuité de f en x.
b) Il s’agit de construire une suite de fonctions en escaliers (ϕn )n∈N∗ qui converge uniformément
vers f . Pour tout n ∈ N∗ , on définit ϕn : [0, 1] → R par

ϕn(x) = f (x) si ∃(p, q ) ∈ N × N ∗, p ∧ q = 1, q ≤ n, x = p/q,
ϕn(x) = 0 sinon.
Pour tout n, ϕ n est nulle partout sauf en un nombre fini de points, c’est donc une fonction en
escalier. Par ailleurs, on a
1
∀x ∈ [0, 1], |f (x) − ϕn (x)| < .
n
En effet,
— si x ∈ RQ, ϕ n(x) = f (x) = 0,
— si x = p/q avec p ∧ q = 1 et q ≤ n, on a ϕ n(x) = f (x),
— si x = p/q avec p ∧ q = 1 et q > n, on a ϕ n(x) = 0 et f (x) = 1/q < 1/n.
Ainsi, la suite de fonctions en escaliers (ϕn) converge uniformément vers f , donc f est une
fonction réglée.
Remarque. Comme f est réglée, elle vérifie les assertions du théorème 4 de la page 99,
ce qui entraı̂ne que l’ensemble des points de discontinuité de f est au plus dénombrable.
Ceci est en accord avec le résultat de la question a).
– On peut prouver (en utilisant le théorème de Baire décrit dans l’annexe A) qu’il n’existe
pas de fonction continue en tout point de Q et discontinue en tout point de RQ.
4. PROBLÈMES 115

Problème 12 (Fonctions positivement homogènes). La lettre E désigne un R-


espace vectoriel. Une application f : E → R est dite positivement homogène de degré α
(avec α > 0) si
∀x ∈ E, ∀λ ∈ R+, f (λx) = λα f (x).
Une application g : E → R est dite convexe si
∀x, y ∈ E, ∀λ ∈ [0, 1], g(λx + (1 − λ)y ) ≤ λg (x) + (1 − λ)g (y ).
On rappelle qu’une semi-norme sur E est une application N de E dans R+ qui vérifie les
propriétés d’une norme sauf la propriété (N (x) = 0 ⇐⇒ x = 0) (voir la remarque 1
page 8). On s’intéressera ici aux fonctions positivement homogènes de degré 1.
1/ Soit f : E → R une fonction positivement homogène de degré 1.
a) Montrer que f est convexe sur E si et seulement si
∀(x, y ) ∈ E2 , f (x + y ) ≤ f (x) + f (y ).
b) Si f est à valeurs positives, montrer que f est convexe sur E si et seulement si l’ensemble
C = {x ∈ E | f (x) ≤ 1} est convexe.
c) Si f est convexe, à valeurs positives et si elle est paire, montrer que f est une semi-
norme.
d) On suppose ici que E est de dimension finie. Soit Ω un ouvert borné non vide de E.
Montrer qu’il existe une norme N sur E telle que Ω = BN (0, 1) = {x ∈ E | N (x) < 1} si
et seulement si Ω est convexe et admet 0 comme centre de symétrie.
2/ (Étude de normes particulières.) Soit un réel α ≥ 1. Montrer, sans utiliser l’inégalité
de Minkowsky, que l’application
N α : Rn → R + (x1 , . . . , xn) → (|x 1| α + · · · + |xn |α )1/α
définit une norme sur Rn.
Solution. 1/ a) Condition nécessaire. Si f est convexe sur E , alors
 
2 1 x+y 1
∀(x, y ) ∈ E , f (x + y ) = f ≤ (f (x) + f (y)) donc f (x + y ) ≤ f (x) + f (y).
2 2 2

Condition suffisante. Il suffit d’écrire que pour (x, y) ∈ E 2 et λ ∈ [0, 1], on a


f (λx + (1 − λ)y ) ≤ f (λx) + f ((1 − λ)y ) = λf (x) + (1 − λ)f (y ).

b) Condition nécessaire. Si f est convexe, alors pour tout (x, y ) ∈ C 2 et pour tout λ ∈ [0, 1],
f (λx + (1 − λ)y ) ≤ λf (x) + (1 − λ)f (y ) ≤ λ + (1 − λ) = 1, donc λx + (1 − λ)y ∈ C.
Condition suffisante. C’est un peu plus délicat. Soit (x, y) ∈ E 2 . Pour tout ε > 0, on a
 
x y x 1
, ∈ C car f = f (x) < 1 (de même pour y)
f (x) + ε f (y ) + ε f (x) + ε f (x) + ε
(le paramètre ε a été introduit pour éviter de diviser par 0) donc C étant convexe par hypothèse,
 
λx (1 − λ)y
∀λ ∈ [0, 1], f + ≤ 1. (∗)
f (x) + ε f (y ) + ε
Si on choisit λ tel que
 
λ (1 − λ) f (x) + ε
= c’est-à-dire λ = ∈ [0, 1] ,
f (x) + ε f (y ) + ε f (x) + f (y) + 2ε
on obtient en remplaçant dans (*)
 
x+y
f ≤1 donc f (x + y ) ≤ f (x) + f (y ) + 2ε.
f (x) + f (y) + 2ε
116 2. FONCTIONS D’UNE VARIABLE RÉELLE

Ceci dernier résultat est vrai pour tout ε > 0, donc f (x + y ) ≤ f (x) + f (y). On en conclut
maintenant avec a) que f est convexe.
c) Comme f est convexe, l’inégalité triangulaire est vérifiée d’après le résultat de la question
a). Il faut maintenant montrer que pour tout x ∈ E et pour tout λ ∈ R, f (λx) = |λ|f (x). Si
λ ≥ 0, ceci résulte du fait que f est positivement homogène de degré 1. Si λ < 0, ceci est une
conséquence de l’hypothèse de parité vérifiée par f car
f (λx) = f ((−λ)x) = (−λ)f (x) = |λ|f (x).

d) Condition nécessaire. Une norme est une fonction positivement homogène de degré 1, est
convexe d’après a) car elle vérifie l’inégalité triangulaire, et est de plus à valeurs positives. On
en conclut grâce à b) que C = BN (0, 1) est convexe. L’intérieur d’un convexe est convexe, donc

C = Ω est convexe. La symétrie de Ω par rapport à 0 est immédiate puisque
∀x ∈ Ω, N (−x) = N (x) < 1 donc − x ∈ Ω.
Condition suffisante. Commençons par définir N . On pose N (0) = 0. Soit x ∈ E , x = 0. Nous
allons définir N (x) comme étant égal à 1/µx , où µx > 0 est tel que µx x appartient à la frontière
de Ω (ceci car N (µx x) = 1) — voir la figure ci-contre.
x

µ xx

0

Figure 10. L’ensemble Ω et la construction de µ x pour un x donné.

On pose Γx = {λ > 0 | λx ∈ Ω}. Comme Ω est ouvert et contient 0 (il est non vide,
symétrique par rapport à 0 et convexe), Γx est non vide. De plus, Ω est borné donc Γ x est
majoré. Ainsi, µx = sup Γ x est bien défini, et on pose N (x) = 1/µ x. Remarquons que Ω étant
fermé, on a µx ∈ Γx .
Ainsi construite, on vérifie facilement que N est positivement homogène de degré 1, paire et
vérifie N (x) = 0 ⇐⇒ x = 0. Par ailleurs,
N (x) ≤ 1 ⇐⇒ µ x ≥ 1 ⇐⇒ 1 ∈ Γx ⇐⇒ x ∈ Ω,
donc C = {x ∈ E | N (x) ≤ 1} = Ω est convexe. En utilisant le résultat de la question c), on en
déduit que N est une norme.
2/ Soit α ≥ 1. Il est immédiat que l’application Nα est positivement homogène de degré 1,
positive et paire, et vérifie Nα(x) = 0 ⇐⇒ x = 0. Pour prouver que c’est une norme, il suffit
de vérifier, en vertu du résultat de la question 1/c), que l’ensemble
C = {x ∈ R n | N α (x) ≤ 1} = {(x1 , . . . , xn ) ∈ Rn | |x 1 |α + · · · + |xn | α ≤ 1}
est convexe. Pour prouver ceci, on remarque que l’application R+ → R x → xα est convexe
(elle est dérivable et sa fonction dérivée x → αxα−1 est croissante), ce qui entraı̂ne pour x, y ∈ C
∀λ ∈ [0, 1], ∀i ∈ {1, . . . , n}, |λxi + (1 − λ)yi | α ≤ (λ|xi | + (1 − λ)|yi |) α ≤ λ|xi | α + (1 − λ)|yi | α,
donc par sommation sur i = 1, . . . , n
n
 n   n 
  
|λxi + (1 − λ)y i|α ≤ λ |xi| α + (1 − λ) |yi |α ≤ λ + (1 − λ) = 1.
i=1 i=1 i=1
En d’autres termes, λx + (1 − λ)y ∈ C . Ainsi, C est convexe et le résultat est prouvé.
4. PROBLÈMES 117

Problème 13 (Moyennes). Soient I et J deux intervalles de R et f un homéomorphisme


de I dans J . Soient (xi, αi )1≤i≤n n couples de I × R +∗. On dit que y ∈ I est la moyenne
selon f de (xi, αi )1≤i≤n si
 
α1f (x1 ) + · · · + αn f (xn) −1 α 1f (x 1 ) + · · · + α n f (xn)
f (y ) = ou encore y = f .
α1 + · · · + α n α1 + · · · + α n
(Par exemple, si αi = 1 pour tout i, la moyenne selon f (x) = x est la moyenne arithméti-
que, la moyenne selon f (x) = log x est la moyenne géométrique, la moyenne selon f (x) =
1/x est la moyenne harmonique.)
a) Soient I, J, K trois intervalles de R et f : I → J , g : I → K deux homéomorphismes.
On dit que la moyenne selon f est inférieure à la moyenne selon g si pour toute famille
finie de couples (xi , αi )1≤i≤n de I × R+∗ , la moyenne selon f de cette famille est inférieure
à la moyenne selon g de cette famille. Si f est croissante, montrer que la moyenne selon
f est inférieure à la moyenne selon g si et seulement si h = f ◦ g −1 est concave. Que dire
si f est décroissante ?
b) (Application.) Rappelons que pour tout α ≥ 1, l’application
Nα : R n → R (x1 , . . . , xn ) → (|x1|α + · · · + |xn |α )1/α
définit une norme sur Rn (voir par exemple la question 2/ du problème précédent). Soient
α, β deux nombres réels tels que 1 ≤ α ≤ β . Montrer que
1 1
∀x ∈ Rn , Nβ (x) ≤ N α(x) ≤ n α − β Nβ(x).

Solution. a) Soit (x i , αi )1≤i≤n une famille finie de I × R+∗ , y sa moyenne selon f , z sa moyenne
selon g . La fonction f étant croissante, on a
(y ≤ z ) ⇐⇒ (f (y ) ≤ f (z)).
Pour tout i, posons z i = g (xi ), de sorte que f (xi ) = f ◦ g −1 (z i ) = h(zi ). On a
 
α 1 h(z1 ) + · · · + αn h(z n ) α1 z1 + · · · + αnzn
f (y ) = et f (z) = h .
α 1 + · · · + αn α1 + · · · + αn
Ces deux expressions montrent que f (y) ≤ f (z ) si et seulement si l’inégalité caractérisant la
concavité de h est vérifiée pour la famille (zi , α i )1≤i≤n . On en conclut facilement que la moyenne
selon f est inférieure à la moyenne selon g si et seulement si h = f ◦ g−1 est concave.
Si f est décroissante, la fonction −f est croissante. La moyenne selon f étant la même que
la moyenne selon −f (revoir les définitions), on en conclut que la moyenne selon f est inférieure
à la moyenne selon g si et seulement si (−f ) ◦ g −1 est concave, c’est-à-dire f ◦ g −1 est convexe.
b) Démontrons Nβ ≤ N α. Lorsque x = (x1 , . . . , xn ) ∈ Rn vérifie N α(x) = 1, on a |x i| ≤ 1 pour
tout i et comme α ≤ β ceci entraı̂ne |xi|β ≤ |xi | α pour tout i, donc
n
 n

β
|xi | ≤ |xi|α = 1,
i=1 i=1
et donc Nβ (x) ≤ 1. Traitons le cas général. Si x = 0, l’inégalité est triviale, sinon, en notant
λ = 1/Nα (x), on a
1 1
Nα(λx) = 1 donc N β (x) = N β(λx) ≤ = Nα (x).
λ λ
1 1
Démontrons maintenant l’inégalité Nα ≤ n α − β Nβ . Soit x = (x1 , . . . , x n ) ∈ Rn. On remarque
que
 
Nα (x) |x1 |α + · · · + |xn |α 1/α
=
n 1/α n
est la moyenne de la famille (|x i |, 1)1≤i≤n selon la fonction f : R+ → R x → x α. De même,
Nβ (x)/n1/β est la moyenne de cette même famille selon g : x → xβ . Comme f ◦ g−1 = xα/β
118 2. FONCTIONS D’UNE VARIABLE RÉELLE

est une fonction concave (la fonction dérivée x → (α/β )xα/β−1 est décroissante car α ≤ β ), et
que f est croissante, le résultat de la question précédente entraı̂ne que la moyenne selon f est
inférieure à la moyenne selon g , en particulier
Nα (x) Nβ (x)
1/α
≤ 1/β ,
n n
d’où l’inégalité désirée.
Remarque. La dernière inégalité est une égalité pour x = (1, . . . , 1), on ne peut donc pas
remplacer n1/α−1/β par une constante plus petite. Remarquez d’ailleurs qu’il aurait été
difficile de prouver cette inégalité sans utiliser le résultat de la question a).

Problème 14 (Fonctions à variation bornée). Pour tout segment [a, b] de R, on


note sub([a, b]) l’ensemble des subdivisions σ de [a, b] : a = x0 < x 1 < · · · < x n = b. Soit
une application f : [a, b] → R. Pour toute subdivision σ : a = x 0 < x1 < · · · < xn = b de
[a, b], on note
n−1

Varσ (f ) = |f (xi+1 ) − f (xi )|.
i=0
On suppose qu’il existe M > 0 tel que Varσ (f ) ≤ M pour toute subdivision σ de [a, b]
(on dit alors que f est à variation bornée ) et on pose
b
f = sup Varσ (f ).
a σ∈sub([a,b])

1/ a) Soit I = [c, d] un segment inclus dans [a, b]. Montrer que la restriction f|I de f à I
est à variation bornée. On peut ainsi définir
d
f = sup Varσ (f|I ).
c σ∈sub([c,d ])
y z z
b) Si a ≤ x < y < z ≤ b, montrer la relation de Chasles f+ f= f.
x y x
c) Si g : [a, b] → R est de classe C1 , montrer que g est à variation bornée et que
b  b
g= |g(t)| dt.
a a

2/ On considère une application f : [a, b] → R.


a) Montrer que f est à variation bornée si et seulement s’il existe deux fonctions croissantes
g, h : [a, b] → R telles que f = g − h.
b) Montrer que si f est à variation bornée, f est une fonction réglée.
3/ (Un exemple de fonction continue qui n’est pas à variation bornée.) Montrer que
l’application
 
1
f : [0, 1] → R, f (x) = x cos si x = 0 et f (0) = 0
x
n’est pas à variation bornée bien qu’elle soit continue.
Solution. 1/ a) Soit σ : c = x0 < x1 < · · · < xn = d une subdivision de [c, d]. Alors
σ : a ≤ x0 < · · · < xn ≤ b est une subdivision de [a, b] et donc
Var σ(f|I ) ≤ Varσ  (f ) ≤ M.
Cette majoration étant valable pour tout σ ∈ sub([c, d]), on en déduit que f |I est à variation
bornée.
4. PROBLÈMES 119

b) On considère deux subdivisions


σ1 : x = x 0 < x 1 < · · · < x p = y ∈ sub([x, y ]) σ2 : y = y0 < y 1 < · · · < y q = z ∈ sub([y, z ]).
En les concaténant, on obtient une subdivision σ : x = x 0 < · · · < xp = y 0 < y 1 < · · · < y q = z
de [x, z ]. Ainsi
z
Var σ1 (f ) + Var σ2 (f ) = Varσ (f ) ≤ f.
x
Cette majoration étant valable pour tout σ 1 ∈ sub([x, y]) et σ 2 ∈ sub([y, z ]), on en déduit
y z z
f+ f≤ f. (∗)
x y x

Considérons maintenant une subdivision σ de [x, z ]. En lui ajoutant le point y (s’il ne fait
pas déjà parti de σ ), on obtient une subdivision σ  de [x, z ] qui vérifie Varσ (f ) ≤ Var σ (f ). On
peut noter σ sous la forme
σ : x = x0 < x 1 < · · · < xp = y = y 0 < · · · < yq = z.
Considérons alors les subdivisions
σ1 : x = x0 < x 1 < · · · < x p = y ∈ sub([x, y ]) σ2 : y = y0 < y1 < · · · < yq = z ∈ sub([y, z ]).
On a z y
Var σ(f ) ≤ Varσ  (f ) = Varσ1(f ) + Var σ2 (f ) ≤
f+ f.
x y
z y 
Ceci étant vrai pour toute subdivision σ de [x, z], on en déduit x f ≤ x f + zyf , d’où le
résultat avec (*).
c) Soit σ : a = x0 < x1 < · · · < xn = b une subdivision de [a, b]. Pour tout i, on a
 xi+1   x i+1
 

|g (xi+1) − g (x i)| =  g (t) dt  ≤

|g (t)| dt
xi xi
b 
donc par sommation sur i, Varσ (g ) ≤ a |g (t)| dt. Ceci étant vrai pour tout σ ∈ sub([a, b]) on
en déduit que g est à variation bornée et que
b  b
g≤ |g (t)| dt. (∗∗)
a a
Il nous faut maintenant prouver l’inégalité réciproque. Si σ : a = x 0 < x1 < · · · < xn = b
est une subdivision de [a, b], on peut écrire pour tout i, en vertu du théorème des accroissements
finis,
g (xi+1 ) − g (xi ) = (xi+1 − xi )g (θi ) avec θi ∈ ]xi, xi+1 [,
de sorte que
n−1

Varσ (g) = (x i+1 − x i )|g (θi )| avec ∀i, θ i ∈ ]xi, xi+1 [.
i=0
Cette dernière expression est une somme de Riemann relative à la subdivision σpour la fonction
b
|g |. En faisant tendre le pas de σ vers 0, on voit donc que Varσ (g) tend vers a |g (t)| dt, d’où
b  b
a |g (t)| dt ≤ a g. On en déduit avec (**) le résultat.
2/ a) Condition suffisante. Une fonction croissante ϕ : [a, b] → R est à variation bornée car
pour toute subdivision σ : a = x0 < x1 < · · · < xn = b de [a, b],
n−1
 n−1

Varσ (f ) = |f (x i+1) − f (xi )| = [f (xi+1 ) − f (xi )] = f (b) − f (a).
i=0 i=0
La différence de deux fonctions à variation bornée étant à variation bornée (c’est immédiat), on
en déduit que f = g − h est à variation bornée. 
Condition nécessaire. Soit g : [a, b] → R x → xa f . D’après le résultat de la question 1/ b), g
est une fonction croissante. Posons h = g − f . La fonction h est croissante car
y y
∀x < y, h(y ) − h(x) = f − [f (y ) − f (x)] ≥ 0 car |f (x) − f (y)| ≤ f
x x
120 2. FONCTIONS D’UNE VARIABLE RÉELLE

(cette dernière assertion est vraie car σ : x < y est une subdivision de [x, y ]). Ainsi, f = g − h
est la différence de deux fonctions croissantes.
b) D’après la question précédente, on peut écrire f = g − h où g et h sont deux fonctions
croissantes. Une fonction monotone est réglée (voir la conséquence du théorème 4 page 99),
donc f , différence de deux fonctions réglées, est une fonction réglée.
3/ Considérons pour tout entier n ≥ 2 la subdivision de [0, 1] définie par
1 1 1 1
σn : 0 < < < ··· < < < 1.
nπ (n − 1)π 2π π
1
Pour tout k on a f ( kπ ) = cos(kπ )/(kπ) = (−1)k /(kπ) donc

 
n−1
1
  
1  1 
n−1 
1 1
 n−1
1 2
Varσn (f ) ≥ 
f (k + 1)π − f kπ  = π k+1
+
k

π k+1
.
k=1 k=1 k=1
 1
La série k k+1 diverge, donc l’ensemble (Varσ ) σ∈sub([0,1]) n’est pas majoré, ce qui prouve que
f n’est pas à variation bornée.

Problème 15 (Fonctions presque-périodiques). Soit E un espace vectoriel normé


(sur R ou C). On note F l’ensemble des fonctions de R dans E . Pour tout f ∈ F , on note
f  ∞ = sup f (x) ∈ R+ ∪ {+∞} et ∀τ ∈ R, fτ : R → E x → f (x + τ ).
x∈R

On dit que f ∈ F est presque-périodique si f est continue sur R et si


∀ε > 0, ∃Λ > 0, ∀α ∈ R, ∃τ ∈ [α, α + Λ[, f − fτ ∞ < ε. (∗)
On note P le sous-ensemble de F constitué des fonctions presque-périodiques.
a) Donner une classe de fonctions classique incluse dans P .
b) Si f ∈ P, montrer que f est bornée et uniformément continue sur R.
c) On note B le s.e.v de F constitué des fonctions continues et bornées, et on munit B
de la norme  . ∞ . Pour tout f ∈ B, on note A(f ) = {f a , a ∈ R}. Si f ∈ B, montrer
l’équivalence
(f ∈ P) ⇐⇒ (A(f ) est précompact dans B).
(La précompacité est définie dans la preuve du théorème de Bolzano-Weierstrass, page 29).
d) Soient f, g ∈ P. Montrer que f + g ∈ P et f g ∈ P si E est une algèbre normée.
e) Montrer que P est une partie fermée de B.

Solution. a) Il est clair que la classe des fonctions continues périodiques est incluse dans P (il
suffit, dans (*), de choisir Λ égal à la période).
b) Une fonction presque-périodique f est bornée. En effet, en choisissant ε = 1 dans (*) et en
considérant le Λ correspondant, on a pour tout nombre réel x
∃y ∈ [x − Λ, x[, f − fy ∞ < 1
donc f (x − y) − f (x) < 1 ce qui entraı̂ne
f (x) < 1 + f (x − y) ≤ 1 + sup f (t) car x − y ∈ [0, Λ],
t∈[0,Λ]

et ceci pour tout x ∈ R d’où le caractère borné de f .


Montrons maintenant qu’une fonction presque-périodique f est uniformément continue. Soit
ε > 0 et considérons le Λ correspondant dans (*). La fonction f étant continue sur le compact
[0, Λ + 1], elle y est uniformément continue donc
∃η ∈ ]0, 1[, ∀x, x ∈ [0, Λ + 1], |x − x | < η, f (x) − f (x ) < ε.
4. PROBLÈMES 121

Considérons alors deux réels x, x tels que |x − x| < η, avec x ≤ x . La fonction f étant presque
périodique,
∃τ ∈ [x − Λ, x[, f − f τ ∞ < ε.
Ceci entraı̂ne
f (x − τ ) − f (x) < ε et f (x − τ ) − f (x ) < ε,
donc comme x − τ et x  − τ appartiennent à [0, Λ + 1],
f (x) − f (x ) ≤ f (x) − f (x − τ ) + f (x − τ ) − f (x − τ ) + f (x − τ ) − f (x) < ε + ε + ε = 3ε.
Ceci est vrai pour tout couple (x, x) de réels tel que |x − x | < η, d’où l’uniforme continuité de
f sur R.
c) Condition nécessaire. Soit f ∈ P, ε > 0 et le Λ correspondant dans (*). Comme f est
uniformément continue sur R, on voit que
∃(η1, . . . , ηn ) ∈ [0, Λ] n , ∀y 0 ∈ [0, Λ], ∃i, f y0 − fη i ∞ < ε. (∗∗)
Maintenant, soit y ∈ R. D’après (*), il existe τ ∈ [−y, −y + Λ[ tel que f − f τ∞ < ε. Ceci
s’écrit aussi fy − f ρ  ∞ < ε avec ρ = τ + y ∈ [0, Λ[. D’après (**), il existe i ∈ {1, . . . , n} tel que
fρ − fη i ∞ < ε, et on en déduit
f y − f ηi ∞ ≤ fy − f ρ∞ + fρ − f η i ∞ < ε + ε = 2ε.
n

En d’autres termes on a A(f ) ⊂ B(fηi , 2ε), d’où la précompacité de A(f ) dans B.
i=1
Condition suffisante. Soit ε > 0. Par hypothèse, il existe une famille finie (fa i )1≤i≤n de A(f )
telle que
n
A(f ) ⊂ B(f ai , ε). (∗∗∗)
i=1
Posons Λ = 1 + 2µ où µ = sup{|a1|, . . . , |an |} et considérons maintenant un réel quelconque α.
D’après (***),
∃i ∈ {1, . . . , n}, f α+µ − fai ∞ < ε ce qui équivaut à fα+µ−a i − f ∞ < ε.
Ainsi, pour tout réel α, nous avons trouvé y ∈ [α, α+Λ[ (ici y = α + µ − ai) tel que f y − f ∞ < ε.
Ceci suffit pour montrer que f est presque-périodique.
d) D’après la question précédente, A(f ) et A(g ) sont précompacts. Ainsi, si on considère ε > 0,
il existe deux familles finies (f ai ) 1≤i≤m de A(f ) et (gbj ) 1≤j ≤n de A(g ) telles que
m
 n

A(f ) ⊂ B(f ai , ε) et A(g ) ⊂ B(g bj , ε).
i=1 j=1

On en conclut 
A(f + g) ⊂ A(f ) + A(g ) ⊂ B(f ai + gbj , 2ε),
1≤i≤m
1≤j ≤n

donc A(f + g) est précompact, donc f + g ∈ P d’après la question précédente.


Supposons maintenant que E soit une algèbre normée. Comme f et g sont bornées, M =
f ∞ et N = g  ∞ sont finis. Soient a ∈ R et i, j tels que fa − f ai ∞ < ε et ga − gb j ∞ < ε.
L’inégalité

∀x ∈ R, faga (x) − fai gbj (x) ≤ fa g a (x) − fai g a(x) + fai ga (x) − fai g bj(x)
≤ f a(x) − fa i(x) · ga (x) + f ai (x) · g a(x) − gbj (x) ≤ N ε + M ε
montre que

A(f g) ⊂ B(fai gb j , (M + N )ε),
1≤i≤m
1≤j ≤n

donc comme précédemment A(f g) est précompact, donc f g ∈ P.


122 2. FONCTIONS D’UNE VARIABLE RÉELLE

e) Soit (fn) n∈N une suite de P qui converge vers f ∈ B. Montrons que f ∈ P. Soit ε > 0 et
N ∈ N tel que f − fN∞ < ε. Comme fN est presque-périodique,
∃Λ > 0, ∀α ∈ R, ∃τ ∈ [α, α + Λ[, (fN ) − (fN ) τ ∞ < ε
donc
f − fτ ∞ ≤ f − f N ∞ + (fN ) − (f N )τ  ∞ + (fN ) τ − fτ ∞ < ε + ε + ε = 3ε,
ce qui suffit pour prouver que f ∈ P.
Remarque. Les fonctions presque périodiques ont été introduites par Bohr au début du
vingtième siècle, dans le cadre de l’étude des séries de Dirichlet a n /ns , et généralisent
les séries de Fourier. On peut  Tmontrer que pour toute fonction presque périodique f ,
1
la limite M (f ) = limT →∞ T 0 f (t) dt existe (on l’appelle valeur moyenne de f ). Une
fonction complexe f est presque périodique si et seulement si on peut écrire
+∞

f (t) = an eiλ nt , avec ∀n ∈ N an ∈ C, λn ∈ R,
n=0
où la convergence est uniforme sur R (la condition suffisante de ce résultat est une
conséquence
des résultats démontrés dans ce problème), et on a l’équivalent de l’inégalité
de Bessel |a n| 2 ≤ M (f 2 ). Un exemple typique de fonction presque périodique est
t → ζ (σ + it) (avec σ > 1 donné). Ceci entraı̂ne par exemple que pour tout ε > 0 et
pour tout T > 0, il existe t > T tel que ζ (σ ) − ε < |ζ (σ + it)| ≤ ζ (σ ).
CHAPITRE 3

Intégration

Lasurface.
théorie de l’intégration naquit avec la recherche du calcul de l’aire d’une
Archimède savait déjà évaluer l’aire d’une surface délimitée par une
parabole et une droite. Ses calculs furent repris au neuvième siècle par les
savants arabes. Dès 1636, Pierre de Fermat carra les courbes x → axm où m
est un entier naturel.
Au cours de la seconde moitié du XVIIe siècle, Newton et Leibniz fondèrent
le calcul infinitésimal. Newton calcula l’aire d’une courbe y = f (x) en inver-
sant les opérations de dérivation (aujourd’hui on dirait : en utilisant la notion
de primitive). A l’inverse, Leibniz interpréta les aires comme des sommes de
rectangles infinitésimaux.
En 1823, Cauchy rassembla leur résultats et donna le premier une défini-
tion précise de l’intégrale. C’est surtout Riemann qui, en 1854, développa la
théorie de l’intégration. Il définit son intégrale à l’aide des fameuses “sommes
de Riemann”.
Enfin, Lebesgue, dans sa thèse de 1902, présenta des idées révolutionnaires
sur le concept d’intégrale. Il éclaira bien des difficultés des discussions du XIXe
siècle, et fournit un cadre général simplifié à de nombreux théorèmes, alors que
la théorie de Riemann multipliait les hypothèses et les conditions restrictives.

L’intégrale de Lebesgue n’est pas au programme des classes préparatoires scientifiques,


et nous étudierons ici l’intégrale de fonctions continues par morceaux.

1. Intégrale sur un segment de R


Nous traiterons directement le cas des fonctions à valeurs dans un espace de Banach
(c’est-à-dire un e.v.n complet), la théorie étant presque identique à celle des fonctions à
valeurs réelles. Dans cette section, la lettre E désigne un espace de Banach sur K (avec
K = R ou C), [a, b] un segment de R non réduit à un singleton.
1.1. Définition de l’intégrale sur un segment de R
Intégrale des fonctions en escalier.
Définition 1. On appelle subdivision de [a, b] toute partie finie de [a, b] contenant a et b.
Si σ est une subdivision de [a, b], on peut écrire σ = {x0 , x 1, . . . , xn } avec a = x0 < x1 <
. . . < x n = b. C’est en général la notation employée pour désigner une subdivision.
On appelle pas (ou module ) de la subdivision σ et on note |σ | le réel sup1≤i≤n (xi −xi−1).
Définition 2. Une application ϕ : [a, b] → E est dite en escalier s’il existe une subdi-
vision de [a, b]
σ : a = x0 < x 1 < . . . < xn = b
telle que pour tout i ∈ {1, . . . , n}, ϕ soit constante sur ]xi−1, x i[. Une telle subdivision σ
est dite alors bien adaptée à ϕ.
Définition 3 (Intégrale d’une fonction en escalier). Soit ϕ : [a, b] → E une fonction en
escalier. Soit σ : a = x0 < x 1 < · · · < xn = b une subdivision de [a, b] bien adaptée à ϕ,
124 3. INTÉGRATION

de sorte que pour tout i ∈ {1, . . . , n}, il existe ci ∈ E telle que ϕ(x) = ci sur ]x i−1, xi [. La
valeur
n

I(σ, ϕ) = (x i − x i−1)ci
i=1

est indépendante de la subdivision σ adaptée à ϕ. On la note alors I (ϕ) ou encore [a,b]
ϕ
b
ou a ϕ(x) dx, et on l’appelle intégrale de ϕ.

Remarque 1. — Toute fonction en escalier sur [a, b] à valeurs dans K est combinaison
linéaire de fonctions caractéristiques de segments contenus dans [a, b].
— L’ensemble E ([a, b], E) des fonctions en escalier sur [a, b] est un K-e.v, et l’applica-
tion E → E ϕ → I (ϕ) est linéaire.
— Pour toute fonction en escalier ϕ, ϕ est une fonction en escalier et I (ϕ) ≤
I (ϕ).
— Si ϕ et ψ sont en escalier, à valeurs réelles, et si ϕ ≤ ψ , alors I (ϕ) ≤ I (ψ).
Intégrale d’une fonction continue par morceaux.
Bien qu’on puisse définir l’intégrale de classes de fonctions beaucoup plus générales,
nous nous limiterons à l’intégrale des fonctions continues par morceaux sur un segment
[a, b] ⊂ R dont la définition se trouve page 98.
 Définition 4 (Intégrale d’une fonction continue par morceaux). Soit une
fonction f : [a, b] → E continue par morceaux. Alors tout n ∈ N∗ , il existe une fonction
en escalier ϕn : [a, b] → E telle que f − ϕ n < 1/n sur [a, b]. La suite (un ) définie par
b
un = a ϕn (t) dt est alors une suite de Cauchy dans E , donc convergente. Sa limite ne
b 
dépend pas du choix des fonctions en escaliers ϕn , on la note a f (t) dt (ou encore [a,b] f )
et on l’appelle intégrale de f .
Démonstration. Comme f est continue par morceaux, c’est une fonction réglée, donc pour tout
n ∈ N∗ il existe bien une fonction en escalier ϕ n vérifiant f − ϕn  < 1/n sur [a, b] (voir la
proposition 5 page 99). La suite (un ) vérifie bien le critère de Cauchy car d’après la remarque 1,
et comme ϕp − ϕ q ≤ ϕp − f  + f − ϕ q ≤ 1/p + 1/q, on a
∀p, q ∈ N, p, q ≥ n up − u q = I (ϕ p − ϕq ) ≤ I (ϕp − ϕ q ) ≤ I (2/n) = 2(b − a)/n.
Comme E est complet par hypothèse (c’est un espace de Banach), la suite (u n) converge. On
note  sa limite.
Unicité de la limite. Soit (ψn ) une autre suite de fonctions en escalier vérifiant f − ψ n < 1/n
sur [a, b]. On note  la limite de vn = I (ψn ). L’inégalité
ψ n − ϕn  ≤ ψ n − f  + f − ϕ n ≤ 2/n
entraı̂ne
∀n, vn − u n = I (ψn − ϕ n) ≤ I (ψ n − ϕ n ) ≤ I (2/n) = 2(b − a)/n
donc limn→∞ u n − v n  = 0. Donc  = . 

Remarque 2. — Lorsque f est à valeurs réelles, on peut définir


I −(f ) = sup I (ϕ) et I +(f ) = inf I (ψ ),
ϕ∈E ψ∈E
ϕ≤f ψ≥f

où E désigne l’ensemble des fonctions en escalier de [a, b] dans R. Lorsque f est
continue par morceaux, on a I −(f ) = I + (f ), et cette valeur commune est un moyen
équivalent de définir l’intégrale de f . La définition 4 permet de ne pas se limiter
au cadre où E = R et donne une définition intrinsèque de l’intégrale sur tout e.v.n
complet, en particulier sur C et sur tout e.v de dimension finie (voir la remarque 3).
1. INTÉGRALE SUR UN SEGMENT DE R 125

— Lorsque f et g sont deux fonctions continues par morceaux qui diffèrent seulement
en un nombre fini de points, leurs intégrales sont identiques.
— L’intégrale d’une fonction en escalier donnée par la définition 4 est cohérente avec
celle donnée dans la définition
 b 3. a
— Lorsque a > b, on définit a f (x) dx = − b f (x) dx.
— La définition précédente s’étend facilement pour définir l’intégrale d’une fonction
réglée. On peut de manière plus générale définir les fonctions Riemann-intégrables,
qui sont les fonctions f telles que pour tout ε > 0, on peut trouver ϕ et µ en
escalier telles que f − ϕ < µ et I (µ) < ε. En faisant tendre ε vers 0, les valeurs
I (ϕ) convergent vers une valeur unique appelée intégrale de Riemann de f . Toute
fonction continue par morceaux, toute fonction réglée, est Riemann-intégrable.

Proposition 1. Soient E un K-e.v.n de dimension finie (toujours avec K = R ou C),


n= (e1, . . . , e n ) une base de E et f : [a, b] → E une application. On peut écrire f =
B
i=1 f i e i où pour tout i, l’application f i prend ses valeurs dans K. L’application f est
continue par morceaux si et seulement si chacune des applications f i est continue par
morceaux et on a alors
 b n 
 b 
f (x) dx = f i (x) dx e i.
a i=1 a

Remarque 3. — En particulier, C est un R-e.v dont (1, i) est une base. L’application
f : [a, b] → C s’écrit sous la formef = f1 + if2 ( où f1 , f2 sont à valeurs réelles) et
b b b
on a a f (x) dx = a f 1 (x) dx + i a f2(x) dx .
— On peut définir l’intégrale d’une fonction f à valeurs dans un R-e.v de dimension
finie E à partir de l’intégrale des fonctions à valeurs réelles, en  procédant comme
n
suit : on considère une base B = (e1 , . . . , en ) de E , on
 écrit f =  i=1 fi ei où l es if
b n  b
sont à valeurs réelles, et on pose a f (x) dx = i=1 a fi (x) dx e i. Il faut ensuite
vérifier que cette définition ne dépend pas de la base B choisie. L’avantage de la
définition que nous avons adoptée (voir définition 4) est qu’elle est intrinsèque (i. e
elle ne privilégie pas de base).
1.2. Propriétés des intégrales
Nous commençons par donner pêle-mêle les propriétés les plus élémentaires.
— Relation de Chasles. Soit f : [a, b] → E continue par morceaux et c ∈ ]a, b[. On a
b c b
a
f (x) dx = a f (x) dx + c f (x) dx.
— Linéarité de l’intégrale. L’ensemble Cm ([a, b], E) des fonctions continues par mor-
b
ceaux sur [a, b] est un K-e.v et l’application Cm([a, b]) → E ϕ → a ϕ(x) dx est
linéaire.
— Positivité de l’intégrale. Si f, g : [a, b] → R sont continues par morceaux et si f ≥ g
b b
sur [a, b], alors a f (x) dx ≥ a g (x) dx (le cas de l’inégalité stricte est plus délicat ;
voir la proposition 4).
b b
— Si  .  est une norme sur E , alors  a f (x) dx ≤ a f (x) dx.

Proposition 3. Soit (fn ) une suite de fonctions continues par morceaux de [a, b] dans
E qui converge uniformément sur [a, b] vers une fonction continue par morceaux f . Alors
 b  b
lim f n(x) dx = f (x) dx. (∗)
n→+∞ a a
126 3. INTÉGRATION

Remarque 4. Le théorème de convergence dominée (voir théorème 3 page 151) offre un


cadre beaucoup plus commode pour obtenir la convergence d’intégrales d’une suite de
fonctions, et c’est ce dernier que l’on utilise le plus souvent.
Proposition 4. Soient f et g : [a, b] → R deux fonctions continues. Si f ≥ g sur [a, b]
b b
et s’il existe c ∈ [a, b] tel que f (c) > g(c), alors a f (x) dx > a g (x) dx.
Démonstration. On pose γ = f (c) − g (c) > 0. La continuité de f − g en c entraı̂ne l’existence
d’un segment non réduit à un singleton [α, β] contenant c tel que f (t) ≥ g (t) + γ/2 pour tout
t ∈ [α, β]. Ainsi,
β  β  β  β
γ (β − α)γ
f (x) dx ≥ g (x) + dx = g (x) dx + > g (t) dt.
α α 2 α 2 α
Comme f ≥ g sur [a, b], on a par ailleurs
α  α  b  b
f (x) dx ≥ g (x) dx et f (x) dx ≥ g (x) dx.
a a β β
On en déduit facilement le résultat avec la relation de Chasles. 

Normes et intégrales.
Théorème 1 (In égalité de Schwarz). Soient f, g : [a, b] → C deux applications
continues par morceaux. Alors
 b  2  b   b 
 
 f (x)g (x) dx ≤ 2
|f (x)| dx · 2
|g (x)| dx .

a a a
Si f et g sont continues et f non identiquement nulle, cette inégalité est une égalité si et
seulement s’il existe α ∈ C tel que g (x) = αf (x) pour tout x ∈ [a, b].
Démonstration. Désignons par C m([a, b]) l’algèbre des fonctions continues par morceaux de [a, b]

dans C, et considérons la forme hermitienne positive Φ : Cm ([a, b]) → R f → ab f (x)f (x) dx,
dont la forme polaire est
b
2
ϕ : Cm ([a, b]) → C (f, g ) → f (x)g (x) dx
a
(voir le tome Algèbre). L’inégalité de Schwarz appliquée à ϕ donne
∀f, g ∈ Cm ([a, b]), |ϕ(f, g)| 2 ≤ Φ(f )Φ(g ),
d’où la première assertion du théorème. La restriction de Φ à l’e.v des fonctions continues sur
[a, b] est définie, et on sait alors que l’inégalité de Schwarz est une égalité si et seulement si f et
g forment une famille liée, d’où la seconde assertion du théorème. 
Conséquence : Sur l’e.v C([a, b], K) des fonctions continues sur [a, b], les applications

 b  b
N1 (f ) = |f (t)| dt, N2 (f ) = |f (t)|2 dt, N ∞(f ) = sup |f (t)|
a a t∈[a,b]

sont des normes. L’inégalité de Schwarz entraı̂ne que N2 vérifie bien l’inégalité triangu-
laire ; la nullité de N1(f ) ou N2 (f ) entraı̂ne bien celle de f d’après la proposition 4.
(i) La norme N1 s’appelle norme de la convergence en moyenne.
(ii) La norme N2 s’appelle norme de la convergence en moyenne quadratique.
(iii) La norme N∞ (encore notée  · ∞ ) s’appelle norme de la convergence uniforme.
Ces normes vérifient les inégalités

N1 (f ) ≤ b − a N2 (f ) ≤ (b − a)f ∞ .
1. INTÉGRALE SUR UN SEGMENT DE R 127

x
Étude de la fonction x → a
f (t) dt.

 Théorème 2. Soit f : [a, b] → E une fonction continue par morceaux sur [a, b]. Alors
x
l’application F : [a, b] → E x → a f (t) dt est C1 par morceaux et continue sur [a, b]. De
plus, F est dérivable à gauche et à droite en tout point x de I , et on a Fg (x) = lim t→x f (t)
t<x
et F d(x) = lim t→x f (t). En particulier, si f est continue sur [a, b] alors F est de classe C1
t>x
sur [a, b] et F  (x) = f (x) pour tout x ∈ [a, b].
Corollaire 1. Toute application continue f : [a, b] → R admet au moins une primitive
b  b
F , et pour toute primitive F de f, on a a f (x) dx = F = F (b) − F (a).
a
C’est ce dernier résultat qui amène à rechercher des primitives d’une fonction pour
calculer son intégrale. Ce problème sera étudié plus particulièrement dans la partie 2 de
ce chapitre. En l’appliquant à f = uv  + u  v dont la primitive est F = uv , on obtient le
résultat qui suit.
 Théorème 3 (Intégration par parties). Soient u, v : [a, b] → C deux fonctions de
classe C 1 . Alors b  b  b

u(x)v (x) dx = u · v − u(x)v (x) dx.
a a a
Citons enfin un dernier résultat, particulièrement utilisé lors de calculs de primitives.
 Théorème 4 (Changement de variable). Soit ϕ : [a, b] → R une application de
classe C1 et f : I ⊂ R → E une application continue par morceaux telle que ϕ([a, b]) ⊂ I
( où I est un intervalle de R). Alors
b  ϕ(b)
  
f ϕ(t) ϕ (t) dt = f (u) du.
a ϕ(a)

Conséquence : En conjuguant le théorème du changement de variable avec la relation de


Chasles, on obtient les résultats qui suivent.
— Soit f une application f : [−a, a] → E une continue par morceaux. Si f est paire,
a a a
alors −a f (x) dx = 2 0 f (x) dx. Si f est impaire, alors −a f (x) dx = 0.
— Soit f : R → E une application continue par morceaux et T -périodique. Alors
 a+T  T
∀a ∈ R, f (t) dt = f (t) dt.
a 0

Première et seconde formule de la moyenne.


Théorème 5 (Première formule de la moyenne). Soit f : [a, b] → R une fonction
continue et g : [a, b] → R+ une fonction continue par morceaux et positive. Alors
 b  b
∃c ∈ [a, b], f (x)g (x) dx = f (c) g (x) dx.
a a
Démonstration. Posons m = inf x∈[a,b] f (x) et M = supx∈[a,b] f (x). On a
b  b  b
m g (x) dx ≤ f (x)g (x) dx ≤ M g (x) dx.
a a a
b b
Si a g = 0, l’inégalité précédente montre que a f g = 0 et le résultat est évident. Sinon, on a
 
m ≤ ( ab f g)/( ab g ) ≤ M et on conclut en appliquant le théorème des valeurs intermédiaires à
la fonction continue f . 
Remarque 5. Attention, la fonction g doit être positive.
128 3. INTÉGRATION

Théorème 6 (Seconde formule de la moyenne). Soit f : [a, b] → R une fonction


positive décroissante de classe C1 et g : [a, b] → R une fonction continue. Alors il existe
c ∈ [a, b] tel que
 b  c
f (t)g (t) dt = f (a) g (t) dt.
a a
t
Démonstration. L’application G : [a, b] → R t → a
g (x) dx est de classe C 1 donc continue sur
[a, b]. Ceci assure l’existence des réels
m = inf G(t) et M = sup G(t).
t∈[a,b] t∈[a,b]
b
Montrons mf (a) ≤ a f (t)g (t) dt ≤ M f (a), ce qui prouvera le résultat en appliquant à G le
théorème des valeurs intermédiaires. En intégrant par parties, on a
 b  b  b  b

f (t)g (t) dt = G(t)f (t) − G(t)f (t) dt = G(b)f (b) − G(t)f  (t) dt.
a a a a
Or mf (b) ≤ G(b)f (b) ≤ M f (b) et
 b  b  b
   
 
m f (a) − f (b) = −m f (t) dt ≤ − G(t)f (t) dt ≤ −M f (t) dt = M f (a) − f (b) ,
a a a
b
donc finalement mf (a) ≤ a
f (t)g (t) dt ≤ M f (a). Ceci prouve le résultat en vertu de la conti-
nuité de G. 

Remarque 6. — Cette formule n’est pas au programme des classes préparatoires mais
elle peut rendre de précieux services (par exemplepour démontrer la convergence
+∞
de certaines intégrales semi-convergentes comme 1 sin(t)/t dt ; voir également
la règle d’Abel formulée dans le théorème 5). Il faut savoir refaire sa démonstration
qui est simple.
— Ce résultat reste vrai si on suppose uniquement f continue (voir l’exercice 8
page 135), et même si f et g sont uniquement supposées Riemann-intégrables.
1.3. Sommes de Riemann
Sommes de Riemann.
Notation. Soient f : [a, b] → E une application bornée, σ : a = x0 < x1 < · · · < x n = b
une subdivision de [a, b] et ξ = (ξ i)1≤i≤n une famille de n réels telle que ξ i ∈ [xi−1 , x i ] pour
tout i ∈ {1, . . . , n}. Le couple (σ, ξ ) s’appelle une subdivision pointée.
On appelle somme de Riemann de la fonction f pour la subdivision pointée (σ, ξ ) la
grandeur notée S(f, σ, ξ) définie par
n

S(f, σ, ξ) = (x i − xi−1)f (ξ i).
i=1

On rappelle que le pas de σ est le réel sup1≤i≤n (x i − x i−1), noté |σ |.

 Théorème 7. Soit une application f : [a, b] → E continue par morceaux. Pour tout
ε > 0, il existe α > 0 tel que pour toute subdivision pointée (σ, ξ ) de [a, b] vérifiant
|σ | < α, on ait
 b 
 
 f (x) dx − S (f, σ, ξ) ≤ ε.
 
a
Démonstration. Nous allons prouver le résultat en trois étapes.
Étape 1. Supposons que f soit de la forme f = χ[c,d] · e, où χ[c,d] est la fonction caractéristique
1. INTÉGRALE SUR UN SEGMENT DE R 129

d’un segment [c, d] inclus dans [a, b] et e ∈ E . Soit (σ, ξ ) une subdivision pointée de [a, b].
Écrivons
σ : a = x0 < x1 < · · · < xn = b, ξ = (ξ i)1≤i≤n .
On remarque que
n  xi   b  
n  xi

    
   
S(f, σ, ξ) = f (ξi ) dx donc S(f, σ, ξ) − f (x) dx ≤  (f (ξ i ) − f (x)) dx .
xi−1 a  x i−1 
i=1 i=1
Parmi les intervalles [xi−1, xi ] (1 ≤ i ≤ n), il y en a au plus deux sur lesquels f ne soit pas
constante. On en conclut facilement
  b 
 
S(f, σ, ξ) − f (x) dx 
  ≤ 2|σ| · e,
a
d’où le résultat.
Étape 2. Supposons que f soit une fonction en escalier. On peut écrire f comme une somme finie
de fonctions du type de celles traitées dans l’étape 1, et le résultat s’obtient ensuite facilement
par linéarité de l’intégrale et de l’application f → S (f, σ, ξ).
Étape 3. Traitons maintenant le cas général. Soit f : [a, b] → E une fonction continue par
morceaux. Soient ε > 0 et une fonction en escalier ϕ telle que f − ϕ ≤ ε sur [a, b]. L’étape
précédente nous assure l’existence d’un réel α > 0 tel que pour toute subdivision pointée (σ, ξ)
vérifiant |σ | < α, on ait  
 b
 
S(ϕ, σ, ξ) − ϕ(x ) dx  < ε,
 
a
Ainsi, pour une telle subdivision pointée (σ, ξ), on a
  b 
 
 S(f, σ, ξ) − f (x ) dx 
 
a
 b   b b 
   

≤ S(f, σ, ξ) − S (ϕ, σ, ξ) + S(ϕ, σ, ξ) −  
ϕ(x) dx  +  ϕ(x) dx − f (x) dx

a a a
 b
≤ S(f − ϕ, σ, ξ) + ε + ϕ(x) − f (x) dx
a
≤ (b − a)ε + ε + (b − a)ε = (1 + 2(b − a))ε.
d’où le théorème. 
Conséquence : Soit f : [a, b] → E une application continue par morceaux. Alors
n   n−1    b
b−a  b−a b−a  b−a
lim f a+i = lim f a+i = f (x) dx.
n→+∞ n i=1 n n→+∞ n
i=0
n a

Exemple 1. En appliquant ce dernier résultat à f : [0, 1] → R t → 1/(1 + t), on obtient


n   n  1  1
1 i  1 dt
lim f = lim = = log(1 + t) = log 2.
n→+∞ n n n→+∞ n+i 0 1+t 0
i=1 i=1

Remarque 7. - Le théorème 7 est également vrai sur les fonctions Riemann-intégrables.


Réciproquement, si E est un e.v.n de dimension finie, on peut même montrer qu’une
fonction est Riemann-intégrable si et seulement si ses sommes de Riemann “convergent”
lorsque le pas des subdivisions tend vers 0.
– Sous certaines hypothèses de régularité
 bsur f , il est possible de donner un développement
b−a
 b−a
asymptotique de n i f (a + i n ) − a f (x) dx. Ceci est un problème classique qu’il est
bon de savoir résoudre (voir l’exercice 6 et le sujet d’étude 3 page 321).
– Attention, le résultat de ce théorème n’est pas vrai pour les fonctions intégrables ou les
intégrales généralisées, sauf sous certaines hypothèses (voir l’exercice 5 page 156 qui est
classique).
130 3. INTÉGRATION

1.4. Exercices
 Exercice 1 (Int égrales de Wallis). Pour tout n ∈ N, on pose
 π/2
In = sinn x dx.
0

a) Donner une expression explicite de In pour tout n ∈ N.


b) En déduire la formule de Wallis
 2
1 2p(2p − 2) · · · 2
lim = π,
p→+∞ p (2p − 1)(2p − 3) · · · 1

π
puis montrer que lorsque n → +∞, In ∼ .
2n
Solution. a) En intégrant par parties, on a
 π/2  π/2  π/2
n−1 n−1
∀n ≥ 2, In = sin x sin x dx = − sin x cos x +(n −1) sinn−2 x cos2 x dx
0 0 0
n−1
= (n − 1)(In−2 − I n) d’où In = I n−2 .
n
 π/2
Comme I0 = π/2 et I 1 = − cos x = 1, on en déduit
0
(2p − 1)(2p − 3) · · · 1 π 2p(2p − 2) · · · 2
∀p ∈ N∗, I 2p = et I2p+1 = . (∗)
2p(2p − 2) · · · 2 2 (2p + 1)(2p − 1) · · · 1

b) En remarquant que
 π

∀p ∈ N , ∀x ∈ 0, , 0 ≤ sin2p+1 x ≤ sin 2p x ≤ sin 2p−1 x,
2
on tire, par intégration
I2p I2p−1 2p + 1
∀p ∈ N ∗, I2p+1 ≤ I2p ≤ I 2p−1 donc 1 ≤ ≤ = ,
I2p+1 I2p+1 2p
la dernière égalité étant une conséquence de (*). Par conséquent
I2p
lim =1
p→+∞ I 2p+1
et on en déduit la formule de Wallis avec la formule (*).
De la formule de Wallis, on déduit
(2p − 1)(2p − 3) · · · 1 1
∼ √
(2p)(2p − 2) · · · 2 pπ
donc grâce à (*), on tire
 
1 π 1 π
I 2p ∼ et I 2p+1 ∼ I 2p ∼ .
2 p 2 p
On en déduit l’équivalent demandé.

Exercice 2. Soit f : [a, b] → R une fonction positive et continue sur [a, b].
a) Montrer que
 b 1/n
n
lim f (t) dt = M où M = sup f (t).
n→+∞ a t∈[a,b]
1. INTÉGRALE SUR UN SEGMENT DE R 131

b) Soit g : [a, b] → R une fonction continue prenant des valeurs strictement positives sur
[a, b]. Montrer que
 b 1/n
n
lim f (t) g (t) dt = M où M = sup f (t).
n→+∞ a t∈[a,b]

Solution. a) L’inégalité f (t)n ≤ M n pour tout t ∈ [a, b] entraı̂ne


 b 1/n  b  1/n
∗ n n
∀n ∈ N , f (t) dt ≤ M = (b − a)1/n M. (∗)
a a
Par ailleurs, f étant continue sur le compact [a, b], il existe c ∈ [a, b] tel que f (c) = M. Pour
tout ε > 0, il existe donc un segment [α, β ] contenant c, non réduit à un singleton, tel que
∀t ∈ [α, β ], f (t) ≥ M − ε.
Ceci entraı̂ne
 b 1/n  β 1/n  β  1/n
n n n
f (t) dt ≥ f (t) dt ≥ (M − ε) dt = (β − α)1/n(M − ε). (∗∗)
a α α

Comme limn→+∞ µ1/n = 0 pour tout réel µ > 0, on tire de (*) et (**)
 b 1/n
∗ n
∃N ∈ N , ∀n ≥ N, M − 2ε ≤ f (t) dt ≤ M + ε.
a
Ceci étant possible pour tout ε > 0, on en déduit le résultat.
b) La fonction g étant continue sur un compact, elle est bornée et atteint ses bornes. On en
conclut que les réels
k = inf g (t) et K = sup g (t)
t∈[a,b] t∈[a,b]

existent et sont strictement positifs. Par ailleurs, le fait que kf (t)n ≤ g (t)f (t)n ≤ Kf (t) n pour
tout t ∈ [a, b] montre que
 b 1/n  b 1/n  b  1/n
1/n n n 1/n n
k f (t) dt ≤ f (t) g (t) dt ≤K f (t) dt .
a a a
1/n
On conclut avec la question précédente et avec les limites lim k = lim K 1/n = 1.
n→+∞ n→+∞

Exercice 3. Soit f : [a, b] → R une fonction de classe C 1 telle que f (a) = f (b) = 0. Soit
M = supt∈[a,b] |f (t)|.
a) Montrer
 b 
  (b − a)2
 f (t ) dt≤ M. (∗)
  4
a

b) Dans quels cas l’inégalité (*) est elle une égalité ?

Solution. a) Le problème est symétrique par rapport à c = (a + b)/2. On écrit


 t   t
 
∀t ∈ [a, c], 
|f (t)| =  f (x) dx ≤

|f (x)| dx ≤ M (t − a)
a a
et    b
 b 
∀t ∈ [c, b], |f (t)| =  f (x) dx ≤

|f (x)| dx ≤ M (b − t),
t t
132 3. INTÉGRATION

ce qui entraı̂ne
 b   b  c b 
  (b − a) 2
 f (t ) dt ≤ | f (t ) | dt ≤ M
 (t − a
 ) dt + (b − t ) dt = M.
  4
a a a c

b) Les fonctions en présence étant continues, l’égalité se produira si et seulement si chacune des
inégalités précédentes est une égalité. On a donc
∀t ∈ [a, c], |f (t)| = M (t − a) et ∀t ∈ [c, b], |f (t)| = M (b − t),
ce qui entraı̂ne
∀t ∈ [a, c], f (t)2 = M2 (t − a)2 et ∀t ∈ [c, b], f(t)2 = M2 (b − t)2. (∗∗)
La fonction f 2 est de classe C 1 , elle est donc dérivable en c = (a + b)/2. Les formules (**) donnent
 2  2
successivement (f2 ) (c) = 2M 2 b−2 a et (f 2 ) (c) = −2M 2 b−2a , ce qui n’est possible que si
M = 0. Donc f = f (a) = 0. Réciproquement, la fonction nulle est bien solution.

Exercice 4. Soient E un espace euclidien et f : [a, b] → E une fonction continue. On


suppose que  b  b
 
 f (t ) dt= f (t) dt.
 
a a
Montrer qu’il existe e ∈ E de norme 1 tel que f (t) = f (t) · e pour tout t ∈ [a, b].

Solution. Si ab f (t) dt = 0, les hypothèses et la continuité de f entraı̂nent que f est identique-
ment nulle, et le résultat est évident. Sinon, on pose
b  b  b 
f (t) dt  
e1 =  a  de sorte que f (t) dt = 
 f (t) dt 
 e1 .
 b 
 a f (t) dt a a

Le vecteur e1 est de norme 1. On le complète en une base orthonormale (e 1, . . . , e n) de E . On


peut écrire f = f1 e1 + · · · + fn e n où pour tout i, f i est une fonction continue à valeurs réelles.
On a b n  b   b   b 
  
f (t) dt = fi (t) dt ei =  
 f (t) dt  e1 = f (t) dt e 1,
a i=1 a a a
donc en extrayant la composante le long du vecteur e1
b  b
f1 (t) dt = f (t) dt. (∗)
a a
Comme f1 (t) ≤ f (t) = (f 1(t) 2 + · · ·+ fn (t)2 )1/2 et que chacune de ces fonctions sont continues,
l’égalité (*) entraı̂ne
∀t ∈ [a, b], f 1(t) = f (t) = (f1 (t)2 + · · · + fn (t)2 ) 1/2. (∗∗)
Donc f2 (t) = . . . = f n (t) = 0 pour tout t ∈ [a, b], d’où
∀t ∈ [a, b], f (t) = f 1(t)e 1 = f (t)e 1 ,
la dernière égalité étant encore une conséquence de (**).

Exercice 5. Soit f : [0, 1] → R une fonction de classe C1 telle que 0 ≤ f (t) ≤ 1 pour
tout t ∈ [0, 1]. Montrer que
 1  1 2
3
f (x) dx ≤ f (x) dx .
0 0
1. INTÉGRALE SUR UN SEGMENT DE R 133

Solution. On considère les fonctions


t  t
2
ϕ : [0, 1] → R t → f 3(x) dx et ψ : [0, 1] → R t → f (x) dx .
0 0
Il s’agit de montrer ϕ(1) ≤ ψ(1). Les fonctions ϕ et ψ sont de classe C 1 . Comme ϕ(0) = ψ (0) = 0,
ceci sera prouvé si on montre ϕ (x) ≤ ψ (x) pour tout x ∈ [0, 1], c’est-à-dire si
 x
3
∀x ∈ [0, 1], f (x) ≤ 2f (x) f (t) dt.
0
Comme f ≥ 0 (f (0) = 0 et f  est positive), ceci sera prouvé si on montre
x
2
∀x ∈ [0, 1], f (x) ≤ 2 f (t) dt.
0
Chacune de ces deux fonctions étant dérivable et nulle en 0, ce dernier point sera obtenu si on
prouve que la dérivée de la première est inférieure à celle de la seconde sur [0, 1], c’est-à-dire si
∀x ∈ [0, 1], 2f (x)f  (x) ≤ 2f (x).
Ce dernier point est bien réalisé car f est positive et f  ≤ 1 par hypothèse. D’où le résultat.
Remarque. L’égalité se produit lorsque f (x) = x ou lorsque f est la fonction nulle.

Exercice 6. a) Montrer que la suite (un )n∈N∗ définie par


n
∗ n
∀n ∈ N , un = .
k=1
n + k2
2

converge et calculer sa limite .


b) Donner un équivalent de  − un lorsque n tend vers l’infini.
1
Solution. a) En considérant la fonction f : [0, 1] → R x → , on remarque que
1 + x2
n  
1 k
∀n ∈ N∗ , un = f .
n n
i=1
Autrement dit, un est une somme de Riemann de f pour une subdivision de pas 1/n. On en
déduit avec le théorème 7 que (u n) converge et que
 1  1 π
dx
 = lim un = 2
= arctan x = .
n→+∞ 0 1+x 0 4
b) C’est classique. On procède comme suit. On considère une primitive F de f , de sorte que
pour tout n ∈ N ∗,
n   
   n  
k k−1 1  k
 = F (1) − F (0) = F −F et u n = F ,
n n n n
k=1 k=1
donc n       
 k k−1 1  k

∀n ∈ N ,  − un = F −F − F .
n n n n
k=1
D’après la formule de Taylor-Lagrange, pour tout n ∈ N∗ on a
       
k−1 k k−1 k 1  k 1
∀k ∈ {1 . . . , n}, ∃x k ∈ , , F =F − F + 2 F (xk ),
n n n n n n 2n
et comme F  = f  ceci entraı̂ne
n 
  n
−1  1  
 − un = 2
F (xk ) = − f (xk ).
2n 2n2
k=1 k=1
134 3. INTÉGRATION

On a encore affaire à une somme de Riemann ; le théorème 7 donne


n  1
1  1
lim f (x k ) = f (t) dt = f (1) − f (0) = − ,
n→+∞ n 0 2
k=1
1
donc  − u n ∼ lorsque n tend vers +∞.
4n
Remarque. On a montré, de manière plus générale, que pour toute fonction f : [0, 1] → R
de classe C1 1    
n
 k 1  1
f (t) dt = f − f (1) − f (0) + o .
0 n 2n n
k=1
Lorsque la fonction f possède de bonnes propriétés de régularité, il est possible de pour-
suivre ce développement asymptotique (voir le sujet d’étude 3 page 321 sur la formule
d’Euler-Maclaurin).

Exercice 7 (Inégalit é de Young). Soit f : R+ → R une fonction dérivable et


strictement croissante telle que f (0) = 0.
a) Pour tout x > 0, montrer
 x  f (x)
f (t) dt + f −1 (t) dt = xf (x).
0 0

b) En déduire que
 a  b
∀a, b > 0, f (t) dt + f −1(t) dt ≥ ab,
0 0
et que l’égalité se produit si et seulement si b = f (a).

Solution. a) Cette égalité a une interprétation géométrique simple comme le montre la figure
ci contre.

 f (x)
f −1(t) dt
0   (f )
f (x)

 x
f (t) dt
0

0 x
Figure 1. L’aire xf (x) du rectangle est la somme des surfaces des régions
délimitées par ce rectangle et le graphe de f .

On considère l’application
 x  f (x)
+
g : R → R x → f (t) dt + f −1 (t) dt − xf (x).
0 0
Cette application est dérivable, et
 
∀x ≥ 0, g (x) = f (x) + f (x)f −1 (f (x)) − f (x) + xf  (x) = 0.
1. INTÉGRALE SUR UN SEGMENT DE R 135

Donc g est constante, et comme g (0) = 0 on en déduit que g est nulle, d’où le résultat.
b) On fixe b > 0 et on considère l’application
 a b
+
ϕ : R → R a → f (x) dx + f −1 (x) dx − ab.
0 0

Cette application est dérivable, et ϕ (a) = f (a) − b pour tout a ∈ R. Comme f est strictement
croissante, on a donc
∀a < f −1 (b), ϕ (a) < 0 et ∀a > f −1(b), ϕ (a) > 0.
Comme ϕ(f −1(b)) = 0 d’après la question précédente, on déduit de tout ceci que
∀a < f −1 (b), ϕ(a) > 0, ϕ(f −1(b)) = 0, ∀a > f −1 (b), ϕ(a) > 0.
On a donc démontré l’inégalité désirée, et l’égalité se produit si et seulement si a = f −1 (b),
c’est-à-dire b = f (a), d’où le résultat.

Exercice 8. Démontrer que la seconde formule de la moyenne (théorème 6 page 128) reste
vraie si l’on suppose uniquement f continue. Autrement dit, montrer que si f : [a, b] → R
est une fonction continue, positive et décroissante, et si g : [a, b] → R est continue, alors
il existe c ∈ [a, b] tel que
 b  c
f (t)g (t) dt = f (a) g (t) dt.
a a

(on utilisera une transformation d’Abel à partir d’une expression approchant une somme
de Riemann)

Solution. Nous démarrons


t comme dans la preuve du théorème 6. On considère l’application
G : [a, b] → R t → a g (x) dx, continue sur [a, b], ce qui assure l’existence des réels
m = inf G(t) et M = sup G(t).
t∈[a,b] t∈[a,b]
b
On va montrer mf (a) ≤ a f (t)g (t) dt ≤ M f (a), ce qui prouvera le résultat en appliquant à G
le théorème des valeurs intermédiaires.
L’idée est de remplacer l’intégration par parties utilisée dans la preuve du théorème 6 par
une transformation d’Abel à partir d’une somme de Riemann. Par comparaison avec la formule
 b  b
f (t)g (t) dt = G(b)f (b) − G(a)f (a) − f (t)G(t),
a a
1
(valide uniquement lorsque f est C ) on va écrire, pour une subdivision σ de pas suffisamment
petit, avec σ : a = a 0 < · · · < a n = b, une transformation d’Abel de la somme
n−1

I (σ ) = f (ai)(G(ai+1 ) − G(ai )),
i=0

ce qui donne
n−1

I (σ ) = G(a n )f (an−1) − G(a 0)f (a0 ) + (f (a i−1 ) − f (a i))G(ai ).
i=1

Comme G(an )f (an−1) − G(a 0)f (a 0 ) = G(b)f (an−1 ), cette dernière identité entraı̂ne, compte
tenu de la décroissante de la fonction f
n−1
 n−1

G(b)f (an−1 ) + (f (ai−1 ) − f (ai ))m ≤ I (σ) ≤ G(b)f (a n−1) + (f (a i−1 ) − f (ai ))M,
i=1 i=1
136 3. INTÉGRATION

c’est-à-dire
G(b)f (a n−1) + m(f (a0 ) − f (an−1 )) ≤ I (σ) ≤ G(b)f (an−1 ) + M (f (a0 ) − f (an−1 ))
et comme f (an−1 ) ≥ 0 on en déduit
mf (a) ≤ I (σ ) ≤ M f (a). (∗)
 b
Maintenant nous allons approcher I (σ ) par une somme de Riemann de a f g. Soit ε > 0.
Comme g est continue sur un segment, elle y uniformément continue d’après le théorème de
Heine, donc il existe α > 0 tel que si |x − y | < α, alors |g (x) − g(y )| < ε. Ensuite, le théorème
sur les sommes de Riemann nous assure qu’on peut choisir la subdivision (σ) de pas |σ | < α
b
telle que |S (σ ) − a f g| < ε, où
n−1

S (σ ) = (ai+1 − ai )f (ai )g (ai ).
i=0
On écrit maintenant
n−1 n−1  
   a i+1 
|I (σ ) − S (σ )| ≤ f (a i)|G(ai+1 ) − G(ai) − (ai+1 − a i)g (a i)| = f (ai )  (g (t) − g (ai )) dt
i=0 i=0 ai

et le pas de la subdivision étant |σ | < α, on a |g (t) − g (ai )| < ε pour t ∈ [ai , ai+1 ] donc l’inégalité
précédente entraı̂ne
n−1
 n−1

|I (σ ) − S (σ )| ≤ f (a i )(ai+1 − ai)ε ≤ f (a)(ai+1 − a i )ε = (b − a)f (a)ε.
i=0 i=0
On en déduit
     
 b   b 
I (σ ) −  
f (t)g (t) dt ≤ |I (σ ) − S (σ )| + S (σ ) − f (t)g (t) dt  ≤ (b − a)f (a)ε + ε,

a a
et finalement, avec (*) on a
 b
mf (a) − (b − a)f (a)ε − ε ≤ f (t)g (t) dt ≤ M f (a) + (b − a)f (a)ε + ε.
a
 b
Cette inégalité étant vraie pour tout ε > 0, on en déduit bien mf (a) ≤ a
fg ≤ M f (b), d’où le
résultat.
Remarque. - On aurait également pu se ramener au cas du théorème 6 page 128 en
construisant une suite de fonctions (fn ) décroissantes positives de classe C1 , qui converge
uniformément vers f .
– On peut démontrer que la seconde formule de la moyenne est encore vraie si on suppose
seulement f et g Riemann-intégrables.

2. Calcul de primitives
b
La formule a f  (t) dt = f (b) − f (a) (pour f une fonction de classe C 1) nous amène à
calculer les primitives d’une fonction continuepour calculer son intégrale.
Lorsque F est une primitive de f , on note f (t) dt = F (t) + k . Cette notation signifie
que l’ensemble des primitives de f est constitué des fonctions de la forme t → F (t) + k où
k est une constante.
 Cette écriture peut même être valable sur plusieurs intervalles. Par
exemple, écrire dt/t = log |t| + k signifie que sur R +∗, les primitives de t → 1/t sont les
fonctions de la forme t → log t + k , et que sur R−∗ elles sont de la forme t → log(−t) + k.
1  1
Il faut alors se garder d’écrire −1 dt/t = log |t| , ce qui n’aurait aucun sens.
−1
2. CALCUL DE PRIMITIVES 137

2.1. Primitives élémentaires


Dans le tableau suivant, F (x) désigne une primitive de f (x).
f (x) F (x) f (x) F (x)
x α+1
x α, α =
 −1 α+1
coth x log( sh x)
1 1
x
log |x| ch 2x
th x
1
ex ex sh 2 x
− coth x
ax √ 1
a x, a > 0, a = 1 log a 1−x 2
arcsin x
sin x − cos x √ 1 ,a>0 arcsin xa
a 2−x2

cos x sin x √ 1 log(x + 1 + x2 )
1+x 2

tan x − log | cos x| √ 1 , a = 0 log(x + a2 + x 2 )
a 2+x2

cotan x log | sin x| √ 1 log |x + x 2 − 1|
x 2−1

1
tan x √ 1 , a = 0 log |x + x 2 − a2 |
cos2 x x 2−a2

1
− cotan x √ 1 , b = 0 log |x + x 2 + b|
sin2 x x 2+b
1
sh x ch x x2 +1
arctan x
1 1
ch x sh x x2 +a2
, a = 0 a
arctan xa
 x+a 
th x log( ch x) 1
, a = 0 1
log  
a2 −x2 2a x−a

Remarque. On peut aussi exprimer les primitives de t → 1/ t2 + 1 sous la forme t →
argsh t + k , car on a l’égalité bien connue

∀t ∈ R, log(t + t2 + 1) = argsh (t).

De même, lorsque t > 1, on peut aussi exprimer les primitives de t → 1/ t2 − 1 sous la
forme t → argch t + k . Cette expression n’est plus valable lorsque t < −1 car la fonction
argch n’est définie que sur [1, +∞[.
2.2. Primitives des fractions rationnelles

Soit F une fraction rationnelle de R(X ). Pour calculer F , on commence par décompo-
ser F en éléments simples sur R. On est ainsi ramené à calculer les primitives de la forme
 
dx ∗ ax + b
h
(h ∈ N ) et (c2 − 4d < 0, h ∈ N∗ ).
(x − a) (x + cx + d)h
2

– La première primitive se calcule facilement grâce à l’expression


 
1
dx (1−h)(x−a)h−1
+ k si h = 1
h
= .
(x − a) log |x − a| + k si h = 1

– Quant à la seconde, on commence par écrire

ax + b 2α(x − p) β
sous la forme + .
(x2 + cx + d)h 2 2
[(x − p) + q ] h [(x − p)2 + q2 ]h
138 3. INTÉGRATION

— La primitive du premier terme dans le membre de droite est donnée par


 α
  + k si h ≥ 2,
2α(x − p)
dx = (1 − h)[(x − p) 2 + q 2 ]h−1 .
[(x − p) 2 + q 2 ] h α log[(x − p) 2 + q2] + k si h = 1
— Pour celle du second terme, on commence par effectuer 2 le changement de variable
2 −h
t = x − p, ce qui ramène le calcul à celui de Ih = (t + q ) dt. Ensuite, deux
techniques sont à notre disposition.
— Première technique. On effectue le changement de variable
  −π π 
t = q tan θ, θ ∈ , de sorte que dt = q (1 + tan2 θ) dθ,
2 2
donc
  
dt 1 dθ 1
Ih = 2 2 h
= 2h−1 2 h−1
= 2h−1 cos2h−2 θ dθ.
(t + q ) q (1 + tan θ) q
Le calcul de cette dernière primitive est traité dans la section suivante. Après
l’avoir calculée, on remplace θ par arctan(t/q ) puis t par x − p et le tour est
joué.
— Seconde technique. On procède par intégration par parties. Pour tout h ∈ N∗ ,
on écrit

t t2 t
Ih = 2 2 h
+ 2h 2 2 h+1
= 2 2 h
+ 2hI h − 2hq 2Ih+1 ,
(t + q ) (t + q ) (t + q )
ce qui entraı̂ne
t
2hq2Ih+1 = (2h − 1)I h + . (∗)
(t 2 + q2)h
Cette dernière relation permet de calculer I h+1 connaissant Ih. Le calcul de I1
est immédiat car 
dt 1 x
= arctan .
t2 + q2 q q
Remarquez que pour ramener le calcul de I n à celui de I n−1, c’est I n−1 qu’il faut
intégrer par parties.

1−x
Exemple 1. On veut calculer la primitive dx. On commence par écrire
(x + x + 1)2
2

1−x 1 2x + 1 3 1
=− · 2 + ·  .
(x2 + x + 1)2 2 (x + x + 1) 2 2 (x + 1 ) 2 + 3 2
2 4

La primitive du premier terme du membre de droite se calcule de manière immédiate, car



2x + 1 1
dx = − + k.
(x2 + x + 1)2 x2 + x + 1
Pour celle du second terme, on utilise la relation (*) qui donne
 
3 dx dx x + 12
2· · = + ,
4 [(x + 12 )2 + 34 ]2 (x + 12 )2 + 34 (x + 12 )2 + 34
d’où     
dx 2 2 2x + 1 x + 12
1 2 3 2
= √ arctan √ + 2 .
[(x + 2 ) + 4 ] 3 3 3 x +x+1
2. CALCUL DE PRIMITIVES 139


1 3
(On pourrait aussi procéder en effectuant le changement de variable x + 2
= 2
tan θ ).
Finalement, on a

1−x x+1 2 2x + 1
2 2
dx = 2 + √ arctan √ + k.
(x + x + 1) x +x+1 3 3
2.3. Primitives des fonctions en sinus et cosinus

Polynômes en sinus et cosinus. On veut calculer les primitives sinm x cosn x dx,
où m, n ∈ N. Deux cas se présentent :
— L’un des entiers m ou n est impair (par exemple n = 2p + 1). On a alors
 
sin x cos x dx = sinm x(1 − sin2 x) p cos x dx.
m n

En effectuant
 m ensuite le changement de variable t = sin x, on se ramène à la primi-
tive t (1 − t 2) p dt qui est facile à calculer.
— Les entiers m et n sont pairs. On linéarise, en exprimant sinm x cosn x comme
combinaison linéaire de fonctions de la forme cos kx et sin kx. Par exemple, pour
calculer une primitive de cos4 x, on écrit

eix + e −ix
cos x = d’où
2  
4 1 e4ix + e−4ix e2ix + e −2ix cos 4x cos 2x 3
cos x = +4 +3 = + + ,
8 2 2 8 2 8
d’où on déduit facilement la primitive recherchée.
Fractions rationnelles en sinus et cosinus. On veut calculer une primitive d’une
fonction de la forme R(sin x, cos x) où R est une fraction rationnelle en deux variables.
On s’en sort toujours en effectuant le changement   de variable t2 = tan(x/2). Comme
2t 1−t 2 dt
dt = 12 (1 + t2 )dx, le calcul se ramène à celui de R , , c’est-à-dire
1 + t 1 + t 1 + t2
2 2
à celui d’une primitive d’une fraction rationnelle. En procédant de la sorte, on trouve les
primitives suivantes, qu’il faut retenir :
   x     x π 
dx   dx  
= log  tan  et = log tan + .
sin x 2 cos x 2 4
Cette méthode est souvent fastidieuse car elle amène à calculer des primitives de
fractions rationnelles dont le dénominateur est de degré élevé. On commence en général
par essayer d’effectuer l’un des changements de variable t = sin x, t = cos x ou t = tan x
qui simplifie parfois les calculs. On peut à ce sujet utiliser la règle de Bioche présentée ci
dessous :
— si R(sin x, cos x) dx reste inchangé en changeant x en π − x, on pose t = sin x ;
— si R(sin x, cos x) dx reste inchangé en changeant x en −x, on pose t = cos x ;
— si R(sin x, cos x) dx reste inchangé en changeant x en π + x, on pose t = tan x.
N’oubliez pas que le terme dx doit faire parti de l’expression invariante !

sin 3 x 
Exemple 2. On veut calculer dx. Le terme sous le signe reste invariant
1 + cos2 x
en changeant x en −x (il faut prendre en compte le dx), on pose donc t = cos x. On a
dt = − sin x dx, donc le calcul se ramène à celui de la primitive
   
1 − t2 2
(−dt) = 1− dt = t − 2 arctan t + k.
1 + t2 1 + t2
140 3. INTÉGRATION

Il reste à remplacer t par cos x, ce qui donne



sin3 x
dx = cos x − 2 arctan(cos x) + k.
1 + cos 2 x
2.4. Primitives des fractions rationnelles en ex
Pour calculer la primitive une fonction de la forme R (ex ) où R est une fraction
 ration-
x
nelle, on s’en sort toujours en posant t = e ; on se ramène alors au calcul de R(t)/t dt,
primitive d’une fraction rationnelle.
Fractions rationnelles en sinus et cosinus hyperbolique. Pour calculer une pri-
mitive de R( sh x, ch x) où R est une fraction rationnelle en deux variables, trois méthodes
sont utilisables. On peut
— faire le changement de variable t = th (x/2), ce qui ramène le calcul à celui de
  
2t 1 + t 2 2 dt
R ,
1 − t 1 − t 1 − t2
2 2

(technique à éviter si possible),


— tout exprimer en fonction de ex,
— effectuer un éventuel changement  de variable t = sh x, t = ch x ou t = th x en
procédant par analogie avec R(sin x, cos x) dx.
La première et/ou la seconde de ces méthodes donne les primitives classiques suivantes
     x
dx dx 1 ch x − 1  
x
= 2 arctan(e ) + k et = log + k = log  th  + k  .
ch x sh x 2 ch x + 1 2
2.5. Intégrales abéliennes
   
n ax + b
a). On veut calculer les primitives de la forme R x, avec ad − bc = 0 et
cx + d
n ∈ N∗ , où R est une fraction rationnelle en deux variables. On effectue le changement de
variables

ax + b dtn − b
t= n , de sorte que x = g (t) = et dx = g (t) dt.
cx + d a − tn c

On se ramène ainsi à calculer R(g (t), t)g  (t) dt, primitive d’une fraction rationnelle.
Exemple 3. On cherche à calculer

dx
√ √ .
1+x− 3 1+x
 √
On √ remarque que la fonction sous le signe est une fraction en 6 1 + x. On pose donc
t = 6 1 + x, de sorte que t 6 = 1 + x donc 6t5 dt = dx, et le calcul est ramené à celui de

6t 5
dt.
t 3 − t2
 √
b). On veut calculer les primitives de la forme R(x, ax2 + bx + c) avec a = 0, b2 −
4ac = 0, où R est une fraction rationnelle en deux variables. On traite plusieurs cas selon
le signe de a et de b2 − 4ac.
Si b2 − 4ac > 0, plusieurs √
cas se présentent. √  2
— Si a < 0, on écrit y = ax 2 + bx + c sous la forme y = −a q√ − (x − p)2 , puis
on effectue le changement de variable x − p = q cos θ (y devient q −a sin θ ). On se
ramène alors au calcul de primitives de fractions rationnelles en sinus et cosinus.
2. CALCUL DE PRIMITIVES 141

 √
Exemple 4. Pour calculer 1 − x2 dx, on fait le changement de variable x = cos θ
de sorte que dx = − sin θ dθ , ce qui ramène le calcul à celui de
 
2 cos(2θ ) − 1 sin(2θ) θ sin θ cos θ − θ
− sin θ dθ = dθ = − +k = .
2 4 2 2
Donc
 √ √
x 1 − x2 − arccos x
1 − x2 dx = + k.
2
√ √ 
— Si a > 0, on écrit y = ax2 + bx + x sous la forme y = a (x − p)2 − q 2 , puis
on
√ effectue le changement de variable x − p = qε ch t avec ε ∈ {−1, 1} (y devient
q a sh t). On se ramène à calculer les primitives d’une fraction rationnelle en sinus
et cosinus hyperbolique.
— Quel que soit le signe de a, une autre méthode est d’écrire ax2 + bx + c = a(x −
α)(x − β ) où α et β sont deux nombres réels distincts, puis
  
√ x − β
ax 2 + bx + c = |x − α| a
x−α

et on se ramène aux primitives traitées dans la partie a).


Si√b2 − 4ac < 0, alors ax2 + bx + c a le signe de a, on doit donc avoir a > 0. On écrit

y = ax + bx + c sous la forme y√= a q2 + (x − p) 2 , puis on fait le changement de
2

variable x − p = q sh t (y devient q a ch t).


Remarque 1. Lorsque a > 0, on peut également faire le changement de variable
√ √ √
y = ax 2 + bx + c = x a + t (resp. − x a + t).
L’idée
√ sous-jacente est la paramétrisation de la partie d’hyperbole H d’équation y =
2
ax + bx + c par une droite D parallèle à une asymptote de H (D √coupe H en un point

unique). Ce type de droite a pour équation y = x a + t ou y = −x a + t, t ∈ R.

Astuces de calcul. √
— Les primitives de la forme ax2 + bx + c peuvent se calculer en intégrant par
parties.
√
Exemple 5. On veut calculer t2 − 1 dt. En intégrant par parties, on a
 √ √  2 √  2
t (t − 1) + 1
t2 − 1 dt = t t2 − 1 − √ dt = t t2 − 1 − √ dt,
t2 − 1 t2 − 1
d’où
 √ √  √ √
1
2 t 2 − 1 dt = t t2 − 1 − √ = t t 2 − 1 − log |t + t2 − 1| + k.
t2 − 1
√ √
— Le calcul des primitives de fonctions de la forme R(x, αx + β, γx + δ) où R
est une fraction
√ rationnelle en trois variables se ramène, après le changement
√ de
2
variable t = γx + δ, à un calcul de primitives de la forme F (t, at + bt + c)
où F est une fraction rationnelle.

dx
— Le calcul des primitives  est considérablement simplifié
(x + a)n αx2 + βx + γ
en effectuant le changement de variable t = 1/(x + a).
142 3. INTÉGRATION

2.6. Produit d’un polynôme et d’une exponentielle



Les primitives P (x)erx dx où P est un polynôme et r un nombre complexe non nul
s’écrivent sous la forme Q(x)erx + k où Q est un polynôme ayant même degré que P
et k une constante. On trouve les coefficients de Q par identification dans l’expression
rQ + Q = P .  
Les primitives P (x)erx cos mx dx ou P (x)erx sin mx dx (r, m ∈ R, P polynôme)
s’obtiennent en les écrivant comme les parties réelles et imaginaires de P (x)e (r+im)x dx.
2.7. Exercices
Exercice 1. Calculer les primitives des fonctions suivantes
1 x+1 x2 1
a) b) c) d) .
x − x2 − 2
4 (x2 + 1) 2 x6 − 1 x(x 2 + 1)2

Solution. a) On décompose en éléments simples


1 1 1 1
= 2 = − ,
x4 2
−x −2 2
(x − 2)(x + 1) 2
3(x − 2) 2
3(x + 1)
et on en déduit
 

dx 1 x − √ 2  1
 
= √ log  √  − arctan x + k, k ∈ R.
x4 − x 2 − 2 6 2 x + 2 3

b) On a
x+1 1 2x 1
2 2
= · 2 2
+ 2 ,
(x + 1) 2 (x + 1) (x + 1)2
donc  
x+1 1 dx
2 2
dx = − 2 + .
(x + 1) 2(x + 1) (x + 1)2
2

Pour calculer la primitive du second membre de cette dernière égalité, on fait le changement de
variable x = tan θ , −π/2 < θ < π/2. On a dx = (1 + tan2 θ) dθ et
   
dx dθ 2 cos(2θ) + 1 sin(2θ) θ
2 2
= 2
= cos θ dθ = dθ = + +k
(x + 1) 1 + tan θ 2 4 2
2x 1
= 2
+ arctan x + k.
4(1 + x ) 2
Finalement

x+1 1 x 1
dx = − + + arctan x + k, k ∈ R.
(x2 + 1) 2 2(x 2 + 1) 2(1 + x2 ) 2

c) Il y a une petite astuce de calcul. On pose t = x3 , de sorte que dt = 3x 2 dx et


     3 
x2 1 dt 1 t − 1  1 x − 1 
dx = = log    
= log  3 .
x6 − 1 3 t2 − 1 6 t+1 6 x + 1

d) La décomposition en éléments simples de la fraction rationnelle considérée est


1 1 x x
= − 2 − 2
x(x2 + 1) 2 x x + 1 (x + 1)2
donc 
dx 1 1 1
= log |x| − log(x2 + 1) + · + k, k ∈ R.
x(x 2 + 1)2 2 2 1 + x2
2. CALCUL DE PRIMITIVES 143

Exercice 2. Calculer les primitives des fonctions suivantes


cos x sin x
a) 2
b)
sin x + 2 tan2 x cos3x + sin3 x
1 1
c) d) √ .
sin x + cos x + 2 ch x ch 2x

Solution. a) Il y a invariance de l’expression intégrée par le changement de variable x → π − x.


La règle de Bioche nous invite à effectuer le changement de variable t = sin x. On a
cos x cos x dx dt (1 − t 2) dt
2
dx = = = .
sin x + 2 tan2 x 2 sin2 x 2
t2 3t2 − t 4
sin x + 2 t +2
1 − sin2 x 1 − t2
De plus
X −1 1 2 t2 − 1 1 2
= + donc = + ,
X (X − 3) 3X 3(X − 3) t2(t 2 − 3) 3t2 3(t2 − 3)
d’où  

(1 − t2 ) dt 1 1  t − √3 
 
= − + √ log  √ 
3t 2 − t 4 3t 6 3 t + 3 
et finalement  

cos x 1 1  sin x − √3 
 
2
=− + √ log  √ .
2
sin x + 2 tan x 3 sin x 6 3  sin x + 3 

b) L’expression intégrée est invariante par le changement de variable x → x + π . La règle de


Bioche nous invite à faire le changement de variable t = tan x. On a alors dt = dx/(cos2 x) et
sin x dx tan x dx t
3 = 3 2
= dt.
cos 3 x +
sin x 1 + tan x cos x 1 + t3
Maintenant, la décomposition en éléments simples
t t 1 t+1
3
= 2
=− + 2
1+t (1 + t)(1 − t + t ) 3(t + 1) 3(t − t + 1)
entraı̂ne
   
t 1 1 2t − 1 1 1
dt = − + + · dt
1 + t3 3(t + 1) 6 t2 − t + 1 2 (t − 12 ) 2 + 34
  
1   1 1 2 2 1
= − log  1 + t + · log(t − t + 1) + · √ · arctan √ · t −
2
3 6 2 3 3 2
donc
  
sin x 1   1   1 2 tan x − 1
  2
dx = − log 1 + tan x + log tan x − tan x + 1 + √ · arctan √
cos3 x + sin 3 x 3 6 3 3

c) On commence par écrire sin x + cos x = 2 sin(x +π/ 4), puis on fait le changement de variable
t = x + π/4. On se ramène ainsi à évaluer la primitive
 
dt 1 dt
√ =√ √ .
2 sin t + 2 2 sin t + 2
Pour évaluer cette dernière primitive, on effectue le changement de variable u = tan(t/2) de
sorte que dt = 2du/(1 + u2 ) et
   √ √
1 dt du du
√ √ = √ =  √ 2
= 2 arctan( 2u + 1) + k.
2 sin t + 2 u 2 + 2u + 1 u+ 2 +1 2 2
144 3. INTÉGRATION

Finalement, on a
 √ x π 
dx √
= 2 arctan 2 tan + + 1 + k, k ∈ R.
sin x + cos x + 2 2 8
d) On a
1 1
ch 2x = ch 2x + sh 2x = ch 2x(1 + th 2 x) donc √ =  .
ch x ch 2x ch x 1 + th 2x
2

On fait donc le changement de variable t = th x. On a dt = dx/ ch 2x et


  
dx dt  
√ = √ = log t + t2 + 1 + k,
ch x ch 2x 1 + t2
donc  
dx  
√ = log th x + 1 + th 2 x + k, k ∈ R.
ch x ch 2x

Exercice 3. Calculer les primitives des fonctions suivantes



x−2 1 √
a) x b) √ c) −x2 + 4x + 10
x+1 (x + 1) x2 + x + 1
1 1
d) √ e) √
3
.
x + x2 + 2x 1 + x3

Solution. Il y a en général plusieurs moyens de calculer les primitives de chaque fonction


présentée. Nous nous limiterons à un seul type de résolution.
a) On fait le changement de variable

x−2 2 + t2 6t
t= ou encore x = ; on a alors dx = dt
x+1 1 − t2 (1 − t2 )2
et     2
x−2 2 + t2 6t 6t (2 + t2 )
x dx = · t · dt = dt.
x+1 1 − t2 (1 − t2 )2 (1 − t2)3
De la décomposition en éléments simples
 
6t2 (2 + t2 ) 1 3 3 21 21 18 18
= − − − + − ,
(1 − t2)3 8 t + 1 t − 1 (t + 1)2 (t − 1)2 (t + 1)3 (t − 1)3
on déduit
  
6t 2(2 + t 2 ) 1 21 21 9 9
dt = 3 log |t + 1| − 3 log |t − 1| + + − + +k
(1 − t2 )3 8 t + 1 t − 1 (t + 1) 2 (t − 1) 2

x−2
Pour obtenir les primitives de la fonction proposée, il suffit ensuite de remplacer t par .
x+1
Si on simplifie au mieux l’expression, on parvient finalement à
   
x−2 2x − 5  2 3   
x dx = x − x − 2 + log 2 x2 − x − 2 + 2x − 1 + k, k ∈ R.
x+1 4 8

b) Il faut utiliser l’une des astuces décrites à la page 141 : on fait le changement de variable
t = 1/(x + 1). Après calculs, on est ramené à la primitive
  
dt  1  2 
ε √2 = −ε log t − + t − t + 1  + k,
 k∈R
t −t+1 2
où ε ∈ {−1, 1} a le signe de t. On en déduit le résultat en remplaçant t par 1/(x + 1).
2. CALCUL DE PRIMITIVES 145

c) On résout le problème en intégrant par parties. On a


   
x2 − 2x
−x2 + 4x + 10 dx = x −x 2 + 4x + 10 + √ dx.
−x2 + 4x + 10
Or
  
x2 − 2x −x 2 + 4x + 10 2x + 10
√ dx = − √ dx + √ dx
−x2 + 4x + 10 −x2 + 4x +10 −x 2 + 4x + 10
 2x + 10
=− 2
−x + 4x + 10 dx + √ dx
−x 2 + 4x + 12
donc
    
2 2
−2x + 4 dx
2 −x + 4x + 10 dx = x −x + 4x + 10− √ dx+14 √
2
−x + 4x + 10 2
−x + 4x + 10
   
x−2
= x −x2 + 4x + 12 − 2 −x2 + 4x + 10 + 14 arcsin √ + k, k ∈ R.
14

d) On pose x2 + 2x = −x + t, de sorte que
t2 t2 + 2t
x= et dx = dt.
2(t + 1) 2(t + 1)2
On se ramène ainsi à
  
1 t+2 1 dt 1 dt 1   1
dt = + = log t + 1 − +k
2 (t + 1) 2 2 t+1 2 (t + 1)2 2 2(t + 1)
1    1
= log 1 + x + x2 + 2x  − √ + k, k ∈ R.
2 2(1 + x + x2 + 2x)

e) On fait d’abord le changement de variable t = 1/x, ce qui ramène le calcul à celui de


 
dt t2 dt
− √ = − √ ,
t 3 1 + t3 t 3 3 1 + t3
puis on pose u = t 3, nous ramenant à

1 du
− √
3
.
3 u 1+u
√3
On pose ensuite v = 1 + u, ce qui donne

v
− dv.
−1 v3
On décompose la dernière intégrande en éléments simples
 
v 1 1 v−1
= − 2
v 3− 1 3 v−1 v +v +1
et après un calcul classique, on trouve
  
v 1 1 2 1 2v + 1
− dv = − log(v − 1) + log(v + v + 1) − √ arctan √ + k, k ∈ R.
v3 − 1 3 6 3 3

3 1
Il reste alors à remplacer v par 1 + 3 et c’est terminé.
x

Exercice 4. Calculer l’intégrale


 3π/ 2
dx
I= .
π/2 3 + sin x
146 3. INTÉGRATION

Solution. On commence par rechercher une primitive de la fonction x → 1/(3 + sin x). Pour
cela, posons
x π   3π  2 dt
t = tan , x ∈ ,π ou x ∈ π, , de sorte que dx = .
2 2 2 1 + t2
On trouve, après calculs,
     
dx 2 dt 1 3t + 1 1 3 tan(x/2) + 1
= = √ arctan √ = √ arctan √ .
3 + sin x 3t2 + 2t + 3 2 2 2 2 2 2
 
1 3 tan(x/2) + 1
La fonction F : x → √ arctan √ est donc une primitive de x → 1/(3 + sin x),
2 2 2
mais seulement sur chacun des intervalles [π/2, π[ et ]π, 3π/ 2]. Il faut donc se garder décrire
I = F (3π/3) − F (π/2), ce qui n’aurait aucun sens. On va résoudre le problème en écrivant
π  3π/2
dx dx
I= + .
π/2 3 + sin x π 3 + sin x
On a
 π  X  π √
dx dx π arctan 2
= lim = lim F (X ) − F = √ − √ ,
π/2 3 + sin x X →π
X<π π/2 3 + sin x X →π
X<π
2 2 2 2
de même
 3π/2  3π/ 2   √
dx dx 3π arctan(1/ 2) π
= lim = lim F − F (X ) = − √ + √ .
π 3 + sin x XX>π
→π
X 3 + sin x X →π
X>π
2 2 2 2
Compte tenu de la classique relation arctan x + arctan(1/x) = π/2 pour x > 0, on en déduit
√ √
π arctan 2 + arctan(1/ 2) π π π
I= √ − √ = √ − √ = √ .
2 2 2 2 2 2 2

Exercice 5. Donner une relation de récurrence permettant de calculer les intégrales


suivantes :
 π/4  π/4  e
n dx
a) In = tan x dx, b) In = n
, c) I n = log n x dx.
0 0 cos x 1

Solution. a) Il suffit de remarquer que


 π/4  π/4
n 2 tann+1 x 1
∀n ∈ N, I n + I n+2 = tan x(1 + tan x) dx = = .
0 n+1 0 n+1
Cette relation permet de calculer In sachant que
π  π/4 log 2
I0 = et I 1 = − log(cos x) = .
4 0 2
b) Pour tout n ∈ N, on a, en intégrant par parties
 π/4  π/4  π/4
1 dx 1 sin x
In+2 = n 2
= n
tan x −n tan x dx
0 cos x cos x cos x 0 0 cosn+1 x
√ n  π/4 √ n
sin 2 x
= ( 2) − n dx = ( 2) − n(In+2 − In )
0 cos n+2 x
donc √
√ ( 2)n n
(n + 1)I n+2 = ( 2) n + nIn ou encore In+2 = + In ,
n+1 n+1
3. INTÉGRALE SUR UN INTERVALLE QUELCONQUE 147

d’où un moyen de calculer chaque In , sachant que


   x π π/4  
π 3π
I0 = et I 1 = log tan + = log tan .
4 2 4 0 8

c) En intégrant par parties, on a


 e  e
∀n ∈ N ∗, In = x logn x − n logn−1 x dx = e − nI n−1.
1 1
Cette relation de récurrence permet de calculer chaque terme In , sachant que I0 = e − 1.

3. Intégrale sur un intervalle quelconque


Dans toute cette partie, I désigne un intervalle quelconque de R dont les extrémités
sont notées a et b : −∞ ≤ a < b ≤ +∞, et E désigne un K-espace e.v.n complet (avec
K = R ou C).
3.1. Définition d’une fonction intégrable
Définition 1. Une fonction est dite continue par morceaux sur I si elle est continue par
morceaux sur tout segment J inclus dans I .
Fonctions positives intégrables.
 Définition 2 (Fonction positive intégrable). Soit f une fonction positive et
continue par morceaux sur I . On dit que f est intégrable (ou sommable) sur I si il existe
M ≥ 0 tel que, pour tout segment J ⊂ I , on a J f ≤ M . On note alors
 
f = sup f.
I J ⊂I J

Proposition 1. Soit f positive et continue par morceaux sur I . Alors f est intégrable si
et seulement s’il existe une suite croissante
 de segments Jn = [an , bn ] inclus dans I telle
que ∪n Jn = I et telle que la suite Jn f soit bornée. Dans ce cas, pour toute suite (J n) de
ce type on a
    bn
f = sup f = lim f = lim f (t) dt.
I n Jn n→∞ Jn n→∞ an

Exemple 1 (Exemples fondamentaux). — Cas b = +∞, a ∈ R.


α
— Pour a > 0, t → 1/t est intégrable sur [a, +∞[ si et seulement si α > 1.
— t → e−λt est intégrable sur [a, +∞[ et seulement si λ > 0.
— Cas a et b finis.
— La fonction t → 1/(b − t)α est intégrable sur [a, b[ si et seulement si α < 1.
— De même, t → 1/(t − a)α est intégrable sur ]a, b] et seulement si α < 1.
Fonctions intégrables à valeurs quelconques.
 Définition 3 (Fonction intégrable). Une fonction f : I → E continue par mor-
ceaux est dite intégrable (ou sommable) sur I si f  est intégrable sur I . Dans ce cas,
 toute suite croissante (Jn ) de segments inclus dans I telle que ∪ nJ n = I, la limite de
pour
( Jn f ) existe et ne dépend pas du choix de la suite (J n ). Cette limite s’appelle l’intégrale

de f et est notée I f .
Démonstration. Notons Jn = [a n , bn], de sorte que la suite (a n) est décroissante
 et tend vers a,
et (bn) est croissante et tend vers b. Montrons d’abord que la suite u n = Jn f est de Cauchy.
148 3. INTÉGRATION


Soit ε > 0. Comme f  est intégrable sur I, la suite (Un) = ( Jn f ) converge donc il existe
N ∈ N tel que pour tout p, q ≥ N , |Up − U q| < ε. Ceci entraı̂ne, pour p, q ≥ N (et p < q),
    
 
 
u p − u q  =  f+ f ≤ f  + f  = Uq − U p < ε.
 [a q,ap ] [bp ,bq]  [aq ,ap ] [bp ,b q]

Finalement, (un ) est bien une suite de Cauchy dans l’e.v.n complet E, donc elle converge. Notons
 sa limite.
Unicité de la limite. Soit (Kn ) une autre suite d’intervalles
 vérifiant les mêmes hypothèses
que (Jn ), et notons  la limite de la suite vn = Kn f . Construisons la suite de segments

Ln = J n ∪ K  n = [c n , dn ], qui est bien croissante et vérifie ∪n Ln = I, et notons  la limite de la
suite wn = Ln f . Notons Wn = Ln f . Comme Jn ⊂ Ln on a
    
 
 
wn − un =  f+ f ≤ f  + f  = Wn − U n ,
 [c n,an ] [bn ,dn]  [c n,a n ] [b n,dn ]

et comme (Un ) et (W n) ont même limite (c’est I f ), on en déduit que limn→+∞ wn − u n = 0.
Donc  = . On montrerait de même que  =  , on a donc bien démontré que  =  . 

Remarque 1. — Si f est  à valeurs réelles, une définition équivalente à la précédente


est I f = I f + − I f − , où f+ = max(f, 0) et f − = max(−f, 0). La définition 3
que nous proposons permet de ne pas se limiter au cadre où E = R et donne une
définition intrinsèque de l’intégrale sur tout e.v.n complet, en particulier sur C et
sur tout e.v de dimension finie (voir également la remarque 2 page 124).
— La proposition 1 page 125 se généralise aisément ici : Une fonction continue par
morceaux  f : I → C est intégrable sur I si et seulement si (f ) et (f ) le sont,
et on a I f = I (f ) + i I (f ). De même, considérons un e.v.n E de dimension
finie dont (e i ) est une base, et une fonction continue par morceaux f : I → E qui
s’écrit f = i fi ei (avec fi ∈ Cm (I, R)).Alors  f est intégrable sur I si et seulement
si les fi sont intégrables sur I, et on a I f = i( I fi)ei .
— Si f est continue par morceaux sur un segment [a, b], alors f est bien intégrable au
sens de la définition précédente, et la définitionde son intégrale
 est
 bien égale
 à
celle de la définition 4 page 124, et de plus on a [a,b] f = ]a,b] f = [a,b[ f = ]a,b[ f .
— De manière générale, f est intégrable sur I si et seulement si elle est intégrable sur
◦   b
l’intérieur I de I , et on a I f = ◦ f . Ceci permet d’utiliser la notation a f (t) dt =
 b I a
I
f . Si a > b, on définit a = − b .
— Les propriétés élémentaires des intégrales sur un segment (linéarité, positivité, rela-
tion
 de Chasles,
 . . . ) restent vraies pour les fonctions intégrables. On a notamment
 I f  ≤ I f .
Critères d’intégrabilité.
Proposition 2. Soit I = [a, b[ et f : I → E une fonction continue par morceaux sur I .
Les assertions suivantes sont équivalentes
(i) f est intégrable sur [a, b[
x
(ii) x → a f (t) dt est bornée sur [a, b[
x
(iii) lim x→b a f (t) dt existe
x<b
b
(iv) lim x→b x f (t) dt = 0
x<b
y
(v) ∀ε > 0, ∃A ∈ I tel que ∀x, y ∈ [A, b[ (x < y), x
f (t)dt < ε (critère de Cauchy)
3. INTÉGRALE SUR UN INTERVALLE QUELCONQUE 149


Proposition 3. Soit f : I → E une fonction continue par morceaux et soit c ∈ I. Soit
Ig = I ∩ ] − ∞, c] et I d = I ∩ [c, +∞[. Alors f est intégrable
 sur I si et seulement si f est
intégrable sur Ig et intégrable sur I d, et on a I f = Ig f + Id f .

Proposition 4. Soient f : I → E et ϕ : I → R+ continues par morceaux.


 
(i) Si f  ≤ ϕ sur I et si ϕ est intégrable, alors f est intégrable et on a  I f  ≤ I ϕ.
(ii) Si f est à valeurs positives et non intégrable, et si f ≤ ϕ, alors ϕ est non intégrable.
Proposition 5. Soient f : [a, b[ → E et g : [a, b[ → R+ continues par morceaux.
(i) Si f (x) = O (g (x)) lorsque x → b et si g est intégrable, alors f est intégrable.
(ii) Si f (x) ∼ g (x) lorsque x → b (et f à valeurs réelles), alors f est intégrable si et
seulement si g est intégrable.
On utilise souvent les propositions précédentes pour comparer les fonctions que l’on
intègre avec les fonctions de comparaison introduites dans l’exemple 1. Par exemple
2 2
lim t 2e −t = 0 donc e−t = O (t−2 ) (t → +∞),
t→+∞

et comme t → 1/t2 est intégrable sur [1, +∞[, on en déduit avec l’assertion (i) de la
2
proposition 5 que t → e−t est intégrable sur [1, +∞[. On peut également utiliser des
comparaisons avec les intégrales de Bertrand (proposition 6).
Intégrales de Bertrand. En plus des exemples fondamentaux de l’exemple 1 les inté-
grales de Bertrand fournissent d’autres fonctions de comparaison qui permettent parfois,
à l’aide des propositions précédentes, de décider de la convergence d’une intégrale.

Proposition 6 (Intégrales de Bertrand). Soient α et β des nombres réels. Alors


 
1
t → α β est intégrable sur [e, +∞[ ⇐⇒ ((α > 1) ou (α = 1 et β > 1))
t log t
et
 
1
t → est intégrable sur ]0, 1/e] ⇐⇒ ((α < 1) ou (α = 1 et β > 1)).
t | log t|β
α

Démonstration. Montrons tout d’abord la première partie de la proposition.


— Si α > 1, on écrit α = 1 + 2h avec h > 0. Pour tout β ∈ R, on a
 
1 1 1 1 1
lim = 0 donc = 1+h =O
t→+∞ th log β t tα logβ t th logβ t t 1+h
donc t → t−α log−β t dt est intégrable sur [e, +∞[.
— Si α = 1, deux cas se présentent.
— Si β > 1, comme
X  X
dt log 1−β t log 1−β X − 1
∀X > e, = = ,
e t logβ t 1−β
e
1−β

on en conclut que la fonction est bien intégrable sur [e, +∞[.


— Si β ≤ 1, on écrit
 X  X  X
dt dt
∀X > e, ≥ = log(log t) = log log X,
e t logβ t e t log t e

ce qui prouve que la fonction n’est pas intégrable.


150 3. INTÉGRATION

— Si α < 1, on écrit α = 1 − 2h avec h > 0. On a pour tout β ∈ R


th 1 th 1 1
lim β
= +∞ donc ∃A ≥ e, ∀t > A, β
= β
≥ ,
t→+∞ log t tα log t log t t1−h t1−h
ce qui montre que la fonction n’est pas intégrable car t → 1/t1−h n’est pas intégrable sur
[A, +∞[.
La seconde partie de la proposition se déduit de la première par le changement de variable
u = 1/t grâce au théorème du changement de variable (théorème 1). 

3.2. Propriétés des intégrales de fonctions intégrables


Comme pour les intégrales sur un segment de R, les intégrales des fonctions intégrables
possèdent les propriétés qui suivent.

Théorème 1 (Changement de variable). Soit f : J → E une fonction continue


par morceaux sur J , et ϕ : I → J une bijection de classe C1 où J est un intervalle de R.
Alors f est intégrable sur J si et seulement si (f ◦ ϕ) × ϕ l’est sur I . De plus, en notant
a et b les extremités de l’intervalle I et ϕ(a) et ϕ(b) les limites de ϕ en a et b, on a
b  ϕ(b)

f (ϕ(t)) ϕ (t) dt = f (x) dx.
a ϕ(a)

Théorème 2 (In égalité de Schwarz). Soient f, g : I → K deux applications conti-


nues par morceaux et telles que f 2 et g2 sont intégrables sur I . Alors f g est intégrable sur
I et on a
 2    
 
 f g ≤ |f | 2 · |g |2 .
 
I I I

Conséquence : On en déduit en particulier que si f et g 2 sont intégrables, alors f + g


2

est également de carré intégrable.


Convergence en moyenne, convergence quadratique.
Proposition 7 (Norme de la convergence en moyenne). L’ensemble L1c (I, E) des fonc-
tions continues de I dans E et intégrables sur I est un espace vectoriel. L’application

1 +
N1 : Lc (I, E) → R f → f 
I

est une norme sur L 1c (I, E), appelée norme de la convergence en moyenne.
Proposition 8 (Norme de la convergence en moyenne quadratique). L’ensemble L2c (I, K)
(K = R ou C) des fonctions continues de I dans K de carré intégrable (i.e. f 2 intégrable)
sur I est un espace vectoriel. L’application

2 2
L c(I, K) → K, (f, g) → f g
I

est un produit scalaire sur L2c (I, K) qui fait de L2c (I, K) un espace préhilbertien. La norme
associée est l’application

N2 : L2c (I, K) → R + f → |f | 2
I

appelée norme de la convergence en moyenne quadratique.


3. INTÉGRALE SUR UN INTERVALLE QUELCONQUE 151

Théorème de convergence dominée. Voici maintenant le théorème le plus important


du chapitre. Il correspond à une version aménagée, au niveau des classes préparatoire, du
théorème plus général de convergence dominée dans le cadre de l’intégrale de Lebesgue.
Il est puissant et très commode à utiliser, là où des approches fondées sur la convergence
uniforme sont souvent plus fastidieuses.
 Théorème 3 (convergence dominée). Soit (f n ) une suite de fonctions continues
par morceaux de I dans E , vérifiant les conditions suivantes
(i) Il existe une fonction positive ϕ, continue par morceaux, intégrable sur I telle que
fn  ≤ ϕ pour tout n (Hypothèse de domination).
(ii) La suite de fonctions (f n) converge simplement vers une fonction f : I → E continue
par morceaux,
Alors les fn et f sont intégrables sur I et on a
 
lim fn = f.
n→∞ I I
La preuve de ce résultat est hors programme des classes préparatoires. On en trouvera
deux démonstrations différentes dans les problèmes 20 page 194 et 21 page 196.
3.3. Intégrale généralisée
Définition 4. Soient [a, b[ un intervalle de R (avec−∞ < a < b ≤ +∞) et f : [a, b[→ E
x
une fonction continue par morceaux. Si  = lim x→b a f (t) dt existe et est finie, on dit que
b x<b

l’intégrale généralisée (ou impropre) a f (t) dt converge (ou qu’elle est convergente) et on
b b
pose a f (t) dt = . Dans le cas contraire, on dit que l’intégrale a f (t) dt diverge (ou
qu’elle est divergente).
b
Remarque 2. — Si f est intégrable sur [a, b[, l’intégrale généralisée a f (t) dt converge
b
(on dit aussi que l’intégrale a f (t) dt est absolument convergente), et la valeur de
l’intégrale donnée par la définition précédente est identique à celle de la définition 3
page 147. La réciproque est fausse ; les intégrales généralisées qui convergent mais
ne convergent pas absolument sont appelées intégrales semi-convergentes. b
— Si f est positive, f est intégrable si et seulement si l’intégrale a f (t) dt converge.
— Lorsque f est continue par morceaux sur ]a, b] (−∞ ≤ a < b < +∞), on définirait
b b
de même a f (t) dt = lim xx>a →a
x
f (t) dt lorsque cette limite existe.
b b
— Pour tout c ∈ [a, b[, les intégrales c f (t) dt et a f (t) dt sont de même nature.
— Les propriétés élémentaires vérifiées par les fonctions intégrables restent vraies pour
les intégrales généralisées (linéarité, positivité, relation de Chasles, . . .).

Définition 5. Soient ]a, b[ un intervalle de R (avec −∞ ≤ a < b ≤ +∞) et f : ]a, b[→ E


une fonction continue par morceaux sur ]a, b[. Si
 y
 = lim f (t) dt
(x,y)→(a,b) x
b b
existe, on dit que l’intégrale a f (t) dt converge et on note f (t) dt = . Dans le cas
b a
contraire, on dit que a f (t) dt diverge.
Remarque 3. — Cette définition est cohérente
 b avec la précédente : si f est continue par
morceaux sur [a, b[ (on ferme en a) et si a f (t) dt converge au sens de la première
définition, alors cette même intégrale converge au sens de la seconde définition.
152 3. INTÉGRATION

— Pour tout c ∈ ]a, b[ on a l’équivalence


 b   c  b 
f (t) dt converge ⇐⇒ f (t) dt converge et f (t) dt converge .
a a c
Cette équivalence nous permet de nous limiter à l’étude de la convergence des
intégrales généralisées du type de celles introduites dans la définition 4.
+∞
— En particulier, si f : R → E est continue par morceaux, l’intégrale −∞ f (t) dt
0  +∞
converge si et seulement si chacune des deux intégrales −∞ f (t) dt et 0 f (t) dt
x
converge. Ceci n’est pas équivalent à dire que la limite limx→+∞ −x f (t) dt existe
(la condition est nécessaire mais pas suffisante comme le montre le contre-exemple
de la fonction f (t) = t).
Citons enfin le théorème qui suit, qui rend parfois quelques services.
Théorème 4. Soit a ∈ R et f : [a, +∞[→ E une fonction uniformément continue sur
 +∞
[a, +∞[. Si l’intégrale a f (t) dt converge, alors lim t→+∞ f (t) = 0.
Démonstration. Soit ε > 0. L’uniforme continuité de f entraı̂ne
∃η > 0, ∀x, y ≥ a, |x − y | < η, f (x) − f (y) < ε,

et la convergence de l’intégrale a+∞ f (t) dt entraı̂ne
 
 y 
∃A > a, ∀x, y ≥ A,  f (t) dt
  ≤ εη.
x
Considérons maintenant x > A. On a
  x+η   x+η   
       x+η 

η f (x) =   
f (x) dt ≤  f (x) − f (t) dt  f (t) dt
 + 
x x x
 x+η
≤ ε dt + εη = 2εη,
x
donc f (x) ≤ 2ε. Ceci étant vrai pour tout x > A, on en déduit le résultat. 

Remarque 4. Le théorème est faux si f est seulement supposée continue (pour un contre-
exemple, voir le troisième alinéa de la remarque 6 — on peut également construire un
contre-exemple d’une fonction continue positive non bornée dont l’intégrale sur R converge,
en considérant une fonction nulle sauf en certains endroits où elle possède des “pics” de
plus en plus grands et de plus en plus étroits lorsque x → +∞).
3.4. Intégrales semi-convergentes
On appelle ainsi les intégrales qui convergent mais ne convergent pas absolument. Le
résultat qui suit permet souvent de montrer la convergence de telles intégrales.

Théorème 5 (Règle d’Abel). Soient f : [a, b[ → R de classe C 1 et g : [a, b[ → R


continue sur [a, b[ vérifiant
(i) f est décroissante et limx→b f (x) = 0 (en particulier, f est positive),
 x 
 
(ii) il existe M > 0 tel que pour tout x ∈ [a, b[,  g (t) dt ≤ M .
a
b
Alors a f (t)g (t) dt est convergente.
Démonstration. Soit ε > 0. L’hypothèse (i) assure l’existence de A > 0 tel que f (A) ≤ ε. En
utilisant la deuxième formule de la moyenne (voir page 128), on a
 y c
∀x, y ∈ R, a ≤ x < y < b, ∃c ∈ [x, y ], f (t)g (t) dt = f (x) g (t) dt,
x x
3. INTÉGRALE SUR UN INTERVALLE QUELCONQUE 153

ce qui entraı̂ne,
 
 y 
∀x, y ∈ R, A ≤ x < y,  f (t)g (t) dt ≤ f (x) · 2M ≤ f (A) · 2M ≤ 2M ε,

x

d’où le résultat en vertu du critère de Cauchy. 

Remarque 5. - Ce résultat est une version continue du théorème 7 page 215 sur les séries.
Il repose essentiellement sur la deuxième formule de la moyenne (page 128). Cette dernière
n’est pas au programme (la règle d’Abel pour les intégrales ne l’est pas non plus), mais
nous avions vu que lorsque f est C 1 et g continue, sa preuve s’obtient facilement. La
remarque 6 donne un exemple typique de preuve directe.
– En utilisant la proposition 1 page 125, il est clair que ce théorème reste vrai si g est à
valeurs dans un R-e.v de dimension finie, en particulier sur C.
Conséquence : On considère la fonction g : [a, +∞[ → C t → eiλt ( où a, λ ∈ R). On a
 x   iλx 
   e − eiλa 
∀x > a,  g (t ) dt =  ≤ 2 .
   λ  |λ|
a

En appliquant la règle d’Abel, on en déduit que pour toute fonction f : [a, +∞[, → R
décroissante et tendant vers 0 à l’infini, l’intégrale
 +∞
f (t)eiλt dt
a
 +∞
converge. En particulier, pour tout α > 0, l’intégrale 1
eit/t α dt converge.
Remarque 6. — On peut prouver facilement la convergence de l’intégrale
 +∞ it
e
dt (∗)
1 tα
pour α > 0 sans utiliser la règle d’Abel (qui rappelons le, n’est pas au programme).
Il suffit d’intégrer par parties, en écrivant
 X it  it X 
e e α X eit
∀X > 1, dt = + dt.
1 tα it α 1 i 1 tα+1
On remarque ensuite que le terme de gauche dans le membre de droite de cette
dernière égalité converge lorsque X → +∞; quant à son terme de droite, il converge
+∞
également quand X → +∞ car l’intégrale 1 eit/tα+1 dt converge absolument. De
tout ceci, on en déduit la convergence de l’intégrale (*). En particulier, les parties
réelles et imaginaires de cette intégrale convergent, c’est-à-dire que pour tout α > 0,
on a la convergence des intégrales
 +∞  +∞
sin t cos t
dt et dt.
1 tα 1 tα
— Lorsque 0 < α ≤ 1, l’intégrale (*) est semi-convergente (elle est convergente mais
non absolument convergente). Lorsque α ≤ 0, elle est divergente. 
+∞
— Grâce au changement de variable u = t2, on montre que l’intégrale 1 sin(t 2) dt a
 +∞ √
la même nature que 1 (sin u)/ u du, donc convergente d’après ce que l’on vient
de voir. Ceci est un exemple d’intégrale convergente dont la fonction intégrée ne
tend pas vers 0 à l’infini.
154 3. INTÉGRATION

— Profitons en ici pour rappeler que si deux fonctions sont équivalentes en l’infini
et si elles ne sont pas de signe constant, leurs intégrales ne sont pas forcément de
même nature. Par exemple, les fonctions
e it eit 1
f : [1, +∞[→ C t → √ et g : [1, +∞[→ C t → √ +
t t t
 +∞
sont équivalentes en l’infini ; pourtant 1 f (x) dx converge (on vient de le voir)
 +∞  +∞    +∞
et 1 g (x) dx diverge (si elle convergeait, 1 g (t) − f (t) dt = 1 dt/t conver-
gerait, ce qui est faux).
3.5. Exercices
Exercice 1. Étudier la nature des intégrales suivantes :
1  +∞    +∞ √
ch t − cos t 1 t sin(1/t 2 )
a) dt b) log cos dt c) dt
0 t 5/2 2/π t 0 log(1 + t)
 +∞  +∞
sin t log(1 + t α)
d) α
dt, α ∈ R e) β
dt, (α, β) ∈ R 2 .
0 t 0 t

Solution. a) La fonction f : t → ( ch t − cos t)t−5/2 est continue sur ]0, 1]. Le problème se situe
donc en 0. Au voisinage de 0, on a
   
t2 2 t2
ch t − cos t = 1 + + o( t ) − 1 − + o(t ) = t2 + o(t2 ) ∼ t2,
2
2 2
donc f (t) ∼ t−1/2 lorsque t → 0+ . On en déduit (grâce à la proposition 5) que l’intégrale
1
0 f (t) dt est absolument convergente.

b) Ici, la fonction f : t → log(cos(1/t)) est continue sur ]2/π, +∞[. Étudions le comportement
de cette fonction aux deux bornes de cet intervalle. Lorsque t → 0 +,
       
2 π 1 π π2
f + t = log cos · = log cos − t + o( t)
π 2 1 + tπ/2 2 4
  2   2   2 
tπ tπ π
= log sin + o( t) = log + o(t) = log t + log + o(1) ∼ log t,
4 4 4
1
donc 2/π f (t) dt converge absolument. Lorsque t → +∞,
    
1 1 1 1 1
f (t) = log 1 − 2 + o 2 = − 2 +o 2 ∼ − 2,
2t t 2t t 2t
 +∞
ce qui montre que l’intégrale 1 f (t) dt converge absolument.
L’intégrale proposée est donc absolument convergente.

t sin(1/t2 )
c) La fonction f : t → est continue sur ]0, +∞[. En 0+, on a
log(1 + t)
√  
t 1 1
|f (t)| ≤ ∼ √ donc f (t) = O √ ,
log(1 + t) t t
1
ce qui montre que 0 f (t) dt est absolument convergente. En +∞, on a
√  
t 1 1 1
f (t) ∼ = 3/2 = O 3/2 ,
log t t2 t log t t
 +∞
donc 1 f (t) dt est absolument convergente. On en déduit que l’intégrale proposée est absolu-
ment convergente.
3. INTÉGRALE SUR UN INTERVALLE QUELCONQUE 155

 +∞
d) La fonction fα : t → t −α sin t est continue sur ]0, +∞[. On sait que l’intégrale 1 fα(t) dt
converge si et seulement si α > 0 (voir la remarque 6). Par ailleurs, lorsque t → 0+ on a
1
fα (t) ∼ 1/tα−1 , donc 0 fα (t) dt converge si et seulement si α < 2. Finalement, l’intégrale
proposée converge si et seulement si 0 < α < 2 (elle est semi-convergente pour 0 < α ≤ 1 et
absolument convergente pour 1 < α < 2).
e) L’application fα,β : x → t −β log(1 + t α) est continue sur ]0, +∞[. En 0 + , trois cas se
produisent : 
— Si α > 0, on a f α,β (t) ∼ tα−β donc 01 f α,β(t) dt converge si et seulement si β − α < 1.
— Si α = 0, on a fα,β (t) = (log 2)t−β donc l’intégrale converge si et seulement si β < 1.
1
— Si α < 0, on a fα,β (t) ∼ α(log t)t −β donc 0 f α,β (t) dt converge si et seulement si β < 1
(voir les intégrales de Bertrand, proposition 6).
En +∞, on traite également trois cas : 
— Si α > 0, alors f α,β (t) ∼ α(log t)t−β donc 1+∞ fα,β(t) dt converge si et seulement si
β > 1.  +∞
— Si α = 0, alors fα,β (t) ∼ (log 2)t−β donc 1 fα,β (t) dt converge si et seulement si β > 1.

— Si α < 0, alors fα,β (t) ∼ t α−β donc 1+∞ f α,β(t) dt converge si et seulement si β − α > 1.
 +∞
De tout ceci, on déduit que 0 fα,β (t) dt converge si et seulement si 1 + α < β < 1 ou
1 < β < α + 1.

 +∞
Exercice 2. Soit f : [1, +∞[→ R une fonction continue telle que l’intégrale 1
f (t) dt
converge. Montrer que pour tout nombre réel a > 0, l’intégrale
 +∞
f (t)
dt
1 ta
converge.

Solution. Il s’agit en fait d’un cas particulier de la règle d’Abel (théorème 5). Nous allons ce-
pendant prouver le résultat directement sans faire appel à cette dernière (la preuve est d’ailleurs
1
tout-à-fait représentative de celle de la règle d’Abel dans x le cas où f est continue et g est C ).
On considère l’application F : [1, +∞[→ R x → 1 f (t) dt. Par hypothèse, cette applica-
tion converge lorsque x → +∞, elle est donc bornée. En intégrant par parties, on a pour tout
a>0
 X   X
f (t) F (t) X F (t)
∀X > 1, a
dt = a
+a dt.
1 t t 1 1 t1+a
La dernière intégrale converge absolument lorsque X → +∞ car F est bornée et a > 0. Quant
au terme entre crochets, il converge également lorsque X → +∞ toujours parce que F est bornée
et a > 0. On en déduit le résultat.

Exercice 3. Donner la nature des deux intégrales suivantes :


 +∞  +∞
dx
a) 4 2
b) | sin x|x dx.
0 1 + x sin x 1

Solution. a) La nature de cette intégrale ne peut pas être décidée en utilisant “les méthodes
usuelles” de comparaison avec des fonctions dont la nature de l’intégrale est connue. On s’en
sort autrement en utilisant une comparaison série-intégrale. Pour tout n ∈ N, on pose
 (n+1)π
1
un= f (x) dx où f (x) = .
nπ 1 + x sin 2 x
4
156 3. INTÉGRATION

  +∞
La fonction f étant positive, la série un et l’intégrale 0 f (x) dx ont même nature (en effet,
 
l’égalité 0nπ f (t) dt = n−1k=0 uk montre que si l’une est bornée l’autre l’est également). Nous
sommes donc ramené à donner la nature de la série u n. Nous allons prouver qu’elle converge.

Pour tout n ∈ N , on a
 (n+1)π  π  π/2
dt dt dt
un ≤ 2
= 2
= 2 = 2In.
nπ 1 + n 4π 4 sin t 4 4
0 1 + n π sin t 0 1 + n4 π4 sin 2 t
La minoration classique sin t ≥ 2t/π sur [0, π/ 2] (que l’on peut obtenir, par exemple, en utilisant
la concavité de la fonction sinus sur cet intervalle) entraı̂ne
 π/2  π/2
∗ dt dt
∀n ∈ N , In ≤ 4 4 2 2
= ,
0 1 + n π 4t /π 0 1 + 4n 4π2 t2
ce qui en effectuant le changement de variable u = 4n 2πt entraı̂ne
 2n 2π2  +∞
∗ 1 du 1 du
∀n ∈ N , In ≤ 2 2
≤ 2
,
4n π 0 1+u 4n π 0 1 + u2

autrement dit, u n ≤ 2In = O(1/n 2). Ceci suffit pour conclure que la série un converge.
b) Ici, comme précédemment, on ne peut pas s’en tirer en utilisant les méthodes usuelles de
critère de convergence d’une intégrale. On procède également par comparaison série-intégrale.
Pour tout n ∈ N∗ , on pose  nπ
un = | sin x| x dx.
(n−1)π
La
 fonction intégrée étant positive, nous avons montré précédemment
 que l’intégrale et la série
u n avaient même nature. Nous allons montrer cette fois que un diverge. Pour tout n ∈ N∗ ,
on a   
nπ π π/2
un ≥ | sin x|4n dx = sin4n x dx = 2 sin4n x dx = 2I n.
(n−1)π 0 0
L’intégrale I n est une intégrale classique : c’est une intégrale de Wallis, dont on sait (voir

l’exercice
 1, page 130) qu’elle est équivalente à π/(8 n). Donc In diverge, et comme un ≥ 2In ,
u n diverge. L’intégrale proposée diverge donc.

Exercice 4. Soit f : [0, +∞[→ R une fonction continue par morceaux, positive et
décroissante, et intégrable sur R+ . Montrer que f (x) = o(1/x) lorsque x → +∞.
Solution. Soit ε > 0. Comme f est intégrable, le critère de Cauchy s’applique, donc
 2x
∃X > 0, ∀x > X, f (t) dt < ε,
x
et on en déduit, la fonction f étant décroissante,
 2x  2x
∀x > X, xf (2x) = f (2x) dt ≤ f (t) dt < ε.
x x
Donc 0 ≤ (2x)f (2x) ≤ 2ε pour tout x > X. Ceci entraı̂ne limx→+∞ xf (x) = 0 d’où le résultat.
Remarque. Ce résultat est une version continue de celui de l’exercice 2 page 219.

1
Exercice 5. a) Soit f : ]0, 1[→ R une fonction croissante telle que l’intégrale 0
f (t) dt
converge. Montrer que
n−1    1
1 k
lim f = f (t) dt.
n→+∞ n n 0
k=1
3. INTÉGRALE SUR UN INTERVALLE QUELCONQUE 157

b) (Application.) Montrer que pour tout nombre réel α > 0, on a


n
 nα
kα−1 ∼ lorsque n → +∞.
α
k=1

Solution. a) C’est ultra-classique. Il suffit d’écrire, la fonction f étant croissante, que


 k/n    (k+1)/n
1 k
∀k ∈ {1, . . . , n − 1}, f (t) dt ≤ f ≤ f (t) dt,
(k−1)/n n n k/n

puis de sommer cette relation pour k allant de 1 à n, ce qui donne


 n−1   1
1−1/n
1 k
f (t) dt ≤ f ≤ f (t) dt,
0 n n 1/n
k=1

d’où le résultat en faisant tendre n vers l’infini puisque chacun des termes extrêmes de ces
1
inégalités tend vers 0 f (t) dt.
b) En appliquant le résultat précédent à la fonction f : x → xα−1 , on obtient
n−1   α−1 1 n−1
1 k α−1 1 1  α−1 1
lim = t dt = donc lim k = ,
n→+∞ n n 0 α n→+∞ n α α
k=1 k=1

et on en déduit facilement le résultat.

Exercice 6. Soient ϕ : R → C une fonction continue par morceaux sur R, T -périodique,


et f : I → C une fonction continue par morceaux et intégrable sur un intervalle I de R.
On note a et b les extrémités de l’intervalle I, avec −∞ ≤ a < b ≤ +∞.
a) Montrer
b  T   b 
1
lim f (t)ϕ(nt) dt = ϕ(t) dt · f (t) dt . (∗)
n→+∞ a T 0 a
b
b) (Lemme de Riemann-Lebesgue) Montrer que limn→+∞ a f (t)eint dt = 0.
b
c) Si I est un segment de R et f de classe C 1 sur I, montrer a f (t)e int dt = O(1/n).
d) On suppose I = R. En utilisant le changement de variable u = t − π/n, obtenir
directement (sans utiliser le résultat de la question a)) le lemme de Riemann-Lebesgue
(question b)).
T
Solution. a) Posons K = 1T 0 ϕ(t) dt. Il s’agit de montrer que
 b  b
 
lim f (t) · ϕ(nt) − K dt = 0 ou encore lim f (t)ψ (nt) dt = 0 (∗∗)
n→+∞ a n→+∞ a

avec ψ = ϕ − K . Notons que la fonction ψ vérifie 0T ψ (t) dt = 0. Pour prouver (**), nous
allons procéder en trois étapes : d’abord lorsque f est la fonction caractéristique d’un segment
inclus dans I , puis lorsque f est en escalier sur un segment de I , puis lorsque f est continue par
morceaux et intégrable sur I .
(i) Si f est la fonction caractéristique d’un segment J = [α, β] inclus dans I , on a
 b  β 
1 nβ
In= f (t)ψ (nt) dt = ψ (nt) dt = ψ (t) dt.
a α n nα
158 3. INTÉGRATION

Soit p l’entier naturel tel que nα + pT ≤ nβ < nα + (p + 1)T (p est la partie entière de
n(β − α)/T ). On peut écrire
 nα+pT  nβ 
1
In = ψ (t) dt + ψ (t) dt
n nα nα+pT
  T  nβ  
1 1 nβ
= p ψ (t) dt + ψ (t) dt = ψ (t) dt,
n 0 nα+pT n nα+pT
et comme 0 ≤ nβ − (nα + pT ) < T ceci entraı̂ne

1 T
|In | ≤ |ψ (t)| dt.
n 0
On en déduit que (**) est bien vérifié pour f .
(ii) Si f est une fonction en escalier sur un segment inclus dans I , f est combinaison linéaire
de fonctions caractéristiques de segments inclus dans I , on en déduit par linéarité que
(**) reste vrai pour f .
(iii) Traitons maintenant le cas où f est continue par morceaux et intégrable sur I . Soit
ε > 0. Comme f est intégrable, il existe un segment J = [c, d] inclus dans I tel que
c b
a |f (t)| dt + d |f (t)| dt < ε. Ensuite, la restriction de f à J étant continue par morceaux,
elle est réglée donc il existe une fonction en escalier g telle que |f − g | < ε/(d − c) sur
J. L’étape précédente nous assure l’existence de N ∈ N tel que pour tout n ≥ N ,
d
| c g (t)ψ (nt) dt| < ε. Ainsi, en notant M = supt∈[0,T ] |ψ (t)| = supt∈R |ψ (t)|, on a, pour
n≥N
 d  d  d 
   
 
f (t)ψ (nt) dt ≤ 
|f (t) − g (t)| · |ψ (nt)| dt +  g (t)ψ (nt) dt  ≤ M ε + ε.

c c c
Ceci entraı̂ne que lorsque n ≥ N ,
 b  c   
   d  b
 f (t )ψ (nt) dt  ≤ M | f (t ) | dt +  
f (t)ψ (nt) dt + M |f (t)| dt ≤ 2M ε + ε.
  
a a c d

Ceci termine la solution de la question a).


b) C’est une conséquence directe du résultat de la question précédente, car t → eit est 2π -
 2π
périodique et 0 eit dt = 0.
c) Lorsque f est de classe C1 sur le segment I = [a, b], on procède en intégrant par parties :
 b  b 
int eint 1 b 
In= f (t)e dt = f (t) − f (t)e int dt (∗∗∗)
a in a in a
donc  
b
1 
|I n| ≤ |f (b)| + |f (a)| + |f (t)| dt ,
n a
d’où In = O(1/n).
d) Lorsque n ∈ N∗, le changement de variable u = t − π/n donne
 +∞  +∞  +∞
In = f (t)eint dt = f (u + π/n)einu+iπ du = − f (t + π/n)eint dt,
−∞ −∞ −∞

d’où on déduit  +∞
2I n = (f (t) − f (t + π/n))eint dt. (∗∗∗∗)
−∞
L’intégrande de cette intégrale converge simplement vers 0 lorsque n → ∞, mais son module
n’est pas majoré par une fonction intégrable ϕ indépendamment de n. La convergence de (****)
 +∞
vers 0 s’obtient en écrivant |I n| ≤ 12 −∞ |f (t) − f (t + π/n)| dt et en procédant comme dans la
solution de la question a) du problème 6 page 180.
3. INTÉGRALE SUR UN INTERVALLE QUELCONQUE 159

Remarque. - Lorsque I est un segment et si f est suffisamment régulière, on peut obtenir


un développement asymptotique de In en itérant l’intégration par parties dans (***).
– Le résultat de la question c) se généralise aisément lorsqu’on remplace eint par une
T
fonction continue T -périodique ϕ telle que 0 ϕ(t) dt = 0.

Exercice 7. a) Soitf : [0, +∞[→ R une fonction continue par morceaux et décroissante,
+∞
telle que l’intégrale 0 f (t) dt converge et est non nulle. Pour tout t > 0, prouver la
convergence de
+∞

f (nt)
n=1
et donner un équivalent de cette dernière expression lorsque t → 0+.
b) (Application.) Donner un équivalent, lorsque x → 1 −, de la série entière
+∞
 2
x → xn .
n=1

Solution. a) La fonction f décroı̂t et son intégrale converge, on en déduit que f tend vers 0 à
l’infini. En utilisant encore la décroissance de f , on en déduit que f est positive.
Ensuite, on procède comme dans l’exercice 5. La fonction f étant décroissante, on a
 (n+1)t  nt

∀t > 0, ∀n ∈ N , f (x) dx ≤ tf (nt) ≤ f (x) dx. (∗)
nt (n−1)t

La dernière inégalité entraı̂ne par sommation


N
  Nt  +∞

∀t > 0, ∀N ∈ N , t f (nt) ≤ f (x) dx ≤ f (x) dx,
n=1 0 0

autrement dit, les sommes partielles de la série étudiée sont majorées (lorsque t > 0 est fixé).
Les termes de la série étant positifs, on en déduit qu’elle converge, et ceci pour tout t > 0.
Maintenant, par sommation de (*) sur n ∈ N ∗, on obtient
 +∞ +∞
  +∞
∀t > 0, f (x) dx ≤ t f (nt) ≤ f (x) dx,
t n=1 0

et on en déduit que
+∞
  +∞ +∞
  +∞
1
lim t f (nt) = f (x) dx donc f (nt) ∼ f (x) dx
t→0+
n=1 0 n=1
t 0

(cette dernière assertion a bien un sens car l’intégrale est non nulle par hypothèse).
2
b) En posant x = e−t , on a
+∞
 +∞
 +∞

2 2
∀x ∈ ]0, 1[, xn = e−(nt) = f (nt)
n=1 n=1 n=1
−u2
 +∞ −u2
avec f : u → e . En posant c = 0
e du, on en déduit grâce au résultat de la question
précédente que
+∞
 2 c c c
xn ∼ =√ ∼ √ .
n=1
t − log x 1−x

Remarque. On peut montrer que c = π/2 (voir l’exercice 2 page 167).
160 3. INTÉGRATION

Exercice 8. Soit f : [0, +∞[→ R une fonction continue telle que l’intégrale
 +∞
f (t)
dt
1 t
converge.
a) Soient deux nombres réels strictement positifs a et b. Montrer que l’intégrale
 +∞
f (at) − f (bt)
dt
0 t
converge et calculer sa valeur.
b) (Application.) Si a, b > 0, calculer l’intégrale
 +∞ −at
e − e−bt
dt.
0 t

Solution. a) Nous allons en même temps prouver la convergence de l’intégrale et donner sa


valeur. Posons
f (at) − f (bt)
g : ]0, +∞[ → R t → .
t   
Le changement de variable x = αt montre que pour tout α > 0, 1+∞ f (αt)/t dt est de même
 +∞    +∞
nature que α f (t)/t dt, c’est-à-dire convergente. L’intégrale 1 g (t) dt converge donc. En
définitive, il s’agit pour nous de prouver l’existence et donner la valeur de
 +∞
lim g (t) dt.
x→0
x>0 x

En effectuant les changements de variable u = at et v = bt, on trouve respectivement


 +∞  +∞  +∞  +∞
f (at) f (u) f (bt) f (v )
∀x > 0, dt = du et dt = dv,
x t ax u x t bx v
d’où on tire
 +∞  +∞  +∞  bx
f (t) f (t) f (t)
∀x > 0, g (t) dt = dt − dt = dt.
x ax t bx t ax t
La fonction f étant continue, on a d’après la première formule de la moyenne
 +∞  bx  bx
f (t) dt b
∀x > 0, ∃cx ∈ ]ax, bx[, g (t) dt = dt = f (cx ) = f (cx ) log .
x ax t ax t a
Comme f est continue en 0 et que cx tend vers 0 avec x, on en déduit que
 +∞
b
lim g (t) dt = f (0) log .
x→0
x>0 x a
Autrement dit, l’intégrale proposée converge et sa valeur est f (0) log(b/a).
b) Il suffit d’appliquer le résultat de la question précédente à la fonction f : x → e −x qui vérifie
bien les hypothèses requises. En 0, cette fonction prend la valeur 1, donc l’intégrale converge et
 +∞ −at
e − e−bt b
dt = log .
0 t a

Exercice 9. Soit f : R+ → R une fonction continue par morceaux, positive, telle que
f 2 est intégrable sur R+ . Montrer que lorsque x → +∞,
 x
√ 
f (t) dt = o x .
0
4. INT ÉGRALES DÉPENDANT D’UN PARAM ÈTRE, ÉQUIVALENTS D’INTÉGRALES 161

Solution. On pense bien sûr à utiliser l’inégalité


√ de Schwarz. En l’utilisant sur le domaine [0, x]
on montre seulement que 0x f (t) dt = O ( x), mais on n’a pas le petit o. On procède autrement.
 +∞
Soit ε > 0 et soit x0 > 0 tel que x0 f 2(t) dt < ε 2 . Pour tout x > x0, l’inégalité de Schwarz
entraı̂ne
 x  x  x 1/2  x 1/2
2 √
f (t) dt = 1 · f (t) dt ≤ dt · f (t) dt ≤ ( x − x 0 )ε,
x0 x0 x0 x0
 x0 √
donc si on choisit x1 > x 0 tel que 0 f (t) dt ≤ ε x1 , on a
 x  x0
√ √ √ √
∀x > x1 , f (t) dt ≤ f (t) dt + ε x − x0 ≤ ε( x 1 + x) ≤ 2ε x.
0 0
x
Par ailleurs f est positive donc 0 f également. On en déduit le résultat.

4. Intégrales dépendant d’un paramètre, équivalents


d’intégrales
4.1. Intégrales dépendant d’un paramètre
Dans cette sous-partie, I désigne un intervalle quelconque de R, d’extrémités a et b
(avec −∞ ≤ a < b ≤ +∞), et E désigne un e.v.n complet.

 Théorème 1 (Continuit é sous le signe intégral). Soit A un espace métrique et


une application
f : A × I → E (x, t) → f (x, t)
vérifiant les propriétés suivantes
(i) pour tout x ∈ A, l’application f (x, ·) : t → f (x, t) est continue par morceaux sur I ,
(ii) pour tout t ∈ I , l’application f (·, t) : x → f (x, t) est continue sur A
(iii) il existe une fonction positive ϕ, continue par morceaux et intégrable sur I , telle que
f (x, t) ≤ ϕ(t) pour tout x ∈ A (Hypothèse de domination).
Alors l’application
 b
Φ: A→E x → f (x, t) dt
a
est bien définie, et elle est continue sur A.
Démonstration. L’hypothèse de domination montre que f (x, ·) est intégrable pour tout x ∈ A,
donc Φ est bien définie. Pour prouver qu’elle est continue en tout point x ∈ A, il suffit de montrer
que pour toute suite (xn ) dans A convergeant vers x, on a bien limn→∞ Φ(xn ) = Φ(x). Soit (xn )
une telle suite. La suite de fonctions (fn ) définie par fn : I → E t → f (x n, t) converge
simplement vers t → f (x, t) d’après l’hypothèse (ii). L’hypothèse (iii) nous permet d’appliquer
 
le théorème de convergence dominée, qui donne limn→∞ ab fn (t) dt = ab f (x, t) dt, c’est-à-dire
limn→∞ Φ(xn) = Φ(x). 

 Théorème 2 (D érivation sous le signe intégral). Soit A un intervalle de R et


f : A × I → E (x, t) → f (x, t) une application vérifiant les propriétés suivantes
(i) pour tout x ∈ A, l’application f (x, ·) : t → f (x, t) est continue par morceaux et
intégrable sur I ,
∂f
(ii) f admet une dérivée partielle ∂x
vérifiant les hypothèses du théorème précédent.
162 3. INTÉGRATION

Alors l’application
 b
Φ: A→E x → f (x, t) dt
a
est de classe C 1 sur A et on a
 b
 ∂f
∀x ∈ A, Φ (x) = (x, t) dt.
a ∂x
Démonstration. Soit x ∈ A et (xn ) une suite dans A{x} convergeant vers x. La suite de
fonctions (gn) définie par gn : I → E t → (f (x n , t) − f (x, t))/(xn − x) converge simplement
vers ∂f
∂x
(x, ·) sur I . La fonction g n est bien continue par morceaux et intégrable sur I . De plus,
comme  ∂f ∂x (y, t ) ≤ ϕ(t) pour tout y ∈ I , l’inégalité des accroissements finis entraı̂ne que
gn (t) ≤ ϕ(t). Ainsi, on peut
 ∂fappliquer le théorème
 de convergence dominée qui nous assure la
convergence de I gn vers I ∂x (x, ·). Comme I g n = (Φ(x n) − Φ(x))/(xn − x), nous venons de

démontrer que Φ est dérivable en x et que Φ(x) = I ∂f ∂x (x, ·). La dernière intégrande vérifiant
les hypothèses du théorème de continuité sous le signe intégral, on en déduit que Φ est continue.

Remarque 1. - Les résultats des deux théorèmes précédents restent vrais lorsque l’hy-
pothèse de domination est vérifiée uniquement sur un voisinage de tout point de A (la
continuité, la dérivabilité, sont des propriétés locales). C’est en particulier le cas si A ⊂ Rn
et si l’hypothèse de domination est vraie sur tout compact K de A.
– Lorsque les intégrales définissant Φ sont semi-convergentes, les théorèmes
 précédents ne
s’appliquent plus. On passe en général par une suite de fonctions f n (x) = K n f (x, ·) où les
Kn sont des segments de I qui tendent vers I , puis on prouve des résultats de convergence
uniforme pour (fn ) (voir un exemple dans la solution 2/b) de l’exercice 4 page 168).
Dans le cas où I est un segment de R (et A un intervalle de R), et f continue,
l’hypothèse de domination n’est plus nécessaire, comme l’exprime le corollaire suivant.

 Corollaire 1. Soit A un intervalle de R et [a, b] un segment de R. Soit une application


f : A × [a, b] → E (x, t) → f (x, t) continue sur A × [a, b]. Alors l’application
 b
Φ : A → E x → f (x, t) dt
a

est continue sur A. Si de plus, f est dérivable par rapport à x et si ∂f


∂x
est continue sur
1 
 b ∂f
A × [a, b], alors Φ est de classe C sur A et on a Φ (x) = a ∂x (x, t) dt.
Démonstration. Soit K un compact de A. On peut appliquer le théorème 1 sur K × [a, b] (l’hy-
pothèse de domination est vérifiée car f , continue sur le compact K × [a, b], y est bornée) qui
prouve que Φ est continue sur K. Donc Φ est continue sur tout compact de A, donc sur A tout
entier. On montre de la même manière les résultats relatifs à la dérivation de Φ. 

Remarque 2. On peut également obtenir ce corollaire sans passer par le théorème de


convergence dominée, à partir de l’uniforme continuité de f (et ∂f
∂x
) sur K × [a, b].
La fonction gamma. La fonction gamma est une fonction classique définie par
 +∞
Γ : ]0, +∞[ → R x → e−t t x−1 dt.
0

Cette fonction vérifie les propriétés suivantes (démontrées dans le sujet d’étude 1 page 315,
plus largement consacré à l’étude de cette fonction)
4. INT ÉGRALES DÉPENDANT D’UN PARAM ÈTRE, ÉQUIVALENTS D’INTÉGRALES 163

— La fonction Γ est de classe C ∞ sur ]0, +∞[ et pour tout n ∈ N ∗ on a


 +∞
(n)
∀x > 0, Γ (x) = (log t) ne−ttx−1 dt.
0
— On a la relation fonctionnelle Γ(x + 1) = xΓ(x) pour tout x > 0.
— En particulier, Γ(1)
√ = 1 et Γ(n + 1) = n! pour tout entier n ∈ N.
2
— On a Γ(1/2) = π (le changement de variable t = u donne Γ(1/2) = 2I avec
+∞ 2
I = 0 e−u du et le calcul de I est classique — voir l’exercice 2 page 167).

4.2. Équivalents d’intégrales


Intégration des relations de comparaison. Le théorème qui suit complète le résultat
de la proposition 5 de la page 149.
 Théorème 3. Soient [a, b[ un intervalle semi-ouvert de R (avec −∞ < a < b ≤ +∞),
E un R-espace de Banach, f : [a, b[ → E et g : [a, b[ → R+∗ deux applications continues
par morceaux sur [a, b[.
b
(i) Si l’intégrale a g (t) diverge, alors lorsque x → b− , 
 x x
— la relation f = O (g ) entraı̂ne f (t) dt = O g (t) dt ,
 xa  x a 
— la relation f = o(g ) entraı̂ne f (t) dt = o g (t) dt ,
 x a  x a

— la relation f ∼ g entraı̂ne f (t) dt ∼ g (t) dt.


b a a
(ii) Si l’intégrale a g (t) dt converge, alors lorsque x → b −,
 b  b 
— la relation f = O (g ) entraı̂ne f (t) dt = O g (t) dt ,
 bx  b x 
— la relation f = o(g ) entraı̂ne f (t) dt = o g (t) dt ,
 b x  b x

— la relation f ∼ g entraı̂ne f (t) dt ∼ g (t) dt.


x x
Démonstration. Montrons la première assertion de (i). Si f = O (g ), alors
∃c ∈ ]a, b[, ∃M > 0, ∀t ∈ [c, b[, f (t) ≤ M g(t).
Ainsi,   x
 x 
∀x ∈ [c, b[,  f (t) dt ≤ M g (t) dt.
 
c c
b c d
Or a g (t) dt diverge, donc il existe d ∈ [c, b[ tel que  a f (t) dt ≤ a g (t) dt. On a donc
 x  d  x  x
 
∀x ∈ [d, b[,  
f (t) dt ≤ g (t) dt + M g (t) dt ≤ (M + 1) g (t) dt,

a a c a
d’où le premier résultat.
Prouvons maintenant la seconde assertion de (i). Donnons nous ε > 0. D’après les hypothèses,
∃c ∈ ]a, b[, ∀t ∈ [c, b[, f (t) ≤ εg (t).
Ainsi,    x
 x 
∀x ∈ [c, b[,  
f (t) dt  ≤ ε g (t) dt.

c c
b c d
Or a g (t) dt diverge, donc il existe d ∈ [c, b[ tel que  a f (t) dt ≤ ε a g (t) dt. On a donc
 x   d x  x
 
∀x ∈ [d, b[,  
f (t) dt ≤ ε g (t) dt + ε g (t) dt ≤ 2ε g (t) dt,

a a c a
164 3. INTÉGRATION

d’où le second résultat.


Si f ∼ g et toujours sous les hypothèses de (i), la troisième assertion se montre en écrivant
f − g = o(g ) et en utilisantle résultatprécédent.
Les assertions de (ii) se montrent de manière analogue (et c’est plus facile). 

Exemple 1. Au voisinage de +∞, on a pour tout α > 0


   x  x 
1 1 dt α−1
=o 1−α
donc log x = =o t dt = o(x α ).
x x 1 t 1

Méthode de Laplace. Nous allons donner un résultat général sur les équivalents d’inté-
grales dont l’intégrande dépend d’un paramètre, qui s’applique aux intégrales de la forme
 +∞
I (t) = g (x) e th(x) dx lorsque t → +∞.
0
La démarche utilisée s’appelle la méthode de Laplace. Elle ne figure pas au programme
des classes préparatoires, mais les techniques utilisées sont instructives et permettent de
résoudre les nombreux exercices derrière lesquels se cache cette méthode.
Commençons par l’examen d’un cas particulier.
Lemme 1. Soient α > −1, β > 0, c > 0 et b vérifiant 0 < b ≤ +∞. Alors lorsque
t → +∞,
 b  
α −tcx β 1 α+1
J (t) = x e dx ∼ Γ (ct)−(α+1)/β ,
0 β β
où la fonction Γ est définie page 162.
Démonstration. Il suffit d’effectuer le changement de variable u = tcxβ , ce qui donne
 cb βt
1 −(α+1)/β
J (t) = (ct) u (α+1)/β−1e −u du,
β 0
d’où le résultat lorsque t → +∞. 
Nous pouvons maintenant énoncer le résultat principal de cette sous-partie.
Théorème 4. Soient g et h : ]0, +∞[ → R deux applications continues par morceaux
vérifiant
(i) la fonction x → g (x)eh(x) est intégrable sur R +,
(ii) ∃δ0 > 0, ∀δ ∈ ]0, δ0 [, ∀x ≥ δ, h(x) ≤ h(δ ),
+
(iii) Lorsque x → 0 , on a
g (x) ∼ Axα (α > −1) et h(x) = a − cx β + o(xβ ) (c > 0, β > 0).
Alors lorsque t → +∞, on a
 +∞  
th(x) A α + 1 at
I (t) = g (x)e dx ∼ Γ e (ct) −(α+1)/β .
0 β β
Démonstration. Multipliant par e−at /A, on peut déjà supposer a = 0 et A = 1. Posons d’abord,
pour simplifier l’écriture,  
1 α+1
ϕ(t) = Γ (ct) −(α+1)/β .
β β
Nous allons, pour ε ∈ ]0, 1[, montrer successivement
(i) ∃δ ∈ ]0, δ 0[, ∃t 1 > 0 tels que, pour tout t ≥ t 1 ,
δ
2 −(α+1)/β
(1 − ε) (1 + ε) ϕ(t) ≤ g (x)e th(x) dx ≤ (1 + ε)2 (1 − ε) −(α+1)/β ϕ(t);
0
4. INT ÉGRALES DÉPENDANT D’UN PARAM ÈTRE, ÉQUIVALENTS D’INTÉGRALES 165

(ii) δ > 0 et t 1 étant ainsi fixés, déterminer t 2 > 0 tel que


 +∞
∀t ≥ t 2 , |g (x)| eth(x) dx ≤ εϕ(t).
δ
On en conclura que pour tout t ≥ sup{t1 , t 2}, on a
   
(1 − ε)2 (1 + ε) −(α+1)/β − ε ϕ(t) ≤ I (t) ≤ (1 + ε)2 (1 − ε) −(α+1)/β + ε ϕ(t),
ce qui, en vertu de la continuité des fonctions de ε qui interviennent, prouvera le théorème.
1. Par hypothèse, il existe δ ∈ ]0, δ0 [ tel que

(1 − ε)x α ≤ g (x) ≤ (1 + ε)xα
∀x ∈ ]0, δ] , ,
−c(1 + ε)x β ≤ h(x) ≤ −c(1 − ε)xβ
donc   
δ δ δ
α −c(1+ε)tx β th(x) β
(1 − ε) x e dx ≤ g (x)e dx ≤ (1 + ε) xα e−c(1−ε)tx dx. (∗)
0 0 0
Le nombre δ étant fixé, on sait d’après le lemme 1 que les deux membres extrêmes de (*) on
respectivement pour équivalent, lorsque t → +∞
 
1−ε α+1  −(α+1)/β
Γ c(1 + ε)t = (1 − ε)(1 + ε)−(α+1)/β ϕ(t)
β β
et  
1+ε α+1  −(α+1)/β
Γ c(1 − ε)t = (1 + ε)(1 − ε)−(α+1)/β ϕ(t).
β β
Par définition d’un équivalent, il existe donc t1 > 0 tel que pour tout t ≥ t 1, on ait
 δ  
β
(1 − ε) xαe −c(1+ε)tx dx ≥ (1 − ε) (1 − ε)(1 + ε) −(α+1)/β ϕ(t)
0
et  δ  
α −c(1−ε)txβ −(α+1)/β
(1 + ε) x e dx ≥ (1 + ε) (1 + ε)(1 − ε) ϕ(t) .
0
Il est donc clair, d’après (*), que l’on a (i) pour tout t ≥ t 1.
2. Posons h(δ) = −µ < 0. D’après les hypothèses, on a h(x) + µ ≤ 0 pour tout x ≥ δ , donc
∀x ≥ δ, ∀t > 1, th(x) = (t − 1)h(x) + h(x) ≤ −(t − 1)µ + h(x)
 +∞
ce qui entraı̂ne la convergence de δ |g (x)|eth(x) dx et l’inégalité
 +∞  +∞
th(x) −(t−1)µ
|g (x)|e dx ≤ e |g (x)|eh(x) dx.
δ δ
Or µ > 0, d’où l’existence d’un réel t2 > 0 pour lequel on a (ii). 
b
Remarque 3. Le cas d’intégrales de la forme 0 g (x)eth(x) dx avec 0 < b < +∞, lorsque g
et h sont continues par morceaux sur ]0, b], se ramène au cas du théorème en prolongeant
g et h sur ]0, +∞[ par g (x) = 0 et h(x) = a − 1 pour x > b.
Le cas typique d’application du théorème précédent fait l’objet du corollaire qui suit.
Corollaire 2. Soient g et h deux fonctions réelles de classe C 2 sur ]a, b[ (avec −∞ ≤
a < b ≤ +∞) telles que
(i) La fonction x → g (x)eh(x) est intégrable sur ]a, b[,
(ii) h ne change de signe qu’en un seul point c ∈ ]a, b[ où de plus h atteint son maximum
et où g(c) = 0, h (c) < 0.
Alors lorsque t → +∞, on a
 b   
1 2
g (x)eth(x) dx ∼ Γ g (c)e th(c) .
a 2 −th (c)
166 3. INTÉGRATION

  
Démonstration. On coupe l’intégrale en deux : ab = ac + cb, puis on ramène c à 0 en effectuant
un changement de variable affine. On utilise ensuite le théorème précédent. Pour chacun des
deux cas, on a ici
h (c)
α = 0, A = g (c), β = 2, c=− , a = h(c),
2
et l’équivalent est le double de celui du théorème précédent (il y a deux intégrales). 

Remarque 4. On peut utiliser la valeur Γ(1/2) = π dans le corollaire précédent.
Dans la pratique, l’intégrande n’a pas toujours la forme de celle requise pour appliquer
la méthode de Laplace. On effectue souvent un changement de variable pour amener le
maximum de la fonction apparaissant dans l’exponentielle à une abscisse fixe puis on
applique le corollaire précédent. C’est cette technique qui est utilisée dans l’exemple qui
suit.
Exemple 2. On veut trouver le comportement asymptotique, lorsque x → +∞, de
 +∞
Γ(x + 1) = ex log t−t dt.
0

Comme il est dit plus haut, on cherche l’abscisse t du maximum de la fonction h(t) =
x log t − t (c’est ce maximum qui va dicter le comportement de Γ en +∞). On a h (t) =
x/t − 1, le maximum est donc atteint en t = x. Pour se ramener au cas où h atteint
son maximum en une abscisse indépendante de x, on effectue le changement de variable
u = t/x. On trouve
 +∞
x+1
Γ(x + 1) = x e x(log u−u) du.
0
La fonction u → log u − u atteint son maximum en u = 1, et on peut appliquer le corollaire,
qui donne
  
x+1 1 −x 2 √
Γ(x + 1) ∼ x Γ e = 2π x x+1/2e −x .
2 x
Ce résultat est connu sous le nom de formule de Stirling. Elle généralise celle obtenue
dans l’exercice 3 page 219 pour x entier (car Γ(n + 1) = n! pour tout n ∈ N∗ ).
Remarque 5. Dans les exercices qui suivent, on utilisera tels quels les résultats concer-
nant la méthode de Laplace. Néanmoins, le jour du concours, il faut être capable de
tout redémontrer (en appliquant directement aux fonctions concernées les démonstrations
précédentes).
4.3. Exercices
 Exercice 1. On considère une application f : [0, 1] → R strictement croissante telle que
f (0) = 0 et f (1) = 1. Montrer que
 1
lim f (t) n dt = 0.
n→+∞ 0

Solution. C’est classique. On procède comme suit.


Comme f est strictement croissante sur [0, 1], on a 0 = f (0) ≤ f (x) < f (1) = 1 pour
0 ≤ x < 1. Ainsi, la suite de fonctions (f n ) converge simplement vers la fonction ϕ définie par
ϕ(t) = 0 pour t ∈ [0, 1[ et ϕ(1) = 1. Par ailleurs, on a la majoration 0 ≤ f n ≤ 1 indépendante
de
 1 n. On peut donc
 1 appliquer le théorème de convergence dominée, qui assure la converge de
n
f (t) dt vers 0 ϕ(t) dt = 0.
0
4. INT ÉGRALES DÉPENDANT D’UN PARAM ÈTRE, ÉQUIVALENTS D’INTÉGRALES 167

 +∞ 2
Exercice 2 (Intégrale de Gauss : 0 e −t dt). Le but de l’exercice est de donner
deux méthodes pour calculer l’intégrale de Gauss
 +∞
2
I= e −t dt.
0

1/ (Première méthode.) Exprimer l’application


 1 2 2
e −(t +1)x
g : [0, +∞[→ R x → dt
0 t2 + 1
x −t2
en fonction de x → 0
e dt. En déduire I.

2/ (Seconde méthode.) Soit n ∈ N∗ . Pour tout t ∈ [0, n], montrer
 n  −n
t2 −t2 t2
1− ≤e ≤ 1+ .
n n
En déduire I en utilisant les intégrales de Wallis.

Solution. 1/ La fonction
2 2
e−(t +1)x
[0, +∞[ ×[0, 1] (x, t) →
t2 + 1
admet une dérivée partielle par rapport à x continue. On en déduit (théorème de dérivabilité
sous le signe intégral, et plus précisément le corollaire 1 page 162) que g est dérivable et que
 1 1
 −(t 2+1)x2 −x 2 2
∀x ≥ 0, g (x) = −2x e dt = −2xe e−(tx) dt,
0 0
ce qui, après le changement de variable u = tx donne
 x  x
 −x2 −u2  2
g (x) = −2e e du = −2f (x)f (x) avec f : x → e −u du.
0 0
En intégrant, on en déduit
  π
∀x ≥ 0, g(x) − g (0) = − f 2(x) − f 2 (0) donc g (x) = − f2 (x). (∗)
4
Les inégalités
 1
dt
−x2
0 ≤ g (x) ≤ e 2
entraı̂nent lim g (x) = 0,
0 1+t x→+∞

et la fonction f étant positive, on en déduit avec (*) que


  +∞ √
π −t2 π
lim f (x) = ce qui s’écrit I = e dt = .
x→+∞ 4 0 2

2/ La fonction logarithme est concave, elle se trouve donc en dessous de sa tangente en 1, ce


qui s’écrit log(1 + x) ≤ x pour x > −1. En particulier pour tout n ∈ N∗ on a
   
√ t2 t2 t2 t2
∀t ∈ [0, n[, log 1 − ≤− et log 1 + ≤ .
n n n n
En multipliant respectivement par n et −n puis en prenant l’exponentielle, on en déduit
 n  −n
√ t2 −t2 t2
∀t ∈ [0, n[, 1− ≤e ≤ 1+ .
n n
On a donc
n n +∞  −n
 √  √ 
n
∗ t2 −t 2 t2
∀n ∈ N , 1− dt ≤ e dt ≤ 1+ dt.
0 n 0 0 n
168 3. INTÉGRATION

√ √
En effectuant le changement de variable t = n cos u dans le membre de gauche et t = n cotan u
dans le membre de droite, cette dernière assertion s’écrit aussi
 π/2  √n  π/2
∗ √ 2n+1 −t2 √
∀n ∈ N , n sin u du ≤ e dt ≤ n sin2n−2 u du. (∗∗)
0 0 0

Les intégrales de Wallis vérifient l’équivalent (voir l’exercice 1 page 130)


 π/2 
n π
sin u du ∼ lorsque n → +∞.
0 2n

Ceci montre que les deux termes extrêmes de (**) tendent vers π/2 lorsque n → +∞, et on
en déduit en faisant tendre n vers l’infini dans (**) que
 +∞  √x √
−t2 −t2 π
I= e dt = lim e dt = .
0 x→+∞ 0 2

Remarque. Il existe d’autres moyens de calculer cette intégrale (voir l’exemple 2 page 355
par exemple).

Exercice 3. Soit f : R → R une fonction de classe C ∞ nulle en 0. Montrer que


l’application

f (x)/x si x = 0
g : R → R x →
f (0) si x = 0
est de classe C ∞.
x
Solution. Comme f (0) = 0, on a f (x) = 0 f (t) dt pour tout x ∈ R. Pour x fixé, le changement
de variable t = λx fournit
 1  1

f (x) = x f (λx) dλ donc g (x) = f (λx) dλ.
0 0

La fonction f étant de classe C∞ , on en déduit en appliquant par récurrence le théorème de


dérivation sous le signe intégral, que g est de classe C∞.
Remarque. Ce résultat est un cas particulier du lemme d’Hadamard lorsque n = 1 (voir
l’exercice 4 page 331).

Exercice 4. 1/ Calculer, pour tout x ≥ 0, la valeur de l’intégrale


 +∞
sin(xt) −t
I (x) = e dt.
0 t

2/ a) Soit α > 0. Calculer, pour tout x ≥ 0, la valeur de


 +∞
I (x) = e−t eixt t α−1 dt
0
 +∞
(on exprimera I en fonction de Γ(x) = 0 tx−1 e −t dt).
b) En déduire, pour tout α ∈ ]0, 1[, la valeur de
 +∞
J (α) = t α−1 eit dt.
0
4. INT ÉGRALES DÉPENDANT D’UN PARAM ÈTRE, ÉQUIVALENTS D’INTÉGRALES 169

3/ Pour tout x ≥ 0, calculer


 +∞
2
I (x) = e−t e2iπtx dt.
−∞

Solution. L’idée est de trouver une équation différentielle vérifiée par chacune de ces intégrales
puis de la résoudre pour exprimer explicitement l’intégrale correspondante.
1/ L’intégrande est bien intégrable sur ]0, +∞[ (elle est égale à x + o(1) lorsque t → 0). La dérivée
de l’intégrande par rapport à x est e−t cos xt, dont la valeur absolue est majorée indépendamment
de x par e−t, intégrable sur ]0, +∞[. En vertu du théorème de dérivation sous le signe intégral,
on en déduit que I est dérivable et que
 +∞  +∞   
+  −t −t+ixt 1 1
∀x ∈ R , I (x) = e cos xt dt =  e dt =  = .
0 0 1 − ix 1 + x2
On en déduit  x
+ dt
∀x ∈ R , I(x) = I (0) + = arctan x.
0 1 + t2
2/ a) En procédant comme dans la solution de la question précédente, on montre que I est
dérivable et que
 +∞
+ 
∀x ∈ R , I (x) = i e −teixt tα dt.
0
Par ailleurs, une intégration par parties donne
     +∞
t α +∞ −1 + ix i+x 
∀x ≥ 0, I(x) = e(−1+ix)t · − e−t eixt tα dt = − I (x).
α 0 α 0 α
Cette équation différentielle vérifiée par I s’intègre, compte tenu du fait que
 
dx x−i 1
= 2
dx = log(x 2 + 1) − i arctan x + k, k ∈ C,
i+x x +1 2
et on tire
  
1 2
∃C ∈ C, I(x) = C exp −α log(x + 1) − i arctan x = C (x2 + 1) −α/2 eiα arctan x .
2
Or I (0) = C = Γ(α), donc finalement
∀x ∈ R +, I(x) = Γ(α)(x2 + 1) −α/2e iα arctan x .

b) Le changement de variable u = xt dans l’intégrale définissant I (x) donne


 +∞
1
∀x > 0, I(x) = α e−u/xeiuu α−1 du.
x 0
Lorsque x → +∞, on a e−u/x → 1 et on s’attend à ce que la dernière intégrale tende vers J (x).
C’est ce dernier point que nous montrons maintenant. On ne peut pas ici utiliser directement
le théorème de continuité sous le signe intégral sur tout le domaine d’intégration. Définissons la
fonction  +∞
I : R+ → C y → e−uy eiuuα−1 du.
0

Pour y > 0, l’existence de I(y ) est immédiate. Pour y = 0, son existence est une conséquence
de la règle d’Abel (voir le théorème 5 page 152). Pour montrer la continuité de I en 0, on va
montrer que  I est limite uniforme de fonctions continues. On considère la suite de fonctions (
In )
définie par  n
∀n ∈ N ∗, ∀y ≥ 0, In(y) = e−uy eiu uα−1 du.
1/n
170 3. INTÉGRATION

La seconde formule de la moyenne donne


∀n ∈ N ∗, ∀y ≥ 0, ∀X > n, ∃a, b ≥ n,
X  a  b 
−uy iu α−1 −ny α−1 α−1
e e u du = e cos u u du + i sin u u du
n n n
donc pour tout n ∈ N ∗ et pour tout y ≥ 0,
   1/n  t   
     t 

In(y ) − I (y ) ≤ u α−1 
du + sup  cos u u α−1
du + sup 
 sin u u α−1
du ,
0 t≥n n t≥n n

et on en déduit que ( 
In ) converge uniformément vers  I sur R+ . Le théorème de continuité sous
le signe intégral assure la continuité de In en 0 pour tout n ∈ N∗ , donc  I, limite uniforme de
fonctions continues sur R+ , est continue sur R+ . En particulier, I est continue en 0 + ce qui
s’écrit aussi
lim xα I (x) = J (α),
x→+∞
et d’après le résultat de la question précédente on en déduit
J (α) = Γ(α)eiαπ/2.

3/ En procédant comme dans la solution de la question 1/, on montre que I est dérivable et que
 +∞
+  2
∀x ∈ R , I (x) = 2iπ te−t e2iπtx dt.
−∞
Par ailleurs, une intégration par parties donne
   +∞
1 −t2 2iπxt +∞ 1 2 1 1 
∀x ≥ 0, I(x) = e e + 2te −t e2iπxt dt = I (x).
2iπx −∞ 2iπx −∞ iπx 2iπ
En résolvant cette équation différentielle vérifiée par I , on trouve
2 2
I(x) = I (0)e−π x .
∀x ≥ 0,

La constante I (0) est calculable : elle vaut π (voir l’exercice 2).

Exercice 5. Pour tout t > 0, on pose


 +∞
e−tx
I (t) = √3
dx.
1 x3 + 1
Donner un équivalent de I (t) lorsque t tend vers 0+.

Solution. La convergence de l’intégrale donnant I (t) pour tout t > 0 est immédiate. Pour trouver
son équivalent lorsque t → 0+ il est commode de commencer par effectuer le changement de
variable u = tx (ceci pour localiser la zone qui contribue au terme dominant de l’intégrale), ce
qui donne  +∞
e −u
∀t > 0, I(t) = √
3 3
du.
t u + t3
Lorsque t → 0+ , la contribution prépondérante de l’intégrale se produit lorsque u est proche de
t. Dans cette zone, l’intégrande se comporte comme (u3 + t3 )−1/3 . Lorsque u est proche de 0
mais grand par rapport à t, la contribution de ce terme reste significative, et  1il est alors bien
approché par 1/u. On s’attend par conséquent à ce que I (t) soit équivalent à t du/u = − log t.
C’est ce que nous allons montrer rigoureusement dans les lignes qui suivent.
Pour tout t > 0, on écrit
 1  1   1  +∞
du 1 1 e−u − 1 e−u
I (t) − = − du + du + du . (∗)
t u t (u3 + t 3) 1/3 u 3
t (u + t )
3 1/3
1 (u3 + t 3)1/3
        
I1 (t) I2(t) I3(t)
4. INT ÉGRALES DÉPENDANT D’UN PARAM ÈTRE, ÉQUIVALENTS D’INTÉGRALES 171

Nous allons prouver que les intégrales I1 (t), I2 (t) et I3 (t) du second membre de cette égalité
sont bornées lorsque t → 0+ . Pour les deux dernières c’est facile car il suffit d’écrire (compte
tenu de la classique inégalité 1 − e−u ≤ u)
 1  1  +∞
u
|I2 (t)| ≤ 3 + t3)1/3
du ≤ du ≤ 1 et | I3 (t )| ≤ e−u du = 1.
t (u t 1

Pour l’intégrale I1 (t) on écrit


    
1
1 t −1/3
∀t > 0, |I1 (t)| = 1− 1+ du,
t u u

et compte tenu du fait que 1 − (1 + x)−1/3 ≤ x/3 pour tout x ≥ 0 (inégalité que l’on obtient par
exemple en utilisant la convexité de la fonction x → (1 + x)−1/3 ), on a
 1 
1 t t 1 du 1
∀t > 0, |I1 (t)| ≤ du = = .
t u 3u 3 t u2 3
Ainsi nous avons montré I 1(t) + I2 (t) + I3 (t) = O(1) lorsque t → 0+, donc grâce à (*) on en
déduit
I (t) + log t = O (1) donc I (t) = − log t + O (1) ∼ − log t lorsque t → 0 +.

Exercice 6. 1/ Donner un équivalent, lorsque t → +∞ de


 +∞  π
−αx x
a) I (t) = x t dx (α > 0) b) I (t) = (sin x)xt dx
0 0

(on pourra utiliser le théorème 4 page 164 sur la méthode de Laplace).


2/ Donner un équivalent, lorsque t → 0 +, de
 +∞  α 
x
I (t) = exp − tx dx (0 < α < 1).
0 α

Solution. 1/ a) On ne peut pas appliquer tel que le théorème 4 car dans l’expression tx = e (log t)x,
la fonction dans l’exponentielle est croissante en x. On procède comme il est indiqué dans le
commentaire du corollaire 2.
On écrit x−αx tx = exp(f (x) ) où f (x) = log t · x − αx log x. Comme f (x) → −∞ lorsque
x → +∞, l’intégrale I (t) converge. Cherchons l’abscisse x du maximum de f . On a
f  (x) = log t − α(1 + log x)
donc le maximum de f est atteint au point x = t1/α /e. Pour le ramener à une abscisse fixe, on
effectue le changement de variable x = ut1/α/e. On obtient
 +∞
t 1/α
I (t) = s esh(u) du où s = et h(u) = αu(1 − log u).
0 e
La fonction h admet un unique maximum en u = 1, et on a h(1) = α, h (1) = −α. On peut
donc appliquer le corollaire 2 à cette dernière intégrale, ce qui donne
 +∞   
sh(u) 1 αs 2
e du ∼ Γ e .
0 2 αs

Donc, compte tenu de l’expression de s en fonction de t et du fait que Γ(1/2) = π, on trouve
  
2π 1/(2α) αt1/α
I (t) ∼ t exp .
eα e
172 3. INTÉGRATION

b) Pour se ramener aux hypothèses usuelles d’application de la méthode de Laplace, on com-


mence par effectuer le changement de variable u = π − x, ce qui donne
 π π
t
I (t) = (sin u)(π − u) du = g (u)e th(u) du
0 0
u
avec g (u) = sin u ∼ u et h(u) = log(π − u) = log π − + o(u) (u → 0+ ). On peut donc appliquer
π
le théorème 4 qui donne
 −2
t t πt+2
I (t) ∼ Γ(2) π = 2 .
π t
2/ On ne peut pas appliquer telle quelle la méthode de Laplace car l’intégrale est étudiée lorsque
le paramètre t tend vers 0+ . Pour se ramener à un paramètre qui tend vers +∞, on effectue le
changement de variable u = tx, qui donne
  α 
1 +∞ u
I (t) = exp − u du.
t 0 αtα

La fonction f (u) = α −u prend son maximum pour u = t α/(α−1) , ce qui nous amène à effectuer
αt
le changement de variable u = v tα/(α−1) . On trouve

s +∞ sh(v) vα
I (t) = e dv, où s = tα/(α−1) et h(v) = − v.
t 0 α
La fonction h admet un maximum unique qu’elle atteint en v = 1. De plus,
1−α
h(1) = et h (1) = α − 1.
α
En appliquant le corollaire 2, on obtient donc
 +∞   
1 2
esh(v) dv ∼ Γ e (1−α)s/α (s → +∞).
0 2 (1 − α)s
Or s = t−α/(1−α) et 0 < α < 1. Lorsque t → 0+ , on a donc s → +∞, et finalement
  
2π 2(1α−2 1 − α − α
I (t) ∼ t −α) exp t 1−α , (t → 0 +).
1−α α

Exercice 7. Soient a ∈ R et g : [a, +∞[ → R une application de classe C 1 . On suppose


que g ne s’annule pas au voisinage de l’infini et que lorsque x → +∞, on a
g (x) µ
∼ , µ = 0, µ =
 −1.
g (x) x
Montrer que
x
 +∞ xg (x)
si µ > −1, a
g (t) dt diverge et g (t) dt ∼ (x → +∞)
a µ+1
 +∞
 +∞ xg (x)
si µ < −1, a
g (t ) dt converge et g (t ) dt ∼ − (x → +∞).
x µ+1

Solution. On traite d’abord le cas µ > −1. Le théorème 3 (page 163) sur les équivalents de
primitives entraı̂ne
 x   x
g (t) dt
dt ∼ µ autrement dit log g (x) ∼ µ log x.
a g (t) a t
En particulier,
∀ε > 0, ∃X > a, ∀x ≥ X, log g (x) ≥ (µ − ε) log x ou encore g (x) ≥ xµ−ε .
4. INT ÉGRALES DÉPENDANT D’UN PARAM ÈTRE, ÉQUIVALENTS D’INTÉGRALES 173

 +∞
En choisissant ε > 0 suffisamment petit pour que µ − ε > −1, on en déduit que a g (t) dt
diverge. Par ailleurs, en intégrant par parties, on a
 x  x  x  x
 
g (t) dt = tg (t) − tg (t) dt donc g (t) + tg  (t) dt = xg (x) − ag (a).
a a a a
Or  
 g  (t)
g (t) + tg (t) = g (t) 1 + t ∼ g (t)(1 + µ),
g (t)
donc en vertu du théorème 3, on a
 x  x
 
xg (x) − ag (a) = g (t) + tg (t) dt ∼ (µ + 1) g (t) dt.
a a
On en déduit le résultat.
Le cas µ < −1 se traite de la même manière.

Exercice 8. On considère le logarithme intégral, qui est l’application définie par


 x
dt
∀x ≥ 2, Li (x) = .
2 log t
Pour tout n ∈ N ∗, donner un développement asymptotique de Li (x) à n termes lorsque
x → +∞.
Solution. En intégrant une fois par parties, on a
x    x
dt t x dt
∀x ≥ 2, = + 2
.
2 log t log t 2 2 log t
En itérant le procédé, on tombe au bout de n − 1 étapes sur la relation
   x    x x
t x t t x t dt
Li (x) = + 1! 2 + 2! 3 + · · · + (n − 1)! n + n! n+1 . (∗)
log t 2 log t 2 log t 2 log t 2 2 log t
Nous cherchons à obtenir un équivalent de la dernière primitive. Le logarithme est une fonction
qui varie très peu (en poussant,
 x −(n+1) on peut dire que log x est presque une constante vers l’infini),
et on s’attend à ce que 2 log t dt ∼ x/ log n+1 x. Pour prouver ceci, on écrit
 
d t 1 (n + 1) 1
n+1 = n+1 − n+2 ∼ n+1 , (t → +∞).
dt log t log t log t log t
Le théorème 3 nous autorise à intégrer cet équivalent. En d’autres termes, lorsque x → +∞ on
a
 x  x    x  x
x d t dt dt x
n+1
= n+1
dt ∼ n+1
donc n+1
∼ n+1
.
log x 2 2 dt log t 2 log t 2 log t log x
Avec (*), on en déduit finalement
 
x 1! x 2! x n! x x
Li (x) = + + +··· + +o .
log x log2 x log3 x log n+1 x log n+1 x

Remarque. Tous les termes de ce développement asymptotique tendent vers +∞ lorsque


x → +∞.

Exercice 9. Donner un équivalent, lorsque t → +∞, de


 1
I (t) = cos x eit ch x dx.
0
174 3. INTÉGRATION

Solution. Commençons par analyser intuitivement la situation. Lorsque t est grand, x → eit ch x
tourne très vite autour de l’origine dans C, ce qui a pour effet de rendre l’intégrale petite. Mais
lorsque x est proche de 0, l’application x → ch x varie peu (sa dérivée est nulle en 0), et cette
rotation est très ralentie (on dit que la phase t ch x est stationnaire). La partie de l’intégrale
correspondant à ce voisinage apporte donc une contribution prépondérante. Pour cette raison,
on va travailler au voisinage de 0.
Le développement limité
x2
ch x = 1 + + o( x 2 ) x → 0+
2
nous invite à écrire ch x = 1 + 2 sh 2 (x/2) et d’effectuer le changement de variable u = sh (x/2)
dans l’intégrale. On trouve
α
it 2 1 cos(2 argsh u)
I (t) = 2e J (t), où J (t) = g (u)e2itu du avec α = sh , g(u) = √ .
0 2 1 + u2
La fonction g est de classe C∞ et g (0) = 1. Nous allons montrer, comme on s’y attend suite à la
discussion que nous avons eu plus haut, que
 α
2
J (t) ∼ e2itu du lorsque t → +∞.
0
Le changement de variable v = 2tu 2 fournit
 α  2tα2 iv  +∞ iv
2itu 2 1 e K e
e du = √ √ dv ∼ √ avec K = √ dv. (∗).
0 2 2t 0 v 2 2t 0 v
Par ailleurs, on a
 α  α
2 2
J (t) − e2itu du = g 1(u)e2itu du avec g1(u) = g (u) − 1. (∗∗)
0 0
La fonction g1 est de classe C∞ et elle est nulle en 0, on peut en déduire facilement que u →
g1 (u)/u se prolonge en une fonction de classe C 1 sur [0, α] (cette dernière est même de classe
C ∞, voir l’exercice 3 page 168). On peut donc écrire
 α  α 
2itu2 1 g1 (u) d  2itu 2
g 1(u)e du = e du
0 4it 0 u du
   α  
1 g1 (u) 2itu 2 α 1 d g1 (u) 2itu2
= e − e du,
4it u 0 4it 0 du u
ce qui montre, avec (**)
 α
 
2itu2 1
J (t) − e du = O , (t → +∞).
0 t
Avec (*) on en déduit
 +∞
it Keit eiv
I (t) = 2e J (t) ∼ √ avec K = √ dv.
2t 0 v
√ iπ/4
Remarque. On a K = Γ(1/2) eiπ/4 = πe (voir la question 2/b) de l’exercice 4).
– Il existe une technique générale appelée la méthode
 bde la phase stationnaire qui permet
ith(x)
de donner un équivalent des intégrales de la forme a g (x)e dx lorsque t → +∞.

Exercice 10. Soit a > 0. Donner un équivalent, lorsque n → +∞, de


 +∞
dx
In = .
0 (1 + xa)(2 + xa ) · · · (n + xa )
4. INT ÉGRALES DÉPENDANT D’UN PARAM ÈTRE, ÉQUIVALENTS D’INTÉGRALES 175

Solution. Pour n fixé, l’intégrande de In est équivalente à x −na lorsque x → +∞, ce qui montre
que In existe dès que n > 1/a.
En effectuant le changement de variable t = xa, on voit que

1 +∞ tα dt 1 1
In = = Jn où α = − 1.
a 0 (1 + t)(2 + t) · · · (n + t) a a
On s’est ainsi ramené à rechercher un équivalent de Jn lorsque n → +∞. On sent intuitivement
que c’est la partie correspondant aux valeurs de x proches de 0 qui va donner une contribution
prépondérante à l’intégrale. Ceci nous amène à rechercher un équivalent de
 1
t α dt
Kn = .
0 (1 + t)(2 + t) · · · (n + t)
Cette dernière intégrale a des bornes d’intégrations finies, ce qui nous simplifiera la tache dans
la suite du calcul. Nous prouverons ultérieurement que l’on a bien K n ∼ Jn.
Poursuivons. On écrit
 1
Ln t α dt
Kn = , Ln = ,
n! 0 (1 + t)(1 + t/2) · · · (1 + t/n)
nous ramenant ainsi à trouver un équivalent de L n. Commençons par donner l’idée. On va écrire
n    n    n 
 t  t  t
1+ = exp log 1 +  exp  exp(t log n).
k k k
k=1 k=1 k=1

Il s’agit de rendre rigoureuse cette affirmation. On part de l’inégalité classique


u2
∀u ≥ 0, | log(1 + u) − u| ≤
2
(conséquence de l’inégalité de Taylor-Lagrange appliquée à t → log(1 + t) sur [0, u] à l’ordre 2).
Donc pour tout n ∈ N ∗ et pour tout t ∈ [0, 1],
 n   n   n 
 t  1  t2  1
+∞
1 1
 2
 log 1 + − t  ≤ ≤ Kt avec K = . ( ∗)
 k k  2 k2 2 k2
k=1 k=1 k=1 k=1
n
Il est classique que k=1 1/k = log n + un où ( un ) est une suite qui tend vers la constante
d’Euler γ . La suite (un ) est en particulier bornée, et si on désigne par M un majorant de |un |,
l’assertion (*) entraı̂ne
 n   
 t 
 
∀n ∈ N ∗, ∀t ∈ [0, 1],  log 1 + − t log n ≤ t |un | + Kt2 ≤ (M + K )t = L t,
 k 
k=1

inégalité que l’on peut aussi écrire sous la forme


n
  
∗ t
∀n ∈ N , ∀t ∈ [0, 1], t (log n − L) ≤ log 1 + ≤ t (log n + L).
k
k=1

En prenant l’exponentielle, ceci s’écrit


n 
 
∗ t(log n−L) t
∀n ∈ N , ∀t ∈ [0, 1], e ≤ 1+ ≤ et(log n+L)
k
k=1

donc finalement
 1  1
∗ α −t(log n+L)
∀n ∈ N , t e dt ≤ Ln ≤ tα e −t(log n−L) dt. (∗∗)
0 0
Or, de manière générale on a
1  λ
1 Γ(α + 1)
t αe−λt dt = t αe −t dt ∼ , λ → +∞.
0 λα+1 0 λ α+1
176 3. INTÉGRATION

Ainsi, les termes extrêmes des inégalités (**) sont tout deux équivalents à
Γ(α + 1)
(∗∗∗)
(log n) α+1
(car log n − L ∼ log n et log n + L ∼ log n lorsque n → +∞). On en déduit d’après (**) que
(***) est un équivalent de Ln , donc finalement
1 1 Γ(α + 1)
Kn = Ln ∼ , n → +∞. (∗∗∗∗)
n! n! (log n)α+1
Pour achever notre calcul, il reste à montrer que Jn ∼ Kn. Pour cela, on écrit d’abord
 +∞
fN (t) dt tα
Jn − Kn = , fN (t) =
1 (N + t) · · · (n + t) (1 + t)(2 + t) · · · (N − 1 + t)
Fixons un entier N > α + 2, de sorte que f N est intégrable sur [1, +∞[. Pour n > N on a
 +∞   
fN (t) dt (N !) 1+∞ f N (t) dt 1
0 ≤ Jn − Kn ≤ = =O .
1 (N + 1) · · · (n + 1) (n + 1)! (n + 1)!
Avec (****), on en déduit que Jn ∼ K n . Finalement, compte tenu du fait que α = 1/a − 1, on
a montré  
1 1 1 1 1
In = Jn ∼ 1/a
Γ .
a a n! (log n) a

Remarque. On peut utiliser la formule de Stirling pour exprimer l’équivalent obtenu sous
une forme ne faisant pas intervenir de factorielle.

5. Problèmes
Problème 1. Soient [a, b] un segment de R non réduit à un singleton et f : [a, b] → C
une application de classe C1 telle que f (a) = 0. Montrer les deux inégalités suivantes et
caractériser l’égalité.
 b 
2 (b − a)2 b 
a) |f (x)| dx ≤ |f (x)| 2 dx
a 2 a
 b   b
b−a
b) |f (x)f (x)| dx ≤ |f (x)| 2 dx.
a 2 a

Solution. a) L’inégalité de Schwarz entraı̂ne


 x 2  x    
  x b
2
∀x ∈ [a, b], |f (x)| =    
f (t) dt ≤  2
|f (t)| dt · 1 dt ≤ (x − a) |f  (t)| 2 dt (∗)
a a a a
d’où l’inégalité désirée en intégrant (*) sur [a, b].
Les fonctions en présence étant continues, l’égalité se produira si et seulement si chacune des
inégalités précédentes est une égalité. En particulier, s’il y a égalité, la dernière inégalité de (*)
doit être une égalité, ce qui entraı̂ne
 x  b

∀x ∈ ]a, b], |f (t)| dt = |f (t)| dt.
a a
Comme |f  | est continue, ceci entraı̂ne que f  est nulle sur ]a, b] donc sur [a, b] par continuité. La
fonction f est donc constante, et comme f (a) = 0, f est nulle. Réciproquement, si f est nulle,
il y a bien égalité.
b) On pose  x
∀x ∈ [a, b], u(x) = |f (t)| dt.
a
5. PROBLÈMES 177

La fonction u vérifie
  x
 x 
∀x ∈ [a, b], |f (x)| =  f (t) dt  ≤

|f (t)| dt = u(x).
a a

En particulier,
 b  b  b  b  b 2
   u2(x) 1 
|f (x)f (x)| dx ≤ |f (x)| u(x) dx = u (x)u(x) dx = = |f (x)| dt .
a a a 2 a 2 a

En appliquant l’inégalité de Schwarz, on a


 b 2  b   b   b
 
|f (x)| dx ≤ 1 dx · 2
|f (x)| dt = (b − a) |f  (x)|2 dt, (∗∗)
a a a a

d’où l’inégalité désirée.


S’il y a égalité, alors l’inégalité de Schwarz (**) est une égalité, donc les fonctions t → f (t)
et t → 1 sont liées, autrement dit f  est constante et f est affine. Réciproquement, si f est affine,
chacune des inégalités précédente est une égalité. Finalement, l’inégalité b) est une égalité si et
seulement si f est affine (et toujours f (a) = 0).

Problème 2. On considère l’application


 x2
dt
F : [0, 1[ → R x → .
x log t
Montrer l’existence de limx→1 − F (x) et calculer cette limite. En déduire la valeur de
l’intégrale
 1
t−1
I= dt.
0 log t

Solution. On ne sait pas exprimer une primitive de t → 1/ log t avec des fonctions usuelles (en
fait, on ne peut pas !). Cependant, lorsque t est voisin de 1, 1/ log t est proche de 1/(t log t) dont
on connaı̂t une primitive. C’est cette idée que nous mettons en forme. Pour tout x ∈ ]0, 1[, on a
x2 1 x
∀t ∈ [x 2 , x], ≤ ≤ ,
t log t log t t log t
donc par intégration
 x2  x2
2 dt dt
x ≤ F (x) ≤ x .
x t log t x t log t
Sachant que
 x2  x 2
dt
= log(log t) = log 2,
x t log t x

on a donc x2 log 2 ≤ F (x) ≤ x log 2 pour tout x ∈ ]0, 1[. On en déduit lim x→1− F (x) = log 2 et
F (0) = 0.
Donnons maintenant la valeur de I . La fonction F est dérivable sur ]0, 1[ et on a
2x 1 x−1
∀x ∈ ]0, 1[ , F  (x) = 2
− = .
log(x ) log x log x
Donc
 1  1
t−1
I= dt = F (t) dt = lim− F (x) − F (0) = log 2.
0 log t 0 x→1
178 3. INTÉGRATION

Problème 3 (Int égrale de Dirichlet). Calculer l’intégrale


 π/2
I= log(sin x) dx.
0

Solution. La convergence de l’intégrale est immédiate (en 0, log(sin x) = log(x + o(x)) =


log x + log(1 + o(1)) ∼ log x). On ne sait pas exprimer de primitive de l’intégrande de I avec des
fonctions usuelles (c’est d’ailleurs là que réside tout l’intérêt de l’exercice).
Pour calculer I , on utilise la formule sin x = 2 sin(x/2) cos(x/2) qui entraı̂ne
 π/2  π/2  x  π/2  x
I= log 2 dx + log sin dx + log cos dx
0 0 2 0 2
 π/4  π/4
π log 2
= +2 log(sin x) dx + 2 log(cos x) dx. (∗)
2 0 0
Le changement de variable u = π/2 − x montre que
 π/4  π/2
log(cos x) dx = log(sin x) dx.
0 π/4

π log 2 π log 2
L’assertion (*) s’écrit donc I = + 2I , donc finalement I = − .
2 2
1
 π/2
Remarque. On peut aussi calculer I en l’écrivant sous la forme I = 2 0
log(1−cos2 x) dx
et en développant le logarithme en série au voisinage de 1.

Problème 4. Pour tout a ∈ ] − 1, +∞[, calculer


 π/2
log(1 + a cos x)
F (a) = dx.
0 cos x

Solution. L’intégrande n’admet pas de primitive exprimable avec les fonctions usuelles. L’idée
est de dériver F par rapport à a et de remarquer qu’ainsi, on se ramène à une intégrale que l’on
sait calculer.
Lorsque a ∈ ] − 1, +∞[, l’intégrande est continue sur [0, π/2[ et elle est prolongeable par
continuité en π/2 car au voisinage de ce point
log(1 + a cos x) a cos x + o(cos x)
= = a + o(1).
cos x cos x
Ainsi prolongée, la fonction f : (a, x) → log(1 + a cos x)/ cos x admet une dérivée partielle par
rapport à a qui est continue sur ] − 1, +∞[ ×[0, π/ 2] et qui vaut (1 + a cos x) −1. Le théorème de
dérivation sous le signe intégral permet donc d’affirmer que F est dérivable et que
 π/2
 dx
∀a > −1, F (a) = .
0 1 + a cos x
On calcule cette dernière intégrale en effectuant le changement de variable u = tan(t/2), ce qui
donne  1
 du
∀a > −1, F (a) = 2 .
0 (1 + a) + (1 − a)u 2
On traite ensuite deux cas selon la position de a par rapport à 1.
— Si a ∈ ] − 1, 1[, on peut écrire a = cos 2θ avec θ ∈ ]0, π/2[. On a alors
 1  1  tan θ
 du 1 du 2 dv 2θ
F (a) = 2 2 2 = 2 2 2 = 2
= ,
0 cos θ + u sin θ cos θ 0 1 + u tan θ sin 2θ 0 1+v sin 2θ
5. PROBLÈMES 179

et finalement
arccos a
∀a ∈ ] − 1, 1[, F  (a) = √ . (∗)
1 − a2
— Si a > 1, on écrit a = ch 2θ avec θ > 0, de sorte que
 1  1  th θ
 du 1 du 2 dv 2θ
F (a) = 2 2 2 = 2 2 = 2
= .
0 ch θ − u sh θ ch θ 0 1 − u 2 th θ sh 2θ 0 1+v sh 2θ
Donc
argch a
∀a > 1, F  (a) = √ . (∗∗)
a2 − 1
Ceci étant, on tire grâce à (*) que pour a ∈ ] − 1, 1[,
 a  a  a
 arccos x (arccos x)2 π2 (arccos a) 2
F (a) = F (0) + F (x) dx = √ dx = − = − .
0 0 1− x2 2 0 8 2
La fonction F étant continue en a = 1, cette expression permet de montrer que
π2
F (1) = lim− F (a) = .
a→1 8
On a maintenant avec (**)
 a
argch x π2 ( argch a) 2
∀a > 1, F (a) = F (1) + √ dx = + .
1 x2 − 1 8 2

Remarque. On peut également résoudre cet exercice en utilisant une inversion de som-
mations dans les intégrales doubles. On écrit
a  π/2  a 
log(1 + a cos x) dt dt
dx = donc F (a) = dx,
cos x 0 1 + t cos x 0 0 1 + t cos x
puis on applique le théorème de Fubini qui nous autorise à inverser les sommations :
 a  π/2 
dx
F (a) = dt,
0 0 1 + t cos x
et on poursuit comme plus haut.
Ceci est un fait général : on peut prouver l’interversion de sommation dans les intégrales
doubles grâce au théorème de dérivation sous le signe intégral. Considérons en effet une
fonction continue f : [a, b] × [c, d] → R. On définit l’application
 b  x 
F : [c, d] → R x → f (t, u) dt du.
a c
En appliquant le théorème de dérivation sous le signe intégral, on voit que F est dérivable
et que  b
∀x ∈ [c, d], F  (x) = f (x, u) du.
a
On en déduit
 b  d   d  d  b 

f (t, u) dt du = F (d) = F (c) + F (x) dx = f (x, u) du dx.
a c c c a

Problème 5. Soit f : [0, +∞[ → R une fonction de classe C2 , nulle en 0. On pose


 +∞  +∞  +∞

I0 = 2
f (x) dx, I1 = 2
f (x) dx, et I2 = f (x)2 dx.
0 0 0
 2  2
Montrer que si f et (f ) sont intégrables sur R , alors (f ) également et I12 ≤ I0 I2 .
2 +
180 3. INTÉGRATION

Solution. Nous allons utiliser l’inégalité de Schwarz. Commençons par montrer la convergence
de l’intégrale I1. On intègre par parties, en écrivant, pour tout X > 0,
 X  X  X  X
f (x) 2 dx = f (x)f (x) − f (x)f  (x) dx = f (X )f(X ) − f (x)f (x) dx. (∗)
0 0 0 0

L’inégalité de Schwarz nous assure que la fonction f f  est intégrable sur R+ et que de plus
 +∞  +∞ 1/2  +∞  1/2
 2  2 1/2 1/2
|f (x)f (x)| dx ≤ f (x) dx · f (x) dx = I 0 I2 . (∗∗)
0 0 0
X
L’égalité (*) entraı̂ne donc le fait que 0 f  (x)2 dx − f (X )f  (X ) converge lorsque X → +∞.

Si l’intégrale 0+∞ f (x)2 dx diverge, alors f (X )f  (X ) → +∞ lorsque X → +∞, donc f 2 (X) =

2 0X f (x)f (x) dx tend vers +∞, ce qui est impossible car f 2 est intégrable sur R + par hypothèse.
 +∞
L’intégrale 0 f (x) 2 dx converge donc, et alors (*) montre que f (X )f  (X ) converge lorsque
X → +∞. Notons  = limx→+∞ f (x)f (x). Si  = 0, on a d’après le théorème 3 page 163
(dernière assertion de (i)),
 X
f 2 (X) = 2 f (x)f (x) dx ∼ 2X (X → +∞),
0

ce qui est impossible car f 2 est intégrable sur R +.


Finalement, on a f (X )f  (X ) → 0 lorsque X → +∞. En faisant X → +∞ dans (*), on
obtient
 +∞  +∞
 2
f (x) dx = − f (x)f (x) dx,
0 0
d’où le résultat avec (**).

Problème 6. Soit une fonction f : R → R continue et intégrable sur R. On pose


 +∞
I (a) = |f (t + a) − f (t)| dt.
−∞

a) Montrer que lim a→0 I (a) = 0.


b) Montrer l’existence et donner la valeur de lim a→+∞ I (a).

Solution. a) On ne peut pas ici appliquer le théorème de convergence dominée car il est im-
possible d’obtenir une majoration du type |f (t + a) − f (t)| ≤ ϕ(t) indépendamment de a. Pour
contourner le problème, on se donne ε > 0 et on considère A > 0 tel que
  −A  +∞
|f (t)| dt = |f (t)| dt + |f (t)| dt < ε.
|t|≥A −∞ A

On pose B = A + 1. Pour tout nombre réel a tel que |a| < 1, on a


  
|f (t + a) − f (t)| dt ≤ |f (t + a)| dt + |f (t)| dt
|t|≥B |t|≥B |t|≥B
 −B +a  +∞ 
= |f (t)| dt + |f (t)| dt + |f (t)| dt ≤ ε + ε = 2ε
−∞ B +a |t|≥B

car −B + a < −A et B + a > A. Ceci entraı̂ne


   B
I (a) = |f (t+ a)− f (t)| dt+ |f (t+ a)− f (t)| dt ≤ |f (t+ a)− f (t)| dt+2ε. (∗)
|t|≤B |t|≥B −B
5. PROBLÈMES 181

B
Lorsque a → 0, l’intégrale −B |f (t + a) − f (t)| dt tend vers 0 car le domaine d’intégration est
un segment de R et la fonction |f (t + a) − f (t)| est continue par rapport à a et nulle lorsque
a = 0. Ainsi  B
∃α > 0, (α < 1), ∀a ∈ [−α, α], |f (t + a) − f (t)| dt < ε.
−B
 +∞
Avec l’inégalité (*) ceci entraı̂ne ∀a ∈ [−α, α], −∞ |f (t + a) − f (t)| dt ≤ 3ε, d’où le résultat.
b) Lorsque f est une fonction à support compact (i. e. nulle en dehors d’un compact) on voit
+∞
facilement que la limite recherchée existe et vaut 2 −∞ |f (t)| dt. C’est ce que nous allons prouver
dans le cas général.
On
 +∞commence par symétriser +∞le problème.
   Le changement
  de variable u = t + a/2 montre
 a a 
que −∞ |f (t + a) − f (t)| dt = −∞ f u + 2 − f u − 2 du Ainsi, on est ramené à prouver
l’existence et donner la valeur de
 +∞
lim |f (t + a) − f (t − a)| dt. (∗∗)
a→+∞ −∞
On note fa la fonction t → f (t + a). On va montrer que la contribution de l’intégrale précédente
pour t ≥ 0 converge vers R |f |. On écrit, pour a > 0,
     
|f a − f −a | − |f | = |fa − f−a | − |f−a | = (|fa − f −a| − |f−a |) + |f−a|.
R+ R R+ R R+ R−
 
Avec l’inégalité  |fa − f−a | − |f−a | ≤ |f a| on en déduit
     
 
 |fa − f −a | − |f | ≤ |fa| + |f−a| = |f (t)| dt.

R+ R R+ R− |t|≥a
 
Comme f est intégrable
 sur R, ceci entraı̂ne que
 R+ |f a −f −a| converge vers R |f |. On montrerait
de même que R− |fa − f−a | converge vers R |f |, et par sommation on a finalement démontré
que (**) converge vers 2 R |f | lorsque a → +∞.

Problème 7. Soit f : R+ → R une fonction de classe C 1, intégrable sur R+ , telle que


(f )2 est intégrable sur R+. Montrer que f est bornée sur R +.

Solution. Nous allons montrer que f tend vers 0 en +∞, ce qui montrera le résultat. D’après le
+∞
théorème 4 page 152, comme 0 f (t) dt converge, il suffit de montrer que f est uniformément
continue sur R+.
Soient deux nombres réels x et y tels que 0 ≤ x < y. L’inégalité de Schwarz entraı̂ne
 y   y 1/2  y 1/2  
    2  2 √

|f (y ) − f (x)| =   
f (t) dt  ≤ f (t) dt · 1 dt ≤ (f ) y − x.
x x x R+
Ceci entraı̂ne que f est uniformément continue sur R+ , d’où le résultat.

Problème 8. Soit f : [1, +∞[→ R une fonction continue et croissante. On suppose que
la fonction  x
f (t)
F : [1, +∞[→ R x → dt
1 t
vérifie F (x) ∼ x lorsque x → +∞. Montrer que f (x) ∼ x lorsque x → +∞.

Solution. Soit δ ∈ ]0, 1[. D’après les hypothèses,


 x
f (t)
∃x 0 > 1, ∀x ≥ x 0, (1 − δ )x ≤ dt ≤ (1 + δ)x.
1 t
182 3. INTÉGRATION

Ceci entraı̂ne pour tout ε ∈ ]0, 1[


 x+εx
f (t)
∀x ≥ x1 , dt ≤ (1 + δ)(1 + ε)x − (1 − δ)x = (ε + 2δ + δε)x,
x t
et comme f est croissante
 x+εx
f (x) f (t) ε + 2δ + δε
∀x ≥ x1 , εx ≤ dt ≤ (ε + 2δ + δε)x donc f (x) ≤ (1 + ε)x.
x(1 + ε) x t ε

En choisissant ε = δ, on en déduit
√ √ √
∀x ≥ x 1, f(x) ≤ (1 + 2 δ + δ )(1 + δ )x = (1 + δ) 3x.
En d’autres termes, nous venons de montrer
∀α > 0, ∃x 1 > 1, ∀x ≥ x 1 , f(x) ≤ x(1 + α).
On montrerait de même
∀α > 0, ∃x 2 > 1, ∀x ≥ x 2 , f(x) ≥ x(1 − α).
On en déduit f (x) ∼ x lorsque x → +∞.

Problème 9. On désigne par E l’ensemble des fonctions continues sur [0, 1] à valeurs
dans R +∗. Pour tout f ∈ E , on note
 1   1 
dt
I (f ) = f (t) dt ·
0 0 f (t)

et on note Γ = I (E ) = {I (f ), f ∈ E }.
a) Déterminer m = inf Γ. Pour quelles fonctions de E a-t-on I (f ) = m ?
b) Déterminer sup Γ.

Solution. a) Soit f ∈ E . D’après l’inégalité de Schwarz


 2    1 
1 1
1 dt
1= f (t)  dt ≤ f (t) dt · = I (f ), (∗)
0 f (t) 0 0 f (t)

ce qui montre que I (f ) ≥ 1. L’image de la fonction constante égale à 1 par I est égal à 1, on a
donc m = 1. √Les foncti
√ons f de E étant continues, l’inégalité de Schwarz (*) se produira si et
seulement si f et 1/ f sont proportionnelles, c’est-à-dire si et seulement si
 λ
∃λ > 0, ∀x ∈ [0, 1], f (x) =  ou encore f (x) = λ.
f (x)
Les fonctions f telles que I (f ) = 1 sont donc les fonctions constantes de E .
b) Pour tout n ∈ N∗ , il est possible de construire une fonction f n de E telle que
   
1 2
∀x ∈ 0, , f(x) = 1 et ∀x ∈ ,1 , f(x) = n
3 3
(on peut prendre par exemple une fonction définie comme telle sur [0, 1/3] ∪ [2/3, 1] et affine sur
[1/3, 2/3]). On a alors
n 1 n
∀n ∈ N ∗ , I(fn ) ≥ · = ,
3 3 9
donc sup Γ = +∞.
Remarque. On peut également traiter la question a) en écrivant
  
f (x) f (y )
2I (f ) = + dx dy,
[0,1] 2 f (y ) f (x)
5. PROBLÈMES 183

(on obtient cette expression en appliquant le théorème de Fubini) puis en remarquant que
f (x)/f (y ) + f (y )/f (x) ≥ 2 (en posant α = f (x)/f (y ), on a α + 1/α = 2 + (α − 1)2 /α ≥ 2).

Problème 10. Pour tout ρ ∈ R{−1, 1}, on pose


 π
I (ρ) = log(1 − 2ρ cos θ + ρ 2) dθ.
0

a) Comparer I (ρ) et I (ρ2 ). En déduire la valeur de I (ρ).


b) Retrouver la valeur de I (ρ) en utilisant les sommes de Riemann.

Solution. L’égalité 1 − 2ρ cos θ + ρ2 = (1 − ρ cos θ )2 + ρ2 sin 2 θ = |1 − ρe iθ | 2 montre que I (ρ)


existe pour tout ρ ∈ {−1, 1}.
a) Le changement de variable ψ = π − θ montre que la fonction ρ → I (ρ) est paire. Donc si
ρ ∈ {−1, 1},  π
 
2I (ρ) = I (ρ) + I (−ρ) = log |1 − ρe iθ |2 |1 + ρeiθ |2 dθ.
0
Comme (1 − ρeiθ )(1 + ρeiθ ) = 1− ρ2 e2iθ ceci entraı̂ne
 π 
1 2π
2 2iθ 2
2I (ρ) = log(|1 − ρ e | ) dθ = log(|1 − ρ2 eiθ | 2 ) dθ.
0 2 0
Le changement de variable ϕ = θ − π donne
 2π  π
2 iθ 2
log(|1 − ρ e | ) dθ = log(|1 + ρ 2e iϕ|) dϕ = I (−ρ 2 ) = I (ρ 2),
π 0
1 2
on a donc montré 2I (ρ) = + I (ρ )) = I (ρ 2 ). On en conclut facilement
2 (I (ρ )
2

1  n
∀n ∈ N∗ , I(ρ) = n I ρ2 . (∗)
2
— Si |ρ| < 1, la continuité de I en 0 entraı̂ne I (ρ) = 0 (il suffit de faire n → +∞ dans (*)).
— Si |ρ| > 1, c’est un peu plus délicat. On commence par montrer I (ρ) ∼ 2π log ρ lorsque
ρ → +∞. On a
   
2 2 1 2 2 2 1
∀θ ∈ [0, π], ρ 1 − + 2 ≤ 1 − 2ρ cos θ + ρ ≤ ρ 1 + + 2
ρ ρ ρ ρ
donc par intégration
   
2 1 2 1
2π log ρ + π log 1 − + 2 ≤ I (ρ) ≤ 2π log ρ + π log 1 + + 2 ,
ρ ρ ρ ρ
ce qui entraı̂ne bien I (ρ) ∼ 2π log ρ lorsque ρ → +∞. En particulier, si on fixe ρ > 1, on
a, lorsque l’entier n tend vers +∞
n
I (ρ2 ) ∼ 2n+1 π log ρ,
et on en déduit en faisant tendre n vers l’infini dans (*) que I (ρ) = 2π log ρ. Si ρ < −1,
on a I (ρ) = I (−ρ). Finalement, on a montré
∀ρ ∈ R, |ρ| > 1, I(ρ) = 2π log |ρ|.

b) Soit ρ ∈ R{−1, 1}. En appliquant le théorème sur les sommes de Riemann, on a


 n    
π  kπ
I (ρ) = lim un avec un = log 1 − 2ρ cos + ρ2 .
n→+∞ n n
k=1
Or  
2 kπ
∀k ∈ Z, ρ − 2ρ cos + 1 = (ρ − ω k )(ρ − ω−k ), (ω k = eikπ/n ),
n
184 3. INTÉGRATION


et comme n−1 k=−n(X − ωk ) = X
2n
− 1 (les 2n nombres complexes ω k pour −n ≤ k < n sont des
2n
racines distinctes du polynôme X − 1), on en déduit
n
 n−1 
  ρ − ωn ρ+1
(ρ − ωk )(ρ − ω −k) = (ρ − ωk ) = (ρ2n − 1)
ρ − ω0 ρ−1
k=1 k=−n

et donc  2n 
∗ π (ρ − 1)(ρ + 1)
∀n ∈ N , un = log . (∗∗)
n ρ−1
— Si |ρ| < 1, on a facilement lim n→+∞ u n = 0 d’après (**), donc I (ρ) = 0.
— Si |ρ| > 1, on transforme (**) en
  
π ρ+1 1
un = 2π log |ρ| + log 1 − 2n ,
n ρ−1 ρ
ce qui entraı̂ne I (ρ) = lim n→+∞ un = 2π log |ρ|.
Remarque. On peut montrer que les intégrales I (1) et I (−1) existent et qu’elles sont
nulles.

Problème 11. a) Soit f : [0, 1] → R une fonction continue par morceaux. Montrer que
 1 +∞

lim
q→1
I (q ) = f (t) dt où ∀q ∈ ]0, 1[, I(q) = (1 − q) q nf (q n ).
q<1 0 n=0

b) En utilisant le résultat de la question précédente, calculer


 1
t−1
dt.
0 log t

Solution. a) Pour tout q ∈ ]0, 1[, I (q) est bien définie car f est bornée et on peut écrire
+∞

I (q ) = (qn − q n+1 )f (qn ).
n=0
Cette expression peut s’apparenter à une somme de Riemann infinie pour la subdivision infinie
1, q, q2, . . . , q n , . . .. Le théorème 7 page 128 sur les sommes de Riemann ne s’applique que pour
des subdivisions finies, nous allons donc nous y ramener. Soit ε > 0 On sait qu’il existe α > 0
tel que pour toute subdivision σ : 0 = x0 < x1 < · · · < xn = 1 de [0, 1] vérifiant |σ | < α, on ait
 n  1 
 
 
 (xi − x i−1 )f (xi ) − f (t) dt < ε.
 0 
i=1

Soit q tel que 1 − α < q < 1 et N ∈ N ∗ tel que q N < α. Pour tout n ≥ N , la subdivision
σn : 0 < q n < qn−1 < · · · < q < 1
vérifie |σ n| < α (on a q i − q i+1 = q i (1 − q) ≤ 1 − q < α pour tout i et q n − 0 ≤ q N < α), donc
  1 
 
(1 − q )f (1) + (q − q )f (q) + · · · + (q
2 n−1 n
− q )f (qn−1 n n
) + (q − 0)f (q ) − f (t) dt < ε.

0
Ceci est vrai pour tout n ≥ N . En faisant tendre n vers +∞, on en déduit
+∞ 1 
 
 
 (qi − q i+1)f (q i ) − f (t) dt ≤ ε.
 0 
i=0

Ainsi, pour tout q vérifiant 1 − α < q < 1, on a |I (q) − 01 f (t) dt| ≤ ε. D’où le résultat.
5. PROBLÈMES 185

b) Ici, la valeur correspondante de I (q) pour la fonction intégrée est


+∞
 +∞ +∞ 2n

 qn − 1 q − 1  qn  q
∀q ∈ ]0, 1[, I(q) = (1 − q) qn = − ,
n=0
n log q log q n=0
n n=0
n

et le développement en série entière du logarithme fournit


q − 1  q−1
∀q ∈ ]0, 1[, I(q) = − log(1 − q) + log(1 − q2) = log(1 + q ).
log q log q
Comme log q ∼ q − 1 lorsque q → 1, cette formule entraı̂ne limq→1 − I (q ) = log 2, et finalement

on a 01 (t − 1)/ log t dt = log 2 d’après la question précédente.
Remarque. La valeur de cette dernière intégrale est calculée avec une méthode différente
dans le problème 2 page 177.

Problème 12 (Preuve du théor ème de d’Alembert). 1/ Soit f : R → C∗ une


fonction de classe C1 à valeurs complexes, 2π -périodique et ne s’annulant pas. On définit
 2π 
1 f (θ )
I (f ) = dθ.
2iπ 0 f (θ )
  
t f (θ)
En s’appuyant sur la fonction ϕ(t) = exp 0 f (θ ) dθ , montrer que I (f ) ∈ Z.
2/ Soit P un polynôme complexe non constant. Pour tout r > 0. On pose fP,r (t) = P (reit).
a) (Théorème de d’Alembert). Pour P fixé, en étudiant la fonction r → I (f P,r ) pour r > 0,
prouver que P a au moins une racine complexe.
b) Calculer la valeur de I (fP,r ) en fonction de r et des zéros de P .

f  (t)
Solution. 1/ La fonction ϕ est de classe C 1 et elle vérifie ϕ(t) = f (t) ϕ(t), elle est donc solution
 f
de l’équation différentielle y − f y = 0. Comme f est solution de cette équation différentielle et
que l’espace de ses solutions est de dimension 1, il existe λ ∈ C tel que ϕ = λf . Comme f est 2π -
 2π 
périodique c’est aussi le cas de ϕ, en particulier ϕ(2π) = ϕ(0) ce qui entraı̂ne 0 ff ((θθ)) dθ ∈ 2iπ Z
donc I (f ) ∈ Z.
2/a) Raisonnons par l’absurde et supposons que P ne s’annule pas sur C. Alors la fonction
 2π  iθ iθ
+ 1 P (re )re
g : R → C, r → I (f P,r ) = dθ (∗)
2π 0 P (re iθ )
est définie sur R + tout entier. En vertu de la continuité de la fonction intégrée par rapport à r,
le théorème de continuité sous le signe intégral montre que g est continue. Hors elle est à valeur
dans Z d’après le résultat de la question précédente, elle est donc constante. Comme g (0) = 0, on
en déduit que g est identiquement nulle. Regardons maintenant ce qui se passe lorsque r → +∞.
Soit n le degré de P et a son coefficient dominant. Lorsque |z | → +∞, on a P (z ) ∼ az n et
P  (z ) ∼ nazn−1 , donc zP (z )/P (z ) converge vers n. Ainsi, lorsque r → +∞, l’intégrande de (*)
converge vers n. De plus cette intégrale est bornée indépendamment de r (car zP  (z )/P (z) a une
limite finie lorsque |z| → +∞, donc est bornée sur C), donc d’après le théorème de convergence
dominée on a
 2π  
1 P  (reiθ )reiθ
lim g (r ) = lim dθ = n,
r→+∞ 2π 0 r→+∞ P (reiθ )
ce qui est contradictoire avec le fait que g est identiquement nulle. Ainsi, le polynôme complexe
P doit nécéssairement s’annuler sur C.
186 3. INTÉGRATION

b) Notons α1 , . . . , α n les zéros de P , éventuellement avec leur multiplicité, de sorte que P (z ) =


a(z − α 1) · · · (z − αn). On a
n n
zP (z )  z  zP  (z )
k
= = , où Pk (z) = z − αk . (∗∗)
P (z ) z − αk Pk (z )
k=1 k=1

Si aucune racine de P ne se trouve  sur le cercle |z | = r , la valeur I (f P,r) est bien définie et
l’identité (**) montre que I (f P,r ) = nk=1 I (f Pk,r ). De la même manière que ce que l’on a vu
pour P précédemment, pour tout k, la valeur r → I (fPk ,r ) est une fonction continue de r sur les
intervalles où elle est définie, donc sur l’intervalle [0, |αk |[ et sur l’intervalle ]|αk |, +∞[ . De plus
elle est à valeur dans Z, donc constante sur chacun de ces intervalles. En particulier, si r < |αk |
on a I (fP k ,r) = I (fP k,0) = 0, et si r > |αk |, alors de la même manière que précédemment on a
I (fP k,r ) = limt→+∞ I (f Pk ,t ) = deg(Pk) = 1. En conclusion, la valeur I (f P,r ) est définie lorsque
qu’aucune des racines de P n’est sur le cercle |z | = r , et égale au nombre de racines de P
(comptées avec leur ordre du multiplicité) dans le disque |z | < r.
Remarque. - La valeur I (f ) est appelée indice de la courbe f sur [0, 2π ]. On peut montrer
(c’est une conséquence du théorème du relèvement) qu’on peut écrire f (θ ) = ρ(θ )eiϕ(θ) ,
où ρ : R → R+∗ et ϕ : R → R sont de classe C1 . On a ρ = |f | et ϕ représente l’argument
de f obtenu par continuité. Comme f est 2π -périodique, ϕ(0) ≡ ϕ(2π ) (mod 2π ) donc
ϕ(2π ) − ϕ(0) = 2kπ avec k ∈ Z. On peut montrer facilement que k = I (f ) ; ainsi I (f )
représente le nombre de tours autour de 0, effectués dans le sens trigonométrique, que fait
la courbe fermée f restreinte à [0, 2π ].
– Le calcul de l’intégrale correspondant à I (fPk,r) peut aussi se faire directement, à partir
d’une primitive de l’intégrande ou en la développant en série. La méthode que nous avons
présentée est plus élégante dans le cadre du problème.
– Deux autres preuves du théorème de d’Alembert (encore appelé théorème fondamental
de l’algèbre) sont proposées dans le tome Algèbre.

Problème 13. Déterminer


 1 les fonctions continues f : R → R telles que, pour tout
f (x + t) − f (x)
x ∈ R, l’intégrale dt converge.
0 t2

Solution. Si f est dérivable sur R, on a nécessairement f (x) = 0 pour tout x ∈ R. En effet, s’il
existe x ∈ R tel que f (x) = 0, l’équivalent (f (x + t) − f (x))/t 2 ∼ t→0 f  (x)/t montre que cette
fonction ne serait pas intégrable sur [0, 1], ce qui est contraire aux hypothèses. Ainsi, si f est
dérivable, sa dérivée s’annule sur R donc f est nécéssairement une fonction constante.
Montrons que ce résultat reste vrai si f est seulement supposée continue. Pour cela raisonnons
par l’absurde et supposons f non constante. L’idée est de déterminer un x tel que au voisinage
de t = 0+ , on puisse écrire |f (x + t) − f (x)| ≥ αt avec α > 0. Comme f est non constante, il
existe a, b ∈ R, a < b, tels que f (a) =  f (b). Quitte à considérer la fonction f − f (a), on peut
supposer f (a) = 0. Quitte à considérer −f , on peut supposer f (b) > 0. Quitte à considérer
t → f (a + (b − a)t) on peut supposer que a = 0 et b = 1. En résumé, on peut supposer que
0 = f (0) < f (1). Considérons la fonction ϕ : [0, 1] → R t → f (1)/3 + tf (1)/3, et notons
A = {t ∈ [0, 1] | f (t) ≤ ϕ(t)}. L’ensemble A est non vide puisque 0 = f (0) < ϕ(0) = f (1)/3,
donc 0 ∈ A. Par ailleurs 1 ∈ A car f (1) > ϕ(1) = 2f (1)/3. On peut donc définir c = sup A, et on
a c < 1 par continuité de f et ϕ. Comme A est fermé on a c ∈ A donc f (c) ≤ ϕ(c). Si on avait
f (c) < ϕ(c), alors par continuité de f et ϕ on pourrait trouver d ∈ ]c, 1] tel que f (d) < ϕ(d), ce
qui est en contradiction avec la définition de c. Donc f (c) = ϕ(c). De plus, par définition de c
on a f (t) > ϕ(t) pour tout t ∈ ]c, 1]. On en déduit que
∀t ∈ ]c, 1], f(t) − f (c) > ϕ(t) − ϕ(c) = (t − c)f (1)/3.
5. PROBLÈMES 187

Ainsi on a [f (c + t) − f (c)]/t2 > α/t pour t ∈ ]0, 1 − c] avec α = f (1)/3 > 0, donc l’intégrale
1 2
0 [f (c + t) − f (c)]/t dt diverge. Ceci est contraire aux hypothèses, on en déduit que f est une
fonction constante.
Réciproquement, il est évident que toute fonction constante f vérifie les propriétés de
l’énoncé.

Problème 14. Soit f une application définie sur [0, 1], à valeurs strictement positives,
et continue. Calculer  1 1/α
α
lim
α→0
f (t) dt .
α>0 0

Solution. On note  1
F : [0, +∞[ → R α → f α(t) dt.
0
La fonction ϕ : R+ × [0, 1] → R (α, t) → f α (t) est continue, dérivable par rapport à α et
∂ϕ
∀(α, t) ∈ R+ × [0, 1], (α, t) = log f (t) · f α(t)
∂α
est continue sur R+ × [0, 1]. Donc F est dérivable sur R+ et
 1  1
 α 
∀α ≥ 0, F (α) = log f (t) · f (t) dt, en particulier F (0) = log f (t) dt.
0 0
Or F (0) = 1, donc lorsque α → 0 +,
   
log F (α) log(1 + αF (0) + o(α))
F (α ) 1/α
= exp = exp = exp(F  (0) + o(1)).
α α
Finalement, la limite recherchée vaut
 1


exp(F (0)) = exp log f (t) dt .
0

Remarque. On a déjà calculé la limite de la même fonction lorsque le paramètre α tend


vers +∞ (voir l’exercice 2 page 130).

Problème 15 (Irrationalité de π 2). a) Soit g : R → R une fonction polynôme à


coefficients entiers. On considère la fonction polynôme
xn g (x)
h : R → R x → .
n!
Pour tout k ∈ N, montrer que h(k) (0) est entier.
b) On suppose que π 2 est rationnel, de sorte qu’il existe a, b ∈ N∗ tel que π2 = a/b. Pour
tout n ∈ N ∗ on pose
 1
n xn (1 − x)n
In = π a fn (x) sin(πx ) dx, où fn(x) = .
0 n!
Montrer que I n est un entier. Conclure.
p
Solution. a) Écrivons g (x) = k=0 a k x k . On a
p
1 
h(x) = ak xk+n , (ak ∈ Z).
n!
k=0
188 3. INTÉGRATION

Donc
- si 0 ≤ k ≤ n − 1, h(k) (0) =0 ∈ Z
k!
- si n ≤ k ≤ n + p, h(k) (0) = a = k (k − 1) · · · (n + 1) a k−n ∈ Z
n! k−n
- si n + p < k, h(k) (0) =0 ∈ Z.

b) En intégrant deux fois par parties, on trouve


 1 1
fn (1) − fn (0) 1
fn (x) sin(πx ) dx = − 2 fn (x) sin(πx ) dx,
0 π π 0
puis en itérant, on trouve
 1
fn (1) − fn(0) f n(1) − fn(0)
fn(x) sin(πx ) dx = − +···
0 π π3
(2n) (2n)  1
f n (1) − fn (0)
···+ (−1)n + (−1) n+1
fn(2n+2)(x) sin(πx ) dx.
π2n+1 0

Cette dernière intégrale est nulle car fn est une fonction polynôme de degré 2n, donc finalement
 (2n) (2n)

f (1) − f  (0) f (1) − f (0)
n n
In = a n fn (1) − fn (0) − n n
+ · · · + (−1) n .
π2 π2n
(2k)
D’après le résultat de la question a), on a f n (0) ∈ Z pour tout entier k, et comme f n(1 − x) =
(2k)
fn(x) on a aussi f n (1) ∈ Z. Ceci entraı̂ne, pour tout entier k, 0 ≤ k ≤ n
(2k) (2k) k 
fn (1) − fn (0) n b
  
an 2k
= a k
fn(2k)(1) − f n(2k) (0) = an−k bk fn(2k)(1) − f (2k)
n (0) ∈ Z.
π a
Ainsi, In ∈ Z. Or 0 < I n < πa n/n! et comme a n/n! tend vers 0 lorsque n → +∞ (c’est classique,
on sait même que la série entière z n /n! a un rayon de convergence infini), il existe un entier
n
n pour lequel πa /n! < 1. Ceci entraı̂ne 0 < In < 1, ce qui est impossible puisqu’on a montré
que In était entier.
Le nombre réel π 2 est donc irrationnel (ce qui entraı̂ne que π est irrationnel).
Remarque. On peut montrer que π est transcendant (voir une preuve dans le tome
Algèbre), mais c’est beaucoup plus difficile.

Problème 16 (Moyenne arithmético-géom étrique). Soient a, b ∈ R, a, b > 0.


1/ On considère les deux suites (u n) et (v n) définies par
un + vn √
u 0 = a, v 0 = b et ∀n ∈ N, un+1 = , v n+1 = un vn .
2
Montrer que les suites (un ) et (bn ) convergent vers la même limite.
2/a) On note M (a, b) la limite commune des ces deux suites (on l’appelle la moyenne
arithmético-géométrique de a et b). Montrer que
 π/2
π 1 dθ
= T (a, b), où T (a, b) = √ (∗)
2 M (a, b) 0 a2 cos2 θ + b2 sin2 θ

(on pourra montrer que T (a, b) = T ((a +b)/2, ab) en effectuant le changement de variable
t = b tan θ puis le changement de variable u = 12(t − ab/t)).
5. PROBLÈMES 189

b) (Constante de Gauss.) Montrer que


 1
1 2 dt
√ = √ .
M ( 2, 1) π 0 1 − t4

Solution. 1/ Il est commode de supposer que a ≥ b, on se ramène facilement à ce cas car les
valeurs de u1 et v1 ne changent pas si on intervertit a et b. Une récurrence immédiate montre
que toutes les valeurs des termes des suites (u n ) et (vn ) sont strictement positives. On remarque
ensuite que vn ≤ un : pour n = 0 c’est vrai car a ≥ b, et lorsque n ∈ N∗ ceci découle de
1 √  1√ √ 2
un − v n = un−1 − 2 un−1 vn−1 + vn−1 = u n−1 − v n−1 ≥ 0.
2 2
La suite (un ) décroı̂t, la suite (v n) croı̂t, car

1 vn+1 un
∀n ∈ N, un+1 − u n = (vn − u n) ≤ 0 et = ≥ 1.
2 vn vn
Finalement, nous avons montré
∀n ∈ N, v0 ≤ v 1 ≤ · · · ≤ vn ≤ u n ≤ · · · ≤ u 1 ≤ u 0.
La suite décroissante (un ) est minorée par l’un quelconque des termes de (v n), elle est donc
convergente. De même, (vn) est croissante et majorée par l’un quelconque des termes de (u n),
donc convergente. Enfin, on montre facilement par récurrence que
∀n ∈ N, 0 ≤ u n − vn ≤ (a − b)/2n . (∗∗)
En effet, ceci est vrai pour n = 0, et si on suppose la propriété vraie pour n on a
1 √ √ 2 1 u −v √ √  u − vn
u n+1 − v n+1 = u n − vn = √ n √n un − vn ≤ n .
2 2 un + v n 2
L’encadrement (**) prouve que les limites de (un) et (vn ) sont égales. Nous aurions pu montrer
l’égalité des limites en procédant par continuité : en notant
√ U et V les limites respectives de
(un) et (v n), on a par continuité U = (U + V )/2 et V = U V , donc U = V . Néanmoins (**) est
plus intéréssant car il indique une convergence rapide de (un ) et (v n) vers leur limite commune.
Notons que nous avons prouvé que (un ) et (vn ) sont des suites adjacentes.
2/a) Comme indiqué, on effectue d’abord le changement de variable t = b tan θ. On obtient
 +∞
dt
T (a, b) =  . (∗∗∗)
0 (t2 + a2 )(t2 + b 2)
On effectue maintenant le changement de variable √ u = (t − ab/t)/2. Les calculs ne sont pas si
faciles à obtenir. En notant c = (a + b)/2 et d = ab, on écrit
(t2 − ab) 2 (a + b)2 t 4 + (a2 + b 2 )t2 + a 2b2 (t 2 + a2 )(t2 + b 2)
u 2 + c2 = + = =
4t 2 4 4t 2 4t2
2
(t − ab) 2 √ 2
4 2
t + 2abt + a b 2 2 2
(t + ab) 2
u2 + d2 = 2
+ ab = 2
= .
4t 4t 4t2
Comme du = (t2 + ab)/(2t 2) dt, on en déduit
 
1 +∞ du 1 +∞ 4t 2 1 t2 + ab
T (c, d) =  =  dt,
2 −∞ (u 2 + c 2)(u2 + d 2 ) 2 0 (t2 + a 2)(t 2 + b 2) t2 + ab 2t2
la première égalité provenant de la parité de la fonction intégrée. Dans la dernière intégrale, après
simplification, on reconnait l’intégrande de l’intégrale (***), on en déduit
√ T (c, d) = T (a, b).
Prouvons maintenant l’identité (*). L’égalité T (a, b) = T ( a+2b , ab), entraı̂ne T (un, vn ) =
T (un+1 , vn+1 ) pour tout n ∈ N, donc T (a, b) = T (u0 , v 0) = T (u n , vn) pour tout n ∈ N. La
fonction T (a, b) est l’intégrale d’une fonction continue en (a, b), intégrée sur le segment [0, π/ 2],
190 3. INTÉGRATION

elle est donc continue en (a, b). On peut maintenant passer à la limite dans l’égalité T (a, b) =
T (un , vn ), ce qui donne T (a, b) = T (M (a, b), M (a, b)). On en déduit le résultat car
 π/2
dθ π 1
T (M (a, b), M (a, b)) = = .
0 M (a, b) 2 M (a, b)

b) En effectuant le changement de variable t = cos θ , on obtient


1  π/2  π/2  π/2
dt 1 sin θ dθ dθ dθ
√ = √ √ = √ =  .
4 2 2 2
0 1−t 0 1 + cos θ 1 − cos θ 0 1 + cos θ 0 2 cos 2 θ + sin2 θ

La dernière intégrale est T ( 2, 1), on en déduit le résultat grâce à l’identité (*).
Remarque. L’identité (*) a été prouvée par Gauss. L’intégrale de (*) se ramène à l’intégrale
 π/2 √
elliptique de première espèce K (k) = 0 (1 − k 2 sin2 θ) −1/2 dθ = T (1, 1 − k2 ).
√ 1 √
- La valeur G = 1/M ( 2, 1) est appelée constante de Gauss. L’intégrale 0 dt/ 1 − t4
correspond à la longueur d’un arc de lemniscate qui est la courbe définie par l’équation
(x 2 + y 2 )2 = x2 − y 2 . Les suites (un ) √
et (v n ) convergent très rapidement (la
√ convergence
n
est quadratique, dans le cas de a = 2 et b = 1 on a |M (a, b) − u n| ≤ ( 2 − 1) 2 ), et
Gauss calcula les premières valeurs de (un ) et (v n) pour obtenir 11 décimales de G.
- Ces résultats obtenus par Gauss ont introduit la théorie des intégrales et des fonctions
elliptiques, qui joue un rôle important en théorie des nombres.

Problème 17. On considère, pour β > 0 l’intégrale


 +∞
cos(xt) −βt
Iβ (x) = e dt, x ∈ R.
0 1 + t2

a) Montrer que la fonction x → Iβ (x) est deux fois dérivable sur R et qu’elle vérifie
β
Iβ (x) − Iβ(x) = .
β2 + x2

b) Démontrer l’identité
 +∞
ex F β(x) + e−x Fβ(−x) βe−t
I β (x) = , avec Fβ (x) = dt.
2 x β 2 + t2

c) En déduire les valeurs



 +∞  +∞
π −x
e (x > 0)
cos(xt) π t sin(xt) 2
C (x) = dt = e−|x|, S(x) = dt = 0 (x = 0)
0 1+t 2 2 0 1 + t2  π x
− 2 e (x < 0)

Solution. a) Iβ (x) est bien définie car son intégrande est majorée en valeur absolue par e−βt ,
intégrable sur R+ . Notons f (x, t) l’intégrande de Iβ(x). Sa dérivée partielle par rapport à x est
∂f/∂x = −t sin(xt)/(1 + t2)e−βt , donc |∂f/∂x| est majorée sur R+ par la fonction intégrable
t → e−βt , donc I β est bien dérivable et
 +∞
 t sin(xt) −βt
I β (x) = − e dt.
0 1 + t2
5. PROBLÈMES 191

On montre de même que I β est dérivable et que


 +∞ 2
 t cos(xt) −βt
Iβ (x) = − e dt.
0 1 + t2
La relation souhaitée s’obtient maintenant facilement :
 +∞  +∞  
 (1 + t2) cos(xt) −βt (ix−β )t 1 β
Iβ (x) − Iβ (x) = 2
e dt = (e ) dt =  = 2 .
0 1+t 0 β − ix β + x2

b) Notons Jβ (x) = 12 (ex Fβ (x) + e−x Fβ (−x)). Nous allons montrer que Jβ vérifie la même
équation différentielle que Iβ . Partant de l’égalité
β
ex Fβ (x) = − 2
β + x2
(qui entraı̂ne en particulier que ex Fβ (x) est paire) on obtient par dérivations successives
2J β (x) = ex Fβ (x) − e −x Fβ(−x) + ex Fβ (x) − e−xF β (−x) = ex Fβ (x) − e −xFβ (−x),

2Jβ (x) = ex Fβ (x) + e −x Fβ(−x) + ex Fβ (x) + e−xF β (−x) = 2Jβ (x) − 2 .
β + x2
En divisant par 2 cette dernière égalité, on voit que Jβ est une solution particulière de l’équation
différentielle linéaire vérifiée par Iβ . La différence Kβ = Iβ − Jβ vérifie l’équation linéaire d’ordre
deux Kβ − Kβ = 0, donc Kβ(x) = λex + µe −x avec λ, µ ∈ R. Ainsi, on a
Iβ (x) = Jβ (x) + K β(x) = J β (x) + λex + µe−x .
 +∞ −βt
Or les fonctions Iβ (x) et Jβ (x) sont bornées sur R (on a |I β (x)| ≤ 0 e dt et |ex F β (x)| ≤
 +∞ 2 2
−∞ β/(β + t ) dt), on a donc forcément λ = µ = 0 d’où le résultat.
c) Il est immédiat que Iβ est également définie pour β = 0. Fixons x ∈ R. La fonction
cos(xt) −βt
f : [0, +∞[×[0, +∞[→ R (β, t) → e
1 + t2
est continue et |f (β, ·)| est majorée par la fonction intégrable t → 1/(1 + t 2 ). L’hypothèse de
domination est donc bien vérifiée, ce qui montre que la fonction β → Iβ (x) est continue sur R+.
En particulier
C (x) = I0 (x) = lim I β(x).
β→0
β>0

Utilisons maintenant l’expression de Iβ obtenue à la question précédente, en faisant tendre β


vers 0 en considérant la suite (1/n)n>0 . Lorsque n ∈ N∗ un changement de variable donne
 +∞ −t/n  +∞  −t/n
e e /(1 + t2 ) si t ≥ nx,
F1/n (x) = dt = fn (t ) dt, f n (t) =
nx 1 + t2 −∞ 0 si t < nx.
On a la majoration |fn (t)| ≤ e−x/(1 + t 2) par une fonction intégrable sur R. Lorsque x > 0, la
suite de fonction (fn ) converge simplement vers la fonction nulle, et lorsque x < 0, (fn ) converge
simplement vers t → 1/(1 + t2 ) sur R. Ainsi, le théorème de convergence dominée donne
 +∞
dt
lim F1/n(x) = 0 (x > 0), lim F1/n(x) = 2
= π (x < 0).
n→∞ n→∞ −∞ 1 + t
A partir de la relation établie dans la question précédente, on en déduit lorsque x > 0
1x  π
C (x) = lim I1/n (x) = e lim F1/n(x) + e −x lim F1/n (−x) = e−x .
n→∞ 2 n→∞ n→∞ 2
π −|x|
Par parité on obtient C (x) = 2 e pour x < 0. La valeur de C (0) = π/ 2 est immédiate.
Pour calculer S (x), le principe est maintenant de dériver l’expression précédente, mais c’est
délicat car l’intégrande dérivée n’est pas intégrable sur R+. Supposons x > 0 et commençons
par montrer que l’intégrale impropre S (x) converge. En effet, une intégration par parties donne
 T  T  T
t sin(xt) cos(xt) t cos(xt) 1 − t 2
ST (x) = dt = − + dt.
0 1 + t2 x 1 + t2 0 0 x (1 + t2 ) 2
192 3. INTÉGRATION

Le premier terme de la dernière somme converge lorsque T → +∞, et le dernier également


puisque son intégrande est intégrable sur R+ . L’intégrale S (x) converge donc
 n pour tout x 2> 0.
Considérons maintenant la suite de fonction (Cn ) définie par Cn (x) = 0 cos(xt)/(1 + t ) dt
(qui converge simplement vers C (x)). Pour tout n ∈ N, Cn est dérivable, et C n (x) = S n(x). En
intégrant par parties, on a
 +∞  +∞
t sin(xt) cos(xn) n cos(xt) 1 − t2
Sn (x) − S (x) = dt = + dt,
n 1 + t2 x 1 + n2 n x (1 + t2)2
donc on a la majoration, pour a > 0 fixé
 +∞
1 1 1
∀x ≥ a, |C n(x) − S (x)| = |Sn (x) − S (x)| ≤ + dt.
an n a 1 + t2
Le terme à droite de la dernière inégalité tend vers 0 lorsque n → +∞, on en déduit que la
suite de fonctions (C n) converge uniformément vers S sur [a, +∞[. Donc C est bien dérivable
sur [a, +∞[ et C (x) = S (x) pour x ≥ a. Ceci étant vrai pour tout a > 0, on en déduit que C est
dérivable sur ]0, +∞[ et que C  = S sur cet intervalle. A partir de la forme close C (x) = π2 e −x
on en déduit S (x) = C  (x) = − π2 e−x. Le résultat pour x < 0 s’en déduit car S est une fonction
impaire, et pour x = 0 car l’intégrande de S est nulle dans ce cas.

Problème 18 (Théorème des résidus, version faible). On considère une fraction


rationnelle R à coefficients complexes, intégrable sur R. On note P + = {z ∈ C | (z ) > 0}
et a1, . . . , an la liste des pôles de R (ai = aj pour i = j ). Pour tout k , on note Res R(ak)
le coefficient de 1/(X − ak ) dans l’écriture de R en éléments simples.
a) Montrer
n
ResR (ak ) = 0.
k=1

b) Montrer
 +∞ 
I= R(t) dt = 2iπ ResR(ak ).
−∞ a k∈P +

Solution. a) On peut écrire R = P/Q avec P, Q ∈ C[X ]. Comme R est intégrable sur R, on a
deg(Q) ≥ deg(P ) + 2. Pour tout k, 1 ≤ k ≤ n, notons αk l’ordre de multiplicité du pôle ak . On
peut écrire
n
α 
 k
A k,
R(X ) = Ak, ∈ C et Ak,1 = Res R (a k).
(X − ak ) 
k=1 =1

Comme R = P/Q avec deg(Q) ≥ deg(P ) + 2, on a


n
α  n n
  k
Ak, t  
0 = lim tR(t) = lim = A k,1 = ResR (ak).
t→+∞ t→+∞ (t − a k) 
k=1 =1 k=1 k=1

b) La fonction R est intégrable sur R donc aucun de ses pôles a k n’est réel. De plus, pour tout
k, on a
 +∞   +∞
dt 1
∀ ≥ 2, 
= = 0,
−∞ (t − ak ) (1 − )(t − a k)−1 −∞
donc  n 
 +∞  Ak,1
I= dt. (∗)
−∞ t − ak
k=1
5. PROBLÈMES 193

On ne peut pas à ce stade intervertir les signes de sommation. On va s’en sortir en montrant que
T
−T
dt/(t − a k ) converge lorsque T → ∞. Pour tout k, écrivons a k = xk + iy k avec xk, yk ∈ R
(et yk = 0 puisque a k ∈ R). La primitive suivante
   
dt (t − xk ) + iyk 1  2 2
 t − xk
= dt = log (t − xk ) + y k + i arctan + K,
t − ak (t − xk )2 + yk2 2 yk
permet d’affirmer
 T    
dt 1 (T − x k) 2 + y2k T − xk T + xk
= log + i arctan + arctan
−T t − ak 2 (T + x k) 2 + y2k yk yk
d’ou on déduit 
 T
dt iπ si yk > 0,
lim =
T →+∞ −T t − ak −iπ si yk < 0.
Donc d’après (*) on a
 
n
   T
  
dt
I= Ak,1 lim = iπ  ResR (ak ) − ResR (ak )  .
T →+∞ −T t − ak yk >0 y k<0
k=1
 
D’après la question a), on a ResR (a k) = − yk>0 Res R(a k ), donc finalement
yk <0
 
I = 2iπ ResR(a k ) = 2iπ Res R(ak ).
y k>0 ak ∈P+

Remarque. Ce résultat est en fait une conséquence du théorème des résidus. Ce dernier
s’applique dans le cadre beaucoup plus général de la théorie des fonctions analytiques.

Problème 19. a) En utilisant le résultat du problème précédent, calculer, pour tout


(n, m) ∈ N2, n < m, l’intégrale
 +∞
x 2n
I= 2m
dx.
−∞ 1 + x

b) En déduire, pour tout α > 1, la valeur de


 +∞
dt
J (α) = .
0 1 + tα

Solution. a) L’intégrande est bien intégrable sur R car son dénominateur ne s’annule par sur
R et à l’infini, elle est équivalente à x2(n−m) et n − m ≤ −1. En vue d’appliquer le résultat du
problème précédent, recherchons les résidus de l’intégrande. Son dénominateur s’écrit
2m−
1    
2m (2k + 1)iπ 2k+1 iπ
X +1= (X − ξk ), ξ k = exp =α avec α = exp
2m 2m
k=0

donc tous les pôles sont simples, et on a la décomposition en éléments simples


2
m−1
X 2n Ak ξ 2n
k ξk2n+1 α (2n+1)(2k+1)
R(X ) = 2m = avec Ak = = − = − ,
X +1 X − ξk
k=0
2mξ2km−1 2m 2m

autrement dit, on a 2m résidus A k associés aux pôles ξ k qui sont les valeurs
 
β 2k+1 2n+1 (2n + 1)iπ
Ak = − , β=α = exp .
2m 2m
194 3. INTÉGRATION

Comme les résidus de R dont les parties imaginaires sont strictement positives sont les A k pour
0 ≤ k ≤ m − 1, le résultat du problème précédent entraı̂ne
m−1
 m−1
2iπ  2k+1 iπβ 1 − β2m iπ 2 π
I = 2iπ Ak = − β =− 2
= = 2n+1  .
2m m 1−β m β − 1/β m sin 2m π
k=0 k=0

b) Nous allons exploiter l’idée suivante : si α est de la forme 2m/(2n + 1), le changement de
variable u = t1/(2n+1) nous ramène à une intégrale du type de a).
La fonction fα : t → 1/(1 + t α) est intégrable sur R+ pour tout α > 1. Si a > 1, l’application
α → J (α) est continue sur [a, +∞[ (on a f α ≤ fa pour α ≥ a donc l’hypothèse de domination
est vérifiée). Ceci étant vrai pour tout a > 1, J est continue sur ]1, +∞[.
Fixons maintenant α > 1. Comme Q est dense dans R, on peut trouver deux suites d’entiers
(pn) et (qn ) tendant chacune vers l’infini, telles que (pn/qn) converge vers α. Comme qn → +∞,
la suite (2pn /(2qn + 1)) converge aussi vers α. Le fait que α > 1 assure l’existence d’un rang à
partir duquel pn > qn .
Lorsque p > q sont deux entiers naturels, le changement de variable u = t1/(2q+1) donne
   +∞ 2q    −1
2p u du 2q + 1 +∞ u2q du 2p π
J = (2q + 1) = =π sin .
2q + 1 0 1 + u2p 2 −∞ 1 + u
2p 2q + 1 2p
2q+1

Ainsi, la fonction J étant continue en α , on a


    −1
2pn 2p n π π
J (α) = lim J = lim π sin 2p = .
n→+∞ 2q n + 1 n→ + ∞ 2qn + 1 n
2q +1
α sin(π/α)
n

Problème 20 (Une preuve du théorème de convergence dominée). On note


Cm(I, F ) l’e.v. des fonctions continues par morceaux sur I à valeurs dans F .
1/ Soit S = [a, b] un segment de R et f ∈ C m(S, R). Soit ε > 0. Montrer qu’il existe deux
fonctions continues f − et f+ dans de S dans R telles que
    
f− ≤ f ≤ f + et f+ − ε < f< f− + ε.
S S S

2/ Soit I un intervalle de R et E un e.v.n complet. Soit (u n) une suite d’éléments de


Cm(I, E), telle que u n est intégrable sur I pour tout n, et telle que la série de fonc-
tions
  un converge simplement vers une fonction f ∈ C m(I, E) sur I . On suppose que
n I u n converge.   
a) Soit S = [a, b] un segment inclus dans I . Montrer que S f  ≤ ∞ n=0 S u n  (com-
mencer par le cas où f et les un sont continues, en appliquant à f  et un  un argument
similaire à celui utilisé pour la preuve du premier théorème de Dini, puis adapter le rai-
sonnement).
b) Montrer que f est intégrable sur I et que
 ∞    ∞ 
f  ≤ un  et f= un .
I n=0 I I n=0 I

3/ Soit (fn ) une suite d’éléments de Cm (I, R+) convergeant simplement vers la fonction
nulle sur I , et telle qu’il existe une fonction ϕ ∈ Cm (I, R+ ), intégrable sur I , vérifiant
fn ≤ ϕ pour tout n.
a) Soit n ∈ N. Pour p ≥ n,on pose fn,p = max(fn , fn+1 , . . . , f p). Montrer qu’il existe
pn ≥ n tel que p n ≥ p n−1 et | I fn,p − I f n,pn | < 2−n pour tout p ≥ p n.
b) On pose g n = f n,pn . Montrer que |gn+1 − gn | ≤ 2(fn,pn+1 − f n,p n) + g n − gn+1 .
5. PROBLÈMES 195


c) En déduire que limn→∞ I f n = 0.
4/ Démontrer le théorème de convergence dominée (page 151).

Solution. 1/ Si f est continue c’est évident. Sinon, notons x1 < x2 < · · · < xn les points de
discontinuité de f . Soient deux réels m et M tels que m < inf x∈S f (x) et M > sup x∈S f (x).
Soit α > 0 avec α < 12 min(x i+1 − x i). Considérons la fonction continue ϕ − définie sur S par
|x − x i |
ϕ−(x) = m + (M − m) , si x ∈ J i = [xi − α, x i + α ] ∩ S, ϕ−(x) = M ailleurs.
α
L’application f− = min(f, ϕ−) vérifie f − ≤ f par construction et est continue, car
— si x n’est pas l’un des xi , f est continue en x donc f − = 12 (f + ϕ− − |f − ϕ−|) l’est aussi ;
— pour tout i, f− (x i) = m donc f− = ϕ − sur un voisinage de x i, donc f − est continue en
xi .
Lorsque x n’est pas dans l’un des Ji , on a f− (x) = f (x) car f (x) < M = ϕ −(x). On a donc
 n  n 
(f − f− ) = (f − f−) ≤ (M − m) ≤ 2αn(M − m)
S i=1 Ji i=1 Ji

En choisissant α ≤ ε/(2(M − m)n), on a donc S(f − f −) < ε. On construit de manière analogue
f+ = max(ϕ +, f) avec ϕ+ = M − (M − m)|x − x i |/α sur Ji et ϕ + = m ailleurs.
2/ a) Comme indiqué, nous allons commencer par le cas où f et les u n sont continues. Soit
ε > 0. Pour tout n, considérons l’ensemble
 n


Fn = x ∈ S : f (x) ≥ ε + u k(x)
k=0
La suite (Fn ) est une suite décroissante de fermés
 de S (donc compacts). On a ∩ n≥0Fn = ∅
car pour tout x ∈ S , il existe n tel que f (x) − nk=0 uk (x) < ε, ce qui entraı̂ne x ∈ Fn. Une
suite décroissante de compacts
 non vide est non vide, donc il existe forcément n tel que Fn = ∅,
autrement dit f  < ε + nk=0 uk  sur S . Ceci entraı̂ne
 n  ∞ 

f  ≤ (b − a)ε + uk  ≤ (b − a)ε + uk .
S k=0 S k=0 S

Cette dernière majoration est vraie pour tout ε > 0 donc on a bien le résultat attendu.
Traitons maintenant le cas général. Soit ε > 0. D’après 1/, il existe une fonction continue g
sur S et une suite (vn ) de fonctions continues sur S telles que
   
ε
g ≤ f  avec f  < ε + g, et ∀n ∈ N, un ≤ v n avec vn ≤ n + u n .
S S S 2 S
n
On note Gn = {x ∈ S : g (x) ≥ ε + k=0 vk (x)}, de sorte  n que (Gn ) est une suite décroissante
de
n compacts. Leur intersection est vide (car f ( x ) − k=0 u k(x)
 < ε entraı̂ne g (x) < ε +
n
v
k=0 k (x)), donc il existe n avec Gn = ∅ . Ceci entraı̂ne g < ε + k=0 v k sur S, donc
   n   n  ∞ 

f  ≤ ε+ g ≤ (1+ b − a)ε + v k ≤ (3+b − a)ε + uk  ≤ (3+b − a)ε + uk .
S S k=0 S k=0 S k=0 S

On conclut comme précédemment.


 
b) Comme S uk  ≤ I u k, le résultat précédent entraı̂ne la majoration
 ∞ 
f  ≤ uk ,
S k=0 I

et cecipour tout segment S de I . Par définition, f est donc intégrable sur I et on a f  ≤
∞ n I
k=0 I uk . En appliquant ce même résultat à f − k=0 uk on obtient
  n

 ∞
 
 
f − uk  ≤ uk ,
I  
k=0 I k=n+1
196 3. INTÉGRATION

     
donc ( I f − nk=0 uk)n∈N tendvers 0 lorsque n → ∞. Comme  I f − nk=0 I uk  ≤ I f −
n ∞ 
k=0 uk , on en déduit limn→∞ I f = n=0 I un .
 
3/ a) Fixons n. Remarquons que les fn,p sont dans Cm(I, R) (le maximum de deux fonctions a
et b de Cm(I, R) s’écrit 12 (a + b + |a − b|) donc est dans C m (I, R)) et intégrables car f n,p ≤ ϕ. La
suite (fn,p )p est croissante,
 donc la suite (I n,p)p définie par In,p = I f n,p est croissante. Comme
elle est majorée par I ϕ, elle converge, donc c’est une suite de Cauchy, donc il existe p n tel que
|In,p − In,q | < 2−n pour p, q ≥ p n . On peut choisir pn aussi grand que voulu, d’où le résultat.
b) Si gn+1 − gn < 0, c’est immédiat car f n,pn+1 − f n,pn ≥ 0. Sinon il suffit de remarquer que
gn+1 − gn = fn+1,pn+1 − fn,pn ≤ f n,pn+1 − f n,pn .
c) Posons un = g n − gn+1 . L’inégalité précédente entraı̂ne
    
|un | ≤ 2|In,pn+1 − In,pn | + gn − g n+1 ≤ 2 1−n + gn − gn+1,
I I I I I
par sommation on obtient
p 
 p
   
1−n
∀p ∈ N, |un | ≤ 2 + g0 − gp+1 ≤ 4 + g 0.
n=0 I n=0 I I I

Par ailleurs, la suite de fonctions (gn ) converge simplement
 vers 0, donc on a k≥n uk = g n .
Finalement nous avons montré que la série de fonctions k≥n uk vérifie les hypothèses de la
partie 2/ du problème, donc
 
    
0 ≤ fn ≤ g n =  
uk = uk .
I I I k≥n k≥n I

Le dernier terme est le reste d’une série absolument convergente, d’où limn→∞ I fn = 0.
4/ Reprenons les notations du théorème de convergence dominée, page 151. On a f  ≤ ϕ donc
f est bien intégrable. Ensuite, il suffit d’appliquer le résultat de la question précédente à la suite
(fn − f ), majorée
 par la fonction intégrable 2ϕ, et
 qui converge
 simplement
 vers la fonction
nulle. Donc ( I fn − f ) tend vers 0, et comme  I fn − I f  ≤ I fn − f , ceci termine la
preuve du théorème de convergence dominée.

Problème 21 (Deuxième preuve du théorème de convergence dominée).


Quelques définitions et remarques.
— Une partie bornée E de R est dite élémentaire si E est réunion finie d’intervalles
bornés, i. e. si χE (fonction caractéristique de E ) est
 une fonction en escalier. On
peut alors définir la mesure de E par m(E ) = R χ E(t) dt. De même, si f est
 b
continue par morceaux sur [a, b] ⊃ E , on définit E f = a f (t)χE (t) dt.
— Si E et F sont deux parties élémentaires
 disjointes
 et f une fonction réelle continue
par morceaux,
 on a facilement E ∪F f = E f + F f ; si |f (x)| ≤ K pour tout x ∈ E ,
alors | E f | ≤ K m(E).
— Il est également clair que si E est élémentaire, alors pour tout ε > 0, il existe un
ensemble H, borné, fermé et élémentaire, tel que H ⊂ E et m(H ) > m(E ) − ε.
sous ensembles bornés de R telle que ∩ n∈N∗ An =
a) Soit (An ) n∈N∗ une suite décroissante de sous-suite

∅. Pour tout n ∈ N , on note
αn = sup Γ n où Γn = {m(E ), E élémentaire et E ⊂ A n}.
Montrer que limn→+∞ α n = 0.
b) Soit (fn) une suite de fonctions de [a, b] dans un e.v.n complet A, continue par mor-
ceaux, convergeant simplement vers une fonction f : [a, b] → A continue par morceaux,
5. PROBLÈMES 197

et uniformément bornée, c’est-à-dire


∃K > 0, ∀n ∈ N∗ , ∀x ∈ [a, b], fn (x) ≤ K.
Montrer que  
b b
lim f n(t) dt = f (t) dt.
n→+∞ a a

c) En déduire une preuve du théorème de convergence dominée (page 151).

Solution. a) Toute partie élémentaire incluse dans A n+1 est incluse dans A n , donc Γ n+1 ⊂ Γn ,
donc αn+1 ≤ αn . Autrement dit, la suite (αn ) est décroissante. Comme elle est minorée par 0,
elle converge. On note  sa limite.
Il s’agit de montrer  = 0. Pour cela, on raisonne par l’absurde en supposant  > 0. On fixe
un nombre réel δ tel que 0 < δ < . Pour tout n, il existe une partie En ⊂ An élémentaire et
fermée, telle que m(En) > αn − δ/2n . Pour tout n, l’ensemble
n

Hn = En
i=1
est élémentaire et fermé, et la suite (Hn ) est une suite décroissante de fermés. Si on montre que
H n est non vide pour tout n, alors ∩n∈N ∗ Hn sera non vide (suite décroissante de compacts non
vides), ce qui sera en contradiction avec le fait que
 
Hn ⊂ A n = ∅,
n∈N ∗ n∈N∗
et on en conclura le résultat.
Soit n ∈ N ∗ . Fixons une partie élémentaire E ⊂ A n telle que m(E ) > δ. On a
n
 n

E Hn = (E E i ) donc m(E Hn ) ≤ m(E E i ). (∗)
i=1 i=1
Or pour tout i, 1 ≤ i ≤ n,
δ
m(E Ei ) + m(E i ) = m(E ∪ Ei) ≤ α i donc m(E Ei ) ≤ α i − m(Ei ) ≤ ,
2i
ce qui d’après (*) entraı̂ne
n
 δ
m(E H n ) ≤ < δ.
2i
i=1
Ceci entraı̂ne H n = ∅ car m(E ) > δ par construction. D’où le résultat.
b) Pour tout n ∈ N∗ , posons g n = f − fn . On aura prouvé le résultat si on montre
 b
lim gn (t) dt = 0. (∗∗)
n→+∞ a
La suite de fonctions (gn ) converge simplement vers 0, et on a 0 ≤ gn ≤ 2K sur [a, b].
Ceci étant, considérons ε > 0. Pour tout n ∈ N∗, on pose
A n = {x ∈ [a, b] | ∃i ≥ n, g i (x) ≥ ε}.
La suite (An ) est décroissante et on a ∩ n∈NA n = ∅. D’après le résultat de la question a), on peut
donc trouver N ∈ N∗ tel que tout n ≥ N et pour toute partie élémentaire E ⊂ An , m(E ) < ε.
Soit n ≥ N , et soit s une fonction en escalier telle que s ≤ gn .
On pose E = {x ∈ [a, b] | s(x) ≥ ε} et F = [a, b]E . Les ensembles E et F sont élémentaires
(car s est en escalier). Comme E ⊂ An , on a m(E ) < ε donc
b    
s(t) dt = s+ s≤ 2K + ε ≤ 2Km(E) + (b − a)ε ≤ M ε, M = 2K + (b − a).
a E F E F
Ceci étant vrai pour toute fonction en escalier s inférieure à la fonction continue par morceaux

gn, on en déduit ab g n (t) dt ≤ M ε. Ceci étant vrai pour tout n ≥ N , on en déduit (**).
198 3. INTÉGRATION

c) On reprend les notations du théorème deconvergence  dominée


 page 151. Soit ε > 0, et soit
un segment J = [a, b] inclus dans I tel que I J ϕ = I ϕ − J ϕ < ε. D’après le résultat de la
question précédente appliqué au segment J (que l’on peut bien appliquer car ϕ est bornée sur
cet intervalle) il existe N ∈ N ∗ tel que  J fn − J f  < ε pour tout n ≥ N . Ainsi, pour tout
n ≥ N on a
          
          
 fn − f ≤  fn − fn  +  fn − f +  f − f ≤ ϕ + ε + ϕ ≤ 3ε,
       
I I I J J J J I I J I J
d’où le résultat.
Remarque. On adapte facilement cette démonstration dans les cas où les fn et f sont
seulement supposées Riemann-intégrables.
CHAPITRE 4

Suites et séries

Lautilisée
notion de suite et de limite naquit avec la méthode d’exhaustion, technique
par les mathématiciens grecs de l’Antiquité pour le calcul de longueurs,
d’aires et de volumes. C’est ainsi qu’Archimède approximait l’aire d’un cercle
en y inscrivant une suite de polygones réguliers.
Jusqu’au début du dix-neuvième siècle, le concept de convergence revêtait
deux formes : numérique et formelle. La conception formelle reposait sur des
règles formelles permettant de sommer des séries divergentes (Euler en fit
beaucoup usage), et occulta au cours du dix-huitième siècle la conception
numérique (qui correspond à celle d’aujourd’hui). Les dysfonctionnements du
point de vue formel dans le domaine des séries entières et des séries trigo-
nométriques provoquèrent au début du dix-neuvième siècle le développement
d’un point de vue purement numérique, marqué par les travaux de Gauss
(1813), Fourier (1807) et Bolzano (1817), puis Cauchy dans son cours d’ana-
lyse de l’École Polytechnique (1821). Les problèmes de convergence furent
approfondis par Abel, Dirichlet, Liouville, Riemann et Weierstrass. Abel, en

exhibant la série de fonctions (−1) n−1 sin(nx)/n, attira l’attention de ses
contemporains sur le fait qu’une limite simple d’une série de fonctions conti-
nues n’est pas forcément continue, résultat que l’on croyait vrai auparavant.
C’est Weierstrass qui donna à la définition de limite sa forme actuelle. Il reprit
aussi la notion de convergence uniforme introduite par son maı̂tre Gudermann
en 1838, et en donna une définition claire et précise dans un article écrit en
1841. Il énonça et démontra correctement les théorèmes sur la continuité,
dérivabilité et intégrabilité de la somme d’une série de fonctions. La notion
de convergence normale fut, elle, introduite par Baire en 1908.

1. Suites numériques
1.1. Rappels sur les suites
Nous avons déjà abordé la notion de suite au cours du chapitre de topologie sur les
espaces métriques. Rappelons en les grandes lignes.
— On appelle suite à valeurs dans un ensemble E toute application u de N dans E.
Le terme u(n) est appelé le n-ième terme de la suite et on le note un . La suite u
est notée (u n )n∈N.
— Une suite (un ) à valeurs dans un espace métrique (E, d) est dite convergente s’il
existe  ∈ E tel que
∀ε > 0, ∃N ∈ N, ∀n ≥ N, d(u n , ) ≤ ε.
Dans ce cas, il existe une seule valeur  vérifiant cette propriété, et on dit que  est
la limite de (un ) ou que (un) converge vers . On note alors  = lim n→+∞ u n.
— On appelle sous-suite (ou suite extraite) de (un) toute suite (v n) de la forme v n =
uϕ(n) où ϕ est une application strictement croissante de N sur N.
Avec ces définitions, on a les propriétés qui suivent.
— Toute sous-suite d’une suite convergente est convergente et possède la même limite.
200 4. SUITES ET SÉRIES

— Une suite (un ) à valeurs dans un espace métrique est dite bornée s’il existe M > 0
tel que d(u0 , un ) ≤ M pour tout n ∈ N. Toute suite convergente est bornée.
— Si (un ) est une suite bornée à valeurs dans Rp , on peut en extraire une sous-suite
convergente (voir la proposition 11 page 30).
— Une suite (un ) à valeurs dans un espace métrique (E, d) est dite de Cauchy si
∀ε > 0, ∃N ∈ N, ∀p ≥ N, ∀q ≥ N, d(up, uq) < ε.
Toute suite convergente est de Cauchy, toute suite de Cauchy est bornée. Si E est
complet, toute suite de Cauchy converge. En particulier, toute suite de Cauchy
dans Rp converge.
Suites réelles.
— Une suite réelle (un) est dite majorée (resp. minorée) s’il existe M ∈ R tel que
un ≤ M (resp. un ≥ M ) pour tout n ∈ N. Une suite croissante et majorée converge.
— Les inégalités larges sont conservées par passage à la limite.
— Si trois suites (un ), (vn ) et (wn ) vérifient
∀n ∈ N, v n ≤ u n ≤ wn
et si (vn ) et (wn) convergent vers une même limite , alors (u n) converge vers .
— Deux suites réelles (un) et (v n ) sont dites adjacentes si l’une est croissante, l’autre
décroissante, et si
lim u n − vn = 0.
n→+∞
Dans ce cas, (un ) et (v n ) sont convergentes et convergent vers la même limite.
1.2. Suites récurrentes
Définition 1. Soient (E, d) un espace métrique et h un entier naturel non nul. Une suite
(u n)n∈N à valeurs dans E est dite récurrente d’ordre h si on peut écrire
∀n ≥ h, un = f (un−1 , un−2 , . . . , un−h ) (∗)
où f est une application de E h dans E.
Les premières valeurs u 0, . . . , u h−1 de (u n) étant données, la relation (*) permet de
calculer de manière itérative tous les autres termes de la suite. Le plus souvent, les suites
récurrentes que nous traiterons seront d’ordre 1.
La proposition suivante permet souvent de calculer la limite d’une suite récurrente si
on sait par ailleurs qu’elle converge.
Proposition 1. Soit (E, d) un espace métrique et (un ) une suite récurrente d’ordre
h ∈ N∗ vérifiant
∀n ≥ h, un = f (un−1 , un−2 , . . . , u n−h)
où f : E h → E est une application. Si la suite (un ) converge vers une limite  et si
l’application f est continue au point (, . . . , ), alors on a
 = f (, , . . . , ).
La preuve est simple, il suffit de faire tendre n vers +∞ dans la relation (*) et d’utiliser
la continuité de f au point (, . . . , ).
Monotonie des suites réelles récurrentes d’ordre 1. Soit un intervalle I de R et
une fonction f : I → R telle que f (I ) ⊂ I . Considérons une suite (un ) vérifiant
u0 ∈ I et ∀n ∈ N, u n+1 = f (u n).
— Si f est croissante, la suite (u n) est monotone et son sens de monotonie est donnée
par le signe de u1 − u0 (immédiat par récurrence).
1. SUITES NUMÉRIQUES 201

— Si f est décroissante, la fonction f ◦ f est croissante. On en déduit que les suites


(u2n ) et (u2n+1 ) sont monotones, et leur sens de monotonie est opposé (ceci car le
signe de u2 − u0 est l’opposé du signe de u3 − u1 = f (u2 ) − f (u0 )).
1.3. Quelques familles de suites classiques
Suites arithmétiques. On appelle ainsi les suites (un ) à valeurs dans un e.v E vérifiant
une relation de récurrence de la forme u n+1 = un + a où a ∈ E . On a alors u n = u0 + na
pour tout n ∈ N, et on dit que (un ) est une suite arithmétique de raison a.
Suites géométriques. Ce sont les suites à valeurs réelles (ou complexes) vérifiant une
relation de récurrence de la forme un+1 = q un . On a alors un = q n u0 pour tout n ∈ N et
on dit que (un ) est une suite géométrique de raison q . Si |q | > 1, la suite (un) diverge ; si
|q | < 1, elle converge et a pour limite 0 ; si q = 1, elle est constante.
Suites arithmético-géométriques. Ce sont les suites à valeurs réelles (ou complexes)
vérifiant une relation de récurrence de la forme un+1 = q un + a. Lorsque q = 1 on a affaire
à une suite arithmétique, et si q = 1, on a
a
∀n ∈ N, u n = q n(u0 − r ) + r avec r= .
1−q
Récurrences homographiques. On dit qu’une suite (un ) (réelle ou complexe) vérifie
une récurrence homographique si elle vérifie une relation de récurrence du type
ax + b
∀n ∈ N, un+1 = h(u n) avec h(x) = , ad − bc = 0. (∗)
cx + d
Une telle suite est définie pour tout n si et seulement si aucune de ses valeurs n’annule le
dénominateur de h. La proposition qui suit permet d’exprimer explicitement un.
Proposition 2. Soit (u n) une suite complexe vérifiant (*). On considère l’équation
h(x) = x ⇐⇒ cx2 − (a − d)x − b = 0. (E )
— Si (E ) admet deux racines distinctes α, β alors on a
un − α u0 − α a − αc
∀n ∈ N, = kn où k = .
un − β u0 − β a − βc
— Si (E ) admet une racine double α , alors
1 1 c
∀n ∈ N, = + k n où k = .
un − α u0 − α a − αc
Démonstration. Dans le premier cas, il suffit de remarquer que
h(x) − α x−α
=k ,
h(x) − β x−β
ce que le lecteur vérifiera facilement. Dans le second cas, il suffit de remarquer
1 1
= + k.
h(x) − α x−α

Remarque 1. Ces formules permettent de décider s’il existe un entier n qui annule le
dénominateur de h, en quel cas les termes ultérieurs de la suite ne sont pas définis.
– On peut montrer que si (E ) a deux racines distinctes, la valeur k est aussi égale à
k = cβ +d
cα+d
2c
; lorsque (E ) a une racine double, on a l’égalité k = a+ d
.
202 4. SUITES ET SÉRIES

Récurrences linéaires à coefficients constants. On dit qu’une suite (un ) à valeurs


complexes vérifie une récurrence linéaire (homogène) d’ordre h à coefficients constants si
∀n ≥ h, un = a1 u n−1 + a 2u n−2 + · · · + ahun−h (a 1 , . . . , ah ∈ C). ( ∗)
La proposition qui suit permet de calculer explicitement le terme général d’une telle suite.
Proposition 3. L’équation
xh − a 1 xh−1 − · · · − ah = 0 (E)
s’appelle équation caractéristique de la récurrence (*). Si on note r1 , . . . , rq ses racines
et α1 , . . . , αq leur ordre de multiplicité respectifs, alors l’ensemble des suites (u n) vérifiant
(*) est l’ensemble des suites de la forme
un = P1 (n) r n1 + · · · + Pq (n) rqn, (∗∗)
où pour tout i, Pi est un polynôme vérifiant deg(Pi) < αi .
Une preuve de ce résultat fait l’objet du problème 5 page 285. Dans la pratique, si on
se donne une suite (un ) vérifiant (*), les coefficients des polynômes Pi correspondant dans
(**) sont déterminés à partir des h premiers termes u0 , . . . , uh−1 de la suite.
Remarque 2. On rencontre souvent des récurrences linéaires à coefficients constants d’or-
dre 2 :
u0, u 1, ∀n ≥ 2, un = au n−1 + bun−2 . (∗∗∗)
L’équation caractéristique correspondante est
x2 − ax − b = 0, (E )
et dans ce cas, la proposition précédente s’énonce comme suit.
— Si (E ) possède deux racines distinctes x 1, x2 , les suites vérifiant (***) sont celles
de la forme
un = λxn1 + µxn2 . (∗∗∗∗)
Les coefficients λ et µ sont déterminés à partir des équations u0 = λ + µ et u1 =
λx1 + µx2 .
— Si (E ) possède une racine double x, les suites vérifiant (***) sont celles de la forme
u n = (λn + µ)x n.
On détermine λ et µ grâce aux équations u 0 = µ et u 1 = (λ + µ)x.
Lorsque a et b sont réels et que le discriminant ∆ = a2 + 4b de (E ) est strictement négatif,
l’expression (****) fait intervenir des nombres complexes. Dans ce cas, on peut en donner
une expression ne faisant intervenir que des nombres réels en écrivant les racines de (E)
sont la forme ρeiθ , ρe−iθ , de sorte que (****) se met sous la forme
un = ρn (γ cos(nθ) + δ sin(nθ )).
1.4. Exercices
Exercice 1. Étudier la suite (un) définie par
1
u0 > 0 et ∀n ∈ N, un+1 = √ .
2 − un

Solution. Pour que la suite (un ) soit bien définie, il faut avoir un ≥ 0 pour tout n, ce qui sera
vérifié si et seulement si 0 ≤ u n < 4 pour tout n.
Ceci étant, on a
1
∀n ∈ N, un+1 = f (un) où f : [0, 4[→ R x → √ .
2− x
1. SUITES NUMÉRIQUES 203

La monotonie de (un ) est dictée par le signe de g (x) = f (x) − x ; un calcul simple montre que
√ √ √  √ √ 
1+ 5 1− 5
( x − 1) x− 2 x− 2
∀x ∈ [0, 4[, g(x) = f (x) − x = √ ,
2− x
d’où le tableau suivant, donnant le comportement de f et le signe de g :

3+ 5
x 0 1 2 4

1 3+ 5
f (x) 2  1  2  +∞
1
g (x) 2 + 0 − 0 +

Nous avons vu que forcément, u0 ∈ [0, 4[ pour que la suite (un ) soit définie. Pour étudier (u n),
nous traitons plusieurs cas selon la position de u0 par rapport aux points fixes de f .
(i) u0 ∈ [0, 1[. Le tableau montre que [0, 1[ est stable par f , on a donc u n ∈ [0, 1[ pour tout
n. Par ailleurs, g est positive sur cet intervalle, donc la suite (un ) est croissante. Comme
elle est majorée (par 1), elle converge. Sa limite  vérifie f () = , c’est-à-dire g () = 0.
Comme de plus  ∈ [0, 1] car la suite prend ses valeurs dans [0, 1[, on a forcément  = 1.
En résumé, (un ) tend vers 1 en croissant.
(ii) u0 = 1. Alors√la suite (u n) est stationnaire à 1.√
(iii) u 0 ∈ ]1, (3+ 5)/2[. Comme l’intervalle ]1, (3+ 5)/2[ est stable par f , tous les éléments
de la suite appartiennent à cet intervalle. Comme g y est négative, (un ) décroı̂t. De plus,
(un) est minorée
√ (par 1), on en déduit qu’elle converge. Sa limite  vérifie g () = 0 et
1 ≤  < (3 + √ 5)/2, donc  = 1. En résumé, (un ) tend vers 1 √ en décroissant.
(iv) u0 = (3 +√ 5)/2. Alors la suite (u n) est stationnaire à (3 + 5)/2. √
(v) u 0 > (3 + 5)/2. Alors la suite (u n ) est croissante (g est positive √ sur ](3 + 5)/2, 4[) ; si
elle était majorée par 4, elle convergerait vers un réel  > (3 + 5)/2 point fixe de f , ce
qui n’est pas possible. Ainsi, il existe n tel que un > 4 et la suite (un ) n’est pas définie.

 Exercice 2 (Moyenne de C ésaro). Soit (an)n≥1 une suite complexe convergente, de


limite . Montrer que la suite (b n)n≥1 définie par
a1 + · · · + an
∀n ≥ 1, bn =
n
converge vers .
Solution. Donnons nous ε > 0. La suite (an ) converge vers  donc
∃N ∈ N∗ , ∀n > N, |a n − | ≤ ε.
Ainsi, pour tout n > N ,

∀n > N, |a1 + a 2 + · · · + an − n| ≤ |a1 + · · · + a N − N | + |aN+1 + · · · + an − (N − n)|


≤ |a 1 + · · · + aN − N | + |aN+1 − | + · · · + |an − | ≤ K + (n − N )ε ≤ K + nε,
où K = |a1 + · · · + aN − N |, ce qui entraı̂ne
 
 a1 + · · · + an  K
∀n > N, |bn − | =  −   ≤ + ε.
n n
Si on fixe un entier N 1 > N tel que K/N1 < ε, on a finalement
K
∀n ≥ N 1, |bn − | ≤ + ε ≤ 2ε,
N1
d’où le résultat.
204 4. SUITES ET SÉRIES

Remarque. En procédant de la même manière, on peut  montrer plus généralement que


pour toute suite de réels positifs (εn ) telle que la série ε n diverge, on a
ε 1 a1 + · · · + ε nan
lim = .
n→+∞ ε1 + · · · + ε n

Exercice 3. Soit (un ) une suite définie par



u 0 > 0, u 1 > 0, λ > 0, ∀n ∈ N, un+2 = λ un+1 un .
Expliciter le n-ième terme un de la suite en fonction de n.

Solution. Une récurrence immédiate montre que chaque terme de la suite est positif, donc la
suite (u n ) est bien définie. La suite (un ) est récurrente d’ordre 2, mais elle n’entre pas dans une
des familles classiques étudiées dans la partie 1.3. Pour se ramener à une récurrence classique,
nous allons considérer la suite (v n) définie par vn = log un. Elle vérifie
v n+1 vn
∀n ∈ N, v n+2 = + + log λ. (∗)
2 2
Pour rendre homogène cette récurrence linéaire, nous commençons par en rechercher une solution
particulière (c’est comme pour les équations différentielles). Aucune suite constante ne convient,
on recherche donc une solution particulière (wn ) sous la forme w n = αn. Elle vérifie (*) si et
seulement si
 
1 2
∀n ∈ N, α(n + 2) = α n + + log λ, ce qui équivaut à α = log λ.
2 3
Ainsi, la suite (wn) définie par w n = (2 log λ/3)n vérifie (*), ce qui en retranchant à (*) montre
que la suite (xn ) définie par x n = vn − w n vérifie
xn+1 xn
∀n ∈ N, x n+2 = + .
2 2
On sait résoudre ce type de récurrence. L’équation caractéristique correspondante est
1 1 1
r2 − r − = 0 dont les solutions sont r = − , r = 1.
2 2 2
Ainsi, il existe a, b ∈ R tels que
 n  
1 1 n 2 log λ
∀n ∈ N, xn = a + − b donc ∀n ∈ N, vn = x n + w n = a + − b+ n,
2 2 3
et finalement
n
∀n ∈ N, u n = exp(v n) = A · B (−1/2) · λ2n/3 , A = ea , B = eb .
Pour déterminer A et B , on écrit
1 1/3 2/3 2/3 −2/3 4/9
u0 = AB, u 1 = A · √ λ 2/3 d’où on déduit A = u0 u1 λ−4/9 , B = u0 u1 λ .
B
Finalement, nous avons montré
 (2/3)(−1/2)n
1/3 2/3 u0 λ 2/3
∀n ∈ N, un = u0 u 1 λ−4/9 λ2n/3 .
u1

Remarque. Retenez le procédé qui consiste à rechercher une solution particulière pour
résoudre des récurrences linéaires non homogènes du type (*).
1. SUITES NUMÉRIQUES 205

Exercice 4. Soit α > 0 un nombre irrationnel et (rn ) une suite de nombres rationnels
qui converge vers α. Pour tout n, on écrit rn = pn /qn où pn ∈ Z, q n ∈ N ∗ . Montrer que
lim pn = lim q n = +∞.
n→+∞ n→+∞

Solution. Donnons nous un entier naturel non nul N . Notons Γ l’ensemble des rationnels de la
forme a/b qui se trouvent dans l’intervalle [α − 1, α + 1] et vérifient a ∈ Z, b ∈ N, 1 ≤ b ≤ N .
L’ensemble Γ est fini puisque
  
p
Γ= Γq avec Γ q = r ∈ [α − 1, α + 1] | ∃p ∈ Z, r =
q
1≤q≤N

et que pour tout q ∈ N∗, Γ q est fini (car si p/q ∈ Γ q , on a forcément (α − 1)q ≤ p ≤ (α + 1)q).
Ceci étant, le nombre α est irrationnel donc n’appartient pas à Γ. Comme Γ est fini, on en
déduit que
∃ρ ∈ ]0, 1[ , ∀x ∈ Γ, |x − α| > ρ. (∗)
La suite (rn ) converge vers α, donc il existe n0 ∈ N tel que |rn − α| < ρ pour tout n ≥ n0 .
D’après (*), on a donc rn ∈ Γ pour tout n ≥ n 0 , ce qui entraı̂ne qn > N (si qn ≤ N , comme
rn = p n /qn ∈ [α − 1, α + 1], on a rn ∈ Γqn ⊂ Γ, impossible). Finalement, nous avons prouvé
∃n0 ∈ N, ∀n ≥ n0 , q n ≥ N.
Ceci étant possible pour tout N ∈ N∗ , la suite (qn ) tend vers +∞. Comme pn ∼ αqn, la suite
(pn) tend également vers +∞.

 Exercice 5 (Sous-groupes additifs de R). a) Soit Λ un sous-groupe de (R, +),


Λ = {0}. Montrer que Λ vérifie l’une des deux assertions suivantes :
(i) il existe m > 0 tel que Λ = mZ,
(ii) Λ est dense dans R.
b) Soient a et b deux nombres réels non nuls. Montrer que l’ensemble aZ + bZ est dense
dans R si et seulement si a/b ∈ Q.
c) Soient a et b deux nombres réels strictement positifs. Montrer que aN − bN est dense
dans R si et seulement si a/b ∈ Q.
d) (Application.) Quelles sont les valeurs d’adhérence de la suite (un) définie par un =
sin n ?
Solution. a) C’est très classique. Notons Λ+∗ = Λ ∩R+∗ et m = inf Λ+∗ . Deux cas se présentent.
— Si m > 0, alors m ∈ Λ +∗ . En effet, si m ∈ Λ+∗ , on a par définition de la borne inférieure
∃α, β ∈ Λ+∗ , m < α < β < 2m,
donc 0 < β − α < m, et comme Λ est un groupe additif, β − α ∈ Λ+∗, ce qui est absurde
car m = inf Λ+∗.
Comme m ∈ Λ et que Λ est un groupe, on a mZ ⊂ Λ. Nous allons montrer l’inclusion
réciproque, ce qui prouvera mZ = Λ. Soit x ∈ Λ. Si n est la partie entière de x/m, on a
mn ≤ x < m(n +1), donc 0 ≤ x − mn < m. Comme x − mn ∈ Λ, ceci entraı̂ne x − mn = 0
car m = inf Λ+∗, autrement dit x = mn ∈ mZ. Finalement, on a montré Λ = mZ.
— Si m = 0, nous allons montrer que Λ est dense dans R. Soient a < b deux nombres réels.
Comme 0 = inf Λ+∗, il existe x ∈ Λ tel que 0 < x < b − a, et alors il existe n ∈ Z tel que
a < nx < b. Ainsi, ]a, b[ ∩ Λ = ∅, et ceci pour tout intervalle ouvert ]a, b[ de R, d’où la
densité de Λ dans R.
b) L’ensemble Λ = aZ + bZ est un sous-groupe de (R, +).
Condition nécessaire. Supposons a/b ∈ Q. Alors il existe deux entiers non nuls premiers entre
206 4. SUITES ET SÉRIES

eux p et q tels que a/b = p/q. On a donc


  
a  p b b
aZ + bZ = b Z + Z = b Z + Z = (qZ + pZ) = Z
b q q q
(on a pZ + qZ = Z car les deux entiers p et q sont premiers entre eux), ce qui entraı̂ne que Λ
n’est pas dense dans R. Ceci est contraire aux hypothèses, on en déduit a/b ∈ Q.
Condition suffisante. Supposons a/b ∈ Q et Λ non dense dans R. D’après la question précédente,
il existe m ∈ R tel que Λ = mZ. En particulier a ∈ Λ, donc il existe p ∈ Z tel que a = mp. De
même, il existe q ∈ Z tel que b = mq. Donc a/b = p/q ∈ Q, ce qui est absurde. L’ensemble Λ est
donc dense dans R.
c) Condition nécessaire. Supposons aN − bN dense dans R. Alors il en est de même pour aN −
bN ⊃ aZ − bZ, donc d’après la question précédente, a/b ∈ Q.
Condition suffisante. Si a/b ∈ Q, pour montrer que aN − bN est dense dans R, nous allons
commencer par montrer que
∀ε > 0, ∃(p, q ) ∈ N2, 0 < ap − bq < ε. (∗)
Quitte à diminuer ε > 0, on peut supposer ε < inf {a, b}. L’ensemble aZ + bZ est dense dans R
d’après la question précédente, donc
∃(p, q ) ∈ Z2, 0 < ap − bq < ε.
Les entiers p et q sont de même signe car ε < inf {a, b}. S’ils sont tous les deux positifs, on a
prouvé (*). Sinon, on réutilise la densité de aZ + bZ dans R qui entraı̂ne
ap − bq ε
∃(p , q ) ∈ Z 2, 0 < ap − bq  < < , K = sup{|p|, |q|}. (∗∗)
K K
Une nouvelle fois, p et q sont de même signe. S’ils sont positifs, on a prouvé (*). Sinon, (**)
entraı̂ne
0 < (ap − bq ) − K (ap − bq ) = a(p − Kp ) − b(q − Kq) < ε. (∗∗∗)
   
L’entier p − Kp est positif car p ≤ −1 donc p − Kp ≥ p + K ≥ 0 ; de même q − Kq ∈ N, et
finalement, (***) entraı̂ne (*).
L’ensemble aN − bN étant stable par addition, l’assertion (*) entraı̂ne la densité de aN − bN
dans R+. De même, bN − aN est dense dans R + , donc aN − bN est dense dans R− , et finalement
aN − bN est dense dans R.
d) Nous allons montrer que l’ensemble des valeurs d’adhérence de la suite (u n) est [− 1, 1]. Il
suffit pour cela de prouver que X = {sin n, n ∈ N} est dense dans [−1, 1]. Considérons la
fonction f : R → R x → sin x. On a f −1 (X) = N + 2π Z, et comme N − 2π N est dense dans R
(car 2π est irrationnel), on en déduit que f −1(X ) est dense dans R. Donnons nous a, b ∈ [−1, 1],
a < b. Comme f −1( ]a, b[ ) est ouvert (f est continue), on a
f −1 (]a, b[ ∩ X) = f −1 ( ]a, b[ ) ∩ f −1 (X ) = ∅,
donc ]a, b[ ∩ X = ∅. Ainsi X est dense dans [−1, 1], d’où le résultat.

Exercice 6. Soit (un ) une suite telle que



1 + un
u0 ≥ 0, et ∀n ∈ N, u n+1 = .
2
Prouver que la suite (vn ) définie par
n

∀n ∈ N, vn = ui
i=1
est convergente et calculer sa limite.
Solution. Nous traitons plusieurs cas, selon la position de u 0 par rapport à 1.
1. SUITES NUMÉRIQUES 207

– u0 < 1. Soit θ ∈ ]0, π/2] tel que u 0 = cos θ. La formule trigonométrique


  
u 1 + cos u θ
∀u ∈ R, cos = entraı̂ne immédiatement ∀n ∈ N, u n = cos .
2 2 2n
En utilisant maintenant l’identité
sin 2u
∀u ∈ π Z, cos u = ,
2 sin u
on voit que pour tout entier naturel non nul n,
     
θ θ θ sin θ sin(θ/2) sin(θ/2n−1) sin θ
vn = cos cos · · · cos n = · ··· n
= n ,
2 4 2 2 sin(θ/2) 2 sin(θ/4) 2 sin(θ/2 ) 2 sin(θ/2n)
et on en conclut que

sin θ 1 − u20
lim v n = = .
n→+∞ θ arccos u0
– u0 = 1. Dans ce cas, on a un = 1 pour tout n et tout est trivial.
– u0 > 1. On procède comme dans le cas u0 < 1 en remplaçant les fonctions trigonométriques
par les fonctions hyperboliques correspondantes. Si on écrit u 0 = ch θ, on montre que

sh θ u20 − 1
lim vn = = .
n→+∞ θ argch u 0

Remarque. En partant de u0 = 0, la suite (vn ) converge vers 2/π, ce qui s’écrit



    
 
2 1 1 1 1 1 1 1 1 1
= · + · + + ··· ,
π 2 2 2 2 2 2 2 2 2
formule découverte par François Viète (1540-1603), qu’il obtint en considérant l’aire de
polygones réguliers à 2n côtés.

Exercice 7. Soit (un ) n∈N∗ la suite définie par



u1 = 1, ∀n ≥ 2, un = n + u n−1 .
Donner un équivalent puis calculer les deux premiers termes du développement asympto-
tique de (un) lorsque n → +∞.

Solution. Une autre manière de voir les choses est d’écrire


 


∀n ∈ N , un = n + n − 1 + n − 2 + · · ·.

Dans cette expression, on “intuite” que seul le premier√terme n est prépondérant dans l’ex-
pression de (un ). Nous allons donc montrer que √u n ∼ n lorsque n → +∞. Pour cela, nous
commençons par montrer par récurrence u n ≤ 2 n pour tout n ∈ N∗ . Pour n = 1, c’est vrai.
Pour passer au rang n au rang n + 1, on écrit
√ √  √ 2
u2n+1 = n + 1 + un ≤ n + 1 + 2 n = ( n + 1)2 ≤ 2 n + 1 .
Maintenant, il suffit d’écrire
 
∗ √ √ √ √ 2
∀n ∈ N , n ≤ un ≤ n + u n−1 ≤ n + 2 n = n 1 + √ ,
n

ce qui entraı̂ne immédiatement un ∼ n.
208 4. SUITES ET SÉRIES

Pour calculer le second terme du développement asymptotique de (un), on écrit



 √
∗ √ √ √ n−1
∀n ∈ N , u n = n + un−1 = n + n − 1 (1 + o(1)) = n 1 + (1 + o(1))
n
 √   
√ 1 n−1 √ 1 1 √ 1
= n 1+ (1 + o(1)) = n 1 + √ (1 + o(1)) = n + + o(1).
2 n 2 n 2

Remarque. En itérant le procédé, on peut en fait calculer un nombre quelconque de termes


du développement asymptotique de (un ).

Exercice 8. Soit (un )n≥1 une suite réelle positive majorée. On dit que A ⊂ N est de
densité nulle si limn→+∞ n1 Card(A ∩ [0, n]) = 0. Montrer que (i) et (ii) sont équivalents :
n
1
(i) La suite (Sn ) définie par Sn = u k converge vers 0.
n k=1
(ii) Il existe une partie A ⊂ N de densité nulle telle que n→
lim+∞
un = 0.
n∈A

Solution. Commençons par prouver que (ii) =⇒ (i). Soit ε > 0. Soit M un majorant de (un ) et
soit N ∈ N tel que u n < ε pour tout n > N et n ∈ A. Lorsque n > N on a
N N
1  1  1  1 Card(A ∩ [0, n])
|Sn | ≤ uk + uk + uk < uk + ε + M .
n n N<k≤n n N<k≤n n n
k=1 k=1
k∈A k∈A

Lorque n → +∞ les termes de gauche et de droite de la dernière somme, tendent vers 0. Donc il
existe N ≥ N tel que pour n ≥ N , ces deux termes soient chacuns inférieurs à ε. On en déduit
que |Sn | < 3ε pour n ≥ N  , ce qui montre le résultat voulu.
Montrons maintenant (i) =⇒ (ii). La suite (Sn ) converge vers 0 donc la suite (αn ) définie
par αn = supk≥n S k converge également vers 0. On peut se placer dans le cas où la limite de
(un) n’est pas nulle, sinon il suffit de choisir A = ∅ et (ii) est prouvé. Dans ce cas on a α n > 0.
√ √
Définissons A = {n ∈ N∗ | u n ≥ α n }. Lorsque n ∈ A on a 0 ≤ un < α n donc lim un = 0.
n→+∞
n∈A
√ √
Lorsque k ∈ A ∩ [0, n], on a uk ≥ α k ≥ αn ce qui entraı̂ne
 n
uk 1  Card(A ∩ [0, n]) S √
Card(A ∩ [0, n]) ≤ √ ≤ √ uk donc ≤ √ n ≤ α n.
αn αn n αn
k∈A k=1
1≤k≤n

On en déduit que A est de densité nulle, et on a bien prouvé (ii).

2. Séries numériques
2.1. Généralités
Définition 1. Soit (un ) n∈N une suite à valeurs dans un espace vectoriel E . On appelle
série de terme général un la suite (S n)n∈N définie par
∀n ∈ N, Sn = u0 + u1 + · · · + un.

On note cette série un . Pour tout n ∈ N,  un s’appelle le terme d’indice n, Sn s’appelle
la somme partielle d’indice n, de la série un .
Lorsque E est un e.v.n, on dit que un converge si lasuite (Sn) converge. Dans ce
cas, la limite s’appelle la somme de la série et on la note +∞ n=0 un . Pour tout n ∈ N, on
2. S ÉRIES NUM ÉRIQUES 209

appelle alors reste d’indice n l’élément Rn défini par


+∞ n +∞
 
Rn = uk − uk = uk .
k=0 k=0 k=n+1

Exemple 1. — Séries arithmétiques. Les séries de la forme n∈N na où a est une
constante sont toujours divergentes dès que a = 0. Les sommes partielles de cette
séries peuvent s’exprimer de manière explicite grâce à la relation
n
n(n + 1)
∀n ∈ N, k= .
k=0
2

— Séries géométriques. Les séries n∈N qn où q est un nombre complexe, convergent
si et seulement si |q | < 1. Lorsque q = 1, les sommes partielles sont données par
n
 1 − q n+1
qk =
1−q
k=0
et si |q | < 1, la somme et les restes de la série s’expriment explicitement :
+∞
 ∞

1 qn
qk = et ∀n ∈ N, qk = .
1−q 1−q
k=0 k=n

Critère de Cauchy pour les séries. Le critère de Cauchy pour les suites s’étend
aisément pour les séries et donne le résultat suivant.

Proposition 1. Une série un à valeurs dans un espace de Banach converge si et
seulement si
∀ε > 0, ∃N ∈ N, ∀n ≥ N, ∀p ∈ N, un + · · · + un+p  < ε.

Corollaire 1. Si un est une série convergente, alors limn→+∞ un = 0.
Remarque
 1. La réciproque de ce corollaire est fausse ; par exemple, la série harmonique
1/n diverge (voir la proposition 2).
Séries absolument convergentes. Voyons une autre conséquence importante du cri-
tère de Cauchy pour les séries :

Th
 éor ème 1. Soit u n unesérie à valeurs dans un R-espace de Banach. Si la série
 un converge, on dit que un est absolument convergente, et dans ce cas, la série
u n est convergente.
Ainsi, on est souvent ramené à prouver la convergence d’une série à termes positifs. Le
but de la partie qui suit est de donner des conditions suffisantes pour assurer la convergence
d’une série à termes positifs.
2.2. Séries à termes positifs
Toute suite réelle croissante et majorée converge, et comme conséquence immédiate,
on a le résultat suivant.

Théorème 2. Une série u n à termes réels positifs converge si et seulement si la suite
(Sn) des sommes partielles est majorée.
On en déduit facilement :
 
Théorème 3. On considère deux séries réelles un et vn telles que
∀n ∈ N, 0 ≤ un ≤ vn .
   
Alors si vn converge, un converge ; si un diverge, v n diverge.
210 4. SUITES ET SÉRIES

 
 Théorème 4. Soient u n et vn deux séries
 à termes positifs. 
(i) Si v n = O (u n ) lorsque n → +∞ et si u converge, alors vn converge ;
   n  
(ii) si un ∼ vn lorsque n → +∞, alors les séries u n et v n sont de même nature.

Remarque 2. Attention, l’assertion (ii) de ce dernier théorème n’est vraie que pour des
séries à termes positifs (voir l’exercice 7 page 223 pour un contre-exemple avec des séries
à termes non positifs).

 Proposition 2 (Séries de Riemann). Soit α un nombre réel. La série de Riemann


 1

n≥1

converge si et seulement si α > 1.

Équivalents des sommes partielles et des restes. Le résultat qui suit est crucial
dans beaucoup d’exercices. Il complète le théorème 4.
 
 Théorème 5. Soient un et vn deux séries à termes positifs, telles que un ∼ v n
lorsque n → +∞. Alors 
(i) si un converge, v n converge et les restes vérifient
+∞
 +∞

uk ∼ vk , n → +∞ ;
k=n k=n
 
(ii) si un diverge, vn diverge et les sommes partielles vérifient
n
 n

uk ∼ vk , n → +∞.
k=0 k=0
 
Démonstration. On sait déjà que les séries un et vn ont même nature (c’est l’assertion (ii)
du théorème 4).
(i). Soit ε > 0. L’équivalence un ∼ vn entraı̂ne
∃N ∈ N, ∀k ≥ N, (1 − ε)u k ≤ vk ≤ (1 + ε)u k ,
donc
+∞
 +∞
 +∞

∀n ≥ N, (1 − ε) uk ≤ vk ≤ (1 + ε) uk ,
k=n k=n k=n
d’où (i).
(ii). Soit ε > 0. Comme précédemment, on commence par écrire
∃N ∈ N, ∀k ≥ N, (1 − ε)u k ≤ vk ≤ (1 + ε)u k .
On en déduit
N−1
 n
 n
 N−1
 n

∀n ≥ N, vk + (1 − ε) uk ≤ vk ≤ v k + (1 + ε) uk .
k=0 k=N k=0 k=0 k=N

Comme un diverge,
n chacun des
termes extrêmes de ces inégalités sont respectivement équiva-
lents à (1 − ε) k=0 uk et (1 + ε) nk=0 uk On en déduit
n
 n
 n

∃N ≥ N, ∀n ≥ N  , (1 − 2ε) uk ≤ vk ≤ (1 + 2ε) uk ,
k=0 k=0 k=0

d’où le résultat. 
2. S ÉRIES NUM ÉRIQUES 211

Application. Ce dernier résultat permet de donner des développements asymptotiques


de certaines suites ou séries. Pour illustrer ce propos, nous allons donner un développement
asymptotique à l’ordre 3 des nombres harmoniques Hn définis par
1 1 1
∀n ∈ N ∗, Hn = 1 + + + · · · + .
2 3 n

a) On commence par remarquer que lorsque n → +∞,


 
1 1
∼ log 1 + .
n n

Comme 1/n diverge et que les deux séries en présence sont à termes positifs, on peut
appliquer la partie (ii) du théorème 5 qui entraı̂ne
n    n    n 
 1 k+1  k+1
Hn ∼ log 1 + = log = log = log(n + 1),
k k k
k=1 k=1 k=1
autrement dit Hn ∼ log n.
b) Nous avons obtenu le premier terme de notre développement asymptotique. Pour ob-
tenir le suivant, on considère la suite Un = Hn − log n, et on écrit
 
1 1 1 1
U n − Un−1 = − log n + log(n − 1) = + log 1 − ∼− 2 (∗)
n n n 2n

donc la série (Un − Un−1) converge. Comme
n

∀n ∈ N , (Uk − Uk−1 ) = Un − U 1 ,
k=2

la suite (Un ) converge. Notons γ sa limite, de sorte que


Hn = log n + U n = log n + γ + o(1).

c) Poursuivons.
 En appliquant la partie (i) du théorème 5, on en déduit un équivalent des
restes de (U n − U n−1 ), ce qui s’écrit
+∞
 +∞
1  1
γ − Un = (Uk − Uk−1 ) ∼ − .
2 k2
k=n+1 k=n+1

Pour obtenir un équivalent de ce dernier terme, on écrit


 k+1  k
dt 1 dt
∀k ≥ 2, 2
≤ 2 ≤ 2
donc
k t k k−1 t
 +∞ +∞
  +∞ +∞
 1
1 dt 1 dt 1 1
∀n ≥ 1, = 2
≤ 2
≤ 2
= , d’où 2
∼ .
n+1 n+1 t k n t n k n
k=n+1 k=n+1

Finalement, on a démontré
+∞  
1 1 1 1 1
Un − γ ∼ ∼ , donc Hn = log n + Un = log n + γ + +o .
2 k=n k 2 2n 2n 2n
Remarque 3. — Il est important de retenir ce résultat. Le nombre réel γ est une
constante classique appelée constante d’Euler. On a γ = 0.577215664 . . ..
— On aurait pu poursuivre ce développement asymptotique en itérant la méthode.
Un développement asymptotique de Hn à un ordre quelconque fait l’objet du sujet
d’étude 3 page 321, par la formule d’Euler-Maclaurin.
212 4. SUITES ET SÉRIES

— Cette
 méthode est assez générale. On peut l’utiliser par exemple sur la série
log(n) pour obtenir un développement asymptotique de n! à plusieurs termes
(voir le commentaire de l’exercice 3 page 219 sur la formule de Stirling).
Comparaison série-intégrale.
Proposition 3. Soit f : R+ → R une fonction positive, continue par morceaux et
décroissante sur R +. Alors la suite (Un ) définie par
 n
∀n ∈ N, Un = f (0) + f (1) + · · · + f (n) − f (t) dt
0
  +∞
est convergente. En particulier, la série f (n) et l’intégrale 0
f (t) dt ont même na-
ture.
Démonstration. La décroissance de f entraı̂ne
 k+1
∀k ∈ N, f (k + 1) ≤ f (t) dt ≤ f (k).
k
On en déduit

n−1  k+1

∀n ∈ N, Un = f (k) − f (t) dt + f (n) ≥ f (n) ≥ 0
k=0 k

et  n+1
∀n ∈ N, Un+1 − U n = f (n + 1) − f (t) dt ≤ 0.
n
La suite (Un ) est décroissante et minorée, elle converge donc. 

Remarque 4. Ce résultat reste vrai si f est seulement supposée décroissante à partir d’une
certaine abscisse X (reprenez la preuve précédente).
– Si f estC1 à valeurs complexes et f  est intégrable sur R+ , on peut également montrer que
 +∞
la série f (n) et l’intégrale 0 f (t) dt ont même nature (voir l’exercice 12 page 227).
Exemple 2. — On retrouve avec ce résultat celui de la proposition 2 sur les séries de
Riemann.
— Si on applique ce résultat à la fonction f : x → 1/(1 + x), on montre que la suite
(Un) définie par
 n−1
1 1 1 1
Un = 1 + + · · · + − f (t) dt = 1 + + · · · + − log n
2 n 0 2 n
converge. En notant γ la limite de (Un) (c’est la constante d’Euler), on retrouve
ainsi le fait que
1 1
Hn = 1 + + · · · + = log n + γ + o(1).
2 n
Séries de Bertrand. On appelle ainsi les séries de la forme
 1
α β
, (α, β) ∈ R 2. (∗)
n≥2
n log n
On a vu (voir la proposition 6 page 149 sur les intégrales de Bertrand) que
 +∞ 
dt
converge ⇐⇒ ((α > 1) ou (α = 1 et β > 1)).
2 tα logβ t
Si α ≤ 0, il est clair que la série de Bertrand (*) diverge pour tout β ∈ R (on peut dire
par exemple que le terme général est supérieur à 1/n à partir d’un certain rang). Si α > 0,
2. S ÉRIES NUM ÉRIQUES 213

la fonction f (t) = t−α log−β t étant décroissante au voisinage de +∞, on en déduit (voir
la remarque précédente) que
 
 1
α logβ n
converge ⇐⇒ ((α > 1) ou (α = 1 et β > 1)).
n≥2
n

2.3. Quelques recettes


 
Proposition 4. Soient un et v n deux séries à termes strictement positifs, telles
qu’à partir d’un certain rang, on ait un+1 /un ≥ vn+1 /v n. Alors
 
(i) si un converge, vn converge ;
 
(ii) si vn diverge, un diverge.
Démonstration. Soit N ∈ N tel que un+1 /u n ≥ vn+1 /v n pour tout n ≥ N . Une récurrence
immédiate montre que
un vn uN
∀n ≥ N, ≥ ou encore un ≥ Kv n avec K = .
uN vN vN
On conclut facilement en appliquant le théorème 3 page 209. 

Corollaire 2. Soit un une série à termes strictement positifs vérifiant
un+1 1
= , a ∈ R, n → +∞.
un 1 + a/n + o(1/n)

Alors si a > 1, la série un converge ; si a < 1, la série diverge.
Démonstration. Supposons a > 1, et fixons  un nombre réel b tel que 1 < b < a. Considérons la
suite (vn ) définie par v n = n−b . La série vn converge et
vn+1 1 1
= b
= .
vn (1 + 1/n) 1 + b/n + o(1/n)

Comme b < a, on en déduit qu’à partir d’un certain rang, vn+1 /vn ≥ un+1 /un , donc un
converge d’après la proposition précédente.
Si a < 1, on montrerait en procédant de la même manière qu’à partir d’un certain rang,

un+1/u n ≥ vn+1/vn où vn = n−b , b étant fixé tel que a < b < 1. Comme vn diverge, on en

déduit toujours avec la proposition précédente que un diverge. 

Remarque 5. Si a = 1, on ne peut pas conclure quant à−1la nature de la série. Considérons



β
par exemple la série de Bertrand
 un avec u n = n log n. Lorsque β < −1, un
converge ; lorsque β ≥ −1, u n diverge, et on a pour tout β
   −1   −1
u n+1 1 β 1 1 1
= 1+ + +o = 1+ +o .
un n n log n n log n n n

Règle de Raab-Duhamel. Proposition 5 (R ègle de Raab-Duhamel). Soit (un)


une série à termes > 0 telle que
un+1 1
= n → +∞.
un 1 + a/n + O (1/n 2)
Alors il existe λ > 0 tel que un ∼ λ/na lorsque n → +∞.
Démonstration. Il s’agit de montrer que la suite (nau n) converge et a une limite > 0. Pour cela,
on considère la suite (v n) définie par vn = log(n a un). Pour l’étudier, on considère la série wn
214 4. SUITES ET SÉRIES

où pour tout n, wn = vn+1 − v n . On a


         
n+1 a un+1 1 a 1
w n = log + log = a log 1 + − log 1 + + O
n un n n n2
      
a 1 a 1 1
= +O 2
− +O 2
=O ,
n n n n n2

donc la série wn converge. Comme w1 + · · · + wn = vn+1 − v 1 , la suite (vn ) converge. Donc
na un = exp(vn ) converge vers une limite > 0, d’où le résultat. 

Remarque 6. — Cette règle permet de déterminer la nature de la série un : elle
converge si et seulement si a > 1.
— Sans la présence du O (1/n2),le résultat est faux (nous avons vu à la remarque 5 le
cas d’une série convergente un pour laquelle un+1 /un = (1 + 1/n + o(1/n))−1 ).
— La règle de Raab-Duhamel reste vérifiée lorsqu’on remplace O (1/n 2 ) par O (1/nα)
avec α > 1 (pour s’en persuader, reprendre la preuve dans ce cas).
Règle de d’Alembert, règle de Cauchy.


Proposition 6 (Règle de d’Alembert). Soit un une série à termes > 0 telle que
un+1
lim = λ, λ ∈ [0, +∞].
n→+∞ un

Alors 
(i) si λ < 1, un converge ;
(ii) si λ > 1, un diverge ; 
+
(iii) si λ = 1 (i. e. si un+1 /un tend vers 1 en restant supérieur à 1), un diverge.


Proposition 7 (R ègle de Cauchy). Soit un une série à termes > 0 telle que

lim n u n = λ, λ ∈ [0, +∞].
n→+∞

Alors 
(i) si λ < 1, un converge ;
(ii) si λ > 1, un diverge ; 
(iii) si λ = 1 (i. e. si (un) 1/n tend vers 1 en restant supérieur à 1),
+
un diverge.

Remarque 7. On peut montrer que si un+1 /un → λ, alors (u n)1/n → λ. La réciproque est
fausse (par exemple, la suite u n = 2 + (−1)n vérifie (un) 1/n → 1 mais u n+1/un ne converge
pas).
2.4. Séries semi-convergentes
On appelle ainsi les séries convergentes mais non absolument convergentes.

Séries alternées. Ce sont les séries réelles un dontn les termes changent alternative-
ment de signe. Au signe près, on peut les écrire (−1) a n où an ≥ 0 pour tout n.
Théorème 6. Soit  (a n ) une suite à termes positifs, décroissante, tendant vers 0. Alors
la série alternée (−1)na n converge, et les restes
+∞

∀n ∈ N, Rn = (−1)k ak vérifient |Rn | ≤ an+1.
k=n+1
2. S ÉRIES NUM ÉRIQUES 215

n k
Démonstration. Pour tout n ∈ N, on note Sn = k=0 (−1) ak . La suite (an ) étant décroissante,
on a
∀n ∈ N∗, S2n − S2n−2 = a 2n − a2n−1 ≤ 0, S2n+1 − S 2n−1 = a 2n − a 2n+1 ≥ 0.
Autrement dit, la suite (S2n) est décroissante, (S 2n+1 ) est croissante. Or S2n+1 − S 2n = −a2n+1 ,
donc limn→+∞ S2n+1 −S2n = 0. Les suites (S2n ) et (S 2n+1) sont donc adjacentes. Elles convergent
donc vers une même limite S . La suite (Sn ) converge donc vers S et
∀n ∈ N, S 2n+1 ≤ S ≤ S 2n.
Ceci entraı̂ne
∀n ∈ N, |R2n | = |S − S 2n| ≤ S2n − S2n+1 = a2n+1 ,
de même
∀n ∈ N∗ , |R 2n−1| = |S − S 2n−1 | ≤ S2n − S2n−1 = a2n ,
ce qui montre |Rn | ≤ an+1 pour tout n ∈ N. 

Transformation d’Abel. La transformation  d’Abel est aux séries ce que l’intégration



par parties est aux intégrales. Soit une série unavec un = αnv n . On note Sn = nk=0 vk .
Effectuer une transformation d’Abel sur la série u n c’est écrire, pour tout n,
n
 n
 n

uk = αk v k = α 0 v0 + α k(S k − Sk−1 )
k=0 k=0 k=1
n
 n−1
 n−1

= α 0 v0 + αkS k − αk+1 Sk = (αk − αk+1 )Sk + αnSn
k=1 k=0 k=0

(on peut comparer cette expression à celle de l’intégration par parties, en disant que
αk+1 − α k est la “dérivée” de (α k ) et S n est une “primitive” de (v n)). Grâce à cette
technique, on montre le résultat suivant. C’est la version pour les séries du théorème 5 de
la page 152.

Théorème 7 (Règle d’Abel). Soit un une série à valeurs dans un espace de
Banach. On suppose que pour tout n, un = αn v n où
— (αn) est  une suite positive, décroissante et tend vers 0 ;
— la série  v n est bornée.
Alors la série u n est convergente.
Démonstration. Pour tout n, notons Sn = v0 + · · · + vn. 
Par hypothèse, il existe M > 0 tel que
Sn  ≤ M pour tout n. Une transformation d’Abel sur un donne
n
 n−1


∀n ∈ N , uk = (αk − α k+1)Sk + αn Sn . (∗)
k=0 k=0

La série k (αk − α k+1 )Sk converge absolument car
n
 n

∀n ∈ N, (αk − α k+1)Sk  ≤ (αk − α k+1 )M = (α0 − α n+1)M ≤ α 0 M
k=0 k=0

Par ailleurs, (αnS n) tend vers 0 car (Sn ) est bornée et (αn) tend vers 0. On en conclut avec

l’expression (*) que la série un converge. 

Exemple 3. — En prenant αn = an et v n = (−1)n , on retrouve le résultat de conver-


gence du théorème 6.
216 4. SUITES ET SÉRIES


— Une série de la forme αne niθ , où (αn ) est une suite décroissante tendant vers 0
et où θ ∈ R2π Z, converge. En effet, on a
 
   1 − e(n+1)iθ 
2 1
1 + eiθ + · · · + eniθ  =  
∀n ∈ N,  1 − eiθ  ≤ |1 − e iθ | = | sin(θ/2)| ,
donc  la série converge d’après la règle d’Abel. On en déduit en particulier que la
série eniθ /n α converge pour tout α > 0.
2.5. Séries commutativement convergentes, produit de Cauchy, séries doubles

Séries commutativement convergentes. On appelle ainsi les séries n∈N un telles
que pour toute bijection ϕ de N dans N, la série n∈N uϕ(n) converge. En particulier, une
série commutativement convergente est convergente.

Théorème 8. Une série un à valeurs dans un espace de Banach et absolument conver-
gente est commutativement convergente. De plus, pour toute bijection ϕ : N → N, on
a
+∞ +∞
 
un = uϕ(n) .
n=0 n=0

Démonstration. Soit ϕ une bijection de N dans N. Pour tout n ∈ N, on a


n
 +∞

uϕ(k) ≤ uk,
k=0 k=0

donc uϕ(n) converge absolument, donc converge. Montrons l’égalité des sommes des deux

séries. Soit ε > 0 et soit N ∈ N tel que +∞ n=N +1 un  < ε. Comme ϕ : N → N est une bijection,
il existe N ∈ N tel que {0, 1, . . . , N } ⊂ {ϕ(0), ϕ(1), . . . , ϕ(N )}. On en déduit

 +∞   +∞  N   N 
 +∞
    N   N
   +∞
 
       
 u n − uϕ(n)  ≤  u n − un +  u n − u ϕ(n)  +  uϕ(n) − uϕ(n)
       
n=0 n=0 n=0 n=0 n=0 n=0 n=0 n=0
+∞
 +∞
 +∞
 +∞

≤ u n + u n  + u ϕ(n) ≤ 3 u n < 3ε.
n=N +1 n=N +1 n=N  +1 n=N +1
+∞ +∞
Cette inégalité est vraie pour tout ε > 0, on en conclut n=0 un = n=0 u ϕ(n) . 

Produit de Cauchy.

 
Théorème 9 (Produit de Cauchy). Soient p∈N ap et q∈N b q deux séries absolu-
ment convergentes à valeurs dans une algèbre normée complète. Alors la série
 n
cn avec cn = akbn−k ,
n∈N k=0
 
est appelée produit de Cauchy de ap et b q , elle est absolument convergente et on a
+∞
 +∞   +∞ 
  
cn = ap bq . (∗)
n=0 p=0 q=0

Démonstration. La série c n est absolument convergente d’après la majoration
    
N N
   N
 N
c n  ≤ ap  · bq  ≤ ap · b q  =  ap    bq   ≤ AB
n=0 n=0 p+q =n 0≤p,q ≤N p=0 q=0
2. S ÉRIES NUM ÉRIQUES 217

+∞ 
avec A = ap  et B = +∞
p=0 q=0 b q . Pour montrer (*), remarquons maintenant que
  
2n
  n n    
∆n = ck −  ap  bq  = ap bq − ap bq = ap bq + ap b q
k=0 p=0 q=0 0≤p+q≤2n 0≤p,q ≤n p>n q>n
p+q≤2n p+q≤2n

ce qui entraı̂ne
  2n
 n−1
 n−1
 2n

∆ n ≤ a p · b q  + a p  · bq  = ap  b q  + ap  bq,
n<p≤2n n<q≤2n p=n+1 q=0 p=0 q=n+1
q<n p<n
 2n 2n
donc ∆n  ≤ B p=n+1 a p + A q=n+1 b q . Ainsi ∆n converge vers 0, d’où (*). 

Séries doubles. On désigne ainsi les séries de la forme (p,q)∈N2 up,q .
Théorème 10. Soit (up,q )(p,q)∈N2 une suite à double entrée, à valeurs dans un espace de
Banach. Alors les deux assertions suivantes sont équivalentes
   ∞ 
(i) Pour tout q ∈ N, p up,q est absolument convergente, et q p=0 up,q  converge.
  ∞  
(ii) Pour tout p ∈ N, q up,q est absolument convergente, et p q=0 up,q  converge.

Dans ces hypothèses, on a de plus


+∞
 +∞  +∞
 +∞ 
   
u p,q = up,q . (∗)
q=0 p=0 p=0 q=0
+∞
Démonstration. Montrons (i) =⇒ (ii). Notons Aq = p=0 u p,q . Pour p fixé, up,q  ≤ A q et
 
d’après (i), A q converge, donc q up,q  converge. Notons B p la somme de cette série. On a
P +∞
 P  +∞
   
∀P ∈ N, Bp = u p,q  ≤ A q.
p=0 q=0 p=0 q=0

Cette majoration étant indépendante de P , on en conclut que p B p converge, d’où (ii). On
montre de la même manière que (ii) =⇒ (i).   
Montrons maintenant (*). Notons an,q = np=0 up,q et a q = +∞ p=0 up,q . La série  a q converge
absolument car a q  ≤ Aq , donc elle converge. Soit ε > 0 et soit Q ∈ N tel que q>Q Aq < ε.

Notons Cn = 0≤p,q ≤n u p,q . Lorsque n > Q on a
+∞
 Q
 n
 +∞

aq − C n = (aq − an,q ) + (aq − an,q ) + aq . (∗∗)
q=0 q=0 q=Q+1 q=n+1

Pour q > Q, on a aq − a n,q =  p>n up,q  ≤ A q et comme a q ≤ A q, (**) entraı̂ne
 +∞   Q   Q 
    ∞
  
     
 aq − Cn ≤  (aq − a )
n,q  + A q ≤  (aq − a n,q  + ε.
)
     
q=0 q=0 q=Q+1 q=0

Les suites (an,q )n pour 0 ≤ q ≤ Q convergent vers aq donc il existe N0 ≥ Q tel que  Q q=0 (aq −
+∞
an,q ) < ε dès que n ≥ N0 . Ainsi, pour n ≥ N0 on a  q=0 aq − Cn  < 2ε, donc la suite (Cn )
 
converge vers +∞ q=0 a q. On montrerait de même que bp converge et que (C n) converge vers
+∞  +∞
p=0 b p (avec bp = q=0 up,q ). Ainsi, on peut bien intervertir les signes de sommation. 

Remarque 8. Les résultats précédents s’inscrivent naturellement dans le cadre de la théorie


des familles sommables, que nous présentons brièvement. Soit E un espace de Banach, I
un ensemble non vide. On note Pf (I) l’ensemble des parties finies de I. On considère une
famille (ui)i∈I d’éléments de E.
218 4. SUITES ET SÉRIES

(1) La famille (u i) i∈I est dite sommable s’il existe S ∈ E tel que
 
 
 
∀ε > 0, ∃J0 ∈ Pf (I ), ∀J ∈ P f (I ), J 0 ⊂ J,  ui − S < ε.
 i∈J 

La grandeur S est alors appelée somme de (ui )i∈I et notée i∈I ui .
(2) Elle est dite absolument sommablesi la famille (ui )i∈I est sommable. Ceci est
équivalent à dire que l’ensemble { i∈J ui , J ∈ Pf (I )} est borné.
Les familles sommables vérifient les propriétés suivantes :
(i) Si (ui)i∈I est sommable, alors l’ensemble {i ∈ I, u i = 0} est au plus dénombrable.

(ii) Une série n∈N un est absolument convergente si et seulement si la famille (u n)n∈N
est absolument sommable.
(iii) Une famille absolument sommable est sommable. La réciproque est vraie si E est de
dimension finie mais fausse si E est de dimension infinie.

(iv) Une série n∈N un est commutativement convergente si et seulement si la famille
(un )n∈N est sommable.
(v) (Associativité) Si (It) t∈T est une partition de I , alors (u i)i∈I est sommable si et
seulement si pour tout t ∈ T , (ui ) i∈It est sommable, de somme st, et la famille
(st) t∈T est sommable.
Les résultats précédents découlent naturellement des propriétés des familles sommables :
le théorème 8 est la conséquence de (ii), (iii) et (iv). Pour le théorème sur les produits de
Cauchy, la convergence absolue de cn découle de (v) appliqué à la famille (u p vq )(p,q)∈N2
avec la partition ({(p, q ), p + q = n})n∈N de N 2, et l’égalité des limites provient de (v)
appliqué à la famille (upvq )(p,q)∈N 2 et à la même partition. L’équivalence des assertions (i) et
(ii) du théorème d’interversion des limites est la conséquence de la propriété d’associativité
appliquée à la famille (up,q )(p,q)∈N 2, et la formule (*) est la conséquence de (iii) et de (v)
appliqué aux partitions (N × {n})n∈N et ({n} × N) n∈N de N2.
2.6. Exercices
Exercice 1. Soit (un ) la suite définie par
u n+1 n+a
u0 = 1, ∀n ∈ N, = ,
un n+b

où a et b sont deux nombres réels positifs fixés. Donner la nature de la série u n en
fonction de a et b et calculer sa somme lorsqu’elle converge.
Solution. Pour la convergence de la série, il suffit de remarquer que lorsque n → +∞,
un+1 1 1
= = ,
un 1 + (b − a)/(n + a) 1 + (b − a)/n + O(1/n2 )

donc d’après la règle de Raab-Duhamel, il existe λ > 0 tel que un ∼ λ/nb−a . Ainsi, un
converge si et seulement si b − a > 1.
Sommons la série lorsque b − a > 1. La relation (n + b)un+1 = (n + a)un entraı̂ne
n
 n
 n+1
 n

(k + b)uk+1 = (k + a)uk donc (k + b − 1)uk = (k + a)uk,
k=0 k=0 k=1 k=0
ce qui s’écrit  
n

(b − a − 1) uk + (n + b)u n+1 = (b − 1)u 0. (∗)
k=0
2. S ÉRIES NUM ÉRIQUES 219

Lorsque n → +∞, un ∼ λ/n b−a avec b − a > 1, donc (n + b)un+1 → 0, et en faisant n → +∞


dans (*), on obtient finalement
+∞
 +∞
 b−1
(b − a − 1) u n = (b − 1)u0 = (b − 1) donc un = .
n=0 n=0
b−a−1


Exercice 2. Soit (un ) une suite à termes positifs et décroissante. Si la série un
converge, montrer que un = o(1/n) lorsque n → +∞.
 
Solution. Soit ε > 0. La série un converge donc il existe N ∈ N tel que ∞
n=N +1 un < ε. On
en déduit, la suite (un ) étant décroissante, que


∀p > N, (p − N )up ≤ uN+1 + uN+2 + · · · + up ≤ u n < ε,
n=N +1

donc pour tout p > 2N , (p/2)u p ≤ (p − N )u p < ε. Finalement, nous avons 0 ≤ pu p ≤ 2ε pour
tout p > 2N . Ainsi, (nun ) tend vers 0, d’où le résultat.
Remarque. Ce résultat est la version discrète de celui de l’exercice 4 page 156.

 Exercice 3 (Formule de Stirling). On considère la suite (u n) n∈N∗ définie par



∗ nne−n n
∀n ∈ N , un = .
n!
Donner la nature de la série de terme général vn = log(u n+1/u n ). En déduire l’existence
d’un entier k > 0 tel que

n! ∼ k n n n e−n n → +∞. (∗)
Calculer la constante k en utilisant la formule de Wallis (voir l’exercice 1 page 130).

Solution. Estimons v n lorsque n → +∞ :


      
n + 1 n+1/2 −1 1 1
vn = log e = −1 + n + log 1 +
n 2 n
     
1 1 1 1 1
= −1 + n + − +O =O .
2 n 2n 2 n3 n2

Cette expression montre que vn converge. Comme on a v1 + · · · + v n = log un+1 − log u1
pour tout n, la suite (log un ) converge. En notant λ sa limite, on voit que (un) converge vers
k = eλ > 0, d’où l’équivalent (*).
Il nous reste à calculer la constante k. Comme indiqué dans l’énoncé, nous allons utiliser la
formule de Wallis qui est
 2
1 2p(2p − 2) · · · 2
lim = π.
p→+∞ p (2p − 1)(2p − 3) · · · 1

Par ailleurs, en utilisant l’équivalent (*) on a lorsque p → +∞


 2  2
1 2p(2p − 2) · · · 2 1 22p(p!)2 24p k 4p4p+2 e−4p k2
= ∼ = ,
p (2p − 1)(2p − 3) · · · 1 p (2p)! p k2 (2p)4p+1 e−4p 2

donc π = k 2 /2 d’après la formule de Wallis, d’où k = 2π . Finalement, le résultat obtenu est

n! ∼ 2πn nn e−n n → +∞.
220 4. SUITES ET SÉRIES


Remarque. Il faut connaı̂tre ce résultat et savoir le prouver. A partir de la série vn, en
procédant comme on l’a fait page 211 pour donner un développement asymptotique des
nombres harmoniques, il est possible de calculer un développement asymptotique de n!.
Une version continue de la formule de Stirling est traitée dans l’exemple 2 page 166.

Exercice 4. a) Montrer l’égalité


+∞
 +∞
 1
(ζ (k ) − 1) = 1, où ζ (k) = k
.
k=2 n=1
n

b) Montrer l’égalité
+∞
 ζ (k ) − 1 1 1
= 1 − γ, où γ = lim 1 + + · · · + − log n.
k=2
k n→+∞ 2 n

Solution. a) Ils’agit d’un exercice d’interversion de sommation. Pour tout k ≥ 2, la série à


termes positifs n≥2 1/nk est convergente, et sa somme ζ (k) − 1 vérifie

 1 +∞  +∞
1 1 dt 1 1 1
ζ (k ) − 1 = k + k
≤ k+ = k+ ≤ k−1 .
2 n=3
n 2 2 tk 2 (k + 1)2k 2

La série à termes positifs k≥2 (ζ (k ) − 1) est donc convergente. Les séries en présence étant
convergentes et à termes positifs, elles sont absolument convergentes donc on peut donc appliquer
le théorème d’interversion de sommation (voir le théorème 10 page 217) qui entraı̂ne
 +∞  +∞  +∞  +∞
+∞
 +∞
  1   1  1  1
+∞
1

(ζ (k ) − 1) = = = = −
n=2
nk n=2
nk n=2
n(n − 1) n=2 n − 1 n
k=2 k=2 k=2

d’où le résultat.
n
b) Remarquons que si Un = ( k=1 1/k ) − log n, on a
  +∞
 1
1 1 1
∀n ≥ 2, δ n = Un − U n−1 = − log n + log(n − 1) = + log 1 − =−
n n n knk
k=2

où le dernier terme est obtenu grâce


 au développement de log(1 − 1/n) en série entière. Ceci
k
assure la convergence de la série k≥2 1/(kn ). Par ailleurs, l’égalité

N
 N

∀N > 2, δn = (Un − Un−1 ) = UN − U 1 = UN − 1,
n=2 n=2

associée à la convergence de (Un ) vers la constante d’Euler γ (c’est classique, voir


 par exemple la
page 211 sur les sommes harmoniques), prouve que la série à termes négatifs n≥2 δ n converge,
et sa somme est γ − 1. On peut appliquer le théorème d’interversion des limites (les séries sont
toutes à termes négatifs et convergentes, donc absolument convergentes) qui entraı̂ne
+∞ +∞
  +∞  +∞  +∞
  +∞  1   1  ζ (k ) − 1
1−γ = − δn = = =
n=2 n=2
knk n=2
knk k
k=2 k=2 k=2

d’où le résultat.
2. S ÉRIES NUM ÉRIQUES 221

Exercice 5. Soit f : R + → R+∗ une fonction de classe C 1 vérifiant


f (x)
lim = −∞.
x→+∞ f (x)

Montrer que la série f (n) converge et donner un équivalent, lorsque n → +∞, de
+∞

Rn = f (k ).
k=n

Solution. Soit A > 0. Il existe N > 0 tel que pour tout x ≥ N , f  (x)/f (x) ≤ −A. Ainsi, si on
se donne un entier n ≥ N , on peut écrire
 n+p   n+p
f (n + p) f (x)
∀p ≥ n, log = dx ≤ −A dx = −pA,
f (n) n f (x) n

de sorte que f (n + p) ≤f (n)e−pA pour tout p ∈ N ∗. La fonction f étant positive, ceci assure la
convergence de la série f (n). De plus, notre majoration entraı̂ne
+∞
 +∞
 e−A
∀n ≥ N, 0 ≤ Rn+1 = f (n + p) ≤ f (n) e −pA = f (n).
p=1 p=1
1 − e−A

Comme on peut prendre A aussi grand que l’on veut, nous avons en fait montré que
∀ε > 0, ∃N > 0, ∀n ≥ N, 0 ≤ Rn+1 ≤ εf (n).
Autrement dit, Rn+1 = o(f (n)), donc Rn = f (n) + Rn+1 ∼ f (n) lorsque n → +∞.
 2
Remarque. Un exemple d’application de ce résultat est la convergence de
 +∞ e−n et le
2 2
fait que p=n e−p ∼ e−n .


Exercice 6. Soit n∈N un une série à termes > 0.

1/ a) Si un diverge, discuter en fonction du paramètre α > 0 la nature de la série
 un n

, où Sn = uk.
Sαn k=0

b) On suppose u n = o(Sn ) lorsque n → +∞. Exprimer  en fonction de S n un équivalent


α
des sommes partielles (resp. des restes) de la série un /Sn lorsqu’elle diverge (resp.
lorsqu’elle converge).

2/ a) Si un converge, discuter en fonction du paramètre α > 0 la nature de la série
 un +∞

, où Rn = uk .
R αn k=n

b) On suppose un = o(Rn ) lorsque n → +∞. Exprimer  en fonction de R n un équivalent


α
des sommes partielles (resp. des restes) de la série un /Rn lorsqu’elle diverge (resp.
lorsqu’elle converge).

Solution. 1/ a) Ceci dépend de la position de α par rapport à 1, comme on le voit dans le cas
où un = 1 pour tout n (dans ce cas, on a affaire aux séries de Riemann).
— Si α > 1, on écrit
 Sn  n n  Sk  Sn
∗ un S n − S n−1 dt uk dt dt
∀n ∈ N , = ≤ donc α ≤ = ,
Snα Snα Sn−1 t
α Sk Sk−1 t
α
S0 t
α
k=1 k=1
222 4. SUITES ET SÉRIES

et
 comme α > 1, on en conclut que les sommes partielles de la série à termes positifs
un /Snα sont majorées. Si α > 1, la série un/S αn est donc convergente.

— Si α ≤ 1, nous allons montrer que la série diverge. Par hypothèse, u n diverge, donc il
existe N ∈ N tel que Sn ≥ 1 pour tout n ≥ N . On écrit ensuite
q q
un un up+1 + · · · + uq S q − Sp Sp
∀p ≥ N, ∀q > p, α
≥ ≥ = = 1 − . (∗)
n=p+1
Sn n=p+1
Sn S q S q Sq

Pour tout p ≥ N , il existe q > p tel que Sq ≥ 2Sp . On en déduit avec (*) que
q
un 1
∀p ≥ N, ∃q > p, α
≥ .
n=p+1
Sn 2

Le critère deCauchy n’est donc pas vérifié pour la série un /Snα , elle diverge donc.
α
En résumé, la série un /S n converge si et seulement si α > 1.
b) On va utiliser une comparaison série-intégrale. On écrit
 Sn
∗ un S n − S n−1 dt Sn − S n−1 un
∀n ∈ N , α
= α
≤ α
≤ α = α ,
Sn Sn Sn−1 t S n−1 S n−1
et comme un = o(S n), on a S n−1 = Sn − un ∼ Sn lorsque n → +∞, de sorte que notre
encadrement entraı̂ne
 Sn
un dt
α
∼ α
. (∗∗)
Sn Sn−1 t
(Noter au passage
 que dans le cas ou un = o(Sn ), cet équivalent est un autre moyen de parvenir
au fait que un /S αn converge si et seulement si α > 1.) Supposons α ≤ 1. D’après (**) et le
théorème 5 page 210, on a
 n n   Sn
uk  Sk dt dt
α
∼ α
= α
.
Sk Sk−1 t S0 t
k=1 k=1
On en déduit 
n
 uk Sn1−α /(1 − α) si 0 < α < 1
∼ .
Sα log Sn si α = 1
k=0 k

Si maintenant α > 1, la série un /S αn converge et d’après (**) et le théorème 5, on a
+∞
 un   Sk dt  +∞ dt
+∞
1
∼ = = .
Sαn
k=n+1 k=n+1Sk−1 tα
Sn tα
(α − 1) Snα−1

2/ a) Comme précédemment, tout dépend de la position de α par rapport à 1.


— Si α < 1, on écrit pour tout n
 Rn n  R0  R0
un Rn − Rn+1 dt uk dt dt
α
= α
≤ α
donc α ≤ α

Rn Rn R n+1 t Rk Rn+1 t 0 tα
k=0

(le dernière intégrale converge bien car α < 1). On en déduit que les sommes partielles
de la série étudiée sont majorées, la série converge donc.
— Si α ≥ 1, on commence par fixer un entier N ∈ N tel que R n ≤ 1 pour tout n ≥ N , puis
on écrit
q−1
  uk q−1
uk up + · · · + uq−1 Rp − Rq Rq
∀p, q ≥ N, (q > p), α ≥ ≥ = = 1− .
Rk Rk Rp Rp Rq
k=p k=p

L’entier p ≥ N étant fixé, il existe q > p tel que Rq < Rp /2, et la dernière expression
montre alors que notre série ne satisfait pas le critère de Cauchy. Elle diverge donc si
α ≥ 1.
2. S ÉRIES NUM ÉRIQUES 223


Finalement, nous avons montré que un/Rαn converge si et seulement si α < 1.
b) En procédant comme dans la solution de la question 1/ b), on montre que sous l’hypothèse
un = o(Rn),
 Rn+1
un dt
α
∼ .
Rn Rn tα
En poursuivant le raisonnement comme nous l’avions fait plus haut, on en déduit
n
 +∞
 uk log(1/Rn) si α = 1,  un R1−α
n
∼  
α−1 −1
et pour α < 1 ∼ .
R αk (α − 1) R n si α > 1 , R α
n 1 − α
k=0 k=n+1


Exercice 7. a) Discuter en fonction du paramètre α > 0 la nature de la série n∈N∗ un
où
(−1)n−1
∀n ∈ N ∗ , un = α .
n + (−1) n

b) Discuter en fonction des réels θ, ϕ la nature de la série n≥2 un où
eniθ
∀n ≥ 2, un = √ .
n + eniϕ

c) Plus généralement, discuter en fonction des paramètres θ, ϕ ∈ R et α > 0 la nature de


la série n≥2 u n où
eniθ
∀n ≥ 2, un = α .
n + eniϕ

Solution. a) On a affaire à une série alternée. Attention à ne pas commettre l’erreur d’appliquer
directement le théorème 6 page 214, sous prétexte que la suite (nα + (−1)n) est “pratiquement”
décroissante.
Une bonne manière de procéder est de calculer un développement asymptotique de (un) :
 
(−1) n−1 1 (−1) n−1 1 1
un = = + 2α + o = vn + wn ,
nα 1 + (−1)n n−α nα n n2α
(−1)n−1 1
vn = α
, w n ∼ 2α .
n n

La série
 vn converge d’après le théorème 6 page 214, et comme u n = v n + wn, on en déduit
que  un et w n ont même nature. Comme wn ∼ n−2α (et que wn ∈ R), on voit finalement
que un converge si et seulement si α > 1/2.
b) Comme précédemment, le plus sûr est de calculer un développement asymptotique de (u n ) :
   
eniθ 1 1 eniθ e ni(θ+ϕ)
un = √ √ = v n + w n + O , v n = √ , w n = − .
n 1 + eniϕ / n n3/2 n n
  
Ceci montre que la série u n − (vn + w n) converge, donc un est de même nature que (vn +
wn). A ce stade, on traite plusieurs cas.  
(i) Si θ ∈ 2π Z et θ + ϕ ∈ 2π Z, alors chacune des séries v n et wn converge (c’est
 la plus
classique conséquence
 de la règle d’Abel, voir l’exemple 3 page 215), donc (vn + w n )
converge, donc un converge. √ 
(ii) Si θ ∈ 2π Z et  θ + ϕ ∈ 2π Z, on a v n + w n ∼ 1/ n, et comme vn + wn ∈ R, (vn + wn )
diverge, donc u n diverge.
(iii)Si l’un et
 l’un seulement des réels θ, θ + ϕ est un multiple de 2π, alors parmi les séries
vn et wn , l’une est divergente et l’autre convergente
 (ce dernier pointest toujours
justifié par la règle d’Abel). On en déduit que (v n + w n) diverge, donc un diverge.
224 4. SUITES ET SÉRIES


Finalement, nous avons montré que un converge si et seulement si θ ∈ 2πZ et θ + ϕ ∈ 2πZ.
c) On généralise la méthode précédente. L’idée est de calculer un développement asymptotique
de (un ) jusqu’à un terme d’erreur de la forme O (1/na ) avec a > 1.
Si α > 1, la série converge absolument donc converge, sinon 0 < α ≤ 1 et on note p = [1/α] −1
([1/α] est la partie entière de 1/α), de sorte que (p + 1)α ≤ 1 < (p + 2)α. On écrit
 
e niθ 1 1 ke
ni(θ+kϕ)
un = α = u n,0 + u n,1 + · · · + u n,p + O , un,k = (−1) .
n 1 + e niϕn −α n(p+2)α n(k+1)α
Comme (p + 2)α > 1, ceci montre que
 
la série u n a même nature que la série (un,0 + · · · + un,p). (∗)
n n
Deux cas se présentent : 
(i) Si pour tout k ∈ {0, 1, . . . , p}, θ + kϕ ∈ 2πZ, alors chacune des séries n u n,k converge
(pour 0 ≤ k ≤ p) d’après la règle d’Abel, donc un converge d’après le principe (*).
(ii) Sinon, il existe un plus
petit entier k tel que θ + kϕ ∈ 2π Z. Comme θ + ϕ ∈ 2πZ pour
0
≤  < k, les séries n un, (0≤  < k) convergent, et on en déduit d’après (*) que
n u n est de même nature que n (un,k + · · · + un,p ). Notons vn = un,k + · · · + u n,p. On
a  
(−1)k 1
u n,k = (k+1)α et un,k+1 + · · · + un,p = O ,
n n (k+2)α

donc vn ∼ (−1)k /n (k+1)α . A ce stade, on ne peut conclure directement que vn diverge
car vn est un nombre complexe. Pour s’en tirer, on va considérer la partie réelle de vn ,
k (k+1)α
en posant xn = (vn ). Elle vérifie  aussi xn ∼ (−1) /n  et comme xn est de signe
constant et que (k + 1)α ≤ 1, x n diverge. Donc vn diverge (une suite complexe

converge si et seulement si ses parties réelles et imaginaires convergent), donc un
diverge. 
Finalement, nous avons montré que un converge si et seulement si
   
1
(α > 1) ou 0 < α ≤ 1 et ∀k ∈ N, k ≤ − 1, θ + kϕ ∈ 2πZ .
α


Exercice 8. Soit un une série à termes positifs dont le terme général tend vers 0. On
note Sn la sommepartielle d’indice n de cette série. Montrer que si la suite (S n − nu n)
est bornée, alors un converge. La réciproque est-elle vraie ?

Solution. Raisonnons par l’absurde en supposant la série u n divergente. Alors Sn → +∞
donc l’estimation Sn − nu n = O(1) entraı̂ne
 
un 1 1 1
nun = Sn + O(1) donc = +O ∼ . (∗)
Sn n nS n n
à ce stade, nous allons utiliser une technique classique : on écrit
   
Sn un un 1
log = − log 1 − ∼ ∼ .
S n−1 Sn Sn n

On en déduit que la série log(Sn /S n−1) diverge, et on peut sommer terme à terme ces
équivalents (voir le théorème 5 page 210), ce qui donne
n    n
Sk 1
log S n − log S0 = log ∼ ∼ log n
S k−1 k
k=1 k=1

(ce dernier équivalent est hyper-classique, voir par exemple page 211), donc log Sn ∼ log
√ n. En
particulier, il existe N ∈ N tel que log(S n) ≥ (log n)/2 pour tout n ≥ N , et donc S n ≥ n pour
2. S ÉRIES NUM ÉRIQUES 225

tout n ≥ N . On injecte cette minoration dans (*), ce qui donne


   
un 1 1 1 1
= +O = +O .
Sn n nSn n n3/2
Nous possédons maintenant plus d’information que dans (*). Nous allons en tirer parti pour
calculer un développement asymptotique à deux termes de log S n. On a
     2  
1 S n−1 1 un 1 un un 1
+ log = + log 1 − = − +O =O ,
n Sn n Sn n Sn Sn2 n3/2

donc la série 1/n + log(S n−1/S n ) converge. En notant λ sa somme, on a
n   
1 1 1 Sk−1
1 + + · · · + + log S 0 − log S n = + log = λ + o(1),
2 n k Sk
k=1
donc
1 1
log Sn = 1 + + · · · + + log S 0 − λ + o(1) = log n + γ + log S 0 − λ + o(1),
2 n
où γ désigne la constante d’Euler. En passant aux exponentielles, on en déduit
Sn ∼ Kn, K = eγ +log S0 −λ > 0.
Comme un ∼ Sn /n d’après (*), on en déduit que la suite (un ) converge vers la constante non
nulle K, ce qui est contraire aux hypothèses. La série un converge donc.
La réciproque est fausse. Par exemple, la suite (u n) définie par

1/n2 si n ∈ {2k , k ∈ N}
un = √
1/ n si n ∈ {2k , k ∈ N}
est convergente mais (Sn − nun) n’est pas bornée.

 
Exercice
 9. Soit un une série à termes positifs. Comparer la nature des séries un
et vn , où
1
∀n ∈ N∗ , vn = .
1 + n2 un
 
Solution. Si un diverge, on ne peut rien conclure quant à la nature de vn . Par
 exemple

— si un = 1/n pour tout n ∈ N , u n diverge et vn = 1/(1+ n) ∼ 1/n, donc vn diverge ;
— si un = 1 pour tout n ∈ N ∗ , un diverge
 et vn = 1/(1 + n2) ∼ 1/n2 , donc vn converge.
Si un converge, nous montrons que vn diverge. Deux cas se présentent
 :
(i) si (n2 u n) ne tend vers pas +∞, alors (vn ) ne tend pas vers 0 donc vn diverge ;

(ii) si (n u n ) tend vers +∞, alors vn ∼ 1/(n u n ), donc (unv n ) ∼ 1/n, et donc (unvn )1/2
2 2 1/2

diverge. L’inégalité de Schwarz,


n 2  n  n 
√  
∀n ∈ N ∗, un vn ≤ uk vk ,
k=1 k=1 k=1
 
montre alors que les sommes partielles de vn divergent vers +∞, donc vn diverge.

Exercice 10. Pour tout k ∈ N on note log(k) = log ◦ · · · ◦ log le logarithme itéré k fois,
et on note nk ∈ N un entier tel que log (k) n est défini et ≥ 1 pour tout n ≥ n k .
a) Soit k ∈ N∗ et β > 0. Discuter de la convergence de la série
 1
n≥n k
n × log n × · · · × log(k−1) n × (log (k) n) β
226 4. SUITES ET SÉRIES

en fonction de β , et donner un équivalent de ses sommes partielles lorsqu’elle diverge.


b) Pour tout n ∈  N ∗, on note k n le plus grand entier k tel que log(k) n ≥ 1. Donner la
nature de la série n≥1 un où
1
un =
n × log n × log log n × · · · × log (kn) n
et déterminer un équivalent des sommes partielles de cette dernière si elle diverge.
Solution. a) Il suffit d’appliquer la proposition 3 sur la comparaison série-intégrale, page 212,
à la fonction décroissante fk,β : [nk , +∞[ → R, x → 1/(x × · · · × log(k−1) x × (log (k) x) β).
Traitons d’abord le cas β = 1. On montre facilement par récurrence sur k qu’une primitive de
fk,1 est log(k+1) , on en déduit que les sommes partielles de la série divergent car elles vérifient
n n
fk,1 (p) = fk,1 (x) dx + O(1) = log(k+1) n + O (1).
p=nk nk

Lorsque β = 1, une primitive de la fonction fk,β est donnée par (1 − β )−1 (log (k) x)1−β , donc
n
 n
(log(k) n)1−β
fk,β (p) = fk,β (x) dx + O(1) = + O (1).
p=n nk 1−β
k

On en déduit que si β < 1, la série diverge et ses sommes partielles sont équivalentes à
(log(k) n)1−β /(1 − β ). Si β > 1 l’estimation précédente montre que les sommes partielles sont
bornées, et comme la série est à termes positifs elle converge (notons que dans ce cas, on mon-
trerait facilement que les restes de la série sont équivalents à (β − 1)−1 1/(log(k) n) β−1).
b) Définissons la suite (ek ) par e0 = 1 et e k+1 = exp(ek ). Pour tout k ∈ N, on considère
la fonction continue décroissante gk : [ek , ek+1] → R, x → 1/(x × log x × · · · × log (k) x),
puis on considère la fonction g : [1, +∞[→ R définie par g(x) = gk (x) sur chaque intervalle
x ∈ [ek , ek+1[ . L’identité log (k) (ek ) = 1 entraı̂ne gk−1 (ek ) = gk (ek ) donc g est continue, et
comme chaque gk est décroissante on en déduit que g est également décroissante. Par ailleurs,
on a n ∈ [ek n , e kn+1 [ donc un = g (n). On peut donc appliquer le résultat la comparaison série-
intégrale, qui entraı̂ne
n  n k
n −1

up = g (x) dx + O (1) = I j + R n + O (1)


p=1 1 j=0
avec  ej+1  n
Ij = gj (x) dx, Rn = g kn (x) dx.
ej ekn
(j +1)
Nous avons vu plus haut qu’une primitive de gj est log , donc I j = log (j +1) e j+1 − log(j +1) ej ,
et par récurrence sur j on en déduit facilemet Ij = 1 (on peut aussi procéder à partir du
changement de variable t =ex dans l’intégrale I j qui montre que I j = Ij−1). On a 0 ≤ R n ≤
I k n = 1, on en déduit donc np=1 up = kn + O(1). Ainsi la série diverge et les sommes partielles
sont equivalentes à k n .

n
Exercice 11. a) Soit (un ) la suite définie par u n = k=1 log k/k . Montrer l’existence
2
d’une constante C ∈ R telle que un = (log n)/2 + C + o(1).

b) Démontrer la convergence de la série n≥1 (−1)n log n/n et utiliser le résultat de la
question précédente pour calculer la somme de cette série.
Solution. a) Le plus simple est d’appliquer la proposition 3 sur la comparaison série-intégrale,
page 212. Ici on a un = f (0) + f (1) + · · · + f (n − 1) avec f (x) = log(x + 1)/(x + 1). La fonction
f est dérivable et f (x) = (1 − log(x + 1))/(x + 1) 2 donc f est décroissante à partir d’un certain
2. S ÉRIES NUM ÉRIQUES 227

rang. Comme indiquédans la remarque 4 page 212, le résultat de la proposition 3 reste vérifié,
on conclut que un − 0n f (x) dx = un − 12 log2 (n + 1) = un − 12 log2 n + o(1) converge, d’où le
résultat.
On aurait pu aussi procéder comme on l’a fait à la page 211 pour les sommes harmoniques :
pour montrer la convergence de vn = un− (log2 n)/2, on calcule l’estimation v n − v n−1 =
O (log n/n2 ), entraı̂nant la convergence de (v n − v n−1) donc celle de la suite (v n).
 
b) Notons an = 2n k
k=1 (−1) log k/k . Pour montrer la convergence de
n
n≥1 (−1) log n/n il suffit
de montrer celle de (an ) car le terme général de la série converge vers 0, et de plus la somme de
la série égale la limite de (an). On récrit a n sous la forme
n
 2n

log 2k log k 1
an = 2 − = H n log 2 + u n − u2n , avec Hn = 1 + · · · + .
2k k n
k=1 k=1

En utilisant l’asymptotique des sommes harmoniques H n = log n + γ + o(1) (où γ est la constante
d’Euler, voir la page 211), et celle de (un ), on en déduit
log2 n − log 2 (2n) log2 2
an = log 2 log n + γ log 2 + + o(1) = γ log 2 − + o(1).
2 2

La série n≥1 (−1) n log n/n converge donc et sa somme est égale à (γ − 12 log 2) log 2.

 E
 xercice 12. 1/ Soit f : ]0, +∞[→ C une application de classe C 1 telle que l’intégrale
+∞
|f (t)| dt converge. n
1 
a) Montrer que la série n∈N∗ f (n) a même nature que la suite ( 1 f (t) dt) n∈N .
b) (Application.) Lorsque α > 1/2, donner la nature de la série

 ei n
α
. (∗)
n∈N∗
n

2/ En généralisant la technique précédente, donner la nature de la série (*) lorsque 0 <


α ≤ 1/2.

Solution. 1/ a) Nous allons montrer que la suite (s n ) définie par


 n+1 n

∀n ∈ N , sn = f (t) dt − f (p)
1 p=1

est une suite convergente, ce qui montrera le résultat.



 x Soit n ∈ N . En appliquant la formule de Taylor avec reste intégral à la fonction x →
n
f (t) dt à l’ordre 1, on a
 n+1  n+1   n+1   n+1
 
f (t) dt = f (n) +  
(n + 1 − t)f (t) dt donc f (n) − f (t) dt ≤ |f  (t)| dt.
n n n n
   n+1 
Ainsi, la série n∈N ∗ f (n) − n f (t) dt converge absolument donc converge, c’est-à-dire que
(s n) converge, d’où le résultat.

b) Nous allons appliquer le résultat précédent à la fonction f (t) = ei t /tα. On a
 √ √ 
 i ei t ei n  1 α
 
∀t > 0, |f (t)| =  α+1/2 − α 1+α  ≤ α+1/2 + α+1
 2t t  2t t
 +∞
et comme α > 1/2, cette expression montre que 1 |f (t)| dt converge. Ainsi, la série étudiée a
n   +∞
même nature que la suite 1 f (t) dt . Nous allons montrer que l’intégrale 1 f (t) dt converge,
ce qui entraı̂nera la convergence de la série. Le changement de variable v = u2 montre que cette
228 4. SUITES ET SÉRIES

 +∞
dernière intégrale a même nature que 1 eiv /v 2α−1 dv, donc convergente car 2α − 1 > 0 (voir
le début de la remarque 6 page 153).

i t α
2/
 +∞On pose f (t) = e /t . Ici, la technique précédente ne peut pas s’appliquer car l’intégrale
x

1 |f (t )| dt diverge. On généralise, en appliquant pour tout n ∈ N à la fonction x → n f (t) dt
la formule de Taylor avec reste intégral à l’ordre 2, ce qui donne
 n+1  n+1
f (n) (n + 1 − t) 2 
f (t) dt = f (n) + + f (t) dt,
n 2 n 2
ce qui entraı̂ne
   n+1
 n+1
f (n) 
∀n ∈ N , ∗  f (t) dt − f (n) − ≤ |f (t)| dt. (∗∗)
 2 
n n
Ici, on a
√   i√t √  
i t i t
 e 1 ie e 1
∀t > 0, f (t) = − α+1 − α + + α (α + 1) α+2 = O α+1 ,
4t 4 tα+3/2 t t
 +∞     n+1
donc l’intégrale
 1
|f (t)| dt converge, donc d’après (**) la série n
f (t) dt − f (n) −

f (n)/2 converge. Or
√ √
∗  ie i n ei n
∀n ∈ N , f (n) = − α ,
2nα+1/2 n 1+α

et en appliquant le résultat de la question précédente, on voit que f (n) converge.
 Finalement,
 n+1 n 
la série n
f (t) dt − f (n) converge, en particulier, la suite 1 f (t) dt et la série f (n)
sont de même nature. En écrivant
n  √n iv  iv √ n  √ n iv
e e 1 − 2α e
f (t) dt = 2α−1
dv = 2α− 1
− dv,
1 1 v iv 1 i 1 v 2α
 
on s’aperçoit que 1n f (t) dt diverge car α ≤ 1/2. Finalement, nous venons de montrer que la
série (*) diverge pour 0 < α ≤ 1/2.
Remarque. Cette technique de comparaison série-intégrale trouve une généralisation na-
turelle avec la formule d’Euler-Maclaurin (voir le sujet d’étude 3 page 321).

Exercice 13. Soit (un ) une suite vérifiant


 π
u0 ∈ 0, , ∀n ∈ N, un+1 = sin un.
2

a) Montrer que (un ) tend vers 0 puis donner un équivalent de (un ) lorsque n → +∞.
b) Donner un développement asymptotique à deux termes de (un ).

Solution. Il faut avoir fait au moins une fois dans sa vie ce type d’exercice.
a) L’intervalle ]0, π/ 2] est stable par la fonction sinus, donc un ∈ ]0, π/2] pour tout n. De plus,
on a sin x < x sur cet intervalle, donc la suite (u n) est strictement décroissante. Par ailleurs, elle
est minorée par 0, elle converge donc. Sa limite  vérifie sin() = , donc  = 0.
Donnons maintenant un équivalent de (un ). On utilise pour cela une jolie astuce (bravo si
vous l’avez trouvée), en écrivant
1 1 1 1 1 1
− = − 2 =  2 − 2
u2n+1 u 2n sin 2 un un 3 4
un − u n /6 + O (un) u n
   
1 1 1 u 2n 3 1 1
= 2 − 1 = + O (u n) = + O (un ) ∼
un 1 − u2n /3 + O (u3n) u2n 3 3 3
2. S ÉRIES NUM ÉRIQUES 229

(on a bien
 le droit de faire ces développements limités car u n → 0). Cet équivalent montre que
la série (1/u2n+1 − 1/u 2n ) diverge, et en sommant les équivalents, on obtient (on a le droit, voir
le théorème 5 page 210)
n−1
  n−1
1 1  1 1 1 n
− = − ∼ = ,
u2n u20 u2k+1 u2k 3 3
k=0 k=0

autrement dit 1/u2n ∼ n/3 donc un ∼ 3/n.
b) On procède comme plus haut, en cherchant cette fois ci un développement asymptotique à
deux termes de 1/u2n+1 − 1/u 2n . On a
 
u3n u5n 5 u2n u4n 4
un+1 = sin un = un − + + o(un ) = un 1 − + + o(un )
6 120 6 120
donc
 
1 1 u2n u4n 4 1 1 1 u2n 2 u 2n 1
= 2 1+ + + o( u n ) d’où − − = + o ( u n ) ∼ ∼
u 2n+1 un 3 15 u2n+1 u 2n 3 15 15 5n

le dernier équivalent provenant du fait que un ∼ 3/n. Comme précédemment, ces expressions
sont les termes généraux de séries qui divergent, et on peut sommer les équivalents, ce qui donne
n−1
  n−1
1 1 n  1 1 1  1 log n
2
− 2 − = 2 − 2 − ∼ ∼ ,
un u0 3 uk+1 uk 3 5k 5
k=0 k=1

et finalement
 −1  √  
n log n 3 3 3 log n log n
u2n = + + o(log n) d’où un = − √ +o √ .
3 5 n 10 n n n n

Remarque. On peut poursuivre le développement asymptotique en itérant la méthode.


– On peut de même donner un équivalent de toute suite récurrente (u n) qui tend vers 0 et
vérifie un+1 = f (un ) où f est une fonction vérifiant f (x) = x − Axα + o(xα) au voisinage
de 0 (A > 0, α > 1), en calculant un équivalent de u1−α 1−α
n+1 − u n lorsque n → ∞.

Exercice 14. Soit Θ une bijection


 de N∗ dans N∗ .
1/ a) Montrer que la série 1/(nΘ(n)) converge, et donner la majoration la meilleure
possible de la somme de cette série par une expression indépendante de Θ.
b) Montrer que la série Θ(n)/n2 diverge et donner la minoration la meilleure possible
des sommes partielles de cette série par une expression
 indépendante de Θ.
3
c) Que peut-on dire sur la nature de la série Θ(n)/n ?
2/ a) (Inégalité de réarrangement). Soient n ∈ N∗ et des réels (ai )1≤i≤n et (b i )1≤i≤n tels
que a1 ≤ . . . ≤ an et b1 ≤ . . . ≤ bn . Montrer que pour toute permutation σ ∈ Sn de
{1, . . . , n}, on a
a1 bσ(1) + · · · + an bσ(n) ≤ a 1b1 + · · · + anb n. (∗)

b) Soit (un )n∈N ∗ unesuite réelle positive décroissante telle que la série un diverge.
Montrer que la série un Θ(n)/n diverge.

Solution. 1/ a) C’est tout simple. On utilise l’inégalité de Schwarz qui entraı̂ne


N 2  N  N 
 1  1  1

∀N ∈ N , ≤ . (∗)
nΘ(n) n2 Θ(n)2
n=1 n=1 n=1
230 4. SUITES ET SÉRIES

 
La série 1/n2 converge, ainsi que 1/Θ(n) 2 d’après le théorème 8 page 216 (et d’ailleurs, les
sommes de ces deux séries sont égales). L’inégalité (*) montre donc que les sommes partielles
de notre série sont majorées, donc elle converge, et en faisant N → +∞ dans (*), on obtient
∞ 2  ∞  ∞   ∞ 
 1  1  1  1 2 ∞
1 ∞
1
≤ 2 2
= 2
donc ≤ 2
.
n=1
n Θ( n ) n=1
n n=1
Θ(n) n=1
n n=1
nΘ( n) n=1
n
Cette inégalité est la meilleure possible car il y a égalité lorsque Θ est l’identité.
On aurait pu traiter cette question à partir de l’inégalité ab ≤ (a2 + b 2)/2 appliquée à
a = 1/Θ(n) et b = 1/n.
b) La manière la plus immédiate de montrer qu’il y a divergence est certainement de nier le
critère de Cauchy, en écrivant
2N
 2N  N 
 Θ(n) 1  1  N +1 1

∀N ∈ N , ≥ Θ(n) ≥ n = ≥ .
n2 4N 2 4N 2 n=1 8N 8
n=N +1 n=N +1
Si maintenant on veut une minoration fine des sommes partielles de cette série, on peut utiliser
(encore !) l’inégalité de Schwarz qui entraı̂ne, pour tout N ∈ N∗
 N 2  N  2  N  N  N  N 
1  Θ(n) 1  Θ(n)  1  Θ(n) 1
=  ≤ 2
≤ 2
,
n=1
n n=1
n Θ(n) n=1
n n=1
Θ(n) n=1
n n=1
n
N
 N
Θ(n) 1
donc 2
≥ . Cette minoration est optimale car il y a égalité lorsque Θ est l’identité.
n n
n=1 n=1

c)
 On ne peut rien dire dans le cas général. Lorsque Θ est l’identité, la série Θ(n)/n 3 =
1/n 2 converge. Mais on peut construire Θ tel que cette série diverge. Par exemple, notons
A = {(2n)2 | n ∈ N ∗ } et B = N∗ A. On note b1 < b2 < b 3 < . . . les entiers de B rangés dans
l’ordre croissant, et on définit Θ par Θ(2n) = (2n)2 pour n ∈ N∗ et Θ(2n + 1) = b n pour n ∈ N.
Avec ce choix de Θ, on voit que Θ(n)/n3 diverge car
2N
 N
 N
Θ(n) Θ(2n) 1
∀N ∈ N ∗, ≥ = .
n=1
n3 n=1
(2n)3 n=1
2n

2/ a) C’est un classique. On procède par récurrence sur n. Pour n = 1 c’est évident. Supposons
le résultat vrai pour n − 1 et montrons l’inégalité (*) pour n. Si σ (n) = n c’est évident, car
dans ce cas la restriction de σ à {1, . . . , n − 1} est une permutation de {1, . . . , n − 1} et il
suffit d’appliquer l’hypothèse de récurrence. Sinon, on a k = σ−1 (n) < n et  = σ (n) < n, et
on définit la permutation σ  de {1, . . . , n − 1} par σ (i) = σ (i) si i = k, et σ (k) = . On a
(an − a k)(b n − b ) ≥ 0 donc a n b  + a k bn − a k b ≤ a n bn . On en déduit
n
 n−1
 n−1

ai b σ(i) = ai bσ  (i) + an b + ak b n − akb  ≤ a ib σ (i) + a nb n.
i=1 i=1 i=1
n−1  n−1
Le résultat pour n en découle, car d’apres l’hypothèse de récurrence i=1 ai bσ (i) ≤ i=1 a ibi .
b) Soit n ∈ N∗. On note b 1 < . . . < bn les valeurs de Θ(1), . . . , Θ(n) rangées dans l’ordre
croissant. Soit σ ∈ Sn tel que bσ(k ) = Θ(k) pour k = 1, . . . , n. Soit a k = −u k /k . Comme (uk )
est positive et décroissante, (u k /k) également donc on a a1 ≤ . . . ≤ an . On en déduit, d’après le
résultat de la question précédente et après changement de signe, que
n n

uk uk
bσ(k ) ≥ b
k k k
k=1 k=1
et comme bσ(k ) = Θ(k ) et bk ≥ k ceci implique
n
 n

uk
Θ(k) ≥ u k.
k
k=1 k=1
3. SUITES ET SÉRIES DE FONCTIONS 231


On en déduit le résultat car u n diverge.
Remarque. L’inégalité de réarrangement (parfois appelée inégalité de réordonnement)
est classique. Nous aurions pu l’utiliser pour obtenir les résultats des questions 1/a) et
1/b). Elle entraı̂ne de nombreuses
n inégalités, comme l’inégalité de Tchébycheff pour les
n n
sommes : i=1 xi yi ≥ ( i=1 x i )( i=1 y i)/n dès que x1 ≤ . . . ≤ xn et y 1 ≥ . . . ≥ yn . On
peut aussi utiliser l’inégalité de réarrangement pour démontrer l’inégalité de Schwarz, ou
pour démontrer que la moyenne géométrique est inférieure à la moyenne arithmétique.

3. Suites et séries de fonctions


La notion de convergence d’une suite ou d’une série de fonctions est cruciale en analyse.
Rares sont les problèmes d’analyse au concours dans lesquels n’apparaissent pas la notion
de convergence uniforme. En résumé, cette section est certainement la plus importante de
tout le cours d’analyse des classes préparatoires scientifiques.
3.1. Définitions
Il existe principalement deux types de convergence pour une suite de fonctions : la
convergence “point par point” (convergence simple) et la convergence “globale” (conver-
gence uniforme). La seconde est la plus importante car elle entraı̂ne comme on le verra
plus tard des propriétés intéressantes sur la fonction limite.
Définition 1. Soient X un ensemble, (E, d) un espace métrique, et (fn )n∈N une suite de
fonctions de X dans E.
— On dit que (f n) converge simplement (sur X) vers f : X → E si pour tout x ∈ X ,
la suite (fn (x))n∈N converge vers f (x), en d’autres termes si
 
∀ε > 0, ∀x ∈ X, ∃N ∈ N, ∀n ≥ N, d fn (x), f (x) < ε ;
— on dit que (f n) converge uniformément (sur X ) vers f : X → E si
 
∀ε > 0, ∃N ∈ N, ∀n ≥ N, ∀x ∈ X, d fn (x), f (x) < ε.
Remarque 1. — Il est important de saisir complètement la différence entre ces deux
notions. Étant donné ε > 0, la valeur de N pour laquelle d(fn (x), f (x)) < ε pour
tout n ≥ N dépend de x pour la convergence simple, et ne dépend pas de x pour
la convergence uniforme.
— La convergence uniforme entraı̂ne la convergence simple.
— Lorsque les fonctions (fn) sont à variable et valeurs réelles, la convergence uniforme
de (fn ) vers f équivaut à dire que pour tout ε > 0, il existe un rang à partir duquel
le graphe de fn est “coincé” entre le graphe de f − ε et f + ε.
Exemple 1. La suite de fonctions f n : [0, 1[ → R x → xn converge simplement vers
0 sur [0, 1[, mais pas uniformément car pour tout n ∈ N, il existe x ∈ [0, 1[ tel que
|f n(x) − 0| ≥ 1/2 (la définition n’est plus vérifiée dès que ε < 1/2).
Par contre, elle converge uniformément vers 0 sur [0, 1/2] car pour tout n et pour tout
x ∈ [0, 1/2], |f n (x) − 0| ≤ 2−n . Plus généralement, elle converge uniformément vers 0 sur
[0, a] pour tout a < 1.
Ce phénomène est courant. Les intervalles où il y a convergence uniforme sont souvent
différents de ceux où il y a convergence simple.
232 4. SUITES ET SÉRIES

Critère de Cauchy uniforme.


Proposition 1. Une suite de fonctions d’un ensemble X vers un espace métrique complet
(E, d) converge uniformément (sur X ) si et seulement si
 
∀ε > 0, ∃N ∈ N, ∀p ≥ N, ∀q ≥ N, ∀x ∈ X, d fp (x), fq (x) < ε.
Démonstration. La condition nécessaire est immédiate.
Voyons la condition suffisante. Pour tout x ∈ X , la suite (fn (x)) est de Cauchy dans l’espace
complet E donc elle converge, vers une limite que nous notons f (x). On définit ainsi une fonction
f : X → E.
Soit ε > 0, et soit N ∈ N tel que
 
∀p ≥ N, ∀q ≥ N, ∀x ∈ X d fp (x), fq (x) < ε. (∗)
Fixons un entier quelconque
 p ≥ N et un x ∈ X quelconque. En faisant q → +∞ dans (*), on
obtient
 d fp (x), f(x) ≤ ε, et comme p ≥ N et x ∈ X étaient arbitraires, on obtient finalement
d f p (x), f(x) ≤ ε pour tout p ≥ N et pour tout x ∈ X. Il y a donc convergence uniforme. 

Caractérisation de la convergence uniforme sur l’espace des fonctions. On


peut obtenir une caractérisation agréable de la convergence uniforme si on regarde une
suite de fonctions comme une suite de points d’un espace de fonctions.
Définition 2 (Norme de la convergence uniforme). Soit X un ensemble et E un e.v
normé. On note B(X, E ) l’e.v des applications bornées de X dans E , et pour tout f ∈
B(X, E), la norme
f ∞ = sup f (x)
x∈E
fait de B(X, E ) un e.v.n. Cette norme est appelée norme de la convergence uniforme. Une
suite (fn ) de B(X, E ), regardée comme une suite de fonctions de X dans E , converge
uniformément (sur X ) vers f ∈ B(X, E ) si et seulement si fn − f ∞ → 0 (i. e. si fn → f
dans l’e.v.n B(X, E )).
Remarque 2. Si (f n) est une suite de fonction de B(X, E ) et si E est un espace de Banach
(i. e. un e.v.n complet), la condition suffisante de la proposition 1 s’énonce comme suit : si
(fn) est une suite de Cauchy de B(X, E ), alors (f n ) converge vers une fonction f : X → E .
Cette fonction f est bornée car la suite (fn ) est bornée dans B (X, E) (c’est une suite de
Cauchy), donc il existe M > 0 tel que fn ∞ ≤ M pour tout n, donc
∀x ∈ X, ∀n ∈ N, fn(x) ≤ M.
En fixant x ∈ X (quelconque) et en faisant n → +∞, on en déduit que f (x) ≤ M , et
ceci est vrai pour tout x ∈ X donc f est bien bornée. Finalement, nous avons f ∈ B(X, E).
Autrement dit nous venons de montrer que si E est un espace de Banach, alors B(X, E )
est aussi un espace de Banach.
Séries de fonctions,  convergence normale. Comme pour les séries numériques,
une série de fonctions gn est définie comme étant la suite de fonctions (f n ) avec fn =
g0 + · · · + gn . On en rencontre beaucoup dans la pratique, et on dispose pour ces dernières
d’un nouveau type de convergence (convergence normale) qui n’est en fait qu’une condition
suffisante commode pour montrer la convergence uniforme.
Définition 3. Soient X un ensemble  et E un espace de Banach (i. e. un e.v.n complet).
On dit qu’une série de fonctions gn à termes dans B(X, E ) converge normalement si la
série g n ∞ converge.

Remarque 3. Il est équivalent de dire  que la série de fonctions g n converge normalement
s’il existe une série à termes positifs an convergente telle que
∀n ∈ N, ∀x ∈ X, g n(x) ≤ a n .
3. SUITES ET SÉRIES DE FONCTIONS 233

 n 2
Exemple 2. La série de fonctions g n définie par
 gn : [0, 1] → R x → x /n converge
2
normalement sur [0, 1] car gn  ∞ = 1/n donc g n ∞ converge.


 Théorème 1. Une série de fonctions gn à valeurs dans un espace de Banach qui
converge normalement sur un ensemble X converge uniformément sur X .
Démonstration. Il suffit de vérifier le critère de Cauchy uniforme (voir la proposition 1), ce qui
est immédiat car pour tout n, p ∈ N et pour tout x ∈ X ,
gn (x) + · · · + g n+p (x) ≤ gn (x) + · · · + gn+p (x) ≤ gn ∞ + · · · + gn+p ∞

et g n ∞ converge.

Une autre solution est de dire que gn est une série absolument convergente dans B (X, E )
qui est un espace de Banach (voir la remarque 2), donc elle converge dans B(X, E ), donc elle
converge uniformément. 

Remarque 4. De même qu’il existe des séries convergentes mais non absolument conver-
gentes, il existe des séries de fonctions uniformément convergentes qui ne sont pas nor-
malement convergentes (voir par exemple la question b) de l’exercice 2 page 236).
3.2. Propriétés des suites de fonctions
Continuité de la fonction limite.
 Théorème 2. Soient (E, d) et (F, δ) deux espaces métriques et (f n ) une suite de fonctions
de E dans F . Si (f n) converge uniformément sur E vers f : E → F et si toutes les
fonctions f n sont continues en x0 ∈ E , alors f est continue en x0 .
Démonstration. La démonstration est très classique, il faut savoir la refaire. Soit ε > 0. La suite
de fonctions (fn ) converge uniformément vers f , donc
 
∃n ∈ N, ∀x ∈ E, δ f n(x), f(x) < ε.
Or fn est continue en x0 , donc
 
∃α > 0, ∀x ∈ E, d(x, x0 ) < α, δ f n (x), fn (x 0) < ε.
On en déduit, pour tout x ∈ X vérifiant d(x, x0 ) < α,
       
δ f (x), f(x0) ≤ δ f (x), f n(x) + δ fn (x), fn (x0 ) + δ fn (x0 ), f(x 0 ) < ε + ε + ε = 3ε,
d’où le résultat. 

Intégration d’une suite de fonctions. Donnons maintenant une généralisation de la


proposition 3 page 125. La preuve est immédiate, elle est analogue à celle de ce dernier.
 Théorème 3. Soit (fn ) une suite de fonctions continues d’un segment [a, b] de R dans
un espace de Banach E , qui converge uniformément vers f sur [a, b]. Alors f est continue
et  
b b
f (t) dt = lim fn (t) dt.
a n→+∞ a
x
Plus généralement, la fonction F : [a, b] → E x → a f (t) dt est limite uniforme de la
suite de fonctions (Fn ) définie par
 x
∀n ∈ N, Fn : [a, b] → E x → fn (t) dt.
a
 
Remarque. L’égalité f = lim n→∞ fn est également vérifiée sous des hypothèses beau-
coup moins contraignantes dans le cadre du théorème de convergence dominée (voir le
théorème 3 page 151).
234 4. SUITES ET SÉRIES

– Le résultat se généralise si on suppose seulement les fn Riemann-intégrables. Dans ce


cas, la limite uniforme f est également Riemann-intégrable.
En combinant le théorème précédent avec le théorème 1, on en déduit facilement :

Corollaire 1 (Interversion des signes de sommation). Si g n est une série de fonctions
continues d’un segment [a, b] de R dans un espace de Banach E , qui converge normalement
sur [a, b], alors
 b  ∞
 +∞  b
 
g n(t) dt = gn (t) dt .
a n=0 n=0 a

Dérivabilité et dérivée de la fonction limite.


 Théorème 4. Soit (f n ) une suite de fonctions de classe C 1 d’un segment [a, b] de R dans
un espace de Banach E . On suppose que
(i) il existe x 0 ∈ [a, b] tel que la suite (fn (x0 )) converge ;
(ii) la suite de fonctions (fn ) converge uniformément sur [a, b] vers une fonction g .
Alors (f n) converge uniformément sur [a, b] vers une fonction f de classe C 1 et vérifiant
f  = g.

Démonstration. On applique le théorème 3 à la suite  x de fonctions (fn ), et on en conclut que
(f n − f n(a)) converge uniformément vers h : x → a g (t) dt sur [a, b]. En particulier, la suite
(f n(x0) − f n (a))n∈N converge, donc d’après (i), (fn (a)) converge, et nous notons  la limite
correspondante. On voit facilement que (f n) converge uniformément vers f : x → h(x) + .
Cette fonction f vérifie bien les propriétés voulues (en particulier, elle est C 1 car g est continue
comme limite uniforme de fonctions continues). 

Remarque 5. — Il n’est pas difficile d’en déduire que si (fn ) est une suite de fonctions
1
C de I dans E ( où I est un intervalle quelconque de R) qui vérifie les hypothèses
précédentes sur tout segment de I , alors les mêmes conclusions subsistent (sauf la
convergence uniforme de (fn )sur I tout entier).
— Pour une série de fonctions gn , les gn étant declasse C 1 sur [a, b], ce théorème
  suit : s’il existe x0 ∈ [a, b] tel que gn
s’énonce comme (x0) converge et si la série de
fonctions gn converge normalement sur [a, b],  alors g n converge
 ∞  normalement
1 ∞ 
sur [a, b] vers une fonction C sur [a, b] et on a ( n=0 g n) = n=0 gn .
— Le théorème 4 reste vrai lorsque les fonctions (fn ) sont seulement supposées déri-
vables (voir l’exercice 9 page 244).
En utilisant une récurrence fondée sur le théorème précédent, on en déduit le corollaire
suivant qui permet de montrer qu’une fonction limite est de classe C p .
Corollaire 2. Soit p ∈ N∗ et (fn) une suite de fonctions de classe C p d’un segment [a, b]
de R dans un espace de Banach E . On suppose que pour tout k ∈ {0, 1, . . . , p}, la suite
de fonctions (f(nk) )n converge uniformément vers une fonction gk sur [a, b]. Alors la limite
uniforme f = g0 de (f n ) est de classe C p et vérifie f (k) = g k pour tout k ∈ {0, 1, . . . , p}.
Remarque 6. Le corollaire précédent s’étend aisément aux séries de fonctions : si les gn
 
sont Cp et si g (nk) converge normalement sur [a, b] pour 0 ≤ k ≤ p, alors g = ∞ n=0 g n
p (k)
∞ (k)
est de classe C et g = n=0 gn pour 0 ≤ k ≤ p.
Exemple 3. Considérons une algèbre normée complète E, un élément u ∈ E , et la fonction
∞ n
t n
eu : R → E t → exp(tu) = u .
n=0
n!
3. SUITES ET SÉRIES DE FONCTIONS 235

Chaque terme de la série est de classe C ∞ et la série des dérivées p-ièmes est
 t n−p
u n.
n≥p
(n − p)!

Cette série converge normalement sur tout segment S = [a, b] de R (car t n−pu n /(n−p)! ≤
up(M u)n−p /(n − p) ! où M = max{|a|, |b|}). On en déduit que ue est de classe C ∞
sur S et que sa dérivée p-ième est e(up) = upeu . Ceci étant vrai pour tout segment S de R,
c’est vrai également sur R tout entier.
3.3. Complément : le théorème de Weierstrass
Weierstrass démontra le théorème suivant, qui frappa beaucoup ses contemporains.

Théorème 5 (Théorème de Weierstrass). Toute fonction continue f : [a, b] ⊂


R → C est limite uniforme sur [a, b] d’une suite de fonctions polynômes.
Ce résultat est prouvé dans l’exercice 8 page 242 (preuve par les polynômes de Bern-
stein) et par une autre méthode dans le problème 23 page 304 (preuve par la convolution).
Il est important, car il permet parfois de prouver des résultats sur les fonctions continues
en les montrant d’abord pour les fonctions polynômes, puis en concluant par un argument
de densité.
Il existe un théorème analogue sur les fonctions continues 2π -périodiques :
Théorème 6. Toute fonction continue et 2π -périodique de R dans C est limite uniforme
sur R d’une suite de polynômes trigonométriques.
Ce résultat est un avant-goût des séries de Fourier. C’est une conséquence du résultat
prouvé dans le problème 25 page 306.
Remarque 7. — Un autre résultat (théorème de Stone-Weierstrass) propose un cadre
général englobant ces deux théorèmes.
— Plus généralement, le théorème de Müntz (voir le problème 28 page 310) donne
une condition nécessaire et suffisante sur une suite réelle croissante (αn ) pour que
Vect(xαn ) n soit dense (au sens de la norme de la convergence uniforme) dans l’en-
semble des fonctions continues sur [0, 1].
3.4. Exercices
Exercice 1. a) Montrer que la suite de fonctions (fn ) définie par
 x n
+
fn : R → R, f n (x) = 1 − si x ∈ [0, n], f n(x) = 0 si x > n
n
converge uniformément sur R + vers la fonction f : x → e−x.
b) Montrer que la suite de fonctions (fn ) définie par
 z n
fn : C → C z → 1 +
n
converge uniformément sur tout compact de C vers f : z → e z .

Solution. a) Remarquons déjà qu’il y a convergence simple. Pour montrer la convergence uni-
forme, nous allons donner deux méthodes.
Première méthode. Nous allons utiliser une technique générale, qui consiste à montrer que
le maximum sur R+ de |f n(x) − f (x)| tend vers 0 en faisant une étude de fonctions. Fixons un
entier n > 1 et posons ϕ : [0, n] → R x → e−x − fn(x). On a, pour tout x ∈ [0, n[
 x n−1    x  
ϕ (x) = −e−x + 1 − = e−x exp (n − 1) log 1 − +x −1 .
n n
236 4. SUITES ET SÉRIES

Le signe de ϕ (x) est donc celui de ψ (x) = (n − 1) log(1 − x/n) + x. On a ψ  (x) = (1 − x)/(n − x),
on en déduit que ψ croı̂t sur [0, 1] et décroı̂t sur [1, n[. Comme ψ(0) = 0 et que ψ (x) → −∞
lorsque x → n− , on en déduit (faites un tableau de variation) l’existence de α ∈ ]1, n[ tel que
∀x ∈ [0, α], ψ(x) ≥ 0, ∀x ∈ [α, n[ , ψ(x) ≤ 0.
Comme ψ a le signe de ϕ, ϕ est croissante sur [0, α] et décroissante sur [α, n]. Comme ϕ(0) = 0
et ϕ(n) = e−n ≥ 0, on en déduit
∀x ∈ [0, n], 0 ≤ ϕ(x) ≤ ϕ(α) avec ϕ (α) = 0.
Il s’agit donc pour nous de majorer ϕ(α). En exploitant le renseignement ϕ (α) = 0, on a
 α n−1  α  n−1  α  α −α
1− = e−α donc ϕ(α) = e−α − 1 − 1− = e . (∗)
n n n n
Un rapide étude de x → xe−x montre que cette fonction atteint son maximum en x = 1,
donc est majorée par 1/e sur R +, de sorte que (*) entraı̂ne ϕ(α) ≤ 1/(ne). Sur [n, +∞[, on a
|fn (x) − f (x)| = |f (x)| ≤ |f (n)| = |fn (n) − f (n)| ≤ 1/(ne), donc finalement
1
∀x ∈ R+ , |fn (x) − f (x)| ≤ ,
ne
d’où le résultat.
– Seconde méthode. Considérons ε > 0, puis M > 0 tel que e−M < ε. Nous commençons par
approcher log fn par −x sur [0, M ]. D’après la formule de Taylor-Lagrange,
 
1 u2 1
∀u ∈ 0, , ∃θ ∈ ]0, 1[, log(1 − u) = −u + ,
2 2 (1 − θu) 2
donc | log(1 − u) + u| ≤ 2u 2 lorsque u ∈ [0, 1/2]. On en déduit
  x  2M 2
 
∀n ≥ 2M, ∀x ∈ [0, M], n log 1 − + x ≤ ,
n n
donc (log fn ) converge uniformément vers x → −x sur [0, M]. La fonction exponentielle étant 1-
lipschitzienne sur R− (immédiat par l’inégalité des accroissements finis), on en déduit en prenant
l’exponentielle que (fn ) converge uniformément vers f sur [0, M]. En particulier,
∃N ∈ N, ∀n ≥ N, ∀x ∈ [0, M ], |f n (x) − f (x)| < ε. (∗∗)
Or e−M< ε, donc |fn (x) − f (x)| ≤ e −x
≤ e−M< ε sur [M, +∞[, donc finalement l’inégalité
+
(**) est vraie sur R tout entier, d’où le résultat.
b) Ici, les techniques précédentes ne peuvent plus s’appliquer.
Soit C un compact de C et soit M > 0 tel que |z| ≤ M pour tout z ∈ C . L’inégalité
1 n n−1 n−k+1 1 1
∀n ∈ N ∗ , ∀k ≤ n, Cnk k = ··· ≤
n n n n k! k!
entraı̂ne
  n  

 z
 n
z  
  +∞
 zk  n k
Cn k
  1 Ck    zk 
∀z ∈ C, e − 1 + = − z  ≤  − n  |z | +
k  
n  k! nk   k! nk   k! 
k=0 k=0 k=0 k>n
 n    Mk  
1 Cnk k M M n
≤ − M + =e − 1+
k! nk k! n
k=0 k>n
n M
et comme (1 + M/n) → e lorsque n → +∞, on en déduit le résultat.

Exercice 2. Pour tout n ∈ N∗ , on définit l’application


x
u n : R+ → R x → 2 .
n + x2

a) Montrer que la série de fonctions un converge simplement sur R + vers une fonction
3. SUITES ET SÉRIES DE FONCTIONS 237

continue f mais que la convergence  n’est pas uniforme sur R +.


b) Montrer que la série de fonctions (−1)nun converge uniformément sur R + tout entier,

mais que la convergence n’est pas normale sur R+ .

Solution. a) La convergence simple est immédiate car pour tout x ∈ R + fixé, x/(n 2 + x 2)
2 2 2
converge (ceci car x/(n + x ) ∼ x/n lorsque n → +∞).
En revanche, il n’y a pas convergence uniforme sur R+. En effet, la minoration
2p
 2p
∗ x x px
∀x > 0, ∀p ∈ N , 2 2
≥ 2 2
= 2 (∗)
n=p+1
x +n n=p+1
x + (2p) x + 4p2

entraı̂ne
2p

∗ x 1
∀p ∈ N , ∃x > 0, ≥ (prendre x = p dans (*)),
n=p+1
x2 + n 2 5

autrement dit, la série de fonctions un ne vérifie pas le critère de Cauchy uniforme sur R+.
Ceci montre qu’il n’y a pas convergence uniforme sur R +.
Pour montrer la continuité de la limite simple f de un sur R + , il aurait été commode
que la convergence soit uniforme sur R + tout entier, mais ce n’est pas le cas. Pour contourner
le problème, on va montrer que f est continue sur [0, M] pour tout M > 0, ce qui entraı̂nera la
continuité de f sur R+ tout entier. Fixons donc un réel positif quelconque M . On a
M
∀n ∈ N ∗ , ∀x ∈ [0, M], |u n(x)| ≤ 2 ,
n
 2

et comme la série M/n converge, un converge normalement, donc uniformément, sur
[0, M]. Ainsi f , limite uniforme d’une suite de fonctions continues sur [0, M], est continue sur
[0, M]. D’où le résultat.

b) Si on fixe x ≥ 0, (−1) n un (x) est une série numérique alternée dont la valeur absolue du
terme général décroı̂t ; la série converge donc (on le savait déjà, car on a montré plus haut qu’elle
converge absolument), et de plus les restes sont majorés en valeur absolue par la valeur absolue
du premier terme qui les compose (voir le théorème 6 page 214), donc
+∞  
 x  x x2 + p2 1 1
∗  n
∀p ∈ N ,  (−1) 2  ≤ ≤ = ≤ .
 x +n 2  2
x +p 2 2
x +p 2 2
x +p 2 p
n=p

Cette majoration des restes est indépendante de x≥ 0, et elle montre que les restes tendent
uniformément vers 0 sur R+. La série de fonctions un converge donc uniformément sur R+ .
+ ∗
Il n’y a pas convergencenormale sur R tout entier, car pour tout n ∈ N , sup x≥0 un(x) ≥
un(n) = 1/(2n) et la série 1/(2n) diverge.
Remarque. Retenez la  méthode utilisée pour montrer la continuité de la limite simple
de la série de fonctions un : comme il n’y avait pas convergence uniforme sur R+ tout
entier, nous avons montré la convergence uniforme sur [0, M ] pour tout M > 0. Cette
technique est très classique. On procède aussi souvent ainsi pour montrer la dérivabilité
d’une suite de fonctions lorsqu’il n’y a pas convergence uniforme sur l’intervalle de départ
tout entier.

Exercice 3. Que dire d’une fonction f : R → R limite uniforme sur R d’une suite de
fonctions polynômes (Pn ) ?
Solution. Premier réflexe : f est continue. Mais il y a bien mieux, et nous allons montrer que f
est une fonction polynôme. Le critère de Cauchy uniforme entraı̂ne
∃N ∈ N, ∀n ≥ N, ∀x ∈ R, |Pn(x) − P N (x)| ≤ 1.
238 4. SUITES ET SÉRIES

Ainsi, pour tout n ≥ N , PN − P n est une fonction polynôme bornée sur R, donc constante.
Autrement dit, pour tout n ≥ N , il existe αn ∈ R tel que P n = PN + α n . La suite (Pn (0))
converge, donc la suite (αn)n≥N = (P n(0) − P N (0))n≥N aussi. Notons α la limite de (αn). On a
∀x ∈ R, f (x) = lim Pn(x) = lim PN (x) + αn = P N (x) + α,
n→+∞ n→+∞

donc f = PN + α est une fonction polynôme.

Exercice 4. On considère la suite de fonctions (f n ) définie par


 π
∀n ∈ N, fn : 0, → R x → cos n x · sin x.
2

a) Montrer que (fn ) converge uniformément vers la fonction nulle sur [0, π/2].
b) On considère la suite de fonctions (gn) définie par g n = (n + 1)f n . Montrer que sur
tout intervalle de la forme [δ, π/2] avec 0 < δ < π/2, (gn ) converge uniformément vers la
 π/2
fonction nulle, mais que pourtant, la suite ( 0 gn (t) dt)n ne tend pas vers 0.

Solution. a) On pourrait résoudre l’exercice en essayant de majorer directement le maximum


de |fn | sur [0, π/2] en effectuant une étude de fonction, mais nous allons donner une méthode
différente qui est plus générale et qui a son intérêt.
Analysons la situation. On a cosn x → 0 lorsque n → +∞ uniformément lorsque x est dans
[0, π/2] et n’est pas dans un voisinage de 0. Au voisinage de 0, la fonction sinus est petite. Pour
tirer parti de ces deux informations on procède comme suit.
Un nombre réel ε > 0 étant donné, on considère δ > 0 (et δ < π/2) tel que | sin x| < ε sur
[0, δ] (ceci est possible par continuité de la fonction sinus qui est nulle en 0). Sur le reste de
l’intervalle, on a  π
∀x ∈ δ, , ∀n ∈ N, |fn (x)| ≤ (cos δ )n,
2
et comme (cos δ )n tend vers 0 (car |cos δ | < 1), on en déduit
 π
∃N ∈ N, ∀n ≥ N, ∀x ∈ δ, , |f n (x)| < ε.
2
Comme | sin x| < ε sur [0, δ], on a aussi |fn (x)| < ε sur [0, δ] pour tout n, donc finalement
 π
∀n ≥ N, ∀x ∈ 0, , |fn(x)| < ε.
2
b) La convergence uniforme de (g n ) vers 0 sur [δ, π/2] est une conséquence de l’inégalité
 π
∀x ∈ δ, , ∀n ∈ N, |g n (x)| ≤ (n + 1) cosn x ≤ (n + 1) cosn δ
2
et du fait que (n + 1) cosn δ → 0 lorsque n → +∞ (ceci car |cos δ | < 1).
  π/2
Comme 0π/2 g n(x) dx = − cos n+1 x 0 = 1, il est clair que la suite des intégrales de gn ne
tend pas vers 0.
Remarque. La dernière question de l’exercice est un contre-exemple qui montre que le
théorème de convergence dominée (page 151) est faux lorsque la condition de domination
n’est pas satisfaite.

Exercice 5 (Théorèmes de Dini). a) Soit (fn ) une suite croissante de fonctions


réelles continues et définies sur un segment I = [a, b] de R. Si (f n) converge simplement
vers une fonction f continue sur I , montrer que la convergence est uniforme.
b) Soit (fn ) une suite de fonctions croissantes réelles, continues et définies sur un segment
3. SUITES ET SÉRIES DE FONCTIONS 239

I = [a, b] de R. Si (f n) converge simplement vers une fonction f continue sur I , montrer


que la convergence est uniforme.
Solution. a) Soit ε > 0. Pour tout n, considérons l’ensemble
Fn = {x ∈ I | f (x) ≥ f n(x) + ε}.
La suite (Fn ) est une suite décroissante de fermés de I (donc compacts). On a ∩n≥0 Fn = ∅
car pour tout x ∈ I , fn (x) converge vers f (x) donc il existe n tel que f (x) < ε + fn(x), ce qui
entraı̂ne x ∈ Fn . Une suite décroissante de compacts non vide est non vide, donc il existe N tel
que FN = ∅, autrement dit f < ε + fn sur I pour n ≥ N . Comme la suite (f n ) est croissante,
on a également fn ≤ f , donc finalement f n ≤ f < ε + f n pour n ≥ N d’où le résultat.
b) Tout d’abord, la fonction f est limite simple de fonctions croissantes, elle est donc croissante.
Donnons nous ε > 0. La fonction f est continue sur le compact I, donc d’après le théorème de
Heine
∃η > 0, ∀(x, x ) ∈ I 2 , |x − x | < η, |f (x) − f (x )| < ε.
On considère ensuite une subdivision a = x0 < x1 < · · · < xp = b de I de pas < η, c’est-à-dire
telle que xi+1 − x i < η pour tout i. Pour tout i ∈ {0, 1, . . . , p}, la suite (f n(x i )) tend vers f (x i),
on en déduit (les i étant en nombre fini)
∃N ∈ N, ∀n ≥ N, ∀i ∈ {0, . . . , p}, |f n(xi ) − f (xi )| < ε.
Ceci étant, considérons x ∈ I . Il existe i ∈ {0, . . . , p− 1} tel que x ∈ [xi , x i+1 ], et les fonctions
(f n)n∈N et f étant croissantes
∀n ∈ N, fn (x i ) ≤ f n(x) ≤ fn (xi+1 ) et f (xi ) ≤ f (x) ≤ f (xi+1 ),
donc
∀n ∈ N, f (xi ) − fn (xi+1) ≤ f (x) − f n(x) ≤ f (xi+1 ) − fn(xi ). ( ∗)
Or pour tout n ≥ N ,
|f (xi+1 ) − f n(xi )| ≤ |f (xi+1 ) − f (xi )| + |f (xi ) − fn (xi )| < 2ε,
de même |f (xi) − f n (xi+1)| < 2ε. On en déduit avec (*) que |f (x) − fn (x)| < 2ε, et ceci pour
tout n ≥ N et pour tout x ∈ I , d’où le résultat.

Exercice 6. a) Soit (fn) une suite de fonctions d’un segment I = [a, b] de R à valeurs
dans un R-e.v.n E . On suppose qu’il existe K > 0 tel que toutes les fonctions fn soient
K -lipschitziennes. Si (f n) converge simplement vers une fonction f : I → E , montrer que
la convergence est uniforme.
b) Soit (fn ) une suite de fonctions convexes de ]a, b[ dans R ( où ] a, b[ est un intervalle de
R) qui converge simplement vers une fonction f : ]a, b[ → R. Montrer que sur tout segment
[α, β ] ⊂ ]a, b[, la suite (fn ) converge uniformément vers f . La convergence est-elle uniforme
sur ]a, b[ tout entier ?

Solution. a) La technique ressemble assez à celles utilisées dans l’exercice précédent. Remarquons
tout d’abord que f , limite simple de fonctions K -lipschitziennes, est K -lipschitzienne.
Soit ε > 0 et soit a = x0 < x1 < · · · < xp = b une subdivision de [a, b] de pas < ε (i. e.
xi+1 − xi < ε pour tout i). Pour tout i ∈ {0, 1, . . . , p}, la suite (fn (xi )) converge vers f (xi ) donc
(les i sont en nombre finis)
∃N ∈ N, ∀n ≥ N, ∀i ∈ {0, 1, . . . , p}, f n(xi) − f (x i ) < ε.
Si maintenant on considère x ∈ I , il existe i tel que x ∈ [xi, xi+1 ] donc les fonctions en présence
étant toutes K-lipschitziennes, on a pour tout n ≥ N
f n(x) − f (x) ≤ fn (x) − f n (xi) + f n(xi) − f (xi ) + f (xi ) − f (x)
≤ K (x − x i) + ε + K (x − x i) ≤ (2K + 1)ε.
240 4. SUITES ET SÉRIES

Ceci étant vrai pour tout x ∈ I et pour tout n ≥ N , on en déduit le résultat.


b) L’idée est de se ramener à la question précédente. Soit I = [α, β] ⊂ ]a, b[. Fixons α , β  tels
que a < α < α < β < β  < b. Les suites
   
fn (α) − f n (α) fn(β ) − f n(β  )
et
α − α n∈N β − β n∈N
convergent (conséquence de la convergence simple de (f n )), elles sont donc bornées. Désignons
par K > 0 un majorant de la valeur absolue des termes de ces suites. Chaque fonction f n étant
convexe, on en déduit pour tout n
fn (α ) − fn (α) fn (x) − f (y ) f n(β ) − f n (β )
∀(x, y ) ∈ [α, β ] 2, −K ≤ ≤ ≤ ≤ K,
α − α x−y β − β
donc |fn (x) − f n (y )| ≤ K |x − y|. Ainsi, toutes les fonctions fn sont K-lipschitziennes sur [α, β ],
et on conclut en utilisant le résultat de la question précédente.
Il n’y a pas en général convergence uniforme sur ] a, b[ tout entier, comme le montre le contre-
exemple classique de la suite de fonctions convexes (fn) définies par f n : ]0, 1[→ R x → xn qui
converge simplement vers 0 sur ]0, 1[ mais pas uniformément.

Exercice 7 (Phénomène de Runge). Soit α > 0 et f α la fonction


1
fα : [−1, 1] → R x → 2 .
x + α2
Soit n ∈ N ∗. On considère les 2n points équirépartis dans [−1, 1] définis par a k = (2k +
1)/(2n) pour −n ≤ k ≤ n − 1. On note Pn le polynôme d’interpolation de Lagrange de
degré < 2n déterminé par les conditions Pn (ak) = f α (ak) pour −n ≤ k ≤ n − 1. On veut
montrer que la suite de fonctions polynômes (Pn) ne converge pas forcément uniformément
vers fα sur [−1, 1] (c’est le phénomène de Runge).
a) Montrer la formule
n−1

1 ω n (x)
fα (x) − P n(x) = 2 2
, avec ωn(x) = (x − ak ).
x + α ωn (αi)
k=−n

b) En déduire le premier terme du développement asymptotique de log |fα(1) − Pn(1)|


lorsque n → ∞.
c) Montrer que si α > 0 est suffisamment petit, la suite de fonctions (P n ) ne converge pas
uniformément vers fα .
Solution. a) Il est classique que le polynôme d’interpolation de Lagrange Pn existe et s’écrit
sous la forme (voir le tome Algèbre)
n−1
 fα (a k )ωn(x)
Pn (x) = . (∗)
(x − a k )ω n(a k )
k=−n
La clé est maintenant d’utiliser la décomposition en éléments simples de la fraction f α(x)/ω n (x).
Cette dernière n’a que des pôles simples, qui sont αi, −αi et les (ak ) −n≤k≤n−1 , on peut donc
écrire sa décomposition en éléments simples (voir le tome Algèbre) sous la forme
n−1

f α(x) fα(a k) 1 i 1 i
= 
+ − . (∗∗)
ωn(x) (x − ak )ωn(a k) ωn (−iα) 2α(x + iα) ωn(iα) 2α(x − iα)
k=−n
La relation de symétrie autour de zéro a−j = −a j−1 pour 1 ≤ j ≤ n donne
n
 n
 n
 n−1

ωn (x) = (x − a−j ) (x − aj−1 ) = (x + aj−1 )(x − a j−1 ) = (x2 − a2j )
j=1 j=1 j=1 j=0
3. SUITES ET SÉRIES DE FONCTIONS 241

donc la fonction polynôme ω n(x) est paire, ce qui fournit ωn (−iα) = ω n (iα). Avec (*) et (**),
on en déduit l’égalité fα (x)/ωn(x) = P n(x)/ωn (x) + 1/(x 2 + α 2)/ω n (iα), d’où le résultat.
b) L’expression de ω n (x) que nous avions obtenu précédemment donne
n−1
  n−1
 
ωn (1)  1 − a2j  1 − a 2j
n
= = (−1)
ωn (iα) j=0 −α2 − a2j j=0
α2 + a 2j

D’après le résultat obtenu à la question précédente, on a donc


n−1

1 1 − a 2j
log |f α (1) − Pn(1)| = log + log . (∗∗∗)
1 + α2 α2 + a2
j
j=0

Le dernier terme du membre droit est, au facteur 1/n près, une somme de Riemann de la fonction
2
ϕα (t) = log α1−t2 +t2 pour une subdivision de [0, 1] ; nous sommes ici en présence d’une intégrale
généralisée et on ne peut donc pas appliquer le théorème 7 page 128. On  1 va procéder comme à
l’exercice 5 page 156. Notons d’abord que l’intégrale généralisée I (α) = 0 ϕα(t) dt converge (on
l’obtient facilement en écrivant ϕα(t) = log(1 − t) + log α1+t2 +t2 ). Maintenant, on remarque que

ϕα est décroissante sur ]0, 1] (composition de la fonction décroissante t → (1 − t2 )/(α2 + t2 ) par


a aj
la fonction log qui est croissante), donc ajj+1 ϕα (t) dt ≤ 1n ϕ α (aj ) ≤ aj−1 ϕα(t) dt, ce qui, par
sommation pour 1 ≤ j ≤ n − 2 donne
 n−2 
1−1/(2n)
1 1−3/(2n)
I (α) + o(1) = ϕα(t) dt ≤ ϕα(a j ) ≤ ϕα (t) dt = I (α) + o(1).
3/(2n) n j=1 1/(2n)


Donc n−1 j=0 ϕα (aj ) = nI (α) + o(n) (on a bien ϕα (a 0) + ϕ α (an−1) = o(n)). Avec (***), on en
déduit finalement, lorsque n → ∞,
 1
1 1 − t2
log |fα (1) − P n (1)| = log + n(I (α) + o(1)), avec I (α) = log dt.
1 + α2 0 α2 + t 2

c) Nous allons montrer que I est définie et continue en 0. Lorsque α ∈ [0, 1] et 0 < t ≤ 1, on a
 
2 2 1 1 2 2 1 2
α + t ≤ 2 et 2 2
≤ 2 donc | log(α + t )| ≤ max log 2, log 2 ≤ log 2 .
α +t t t t

Comme la fonction t → log(2/t2 ) est intégrable sur ]0, 1], l’hypothèse de domination est vérifiée

donc la fonction α → 01 log(α 2 +t2) dt est bien définie et continue pour α ∈ [0, 1]. En particulier,
I est définie et continue en α = 0, et comme I (0) = 2 log 2 > 0, on en déduit l’existence de α0 > 0
tel que I (α) > 0 pour 0 ≤ α < α0 . D’après le développement asymptotique obtenu à la question
précédente, lorsque 0 ≤ α < α0 , on a donc limn→+∞ log |fα (1) − P n (1)| = +∞. Ainsi il n’y a
pas convergence simple (et donc pas uniforme) de (Pn ) vers f lorsque 0 ≤ α < α 0 .
Remarque. On peut obtenir la forme explicite I (α) = 2 log 2 − α arctan(1/α) −log(1+ α 2).
On peut montrer que I (α) > 0 pour α < α0 ≈ 0, 5255249.
– Pour éviter de passer par les sommes de Riemann d’une intégrale généralisée dans
b), on aurait pu obtenir le comportement de log |ω n(1)| par la formule de Stirling. Le
comportement de log |ωn (iα)| s’obtient directement à partir d’une somme de Riemann
d’une fonction définie et continue sur le segment [0, 1].
– Ainsi, une suite de polynômes interpolants ne converge pas forcément vers une fonction f
à approcher. Cependant, certaines conditions de majoration des dérivées de f permettent
de garantir qu’un polynôme interpolant est une bonne approximation (voir question b)
de l’exercice 7 page 83).
242 4. SUITES ET SÉRIES

Exercice 8 (Polynômes d’approximation de Bernstein). On note I = [0, 1] et


C l’e.v des fonctions continues de I dans C. On note, pour toute fonction f ∈ C et n ∈ N∗
 n  
k
Bn(f ) : I → C x → f bkn (x), avec bkn (x) = Cnkxk (1 − x)n−k .
k=0
n
 n  2
k
a) Calculer explicitement l’expression − x bkn (x), puis en déduire que pour tout
k=0
n
η > 0 et pour tout x ∈ I ,
 1
bkn (x) ≤ 2 .

k, |k/n−x|≥η

b) (Théorème de Bernstein) Pour tout f ∈ C, montrer que la suite de fonctions (Bn (f ))n∈N
converge uniformément vers f sur [0, 1]. En déduire le théorème de Weierstrass (théorème 5
page 235).
c) Donner une condition nécessaire et suffisante pour que f ∈ C soit limite uniforme sur
[0, 1] de fonctions polynômes à coefficients entiers.
d) Si f est une fonction lipschitzienne, montrer que f − Bn (f )∞ = O (n−1/2 ).
e) Considérons la fonction f : [0, 1] → R définie par f (x) = |x − 1/2|. Donner un
équivalent de Bn (f )(1/2) − f (1/2) lorsque n → ∞ (on pourra utiliser l’égalité kCnk =
k−1
nCn−1 et la formule de Stirling).

Solution.
a) En notant par abus 1, x, x2 les fonctions x → 1, x → x et x → x 2, on a pour tout n ∈ N∗
n  2 n  2 
k k k k
−x bn (x) = − 2x + x b kn(x) = B n (x2 ) − 2xB n(x) + x2 Bn(1).
2
(∗)
n n2 n
k=0 k=0

On se ramène ainsi à exprimer Bn(1), Bn (x) et Bn (x2 ). Pour cela, on part de l’identité bien
connue
n
∗ 2
 n 
∀n ∈ N , ∀(a, b) ∈ R , F (a, b) = a + (1 − b) = Ckn ak (1 − b) n−k .
k=0
On en déduit Bn (1) = F (1, 1) = 1,
n
1  k x ∂F x 
B n(x) = C n k x k(1 − x) n−k = (x, x) = n x + (1 − x))n−1 = x
n n ∂a n
k=0
et après un petit calcul
n  
2 1  k 2 k n−k x ∂ ∂F x(1 − x)
Bn (x ) = 2 C n k x (1 − x) = 2 a (x, x) = x 2 + .
n n ∂a ∂a n
k=0
En remplaçant ces expressions dans (*), on obtient
n  2
k x(1 − x)
− x b kn(x) = .
n n
k=0
La seconde partie de la question s’obtient maintenant en écrivant
 n  2
k 1  k 1 x(1 − x) 1
bn (x) ≤ 2 − x bkn (x) = 2 ≤ 2. (∗∗)
η n η n nη
k,|k/n−x|≥η k=0

b) Analysons la situation. On peut voir l’expression donnant Bn (f )(x) comme un barycentre


des points f (k/n), dont les coefficients les plus significatifs se trouvent dans la région où k/n est
dans un voisinage de x d’après (**). On a donc Bn (f )(x)  f (x).
3. SUITES ET SÉRIES DE FONCTIONS 243

Précisons. Donnons nous ε > 0. La fonction f est continue sur le compact I. Elle est donc
bornée de sorte qu’il existe M > 0 tel que |f (x)| ≤ M sur I . Elle est aussi uniformément continue
d’après le théorème de Heine donc
∃η > 0, ∀(x, x ) ∈ I 2 , |x − x | < η, |f (x) − f (x )| < ε.
On écrit maintenant
 n    
 k  k

∀x ∈ I, ∀n ∈ N , |B n(f )(x) − f (x)| = |B n (f )(x) − f (x)Bn(1)| ≤  f − f (x ) b n (x)
 n 
k=0
     n 
   2M 2M
≤ ε bkn (x)  + 2M  b kn (x) ≤ ε b kn(x) + 2 = ε + 2 .
nη nη
k,|k/n−x|<η k,|k/n−x|≥η k=0
∗ 2
Ainsi, si N ∈ N est choisi tel que 2M/(N η ) < ε, on a montré
∀n ≥ N, ∀x ∈ I, |Bn (f )(x) − f (x)| < 2ε.
La suite de fonctions (Bn (f )) converge donc uniformément vers f sur [0, 1].
Les Bn(f ) sont des fonctions polynômes, on vient donc de montrer que toute fonction conti-
nue sur [0, 1] est limite uniforme de fonctions polynômes sur [0, 1]. On en déduit facilement par
changement de variable affine le théorème de Weierstrass (si f : [a, b] → C est continue, on se
ramène à [0, 1] en considérant g (x) = f [a + (b − a)x]).
c) Montrons que f ∈ C est limite uniforme sur [0, 1] de fonctions polynômes à coefficients entiers
si et seulement si f (0) et f (1) sont des entiers.
La condition est nécessaire : en effet, si f est limite uniforme de fonctions polynômes (Pn )
à coefficients entiers, alors f (0) (respectivement f (1)) est la limite de la suite d’entiers (Pn (0))
(respectivement (Pn (1))). Une suite d’entiers convergente a pour limite un nombre entier, donc
f (0) et f (1) sont des entiers.
La condition suffisante s’obtient à partir du résultat de la question précédente. Supposons
f (0) et f (1) entiers. On considère la suite de polynômes à coefficients entiers (Zn (f )) définie par
n    
k
Zn(f ) : [0, 1] → R x → f Cn xk (1 − x)n−k ,
k
n
k=0
où [ t] désigne la partie entière de t. Pour x ∈ [0, 1], on a
n−1
      
 k f kn C kn
B n (f )(x) − Zn(f )(x) = f − bkn (x)
n C kn
k=1
(les termes de la somme pour k = 0 et k = n sont nuls car f (0) et f (1) sont entiers), et comme
0 ≤ f (k/n) − [f (k/n)Ckn]/Cnk < 1/Cnk ≤ 1/n pour 1 ≤ k ≤ n − 1 on en déduit
n−1
 n
1 k 1 k B (1) 1
0 ≤ Bn (f )(x) − Zn (f )(x) ≤ bn (f )(x) ≤ bn(f )(x) = n = ,
n n n n
k=1 k=0
et ceci pour tout x ∈ [0, 1]. Comme (Bn (f )) converge uniformément vers f on en déduit que
(Zn(f )) converge uniformément vers f .
d) Considérons une fonction λ-lipschitzienne f sur [0, 1]. Soit n ∈ N ∗ et ε = n−1/2 . Pour x ∈ I ,
on a
         
 k  k  k  k
|Bn (f )(x) − f (x)| ≤    
f n − f (x) bn(x) +  f n − f (x)  bn (x)
k,|k/n−x|≤ε k,|k/n−x|>ε
   2
λ k
≤ λεbkn (x) + − x b kn(x)
ε n
k,|k/n−x|≤ε k,|k/n−x|>ε
n
 n 
 2
λ k λ x(1 − x) λ 2λ
≤ λε bkn (x) + − x b kn (x) = λε + ≤ λε + = √ .
ε n ε n nε n
k=0 k=0
244 4. SUITES ET SÉRIES

Ceci étant vrai pour tout x ∈ I , on en déduit le résultat.


e) Soit un = B n (f )(1/2) − f (1/2) = Bn(f )(1/2). L’égalité b kn(1/2) = Cnk/2 n donne
     
1  1 k k 1  k 1 k 1  2k
un = n − Cn + n − Cn = n 1− C kn.
2 2 n 2 n 2 2 n
0≤k<n/2 n/2<k≤n 0≤k<n/2

où on a utilisé la relation de symétrie C nn−k = Cnk. Grâce à la relation kCnk = nC k−1
n−1 on en déduit
 
2n un = Ckn − 2 k−1
Cn−1 .
0≤k<n/2 1≤k<n/2

k−1
Il reste à utiliser la relation du triangle de Pascal Ckn = Cn−1 k
+ Cn−1 qui entraı̂ne
    [(n−1)/2]
2nun = 1 + (C kn − Cn−1
k−1
)− k−1
Cn−1 = C kn−1 − k−1
Cn−1 = C n−1 ,
1≤k<n/2 1≤k<n/2 0≤k<n/2 1≤k<n/2

où [ ( n − 1)/2] désigne la partie entière de (n − 1)/2. La formule de Stirling permet facilement
[(n−1)/2] √
d’obtenir l’équivalent Cn−1 ∼ 2n / 2πn. Finalement on a obtenu l’équivalent
   
1 1 1
u n = Bn (f ) −f ∼ √ .
2 2 2πn
Ainsi, l’estimation obtenue à la question précédente est la meilleure possible dans l’hypothèse
d’une fonction lipschitzienne.
Remarque. Le résultat de la question b) de cet exercice est une version constructive du
théorème de Weierstrass, mais la convergence est lente, comme le montre le résultat de
la question e). Le meilleur approximant polynomial peut s’obtenir par l’algorithme de
Remez.

Exercice 9 (Fonction dériv ée à points de discontinuit és denses).


a) (Généralisation du théorème 4 page 234 lorsque les fonctions fn sont seulement sup-
posées dérivables). Soit (f n) une suite de fonctions dérivables de I = [0, 1] dans un espace
de Banach E . On suppose que la suite des dérivées (fn ) converge uniformément vers une
fonction g sur I , et qu’il existe x 0 ∈ I tel que la suite (f n(x0 ))n∈N converge. Montrer que
(fn) converge uniformément sur I vers une fonction dérivable f qui vérifie f  = g .
b) Exhiber une fonction f : I = [0, 1] → R, dérivable sur I , telle que l’ensemble des
points de discontinuité de f  soit dense dans [0, 1].

Solution. a) Ici, on ne peut pas procéder comme dans la preuve du théorème 4 page 234 car
une fonction dérivée n’est pas forcément continue (ni Riemann-intégrable).
Nous montrons d’abord que (fn ) vérifie le critère de Cauchy uniforme. Soit ε > 0. D’après
les hypothèses,
∃N ∈ N, ∀p, q ≥ N, ( f p (x0 ) − fq (x0) < ε et ∀x ∈ I, f p(x) − fq (x) < ε).
Ainsi, on obtient grâce à l’inégalité des accroissements finis que pour tout p, q ≥ N et pour tout
x ∈ I,
f p(x) − fq (x) ≤ (f p − f q)(x) − (f p − fq )(x0) + (f p − f q)(x 0) < ε|x − x0 | + ε ≤ 2ε.
Autrement dit, la suite de fonctions (fn ) à valeurs dans un espace de Banach vérifie le critère
de Cauchy uniforme, elle converge donc uniformément vers une fonction f sur I .
Il nous reste à montrer que f est dérivable et que f  = g. Fixons x ∈ I et ε > 0. La suite

(f n) vérifie le critère de Cauchy uniforme donc
∃N ∈ N, ∀p ∈ N, ∀t ∈ I, f N (t) − fN +p (t) ≤ ε. (∗)
3. SUITES ET SÉRIES DE FONCTIONS 245

En appliquant (*) à t = x et en faisant p → +∞, on tire f N (x) − g(x) ≤ ε, et par définition


de fN (x), on en déduit
 
f N (t) − fN (x) 
∃α > 0, ∀t ∈ I, 0 < |t − x| < α,  − g (x)
 t−x  ≤ 2ε. (∗∗)

Maintenant, l’inégalité des accroissements finis entraı̂ne avec (*)


∀p ∈ N, ∀t ∈ I, (f N − f N +p)(t) − (f N − f N +p )(x) ≤ ε|t − x|,
et on en déduit en fixant t ∈ I et en faisant p → +∞ que
∀t ∈ I, (fN − f )(t) − (f N − f )(x) ≤ ε|t − x|. (∗∗∗)
De (**) et (***), on tire, pour tout t ∈ I tel que 0 < |t − x| < α,
     
 f (t) − f (x)   (f − f N)(t) − (f − f N )(x)   fN (t) − f N(x) 
  
− g (x)  ≤   
+ − g (x)
 t−x t−x  t−x  ≤ ε + 2ε = 3ε.
En d’autres termes, nous venons de montrer que la fonction f est dérivable au point x et que
f  (x) = g (x). Ceci étant vrai pour tout x ∈ I , on en déduit le résultat.
b) Nous allons construire une telle fonction f à partir de la fonction

x2 sin(1/x) si x = 0
ϕ : [−1, 1] → R x →
0 si x = 0.
Cette fonction est dérivable sur [−1, 0[ ∪ ]0, 1] et on a ϕ (x) = 2x sin(1/x) − cos(1/x) sur cet
ensemble. Or
ϕ(x) − ϕ(0)
∀x ∈ [−1, 1], −x2 ≤ ϕ(x) ≤ x2 donc ∀x =  0, −|x| ≤ ≤ |x|,
x−0
donc ϕ est dérivable en 0 et ϕ (0) = 0. Ainsi, ϕ est une fonction dérivable sur [−1, 1] tout entier
et ϕ est discontinue en 0, continue ailleurs.
L’ensemble Q∩[0, 1] est dénombrable, il existe donc une bijection n → rn de N∗ dans Q∩ [0, 1].
Pour tout n ∈ N∗ , on définit la fonction
1
fn : [0, 1] → R x → ϕ(x − rn).
n2
 
L’expression de ϕ montre que ϕ  est bornée, donc f n converge normalement sur [0, 1]. Il
est clair qu’il existe x0 ∈ [0, 1] tel que (f n (x0 ))nconverge. D’après le résultat de la question
précédente, on en déduit que la série de
fonctions fn converge uniformément vers une fonction
dérivable f sur [0, 1] qui vérifie f  = +∞ f
n=1 n

. Nous allons montrer que f répond à la question.
— Pour tout x ∈ [0, 1]Q, f n est continue en x donc f  est continue en x (limite uniforme

de fonctions continues en x).


— Pour tout x ∈ [0, 1] ∩ Q, il existe N ∈ N ∗ tel que x = r N. Pour tout n = N , on a x = r n
donc fn est continue en x, donc n=N fn est continue en x. Or fN est discontinue en x,

on en déduit que f = f N + n=N fn est discontinue en x.
Finalement, nous avons montré que f est dérivable sur [0, 1], discontinue en tout point rationnel,
continue en tout point irrationnel. D’où le résultat.
Remarque. On peut montrer que l’ensemble des points de continuité d’une fonction dérivée
est dense (voir l’exercice 2 page 419).

Exercice 10 (Th éorème de Helly). Soit (fn ) n∈N une suite de fonctions croissantes
d’un intervalle ouvert I de R dans R telle que, pour tout x ∈ I , la suite (fn (x))n∈N
est bornée. Démontrer qu’il existe une sous-suite (f ϕ(n))n∈N et une fonction croissante
f : I → R, telle que cette sous-suite converge simplement vers f .
246 4. SUITES ET SÉRIES

Solution. L’idée est de procéder par densité, en montrant d’abord le résultat sur les rationnels
(dénombrables) de I , puis d’utiliser la croissance des fonctions fn pour étendre sur I tout entier.
Notons QI = I ∩ Q l’ensemble dénombrable des rationnels de I , et notons x0 , x1 , . . . , xn , . . .
les éléments de QI . On utilise le procédé diagonal, déjà employé dans la solution de l’exercice 2
page 32. De la suite bornée (fn(x0 )) n on extrait une sous-suite convergente (f ϕ0 (n)(x0 ))n . De la
suite bornée (fϕ 0 (n) (x 1))n , on extrait une sous-suite convergente (f ϕ0◦ϕ1 (n) (x1 )) n . En procédant
par récurrence, on peut ainsi construire, pour tout entier naturel p, une sous-suite convergente
(f ϕ0 ◦···◦ϕp (n) (xp ))n∈N . La fonction ψ : N → N définie par ψ (n) = ϕ0 ◦ · · · ◦ ϕn (n) est strictement
croissante, et pour tout entier naturel p, la suite (fψ(n) (x p))n converge (car (fψ(n) (xp )) n≥p est
une sous-suite de (fϕ0 ◦···◦ϕ p(n) (x p))n∈N qui converge).
Construisons maintenant la fonction g définie sur QI par g (x) = limn→∞ fψ(n) (x) pour tout
x ∈ QI. Pour x < y, on a f ψ(n) (x) ≤ fψ(n)(y), et en passant à la limite on voit que g est croissante
sur QI. On étend g sur I tout entier de la manière suivante : pour x ∈ I QI , on définit
g (x) = sup G x, avec Gx = {g (y ) | y ∈ Q I , y < x}
(Gx est non vide car I est ouvert, et majoré car g étant croissante, tous les éléments de Gx sont
inférieurs à g (z ) pour un z > x fixé dans Q I). Ainsi définie, g est croissante sur I , comme on le
vérifie facilement en montrant, g (x) ≤ g (y ) lorsque x < y, d’abord pour x ∈ I QI et y ∈ QI ,
puis pour x ∈ QI et y ∈ I Q I , puis pour x et y dans I QI .
Montrons que pour x dans l’ensemble C ⊂ I des points de continuité de g , la suite (fψ(n)(x))
converge vers g (x). Étant donné ε > 0, comme g est continue en x, il existe a et b dans Q I tels
que a < x < b et
|g (x) − g (a)| < ε, |g (x) − g (b)| < ε donc g (b) − ε < g(x) < g(a) + ε.
Les suites (fψ(n) (a))n et (f ψ(n) (b))n convergent vers g (a) et g (b) respectivement, donc il existe
N > 0 tel que pour tout n ≥ N , |fψ(n) (a) − g (a)| < ε et |fψ(n) (b) − g (b)| < ε, donc
fψ(n) (b) − 2ε < g(b) − ε < g(x) < g(a) + ε < f ψ(n) (a) + 2ε
Comme fψ(n) est croissante, on en déduit fψ(n) (x) − 2ε < g(x) < f ψ(n)(x) + 2ε. Ceci est vrai
pour tout n ≥ N , donc la suite (fψ(n) (x)) converge bien vers g (x), et ceci pour tout x ∈ C .
Il reste le cas des points de discontinuité D = I C de g . La fonction g est croissante donc D
est au plus dénombrable (car une fonction monotone est réglée et l’ensemble des discontinuités
d’une fonction réglée est au plus dénombrable, voir le théorème 4 page 99. On peut aussi obtenir
ce résultat directement, en associant à chaque discontinuité x de g un nombre rationnel différent
en le choisissant dans ]g (x−), g(x+)[). En procédant de la même manière que plus haut, on en
déduit qu’il existe une sous-suite (fψ◦θ (n) ) de (fψ(n) ) telle que (f ψ◦θ (n) (x))n converge pour tout
x ∈ D (attention, cette limite n’est pas forcément égale à g(x)).
Ainsi, pour tout x ∈ I , la suite (fψ◦θ (n)(x))n converge. La fonction f : I → R définie par
f (x) = limn→∞ f ψ◦θ (n) (x) est croissante, et ceci clos la démonstration.

Exercice 11. Soit p ∈ N∗ et (fn )n∈N une suite de fonctions réelles de classe C p sur un
segment [a, b] de R. On suppose que la suite de fonctions (fn(p) )n∈N converge uniformément
vers une fonction g sur [ a, b], et qu’il existe p points distincts x1 , . . . , xp de [a, b] tels que
pour tout i (1 ≤ i ≤ p), la suite (fn(x i))n∈N converge. Montrer que (f n )n∈N converge
uniformément sur [a, b] vers une fonction f de classe C p telle que f (p) = g .
Solution. C’est une généralisation du théorème de dérivation de la limite d’une suite de fonctions.
Notre point de départ est la formule de Taylor avec reste intégral qui donne
∀n ∈ N, ∀x ∈ [a, b], fn (x) = P n(x) + I n(x)
avec
 x
(x − a)p−1 (p−1) (x − t) p−1 (p)
Pn (x) = fn(a) + (x − a)f n(a) + · · · + f (a) et In (x) = f (t) dt.
(p − 1)! n a (p − 1)! n
4. SÉRIES ENTI ÈRES 247

Le polynôme Pn est de degré ≤ p − 1, donc il est égal à au polynôme d’interpolation de Lagrange


égal à Pn (xi ) sur les points (xi) 1≤i≤p, ce qui s’écrit
p 
 j =i(x − xj )
∀x ∈ [a, b], Pn (x) = P n(x i )f i(x), avec fi(x) =  .
i=1 j =i (x i − x j )

Si h est continue sur [a, b], notons h∞ = supx∈[a,b] |h(x)| la norme de la convergence uni-
forme. La fonction g est continue, car c’est la limite uniforme d’une suite de fonctions continues.
 x t) p−1 (b−a)p (p)
Considérons la fonction I (x) = a (x(−p− 1)! g (t) dt. On a I − In ∞ ≤ p! f n − g ∞ donc (In )
converge uniformément vers I . Pour tout i, on a Pn (xi ) = f n(xi) − I n (xi ) donc la suite (Pn(xi ))n
p
converge. Notons yi sa limite, et notons P (x) = i=1 y ifi (x) le polynôme d’interpolation de
Lagrange égal à yi en xi . Posons f (x) = P (x) + I (x). La formule
p

f − f n∞ ≤ P − P n ∞ + I − In ∞ ≤ |yi − P n(xi )| · f i∞ + I − I n ∞
i=1
montre que f − fn ∞ converge vers 0, donc (f n ) converge uniformément vers f . Une récurrence
facile montre que pour 1 ≤ k ≤ p − 1, f est de classe C k et vérifie
x
(k) (k) (x − t)p−k−1
∀x ∈ [a, b], f (x) = P (x) + g (t) dt
0 (p − k − 1)!
x
En particulier, f est de classe C p−1 et vérifie f (p−1)(x) = P (p−1) (x) + 0 g (t) dt, donc f est de
classe C p et f (p) = g (car deg(P ) ≤ p − 1), d’où le résultat.

4. Séries entières
4.1. Définitions

Définition 1. On appelle série entière toute série de fonctions de la forme n∈N an z n
où z est une variable complexe et où (an ) est une suite complexe.
Rayon de convergence. Il est naturel de s’interroger sur le domaine des nombres com-
plexes z pour lesquels une série entière converge.
 Proposition 1 (Lemme d’Abel). Soit  a z n une série entière et z ∈ C tel que la
n 0
suite (an zn0 )n∈N soit bornée. Alors
(i) ∀z ∈ C, |z | < |z0 |, la série an zn est absolument convergente ;
n
(ii) pour tout r, 0 < r < |z0 |, la série de fonctions an z est normalement conver-
gente dans {z ∈ C | |z | ≤ r }.
Démonstration. Si M est un majorant de (|an| |z 0| n), la preuve est simple à partir de la majo-
ration  n  n
 z z
|an z | =   |an | |z0| ≤ M   .
n n
z0 z0

Le lemme d’Abel justifie la définition suivante.

Définition 2 (Rayon de convergence). Soit an z n une série entière. Le nombre
R = sup{r ≥ 0 | la suite (|an | rn ) est bornée}
 n
s’appelle rayon de convergence de an z . D’après le lemme d’Abel,
— pour tout z ∈ C tel que |z | < R,  an zn converge absolument ;
— pour tout z ∈ C tel que |z | > R, an zn diverge ;
— pour tout r tel que 0 ≤ r < R, la série entière an z n converge normalement sur
{z ∈ C | |z | ≤ r }.
248 4. SUITES ET SÉRIES

Le disque ouvert {z ∈ C | |z| < R} est appelé disque de convergence de la série entière.

Remarque 1. — On peut avoir R = 0 ou R = +∞. Si R = +∞, an z n converge
pour tout z ∈ C et la somme de cette série entière définit une fonction de C dans
C appelée fonction entière.
— Sur le cercle |z | = R, la série entière
 peut ou non converger.
n
— Les séries entières a n z et |an| z n ont même rayon de convergence.
Calcul pratique du rayon de convergence. Il existe quelques recettes qui permettent
parfois de calculer explicitement le rayon de convergence d’une série entière. Celles ci sont
des conséquences directes des règles de d’Alembert et de Cauchy que l’on a vu pour les
séries numériques.
Proposition 2 (Règle de d’Alembert). Si limn→ +∞ |a n+1 /a n| = λ avec λ ∈ [0, +∞],
alors le rayon de convergence de la série entière an zn est R = 1/λ (en convenant
1/0 = +∞ et 1/ + ∞ = 0).
1/n
Proposition 3 (Règle de Cauchy). Si lim  n→+n∞ |an| = λ avec λ ∈ [0, +∞], alors le
rayon de convergence de la série entière an z est R = 1/λ (en convenant 1/0 = +∞
et 1/ + ∞ = 0).
Exemple 1. Grâce à larègle de d’Alembert, on montre facilement que
— la série entière zn /n! a un rayon  deα convergence infini ;
n
— pour tout α ∈ R , la série entière n z a un rayon de convergence égal à 1 ;
n
— la série entière n! z a un rayon de convergence nul.
Remarque 2. — Attention, les règles de d’Alembert oude Cauchy ne s’appliquent pas
toujours (essayez de les appliquer, par exemple, à z2n ).
— On peut montrer, pour une série entière donnée, que si la règle de d’Alembert
s’applique alors la règle de Cauchy s’applique (la réciproque est
 fausse).
— Dans tous les cas, on montre que le rayon de convergence de an zn est 1/ρ avec
1/p
ρ = lim n→+∞(supp≥n |ap | ).
 
Somme et produit de séries entières. Soient an z n et bn zn deux séries entières
de rayon de convergence respectivement égal à R > 0 et R > 0. Notons f et g les sommes
de ces séries entières sur leurdisque de convergence D et D .
Somme. La nsérie entière cn z n définie par c n = an + bn est appelée somme des séries
  
entières an z et b n zn . Son rayon de convergence  Rn de vérifie R ≥ inf {R, R } et

sur D ∩ D , f + g est la somme  denla série entière cn z .
Produit. La série entière cn z définie par
∀n ∈ N, c n = a 0 bn + a 1bn−1 + · · · + an−1 b1 + a nb0
 
est appelée produit de Cauchy des séries entières a n z n et bn z n. Son rayon de conver-
   
gence Rvérifie R ≥ inf {R, R } et sur D ∩ D , le produit f g est la somme de la série
entière cn z n (conséquence du théorème 9 page 216 sur le produit de Cauchy de deux
séries absolument convergentes).
Remarque 3. On ne peut rien dire de plus en général sur les rayons de convergence de la
somme
 n oudu produit de Cauchy de deux séries entières. Par exemple, les séries entières
z et −z n ont leur rayon de convergence égal à 1 mais la somme a un rayon de
convergence infini.
4.2. Propriétés des séries entières

Dans toute cette sous-partie, a n zn désigne une série entière de rayon de convergence
R > 0.
4. SÉRIES ENTI ÈRES 249

Continuité.
+∞

Théorème 1. L’application z → an z n est continue sur le disque de convergence
n=0
{z ∈ C, |z | < R}.

Démonstration. Pour tout r ∈ ]0, R[, a n zn converge normalement sur |z| ≤ r , sa somme est
donc continue sur |z| ≤ r (chaque somme partielle est continue), et ceci pour tout r < R, d’où
le résultat. 

Dérivation.
 +∞ n 1
Théorème 2.  L’application f : ]−R, R[ → C x →
 an x n est de classe C . La
n=0
n−1
série entière nan z a même rayon de convergence que a n z , et on a
+∞


∀x ∈ ]−R, R[ , f (x) = nan x n−1 .
n=1

Démonstration. Notons R le rayon de convergence de nan z n−1 . Soit r, 0 ≤ r < R. La suite
(nan r n ) est bornée, donc (an rn ) est bornée donc r ≤ R . Ceci étant vrai pour tout r < R, on
en déduit R ≤ R. Maintenant, soit r < R. Si on fixe r0 tel que r < r0 < R, la suite (an r n0 ) est
bornée, donc la suite (nan rn ) tend vers 0 car nan r n = n(an r n0 )(r/r0 ) n avec r/r0 < 1. On en
conclut r < R , et ceci pour tout r < R donc R ≤ R  . Finalement, on a donc R = R .
La dérivabilité de f et la valeur de f  sont une conséquence du théorème de dérivabilité des
suites de fonctions. 
En appliquant ce théorème par récurrence, on obtient le résultat qui suit.

Corollaire 1. La somme f de la série entière an z n est de classe C ∞ sur ] − R, R[.
De plus, pour tout p ∈ N, f (p) est la somme sur ] − R, R[ d’une série entière de rayon de
convergence R > 0. En outre,
+∞ (p)

f (p) (0) f (0) p
∀p ∈ N, ap = donc ∀z ∈ C, f(z) = z .
p! p=0
p!

Conséquence :  an
— La série entière z n+1 a pour rayon de convergence R et si F désigne la
n+1
somme de cette dernière, on a F  = fsur ] − R, R[.
— Si g est la somme d’une série entière b n zn sur {z, |z | < R } (avec R > 0) et si f
et g coı̈ncident sur un voisinage de 0 dans R, alors pour tout n, bn = g (n)(0)/n! =
f (n)(0)/n! = an . Ceci reste valable si f et g coı̈ncident sur un intervalle de la forme
]0, α[.
Remarque 4 (Dérivation par rapport à la variable complexe). On montre que les fonctions
f définies par des séries entières sont dérivables par rapport à la variable complexe
 sur leur
disque de convergence D , c’est-à-dire que pour tout z0 ∈ D, la limite de f (z 0 + u) −
f (z0 ) /u existe lorsque u tend vers 0 dans C en restant non nul.
Comme on s’y attend, la dérivée de la restriction f de f à l’axe réel et celle de f (au
sens complexe) coı̈ncident sur R.
La condition de dérivabilité par rapport à la variable complexe est très forte. Il ne
suffit pas qu’une fonction de C dans C soit différentiable pour qu’elle soit dérivable par
rapport à la variable complexe (voir l’exercice 8 page 333), mais la réciproque est vraie.
On peut montrer qu’une fonction g : D = {z, |z | < R} → C continûment dérivable par
rapport à la variable complexe sur D (on dit alors que g est holomorphe) est la somme
d’une série entière sur D (voir l’exercice 13 page 265).
250 4. SUITES ET SÉRIES

Principe des zéros isolés.



Théorème 3. Soit f la somme de la série entière an z n sur son disque de convergence.
S’il existe une suite (z p) de nombres complexes non nuls tendant vers 0 telle que f (zp ) = 0
pour tout p, alors an = 0 pour tout n.
Démonstration. Supposons que l’un des an ne soit pas nul, et notons q le plus petit entier naturel
tel que a q = 0. On peut écrire, sur D,
+∞

q
f (z ) = z g (z ) avec g (z ) = an+q zn .
n=0

D’après les hypothèses, g (zp ) = 0 pour tout p, et comme g est continue en 0 (c’est la somme
d’une série entière de rayon de convergence R > 0), on a

aq = g(0) = lim g (zp ) = 0,


p→+∞

ce qui est absurde. Donc a n = 0 pour tout n. 


 
Conséquence : Si les sommes f et g de deux séries entières an z n et bn zn vérifient
f (zp ) = g (zp) pour une suite (zp ) de nombres complexes non nuls tendant vers 0, alors
an = bn pour tout n (appliquer le théorème à f − g ). En particulier, deux séries entières
dont les sommes coı̈ncident sur un voisinage de 0 dans R sont égales. On retrouve ainsi
le résultat énoncé dans la conséquence du corollaire 1.
Formule de Cauchy. La formule suivante n’est pas au programme des classes de
mathématiques spéciales mais elle est d’une importance capitale. On s’en sert souvent
dans les exercices et les problèmes.


 Théorème 4 (Formule de Cauchy). Soit an z n une série entière de rayon de
convergence R > 0, et f la somme de cette série entière sur son disque de convergence.
Alors
 2π
n
∀r ∈ ]0, R[ , ∀n ∈ N, 2π r an = f (reiθ )e−niθ dθ.
0

Démonstration. Fixons r ∈ ]0, R[ et n ∈ N. Il suffit décrire


 
 2π  2π +∞ +∞
  2π
iθ −niθ
f (re )e dθ =  p
a pr e i(p−n )θ  dθ = ap r p
ei(p−n)θ dθ (∗)
0 0 p=0 p=0 0


(on a le droit d’inverser les signes de sommation car la série de fonctions p ap rp e i(p−n)θ est

normalement convergente sur [0, 2π ], ceci parce que |ap | r p converge, le réel r vérifiant 0 ≤

r < R). On conclut à partir de (*) en remarquant que 02π ei(p−n)θ dθ = 0 si p = n, = 2π si
p = n. 

Égalité de Parseval. Le résultat qui suit est la version “série entière” du théorème 1
page 270 (en l’appliquant à la série trigonométrique f (reiθ )).

Théorème 5. Soit an zn une série entière de rayon de convergence R > 0, et f la
somme de cette série entière sur son disque de convergence. Alors pour tout r ∈ ]0, R[, la
série |an | 2 r2n converge et on a
+∞
 
1 2π  2
2 2n
|a n | r = f (reiθ )  dθ.
n=0
2π 0
4. SÉRIES ENTI ÈRES 251

4.3. Développement de fonctions en séries entières


Une fonction (de la variable réelle ou complexe) à valeurs complexes définie dans
un voisinage de 0 est dite développable en série entière sur un voisinage de 0 si sur ce
voisinage, f coı̈ncide avec la somme d’une série entière de rayon de convergence non nul.
Développement en série entière des fractions rationnelles. Soit F une fraction
rationnelle à coefficients dans C. Si 0 n’est pas un pôle de F , nous voulons développer F
en série entière. Après décomposition en éléments simples de F , on se ramène à développer
en série entière les fractions de la forme 1/(z − z 0)p où p ∈ N∗ et z 0 = 0.
Pour tout z ∈ C tel que |z | < |z0|, on a
+∞  
1 1 1 1  z n
=− =− .
z − z0 z 0 1 − z/z0 z0 n=0 z 0

Par produit de Cauchy, on voit donc que pour tout p ∈ N∗, 1/(z − z 0 ) p est développable
en série entière. Pour obtenir son développement en série entière, on dérive (p − 1) fois
1/(x − z0 ) sur ] − |z0 |, |z0 |[, ce qui donne
+∞
  n−p+1
1 (−1)p n! x
∀x ∈ ] − |z0|, |z 0 |[, p
= p ,
(x − z0) z0 (p − 1)! n=p−1 (n − p + 1)! z0

et comme ces deux expressions coı̈ncident sur un voisinage de 0 dans R, elles coı̈ncident
sur tout le disque de convergence |z | < |z0 | (voir la conséquence du corollaire 1).
Ainsi, toute fraction rationnelle complexe F dont 0 n’est pas un pôle est développable
en série entière au voisinage de 0. On peut montrer que le rayon de convergence de cette
dernière est égal au plus petit des modules des pôles de F .
Développement en série entière d’une fonction à variable réelle. Soit f une
fonction complexe de la variable réelle définie sur un voisinage de 0. Si f est développable
en série
entière, il existe α > 0 tel que sur ] − α, α[, f coı̈ncide avec la somme d’une série
entière an z n de rayon de convergence ≥ α. Ceci implique que f est de classe C∞ sur
] − α, α[ et que a n = f (n) (0)/n! pour tout n. Ceci constitue la condition nécessaire de la
proposition qui suit.

 Proposition 4. Soit I un intervalle de R contenant un voisinage de 0. Une fonction


f : I → C de classe C ∞ est développable en série entière sur un voisinage de 0 si et
seulement s’il existe α > 0 tel que la suite de fonctions (Rn ) définie par
n
 f (k) (0)
Rn (x) = f (x) − xk
k!
k=0

tende simplement vers 0 sur ] − α, α[. La série entière


 f (n)(0)
zn (∗)
n!
a alors un rayon de convergence ≥ α et f est égale à la somme de cette série entière sur
] − α, α[.
 f (n) (0) n
Démonstration. Pour tout x ∈ ] − α, α[ la série n! x converge vers f (x) car R n(x) → 0.
(n) n
En particulier, la suite (f (0) x /n!) tend vers 0, donc est bornée, donc la série entière (*) a
un rayon de convergence ≥ |x| (voir la définition 2). Ceci étant vrai pour tout x ∈ ] − α, α[, on
en déduit que le rayon de convergence de (*) est ≥ α. 
252 4. SUITES ET SÉRIES

Dans la pratique, pour montrer que (Rn ) tend simplement vers 0, on peut utiliser la
formule de Taylor qui permet d’écrire Rn (x) comme l’une des expressions
 x
xn+1 (n+1) (x − t) n (n+1)
f (θ x), (θ ∈ ]0, 1[) ou f (t) dt
(n + 1)! 0 n!

(reste de Lagrange, reste intégral). Le reste intégral donne en général des résultats plus
fructueux.
Remarquez que pour montrer que f est développable en série entière, il est inutile de
 f (n) (0) n
commencer par montrer que n!
z a un rayon de convergence non nul. À l’inverse,
si on montre que ce rayon de convergence est nul, alors cela montre que f n’est pas
développable en série entière.
 f(n) (0) n
Remarque 5. La série entière n!
z peut avoir un rayon de convergence non nul et
sa somme peut être différente de f (dans ce cas f n’est pas développable en série entière).
Par exemple, la fonction

e −1/x si x > 0
f : R → R x →
0 si x ≤ 0

est de classe C∞ sur R (voir l’exercice 3 page 79), et vérifie f (n) (0) = 0 pour tout n. La
 f (n) (0) n
série entière n!
z a donc un rayon de convergence infini, et pourtant pour tout
α > 0, f ne coı̈ncide pas avec la somme de cette série entière sur ] − α, α[.
 f(n) (0) n
– La série entière n!
z peut également avoir un rayon de convergence nul bien que

f soit de classe C sur R. On peut montrer par exemple que la fonction
 +∞ −t
+ e
ϕ : R → R x → dt,
0 1 + xt
 +∞
est de classe C∞ sur R + et vérifie ϕ(n)(x) = (−1) n n! 0 e−t /(1 + xt)n dt. En particulier
ϕ(n)(0) = (−1)n n!Γ(n +1) = (−1) n (n!)2 . La fonction paire f (x) = ϕ(x 2) est C ∞ sur R tout
 f (n) (0) n  ϕ(n) (0) 2n
entier, et n!
z = n!
z a un rayon de convergence
 nul. Plus généralement,
n
pour n’importe quelle suite (an ) (en particulier telle que an z a un rayon de convergence

nul) il existe une fonction f de classe C sur R telle que f (n) (0)/n! = an (théorème de
réalisation de Borel, voir le problème 16 page 295).

Exemple 2. La fonction f : R → R x → ex est de classe C ∞ sur R et vérifie f (n) (0) = 1


pour tout n. D’après la formule de Taylor-Lagrange, on a
 
xn xn+1 θx
∀n ∈ N, ∀x ∈ R, ∃θ ∈ ]0, 1[, R n(x) = f (x) − 1 + x + · · · + = e
n! (n + 1)!

donc |Rn (x)| ≤ |x| n+1 e|x| /(n + 1)!. On en tire Rn (x) → 0, i. e. (Rn
) tend simplement vers
0 sur R, et on en déduit grâce à la proposition précédente que z n/n! a un rayon de
convergence infini et que
+∞ n

x x
∀x ∈ R, e = .
n=0
n!

Cette expression nous inviteà prolonger la fonction exponentielle sur C tout entier par le
biais de la fonction entière z n/n!. Ceci fait l’objet de la sous-partie suivante.
4. SÉRIES ENTI ÈRES 253

Calcul d’un développement en série entière. Plusieurs méthodes permettent dans


la pratique de calculer un développement en série entière.
— On peut procéder directement à partir des développements en série entière des
fonctions usuelles (voir plus bas) à l’aide des opérations de somme, produit de
Cauchy, dérivation et intégration des séries entières.
— On peut rechercher une équation différentielle satisfaite par la fonction que l’on
veut développer en série entière ; on trouve ainsi les coefficients du développement
en série entière en procédant par identification sur les coefficients de chaque terme
de l’équation différentielle obtenue (voir par exemple l’exercice 3).
Développement en série entière des fonctions usuelles. En procédant comme
on l’a fait dans l’exemple 2, on obtient les développements en série entière suivants :

x2 xn
∀x ∈ R, ex = 1+x+ +··· + +···
2! n!
x3 x5 x2p+1
∀x ∈ R, sin x = x− + + · · · + (−1)p +···
3! 5! (2p + 1)!
x 2 x4 x2p
∀x ∈ R, cos x = 1− + + · · · + (−1)p +···
2! 4! (2p)!
x3 x5 x2p+1
∀x ∈ R, sh x = x+ + +··· + +···
3! 5! (2p + 1)!
x 2 x4 x2p
∀x ∈ R, ch x = 1+ + +··· + +···
2! 4! (2p)!
α α(α − 1) 2 α(α − 1) · · · (α − n + 1) n
∀x ∈ ] − 1, 1[, (1 + x) α = 1+ x+ x +··· + x +···
1! 2! n!
Le dernier développement est valable pour tout réel α fixé. En particulier, pour tout
x ∈ ] − 1, 1[,
1
= 1 − x + x 2 − · · · + (−1) n xn + · · ·
1+x
√ x 1 1·3 3 1 · 3 · · · (2n − 3) n
1+x = 1+ − x2 + x + · · · + (−1)n−1 x +···
2 2·4 2·4·6 2 · 4 · · · (2n)
1 1 1·3 2 1 · 3 · · · (2n − 1) n
√ = 1− x+ x + · · · + (−1) n x +···
1+x 2 2·4 2 · 4 · · · (2n)
1 1
En intégrant respectivement les développements en série entière de 1+x , , 1 , √1−x
1+x 2 1−x 2
1
2
,
√ 1 (qui sont connus grâce aux formules précédentes), on obtient
1+x 2

x2 x3 xn
∀x ∈ ] − 1, 1[, log(1 + x) = x − + + · · · + (−1)n−1 +···
2 3 n
x3 x5 x2n+1
∀x ∈ R, arctan x = x− + + · · · + (−1)n +···
3 5 2n + 1
x3 x5 x 2n+1
∀x ∈ ] − 1, 1[, argth x = x+ + +··· + +···
3 5 2n + 1
1 3 1 · 3 · · · (2n − 1)
∀x ∈ ] − 1, 1[, arcsin x = x+ x +··· + x 2n+1 + · · ·
2·3 2 · 4 · · · (2n) · (2n + 1)
1 3 1 · 3 · · · (2n − 1)
∀x ∈ ] − 1, 1[, argsh x = x− x + · · · + (−1)n x2n+1 + · · ·
2·3 2 · 4 · · · (2n) · (2n + 1)

Remarque 6. Les fonctions circulaires sont en fait correctement définies à partir des séries
entières (voir la partie qui suit).
254 4. SUITES ET SÉRIES

4.4. Fonctions classiques définies comme sommes de série entières


 n
Fonction exponentielle complexe. La série entière z /n! a un rayon de conver-
gence infini. On définit l’exponentielle complexe par
+∞ n

z z
∀z ∈ C, e = exp(z) = .
n=0
n!

Comme l’a vu, cette fonction coı̈ncide sur R avec la fonction exponentielle “classique”.
Comme sur R, on a e z1 +z2 = ez 1 ez 2 pour tout z1 , z2 ∈ C. Elle permet de définir la puissance
complexe d’un nombre a > 0, par az = exp(z log a) pour tout z ∈ C.
Les parties paire et impaire de ez sont les fonctions cosinus hyperbolique et sinus
hyperbolique définies sur C par
+∞ +∞
e z + e−z  z2n ez − e−z  z 2n+1
ch z = = et sh z = = .
2 n=0
(2n)! 2 n=0
(2n + 1)!
Fonctions circulaires. On définit les fonctions circulaires cosinus et sinus sur C par
+∞ +∞
eiz + e−iz  z 2n eiz − e−iz  z 2n+1
cos z = = (−1)n et sin z = = (−1)n .
2 n=0
(2n)! 2i n=0
(2 n + 1)!

Outre les formules trigonométriques usuelles, ces fonctions vérifient

∀z ∈ C, cos z + i sin z = e iz .
4.5. Exercices

Exercice 1. a) Soit a n z n une série entière de rayon de convergence R > 0 telle que
an > 0 pour tout n . Discuter en fonction du paramètre α ∈ R le rayon de convergence R
α n
de la série entière a nz .
b) Soient an z et bn zn deux séries entières de rayon de
n
convergence respectifs R et
  n
R . Que dire du rayon de convergence R de la série  entière  an bnn z ? (Cette dernière
n
série entière est appelée produit de Hadamard de an z et bn z .)

Solution. a) Les rayons de convergence R et R vérifient R = sup Γ, R = sup Γ , où

Γ = {r ≥ 0 | la suite (an rn) est bornée} et Γ = {r ≥ 0 | (a αn r n) est bornée}.

Maintenant, nous traitons plusieurs cas selon la position de α par rapport à 0.


— Supposons α > 0. L’égalité a αn rn = (an rn/α )α montre que la suite (aαn r n ) est bornée si et
seulement si (an rn/α) est bornée. Ainsi, r ∈ Γ  si et seulement si r1/α ∈ Γ. On en déduit
R = R1/α (avec par convention (+∞)1/α = +∞).
— Si α = 0, a αn = 1 pour tout n donc R = 1.
— Supposons α < 0. Si r ∈ ]0, R[, la suite (a n r n ) tend vers 0, donc la suite (aαn rnα )
tend vers +∞, donc r α ∈ Γ . Ceci étant vrai pour tout r ∈ ]0, R[, on en déduit que
]Rα , +∞[ ∩ Γ = ∅, donc R ≤ R α . On ne peut rien dire de plus en général. Par exemple,
si a2n = 22n et a 2n+1 = 2−2n pour tout n, on a R = 1/2 et R  = 2α < Rα.
b) Pour tout (r, r ) ∈ [0, R[ ×[0, R[, les suites (an r n ) et (bn rn ) sont bornées, donc (a nb n(rr )n )
est bornée. On voit donc que pour tout r ∈ [0, RR [, la suite (a nbn r n) est bornée. Ceci entraı̂ne
R ≥ RR .  
On ne peut rien dire de plus en général. Par exemple, les séries entières z 2n et z 2n+1 sont
de rayon de convergence 1, et leur produit de Hadamard est nul donc son rayon de convergence
est infini.
4. SÉRIES ENTI ÈRES 255

Exercice 2. Après avoir donné leur rayon de convergence R, sommer les séries entières
suivantes :
  xn
2 n
a) n x b) (pour x > 0)
n∈N n∈N
2n + 1
 xn  xn
c) pour x ∈ R d) cos(nθ ).

n(n + 2) n!
n∈N n∈N

Solution. a) En utilisant la règle de d’Alembert, on voit que le rayon de convergence est R = 1.


Pour sommer la série entière, on part de l’égalité
+∞

1
∀x ∈ R, |x| < 1, f(x) = = xn ,
1 − x n=0

d’où on déduit par dérivation que si x ∈ R, |x| < 1,


+∞
 +∞

 n 2 
xf (x) = nx et x f (x) = n(n − 1) xn .
n=0 n=0

Finalement, pour tout x ∈ R, |x| < 1,


+∞
 2x 2 x x2 + x
n2 xn = x2 f (x) + xf  (x) = + = .
n=0
(1 − x)3 (1 − x) 2 (1 − x) 3

Cette formule reste valable pour tout x ∈ C, |x| < 1 car on sait qu’une fraction rationnelle est
développable en série entière pour la variable complexe, et les coefficients sont déterminés si l’on
connaı̂t la somme de la série sur un voisinage de 0 dans R.
b) Toujours grâce à la règle de d’Alembert, on trouve R = 1. On note f la somme de la série
entière proposée. On a
+∞
 +∞

x2n+1 1
∀x ∈ [0, 1[, g(x) = xf (x 2) = et g (x) = x2n = ,
2n + 1 1 − x2
n=0 n=0

donc par intégration


  √  √ 
1 1+x g( x) 1 1+ x
∀x ∈ [0, 1[, g(x) = log d’où f (x) = √ = √ log √ .
2 1−x x 2 x 1− x

c) Ici R = 1 (règle de d’Alembert). Une décomposition en éléments simples fournit


 
1 1 1 1
∀n ∈ N, = −
n(n + 2) 2 n n+2
donc en notant f (x) la somme de la série entière proposée sur ] − 1, 1[,
+∞ +∞
 +∞
1  xn  xn log(1 − x) 1  xn+2
∀x ∈ ]−1, 1[ , x =
 0, f(x) = − =− − 2
2 n=1 n n=1
n+2 2 2x n=1 n + 2
 
log(1 − x) 1 x2 log(1 − x) log(1 − x) 1 1
=− − 2 − log(1 − x) − x − =− + 2
+ + .
2 2x 2 2 2x 2x 4

d) Comme | cos(nθ )/n!| ≤ 1/n!, on a R = +∞. Par ailleurs, on a cos nθ = (eniθ + e −niθ)/2 pour
tout n, donc
+∞
 +∞ +∞

 xn 1  (xe iθ ) n  (xe−iθ )n exp(xeiθ ) + exp(xe−iθ )
∀x ∈ C, cos nθ = + = .
n=0
n! 2 n=0
n! n=0
n! 2
256 4. SUITES ET SÉRIES

Exercice 3. a) Développer en série entière la fonction f : ] − 1, 1[ → R x → (arcsin x)2 .


b) Montrer que la fonction

arcsin x
f : ]0, 1[ → R x → 
x(1 − x)
coı̈ncide sur ]0, 1[ avec la somme d’une série entière, et calculer en les coefficients.

Solution. Tout repose sur la méthode de l’équation différentielle.


a) La fonction arcsinus est développable en série entière sur ] − 1, 1[ (c’est du cours),
 son ncarré
l’est donc également (par un produit de Cauchy). Ainsi, il existe une série entière an x qui
coı̈ncide avec f sur ] − 1, 1[. Pour calculer les an , on recherche une équation différentielle vérifiée
par f . On a
2 arcsin x
∀x ∈ ] − 1, 1[, f (x) = √ donc (1 − x2)f  (x)2 = 4(arcsin x) 2 = 4f (x),
1 − x2
et par dérivation de la dernière égalité
2(1 − x2 ) f (x)f (x) − 2xf  (x)2 = 4f (x) donc (1 − x2 )f  (x) − xf (x) = 2.
En reportant la somme de la série entière dans cette dernière équation, on obtient
 +∞   +∞ 
 
∀x ∈ ] − 1, 1[, (1 − x 2) n(n − 1)an xn−2 − x nan xn−1 = 2
n=2 n=1
+∞
  
donc (n + 1)(n + 2)an+2 − n2 an x n = 2.
n=0

Cette dernière égalité est vraie sur ] − 1, 1[ tout entier, on en déduit par identification des
coefficients (on peut, voir la conséquence du corollaire 1 page 249)
n2
2a 2 = 2 et ∀n ∈ N ∗, an+2 = an . (∗)
(n + 1)(n + 2)
Par ailleurs, a0 = f (0) = 0 et a1 = f (0) = 0. Avec (*), on trouve finalement
 2
∗ 22n−2 (n − 1)!
∀n ∈ N, a 2n+1 = 0 et ∀n ∈ N , a2n = .
n(2n − 1)!
Les coefficients d’indice impair sont nuls, ceci est
 cohérent car f est paire. Remarquez aupassagen
que le rayon de convergence de la série entière a 2n z 2n est égal à 1 (ceci car le rayon de a2n z
est égal à 1 par la règle de d’Alembert).
b) On sait que
 la fonction impaire arcsinus admet un développement en série entière sur ] − 1, 1[
de la forme an x 2n +1 . Donc
√ +∞ +∞
arcsin x 1 1  √ 1 
∀x ∈ ]0, 1[, f(x) =  = √ √ an ( x)2n+1 = √ an xn,
x(1 − x) 1 − x x n=0
1 − x n=0

et comme x → (1 − x)−1/2 coı̈ncide avec la somme d’une série entière sur ]0, 1[, on en déduit par
un produit de Cauchy que c’est aussi  le cas pour f .
Il existe donc une série entière bn xn dont la somme coı̈ncide avec f sur ]0, 1[ (au passage,
elle a forcément un rayon de convergence ≥ 1). Recherchons
√ une équation différentielle vérifiée
par f . Par dérivation de x(1 − x)f (x)2 = (arcsin x)2 , on tire

 2 2 arcsin x 1
∀x ∈ ]0, 1[, 2x(1 − x)f (x)f (x) + (1 − 2x)f (x) = √ √ = f (x),
1−x 2 x
donc
∀x ∈ ]0, 1[, 2x(1 − x)f  (x) + (1 − 2x)f (x) = 1,
4. SÉRIES ENTI ÈRES 257

et finalement
+∞
  
∀x ∈ ]0, 1[, b0 + (2n + 1)b n − 2nbn−1 xn − 1 = 0.
n=1
D’après le principe des zéros isolés, tous les coefficients de cette série entière sont nuls, ce qui
s’écrit
2n 2 · 4 · · · (2n)
b0 = 1, ∀n ∈ N∗ , bn = bn−1 donc ∀n ∈ N ∗ , bn = .
2n + 1 3 · 5 · · · (2n + 1)

 
 Exercice 4. Soient an xn et b n xn deux séries entières
 de rayon de convergence ≥ 1.
On supposeque bn > 0 pourtout n et que la série bn diverge. Pour tout n ∈ N, on
pose An = nk=0 ak et Bn = nk=0 bk.
a) S’il existe  ∈ C tel que
an An
lim =  ou lim = ,
n→+∞ bn n→+∞ Bn
montrer que ∞
an x n
lim n=0
∞ n
= .
n=0 bn x
x→1
x<1

b) Si on suppose simplement
A0 + · · · + A n−1
lim =  avec  ∈ C,
n→+∞ n

montrer que lim x→1 ∞ n
n=0 a n x = .
x<1

c) (Application.) Lorsque x tend vers 1 par valeurs inférieures, montrer les équivalents
+∞ √ +∞
 n +∞

n2 π a log(1 − x) 1
x ∼ √ , x ∼− (a ∈ N, a ≥ 2), (−1) n x4n+1 ∼ .
n=0
2 1−x n=0
log a n=0
2

Solution. a) Supposons tout d’abord lim n→∞ an/b n = . Soit ε > 0 et n 0 ∈ N tel que |an −bn | <
εbn pour tout n ≥ n 0 . Pour tout x ∈ [0, 1[, on a
  n −1  +∞ 
 +∞  +∞ n0 −1
   
0
n n n n
 (an − b n ) x  ≤ |an − b n |x + |a n − bn |x ≤ |an − bn | + ε b n x . (∗)
  n=n0
n=0 n=0 n=0 n=0
 
Or b n est une série à termes positifs divergente, donc lim x→1− ∞ n
n=0 bn x = +∞ (en effet,
∞  
pour tout N ∈ N, limx→1 − n=0 bn xn ≥ limx→1− N n
n=0 bn x =
N
n=0 b n), on en déduit
+∞  n −1
 0

∃λ ∈ ]0, 1[, ∀x ∈ ]λ, 1[, ε bnxn ≥ |an − b n |,


n=0 n=0

donc d’après (*)


+∞  +∞   ∞ 
    n=0 an xn 
   ∞
∀x ∈ ]λ, 1[,  (a n − bn) x n  ≤ 2ε b n xn c’est-à-dire  −   ≤ 2ε,
  n=0bn x n
n=0 n=0
d’où le résultat.
Si An /Bn → , tout repose sur la remarque suivante, conséquence d’un produit de Cauchy :
∞  +∞  +∞  +∞ ∞ +∞
n    n 
n=0 an x n n n n=0 bn x
= an x x = An x , = B n xn . (∗∗)
1−x 1−x
n=0 n=0 n=0 n=0
258 4. SUITES ET SÉRIES

L’étude du cas précédent montre que


∞
A n xn
lim  n=0
∞ n
= ,
n=0 B n x
x→1
x<1

et on en déduit le résultat avec (**).


b) Un produit de Cauchy donne
+∞
 ∞ +∞
x  x 
∀x ∈ [0, 1[, (A0 + · · · + An−1 ) x n = An xn = an xn
n=1
1 − x n=0 (1 − x) 2 n=0

et par ailleurs,
+∞
 +∞
  
n n−1 d 1 x
∀x ∈ [0, 1[, nx = x nx =x = .
n=1 n=0
dx 1−x (1 − x)2

Comme d’après la question précédente


∞ +∞

(A0 + · · · + An−1 ) xn
lim n=0  ∞ n
= , on en déduit lim a n xn = .
n=0 n x
x→1 x→1
x<1 x<1 n=0

  n2 √
c) Pour la série entière an xn = x , on a avec les notations précédentes An = 1 + [ n]
(où la notation [t] désigne la partie entière de t). Or
+∞
 1 · 3 · · · (2n − 1)
1
√ =1+ xn
1−x 2 · 4 · · · (2n)
n=1

et d’après la formule de Wallis


1 · 3 · · · (2n − 1) 1
bn = ∼ √ n → +∞
2 · 4 · · · (2n) nπ
(on
 peut aussi obtenir ce dernier  néquivalent en utilisant la formule de Stirling). On en déduit que
b n diverge et que Bn ∼ √1π k=1 k
√1 (sommation d’équivalents, voir le théorème 5 page 210).

Un équivalent de cette dernière somme s’obtient facilement par comparaison série-intégrale, en


écrivant
 n+1  n  k+1 n n  k
 n  n
dt dt 1 dt dt 1 √
√ = √ ≤ √ ≤ √ = √ , d’où √ ∼ 2 n,
1 t k=1 k t k=1 k k=1 k−1 t 0 t k=1
k
√ √ √
et finalement Bn ∼ 2 n/ π. Ainsi, lim n→∞ A n/B n = π/2, et d’après a) on en déduit
∞ √ +∞
 √
n=0 a nx n π n 2 π
lim−  ∞ n
= , ce qui s’écrit aussi x ∼ √ .
x→1 n=0 bn x 2 n=0
2 1 − x
 n
 an
Pour la série entière an x = x , on a An = 1 + [log n/ log a] ∼ log n/ log a. Or si
b n = 1/n on a
+∞
 1 1
b n x n = − log(1 − x) et Bn = 1 + + · · · + ∼ log n,
n=1
2 n

donc An /Bn → 1/ log a. On en déduit d’après a) que lorsque x → 1−,


+∞
 +∞
 +∞
an n 1  log(1 − x)
x = an x ∼ bn x n = − .
n=0 n=0
log a n=0 log a
 
Pour an xn = (−1) n x4n+1 , on a A8n+k = 1 pour 1 ≤ k ≤ 4 et A 8n+k = 0 pour 5 ≤ k < 8,
donc (A0 + A1 + · · · + An−1 )/n → 1/2, d’où l’équivalent recherché d’après la question b).
4. SÉRIES ENTI ÈRES 259


Exercice 5 (Théorème de Liouville). Soit an zn une série entière dont le rayon
de convergence est infini. Soit f : C → C la somme de cette série entière.
a) Si la fonction entière f est bornée sur C, montrer que f est constante.
b) Plus généralement, s’il existe un polynôme P à coefficients positifs tel que |f (z )| ≤
P (|z |) pour tout z ∈ C, montrer que f est un polynôme.

Solution. a) C’est immédiat si l’on connaı̂t la formule de Cauchy (voir le théorème 4 page 250).
En désignant par M un majorant de |f | sur C, on a en effet
 2π
n 1
∀n ∈ N, ∀r > 0, anr = f (re iθ ) e−niθ dθ donc |an | rn ≤ M.
2π 0

Si n ∈ N∗ , la majoration |a n| ≤ M/rn est vraie pour tout r > 0 et on peut donc faire r → +∞,
ce qui entraı̂ne an = 0 pour tout n ∈ N∗ , d’où le résultat.
b) Notons m = deg(P ). Si m = 0, la réponse a été apportée dans la résolution de la question
précédente. Sinon, on pose
+∞

g (z ) = anz n−m,
n=m

somme d’une série entière de rayon de convergence infini. La fonction g vérifie

f (z ) − a 0 − a 1 z − · · · − am−1 z m−1
∀z ∈ C∗ , g(z) = ,
zm
et comme m = deg(P ), ceci montre que g est bornée sur C tout entier. Comme on l’a vu plus
haut, g est donc constante, donc an = 0 dès que n > m, d’où le résultat.
Remarque. On aurait pu aussi résoudre la question a) de l’exercice en utilisant la formule
de Parseval au lieu de la formule de Cauchy.
– La considération des valeurs complexes de la variable permet seule de comprendre des
phénomènes qui seraient surprenant si on se confinait à la variable réelle. Par exemple, la
fonction sinus est développable en série entière sur R, bornée sur R, mais elle n’est pas
constante.
– Le théorème de Liouville (question a)) admet une généralisation étonnante, connue
sous le nom de théorème de Picard : toute fonction entière f qui évite deux valeurs est
forcément constante. Autrement dit, s’il existe deux valeurs distinctes a et b de C telles
que f (z ) =
 a et f (z ) =
 b pour tout z ∈ C, alors f est constante.


Exercice 6. Soit Φ la somme d’une série entière a n z n dont le rayon de convergence
R est non nul. 
a) Montrer que la série entière an zn /n! 
a un rayon de convergence infini.
b) On note ϕ la somme de la série entière anz n /n!. Montrer
 +∞
∀z ∈ C, |z | < R, Φ(z) = ϕ(zx)e −x dx.
0

Solution. a) Fixons r0 tel que 0 < r0 < R, de sorte que la suite (a n r0n) est bornée (elle tend
même vers 0). Pour tout r > 0, on a an rn /n! = (a n n n
n r0 )(q /n!) (avec q = r/r 0 ), et comme (q /n!)
est bornée (le rayon de convergence de la série z n /n! est infini), on en déduit que la suite
n
(anr /n!) est bornée.
260 4. SUITES ET SÉRIES

b) Soit z ∈ C tel que |z | < R. Commençons par montrer l’existence de l’intégrale. On fixe r tel
|z | < r < R. La suite (an r n) est bornée, donc si M désigne un majorant de (|an |rn ), on a
 
 an (zx)n  n n
∀x ≥ 0, ∀n ∈ N,   = |a n r n| |zx/r | ≤ M (qx) , q=
|z |
< 1, (∗)
 n!  n! n! r
ce qui par sommation entraı̂ne |ϕ(xz )| ≤ M eqx pour tout x ≥ 0. Ainsi, |ϕ(xz)e−x | ≤ M e(q−1)x
pour tout x ≥ 0 et comme q < 1, l’intégrale proposée converge bien.
Il suffit maintenant d’écrire
 +∞  +∞  +∞  an (xz )n

 an z n  +∞
+∞ +∞

−x −x n −x
ϕ(xz)e dx = e dx = x e dx = an zn,
0 0 n! n! 0
n=0 n=0 n=0
 +∞ n −x
car 0 x e dx = Γ(n + 1) = n! pour tout n ∈ N. Nous avons bien le droit d’inverser les
signes de sommations car d’après (*) les sommes partielles de la série vérifient la majoration
N 
 a (zx)n  N
(qx) n −x
 n 
∀N ∈ N∗ ,  e−x  ≤ M e ≤ e(q−1)x
 n!  n!
n=1 n=1

et x → e(q−1)x est intégrable sur R+ , donc l’hypothèse de domination du théorème de convergence


dominée est vérifiée.

Exercice 7. Soient α1 , . . . , αp des entiers naturels non nuls premiers entre eux dans
leur ensemble. Pour tout n ∈ N, on note S n le nombre de solutions (n1 , . . . , np ) ∈ Np de
l’équation
α1 n1 + · · · + αp np = n.
Donner un équivalent de Sn lorsque n → +∞. (Indication : interpréter S n comme le
coefficient d’une série entière qui s’exprime simplement en fonction de α1, . . . , α p.)
Solution. Considérons la série entière définie par le produit de Cauchy des séries entières
   
 
 zα 1n 1 , . . . ,  zαp np .
n1 ∈N np ∈N

Toute l’astuce est de remarquer que le coefficient de z n dans ce produit de Cauchy est le nombre
de manière de combiner les puissances de z de chaque terme du produit pour que leur somme
fasse n. En d’autres termes, on a
 +∞   
+∞
  +∞
 1 1
F (z ) = Sn z n = z α1 n1 · · ·  zαp np  = α
··· ,
n=0 n =0 n =0
1−z 1 1 − zαp
1 p

et toutes les séries entières correspondantes ont leur rayon de convergence égal à 1. La fonction
F (z ) est une fraction rationnelle, dont les pôles se trouvent aux racines α1-ièmes,. . .,αp -ièmes
de l’unité. Le pôle z = 1 est de multiplicité p, et tous les autres pôles ont une multiplicité < p
(en effet, si ωα 1 = · · · = ωα p = 1 avec ω = e 2iaπ/b une racine de l’unité et a ∧ b = 1, alors b
divise α1 a, . . . , αp a donc b divise α1 , . . . , αp d’après le théorème de Gauss, donc b = 1 car les αi
sont premiers entre eux dans leur ensemble). On peut donc écrire la décomposition en éléments
simples de F sous la forme
A   a 1,ω ap−1,ω

F (z ) = + G(z ), G(z) = +··· + , ( ∗)
(1 − z) p ω−z (ω − z ) p−1
ω∈Π
où Π désigne un sous-ensemble fini des racines de l’unité et les ak,ω des constantes complexes
(notez que 1 ∈ Π). On trouve la constante A par les techniques usuelles, en écrivant
   
p 1 1
(1 − z) F (z ) = ··· ,
1 + z + · · · + zα 1−1 1 + z + · · · + z αp−1
4. SÉRIES ENTI ÈRES 261

ce qui en faisant z = 1 dans cette expression fournit A = (α1 · · · αp ) −1 . Maintenant, comme


+∞
 (n + k − 1)!
1 1
= ω −n−k zn (∗∗)
(ω − z ) k (k − 1)! n=0 n!

(voir la page 251), on en déduit que si |ω | = 1, le coefficient de z n dans cette série entière est
un O (nk−1). Ainsi, d’après (*), le coefficient de z n dans la série entière définissant G(z ) est un
O (np−2 ). On en déduit, avec (*) et (**) que
A 1 np−1
Sn = (n + p − 1)(n + p − 2) · · · (n + 1) + O (np−2 ) ∼ .
(p − 1)! α1 · · · αp (p − 1)!

Remarque. En calculant toute la décomposition en éléments simples de F (z ), on peut


obtenir une formule exacte (mais compliquée) pour Sn. En exercice, vous pourrez calculer
exactement le nombre de solutions entières a, b, c de 5a + 3b + 2c = 10000.

Exercice 8 (Théorème de S.Bernstein sur les s éries entières). Soient a > 0


et f : ]−a, a[ → R une fonction de classe C ∞. On suppose que
∀k ∈ N, ∀x ∈ ]−a, a[ , f (2k) (x) ≥ 0.
Montrer que f est développable en série entière sur ]−a, a[. (Indication : commencer par
traiter la fonction F (x) = f (x) + f (−x).)

Solution. Il suffit de montrer que pour tout b ∈ ]0, a[, la fonction f est développable en série
entière sur ]−b, b[ (en effet, les coefficients du développement en série entière sur ] − b, b[ ne
dépendent pas de b). Fixons donc b ∈ ]0, a[.
Suivons l’indication et commencer par développer la fonction F : x → f (x) + f (−x). Cette
fonction est paire donc pour tout k ∈ N, F (2k+1) (0) = 0, donc en appliquant la formule de Taylor
avec reste intégral, on obtient, pour tout n ∈ N et pour tout x ∈ [0, b]

 x
x2 x2n (x − t)2n+1 (2n+2)
F (x) = F (0)+ F  (0)+· · ·+ F (2n)(0)+R n(x), R n(x) = F (t) dt.
2! (2n)! 0 (2n + 1)!
Remarquons que pour tout k ∈ N, F (2k)(0) = 2f (2k) (0) ≥ 0, donc la formule précédente entraı̂ne
0 ≤ Rn (b) ≤ F (b) pour tout n. Pour montrer que Rn(x) tend vers 0 lorsque 0 ≤ x < b, nous
comparons sa valeur à R n(b) en écrivant
 x   x 2n+1  x (b − t)2n+1
x − t 2n+1 (b − t)2n+1 (2n+2)
0 ≤ Rn (x) = F (t) dt ≤ F (2n+2) (t) dt
0 b − t (2n + 1)! b 0 (2n + 1)!
 x 2n+1  x  2n+1
≤ Rn (b) ≤ F (b)
b b
(on a utilisé la majoration (x − t)/(b − t) ≤ x/b < 1 pour tout t ∈ [0, x], qui provient du caractère
décroissant de t → xb−−tt sur [0, x]). Comme 0 ≤ x/b < 1, on en déduit lim n→∞ Rn (x) = 0, ce qui
s’écrit aussi
+∞
 x2n
∀x ∈ [0, b[ , F (x) = F (2n)(0).
n=0
(2 n)!
La fonction F est paire, ce résultat vaut donc sur ]−b, b[.
Il nous reste à montrer le résultat pour f . Fixons x ∈ ]−b, b[. Pour tout n ∈ N, on écrit
 x
x 2n+1 (2n+1) (x − t)2n+1 (2n+2)
f (x) = f (0) + · · · + f (0) + rn (x) avec rn (x) = f (t) dt.
(2n + 1)! 0 (2n + 1)!
262 4. SUITES ET SÉRIES

Comme 0 ≤ f (2n+2) (t) ≤ f (2n+2) (t)+f (2n+2) (−t) = F (2n+2) (t) pour tout t, on a |rn (x)| ≤ Rn (|x|)
donc limn→∞ rn (x) = 0. Ainsi, en notant
p
 xk
∀p ∈ N, Sp (x) = f (k)(0),
k!
k=0
nous venons de montrer que
lim S2n+1 (x) = f (x). (∗)
n→∞
Or
x2n (2n) 1 x2n (2n)
S2n (x) − S2n−1 (x) = f (0) = F (0) donc lim S2n (x) − S2n−1 (x) = 0,
(2n)! 2 (2n)! n→+∞

et on en déduit avec (*) que lim n→+∞ S2n (x) = f (x). Donc d’après (*), on a limn→∞ Sn (x) =
f (x) c’est-à-dire
+∞
 f (n)(0)
∀x ∈ ]−b, b[ , f (x) = xn .
n=0
n!
D’où le résultat.
Remarque. Considérons la fonction f : ]−π/2, π/2[ → R x → tan x. La fonction f 
satisfait les hypothèses précédentes comme on le vérifie facilement, et on en déduit que
f  , donc f , est développable en série entière sur ]−π/2, π/2[.


Exercice 9 (Inverse d’une série enti ère). Soit 1 + n≥1 an zn une série entière
de rayon de convergence non nul, et S la somme de cette série entière sur son disque de
convergence. Montrer que 1/S est développable en série entière autour de l’origine.
Solution. Commençons par montrer le lemme suivant.

Lemme 1. Une série entière un z n a un rayon de convergence non nul si et seulement s’il
existe q > 0 tel que |un| ≤ q pour tout n ∈ N ∗ .
n

En effet : 
Condition nécessaire. Notons r le rayon de convergence de un z n. Soit r  tel que 0 < r < r.
n n
On a un r  → 0, donc il existe M ≥ 1 tel que |un r | ≤ M pour tout n ∈ N∗ , donc
 n
∗ 1 M
∀n ∈ N , |u n | ≤ M  ≤ qn avec q =  .
r r

Condition suffisante. La suite |un (1/q)n| est bornée d’après les hypothèses donc le rayon de
convergence de un z n est supérieur à 1/q, d’où le résultat.

Résolvons maintenant l’exercice. L’hypothétique développement en série entière bn z n
vérifie, s’il existe   +∞ 
+∞
 
n n
1+ anz bn z = 1, (∗)
n=1 n=0
donc par un produit de Cauchy
b 0 = 1, ∀n ∈ N ∗, bn = −a1b n−1 − · · · − an−1b 1 − a nb 0.
Définissons (bn ) comme l’unique suite vérifiant ces récurrences. D’après le lemme précédent, il
existe q > 0 tel que |an | ≤ qn pour tout n ∈ N ∗. Posons r = 2q. On va montrer par récurrence
sur n que |bn| ≤ r n pour tout n. Pour n = 0 c’est vrai car b0 = 1, et pour passer du rang n − 1
au rang n, on écrit
n n
 n
 1 n
|bn| ≤ |ak| |b n−k| ≤ q k r n−k = r ≤ r n.
2k
k=1 k=1 k=1
4. SÉRIES ENTI ÈRES 263


Ainsi, bn z n a un rayon de convergence non nul d’après le lemme. Les relations vérifiées
par (bn ) montrent que sur l’intersection des disques de convergence, l’égalité (*) est bien vérifiée,
d’où le résultat.
Remarque. Si S s’annule, on peut montrer que le rayon de convergence du développement
en série entière de 1/S est égal au plus petit des modules des zéros de S. N’essayez pas
de prouver ce résultat, il ne s’obtient de manière naturelle que dans le cadre général des
fonctions analytiques (voir la remarque de l’exercice 13 page 265).


Exercice 10 (Théor  ème d’Abel). Soit an zn une série entière de rayon de conver-
gence ≥ 1 telle que a n converge. On note f la somme de cette série entière sur le disque
unité. On fixe θ0 ∈ [0, π/2[ et on pose
∆θ 0 = {z ∈ C, |z | < 1 et ∃ρ > 0, ∃θ ∈ [−θ0 , θ 0 ], z = 1 − ρeiθ }
(voir la figure
sous-suite
ci dessous). Montrer que
+∞

lim f (z ) =
z→1
an .
z∈∆θ
0 n=0

∆θ 0 −→
0 1

Figure 1. La région ∆θ 0 . L’écartement du secteur angulaire est 2θ0 .

 n
Solution. Notons S = +∞ n=0 an , Sn = k=0 ak et Rn = S − S n pour tout n ∈ N. Pour majorer
|f (z ) − S |, on va effectuer une transformation d’Abel en écrivant a n = Rn−1 − Rn pour tout n.
Soit z ∈ C∗, |z | < 1. Pour tout N ∈ N ∗, on a
 N  N N−1 N
   
n n n+1
anz − Sn = (R n−1 − R n)(z − 1) = R n (z − 1) − Rn (z n − 1)
n=0 n=1 n=0 n=1
N−1
 N−1

= R n (zn+1 − z n) − R N (z N − 1) = (z − 1) Rn zn − R N (z N − 1),
n=0 n=0
et en faisant tendre N vers +∞ on en déduit
+∞

f (z ) − S = (z − 1) Rn z n. (∗)
n=0
Fixons maintenant ε > 0, puis N ∈ N tel que |Rn | < ε pour tout n > N . D’après (*), pour tout
z ∈ C, |z | < 1,
N   +∞  N 
    |z − 1|
 
|f (z ) − S| ≤ |z − 1|  Rn z n  + ε|z − 1| |z |n ≤ |z − 1| |R n| + ε (∗∗)
  1 − |z|
n=0 n=N +1 n=0

Soit z ∈ ∆θ0 , de sorte que z = 1 − ρe avec ρ > 0 et |ϕ| ≤ θ0 . On a |z | 2 = 1 − 2ρ cos ϕ + ρ2 , et


lorsque ρ ≤ cos θ0 on a la majoration


|z − 1| |z − 1| ρ 2 2 2
= (1 + |z |) = (1 + |z |) ≤ ≤ = .
1 − |z| 1 − |z|2 2ρ cos ϕ − ρ 2 2 cos ϕ − ρ 2 cos θ0 − cos θ 0 cos θ0
264 4. SUITES ET SÉRIES


Si on choisit maintenant α > 0 tel que α N n=0 |Rn | < ε, on voit donc que si z ∈ ∆θ0 et
|z − 1| ≤ inf {α, cos θ0 } la majoration (**) entraı̂ne
 
2 2
|f (z ) − S| ≤ ε + ε =ε 1+ ,
cos θ 0 cos θ 0
d’où le résultat.

Remarque. En appliquant ce résultat à la série (−1)n /(2n + 1), on en déduit
+∞
 +∞
 (−1) n
(−1)n π
= lim xn = x→1
lim arctan x = arctan 1 = .
n=0
2n + 1 x→1
x<1 n=0
2n + 1 x<1
4

De même, on montrerait ∞ n=1 (−1)
n−1
/n = log 2.
 
– Si la série a n converge absolument, le résultat est évident (en effet, an z n converge
alors normalement sur |z| ≤ 1, donc est continue sur |z| ≤ 1, donc en 1).
– La réciproque de ce théorème est fausse. Par exemple, on a
+∞
 1 1
lim (−1) n z n = lim =
z→1
|z|<1 n=0
z→1
|z|<1
1+z 2

et pourtant, (−1)n diverge. Cependant, si a n = o(1/n) (voir l’exercice suivant) ou
mieux, si a n = O(1/n) (voir le problème 27 page 308), la réciproque est vraie.


Exercice 11 (Théor ème taubérien faible). Soit an z n une série entière de rayon
de convergence 1 et f la somme de cette série entière sur le disque unité. On suppose que
∃S ∈ C, lim
x→1
f (x) = S.
x<1
 +∞
Si an = o(1/n), montrer que an converge et que n=0 an = S.
n
Solution. Pour tout n ∈ N, on note Sn = k=0 ak . On a
n
 +∞

∀n ∈ N∗ , ∀x ∈ ]0, 1[ , Sn − f (x) = ak (1 − xk ) − ak xk
k=1 k=n+1
k k−1
et comme (1 − x ) = (1 − x)(1 + x + · · · + x ) ≤ k(1 − x) pour 0 < x < 1, on en déduit
n
 +∞
 k |ak | k sup k>n k |ak |
|Sn − f (x)| ≤ (1 − x) k |a k| + x ≤ (1 − x)M n + ,
n n(1 − x)
k=1 k=n+1

où M désigne un majorant de la suite (k |ak |) (elle est bien majorée car elle tend vers 0). Fixons
maintenant ε > 0 tel que ε < 1. L’inégalité précédente entraı̂ne
  ε  supk>n k |ak |

∀n ∈ N∗ , S
 n − f 1 −  ≤ Mε + ,
n ε
donc si N0 est choisi tel que supk>N 0 k |a k | < ε2 (on peut car kak → 0), on en déduit
  ε 

∀n ≥ N0 , S
 n − f 1 −  ≤ M ε + ε = (M + 1)ε.
n
D’après les hypothèses, f (x) tend vers S lorsque x → 1−, donc il existe N 1 ≥ N0 tel que
|f (1 − ε/n) − S | < ε pour tout n ≥ N 1. Ainsi,
   
 ε    ε 
∀n ≥ N 1, |S n − S| ≤ Sn − f 1 −  + f 1 − − S ≤ (M + 1)ε + ε = (M + 2)ε.
n n
On en déduit que (Sn ) converge vers S, d’où le résultat.
4. SÉRIES ENTI ÈRES 265

Remarque. Ce résultat est une réciproque partielle du théorème d’Abel (voir l’exercice
précédent). Il reste vrai en supposant seulement an = O(1/n) (cf. problème 27 page 308).


Exercice 12. Soit f la somme d’une série entière an z n de rayon de convergence ≥ 1
telle que a n ∈ Z pour tout n ∈ N. On suppose que f est bornée sur le disque unité.
Montrer que f est une fonction polynôme.

Solution.
 Tout découle de l’égalité de Parseval qui entraı̂ne, pour tout r ∈ ]0, 1[, la convergence
de |an |2 r2n et
+∞
  2π
2 2n 1
∀r ∈ ]0, 1[ , |an| r = |f (reiθ )|2 dθ.
n=0
2π 0

Si M désigne un majorant de |f | sur le disque unité, on en conclut que pour tout N ∈ N ∗ ,


N
 M2
∀r ∈ ]0, 1[ , |an |2 r2n ≤ .
n=0

N
En
 faisant r → 1− , on en déduit n=0 |a n| 2 ≤ M 2 /2π , et ceci pour tout N ∈ N∗ , donc la série
2
|an| est bornée, donc convergente. On en déduit que (an ) tend vers 0, et comme les an sont
entiers, ceci entraı̂ne que tous les an sont nuls à partir d’un certain rang. D’où le résultat.

Exercice 13. Pour tout r ∈ ]0, +∞], on note D (r ) = {z ∈ C | |z| < r}. Soit R ∈ ]0, +∞]
et f : D (R) → C une application dérivable par rapport à la variable complexe sur D (R)
et telle que f  (z) soit continue sur D (R) (on rappelle que f est dérivable par rapport à
la variable complexe en z0 si (f (z0 + u) − f (z0))/u converge lorsque u ∈ C tend vers 0 en
restant non nul ; la limite est alors notée f (z 0)).
1/ a) Soit I un intervalle de R et γ : I → D (R) une application de classe C 1 . Montrer
que f ◦ γ est de classe C 1 sur I et calculer (f ◦ γ ).
b) Soit r > 0 et g une fonction définie et continue de {z ∈ C, |z | = r } dans C. Montrer
que l’application
 2π
r g(reit ) e it
D (r) → C z → dt
2π 0 reit − z
est la somme d’une série entière qui converge sur D (r ).
2/ Montrer que
 2π
r f (re it) eit
∀r ∈ ]0, R[ , ∀z ∈ D (r ), f (z) = dt. (∗)
2π 0 re it − z
En déduire que f est la somme d’une série entière qui converge sur D (R) (Indication :
r
 2π f (z +λ(reit−z )) eit
montrer que la fonction λ → 2π 0 reit −z
dt est constante sur [0, 1]).

Solution. 1/ a) On procède comme pour la dérivation par rapport à la variable réelle. Soit
t ∈ I . La fonction f est dérivable par rapport à la variable complexe en γ (t), ce qui s’écrit
f (γ (t) + u) = f (γ (t)) + uf (γ (t)) + o(u) lorsque u → 0 (u ∈ C).
On en conclut que lorsque v est un nombre réel tendant vers 0

f (γ (t + v )) = f (γ (t) + v γ  (t) + o(v)) = f (γ (t)) + (v γ (t) + o(v))f (γ (t)) + o(v )


= f (γ (t)) + v γ  (t)f  (γ (t)) + o(v ),
266 4. SUITES ET SÉRIES

donc f ◦ γ est dérivable en t et (f ◦ γ )(t) = γ (t)f (γ (t)). On déduit de cette dernière expression
que (f ◦ γ ) est continue, donc f ◦ γ est de classe C 1.
b) Il suffit d’écrire que pour tout z ∈ D (r ) on a
 2π  2π  +∞ 
1 g(reit ) 1  zn
dt = g(reit ) e −int dt
2π 0 1 − z/(re it) 2π 0 n=0
r n

+∞    +∞  2π
1  n 1 2π
it −int

n 1
= z g(re )e dt = an z , an = g(re it )e−int dt (∗∗)
2π n=0 rn 0 n=0
2πr n
0

où le coefficient an est indépendant de z . itOn na bien le droit d’échanger les signes de sommation
car si |z | < r, la série de fonctions g(re )(z /r )e n −int (de la variable t) converge normalement
sur [0, 2π ] puisque g est continue, donc bornée, sur le compact |z | = r.
2/ Montrons (*). Fixons r ∈ ]0, R[ et z ∈ D (r). On considère la fonction
 2π
r f (z + λ(reit − z )) eit
ϕ : [0, 1] → C λ → dt.
2π 0 reit − z
Il s’agit de montrer que ϕ(1) = f (z ). La valeur ϕ(0) est un cas particulier de la formule (**)
lorsque t → g (reit ) est la fonction constante égale à f (z), donc

r
 2π
f (z )eit   f (z )  2π
+∞ 
−int
ϕ(0) = f (z ) it − z
dt = n
e dt z n = f (z ),
2π 0 re n=0
2πr 0
 2π
car 0 e −int dt = 0 si n = 0, égal à 2π si n = 0. Il faut donc montrer ϕ(0) = ϕ(1). D’après la
question 1/a), ϕ est dérivable et
 2π
r
∀λ ∈ [0, 1], ϕ (λ) = f (z + λ(re it − z )) e it dt. (∗∗∗)
2π 0
Or, toujours d’après la question a), on a
∂  
f (z + λ(re it − z) = iλreitf  (z + λ(re it − z )),
∂t
donc d’après (***)
1  2π
∀λ, 0 < λ < 1, ϕ  (λ) = f (z + λ(reit − z)) = 0.
2πiλ 0

Ainsi, ϕ est constante donc ϕ(0) = ϕ(1), d’où (*).


La fonction f est continue sur |z | = r car elle y est dérivable par rapport à la variable
complexe. Grâce à (*) et à 1/b), on en déduit que pour tout r ∈ ]0, R[, f est la somme d’une série
entière an z sur D(r ). Or les coefficients an ne dépendent pas de r (ce sont les f (n) (0)/n!).
n

Ce développement en série entière est donc valable sur D (R) tout entier.
Remarque. (Petite digression sur les fonctions analytiques.) Les séries entières rentrent
dans le contexte plus général de l’élégante théorie des fonctions analytiques.
Définition 1. Une fonction f : Ω → C (où Ω est un ouvert de C) est dite analytique
dans Ω si pour tout z0 ∈ Ω, il existe un disque ouvert ∆ : |z − z0 | < r contenu dans Ω
tel que
+∞

∀z ∈ ∆, f (z) = cn (z − z 0)n ,
n=0
où le second membre est une série entière en z − z0 convergente dans ∆.
Elle est dite holomorphe dans Ω si elle est continûment dérivable par rapport à la
variable complexe dans Ω.
5. S ÉRIES DE FOURIER 267

Par exemple, la fonction f : z → 1/z est analytique dans C∗. En effet, pour tout
z0 ∈ C ∗ , et pour tout z que |z − z0 | < |z 0|, on a
+∞  
1 1 1  1 n
f (z ) = = − (z − z0 ) n .
z0 1 + (z − z 0)/z0 z0 n=0 z0
Le résultat de l’exercice montre que toute fonction f holomorphe sur un ouvert Ω de
C est analytique sur Ω. En effet, si z0 ∈ Ω et si r > 0 est tel que D (z0 , r) (disque ouvert
de centre z0 de rayon r ) vérifie D(z0 , r) ⊂ Ω, la fonction g : D(r ) → C z → f (z 0 + z )
est holomorphe sur D (r) donc g est développable en série entière sur D(r ) d’après 2/.
Réciproquement, on montre que toute fonction analytique est holomorphe (c’est plus
facile).
En particulier, la somme d’une série entière est analytique sur son disque de conver-
gence (résultat non évident a priori).
On montre facilement que la composée de deux fonctions holomorphes est holomorphe.
En particulier, si f est la somme d’une série entière qui converge dans un disque D(r ) et
ne s’y annule pas, alors 1/f est holomorphe dans D (r ), donc développable en série entière
sur D (r) d’après le résultat de l’exercice. On obtient ainsi une version plus forte que celle
obtenue à l’exercice 9 page 262, car on a des renseignements sur le rayon de convergence
r de l’inverse (r est le plus petit des modules des zéros de f ).

5. Séries de Fourier
5.1. Séries trigonométriques
Polynômes trigonométriques.
Définition 1. On appelle polynôme trigonométrique de degré ≤ N (N ∈ N) de la variable

réelle x toute fonction de la forme x → N n=−N c n e
inx
(c n ∈ C).
inx
Compte tenu de la relation e = cos nx + i sin nx, il revient au même de dire qu’un po-
lynôme trigonométrique est une fonction de la forme x → 2 + N a0
n=1 (an cos nx + bn sin nx)
où l es a
n , bn sont des nombres co m pl exes reliés aux coefficients cn par

∀m ∈ N, am = c m + c−m et ∀m ∈ N ∗, bm = i(c m − c −m).


N
Remarque 1. Un polynôme trigonométrique P : x → −N cn einx est une fonction conti-
1
 2π
nue 2π-périodique. On a pour tout n la relation cn = 2π 0
P (x)e−inx dx, qui montre que
P est nul si et seulement si cn = 0 pour tout n.
Séries trigonométriques.
Définition 2. On appelle  série trigonométrique une série defonctions de la variable
inx −inx
réelle x de la forme c0 + n∈N ∗(cn e + c−n e ) ; on la note n∈Z cn einx .
Compte tenu de la relation einx = cos nx + i sin nx, il revient  au même de dire qu’une
a0
série trigonométrique est une série de fonctions de la forme 2 + n∈N∗ (an cos nx+bn sin nx)
où l es na, b n sont des nombres complexes reliés aux coefficients c n par les relations vues
précédemment.

Remarque 2. La série trigonométrique n∈Z c n einx converge si et seulement si la suite de

fonctions des sommes partielles ( N n=−N c n e
inx
)N converge, et dans ce cas, la somme de
+∞ inx
la
 série est notée n=−∞ cn e (convention deCauchy). Avec cette convention, la série
n∈Z c n e
inx
peut converger sans que lim p→+∞ pn=q cne inx existe.
q→−∞
268 4. SUITES ET SÉRIES

   
Proposition 1. Si n∈N |cn | et n∈N |c −n | (resp |an | et |bn|) convergent, la série
trigonométrique
  a0  
inx
cn e resp. + (an cos nx + b n sin nx) (∗)
2 ∗
n∈Z n∈N

converge normalement sur R. Sa somme définit une fonction continue et 2π -périodique


sur R.
Proposition 2. Si les suites (cn )n∈N et (c−n )n∈N (resp. (an ) et (b n)) sont réelles, décrois-
santes et tendent vers 0, alors la série trigonométrique (*) converge simplement sur
R2π Z, et uniformément sur tout intervalle de la forme [2kπ + α, 2(k + 1)π − α] (avec
0 < α < π et k ∈ Z).
Démonstration. Il suffit de montrer la convergence uniforme sur [α, 2π − α] pour tout α > 0 fixé.
Pour tout n ∈ N, on note En (x) = nk=0 eikx . On a
 
 i(n+1)x 
1 − e  2 1 1
∀x ∈ [α, 2π − α], |E n (x)| =  ≤ = ≤ . (∗∗)
 1 − eix  |1 − eix| | sin(x/2)| sin(α/2)
Maintenant, une transformation d’Abel fournit
n
 n−1

ikx
∀x ∈ [α, 2π − α], Sn (x) = c ke = (ck − ck+1 )Ek (x) + c n En(x). (∗∗∗)
k=0 k=0

D’après (**) et la décroissance de (cn ), la série (c k − c k+1)Ek (x) converge normalement sur
[α, 2π − α ]. Par ailleurs, d’après (**) et le fait que cn → 0, la suite de fonctions (cnE n(x))
converge uniformément vers 0. On en conclut avec (***) que

la série de fonctions cn einx converge uniformément sur [α, 2π − α]. (∗∗∗∗)
n∈N

Par symétrie, la série n∈N c −ne −inx converge aussi uniformément sur [α, 2π − α ]. On en déduit

que n∈Z c neinx converge uniformément sur [α, 2π − α ]. Ceci est vrai pour tout α ∈ ]0, π[, on
en conclut qu’il y a convergence simple sur ]0, 2π [.
 inx , on en
En prenant respectivement les parties réelles et imaginaires de la série n∈N c ne
 
déduit avec (****) que les séries n∈N cn cos nx et n∈N c n sin nx convergent uniformément sur
[α, 2π − α] pour tout α > 0 dès que (cn ) est une suite réelle décroissante qui tend vers 0. Ainsi, la

série a20 + n∈N∗ (an cos nx + bn sin nx) converge uniformément sur [α, 2π − α] pour tout α > 0,
d’où le résultat. 

Remarque 3. On peut montrer que si une série trigonométrique n∈Z cn einx converge
simplement sur R, alors c n → 0 lorsque |n| → +∞ (théorème de Cantor-Lebesgue, voir la
question 2/ du problème 29 page 312). Si de plus la limite simple de cette série est nulle
en tout point, alors cn = 0 pour tout n (théorème de Cantor, voir la question 3/ du même
problème).
5.2. Définition d’une série de Fourier
Définition 3. Soit f : R → C une application 2π -périodique et continue par morceaux
sur R. On appelle coefficients de Fourier de f les nombres complexes définis par
 2π
1
∀n ∈ Z, cn (f ) = f (t)e−int dt,
2π 0
 
1 2π ∗ 1 2π
∀n ∈ N, a n (f ) = f (t) cos nt dt, ∀n ∈ N , bn (f ) = f (t) sin nt dt.
π 0 π 0
5. S ÉRIES DE FOURIER 269

On appelle série de Fourier associée à f la série trigonométrique


 a 0(f ) 
cn (f )e inx ou encore + (a n(f ) cos nx + bn (f ) sin nx).
2 ∗
n∈Z n∈N

Notez que les coefficients de Fourier vérifient les relations


∀n ∈ N, an (f ) = cn (f ) + c−n (f ), ∀n ∈ N∗ , b n(f ) = i(c n(f ) − c−n (f ))
et que les deux dernières séries trigonométriques sont égales, car
c n (f )einx + c−n (f )e−inx = a n (f ) cos nx + bn (f ) sin nx.
Remarque 4. — Les intégrandes étant 2π -périodiques, on peut remplacer l’intervalle
d’intégration [0, 2π ] par n’importe quel intervalle de longueur 2π.
— On utilise en général les coefficients an (f ), bn(f ) lorsque f est à valeurs réelles.
— Si f est paire (resp. impaire), les coefficients bn(f ) (resp. a n (f )) sont nuls.
— Une série trigonométrique qui converge uniformément sur R est égale à sa série de
Fourier.
— Si f est T -périodique, on peut également définir les coefficients de Fourier de f par
  
1 T 2π
c n (f ) = f (t) exp −i nt dt,
T 0 T
 T     
2 2π 2 T 2π
an (f ) = f (t) cos nt dt, bn (f ) = f (t) sin nt dt.
T 0 T T 0 T
et avec ω = 2π/T , la série de Fourier associée à f est
 a (f ) 
cn (f )e iωnx ou encore 0 + (an (f ) cos ωnx + bn (f ) sin ωnx).
2 ∗
n∈Z n∈N

Dans la suite de cette partie, la période sera toujours T = 2π mais les résultats se
généralisent aisément par normalisation pour toute période T > 0.
L’espace D. Nous aurons besoin de la définition suivante.
Définition 4. Une application f : [a, b] → C ( où [ a, b] est un segment de R) est dite de
classe C n par morceaux sur [a, b] s’il existe une subdivision a = x0 < x1 < · · · < xp = b de
[a, b] telle que pour tout i ∈ {0, . . . , p − 1}, la restriction de f à l’intervalle ]xi , xi+1 [ est
prolongeable par continuité sur [xi , xi+1 ] en une fonction de classe Cn sur [xi , xi+1 ].
Une application f : R → C est dite de classe C n par morceaux si la restriction de f a
tout segment de R est de classe C n par morceaux.
Une fonction f de classe C n par morceaux admet donc en tout point x une limite à
gauche et à droite, que nous notons respectivement f (x−) et f (x+).
Notation. On note D l’e.v des fonctions de R dans C, 2π-périodiques, continues par
morceaux, et telles que
f (x−) + f (x+)
∀x ∈ R, f (x) = .
2
5.3. Convergence en moyenne quadratique
Structure préhilbertienne de D . Sur l’e.v D, l’application
 2π
1
(f, g ) → f, g  = f (t)g (t) dt
2π 0
définit un produit scalaire
 et fait de D un espace préhilbertien complexe, muni de la norme
hermitienne f 2 = f, f .
270 4. SUITES ET SÉRIES

En notant, pour tout n ∈ Z, en l’application x → e inx, (e n)n∈Z constitue une famille


libre orthonormale de D vis-à-vis du produit scalaire défini précédemment.
Proposition 3. Soient n ∈ N et f ∈ D . Le sous-espace vectoriel P n = Vect(e k)−n≤k≤n
vérifie Pn ⊕ Pn⊥ = D. La projection orthogonale p n sur Pn vérifie
n

sn = ck (f )ek = pn(f ),
k=−n

et de plus
 2π n

1
inf f − g 22 = f − s n 22 = 2
|f (t)| dt − |ck(f )| 2 . (∗)
g∈Pn 2π 0 k=−n

Démonstration. Cette proposition est en fait une conséquence directe des résultats généraux sur
les espaces préhilbertiens (voir tome Algèbre) appliqués à D . Nous le redémontrons dans notre
cas.
On remarque que ck(f ) = e k, f = ek , s n si −n ≤ k ≤ n, donc ek , f − s n  = 0 pour
−n ≤ k ≤ n. Autrement dit, f − sn ∈ Pn ⊥ . Comme f = sn + (f − s n) avec sn ∈ Pn (et que
est vrai pour tout f ∈ D ), on en déduit D = Pn + P ⊥
ceci  ⊥
n . Par ailleurs Pn ∩ Pn = {0} car si
g = −n≤k≤n λk ek ∈ P n , alors λ k = g, e k = 0. Finalement on a bien D = Pn ⊕ Pn⊥.

On a montré que f = sn + (f − s n ) avec f − s n ∈ Pn⊥, donc sn = pn (f ). Les éléments sn et


f − sn sont orthogonaux, donc sn 22 + f − sn 22 = f  22 , ce qui fournit la dernière égalité de
(*). Par ailleurs, pour tout g ∈ Pn , on a
f − g 22 = (f − sn) + (s n − g) 22 = f − sn  22 + s n − g 22 ≥ f − sn 22,
et ceci fournit la première égalité de (*). 

Remarque 5. — On interprète (*) en disant que parmi les polynômes trigonométriques


de degré ≤ n, sn est celui qui
nse rapproche le plus de f en moyenne quadratique.
2 1 2π
— La formule
 (*) montre que −n |ck(f )| ≤ 2π 0
|f (t)|2 dt pour tout n > 0. Ainsi,
la série n∈Z |cn (f )|2 converge et la somme de cette série vérifie
+∞
  2π
2 1
|cn(f )| ≤ |f (t)|2 dt (inégalité de Bessel).
−∞
2π 0

En fait, cette inégalité est une égalité (dans ce cas on parle d’égalité de Parseval),
comme il est énoncé dans le théorème qui suit.
Égalité de Parseval.
 Théorème 1 ( Égalité de Parseval).  Soit f : R2 → C une fonction
 2π-périodique et
2 2
continue par morceaux. Alors les séries n∈Z |c n (f )| , |a n (f )| , |b n(f )| convergent
et on a
+∞ +∞ +∞  2π
2 |a 0(f )| 2 1  2 1 2 1
|cn (f )| = + |an (f )| + |bn(f )| = |f (t)|2 dt.
−∞
4 2 n=1 2 n=1 2π 0
Démonstration. Quitte à changer la valeur de f en ses discontinuités, on peut supposer f ∈ D (la
valeurs
 des intégrales faisant intervenir f ne change pas). Il suffit de prouver le résultat sur la série
2 2 2
n∈Z n (f )| car les relations liant les a n(f ), b n (f ) aux cn (f ) entraı̂nent |cn (f )| + |c −n(f )| =
|c
(|an (f )| 2 + |bn (f )|2)/2 pour tout n ∈ N ∗ et |c0 (f )|2 = |a0 (f )|2 /4.
La proposition précédente donne
 2π +∞

1
|f (t)| 2 dt − |cn (f )| 2 = inf f − g 22 , (∗∗)
2π 0 −∞
g∈P
5. S ÉRIES DE FOURIER 271

où P désigne l’e.v des polynômes trigonométriques.


Si f est continue, on sait (conséquence du théorème de Fejér, voir le problème 25 page 306)
qu’il existe une suite de polynômes trigonométriques (g n) qui converge uniformément vers f sur
R, et donc f − gn  22 tend vers 0, ce qui montre inf g∈P f − g 22 = 0, d’où l’égalité de Parseval
d’après (**).
Si f ∈ D n’est pas continue, on peut montrer (c’est facile et peu intéressant) que pour
tout ε > 0, il existe une fonction f ∈ D continue telle que f − f 2 < ε. Comme on l’a vu
précédemment, il existe g ∈ P tel que f − g 2 < ε, donc finalement f − g  < 2ε. Ceci est
possible pour tout ε > 0, donc inf g∈P f − g 22 = 0, d’où le résultat d’après (**). 

Remarque 6. — On peut montrer que l’égalité de Parseval reste vraie pour toute
fonction f 2π -périodique, continue par morceaux et intégrable sur ]0, 2π [.
— Si f est une fonction 2π -périodique et continue par morceaux, on a lim |n|→+∞ cn(f ) =
0 (conséquence de la convergence de n∈Z |cn (f )| 2). On retrouve ainsi le lemme de
Lebesgue dans ce cas particulier (voir l’exercice 6 page 157).
— L’égalité de Parseval entraı̂ne qu’une fonction continue 2π -périodique qui a tout
ses coefficients de Fourier nuls est nulle.
— Soit f : R → C une fonction continue et 2π -périodique. D’aprèsl’égalité de
Parseval et la proposition 3, on a limn→+∞ f − s n 2 = 0 où sn = n−n ck (f )ek .
On en déduit que si la série de Fourier de f converge uniformément sur R, alors
f est égale à sa série de Fourier (sa fonction limite g vérifie f − g 2 = 0 et g est
continue — limite uniforme de fonctions continues — donc f = g ).
5.4. Le théorème de Jordan-Dirichlet
Le résultat qui suit est le résultat principal sur les séries de Fourier.

Théorème 2 (Jordan-Dirichlet). Soient f : R → C une fonction 2π -périodique,


continue par morceaux sur R et t0 ∈ R tels que la fonction
f (t 0 + h) + f (t0 − h) − f (t0+) − f (t 0 −)
h →
h

est bornée au voisinage de 0. Alors n∈Z c n(f )e int0 converge et on a
+∞
 f (t0+) + f (t0 −)
cn (f )e int 0 = .
−∞
2

Démonstration. Quitte à effectuer


n une translation t → t + t0 , on peut supposer t0 = 0. Pour
tout n ∈ N, on note sn = −n c k(f ). Il s’agit de montrer que la suite (u n ) définie par un =
sn − (f (0+) + f (0−))/2 tend vers 0.
On a
n  π  π n
−ipt
2πsn = f (t)e dt = f (t)D n (t) dt, où ∀t ∈ R, Dn (t) = eikt . (∗)
k=−n −π −π k=−n

Le polynôme trigonométrique Dn (t) s’appelle noyau de Dirichlet, on le rencontre souvent lors


de l’étude de séries de Fourier. Il peut être calculé explicitement :
ei(2n+1)t − 1 sin((2n + 1)t/2)
∀t ∈ R2π Z, D n (t) = e −int = .
eit − 1 sin(t/2)
Par ailleurs, Dn est une fonction paire, donc
0  π  π
f (t)D n (t) dt = f (−t)D n (t) dt d’où 2πsn = (f (t) + f (−t)) D n(t) dt.
−π 0 0
272 4. SUITES ET SÉRIES

On en déduit finalement
π  π  
(2n + 1)t
2πun = (f (t) + f (−t) − f (0+) − f(0−)) D n (t) dt = g (t) sin dt,
0 0 2
où g(t) = (f (t) + f (−t) − f (0+) − f (0−))/ sin(t/2) est continue par morceaux sur ]0, π] et bornée
sur un voisinage de 0 d’après les hypothèses. La fonction g est donc intégrable sur ]0, π] et le
lemme de Riemann-Lebesgue (voir l’exercice 6 page 157) entraı̂ne limn→+∞ 2πun = 0, d’où le
résultat. 

 Corollaire 1. Si f est 2π -périodique et de classe C 1 par morceaux, alors pour tout x ∈ R,


f (x+) + f (x−)
la série de Fourier de f converge en ce point x vers . En particulier, si f
2
est continue en x, la série de Fourier de f en x converge vers f (x).

Remarque 7. L’hypothèse C 1 par morceaux est importante. Il existe en effet des fonctions
continues dont la série de Fourier diverge (voir l’exercice 4 page 275).
Convergence uniforme de la série de Fourier.
Lemme 1. Soit f : R → C une fonction 2π-périodique, continue et C1 par morceaux. On
définit ϕ : R → C par ϕ(t) = f  (t) si f est dérivable en t et ϕ(t) = (f  (t+) + f (t−))/2
sinon. Les coefficients de Fourier de ϕ vérifient cn (ϕ) = in c n(f ) pour tout n ∈ Z.
Démonstration. Soit 0 = x 0 < x 1 < · · · < x p = 2π une subdivision de [0, 2π ] telle que f soit C1
sur [xk−1 , xk] pour tout k . En intégrant par parties, on a pour tout k
 xk  x  xk
−int −int k
ϕ(t) e dt = f (t)e + in f (t) e−int dt,
x k−1 x k−1 xk−1

puis la fonction f étant continue, on obtient en sommant cette relation sur k ,


 2π   2π  2π  2π
cn (ϕ) = ϕ(t) e −int dt = f (t)e−int + in f (t) e −int dt = in f (t) e−int dt = in cn (f ).
0 0 0 0

 Théorème 3. Si f : R → C est une fonction 2π -périodique, continue et C 1 par morceaux,


alors la série de Fourier de f converge normalement vers f sur R.
Démonstration. En reprenant les notations du lemme précédent, on a c n (ϕ) = incn (f ). Ainsi,
   
 cn(ϕ)  1 1
∀n ∈ Z ,∗ 
|cn (f )| =   ≤ 2
|cn (ϕ)| + 2 ,
n  2 n
 
et comme n∈Z |cn (ϕ)|2 converge (voir l’égalité de Parseval), on en déduit que n∈Z |cn (f )|
converge, d’où le résultat avec le corollaire 1. 

5.5. Exercices
Exercice 1. Soit f : R → R la fonction 2π -périodique égale à 1 − x 2 /π2 sur [−π, π].
Calculer les coefficients de Fourier de f . En déduire les valeurs de
+∞
 +∞
 +∞

1 1 1
, , .
n=1
n2 n=1
(2 n − 1) 2
n=1
n 4

Solution. La fonction f est paire. Les coefficients bn = bn (f ) sont donc nuls. Par ailleurs,
  
1 π t2 4
a0 = a 0(f ) = 1 − 2 dt =
π −π π 3
5. S ÉRIES DE FOURIER 273

et
 
π
  π
∗ 1 t2 2 4
∀n ∈ N , an = a n(f ) = 1 − 2 cos nt dt = − 3 t 2 cos nt dt = (−1) n+1 2 2
π −π π π 0 nπ
(après une double intégration par parties).
La fonction f est continue et C 1 par morceaux. Sa série de Fourier converge donc simplement
(et même uniformément) vers f , ce qui s’écrit
+∞ +∞
x2 a0  2 4  cos nx
∀x ∈ [−π, π], f(x) = 1 − 2 = + a n cos nx = − 2 (−1) n . (∗)
π 2 n=1
3 π n=1 n2

— En faisant x = π dans (*), on trouve


+∞ +∞
2 4  1  1 π2
0= − 2 donc = .
3 π n=1 n2 n=1
n2 6

— En faisant x = 0 dans (*), on trouve


+∞ +∞
2 4  (−1) n  (−1)n π2
1= − 2 d’où = − ,
3 π n=1 n2 n=1
n2 12

donc
+∞
+∞ +∞
  
 1 1  1  (−1) n 1 π2 π2 π2
2
= 2
− 2
= + = .
n=1
(2n − 1) 2 n=1
n n=1
n 2 6 12 8

— Enfin l’égalité de Parseval s’écrit


+∞ +∞
4 1  16 8  1 π4
+ 4 4
= donc 4
= .
9 2 n=1 n π 15 n=1
n 90

Exercice 2. Soit α ∈ RZ. On désigne par fα l’application 2π -périodique sur R telle


que
∀t ∈ ]−π, π] , f α (t) = cos αt.

a) Calculer la série de Fourier de f α. En déduire


+∞

1 1
∀t ∈ Rπ Z, cotan t = + 2t 2 2π 2
.
t n=1
t − n

b) Montrer alors
 

+∞
t2
 ∞
 N

∀t ∈ ]−π, π[ , sin t = t 1− 2 2 où = lim .
n=1
nπ n=1
N→+∞
n=1

c) Montrer
+∞

1 1
∀t ∈ ]−π, π[ , t =
 0, 2
= .
sin t −∞ (t − nπ ) 2
274 4. SUITES ET SÉRIES

Solution. a) L’application f α est paire donc bn (f α ) = 0 pour tout n ∈ N∗. Par ailleurs,
 
1 π 1 π
∀n ∈ N, an (f α ) = cos αt cos nt dt = [cos(α + n)t + cos(α − n)t] dt
π −π π 0
 
1 sin(α + n)π sin(α − n)π 2α sin απ
= + = (−1)n .
π α+n α−n π (α2 − n2)
La fonction fα est continue et de classe C1 par morceaux, donc la série de Fourier de f α converge
simplement (et même uniformément) vers fα sur R, ce qui entraı̂ne
+∞
sin αt  2α sin απ
∀t ∈ [−π, π], cos αt = + (−1)n 2 − n2 )
cos nt,
απ n=1
π (α

ce qui en faisant t = π et en divisant par sin απ donne


+∞
1 2α
cotan απ = + .
απ n=1 π (α − n2 )
2

Ceci est vrai pour tout α ∈ RZ, d’où le résultat en remplaçant α par t/π.
b) Soit x ∈ ]0, π[. On définit f : [0, x] → R par f (t) = cotan t − 1/t si t = 0, f (0) = 0. La
formule établie à la question précédente montre que
+∞
 2t
∀t ∈ [0, x], f (t) = .
n=1
t2 − n 2 π 2

Comme cette série de fonctions converge normalement sur [0, x], on peut écrire
 x  x
+∞ +∞
  
2t sin x x2
f (t) dt = 2 − n 2π 2
autrement dit log = log 1 − 2 2 .
0 n=1 0 t x n=1

En prenant l’exponentielle de part et d’autre de cette dernière égalité, on voit que le produit
infini existe bien et que
sin x 
+∞
x2

= 1− 2 2 ,
x n=1
n π
d’où le résultat pour 0 < x < π. Comme les fonctions en présence sont impaires et nulles en 0,
on en déduit le résultat pour tout x ∈ ]−π, π[.
c) L’égalité
 
2t 1 1 d 2t 1 1
2 2 2
= + entraı̂ne 2 2 2
=− 2
− .
t −n π t − nπ t + nπ dt
t −n π (t − nπ ) (t + nπ )2
 d  2t

Ainsi, si on fixe x ∈ ]0, π[, la série de fonctions converge normalement sur
dt t 2 − n2π 2
n≥1
[0, x]. En appliquant le théorème de dérivabilité des séries de fonctions, on en conclut que
+∞
 
2t 1
t → 2 − n2 π2
est dérivable sur [0, x], sa dérivée est t → − .
t (t − nπ ) 2
n=1 n∈Z ∗

En dérivant l’identité obtenue dans la question a) au point x, on déduit


−1 1  1
2
+ 2
= − .
sin x x ∗
(x − nπ )2
n∈Z

Ceci vaut pour tout x ∈ ]0, π[. Les fonctions en présence étant paires, cette relation vaut sur
]−π, π [ {0} d’où le résultat
5. S ÉRIES DE FOURIER 275

Exercice
 2π 3. Soit f : R → C une fonction 2π -périodique de classe C1. On suppose que
0
f (t) dt = 0. Montrer
 2π  2π
2
|f (t)| dt ≤ |f (t)|2 dt,
0 0
et caractériser l’égalité.
 2π
Solution. Le coefficient de Fourier c0 (f ) est nul car 0 f (t) dt = 0. En appliquant l’identité
c n(f ) = in cn (f ), avec l’égalité de Parseval appliquée aux fonctions f et f , on trouve donc
 2π     2π
1  2 1
2
|f (t)| dt = 2
|c n (f )| ≤ 2 2
n |cn(f )| = |cn (f )| = |f (t)|2 dt, (∗)
2π 0 ∗ ∗
2π 0
n∈Z n∈Z n∈Z

ce qui prouve l’inégalité voulue.


Il y aura égalité si et seulement si la seule inégalité de (*) est une égalité, c’est-à-dire si et
seulement si |cn (f )| 2 = n 2|cn (f )|2 pour tout n ∈ Z ∗ , ce qui équivaut à c n(f ) = 0 pour tout n tel
que |n| ≥ 2. Or f est de classe C1 , donc sa série de Fourier converge (uniformément) vers f . En
résumé, l’égalité se produira si et seulement f est de la forme f (t) = aeit + be −it, a, b ∈ C.

Exercice 4 (Une fonction continue 2π-périodique dont la série de Fourier


diverge en 0). a) Soit f : R → R la fonction paire, 2π -périodique, telle que
+∞
 1  3 x
p
∀x ∈ [0, π], f (x) = 2
sin (2 + 1) .
p=1
p 2

Vérifier l’existence et la continuité de f sur R.


b) Pour tout ν ∈ N, on pose
 π q

(2ν + 1)t
∀n ∈ N, a n,ν = cos nt sin dt, ∀q ∈ N, s q,ν = a i,ν .
0 2 i=0

Calculer explicitement les a n,ν , montrer que sq,ν ≥ 0 pour tout (q, ν ), et montrer l’existence
d’une constante B > 0 telle que s ν,ν > B log ν pour tout ν ∈ N ∗ .
c) Montrer que la série de Fourier de f diverge en 0.

Solution. a) La série converge normalement sur [0, π], f est donc bien définie et continue sur
[0, π]. On la définit sur [−π, 0[, par f (x) = f (−x). La fonction f est continue sur [−π, π]. De
plus f (−π ) = f (π ), on en déduit que le prolongement de f en une fonction 2π -périodique f sur
R est continu sur R.
b) Le calcul des a n,ν est facile, on a
      
1 π 2ν + 1 2ν + 1
a n,ν = sin + n t + sin − n t dt
2 0 2 2
 
1 1 1 ν + 1/2
= + = .
2 ν + n + 1/2 ν − n + 1/2 (ν + 1/2)2 − n 2
Ainsi, on a a n,ν ≥ 0 pour n ≤ ν , donc s q,ν ≥ 0 pour q ≤ ν.
Pour le cas q > ν, on remarque que les an,ν , sont, au facteur 2/π près, les coefficients de
1
Fourier an (g ν ) de la fonction paire gν (t) = | sin((ν +1/2)t)|. Cette dernière est continue
∞et C par
morceaux, sa série de Fourier converge donc vers gν . En particulier, on a a0,ν /2 + n=1 an,ν =
π
2 gν (0) = 0, donc la suite (sq,ν ) q∈N converge vers a 0,ν /2. Or an,ν est positif pour n ≤ ν , négatif
pour n > ν, donc (sq,ν )q est décroissante à partir de l’indice q = ν . Comme elle converge vers
a0,ν /2, on en déduit que s q,ν ≥ a 0,ν /2 ≥ 0 pour tout q > ν.
276 4. SUITES ET SÉRIES

Il nous reste à obtenir la minoration de s ν,ν. On écrit pour tout ν ∈ N∗ ,


ν ν  n  ν
ν + 1/2 (ν + 1/2) dt (ν + 1/2) dt 1
sν,ν ≥ 2 2
≥ 2 2
= 2 2
= log(4ν + 3),
n=1
(ν + 1/2) − n n=1 n−1
(ν + 1/2) − t 0 (ν + 1/2) − t 2
donc sν,ν ≥ (log ν )/2 pour tout ν ∈ N ∗.
c) Comme f est paire, les coefficients de Fourier b n(f ) sont nuls. Par ailleurs, la parité de f
entraı̂ne
 +∞  π  
2 π 2 1 p3 t
∀n ∈ N, an (f ) = f (t) cos nt dt = sin (2 + 1) cos nt dt,
π 0 π p=1 p2 0 2

(on a le droit de changer les signes de sommation car la série converge normalement sur [0, π]),
donc
+∞ n +∞
2 1 π   1
∀n ∈ N, a n (f ) = a p3 −1 , donc S n = ak (f ) = s 3 .
2 n,2 p −1
π p=1 p2 n,2 2 p=1
p
k=0

Comme les sq,ν sont positifs, et que s ν,ν ≥ (log ν )/2, on en déduit
1 1 p 3−1 p3 − 1
∀p ∈ N, S 2p 3−1 ≥ s p3 −1 p3−1 ≥ log(2 ) = log 2.
p2 2 ,2 2p2 2p 2

Ceci montre que S 2p 3−1 → +∞ lorsque p → +∞, donc la série an(f ) diverge. Autrement dit,
la série de Fourier de f en 0 diverge.
Remarque. Cet exemple d’une fonction continue 2π-périodique dont la série de Fourier
diverge en 0 est dû à Fejér. On peut montrer de manière non constructive que de telles
fonctions existent à partir du théorème de Banach-Steinhaus (voir l’exercice 8 page 425).

Exercice 5. Montrer que la fonction


+∞
 sin nx
f : R → R x →
n=1
n2
est bien définie, qu’elle est 2π-périodique
 et qu’elle est continue. Montrer que les coeffi-
cients de Fourier de f sont tels que n∈N∗ |nbn(f )| 2 converge, mais que pourtant f n’est
pas dérivable en 0.

Solution. La série de fonctions (sin nx)/n2 converge normalement sur R, donc f est définie et
continue sur R. Chaque somme partielle de la série est 2π -périodique, donc f est 2π -périodique.
Comme la série trigonométrique définissant f converge  normalement, f est égale à sa série
de Fourier, donc b n(f ) = 1/n 2 pour tout n ∈ N∗ , et la série |nb n(f )| 2 converge donc.
Montrons que f n’est pas dérivable en 0. Soit N un entier naturel non nul. L’inégalité de
concavité sin u ≥ 2u/π sur [0, π/ 2] entraı̂ne
 π  N
f (x)  sin nx 1  sin nx N
2 1 1  sin nx
∀x ∈ 0, , = + 2
≥ + 2
. (∗)
2N x n=1
nx n n x n=1
π n x n
n>N n>N
Par ailleurs, une transformation d’Abel fournit
 sin nx    n

1 1
= Sn(x) − avec Sn (x) = sin kx,
n2 n2 (n + 1)2
n>N n>N k=0

ce qui grâce à la majoration (on utilise le fait que S n(x) est la partie imaginaire de nk=0 eikx)
 
 1 − ei(n+1)x   sin((n + 1)x/2)  1 1 π
   ≤
|Sn(x)| ≤  = ≤ = ,
 1 − eix   sin(x/2)  sin(x/2) (2/π )(x/2) x
5. S ÉRIES DE FOURIER 277

donne    
  sin nx  π  1 1 π 1 π 1
 
 ≤ − = ≤ .
 n 2  x n 2 (n + 1)2 x (N + 1) 2 x N 2
n>N n>N
Avec (*) on en déduit
 
 π  f (x) 2
N
 1 π
∀x ∈ 0, , ≥ − .
2N x π n=1
n N 2 x2
N 1
Ainsi, en posant x N = π/(2N), on a f (xN )/xN ≥ π2 n=1 n − 4π . La suite (xN ) tend vers 0 et
1
comme n diverge, (f (x N ) − f (0))/(xN − 0) = f (x N )/xN diverge lorsque N → +∞. D’où la
non-dérivabilité de f en 0.

Exercice 6 (Phénom ène de Gibbs). On considère le signal carré ϕ, qui est la fonc-
tion 2π -périodique, égale à 1 sur ]0, π[, à 0 sur ]π, 2π[, et qui vaut 1/2 en ses points de
discontinuité.
a) Calculer la série de Fourier de ϕ, montrer qu’elle converge simplement vers ϕ et même
uniformément sur tout intervalle fermé ne contenant pas les discontinuités de ϕ.
b) Montrer que les sommes partielles d’indice impair s2n−1(t) de la série de Fourier de ϕ
admettent la représentation intégrale

1 1 t sin 2ns
s2n−1 (t) = + ds.
2 π 0 sin s

c) Calculer les points critiques de s2n−1 sur [0, π] et la valeur de son maximum.
d) Montrer que ce maximum converge lorsque n tend vers l’infini vers le nombre

1 1 π sin s
M= + ds,
2 π 0 s
puis conclure (on admet qu’une valeur approximative à 10−3 près est M ≈ 1, 089).

Solution. a) Le signal carré ϕ est C1 par morceaux, et comme ϕ(x) = 12 (ϕ(x−) + ϕ(x+)) en ses
discontinuités, la série de Fourier de ϕ converge simplement vers ϕ. Elle se calcule facilement et
on obtient
+∞
1 2  sin(2ν − 1)t
ϕ(t) = + , t ∈ R.
2 π 2ν − 1
ν=1
Les coefficients de Fourier formant une suite décroissante, la convergence est uniforme sur tout
intervalle fermé ne contenant pas les discontinuités de ϕ d’après la proposition 2 page 268.
t
b) Partant de la représentation intégrale sin(2 ν−1)t
2ν−1 = 0 cos(2ν − 1)s ds, on peut écrire
 t n
1 2
s 2n−1(t) = + Cn (s) ds, Cn (s) = cos(2k − 1)s. (∗)
2 π 0
k=1
On calcule Cn(s) à partir de la partie réelle d’une somme d’exponentielles complexes,
n     
 i2ns
i(2k−1)s is e −1 ins sin ns cos ns sin ns sin 2ns
C n(s) =  e = e = e = = .
e2is − 1 sin s sin s 2 sin s
k=1
On en déduit le résultat en remplaçant cette dernière expression dans (*).
c) La représentation intégrale précédente donne s2n−1 (t) = π1 sin(2nt)/ sin t (et s 2n−1 (0) = 2n/π
par continuité), donc s2n−1 s’annule sur [0, π] en t = xk = kπ/(2n), 0 < k ≤ 2n. Montrons que
son maximum est atteint en x 1. Pour 1 ≤ k ≤ n, on a

1 x 2k sin 2ns
s2n−1(x 2k ) − s2n−1 (x2k−1 ) = ds,
π x2k−1 sin s
278 4. SUITES ET SÉRIES

et comme l’intégrande est négative sur [x2k−1 , x 2k ] on en déduit s 2n−1 (x2k ) < s2n−1(x 2k−1).
Le maximum est donc atteint sur l’un des s2n−1 (x2k−1) pour 1 ≤ k ≤ n. Maintenant pour
1 ≤ k < n, on a
  
1 x2k 1 1
s 2n−1(x 2k+1 ) − s2n−1(x2k−1 ) = sin 2ns − π ds
π x2k−1 sin s sin(s + 2n )
x x x
expression que l’on obtient en découpant en deux l’intégrale x 2k+1 = x 2k + x 2k+1 et en
2k−1 2k−1 2k
effectuant le changement de variable s → s + π/(2n) dans la deuxième. La fonction sinus étant
croissante sur [0, π], et comme sin 2ns est négatif sur [x2k−1 , x 2k ], la dernière intégrande est
négative, donc s2n−1 (x2k+1) < s2n−1 (x2k−1 ). Finalement, ceci montre que le maximum est atteint
en t = x1 = π/(2n) et vaut

1 1 π/(2n) sin 2ns
sup s2n−1 (t) = Mn = + ds.
0≤t≤π 2 π 0 sin s

d) Le changement de variable t = 2ns dans l’intégrale précédente donne



1 1 π sin t
Mn = + dt.
2 π 0 2n sin(t/(2n))
Lorsque x → 0, on a sin x ∼ x. Donc pour tout ε > 0, il existe α > 0 tel que x/(1 + ε) ≤ sin x ≤
x/(1 − ε) pour x ∈ [0, α]. Ainsi, si n > π/(2α) on a t/(1 − ε) ≤ 2n sin(t/(2n)) ≤ t/(1 + ε) sur
[0, π], donc  
1 1 π sin t 1 1 π sin t
+ (1 − ε) dt ≤ M n ≤ + (1 + ε) dt.
2 π 0 t 2 π 0 t
On en déduit que (Mn ) converge vers M lorsque n → ∞.
Concluons. Nous venons de montrer que le maximum des sommes partielles s2n−1 conver-
geait vers un nombre M ≈ 1, 089 qui est strictement plus grand que le maximum de ϕ. Ainsi,
les sommes partielles convergent simplement vers le signal carré ϕ mais pas uniformément.
Remarque. Le graphe ci-contre illustre le phénomène
de Gibbs (pour la somme partielle t → s 2n−1 (t) avec
1
n = 40) de la série de Fourier du signal carré.
– Le phénomène de Gibbs fut observé par Michelson
en 1898 lorsqu’il développa un système mécanique ca-
pable de tracer la série de Fourier d’un signal. Alors
que Michelson soupçonnait un défaut dans la fabrica-
tion de sa machine, Gibbs montra l’année suivante que 0 π
le phénomène était d’origine mathématique.

Exercice 7 (Théor ème de S. Bernstein sur les séries de Fourier). Soit


f : R → C une fonction 2π -périodique. On suppose que
∃α ∈ ]0, 1[ , ∃C > 0, ∀(u, v ) ∈ R 2 , |f (u) − f (v )| ≤ C |u − v |α
(une telle fonction est dite α -höldérienne).
a) Pour tout n ∈ N∗, on pose ρ n = (|cn (f )|2 + |c−n (f )|2 )1/2 . Montrer que
+∞
  π
2 2 1
∀h ∈ R, 4 ρ n sin nh = |f (x + h) − f (x − h)|2 dx.
n=1
2π −π
 C πα
b) En déduire, pour tout ν ∈ N∗ , la majoration ρn ≤ .
2 2ν (α−1/2)
2ν−1 <n≤2 ν
c) Si α > 1/2, montrer que la série de Fourier de f converge normalement vers f sur R.
5. S ÉRIES DE FOURIER 279

Solution. a) Remarquons déjà que f est continue. Ensuite, fixons h ∈ R et considérons la


fonction fh : x → f (x + h) − f (x − h). Les changements de variable u = x + h et u = x − h
conjugués au caractère 2π-périodique des intégrandes entraı̂nent
 π  π
1
∀n ∈ Z, f (x + h)e−inx dx = e inh c n (f ) et f (x − h)e−inx dx = e −inh c n(f )
2π −π −π

et comme e inh − e −inh = 2i(sin nh), ceci montre que les coefficients de Fourier de fh vérifient
c n(fh ) = 2i(sin nh)cn (f ). On conclut en appliquant l’égalité de Parseval à f h.
b) En appliquant l’égalité précédente à h = π/2ν+1 , on a
+∞
  π  π
nπ 1 1 C2 π2α
ρ2n 2
sin ν+1 = 2
|f (x + h) − f (x − h)| dx ≤ (C(2h)α) 2 dx =
n=1
2 8π −π 8π −π 4 22αν

et comme
nπ π 1  C 2 π2α
∀n, 2 ν−1 < n ≤ 2ν , sin2 ν+1
≥ sin2 = , on en déduit ρ 2n ≤ 2 .
2 4 2 4 22αν
2ν−1 <n≤2ν

Il suffit ensuite d’appliquer l’inégalité de Schwarz, qui entraı̂ne


 1/2  1/2
    2 2α  1/2
   2 (ν−1)/2 C π
ρn ≤ 1 ρn ≤2 ,
ν−1 ν ν−1 ν ν−1 ν
2 2 2αν
2 <n≤2 2 <n≤2 2 <n≤2

d’où le résultat.

c) Si α > 1/2, la majoration précédente montre que la série
 à termes positifs ρ n converge, et
comme |cn(f )| ≤ ρ|n| pour tout n ∈ Z, on en déduit que n∈Z |cn (f )| converge. Ainsi, la série
de Fourier de f converge normalement sur R, et on sait alors qu’elle ne peut converger que vers
f (voir le dernier alinéa de la remarque 6 page 271).

Exercice 8. Soit (λn ) une suite positive,


 décroissante et tendant vers 0.
a) Montrer que la série de fonctions λn sin(nx) converge simplement vers une fonction
f sur R, et que f est continue sur ]0, 2π [. 
b) Si λn = o(1/n) lorsque n → +∞, montrer que λn sin(nx) converge uniformément
vers f sur R. 
c) Réciproquement, si λn sin(nx) converge uniformément vers f sur R, montrer que
λn = o(1/n).
d) Plus généralement,
 x si f est continue sur R montrer que λn = o(1/n). (Indication.
Considérer F (x) = 0 f (t) dt.)

Solution. a) On sait d’après la proposition 2 page 268 qu’il y a convergence uniforme sur
[α, 2π − α] pour tout α ∈ ]0, π[. On conclut qu’il y a convergence simple sur ]0, 2π [, et que la
fonction limite f est continue sur [α, 2π − α] pour tout α > 0, donc continue sur ]0, 2π [.
Il y a bien convergence simple en 0 (la série est nulle lorsque x = 0), il y a donc convergence
simple sur [0, 2π [, donc sur R car les fonctions en présence sont 2π-périodiques.
b) C’est un peu technique. Comme les fonctions en présence sont 2π-périodiques et impaires, il
suffit de prouver la convergence uniforme sur [0, π]. Le problème est en x = 0 car on a vu plus
haut qu’il y avait convergence uniforme sur [α, 2π − α] pour tout α > 0.
Commençons par remarquer que
 +∞ 
  πλN
 inx
∀x ∈ ]0, π] , ∀N ∈ N,  λ n e ≤ . (∗)
  x
n=N
280 4. SUITES ET SÉRIES

En effet, une transformation d’Abel fournit


M
 M−1
 n

∀M > N, λn einx = (λn − λ n+1 )En (x) + λ M E M (x) avec En (x) = eikx
n=N n=N k=0
et on a la majoration
   
 1 − e i(n+1)x   sin( n+1 x)  1 π
   2 
∀x ∈ ]0, π] , |En (x)| =  = ≤ ≤
 1−e ix   sin(x/2)  sin(x/2) x
(on a utilisé l’inégalité de concavité sin u ≥ 2u/π sur [0, π/ 2]). En faisant tendre M vers l’infini
on en déduit (*) car (λ n) est décroissante et tend vers 0.
Ceci étant, considérons ε > 0 et N0 ∈ N∗ tel que λ n ≤ ε/n pour tout n ≥ N0 . D’après (*)
+∞

πλN πε
∀x ∈ ]0, π] , ∀N ≥ N0 , |r N (x)| ≤ ≤ où rN (x) = λ n sin(nx).
x Nx
n=N
Ainsi, si N ≥ N 0,  
1 πεN
∀x ∈ ,π , |rN(x)| ≤ = πε,
N N
et si x ∈ ]0, 1/N [, on a, en notant K = [1/x] la partie entière de 1/x
 K 
  K
 ε πε
 
|rN (x)| ≤  λn sin(nx) + |rK+1(x)| ≤ nx + ≤ εKx + πε ≤ ε + πε
  n (K + 1)x
n=N n=N
(on a utilisé la majoration | sin u| ≤ |u| que l’on montre facilement à partir de l’inégalité des
accroissements finis). Ainsi, |r N(x)| ≤ (1 + π )ε pour tout x ∈ [0, π] (en x = 0, c’est trivial), et
ceci est vrai indépendamment de N ≥ N 0. On a donc convergence uniforme sur [0, π].
c) Notons sn les sommes partielles de notre série trigonométrique. Si (sn) converge uniformément
sur R vers f , alors f est continue, et la suite ( sn(1/n)) tend vers 0 (il suffit décrire |s n (1/n)| ≤
|sn (1/n) − f (1/n)| + |f (1/n)| et de remarquer que les deux termes de droite tendent vers 0
lorsque n → +∞). La suite (λ n) étant décroissante, on a par ailleurs
   n       
1 k 1 n 1
sn ≥ λn sin ≥ λn sin ≥ λ n sin ,
n n 2 2 2
k=1 n/2<k≤n

donc 0 ≤ λn ≤ 2sn (1/n)/(n sin(1/2)), et comme sn (1/n) → 0, ceci montre λn = o(1/n).


x
d) C’est difficile. Comme indiqué, nous considérons la fonction F : x → 0 f (t) dt. Soit x ∈ ]0, π]
et α ∈ ]0, x[. La série trigonométrique définissant f converge uniformément sur [α, π ] comme on
l’a vu plus haut, ce qui entraı̂ne
 x   x
+∞ +∞
 λn
f (t) dt = λn sin nt dt = (cos nα − cos nx). (∗∗)
α n=1 α n=1
n
Nous allons prouver que cette expression reste  valable lorsque α = 0.
Pour tout t ∈ ]0, 2π [, on note G(t) = ∞ n=1 λn (cos nt)/n (la série converge simplement sur
]0, 2π [ car la suite (λn /n) est décroissante). Comme la suite (λn /n) est décroissante et tend vers
0, on peut appliquer (*) qui entraı̂ne
 
   
  λn π  λN 3N π
∀N ∈ N ,∗  cos n ≤ = 3λN
 3N 
 n≥N n N π

Comme cos(nπ/(3N )) ≥ cos(π/ 3) = 1/2 pour 1 ≤ n ≤ N , on a donc


 π  1 N−1
 λn N−1
 λn  π 
∀N ∈ N ∗ , G ≥ − 3λN donc ≤ 2G + 6λN . (∗∗∗)
3N 2 n=1 n n=1
n 3N
Comme f est continue en 0, l’égalité (**) montre que G(α) converge +
+
 lorsque α → 0 , donc G est
bornée au voisinage de 0 , et (***) montre donc que la série λn /n est majorée. Cette série
6. PROBLÈMES 281

converge donc (elle est à termes positifs), donc la série trigonométrique


 définissant G converge
normalement sur R. En particulier, ceci montre que G(α) tend vers λn /n lorsque α → 0, et
en faisant tendre α vers 0 dans (*), on obtient
 x +∞
 λn
∀x ∈ ]0, π] , F (x) = f (t) dt = (1 − cos nx).
0 n=1
n
Les termes de la série de l’expression précédente sont positifs donc
π   λn   nπ   λn N λN λN
∀N ∈ N, F ≥ 1 − cos ≥ ≥ = ,
N n N n 2 N 2
N/2<n≤N N/2<n≤N

autrement dit 0 ≤ λN ≤ 2F (π/N ). Comme f est continue et nulle en 0, on a F (x) = o(x)


lorsque x → 0 donc λN = o(π/N) = o(1/N ) d’où le résultat.
Remarque. Si on suppose f bornée au voisinage de 0, on a λn = O(1/n) (immédiat à
partir de l’inégalité 0 ≤ λN ≤ 2F (π/N) obtenue à la fin de la solution de la question d),
et le fait que F (x) = O (x) lorsque x → 0).

6. Problèmes
Problème 1. Donner un équivalent, lorsque n → ∞, d’une suite réelle (u n) vérifiant
1
u 0 > 0, ∀n ∈ N, un+1 = un + α , (α > −1).
un

Solution. On remarque déjà que (u n) diverge vers +∞. En effet, (un ) est croissante. Si elle était
majorée, elle convergerait et sa limite  vérifierait  =  + a/α, ce qui est absurde. La suite (u n )
est donc croissante et non majorée, donc diverge vers +∞.
Pour rechercher un équivalent de (un ), on va appliquer une méthode classique (déjà utilisée
dans l’exercice 7 page 207). On cherche s’il existe β > 0 tel que la suite (uβn+1 − uβn) converge.
Comme α + 1 > 0 et que (u n ) diverge vers +∞, on peut écrire
  β   
β 1 β 1
un+1 = un 1 + α+1 β
= u n 1 + α+1 + o donc u βn+1 − u βn ∼ βuβn−(α+1) .
un un uα+1
n

On choisit β = α + 1, de sorte que uα+1 α+1 ∼ α + 1. En sommant ces équivalents (on peut,
n+1 − un
voir le théorème 5 page 210), on obtient
n−1

uα+1
n − u α+1
0 = (u α+1 α+1
k+1 − uk ) ∼ n(α + 1),
k=0

donc finalement uα+1


n ∼ n(α + 1) donc un ∼ [n(α + 1)]1/(α+1) lorsque n → +∞.

Problème 2 (Nombre moyen de diviseurs d’un entier). Pour tout n ∈ N∗, on


note τ (n) le nombre de diviseurs de n. 1/ Donner, lorsque x → +∞, un équivalent de

F (x) = τ (n).
1≤n≤x

2/ On désigne par γ la constante d’Euler. Lorsque x → +∞, démontrer que



F (x) = x log x + (2γ − 1)x + O ( x)

(on considérera les indices ≤ x dans l’expression de F (x) par une somme double).
282 4. SUITES ET SÉRIES


Solution. 1/ En remarquant que τ (n) = m|n 1 (on somme 1 sur les entiers m qui divisent n),
une inversion de sommation donne
   
F (x) = 1= 1. (∗)
1≤n≤x m|n 1≤m≤x n≤x
m|n

Or n≤x,m|n 1 est le nombre d’entiers naturels inférieurs à x divisibles par m. Ces éléments
sont m, 2m, . . . , [x/m]m ( où [ y ] désigne la partie entière de y ), donc au nombre de [x/m]. Donc
finalement  x 
F (x) = .
1≤m≤x
m
En utilisant l’encadrement x/m − 1 < [x/m] ≤ x/m, en déduit
   
 1  1
x  − x < F (x) ≤ x  ,
1≤m≤x
m 1≤m≤x
m

et comme 1≤m≤x 1/m = log x + O (1) (c’est classique, voir la page 211), on en déduit F (x) =
x log x + O (x), en particulier F (x) ∼ x log x.
2/ Pousser l’asymptotique dans le raisonnement précédent ne permet pas d’obtenir le résultat.
On effectue le changement de variable k = n/m dans la dernière somme de (*), qui donne
  
F (x) = 1= 1. (∗∗)
1≤m≤x 1≤k 1≤km≤x
1≤km≤x

Comme
√ suggéré, découpons cette dernière somme en fonction de la position de k, m par rapport
à x. On peut écrire
    √   x √
F (x) = 1+ 1− 1=2 1 − [ x]2 = 2 − [ x] 2.
√ √ √ √ √ m
1≤k≤ x 1≤m≤ x 1≤k,m≤ x 1≤m≤ x 1≤m≤ x
1≤km≤x 1≤km≤x 1≤km≤x

Ici aussi on utilise l’encadrement y − 1 < [y ] ≤ y, qui permet d’obtenir, à partir de la dernière
expression
 x √ √  1 √
F (x) = 2 + O ( x) − ( x + O (1))2 = 2x − x + O( x). (∗∗∗)
√ m √ m
1≤m≤ x 1≤m≤ x

L’asymptotique d’ordre 3 des nombres harmoniques (voir page 211) permet d’écrire
 1 √ √ 1 √
= log x + γ + O(1/ x) = log x + γ + O(1/ x).
√ m 2
1≤m≤ x

on en déduit le résultat en remplaçant ceci dans la dernière expression de (***).


Remarque. On peut interpréter ce résultat en disant qu’un entier n a en moyenne log n +
2γ − 1 diviseurs.
La technique utilisée est classique dans ce type d’exercice : on fait apparaı̂tre une
somme double puis  on inverse les signes de sommation. On montre par exemple de la
même façon que n≤x σ (n) ∼ (π2 /12)x2 où σ (n) est la somme des diviseurs de n.
√ L’approche consistant à découper la somme de (**) en considérant les cas où k, m ≤
x a été utilisée par Dirichlet et est appelée la méthode de l’hyperbole. Elle porte ce
nom car géométriquement, elle consiste à regrouper les couples d’entiers naturels non √ nuls
(k, m) situés sous l’hyperbole km ≤ x en fonction de la position relative de k, m et x.
Le problème consistant à déterminer la plus petite constante θ telle que F (x) = x log x +
(2γ − 1)x + O (xθ+o(1) ) est célèbre et s’appelle le problème des diviseurs de Dirichlet. Nous
avons montré que θ ≤ 1/2. En 1903, Voronoı̈ a montré que θ ≤ 1/3 puis Hardy et Landau
ont montré en 1915 que θ ≥ 1/4. Il est conjecturé que θ = 1/4 mais le meilleur résultat
obtenu jusqu’à présent est dû à Huxley qui a prouvé en 2003 que θ ≤ 131/416  0, 3149.
6. PROBLÈMES 283

 +∞ 2
Problème 3 (Deux techniques originales pour calculer n=1 1/n ). 1/ Pour
tout m ∈ N∗, on définit la fonction
sin(2m + 1)θ
fm : ]−π, π[ {0} → R θ → .
sin2m+1 θ

a) Écrire f m sous la forme d’un polynôme Pm en cotan 2 θ.


b) En déduire les racines de Pm et calculer leur somme. Conclure.
2/ a) Montrer que la série de fonctions
 1 1 · 3 · · · (2n − 1)
sin t + sin2n+1 t
n∈N ∗
2n + 1 2 · 4 · · · (2n )
converge normalement vers t → t sur [−π/2, π/2]. +∞
b) En déduire par intégration la valeur de +∞ 2
n=0 1/(2n + 1) , puis celle de
2
n=1 1/n .

Solution. 1/ a) En écrivant que sin(2m + 1)θ est la partie imaginaire de (cos θ + i sin θ )2m+1 ,
on trouve
m
∀θ ∈ R, sin(2m + 1)θ = C 22km+1 k
+1 (−1) sin
2k+1
θ cos2(m−k) θ,
k=0
2
m 2k+1 k m−k
donc fm (θ) = Pm (cotan θ ) avec Pm (X) = k=0 C2m+1 (−1) X .
2m+1
b) L’expression fm (θ) = sin((2m + 1)θ )/ sin θ montre que
    
kπ 2 kπ
∀k ∈ N, 1 ≤ k ≤ m, fm =0 donc Pm cotan = 0.
2m + 1 2m + 1
On a ainsi trouvé m racines distinctes du polynôme Pm, et comme deg(P m) = m, on a trouvé
toutes les racines de Pm . La somme des racines de Pm est l’opposé du rapport du coefficient de
X m−1 par celui de X m, donc
m  
kπ C3 2m(2m − 1)
cotan 2
= 21m+1 = . (∗)
2m + 1 C2m+1 6
k=1

Pour en déduire la valeur de 1/n 2 , nous allons comparer cotan 2x et 1/x2 . Montrons
 π 1
∀x ∈ 0, , cotan 2x ≤ 2 ≤ 1 + cotan 2 x. (∗∗)
2 x
L’inégalité des accroissements finis donne tan x ≥ x sur ]0, π/2[, d’où la première inégalité de
(**). Pour la seconde, on utilise la majoration sin x ≤ x sur ]0, π/2[ (qui s’obtient aussi avec
l’inégalité des accroissements finis) qui entraı̂ne 1 + cotan 2x = 1/ sin 2 x ≥ 1/x2 .
De (*) et (**), on tire
m
2m(2m − 1)  (2m + 1) 2 2m(2m − 1)
≤ ≤ +m
6 k2 π2 6
k=1
donc
m
2m(2m − 1) π 2 1 2m(2m − 1) π2 m
2
≤ 2
≤ 2
+ π2 .
(2m + 1) 6 k (2m + 1) 6 (2m + 1)2
k=1

Ceci est vrai pour tout m ∈ N ∗. En faisant m → +∞, on en déduit +∞ 2 2
k=1 1/k = π /6.
2/ a) On sait que la fonction arcsinus est développable en série entière sur ] − 1, 1[, plus
précisément
+∞
 1 1 · 3 · · · (2n − 1)
∀x ∈ ]−1, 1[ , arcsin x = x + un x2n+1 avec un = . (∗∗∗)
n=1
2n + 1 2 · 4 · · · (2n)
L’idée est ensuite de remplacer x par sin t dans cette expression.
284 4. SUITES ET SÉRIES


On pourrait montrer la convergence normale de u n x2n+1 sur [−1, 1] en utilisant la règle
de Raab-Duhamel pour estimer un , mais ici, on peut mieux faire. La positivité des un entraı̂ne
pour tout N ∈ N∗ , d’après (***)
N
 π
∀x ∈ ]−1, 1[ , x+ un x2n+1 ≤ arcsin x ≤ arcsin 1 = ,
2
n=1

faisant tendre x vers 1 on en déduit 1 + N
et en  n=1 un ≤ π/2. Ceci est
vrai pour tout N ∈ N ,

donc un est majorée, et comme les termes de cette série sont positifs, un = |un | converge.
2n+1
Ainsi, un x converge normalement sur [−1, 1]. Sa somme définit donc une fonction conti-
nue sur [−1, 1]. L’égalité (***) vaut sur ] − 1,1[, elle vaut donc sur [−1, 1] par continuité.
Finalement, nous avons montré que x + u nx2n+1 converge normalement vers la fonction
arcsinus sur [−1, 1]. On en déduit le résultat demandé en remplaçant x par sin t (t ∈ [−π/2, π/2]).
b) La convergence normale de la série de fonctions nous autorise à l’intégrer terme à terme, ce
qui donne
 π/2   π/2
+∞ +∞
 +∞

π2 2n+1 2 · 4 · · · (2n) 1
= t dt = 1 + un sin t dt = 1 + un =1+
8 0 n=1 0 n=1
3 · 5 · · · (2n + 1) n=1
(2n + 1)2

où nous avons utilisé la valeur de l’intégrale de Wallis 0π/2 sin 2n+1 t dt (voir l’exercice 1 page 130).

Pour en déduire S = +∞ 2
n=1 1/n , il suffit de séparer les termes d’indices pairs et impairs
+∞
 +∞
 1
1 π2 S π2
S= 2
+ 2
= + d’où S = .
n=0
(2n + 1) n=1
(2n) 8 4 6

Problème 4 (Formule sommatoire de Poisson). a) Soit f : R → C une fonction


de classe C1 vérifiant f (x) = O (1/x 2) et f (x) = O(1/x2 ) lorsque |x| → +∞. Après avoir
justifié l’existence des sommes infinies, montrer que
   +∞
∗ 2iπnx ∗
∀x ∈ R, f (x + n) = f (n) e où ∀n ∈ Z, f (n) = f (t)e −2iπnt dt.
n∈Z n∈Z −∞

(formule sommatoire de Poisson).


b) (Application.) Montrer que
+∞
 +∞

−πn 2 s −1/2 2/s
∀s > 0, e =s e−πk
n=−∞ k=−∞

(on pourra utiliser le résultat établi à la question 3/ de l’exercice 4 page 168).



Solution. a) La série de fonctions n∈Z f (x + n) converge normalement (donc uniformément)
sur tout segment de R. En effet, si M > 0 est tel que |f (x)| ≤ M/x2 pour |x| ≥ 1, il suffit
d’écrire pour tout K > 0 la majoration
M M
∀x ∈ [−K, K], ∀n ∈ Z, |n| > K + 1, |f (x + n)| ≤ 2
≤ .
(x + n) (|n| − K ) 2

En particulier, n∈Z f (x + n) converge simplement  sur R. On note F sa limite simple.
Pour les mêmes raisons, la série de fonctions n∈Z f (x + n) converge uniformément sur tout
segment de R. On peut donc appliquer le théorème de dérivation sur les suites de fonctions qui
entraı̂ne que F est de classe C 1 sur tout segment de R, donc sur R.
Par ailleurs, F est 1-périodique car si on fixe x ∈ R,
N
 N+1

∀N ∈ N, f (x + 1 + n) = f (x + n),
n=−N n=−N +1
6. PROBLÈMES 285

et en faisant N → +∞, on en déduit F (x + 1) = F (x).


Les coefficients de Fourier de F sont donnés par
1  
+∞ 1
−2iπnt
∀n ∈ Z, cn (F ) = F (t)e dt = f (t + n)e−2iπnt dt
0 n=−∞ 0
+∞
  n+1  +∞
= f (t)e −2iπnt
dt = f (t)e −2iπnt dt = f ∗(n)
n=−∞ n −∞

(on a bien le droit d’intervertir les signes de sommation car la série de fonctions définissant F
converge uniformément sur [0, 1] comme on l’a vu ; par ailleurs, la dernière intégrale converge
absolument au vu des conditions de croissance satisfaites par f ). Comme de plus F est de classe
C 1 , sa série de Fourier converge uniformément vers F , d’où la formule sommatoire de Poisson.
2
b) Soit α > 0. Nous appliquons la formule sommatoire de Poisson à la fonction f : x → e−αx .
Les coefficients f ∗ (n) sont donnés par
 +∞  +∞ 
1 √ π −π 2n2/α
∗ −αt2 −2iπnt −u2 −2iπnu/ α
∀n ∈ Z, f (n) = e e dt = √ e e du = e
−∞ α −∞ α
(où on a utilisé le résultat de la question 3/ de l’exercice 4 de la page 168). En appliquant la
formule sommatoire de Poisson avec x = 0, on déduit

 π  −π2n2 /α
−αn2
e = e .
α
n∈Z n∈Z
Ceci est vrai pour tout α > 0, et en changeant α en πs , on obtient le résultat désiré.
Remarque.
 La dernière identité est non-triviale. En considérant la série entière Θ(x) =
n2

n∈Z x , (fonction thêta de Jacobi) elle s’exprime en écrivant s Θ(e−sπ ) = Θ(e−π/s).
Ainsi, le comportement de Θ(x) en x = 1 est relié à son comportement en x = 0.

Problème 5. Soit (un )n∈N une suite complexe vérifiant une récurrence linéaire d’ordre
h ∈ N∗ , c’est-à-dire
∃a1 , . . . , ah ∈ C, ∀n ≥ h, un = a 1un−1 + · · · + ah un−h . (∗)

En considérant la série entière un zn , démontrer la proposition 3 page 202.

Solution. Si la série entière un zn a un rayon de convergence non nul (on ne sait pas encore
si c’est le cas), la récurrence (*) entraı̂ne, après produit par zn et sommation sur n (lorsque z
est dans le disque de convergence) que sa somme f vérifie
f (z ) − Ph (z) = a1 z [f (z ) − P h−1(z)] + · · · + a h−1 z h−1 [f (z ) − P1 (z)] + ah zhf (z ) (∗∗)
où pour tout k, 1 ≤ k ≤ h, Pk (z) = u0 +u1 z + · · ·+ uk−1 z k−1. Ceci s’écrit aussi f (z )Q(z) = P (z )
où
Q(z ) = 1 − a 1 z − · · · − ah z h et P (z) = Ph (z ) − a 1 zPh−1(z ) − · · · − a h−1 zh−1 P 1 (z ).
Ainsi, f (z ) = P (z )/Q(z ) est une fraction rationnelle. Par ailleurs, 0 n’est pas un pôle de f (z )
donc P (z )/Q(z ) est développable en une série entière vn zn dont le rayon de convergence est
non nul (voir page 251). Sa somme g vérifie Q(z )g (z ) = P (z ) sur son disque de convergence,
autrement dit g vérifie la même égalité que f dans (**), donc la suite (vn ) vérifie la récurrence
(*). Par ailleurs, l’égalité (**) vérifiée par g montre que pour tout k, 0 ≤ k < h, le coefficient
de zk dans g (z ) − Ph(z) est nul. Autrement dit, v k = uk lorsque 0 ≤ k < h. Finalement, la suite
(v n) vérifie la même récurrence que (u n) avec  lesn mêmes conditions initiales. Ces suites sont
donc égales, ce qui montre finalement que u n z a bien un rayon de convergence non nul et
que sa somme f vérifie f (z ) = P (z )/Q(z ) sur son disque de convergence.
286 4. SUITES ET SÉRIES

On veut maintenant calculer explicitement un . Soient x1 , . . . , xp les racines de l’équation


caractéristique de (*), d’ordre de multiplicité α 1, . . . , αp , de sorte que
p   p
h h−1 αi h 1
R(X ) = X − a1 X − · · · − ah = (X − x i ) donc Q(X ) = X R = (1 − x iX) αi .
i=1
X i=1

Le polynôme P vérifie deg(P ) < h = deg(Q) par construction, on peut donc écrire la décomposi-
tion en éléments simples de P /Q sous la forme
 
p αi
P (X )   ci,j
=   (ci,j ∈ C).
Q(X ) (X − 1/x i )j
i=1 j=1

On en déduit (voir page 251) que le développement en série entière de f (z) = P (z )/Q(z ) s’écrit
  +∞ 
p αi j j 
 (−1) c i,j x i (n + j − 1)! n n 
f (z ) = xi z .
i=1 j=1
(j − 1)! n=0
n!

On tire, en prenant le coefficient de z n dans cette expression,


 
 p αi
 j j
 (−1) c i,j xi
un = (n + j − 1) · · · (n + 1)  xni,
i=1 j=1
(j − 1)!

donc un = pi=1 Ti (n) xni où pour tout i, Ti est un polynôme de degré < α i .
En désignant par Γ l’e.v des suites vérifiant cette dernière condition, nous avons prouvé que
l’e.v U des suites complexes vérifiant la récurrence linéaire (*) est tel que U ⊂ Γ. Nous voulons
prouver la condition suffisante, c’est-à-dire Γ ⊂ U . Pour cela, on remarque que l’application
U → C h (u n) n∈N → (u0 , u1 , . . . , uh−1 )
est linéaire (c’est évident) et bijective (une suite vérifiant (*) est uniquement déterminée par
ses premiers termes u0 , . . . , u h−1 ). Ainsi, U est de dimension h, et comme Γ est de dimension
α1 + · · · + αp = h, l’inclusion U ⊂ Γ entraı̂ne U = Γ.
Nous avons donc démontré la proposition 3 page 202.
Remarque. On démontre en général  la proposition 3 page 202 en vérifiant directement
que les expressions de la forme pi=1 Pi(n)x ni (avec P i un polynôme de degré < α i ) sont
bien solutions de la récurrence, puis en prouvant que la dimension de l’e.v correspondant
est bien égal à celui des suites vérifiant la récurrence (comme dans la solution présentée
ici). L’intérêt du problème proposé ici est qu’il permet de découvrir la forme générale de
la solution sans la connaı̂tre a priori.

 √
Problème 6 (Nombres de Pisot). a) Montrer que la série sin(π (2+ 3)n ) converge.
b) Plus généralement, on considère un polynôme P de degré d, unitaire, à coefficients
entiers, et on suppose que ses racines ξ1, . . . , ξ d vérifient |ξ1| > 1 et |ξ k| < 1pour tout
k ∈ {2, . . . , d} (on dit que ξ1 est un nombre de Pisot). Montrer que la série sin(πξ n1)
converge.
√ √
Solution. a) Il suffit de remarquer que pour tout n ∈ N, (2 + 3)n + (2 − 3)n est un entier (on
s’en convainc
√ en développant chacun
√ des deux termes par la formule du binôme).
√ Ceci entraı̂ne

| sin(π(2 + √3)n )| = | sin(π(2 − 3)n )| pour tout n ∈ N. Comme | sin(π(2 − 3)n )| ≤ π(2 − 3) n
et que |2 − 3| < 1, on en déduit que notre série converge absolument, donc converge.
b) Le polynôme P est unitaire à coefficients entiers, on en déduit que pour tout n ∈ N, Sn =
ξ n1 + · · · + ξ nd est un entier (S n s’exprime comme un polynôme à coefficients entiers, symétrique
en les ξi, donc s’exprime comme un polynôme à coefficients entiers en les fonctions symétriques
6. PROBLÈMES 287

des racines de P , et ces dernières sont entières donc S n ∈ Z. On peut aussi retrouver ce dernier
résultat grâce aux formules de Newton — voir le tome Algèbre). Ceci entraı̂ne
| sin(πξ 1n )| = | sin (π (ξ2n + · · · + ξnd)) | ≤ π |ξ n2 + · · · + ξnd | ≤ π (|ξ2 |n + · · · + |ξd |n )
et comme |ξ k| < 1 pour k ≥ 2, on en déduit la convergence de la série proposée.

Problème 7. a) Soit α > 0 un nombre irrationnel. Pour tout N ∈ N∗, montrer


 
 
2
∃(p, q ) ∈ N , 1 ≤ q ≤ N,
p
α −  < 1 .
 q  qN
 1
b) Donner la nature de la série (on rappelle que π est un nombre irrationnel,
n∈N ∗
n2 sin2 n
voir le problème 15 page 187).

Solution. a) C’est classique. Nous allons utiliser une méthode connue sous le nom de principe des
tiroirs. Pour tout x ∈ R, notons [x] sa partie entière. On considère les nombres uk = kα − [kα]
pour k = 1, 2, . . . , N + 1. Chacun des uk est élément de [0, 1[, et pour tout k, il existe un unique
i ∈ {0, . . . , N − 1} tel que u k ∈ [ Ni , i+1N [. Autrement dit, nous rangeons les N + 1 nombres uk
dans les N “boites” [0, N [, [ N , N [, . . . , [ N−1
1 1 2
N
, 1[. Il existe donc une boite contenant au moins
deux des nombres uk , ce qui entraı̂ne
1
∃a, b ∈ {1, . . . , N + 1}, a < b, |u b − u a| < ,
N
ce qui s’écrit encore |(b − a)α − ([bα] − [aα])| < 1/N . En posant p = [bα] − [aα] et q = b − a, on
a donc (p, q) ∈ N2 , 1 ≤ q ≤ N et |qα − p| < 1/N, d’où le résultat.
b) La question précédente entraı̂ne l’existence, pour tout n ∈ N∗, d’un couple (p n , qn ) ∈ N2 tel
que 1 ≤ qn ≤ n et |π − pn/q n | ≤ 1/(nq n) ≤ 1/q2n . La suite de rationnels (pn /qn ) tend vers le
nombre irrationnel π , donc (pn) et (q n) tendent vers +∞ (voir l’exercice 4 page 205). Ensuite,
pour tout n ∈ N ∗, on a |p n − πqn | ≤ 1/q n donc
sin 2 pn = sin 2(p n − πqn ) ≤ (pn − πqn )2 ≤ 1/q n2 ,
donc p2n sin2 pn ≤ p2n /q2n , et comme (p n /qn ) converge, la suite (p2n sin 2 pn ) est majorée. Donc la
suite de terme général (p2n sin2 pn )−1 est minorée par une constante > 0. On en déduit que le
terme général de la série proposée ne tend pas vers 0, donc la série diverge.

Problème 8. Soit g : R+ → R une fonction continue, convexe, avec lim x→+∞ g (x) = 0.
a) Montrer que la fonction suivante est bien définie
+∞

S : ]0, +∞[ → R h → (−1) n g (nh).
n=0

g(0)
b) Montrer que lim+ S (h) = .
h→0 2
Solution. a) Si on montre  que g est décroissante sur R+, alors g sera positive (car g(x) → 0
quand x → +∞) et la série (−1)n g (nh) sera convergente pour tout h > 0 (série alternée dont
la valeur absolue du terme général décroı̂t et tend vers 0), ce qui prouvera le résultat.
Soient a, b ∈ R, 0 ≤ a < b. Comme g est convexe, son graphe à droite de b est au dessus de
la corde reliant (a, g(a)) à (b, g (b)) ce qui s’écrit
g (b) − g (a)
∀t > b, g (t) ≥ g (b) + (t − b) .
b−a
288 4. SUITES ET SÉRIES

Comme g (t) tend vers 0 à l’infini, ceci n’est possible que si (g (b) − g (a))/(b − a) ≤ 0, donc
g (b) ≤ g (a). Donc g est bien décroissante.
b) On remarque que R : h → S (h) − g(0)/2 vérifie
+∞
1
∀h > 0, R(h) = (−1) nA n (h) avec A n(h) = g (nh) − g((n + 1)h).
2 n=0
Comme g est décroissante, on a A n(h) ≥ 0 pour tout n et pour tout h > 0. De plus la convexité
de g entraı̂ne
 
∗ (n − 1)h + (n + 1)h g ((n − 1)h) g ((n + 1)h)
∀h > 0, ∀n ∈ N , g(nh) = g ≤ +
2 2 2
c’est-à-dire g (nh) − g((n + 1)h) ≤ g((n − 1)h) − g(nh). Autrement dit, la suite (An (h)) décroı̂t
pour tout h > 0.
Finalement, (−1) n A n (h) apparaı̂t comme une série alternée dont la valeur absolue du
terme général décroı̂t et tend vers 0. On peut donc écrire (voir le théorème 6 page 214)
+∞ 
 
 
∀h > 0, |2R(h)| =  (−1)nA n(h)  ≤ A0(h) = g (0) − g (h).
 
n=0
On en déduit que R(h) tend vers 0 lorsque h → 0+ (g est continue), donc S (h) → g (0)/2 lorsque
h → 0+ .

Problème 9. Donner un équivalent, lorsque x → 1− , de la fonction


+∞

xn
f : [0, 1[ → R x → .
n=1
1 − xn

Solution. L’identité 1 − xn = (1 − x)Pn (x) avec P n(x) = 1 + x + · · · + x n−1 montre que


+∞
 xn
∀x ∈ [0, 1[ , g(x) = (1 − x)f (x) = .
n=1
P n(x )
fixé, on a Pn(x) → n lorsque x → 1 −,
On se ramène ainsi à donner un équivalent de g. Pour n 
ce qui amène à penser que g(x) est équivalent à h(x) = +∞ n
n=1 x /n = − log(1 − x). Montrons
ce dernier résultat. Compte tenu du fait que, pour 0 ≤ x < 1,
n−1
 n−1
 n−1

k
0 ≤ n − Pn(x) = (1 − x ) = (1 − x) Pk(x) ≤ (1 − x) Pn (x) ≤ n(1 − x)Pn(x),
k=0 k=1 k=1
on en déduit, pour tout x ∈ [0, 1[,
+∞
 +∞ +∞
nn − Pn (x)  n n(1 − x)Pn (x) 
0 ≤ g (x) − h(x) = x ≤ x = (1 − x) x n = x ≤ 1.
n=1
nPn (x) n=1
nPn (x) n=1
Ainsi, lorsque x → 1−, g (x) = h(x) + O (1) = − log(1 − x) + O(1) ∼ − log(1 − x), donc
f (x) ∼ log(1 − x)/(x − 1).

Problème 10 (Une série enti ère semi-convergente sur tout son cercle de
convergence). 1/ Soient(a n)n∈N∗ et (bn) n∈N∗ deux  suites complexes. On pose σ0 = 0
∗ n
et pour tout n ∈ N , σn = k=1 ak . Montrer que anbn converge dans les cas suivants :
a) (i) (σn ) √
est bornée, (ii) |bn− bn+1 | converge,
√ (iii) limn→+∞ bn = 0. √
b) (i) (σn/ n) est bornée, (ii) |bn − bn+1| n converge, (iii) lim n→+∞ bn n = 0.
6. PROBLÈMES 289

2/ (Application.) Montrer que la série entière



 (−1)[ n]
zn

n
n∈N

( où [ x] désigne la partie entière de x) est convergente en tout point de son cercle de
convergence |z | = 1, mais n’est absolument convergente en aucun point de ce cercle
(traiter séparément les cas z = 1 et z = 1).

Solution. 1/ Dans les deux cas, on utilisera la relation suivante, conséquence d’une transforma-
tion d’Abel :
n
 n
 n−1

∀n ∈ N ∗, ak b k = (σk − σ k−1 )bk = σk (b k − bk+1 ) + σ nbn . (∗)
k=1 k=1 k=1

a)
 Si M désigne un majorant de la suite (|σn |), on a |σ n(bn − bn+1 )| ≤ M |bn − b n+1|, donc
σ n (bn − b n+1 ) converge absolument d’après (ii),
 et comme |σn bn | ≤ M |b n| tend vers 0 d’après
(iii), on en conclut avec (*) la convergence de akbk.
√ √
b) Soit M un majorant √ de (|σn/ n|), de sorte que |σ n| ≤ M n pour tout n. On a |σn(bn −
b n+1 )|√≤ M |bn − bn+1 | n donc σ n(bn − bn+1) converge absolument d’après (ii). Or |σ nbn | ≤
M |bn | n tend vers 0 d’après (iii), d’où le résultat avec (*).

2/ Commençons par le cas z = 1. D’après la question 1/b) appliquée avec an = (−1)[ n] et
 √
[ n]
b n = 1/n, la conver gence de (−1) /n sera assurée si on montre que les sommes partielles
 √
[ n] √
σn de (−1) sont majorées en valeur absolue par un terme de la forme M n. Montrons
donc ce point.
Pour tout entier naturel impair p on a
(p+2)2 −1 (p+1)2 −1 (p+2) 2−1
 √  √  √
[ n] [ n]
(−1) = (−1) + (−1) [ n]
= (−1) p(2p + 1) + (−1) p+1(2p + 3) = 2.
n=p2 n=p 2 n=(p+1)2

Donnons nous maintenant N ∈ N∗ et notons N 0 le plus grand entier tel que (2N0 + 1)2 ≤ N .
On a
 
(2N 0+1)2 −1 N 0−1 (2k+3) 2 −1 N
0−1
 √   √ √
(−1) [ n] =  (−1)[ n] = 2 = 2N0 ≤ N − 1,
n=1 k=0 n=(2k+1) 2 k=0

et comme (2N 0 + 1) 2 ≤ N < (2N0 + 3)2 ,


 
  N 
 √  √
 (−1) [ n]
≤ N + 1 − (2N + 1) 2
≤ (2N + 3)2
− (2N + 1)2
= 8(N + 1) ≤ 4 N + 4.
  0 0 0 0
n=(2N 0+1) 2 
√ √ √ √
Ceci montre que |σN | ≤ ( N − 1) + (4 N + 4) = 5 N + 3 ≤ 8 N pour tout N ∈ N∗, d’où le
résultat.
Passons maintenant au cas où |z | =√ 1 et z = 1. On écrit z = e iθ avec 0 < θ < 2π , et il s’agit de

montrer la√convergence de (−1) [ n] eniθ /n. Pour cela, on va appliquer 1/a) avec a n = e niθ et
b n = (−1)[ n]/n.
(i). La suite (σn ) est bornée car
 n  
   1 − eniθ  2
 kiθ 
∀n ∈ N ,∗
σn =  e  = e iθ ≤
 .
  1 − eiθ |1 − e iθ|
k=1
290 4. SUITES ET SÉRIES


(ii). Pour montrer que |bn − bn+1| converge, on remarque d’abord
1 1 1 1
si ∃p ≥ 2 (p ∈ N), n + 1 = p 2 alors |bn − b n+1 | = + = 2 + 2
n n+1 p −1 p
1 1 1
si ∃p ∈ N∗ , p2 ≤ n < n + 1 < (p + 1) 2 alors |bn − bn+1 | = − = .
n n+1 n(n + 1)
On en déduit

N N [ N+1]  
  1  1 1

∀N ∈ N , |b n − b n+1| ≤ + 2
+ 2 ,
n(n + 1) p −1 p
n=1 n=1 p=2
  2

et
 comme les séries 1 / (n(n + 1)), 1 / (p − 1) et 1/p2 convergent, on en déduit que
|b n − b n+1 | est bornée donc converge.
(iii) est trivialement vérifiée.
 √
Ainsi, pour tout z ∈ C tel que |z | = 1, la série (−1)[ n]zn /n converge, mais ne converge pas
absolument. En d’autres termes, notre série entière est semi-convergente en tout point de son
cercle de convergence.

 n
Problème 11. a) Montrer que la série entière z /(n!) 2 a un rayon de convergence
infini. Soit f sa somme.
b) Pour tout x > 0, comparer f (x) et
 π/2

I (x) = exp(2 x sin t) dt.
−π/2

c) En déduire, lorsque x → +∞, un équivalent de f (x).

Solution.  n
a) C’est facile, car on a l’encadrement 0 ≤ 1/(n!) 2 ≤ 1/n! et la série z /n! a un rayon de
convergence infini.
 √
b) Fixons x > 0. La série√ de fonctions (2 x sin t) n/n! de la variable t converge
√ normalement
sur [−π/ 2, π/2] (car 2 x sin t y est bornée) vers la fonction t → exp(2 x sin t), donc on est
autorisé à intervertir les signes de sommation en écrivant
  π/2
+∞
2n xn/2
+∞
 2n xn/2  π/2
I (x) = sin n t dt = I n, où ∀n ∈ N, In = sinn t dt. (∗)
−π/2 n! n! −π/2
n=0 n=0

Lorsque n = 2k + 1 est impair, l’intégrande de I2k+1 est impaire donc I2k+1 = 0. Lorsque n = 2k
est pair, I2k s’exprime au moyen des intégrales de Wallis (voir l’exercice 1 page 130)
 π/2
(2k − 1)(2k − 3) · · · 1 π (2k )!
I 2k = 2 sin2k t dt = = 2k π,
0 2k(2k − 2) · · · 2 2 2 (k!)2
et donc (*) s’écrit
+∞
 +∞
2 2k xk 1  xk f (x)
I (x) = I 2k = 2
= .
(2k )! π (k!) π
k=0 k=0

c) Il s’agit de donner un équivalent de l’intégrale I (x) lorsque x → +∞. On pourrait s’en tirer
en appliquant directement la méthode de Laplace (voir le théorème 4 page 164). Comme cette
dernière n’est pas au programme de mathématiques spéciales, nous allons nous mettre dans les
conditions du concours et en faire abstraction.
6. PROBLÈMES 291

√ Pour simplifier l’intégrande, nous commençons par faire le changement de variable u =


2 x sin t qui donne
 2√x
eu
∀x > 0, I(x) = √
√ du.
−2 x 4x − u2

Lorsque x est grand, c’est la partie des u proche de 2 x qui contribue le plus à la valeur de
l’intégrale.
√ Pour ramener ce maximum à une abscisse fixe, on fait le changement de variable
v = 2 x − u qui donne
 4√x √ √  4 √x
e 2 x−v e2 x e −v
∀x > 0, I(x) =  √ dv = 1/4 J (x), J(x) =  dv.
0 4 xv + v2 2x 0 v + v 2x −1/2/4

Lorsque x → +∞, l’intégrande de J (x) tend vers e −v/ v, et on s’attend à ce que J (x) converge
vers l’intégrale correspondante. Pour prouver ce point, nous utilisons l’inégalité |(v + a)−1/2 −
v−1/2 | ≤ a/v 3/2 pour a ≥ 0 (conséquence de l’inégalité des accroissements finis) qui entraı̂ne
  4√ x −v   4 √ x 2  4√ x

 e  v 1 −v 1
∀x > 0, J (x) − √ dv ≤ √ 3/2 e dv = √ v 1/2e−v dv,
 0 v  0 4 x v 4 x 0
 +∞ 1/2 −v  4√ x −v √ √
et comme 0 v e dv converge, on en déduit J (x) = 0 e / v dv + O (1/ x) donc
 √
J (x) converge vers K = 0+∞ e−v v −1/2 dv lorsque x → +∞. Or K = Γ(1/2) = π (voir la
question 2/d) du sujet d’étude 1 page 315), donc finalement, lorsque x → +∞,
√ √
e2 x e2 x √ I (x) 1 √
I (x) = 1/4 J (x) ∼ 1/4 π d’où f (x) = ∼ √ x−1/4 e2 x .
2x 2x π 2 π

Problème 12. Soit α > 0. Donner un équivalent, lorsque n → +∞, de


 n +∞

(αn)k (αn)k
Pn (α) = , et R n (α) = .
k! k!
k=0 k=n+1

Solution. Remarquons que Pn (α) + R n(α) = e αn.


– Cas α > 1. On écrit
n

(αn) n n · · · (n − k + 1) 1
Pn (α) = An (α), avec An(α) = 1 + .
n! nk αk
k=1
On écrit An (α) sous la forme
n
 k

1
A n(α) = 1 + ak−1 (n) , où ak (n) = (1 − j/n).
αk
k=1 j=1

Chaque
 a k(n) converge vers 1 lorsque n → +∞, ce qui suggère que A n (α) converge vers
k K
k≥0 1/α . Prouvons ce résultat. Soit ε > 0. Fixons K ∈ N tel que 1/α < ε et N > K
tel que a K (N ) > 1 − ε. Pour 0 ≤ k ≤ K et n ≥ N , on a ak (n) ≥ a K(N ) > 1 − ε, donc
 1 K  1 n
1 − 1/α K+1
1 + (1 − ε) = 1 + (1 − ε) < An (α) ≤ .
1 − 1/α αk αk
k=1 k=1
α α
On en déduit (1 − ε)2 < An(α) ≤ , et ceci pour tout n ≥ N . Donc A n (α) converge
α−1 α−1
vers α/(α − 1) lorsque n → ∞. Avec la formule de Stirling, on trouve
 e n 1 α α (αe) n
Pn (α) ∼ (αn)n √ = √ et Rn (α) = eαn − Pn (α) ∼ e αn.
n 2πn α − 1 α − 1 2πn
(on a utilisé l’inégalité eα ≥ αe, la fonction concave e x étant au dessus de sa tangente en x = 1).
292 4. SUITES ET SÉRIES

– Cas α < 1. On écrit


+∞

(αn)n nk
Rn (α) = Bn (α), avec Bn (α) = αk.
n! (n + 1) · · · (n + k )
k=1
On écrit Bn (α) sous la forme
+∞
 k

k 1
Bn (α) = bk (n)α , où bk (n) = .
1 + j/n
k=1 j=1

On va montrer que Bn (α) converge vers k≥1 αk . Soit ε > 0. Fixons K ∈ N tel que αK < ε,
puis N ∈ N tel que bK (N ) > 1 − ε. Pour 1 ≤ k ≤ K et n ≥ N , on a bk (n) ≥ b K (N ) > 1 − ε donc
  K +∞
2 α α − αK+1 α
(1 − ε) ≤ (1 − ε) = (1 − ε) αk ≤ Bn (α) ≤ αk = .
1−α 1−α 1−α
k=1 k=1
Donc Bn (α) converge vers α/(1 − α). Avec la formule de Stirling , on en déduit
 n 1 α (αe)n
n e α
Rn (α) ∼ (αn) √ = √ et P n(α) = e αn − R n (α) ∼ eαn .
n 2πn 1 − α 1 − α 2πn

– Cas α = 1. C’est plus délicat. On va d’abord estimer An (1) − Bn (1) ; pour cela, on coupe la
somme sur k jusqu’a un entier m = [nλ ] , où λ ∈ ]0, 1[ sera fixé plus tard ([x] désigne la partie
entière de x). On écrit
m
 n−1
 +∞

An (1)−Bn(1) = 2+α n+βn , avec αn = (ak (n) −bk (n)), βn = ak (n) − b k (n).
k=1 k=m+1 k=m+1

On utilise maintenant la majoration | log(1 + x) − x| ≤ 2x2


sur [−1/2, 1/2] (conséquence du

developpement de Taylor-Young à l’ordre 2 appliqué à log(1 + x)). En notant sk (n) = kj=1 j/n,
elle entraı̂ne, lorsque 0 ≤ k ≤ m (et n assez grand pour que m < n/2, de sorte que k/n ≤ 1/2)
k
 k

log a k(n) = log(1 − j/n) = −sk (n) + xk (n), |xk (n)| ≤ 2 (j/n) 2 ≤ 2m3 /n 2
j=1 j=1
k
 k

log b k(n) = − log(1 + j/n) = −sk (n) + yk (n), |yk (n)| ≤ 2 (j/n) 2 ≤ 2m 3/n2 .
j=1 j=1

On a m3 /n 2 ≤ n 3λ−2, on va donc choisir λ < 2/3 de sorte que m 3 /n2 = o(1). Ceci assure
l’existence d’une constante M > 0 telle que
|ak (n) − bk (n)| = ak (n) |1 − exp(yk (n) − xk (n))| ≤ M ak(n) |y k(n) − x k (n)| ≤ 4M n 3λ−2 ak (n)
donc
|αn | ≤ 4M n3λ−2 A n(1).

Par ailleurs ak (n) ≤ b k (n) (car ak (n)/bk (n) = 1≤j ≤k (1 − j2 /n2 ) ≤ 1) donc
n−1
 +∞
 +∞
 
+∞
1
k
n
|βn | = (bk(n) − ak (n)) + bk (n) ≤ bk (n) ≤ bm (n) ≤ bm(n) .
1 + m/n m
k=m+1 k=n k=m+1 k=1
Or
2λ−1
b m(n) = e−sm(n)+y m (n) = O (e−s m(n) ) = O (e−m(m+1)/(2n)) = O (e −n /2
).
1
On choisit λ = 2 (1/2 + 2/3) = 7/12 de sorte que 2λ − 1 > 0 et λ < 2/3. Avec ce choix de λ, les
estimations précédentes deviennent
1/6/2
|α n | ≤ 4M n−1/4 An (1) et |β n | = O (e−n n 5/12) = o(1).
Comme An (1) ≥ 1 on a β n = o(A n (1)), donc A n (1) − Bn (1) = 2 + αn + βn = 2 + o(An(1)).
Ceci entraı̂ne Pn (1) − Rn (1) = (n n/n!)(A n (1) − Bn (1)) = 2nn /n! + o(P n(1)) = o(en ). Comme
Pn (1) + Rn(1) = en on en déduit P n(1) ∼ Rn (1) ∼ 12 en.
6. PROBLÈMES 293


Problème 13. Soit a
n une série complexe absolument convergence. On suppose que
pour tout entier k ∈ N , +∞
∗ k
n=0 an = 0. Montrer que la suite (an ) est nulle.

Solution. On va raisonner par l’absurde en supposant que la suite (an) n’est  pas nulle, de sorte
que M = supn∈N |an | > 0. Remarquons que (an) converge vers 0 (puisque  k an converge) et
donc si k ∈ N∗ et pour n assez grand, |an |k ≤ |a n| ce qui prouve que an converge. Comme
(an) tend vers 0, il existe n 0 ∈ N tel que |a n | < M/2 pour n ≥ n0 , donc il n’existe qu’un nombre
fini de valeurs de an qui vérifient |an | ≥ M/2, on en déduit que I = {n ∈ N | |a n | = M } est
non vide. Quitte à diviser tous les termes de la suite (an ) par M , on peut même supposer que
k
M = 1. On va mettre de coté les termes an pour n ∈ I , en montrant que Sk = n∈I an tend vers

0. Soit ε > 0 ; on fixe n1 ≥ n0 tel que n≥n 1 |an | < ε. Soit q = supn∈I,n≤n 1 |a n|. Par définition
de I, on a q < 1. et on écrit
 
|S k | ≤ |an |k + |an | ≤ n1 qk + ε.
n∈I,n≤n1 n≥n 1

Comme q < 1 on en déduit que |Sk | <k 2ε 


pour k suffisamment grand. On a donc bien montré que
+∞ k
limk→∞ Sk = 0. Comme S k + n∈I a n = n=0 an = 0, on en déduit que n∈I akn converge vers 0
lorsque k tend vers +∞. En notant z1 , . . . , zp les valeurs distinctes de l’ensemble 
{an , n ∈ I } (on
a |zi | = 1), et en notant ni = Card{n ∈ I | an = z i}, ceci s’écrit aussi limk→+∞ pi=1 ni zik = 0.
On utilise maintenant le résultat plus général suivant
Lemme 1. Soit z1 , . . . , z p des nombres complexes distincts tels que |zi | = 1 pour tout i, et

λ1, . . . , λ p des nombres complexes tels que limk→∞ pi=1 λizik = 0. Alors λ 1 = . . . = λp = 0.
Prouvons ce résultat intermédiaire par récurrence sur p. Pour p = 1 c’est immédiat car |z1 | =
1. Supposons le résultat vrai au rang p − 1 et montrons le au rang p. Notons u k = pi=1 λi z ki .
Comme (uk ) tend vers 0, uk+1 − zp uk tend également vers 0 lorsque k → ∞. Or u k+1 − zp u k =
p−1 k
i=0 (λi z i − λ izp )zi . D’après l’hypothèse de récurrence, on a donc 0 = λ i zi − λ izp = λi(z i − z p)
pour tout i < p, et comme les z i sont distincts ceci entraı̂ne λi = 0 pour i < p. On a donc
limk→∞ λp zpk = 0 et comme |z p| = 1 ceci entraı̂ne également λp = 0. On a donc prouvé le lemme.

Le résultat de l’exercice découle du lemme : comme limk→+∞ pi=1 ni zik = 0, on a forcément
les ni nuls, ce qui est impossible puisque par construction, les n i sont des entiers naturels non
nuls. On a donc aboutit à une contraction, donc la suite (an ) est nulle.

Problème 14. a) Soit f : [0, 1] → R une fonction continue et a 1, . . . , an des points


distincts de [0, 1]. Montrer qu’il existe une suite de polynômes (Pk ) qui converge uni-
formément vers f sur [0, 1] et telle que pour tout k , Pk prend les mêmes valeurs que f
aux points a1 , . . . , a n (on pourra utiliser le théorème de Weierstrass, voir page 235).
b) Soit f : [0, 1] → R une fonction continue qui ne s’annule qu’un nombre fini de fois sur
[0, 1]. Existe t-il une suite de polynômes (Pk ) qui converge uniformément vers f sur [0, 1]
et telle que pour tout k , Pk a les mêmes zéros que f sur [0, 1] ?

Solution. a) D’après le théorème de Weierstrass, il existe une suite de polynômes (Qk ) qui
converge uniformément vers f sur [0, 1]. L’idée est de perturber légèrement les Qk pour que
ceux ci prennent les mêmes valeurs que f sur les points (ai )1≤i≤n . Pour cel à on considère les
polynômes d’interpolation de Lagrange (voir le tome Algèbre) définis par
 x − aj
L i(x) = .
ai − a j
j =i

Ils vérifient
 nL i(xj ) = 0 si i = j , Li (xi) = 1. Pour tout k, on définit maintenant P k(x) =
Qk (x) + i=1(f (ai ) − Q k (a i))Li (x). La fonction P k est un polynôme qui prend les mêmes
valeurs que f sur les points (a i )1≤i≤n , et en utilisant la notation de la norme de la convergence
294 4. SUITES ET SÉRIES

uniforme g ∞ = supx∈[0,1] |g (x)| pour toute fonction réelle continue g sur [0, 1], on a
n
 n

f − Pk ∞ ≤ f − Q k∞ + |f (a i) − Qk (ai )| · Li ∞ ≤ M f − Q k ∞ , M = 1+ |Li∞ .
i=1 i=1

Comme f − Qk ∞ converge vers 0 on en déduit que f − Pk  ∞ également, d’où le résultat.


b) La réponse est oui ! Si f ne s’annule pas il suffit d’approcher f par une suite de polynômes
(P k ) et de remarquer que les Pk ne s’annulent pas sur [0, 1] à partir d’un certain rang (dès que
Pk − f  ∞ < minx∈[0,1] |f (x)|). Si f s’annule on ne peut pas se contenter d’appliquer le résultat
de la question précédentes aux zéros de f , car il faut s’assurer que les Pk n’ont pas d’autres zéros
que ceux de f .
On va s’en sortir en approchant f par une suite de fonctions (gk ) qui s’écrivent sous la forme
gk = P h k , avec P un polynôme fixé qui a les même zéros que f sur [0, 1] et les hk continues ne
s’annulant pas sur [0, 1], puis en approchant hk par un polynôme ne s’annulant pas sur [0, 1].
Pour cela on va choisir g k localement polynomiale au voisinage de chaque zéro de f . Plus
précisément, notons a1 , . . . , an les zéros de f . Pour tout i, notons mi =
 1 si f change de signe
au voisinage de ai , m i = 2 sinon. On choisit pour P le polynôme P = ni=1 (x − ai )m i , de sorte
que f P garde un signe constant sur [0, 1]. Soit k ∈ N∗. Soit δ > 0 suffisamment petit tel que les
segments [ai − δ, ai + δ] soient deux à deux disjoints et tels que |f | < 1/k sur ces segments. On
définit la fonction gk par gk (x) = f (x) lorsque x n’est pas dans l’un des segments [ai − δ, ai + δ],
et sur chacun de ces segments on définit gk affine sur [ai − δ, ai − δ/2] et sur [ai + δ/ 2, a i + δ] de
sorte que
 
gk (ai − δ) = f (ai − δ ), gk (ai + δ/ 2) = εi /k,
et
gk (ai − δ/2) = εi /k gk (ai + δ) = f (a i + δ )

( où εi = f (a i − δ )/|f (a i−δ)| ∈ {−1, 1} a le signe de f (a i−δ ) et εi = f (a i+ δ )/|f (ai + δ )| ∈ {−1, 1}
a le signe de f (ai + δ)). On définit ensuite, pour tout i

(x − ai ) mi εi
∀x ∈ ]ai − δ/2, ai + δ/ 2[ , g k(x) = .
(δ/2) mi k

Ainsi g est définie sur [0, 1], continue sur cet intervalle, a les mêmes zéros que f et de plus
f − gk ∞ < 1/k . On définit maintenant hk (x) = gk (x)/P (x) si x ∈ {a1 , . . . , an} et hk (ai ) =
εi/(k (δ/2) mi ). Ainsi définie, h k est une fonction continue qui ne s’annule pas sur [0, 1] et qui
verifie gk = P hk . Comme on l’a vu au début de la solution de cette question, on peut donc
trouver un polynôme Qk ne s’annulant pas sur [0, 1] tel que h k − Qk  ≤ 1/k .

1 + P ∞
f − P Qk ∞ ≤ f − P hk ∞ + P hk − P Qk  ∞ ≤ f − gk  ∞ + P ∞ · Q k − hk ∞ ≤ .
k
Ainsi, les Pk = P Qk sont des polynômes qui convergent uniformément vers f sur [0, 1] et qui
ont les mêmes zéros que f .

Problème 15. Pour tout entier naturel n, on note ν2(n) le nombre de “1” dans l’écriture
 ν2 (n)
binaire de n. Montrer que la série converge et calculer sa somme.

n(n + 1)
n∈N


Solution. Soit n ∈ N∗ et n = pk=0 ε k 2k son écriture binaire (εk ∈ {0, 1} pour tout k et εp = 1).
On a ν (n) = pk=0 ε k ≤ p + 1. Par ailleurs, 2 p = ε p2p ≤ n donc p ≤ log 2 n, donc finalement
√ √
ν2 (n) ≤ 1 + log2 n. Comme 1 + log2 n = O ( n) lorsque n → +∞, on en déduit ν 2(n) = O ( n),
 ν2 (n)
donc nν(2n(+1)
n)
= O (n −3/2 ). La série à termes positifs n(n+1) est donc convergente.
6. PROBLÈMES 295

Pour calculer sa somme S, on remarque que pour tout entier naturel n, ν2 (2n) = ν2 (n) et
ν2 (2n + 1) = ν2 (n) + 1. En séparant les termes pairs et impairs de la série, ceci entraı̂ne
+∞
 +∞
 +∞
 +∞

ν2 (2n) ν 2 (2n + 1) ν2 (n) ν2(n) + 1
S= + = +
n=1
2n(2n + 1) n=0 (2n + 1)(2n + 2) n=1 2n(2n + 1) n=0 (2n + 1)(2n + 2)
+∞
   +∞

1 1 1
= ν2(n) + +
n=1
2n(2n + 1) (2n + 1)(2n + 2) n=0
(2n + 1)(2n + 2)
+∞
 2ν2 (n)  1
+∞
1
 +∞
S  (−1) n−1
= + − = + .
n=1
2n(2n + 2) n=0 2n + 1 2n + 2 2 n=1 n
+∞ (−1)n−1
Il est bien connu que n=1 n = log 2 (voir par exemple la remarque de l’exercice 10
page 263), donc finalement, on a S = S/2 + log 2, donc S = 2 log 2.

Problème 16 (Théor ème de réalisation de Borel). Soit (a n)n∈N une suite com-
plexe quelconque.
Soit ϕ : R → R une application de classe C∞ telle que ϕ(x) = 1 pour x ∈ [−1, 1] et
ϕ(x) = 0 pour |x| ≥ 2 (une fonction de ce type est construite dans l’exercice 3 page 79).
(n−1)
On pose ϕn (x) = x n ϕ(x), on note Mn un majorant de |ϕn |, |ϕn|, . . . , |ϕn | sur R, et on
choisit
 une suite réelle (λn)n∈N vérifiant λn ≥ 1 pour n ∈ N, tendant vers +∞, et telle
que |an|M n /λn converge. 
Montrer que la fonction f : x → +∞ n
n=0 anx ϕ(λn x) est bien définie, de classe C

sur
(n)
R et vérifie f (0)/n! = an pour tout n ∈ N.

Solution. Notons d’abord que Mn existe bien car ϕ n est à support compact, et qu’on peut choisir
par exemple λn = 1 + n2 (1 + |an|Mn ). La fonction f est bien définie en x = 0, et pour x = 0
également car les termes de la série sont nuls à partir d’un certain rang (dès que |λnx| > 2).
Montrons maintenant par récurrence sur p ∈ N que f est de classe Cp sur R et que
+∞

(p)
f (x) = a nλ pn−n ϕ(np) (λn x). (∗)
n=0

Pour p = 0, l’écriture (*) découle de la définition de ϕn , et la série (*) converge normalement car
pour n ≥ 1, |anλ −n n ϕ n (λn x)| ≤ |an|Mn /λn , d’où la continuité de f sur R. Supposons maintenant
le résultat vrai au rang p et montrons le au rang p + 1. La série des termes dérivés de (*)
 (p+1)
a nλ pn+1−n ϕ n (λn x) converge bien normalement car pour n ≥ p + 2, on a la majoration
p+1−n (p+1)
|an λ n ϕn (λn x)| ≤ |an|M n /λn . On en déduit que f (p) est de classe C1 (donc f de classe
C p+1) et que la formule (*) est bien vraie au rang p + 1.
Il suffit maintenant de remarquer que ϕn (x) = x n pour x ∈ [−1, 1] donc ϕ(pp) (0) = p! et
(p)
ϕn (0) = 0 si n = p. Avec la formule (*), on en déduit que f (p) (0) = p!ap .

Problème 17. Soit (an )n∈N une suite strictement croissante d’entiers, avec a0 > 0. Pour

tout n ∈ N, on pose bn = ppcm (a 0 , a1 , . . . , a n). Étudier la nature de la série 1/bn .

Solution. Nous allons montrer que la série converge. La suite (bn) est croissante. Soit ϕ la
fonction strictement croissante de N dans N telle que
∀n ∈ N, b ϕ(n) = bϕ(n)+1 = · · · = b ϕ(n+1)−1 < b ϕ(n+1) .
Comme bϕ(n) divise b ϕ(n+1), on a bϕ(n+1) ≥ 2bϕ(n), donc b ϕ(n) ≥ 2n pour tout n ∈ N.
296 4. SUITES ET SÉRIES

Par ailleurs,
ϕ(n+1)−1
 1 ϕ(n + 1) − ϕ(n)
∀n ∈ N, Pn = = . (∗)
bk b ϕ(n)
k=ϕ(n)

Si bk = bk+1 , cela signifie que ak+1 | bk . En particulier, les entiers a ϕ(n)+1, . . . , aϕ(n+1)−1 divisent

b ϕ(n) pour tout n. Or tout entier N a au plus 2 N diviseurs, car l’application
 

√ N
{d ∈ N | d | N } → {1, 2, . . . , N } d → inf d,
d

est telle que tout élément de l’image d’arrivée a au plus deux antécédents. Donc b ϕ(n) a au plus
 
2 bϕ(n) diviseurs, donc ϕ(n + 1) − ϕ(n) ≤ 2 bϕ(n). On en déduit avec (*) que

2 2
Pn ≤  ≤ n/2 ,
bϕ(n) 2
 
donc Pn converge, et comme les séries considérées sont à termes positifs, 1/bn converge.

Problème 18 (Transformation d’Euler). 1/ Soit une suite réelle u = (u n)n∈N. On


note ∆u la suite de terme général (∆u) n = un+1 − un . On note ∆ 0u = u et pour p ∈ N∗
on note ∆p u la suite définie par ∆p u = ∆(∆ p−1 u).
a) En désignant par Ckp le coefficient binomial p!/(k !(p − k )!), montrer que

p

p
∀p, n ∈ N (∆ u)n = (−1)p−kCpk un+k .
k=0


b) On suppose que la série (−1)n un converge (on ne suppose rien de plus sur (u n)) et
on note S sa somme. Montrer que pour tout n ∈ N on a

n n+1 ∞
(−1)p p 1  p 
S− (∆ u)0 = n+1 C n+1 Rp, où Rp = (−1)k uk. (∗)
p=0
2p+1 2 p=0 k=p

 p −p−1
c) En déduire que la transformée d’Euler, définie par la série p (−1) 2 (∆ pu) 0 , est
convergente et que
+∞
 +∞

(−1) p p
(∆ u)0 = (−1) n un .
p=0
2 p+1 n=0


2/ a) Appliquer ce résultat à la série (−1)n /(n + 1) en explicitant les termes de la
transformée d’Euler correspondante.

b) Faire de même pour la série (−1)n /(2n + 1).

Solution. 1/a) On procède par récurrence sur p. Pour p = 0 c’est immédiat. Supposons le
résultat vrai pour p ∈ N et montrons le pour p+1. Il suffit d’écrire (∆p+1 u)n = (∆p u)n+1 −(∆p u)n
6. PROBLÈMES 297

et d’appliquer l’hypothèse de récurrence à (∆p u)n+1 et (∆p u)n , ce qui donne


p
 p

p+1 p−k
(∆ u) n = (−1) Cpk un+1+k − (−1)p−kC pk u n+k
k=0 k=0
p+1 p
 
= (−1)p+1−k C k−1
p un+k − (−1)p−k C pk un+k
k=1 k=0
p
= (−1)p+1−k (C k−1
p + C pk) un+k − (−1) p−kun + u n+1+p
k=1
p+1

= (−1)p+1−k C kp+1 un+k ,
k=0

où nous avons utilisé l’identité C k−1


p + C kp = Ckp+1 pour 1 ≤ k ≤ p.
b) Nous allons également procéder par récurrence sur n. Pour n = 0, compte tenu du fait que
R0 = S , on a bien
1 u0 1 1
S − (∆0 u) 0 = S − = (S + S − u0 ) = (R0 + R 1).
2 2 2 2
Supposons maintenant le résultat vrai pour n − 1 et montrons le pour n. L’identité Cpn+1 =
Cpn + C p−1
n pour 1 ≤ p ≤ n permet d’écrire
n+1
 n
 n
 n
 n

Cpn+1 Rp = R0 + C pn R p + Cnp−1 Rp + Rn+1 = Cnp Rp + C pn Rp+1
p=0 p=1 p=1 p=0 p=0
n
 n
 n

= Cnp (2Rp − (−1) p+1u p+1 ) = 2 p
Cn R p − (−1) p+1Cnp up+1 .
p=0 p=0 p=0

D’après le résultat de la question a), la dernière somme est égale à (−1)n+1 (∆n u)0 , on a donc
n
 n+1

n n p
(−1) (∆ u)0 = 2 C np Rp − Cn+1 R p. (∗∗)
p=0 p=0

Or d’après l’hypothèse de récurrence on peut écrire


n
 n−1
 n
(−1)p p (−1)p p (−1)n n 1  p (−1)n n
S− (∆ u) 0 = S − (∆ u) 0 − (∆ u) 0 = C n Rp − (∆ u)0.
p=0
2p+1 p=0
2 p+1 2n+1 2n p=0 2n+1

En remplaçant (−1) n (∆n u)0 par l’expression à droite de (**), on en déduit l’identité (*), ce qui
achève la preuve par récurrence.
−n
n k
c) Compte tenu du résultat de la question précédente, il suffit
 de prouver que 2 k=0 Cn R k
n
tend vers 0 lorsque n → +∞. Soit ε > 0. Comme la série (−1) u n converge, la suite de ses
restes (R k) converge vers 0. Soit N ∈ N ∗ tel que pour tout k ≥ N , |R k| < ε. Comme la suite
(Rk) converge vers 0, elle est bornée, donc ∃M > 0 tel que |R k | ≤ M pour tout k. Pour tout
n ≥ N on a
 
 n  N−1 n N−1 n
 1  M  k ε  k M  k ε  k N nN−1
 n Cnk R k  ≤ n Cn + n Cn ≤ n n + n Cn ≤ M + ε.
2  2 2 2 2 2n
k=0 k=0 k=N k=0 k=0
N−1 n
Comme N est un entier fixé, N n /2 tend vers 0 lorsque n → +∞ donc lorsque n est
suffisamment grand, tout ceci est majoré par 2ε. On en déduit le résultat.
2/ a) Dans notre cas, on a un = 1/(n + 1). La formule de la question 1/a) donne
n k  1 n
  1
n n−k Cn n k k k n (−1) n
(∆ u)0 = (−1) = (−1) (−1) Cn x dx = (−1) (1 − x)n dx = .
k+1 0 0 n+1
k=0 k=0
298 4. SUITES ET SÉRIES

On en déduit, d’après l’identité de la transformée d’Euler obtenu à la question 1/c), que


+∞
 +∞
(−1)n  1
= n+1
.
n=0
n + 1 n=0
(n + 1)2
Notons que la première série a pour somme log 2 (c’est classique, voir par exemple la remarque
de l’exercice 10 page 263), et l’égalité ci dessus se retrouve en écrivant que log 2 = − log(1 − 1/2)
à partir du développement en série entière de log(1 + x).
b) Ici un = 1/(2n + 1) et on a
n
  1 
n
 
n Cnk 1  n
(∆ u)0 = (−1)n−k = (−1)n (−1) k
C kn x2k n
dx = (−1) 1 − x2 dx.
2k + 1 0 0
k=0 k=0
Notons In la dernière intégrale. Une intégration par partie donne, lorsque n ≥ 1,
 1
   
2 n 1 2
 
2 n−1
In = x 1 − x 0
+ (2nx ) 1 − x dx = 2n(I n−1 − In ).
0
On en déduit In = (2n/(2n + 1))I n−1 . Compte tenu de la valeur I0 = 1, une récurrence facile
donne In = (2 · 4 · · · (2n))/(3 · 5 · · · (2n + 1)). L’identité de la transformée d’Euler donne donc
+∞
 +∞
 1 +∞
(−1)n 2 · 4 · · · (2n) 1 n!
= = .
n=0
2n + 1 n=0 2n+1 3 · 5 · · · (2n + 1) 2 n=0 1 · 3 · · · (2n + 1)
La somme de la série de gauche est égale à π/4 = arctan(1) (ici aussi, voir la remarque de l’exer-
cice 10 page 263). La formule obtenue permet donc d’obtenir une série convergeant rapidement
vers π/4.
Remarque. La transformée d’Euler est en général utilisée pour accélérer la convergence
des séries alternées. Sous certaines hypothèses, on peut démontrer que la convergence est
O (1/2n ), comme c’est le cas dans le problème suivant qui généralise la transformée d’Euler
sous des conditions particulières de la suite (un ).

Problème 19 (Accélération de la convergence des séries altern ées). Soit


w une fonction réelle intégrable sur ]0, 1[ . On définit la suite (un ) par
 1
∀n ∈ N, un = tn w (t) dt.
0
On se donne une suite de polynômes (P n ) à coefficients réels telle que ∀n ∈ N, deg Pn = n
et Pn(−1) = 0.
1/a) Montrer que
 1
1 Pn (−1) − Pn (x)
Sn = w (x) dx
Pn (−1) 0 1+x
s’écrit comme une combinaison linéaire des termes (u k)0≤k<n et expliciter ses coefficients
en fonction de ceux de P n.
b) Montrer que la série (−1)n un converge, et que sa somme S vérifie
 1
Mn |w (t)|
|S − Sn | ≤ I, avec Mn = sup |Pn (x)|, I = dt.
|Pn (−1)| x∈[0,1] 0 1+t

2/a) On choisit comme suite de polynômes Pn (x) = (1 − x)n . Expliciter S n et donner une
majoration de |S n − S | en fonction de I et n.
b) Répondre à la même question avec la suite de polynômes P n(x) = (1 − 2x)n .
c) Montrer l’existence et l’unicité d’une suite de polynômes (P n)n∈N vérifiant deg Pn = n
et Pn(sin2 t) = cos(2nt) pour tout t ∈ R. Avec ce choix de polynômes, majorer |S − S n |.
6. PROBLÈMES 299

3/a) Montrer que l’on peut apliquer les processus d’accélération de convergence
 des ques-
tions précédentes aux séries (−1) n /(n + 1)s , où s > 0, et à la série (−1)n (log n)/n.
b) En utilisant l’accélération fondée surles polynômes de la question 2/c) de degré 4, cal-
culer une approximation numérique de +∞ n
n=1(−1) (log n)/n et en déduire une approxima-
+∞
tion numérique de la constante d’Euler γ (on s’appuiera sur l’égalité n=1(−1) n (log n)/n =
(γ − 12 log 2) log 2 établie à l’exercice 11 page 226).
n k
Solution. 1/ a) Ecrivons Pn sous la forme P n(x) = k=0 ak (−x) . On a
 
n
 n
 k−1
 n−1
  n
 
Pn (−1) − Pn (x) = ak 1 − (−x)k = (1 + x) ak (−x)j = (1 + x)  ak  (−x)j .
k=0 k=1 j=0 j=0 k=j +1

On en déduit l’expression de Sn comme combinaison linéaire des (u k )0≤k<n en écrivant


   
 1 n−1
 n n−1
 n
1  1
Sn = ak  (−x)j w (x) dx = (−1)j  ak  u j. (∗)
Pn (−1) 0 j=0 Pn(−1) j=0
k=j +1 k=j +1

b) Compte tenu de la définition des un , on a


n  1 n
  1
k 1 + tn+1 k w (t)
(−1) uk = w (t) dt donc lim (−1) uk = dt,
0 1+t n→+∞ 0 1+t
k=0 k=0
où nous avons utilisé le théorème de convergence dominée, la fonction intégrée dans l’intégrale
de gauche étant majorée en valeur absolue par la fonction intégrable |w | indépendamment de n.
  1 (t)
Ainsi la somme de la série (−1)n un est égale à S = 0 w 1+t dt. On a maintenant
 1  1
1 Pn (x) Mn |w (x)| Mn
S − Sn = w (x) dx donc |S − S n | ≤ dx = I.
P n(−1) 0 1 + x |Pn(−1)| 0 1 + x |Pn(−1)|
Notons que si w est positive, on a I = S et la majoration devient |S − Sn| ≤ |PnM(−
n
1)| S.

2/a) En choisissant Pn (x) = (1 − x)n , on a Pn (−1) = 2n , Pn (x) = nk=0 Cnk (−x)k et M n = 1.
Avec la formule (*) on en déduit
 
n−1
 n
1 I
Sn = n (−1)j  C kn u j , et |S − Sn | ≤ n .
2 j=0 2
k=j +1

b) Avec Pn (x) = (1 − 2x)n , on a Pn (−1) = 3 n, P n (x) = nk=0 2k Cnk (−x)k et Mn = 1, d’où
 
n−1 n
1   I
Sn = n (−1)j  2k Cnk uj et |S − S n | ≤ n .
3 3
j=0 k=j +1

c) L’existence et l’unicité de la suite de polynômes (Pn) découle de l’écriture


n

 2n

cos(2nt) =  (cos t + i sin t) = C 2k
2n(−1)
n−k
cos2k t sin 2(n−k) t = Pn (sin2 t)
k=0
avec
n

Pn (x) = C 2k k
2n(1 − x) (−x)
n−k
. (∗∗).
k=0
Pour majorer |S − Sn |, on remarque déjà que l’égalité Pn (sin2 t) = cos(2nt) entraı̂ne M n = 1.
Calculons maintenant Pn (−1). L’égalité Pn(sin 2 t) = cos(2nt), vraie pour t ∈ R, l’est aussi
pour t ∈ C (en effet, on aurait pu démontrer l’identité cos(2nt) = Pn (sin2 t) en partant de
2 cos(2nt) = (cos t + i sin t)2n + (cos t − i sin t) 2n qui est vraie pour tout t ∈ C. On aurait pu
aussi le démontrer en notant que Pn(sin 2 t) − cos(2nt) est une série entière, nulle sur R donc
sur C d’après le principe des zéros isolés). En choisissant t = t0 tel que sin t 0 = i on va calculer
300 4. SUITES ET SÉRIES

it −it
Pn (−1). L’équation sin t0 = i équivaut à e 0−e 0
= i, ou encore (e it0 ) 2 + 2eit0 − 1 = 0, vérifiée
√ 2i √
si on choisit eit0 = 2 − 1, ce qui est le cas avec t0 = −i log( 2 − 1). On a alors
√ √ √ √
e 2int0 + e−2int0 ( 2 − 1)2n + ( 2 + 1) 2n (3 + 8) n + (3 − 8)n
Pn (−1) = cos(2nt 0) = = = .
2 2 2
√ n
Finalement on obtient la majoration |S − Sn | ≤ 2I/(3 + 8) .
3/a) On part du résultat classique suivant, obtenu avec le changement de variable u = nx
 +∞  +∞
∗ −nx s−1 1 Γ(s)
∀n ∈ N , e x dx = s e−u us−1 du = s
0 n 0 n
où Γ( s) est la valeur de la dernière intégrale. Le changement de variable t = e−x dans la première
intégrale, entraı̂ne
 1  1
Γ(s) n−1 s−1 1 n logs−1 (1/t)
= t log (1 /t) dt donc = t w s(t ) dt, w s(t) = .
ns 0 (n + 1) s 0 Γ(s)
La fonction ws (t) est intégrable
 sur ]0, 1], nous sommes bien dans le cadre des séries de l’exercice
donc la convergence de ( −1) n /(n + 1)s peut être accélérée avec les méthodes précédentes.
Passons maintenant à (−1) n log n/n. En dérivant par rapport à s en s = 1 l’expression
précédente de 1/(n + 1)s (la dérivabilité est facile à partir du théorème de dérivation sous le
signe intégral des fonctions intégrables), on obtient
 1  1
log(n + 1) Γ (1) 1 n 1  
− =− 2
t dt + n
t log log(1/t) dt = t n log log(1/t) − Γ(1) dt.
n+1 Γ(1) 0 Γ(1) 0 0
Ainsi
 nous sommes bien dans le cadre des séries de l’exercice et la convergence de la série alternée
(−1)n log(n + 1)/(n + 1) peut être accélérée avec les méthodes précédentes.
b) En utilisant l’expression (**) du polynôme P4 (x) défini dans la question 2/c), on obtient
P4 (x) = x 4 − 28(1 − x)x 3 + 70(1 − x) 2 x2 − 28(1 − x)3 x + (1 − x)4 = 1 − 32x + 160x 2 − 256x 3 + 128x4

et P4 (−1) = 577. Pour l’accélération de S = +∞ n
n=0 (−1) log(n +1)/(n + 1), on en déduit, compte
tenu de l’expression (*),
 
1 log 2 log 3 log 4 1
S4 = −544 + 384 − 128 = (−336 log 2 + 128 log 3) ≈ −0, 159922 . . .
577 2 3 4 577
1
Pour majorer I = 0 | log log(1/t) − Γ(1)|/(1 + t) dt, on commence par majorer Γ (1). On a
 +∞ 1  +∞
 −t 
Γ (1) = e log t dt donc |Γ (1)| ≤ | log t| dt + e−t t dt = 2.
0 0 0
−x
Ensuite le changement de variable t = e donne
 1  +∞  1  +∞
−x
| log log(1/t)| dt = | log x|e dx ≤ | log t| dt + e−t t dt = 2.
0 0 0 0
On déduit de tout ceci la majoration I ≤ 4. Avec le résultat de la question 2/c) , on a donc

|S − S4 | ≤ 8/(3 + 8) 4 ≤ 0, 007.

Donc S ≈ 0, 159 à 0, 007 près. Or −S = +∞ n 1
n=1 (−1) log n/n = (γ − 2 log 2) log 2 donc γ =
1
−S/ log 2 + 2 log 2 ≈ 0, 577 à 0, 007/ log 2 ≈ 0, 01 près.
Remarque. - L’accélération de convergence de séries alternées obtenue par le choix de
polynômes de la question 2/a) correspond à la transformée d’Euler, rencontrée dans le
problème précédent.
– La majoration de l’erreur obtenue en 3/b) est très grossière, car à partir de S4 on obtient
l’approximation γ ≈ 0, 577292 ce qui est précis à 8 × 10−5 près.
– La fonction Γ introduite dans la solution de 3/a) est étudiée dans le sujet d’étude 1
page 315. Dans la question 3/d) de ce sujet d’étude on montre que Γ (1) = −γ.
6. PROBLÈMES 301

– Les polynômes Pn définis dans la question 2/c) vérifient Pn (x) = T n(1 − 2x) où les
Tn sont les polynômes de Tchébycheff de première nespèce (voir le tome Algèbre). On
peut montrer qu’ils ont la forme explicite P n(x) = k=0 n+k Cn2k+k 4 k(−x) k , ce qui permet
n

d’expliciter l’expression de Sn avec ce choix de polynômes.


Problème 20. Soit an zn une série entière de rayon de convergence ≥ 1, telle que
sa somme f se prolonge en une fonction continue (toujours notée f ) sur le disque unité
fermé. On suppose que
∃α ∈ R, ∃θ > 0, ∀t ∈ [α, α + θ ], f (eit ) = 0. (∗)
Montrer que f est la fonction nulle.

Solution. Soit N ∈ N ∗ tel que N θ > 2π . Considérons la fonction


g : D = {z ∈ C, |z | ≤ 1} → C z → f (z )f (zeiθ ) · · · f (zei(N −1)θ ).

D’après (*), g est nulle sur le cercle unité. Or f est la somme d’une série entière sur D, et par
un produit de Cauchy on en déduit que g est la somme d’une série entière bn z n sur le disque
unité ouvert. Soit n ∈ N. La formule de Cauchy (voir le théorème 4 page 250) donne
 2π
n 1
∀r ∈ ]0, 1[ , bn r = g(re it )e−int dt. (∗∗)
2π 0
La fonction [0, 1]× [0, 2π ] → C (r, t) → g (reit) est continue (car g est continue sur D), et d’après

le théorème de continuité sous le signe intégral, l’intégrale de (**) tend vers 02π g (eit )e−int dt = 0
lorsque r → 1− . On en déduit b n = 0 d’après (**). Ceci est vrai pour tout n ∈ N, donc g est
nulle.
En particulier, g (1/n) = 0 pour tout n ∈ N∗. La forme de g montre donc qu’il existe k et une
infinité d’entiers n pour lesquels f (ekiθ /n) = 0. Comme f , la fonction z → f (eikθ z) est la somme
d’une série entière sur le disque unité. D’après le principe de zéros isolés (voir le théorème 3
page 250) on en déduit que f (eikθ z ) = 0 pour tout z ∈ D . La fonction f est donc nulle.
Remarque. On aurait pu utiliser l’égalité de Parseval au lieu de la formule de Cauchy.

+∞ n
Problème 21 (Th éorème de Bieberbach, cas réel). Soit f (z ) = n=0 a n z la
somme d’une série entière de rayon de convergence 1, à coefficients réels, telle que a0 = 0
et a1 = 1. On suppose que f est injective sur le domaine D = {z ∈ C : |z | < 1}. On veut
montrer que |an | ≤ n pour n ≥ 1.
a) Montrer que pour tout r tel que 0 < r < 1, on a
 π
πan rn
= (f (reiθ)) sin nθ dθ. ((u) désigne la partie imaginaire de u)
2 0

b) Lorsque θ ∈ [0, π], étudier le signe de (f (reiθ )), puis montrer l’inégalité | sin(nθ)| ≤
n| sin θ |. En déduire |an | ≤ n pour tout n ∈ N∗.
c) Montrer que l’inégalité précédente est la meilleure possible.

Solution. a) Soit r ∈ ]0, 1[. On a ak ∈ R pour tout k ∈ N donc (f (reiθ )) = +∞ k
k=0 ak r sin kθ , et
comme cette série est normalement convergente, on peut l’intégrer terme à terme ce qui donne
 π +∞
 π
iθ k
(f (re )) sin nθ dθ = ak r sin kθ sin nθ dθ.
0 k=0 0
302 4. SUITES ET SÉRIES

1
Il
 π suffit ensuite de remarquer que sin kθ sin nθ = 2 (cos(  π k − n)θ − cos(k + n)θ ), et comme
0
cos pθ dθ = 0 si p =
 0, = π sinon, on en déduit 0
sin kθ sin nθ dθ = 0 si k = n, = π/ 2
si k = n, d’où le résultat demandé.
b) On a f (z ) ∈ R si et seulement si f (z ) = f (z ) = f (z ), et comme f est injective, ceci
n’est possible que si z = z, c’est-à-dire z ∈ R. Ainsi, (f (z)) ne s’annule pas sur le connexe
D + = {z ∈ D, (z ) > 0} et comme l’image d’un connexe par une application continue est un
connexe, (f (z )) garde un signe constant sur D +. Comme f (z) = z + o(z) lorsque z → 0, on en
déduit que (f (z )) > 0 sur un voisinage de 0 dans D +, donc dans D + tout entier.
L’inégalité | sin(nθ )| ≤ n| sin θ| pour θ ∈ R se montre facilement par récurrence sur n à partir
de l’identité sin(n + 1)θ = sin nθ cos θ + sin θ cos nθ .
On utilise maintenant l’identité démontrée dans la question précédente, qui implique
 π  π
2 iθ 2
|an| ≤ |(f (re ))| · | sin nθ| dθ ≤ n (f (reiθ )) n sin θ dθ = na1 = n
πrn 0 πr 0
où nous avons une nouvelle fois utilisé l’identité de a) pour n = 1.
 +∞ n
c) L’inégalité est bien la meilleure possible, car on vérifie facilement que n=0 nz = z/(1 − z)2
est injective sur D.
Remarque. Le théorème de Bieberbach dans le cas général s’exprime sans la condition
que les coefficients an soient réels. Il a été annoncé comme une conjecture en 1916 par
Ludwig Bieberbach, qui avait démontré uniquement |a 2| ≤ 2. Après de nombreux résultats
partiels, le cas général n’a été démontré qu’en 1984 par Louis de Branges.

Problème 22 (Fonction z êta de Riemann). Pour tout s > 1, on pose


+∞
 1
ζ (s) = s
.
n=1
n

a) Montrer que ζ définit une fonction de classe C∞ sur ]1, +∞[ et exprimer ses dérivées
successives.
b) Montrer que ζ converge en +∞ et que lorsque s → 1+ , ζ (s) = 1/(s − 1) + γ + o(1) (où
γ désigne la constante d’Euler).
c) On note (pn )n∈N ∗ la suite des nombres premiers rangés dans l’ordre croissant. Montrer
la formule (identité due à Euler)
+∞ 
 1
+∞   N
∀s > 1, ζ(s) = −s
= lim .
n=1
1 − p n n=1
N→+∞
n=1

d) Montrer que la série 1/pn diverge.

Solution. a) Pour démontrer que ζ est de classe C ∞ sur ]1, +∞[ , il suffit de montrer qu’elle est
de classe C∞ sur [a, +∞[ pour tout a > 1. 
Fixons donc a > 1. La fonction ζ est limite simple de la sériede fonctions 1/ns sur
∗ p p s
]1, +∞[. Pour tout p ∈ N , montrons que la série des dérivées p-ièmes (−1) log n/n converge
uniformément sur [a, +∞[. Pour p ∈ N∗ fixé, on a, en écrivant a = 1 + 2h (h > 0)
 
logp n logp n logp n 1 1
lim = 0 donc = =o ,
n→+∞ nh na nh n1+h n1+h

donc la série logp n/na converge. Comme logp n/n s ≤ log p n/n a pour tout s ≥ a, on en déduit
que (−1) p logp n/n s converge normalement donc uniformément sur [a, +∞[. Ainsi (voir le
premier alinéa de la remarque 5 page 234), la fonction ζ est de classe C ∞ sur [a, +∞[ et sur cet
6. PROBLÈMES 303

intervalle, on a
+∞

∗ (p) logp n
∀p ∈ N , ζ (s) = .
n=1
ns

b) On commence par une classique comparaison série-intégrale,


 n+1  +∞
∗ 1 dt 1 dt 1
∀s > 1, ∀n ∈ N , s
≤ s
≤ s donc ζ (s) − 1 ≤ s
= ≤ ζ (s)
(n + 1) n t n 1 t s−1
(par sommation sur n ≥ 1 de la première inégalité). Ceci entraı̂ne 1/(s − 1) ≤ ζ (s) ≤ 1+ 1/(s −1)
pour tout s > 1. On en déduit que ζ (s) ∼ 1/(s − 1) lorsque s → 1+ , et comme 1 ≤ ζ (s) ≤
1 + 1/(s − 1), ζ converge vers 1 en +∞.
Pour obtenir le second terme du développement asymptotique de ζ en s = 1 +, il faut raffiner
 +∞
la technique. Comme 1/(s − 1) = 1 dt/t s, on a
+∞
  n+1
1 1 dt
∀s > 1, ζ(s) − = un (s) où un (s) = s − . (∗)
s − 1 n=1 n n ts

Or
 n+1  n+1
1 1 s dt dt
∀s ∈ [1, 2], 0 ≤ u n (s) ≤ s − s
= s+1
≤2 .
n (n + 1) n t n t2

On en conclut que la série de fonctions un converge normalement sur [1, 2], donc que sa somme
est continue sur [1, 2], en particulier en 1+ . On en déduit
 n
1 1 1
lim ζ (s) − = lim u k (1) = lim 1 + + · · · + − log(n + 1) = γ,
s→1+ s−1 n→∞ n→+∞ 2 n
k=1

d’où le résultat.

c) Si s > 1, on a log(1 − p−sn )
−1
∼ p−s
n ≤ 1/n
−s
lorsque n → +∞, donc n log(1 − p−s n )
−1

converge, ce qui assure l’existence du produit infini pour tout s > 1. L’idée +∞dans ce qui suit
−s −1 −ns
repose sur le fait que pour tout k et pour tout s > 1, on a (1 − pk ) = n=0 pk .
Pour tous les entiers naturels non nuls m et M , pour tout s > 1, on a
 
m M 
 1  1
ik s
= . (∗∗)
k=1 i =0 k
(p ) 0≤i ,...,i ≤M
(p1 · · · pimm ) s
i1
k 1 m

Maintenant, donnons nous N ∈ N∗ .


Soit pm0 le plus grand nombre premier pi et M 0 la plus
grande des puissances ik apparaissant dans toutes les décompositions en facteurs premiers des
N premiers entiers 1, . . . , N . Considérons m ≥ m0 et M ≥ M 0. Tous les entiers compris entre
1 et N se retrouvent dans les entiers pi11 · · · pimm (0 ≤ i 1 , . . . , i m ≤ M ), donc le dernier terme

de (**) est supérieur à N s i1 im
n=1 1/n . Par ailleurs, les nombres p1 · · · pm représentent des entiers
distincts (unicité de la décomposition en facteurs premiers), et finalement, (**) montre
 
N m M +∞
1  1   1
∀s > 1, ≤ ≤ = ζ (s).
n=1
ns (pikk )s n=1
ns
k=1 ik =0

Cette expression est valable pour tout m ≥ m0 et pour tout M ≥ M 0 . En faisant tendre M puis
m vers +∞, on en déduit
 
N
1
+∞
 +∞
 1  
+∞
1

∀s > 1, ≤  = ≤ ζ (s).
ns
n=1
(p ikk )s
k=1 i k =0
1 − p−s
k k=1

Cette expression est valable indépendamment de N ∈ N ∗, on peut donc faire tendre N vers +∞,
ce qui fournit l’égalité voulue.
304 4. SUITES ET SÉRIES


d) Raisonnons par l’absurde. Si 1/p n converge, l’équivalent log[(1 − 1/pn )−1 ] ∼ 1/pn montre
 +∞ −1
que n=1(1 − 1/p n) converge. Notons  la valeur de ce produit infini. Pour tout s > 1,

+∞
1
 +∞
 
1

ζ (s) = ≤ = ,
n=1
1 − p −s
n n=1
1 − 1/p n

ce qui montre que ζ est majorée sur ]1, +∞[. Ceci est impossible puisque l’on a montré ζ (s) ∼
1/(s − 1) lorsque s → 1+, d’où le résultat.
Remarque. Le résultat de c) est généralisé au domaine complexe dans l’annexe C, et
constitue une des clés permettant de démontrer le théorème des nombres premiers.

– On peut montrer que nk=1 (1 −1/p
−γ
k ) ∼ e / log n où γ est la constante d’Euler (formule
n
de Mertens). On en déduit k=1 1/pk ∼ log(log n) lorsque n → +∞. Cette dernière
estimation est aussi une conséquence du théorème des nombres premiers (voir annexe C)
puisque ce dernier entraı̂ne pn ∼ n log n.
– A partir du développement en série entière de la fonction x → un (1+ x) et de l’expression
(*) on peut montrer que pour tout x > 0 on a le développement en série entière
+∞  p 
1  (−1)pγ p p log (1) log p(k ) logp+1 k
ζ (1 + x) − = x, γ p = lim + ··· + − .
x p=0 p! k→+∞ 1 k p+1

On a γ0 = γ et les termes γ p pour p ≥ 1 sont des généralisations de la constante d’Euler.

Problème 23 (Preuve du théor ème de Weierstrass par la convolution).


On note E l’e.v des fonctions continues de R dans C et nulles en dehors d’un compact.
On muni E de la loi de convolution  en définissant, pour tout (f, g) ∈ E 2 la convolée
 +∞
f  g : R → C x → f (x − t)g (t) dt.
−∞

1/ a) Montrer que la loi  est commutative et distributive par rapport à l’addition.


b) (Séquences de Dirac.) On appelle séquence de Dirac (on dit encore approximation de
l’identité) toute suite (χn ) de fonctions positives de E vérifiant
 +∞ 
∀n ∈ N, χn(t) dt = 1, et ∀α > 0, lim χn(t) dt = 0
−∞ n→+∞ |t|≥α
  −α  +∞
( |t|≥α signifie −∞ + α ). Soit (χn ) une telle suite et soit f ∈ E. Montrer que la suite
de fonctions (f  χ n )n∈N converge uniformément vers f sur R.
2/ Pour tout n ∈ N, on pose
1 
(1 − t2 ) n /an si |t| ≤ 1
an = (1 − t 2) n dt et pn : R → C t →
−1 0 sinon.

a) Montrer que (pn ) est une séquence de Dirac.


b) Soit f ∈ E, nulle en dehors de I = [−1/2, 1/2]. Pour tout n ∈ N, montrer que f  pn
est une fonction polynôme. Conclure.
c) En déduire le théorème de Weierstrass : si J est un segment de R et si f : J → C est
continue, alors f est limite uniforme sur J d’une suite de fonctions polynôme.
6. PROBLÈMES 305

Solution. 1/ a) La distributivité par rapport à l’addition de la loi  est immédiate. Pour prouver
qu’elle est commutative, il suffit d’effectuer le changement de variable u = x − t dans l’intégrale
définissant f  g.
b) Fixons ε > 0. La fonction f est continue et nulle en dehors d’un compact, elle est donc
uniformément continue sur R, donc
∃η > 0, ∀(x, y ) ∈ R2 , |x − y| < η, |f (x) − f (y )| ≤ ε.

Désignons par M un majorant de |f | sur R. Choisissons N ∈ N tel que |t|≥η χn (t) dt < ε pour
 +∞
tout n ≥ N . Comme −∞ χ n(t) dt = 1, et que les fonctions χ n sont positives, on a pour tout
n≥N
 +∞ 
   

∀x ∈ R, |f  χn (x) − f (x)| =  f (x − t) − f (x) χn (t) dt
 −∞  η
≤ |f (x − t) − f (x)| χn (t) dt + |f (x − t) − f (x)| χ n(t) dt
|t|≥η −η
 η  +∞
≤ 2M ε + ε χ n(t) dt ≤ 2M ε + ε χn (t) dt = (2M + 1)ε.
−η −∞

Ceci suffit pour conclure.


 +∞ 1
2/ a) On remarque que −∞ pn (t) dt = −1 p n (t) dt = 1 par définition de an . Par ailleurs,
 1  1  1
∗ 2 n 2 n −(1 − t2 )n+1 1
∀n ∈ N , an = 2 (1 − t ) dt ≥ 2 t(1 − t ) dt = =
0 0 n+1 0 n+1
donc si α > 0 (et α < 1),
 1
∗ 2 2
∀n ∈ N , pn (t) dt = (1 − t 2) n dt ≤ (1 − α 2 ) n ≤ 2(n + 1)(1 − α 2) n,
|t|≥α a n α an

et comme |1 − α2| < 1, ceci suffit pour conclure que |t|≥α pn (t) dt tend vers 0 lorsque n → +∞.
b) Pour montrer que f  pn est une fonction polynôme sur I, on commence par écrire
 1/2
∀x ∈ I, (f  pn)(x) = (p n  f)(x) = pn (x − t)f (t) dt. (∗)
−1/2

Lorsque x ∈ I et t ∈ I , on a |x − t| ≤ 1 donc pn (x − t) = (1 − (x − t) 2 )n /an , ce qui en développant



s’écrit sous la forme 2n k
k=0 q k(t)x (où pour tout k , q k est une fonction polynôme). En remplaçant
dans (*), on en déduit
2n
 
 1/2
∀x ∈ I, (f  pn )(x) = qk (t)f (t) dt xk ,
k=0 −1/2

qui est bien une fonction polynôme sur I .


Maintenant, on sait d’après 1/b) que (f  pn ) converge uniformément vers f sur R, en
particulier sur I . En définitive, nous venons de montrer que f est limite uniforme sur I d’une
suite de fonctions polynôme.
c) Soit [a, b] un segment de R et f : [a, b] → C une fonction continue. En considérant un
intervalle plus grand [c, d] avec c < a, b < d, on prolonge f sur [c, a] (resp. sur [b, d]) par une
fonction affine prenant la valeur 0 en c et la valeur f (a) en a (resp. la valeur f (b) en b et la
valeur 0 en d). On prolonge ensuite f sur R tout entier en prenant f (x) = 0 hors de [c, d]. Le
prolongement ainsi construit de f est continu sur R, nul en dehors d’un compact, donc f ∈ E.
On peut ensuite, en effectuant un changement de variable affine, se placer dans le cas où
[c, d] = [−1/2, 1/2]. La fonction f est alors limite uniforme de fonctions polynôme sur [c, d]
d’après la question précédente, en particulier sur [a, b] ⊂ [c, d].
Remarque. Une autre preuve de ce théorème fait l’objet de l’exercice 8 page 242.
306 4. SUITES ET SÉRIES

Problème 24. Dans ce problème, on pourra utiliser le théorème de Weierstrass (voir le


problème précédent ou voir page 235).
1
a) Soit f : [0, 1] → C une fonction continue, telle que pour tout n ∈ N, 0 f (t)t n dt = 0.
Montrer que f est la fonction nulle.  +∞
b) Soit f : R+ → C une fonction continue telle que l’intégrale 0 f (t) dt existe. Montrer
 +∞
que pour tout n ∈ N∗ , l’intégrale In = 0 f (t)e−nt dt existe. Si In = 0 pour tout n ∈ N∗ ,
montrer que f est la fonction nulle.

Solution. a) La propriété vérifiée par f entraı̂ne par linéarité 01 f (t)P (t) dt = 0 pour toute fonc-
tion polynôme P . D’après le théorème de Weierstrass, il existe une suite de fonctions polynôme
(P n) qui converge uniformément vers f sur [0, 1]. De plus, f est bornée sur [0, 1] (continue sur
un compact) donc la suite de fonctions (f Pn ) converge uniformément vers f f = |f |2 sur [0, 1].

Comme 01 f (t)Pn (t) dt = 0 pour tout n, on en déduit
 1  1
2
|f (t)| dt = lim f (t)Pn (t) dt = 0.
0 n→+∞ 0
La fonction |f |2
est continue est positive, elle est donc nulle sur [0, 1] d’où le résultat.
x
b) Considérons la primitive de f définie par F : x → 0 f (t) dt. Si n ∈ N ∗ , une intégration par
parties donne
X  X X  X
−nt −nt −nt −nX
∀X > 0, f (t)e dt = F (t)e +n F (t)e dt = F (X )e +n F (t)e −nt dt.
0 0 0 0
La fonction F est bornée
 +∞ car l’intégrale de f existe sur R, ce qui en particulier entraı̂ne l’existence
de l’intégrale Jn = 0 F (t)e −nt dt. Le membre de droite de la formule précédente converge donc
lorsque X → +∞, ce qui assure l’existence de In et montre que I n = nJn .
Si In = 0 pour tout n ∈ N∗ , on a donc Jn = 0 pour tout n ∈ N∗ , ce qui en effectuant le
changement de variable u = e−t entraı̂ne
 1

∀n ∈ N , Jn = u n−1 F (− log u) du = 0. (∗)
0
La fonction continue ]0, 1] → C u → F (− log u) est prolongeable par continuité sur [0, 1] car
F converge vers +∞ (vers l’intégrale de f ). Ainsi, la formule (*) entraı̂ne, d’après la question
précédente, que F est la fonction nulle. Comme F est une primitive de la fonction continue f ,
on en déduit que f est la fonction nulle.

Problème 25 (Th éorème de Fejér). Soit f : R → C une fonction continue et


2π-périodique. Pour tout k ∈ Z, on note ek : R → C x → eikx . Pour tout n ∈ N, on
définit les fonctions
n
S0 + S1 + · · · + Sn
Sn = ck(f ) e k, Cn =
k=−n
n+1

(où les ck (f ) sont les coefficients de Fourier de f ) et


n
  
Sn = e k, n = S0 + · · · + Sn .
C
k=−n
n+1
π
1
a) Vérifier que 2π C (t) dt = 1 pour tout n ∈ N, et montrer que pour tout α ∈ ]0, π[
−π n
la suite de fonction (Cn ) converge uniformément vers 0 sur [−π, π][−α, α].
b) En déduire le théorème de Fejér : la suite de fonctions (Cn ) converge uniformément
vers f sur R.
6. PROBLÈMES 307

1
π 1
π
Solution. a) On a 2π
e (t) dt
−π k
= 0 pour tout k ∈ Z ∗ et 2π
e (t) dt
−π 0
= 1. On en conclut
   n  π 
1 π
1 π
1  1
∀n ∈ N, Sn (t) dt = 1 et n (t) dt =
C Sk (t) dt = 1.
2π −π 2π −π n+1 2π −π
k=0

Pour montrer le résultat demandé sur la convergence uniforme de Cn , on calcule d’abord Sn .
On reconnaı̂t le noyau de Dirichlet, son calcul est classique et pour x ∈ R2π Z on a
(2n+1)ix
e −1 sin((n + 1/2)x)
∀n ∈ N, Sn(x) = e−inx = ,
e ix − 1 sin(x/2)
et comme
n
 e(n+1)ix − 1 sin((n + 1)x/2)
∀n ∈ N, e(k+1/2)ix = eix/2 ix
= e(n+1)ix/2 ,
e −1 sin(x/2)
k=0

on en déduit, en prenant la partie imaginaire, que


n
  2
n (x) = 1 1 sin((n + 1)x/2)
∀n ∈ N, C Sk (x) = .
n+1 n+1 sin(x/2)
k=0

On en conclut que si 0 < α < π,


 n (x)| ≤ 1
∀n ∈ N, ∀x ∈ [−π, π], |x| > α, |C .
(n + 1) sin 2(α/2)
Ceci suffit pour montrer que (Cn ) converge uniformément vers 0 sur [ −π, π ][−α, α].
b) On remarque que pour tout n ∈ N et pour tout x ∈ R,
n  π   π
1  1
Sn (x) = f (t)e −ikt
dt e ikx
= f (t)Sn(x − t) dt,
2π −π 2π −π
k=0
π
1
donc Cn(x) = 2π  n (x − t) dt. Ainsi Cn s’écrit comme un produit de convolution. Le
f (t)C
−π
changement de variable u = x − t, conjugué au caractère 2π-périodique des intégrandes, entraı̂ne
 π
∗ 1 n (u) du.
∀x ∈ R, ∀n ∈ N , C n(x) = f (x − u)C (∗)
2π −π
Ceci étant, prouvons la convergence uniforme de (Cn ) vers f . On procède comme dans la
question 1/b) du problème 23. Soit ε > 0. La fonction f est continue et 2π-périodique, elle est
donc uniformément continue sur R ce qui entraı̂ne
∃α ∈ ]0, π[ , ∀(x, y ) ∈ R 2 , |x − y| < α, |f (x) − f (y )| < ε.
En désignant par M un majorant de |f | sur R, la formule (*) montre que pour tout x ∈ R et
pour tout n ∈ N,
  
 1 π 

|f (x) − C n (x)| =  
(f (x − t) − f (x))Cn (t) dt
2π −π
  α 
1  1  2M n (t) dt + ε,
≤ 2M C n (t ) dt + ε C n (t ) dt ≤ C
2π α≤|t|≤π 2π −α 2π α≤|t|≤π

et comme (C n ) converge uniformément vers 0 sur [−π, π ][−α, α], il existe N ∈ N tel que


C (t) dt ≤ ε pour tout n ≥ N , de sorte que |f (x) − C n (x)| ≤ (M/π )ε + ε pour tout
α≤|t|≤π n
x ∈ R et pour tout n ≥ N . D’où le résultat
Remarque. Le théorème de Fejér s’exprime en disant que la série de Fourier de toute
fonction continue 2π -périodique converge uniformément en moyenne de Césaro vers f .
En particulier, toute fonction continue 2π -périodique est limite uniforme de polynômes
trigonométriques sur R.
308 4. SUITES ET SÉRIES

Problème 26. a) Soit α ∈ RQ. Montrer que pour toute fonction f : R → C continue
et 2π-périodique, on a
 n
1 2π
1
lim S n(f ) = f (t) dt où Sn(f ) = f (2παk). (∗)
n→+∞ 2π 0 n k=1

(Indication. Utiliser le fait que f est limite uniforme de polynômes trigonométriques sur
R — voir le problème précédent.)
b) En déduire que si α ∈ RQ, l’ensemble Γ = {nα − [nα], n ∈ N} ( où [ x] désigne la
partie entière de x pour tout x ∈ R) est dense dans [0, 1].

Solution. a) Prouvons d’abord le résultat lorsque f est une fonction de la forme ep : R →


C x → eipx (p ∈ Z). Si p ∈ Z∗ , on a (compte tenu du fait que e2iπαp =
 1 car α ∈ Q)
n
1 1 1 − e 2iπαpn 2
∀n ∈ N ∗ , S n(ep ) = e p(2παk) = e 2iπαp donc |S n(e p )| ≤ ,
n n 1 − e 2iπαp n |1 − e2iπαp |
k=1

1
 2π
ce qui montre que Sn (ep) tend vers 0 = 2π 0 ep (t) dt lorsque n → +∞. L’assertion (*) est donc
vraie pour f = ep lorsque p ∈ Z ∗. Elle est trivialement vraie pour e 0.
Finalement, (*) est vraie pour toute les fonctions ep . Par linéarité, on en déduit qu’elle est
vraie pour tout polynôme trigonométrique.
Considérons maintenant une fonction f : R → C, continue et 2π -périodique. Soit ε >
0. On sait qu’il existe un polynôme trigonométrique P (voir le problème précédent) tel que
|f (x) − P (x)| < ε pour tout x ∈ R. Par ailleurs, nous avons montré que (*) était vrai pour P ,
1
 2π
donc il existe N ∈ N tel que |S n (P ) − 2π 0 P (t) dt| < ε pour tout n ≥ N . On en déduit

  2π 
 1 
∀n ≥ N, Sn (f ) − f (t) dt
 2π 0
  2π   2π 
 1  1    
≤ |S n (f ) − Sn (P )| + Sn (P ) − P (t) dt  +  P (t) − f (t) dt ≤ ε + ε + ε = 3ε.
2π 0 2π  0

D’où le résultat.
b) Raisonnons par l’absurde. Si Γ n’était pas dense dans [0, 1], il existerait a, b ∈ R, 0 < a <
b < 1, tels que Γ ∩ [a, b] = ∅. Il est facile de construire une fonction f positive, continue et
2π-périodique sur R telle que f (x) > 0 sur ]2πa, 2πb [ et f (x) = 0 sur [0, 2πa] ∪ [2πb, 2π ]. Comme
pour tout n ∈ N on a nα − [nα] ∈ [a, b], on a f (2παk ) = 0 pour tout k ∈ N donc Sn (f ) = 0 pour
 2π
tout n ∈ N∗ . Ceci est en contradiction avec (*) car la fonction f vérifie 0 f (t) dt > 0.
Remarque. On retrouve facilement, avec la question b), que αN − N est dense dans R
si α ∈ RQ. C’est en substance le résultat démontré (par des moyens différents) dans la
question c) de l’exercice 5 page 205.

Problème 27 (Théorème taub érien fort). Soit (an) unesuite réelle telle que
an = O(1/n) lorsque n → +∞. On suppose que la série entière an z n a un rayon de
convergence  ≥ 1 et que sa somme F vérifie limx→1− F (x) = 0. Notre propos est de montrer
que la série an converge et que sa somme est nulle.
On note Φ l’ensemble des fonctions  ϕ : [0 , 1] → R telle que
1. pour toutx ∈ [0, 1[, la série an ϕ(xn ) converge ;
2. lim x→1 − +∞ n
n=0 a n ϕ(x ) = 0.
6. PROBLÈMES 309

1/ a) Vérifier que toute fonction polynôme p nulle en 0 est élément de Φ.


b) Soit q une fonction polynôme. Montrer l’existence et déterminer
+∞

lim−(1 − x) xn q (xn).
x→1
n=0

2/ a) On considère la fonction g : [0, 1] → R x → 0 si 0 ≤ x < 1/2, x → 1 sinon. Pour


tout ε > 0, montrer qu’il existe deux polynômes p1 et p2 vérifiant
(i) p1 (0) = p2 (0) = 0 et p 1(1) = p2 (1) = 1 ;
(ii) p1 ≤ g ≤ p 2 sur [0, 1] ;
1
p2 (x) − p 1(x)
(iii) q (x) dx < ε avec q(x) = .
0 x(1 − x)
b) Montrer que g ∈ Φ.
3/ En déduire le théorème taubériend’Hardy-Littlewood : Si (b n )
est une suite réelle
+∞ n
vérifiant bn = O(1/n) et si lim x→1− n=0 b n x = , alors la série bn converge et sa
somme vaut .
 
Solution. 1/ a) Si p(x) = xk avec k ∈ N ∗, alors an p(xn ) = an(xk )n converge pour tout
x ∈ [0, 1[, et de plus
+∞
 +∞

∀x ∈ [0, 1[ , an p(x n) = F (x k ) donc lim− an p(xn ) = 0.
x→1
n=0 n=0
Par linéarité, on en déduit que ce résultat est vrai pour toute fonction polynôme nulle en 0.
 n n
b) Le polynôme q est borné sur [0, 1], donc pour tout x ∈ [0, 1[, la série x q (x ) converge
absolument donc converge.
Si q est le monôme x → xk (k ∈ N), alors
+∞
 +∞

n n 1−x 1
∀x ∈ [0, 1[ , (1 − x) x q (x ) = (1 − x) (x k+1)n = = ,
n=0 n=0
1 − x k+1 1 + x + · · · + xk
donc
+∞
  1
n n 1
lim (1 − x) x q (x ) = = q (t) dt.
x→1− k+1 0
n=0
On en déduit par linéarité que le résultat reste vrai pour tout polynôme q.
2/ a) On considère la fonction h : [0, 1] → R définie par
g (x) − x
h(x) = si x ∈ ]0, 1[ , h(0) = −1, h(1) = 1.
x(1 − x)
Compte tenu de la valeur de g la fonction h vérifie
   
1 1 1 1
∀x ∈ 0, , h(x) = − , ∀x ∈ , 1 , h(x) = .
2 1−x 2 x
1
Soit ε > 0. On peut trouver deux fonctions continues s1 et s 2 vérifiant s1 ≤ h ≤ s 2 , 0 s2 (t) −
s1 (t) dt < ε (on s’en convainc en faisant un dessin : prendre deux fonctions égales à h sur [0, 1]
sauf sur un petit voisinage de la discontinuité en x = 1/2 de h, et joindre les extrémités dans
la partie manquante par une ligne continue qui reste toujours du même coté du graphe de h et
qui reste bornée). Comme s1 et s 2 sont continues, on peut trouver deux polynômes t 1 et t2 tels
que |t1 − s1 | < ε et |t2 − s 2| < ε sur [0, 1] (conséquence du théorème de Weierstrass). Ainsi, les
polynômes u1 = t 1 − ε et u2 = t 2 + ε vérifient u1 < s1 ≤ h ≤ s2 < u2 et u2 − u1 = t 2 − t1 + 2ε ≤
s2 − s 1 + 4ε donc
 1  1
(u 2(x) − u1 (x)) dx ≤ (s 2(x) − s 1 (x) + 4ε) dx < 5ε.
0 0
310 4. SUITES ET SÉRIES

Comme g (x) = x + x(1 − x)h(x) sur [0, 1], on en conclut que les polynômes p1 et p2 définis par
p1 (x) = x+x(1−x)u 1(x), p2 (x) = x+x(1−x)u 2(x) vérifient p 1(0) = p 2 (0) = 0, p1(1) = p 2 (1) = 1,
p1 ≤ g ≤ p 2 et
 1
p2 (x) − p1 (x)
le polynôme q (x) = = u 2 (x) − u 1(x) vérifie q (x) dx < 5ε.
x(1 − x) 0
D’où le résultat.
b) Soit ε > 0 et des polynômes p1 , p2 , q vérifiant les conditions (i), (ii) et (iii) de la question
précédente. 
1. La convergence de a n g (xn) pour tout x ∈ [0, 1[ est immédiate car le terme d’indice n
de cette série est nul dès que x n < 1/2.
2. Soit M > 0 tel que |an | ≤ M/n pour tout n ∈ N∗ . Comme p 1 ≤ g ≤ p 2 , on a par ailleurs
 
 +∞ +∞  +∞
 n

n 

∀x ∈ [0, 1[ ,  an g (x ) − a n p1 (x ) ≤ |an|(p 2 − p1 )(x n )
 
n=0 n=0 n=1
+∞
 +∞

xn(1 − x n) n
≤M q (x ) ≤ M (1 − x) x nq (xn) (∗)
n=1
n n=1

(on a utilisé la majoration (1 − x n ) = (1 − x)(1 + x + · · · + xn−1 ) ≤ (1 − x)n). Or p1 ∈ Φ et


 1
limx→1−(1 − x) +∞ n n
n=1 x q (x ) = 0 q (t) dt < ε, on en conclut
 
 +∞
  +∞

 
∃λ ∈ [0, 1[ , ∀x ∈ [λ, 1[ ,  a n p1(x n ) < ε et (1 − x) xnq (xn ) < 2ε.
 
n=0 n=1

L’inégalité (*) entraı̂ne donc


     
 +∞   +∞   +∞ 
 n   n  

n 
∀x ∈ [λ, 1[ ,  an g (x ) ≤  an p 1 (x ) +  a n (g − p1 )(x )  < ε + 2M ε = (2M + 1)ε.
     
n=0 n=0 n=0

Ceci est possible pour tout ε > 0, donc g vérifie bien la condition 2 des éléments de Φ.
3/ La forme de g montre que
+∞ [− log 2/ log x]
 
n
∀x ∈ [0, 1[ , ang (x ) = an,
n=0 n=0

et comme g ∈ Φ, on en conclut en faisant tendre x vers 1− que la série an converge et que sa
somme est nulle.
Le théorème de Hardy-Littlewood s’en déduit facilement en considérant (an) définie par
a0 = b 0 −  et an = b n pour tout n ∈ N∗ .
Remarque. Ce résultat est la version forte du théorème taubérien vu dans l’exercice 11
page 264. La preuve est intéressante car elle fait appel au théorème de Weierstrass là on
ne s’y attendait pas a priori.
Si les bn sont positifs, ce résultat est beaucoup plus facile à prouver.
On peut montrer que le théorème Taubérien d’Hardy-Littlewood reste vrai si la suite
(nbn) est seulement minorée (et non pas bornée comme dans le cadre de l’exercice).

Problème 28 (Th éorème de M üntz). On note C l’e.v des fonctions continues de


[0, 1] dans R. On utilisera, sur C , les deux normes suivantes :
 1 1/2
2
∀f ∈ C, f  2 = f (t) dt et f  ∞ = sup |f (t)|.
0 t∈[0,1]
6. PROBLÈMES 311

Le but de ce problème est de donner une condition nécessaire et suffisante sur une
suite strictement croissante (αn ) à valeurs positives, pour que Vect n∈N(x αn ) soit dense
dans C (par abus, xm désigne la fonction de C définie par x → x m pour m ≥ 0). On
pourra utiliser le théorème de Weierstrass (voir le théorème 5 page 235).
1/ Soit (αn )n∈N ∗ une suite à termes strictement positifs et strictement croissante.
a) Soient m ∈ R+∗ et N ∈ N∗. On note E N = Vect1≤i≤N (xα i). Exprimer en fonction des
αi et de m, la valeur ∆ N (m) = inf f ∈E N xm − f 2 . (Indication. On pourra utiliser les
déterminants de Gram — voir le tome Algèbre).
b) En déduire une condition nécessaire et suffisante sur la suite (αn ) pour que Vectn∈N (xα n)
soit dense dans C pour la norme  . 2 (théorème de Müntz).
2/ Soit (αn) n∈N une suite à termes positifs, strictement croissante, avec α0 = 0 et
vérifiant limn→+∞ αn > 1. Donner une condition nécessaire et suffisante sur (αn ) pour
que Vectn∈N (x αn ) soit dense dans C pour la norme  . ∞.
1
Solution. 1/ a) Munissons C du produit scalaire f, g  = 0 f (t)g (t) dt, dont  .  2 est la norme
euclidienne associée. Le nombre réel ∆N (m) s’interprète comme la distance (au sens de  .  2)
de xm à EN = Vect 1≤i≤N (xαi ). On sait (voir le tome Algèbre) que ceci peut s’exprimer au
moyen des déterminants de Gram. Plus précisément, en notant, pour tout (x1 , . . . , xn ) ∈ Cn ,
G(x1 , . . . , xn ) le déterminant de la matrice (x i, x j ) 1≤i,j ≤n, on a
G(x α1 , . . . , xαN , xm )
∆N (m)2 = . (∗)
G(x α1 , . . . , xαN )
Ici on a xa, x b = a+1b+1 . On trouve dans le tome Algèbre que la valeur d’un déterminant de
 1 
Cauchy det a i +b j i,j
est donnée par
   
  (a − a ) · (b − b )
1 1≤i<j ≤n i j 1≤i<j ≤n i j
det =  .
ai + b j 1≤i,j ≤n 1≤i,j ≤n (a i + b j )

En particulier, pour toute famille (pi )1≤i≤n de nombres réels, G(xp1 , . . . , xpn ) est un déterminant
de Gram car xp i, xp j  = ai +b
1
j
avec a i = pi et bj = p j + 1, et on a donc
 2
p1 pn 1≤i<j ≤n (pi − p j )
G(x , . . . , x ) =  .
1≤i,j ≤n (pi + p j + 1)

En appliquant cette formule avec p 1 = α1 , . . ., pN = αN , pN+1 = m on obtient


   
2
i<j (α i − α j ) · i (αi − m)2
G(xα1 , . . . , xαN , xm) =   
(2m + 1) i,j (α i + α j + 1) · [ i(α i + m + 1)2 ]

(les indices i et j sont pris entre 1 et N ). De même, on trouve


 2
i<j (αi − αj )
G(x α1, . . . , x αN ) =  .
i,j (αi + α j + 1)

On en conclut, avec (*), que


N  
1  αi − m 
∆N (m) = √  . (∗∗)
2m + 1 i=1 α i + m + 1 

b) On note E = Vect(xα i )i∈N∗ . Nous allons montrer qu’une condition nécessaire et suffisante

pour que E = C (où l’adhérence est prise au sens de la norme  . 2) est que la série 1/αn
diverge.
312 4. SUITES ET SÉRIES

Condition nécessaire. Il existe bien sûr m > 0 tel que αn = m pour tout n. La fonction xm
appartient à E = C , donc la suite (∆N (m)) N∈N ∗ tend vers 0, ce qui entraı̂ne, d’après (**)

N  
αi − m
lim = 0. (∗∗∗)
N→+∞
i=1
αi + m + 1

— Si la suite (αn ) est majorée, la série 1/αn diverge.
— Sinon, (αn ) tend vers +∞. Soit N 0 ∈ N tel que αn > m pour tout n ≥ N0 . Alors, dès
que n ≥ N0,
   
αn − m 2m + 1 2m + 1
un = log = log 1 − ∼− (n → +∞), (∗∗∗∗)
αn + m + 1 αn + m + 1 αn
 
et comme d’après (***), n≥N0 un diverge, on en conclut que 1/αn diverge.

Condition suffisante. Réciproquement, si 1/α n diverge, montrons E = C . Pour cela, en vertu
du théorème de Weierstrass, il suffit de montrer que pour tout m ∈ N, x m ∈ E.
Soit m ∈ N. Il s’agit de montrer que (***) est vérifié.
— Si (αn ) est majorée, c’est évident. 
— Sinon, (α n) tend vers +∞, et l’équivalent (****) montre que u n diverge vers −∞, et
on conclut que (***) est vérifié.
2/ Si E = C au sens de la  norme  .  ∞ , on vérifie facilement que E = C au sens de la norme
 . 2 , donc d’après 1/b), n∈N∗ 1/αn  diverge.
Réciproquement, supposons que n∈N∗ 1/αn diverge. Comme lim n→+∞ αn > 1, il existe
N ∈ N tel que αN > 1. Quitte à retirer des termes à (αn ), on peut donc supposer α1 > 1.
En vertu du théorème de Weierstrass, pour montrer E = C, il suffit de montrer que pour toute
fonction
 polynôme P sur [0, 1], P ∈ E . Soit P une fonction polynôme. Soit ε > 0. La série
n∈N ∗ (α n − 1)
−1
diverge, donc d’après 1/b), il existe g ∈ Vect i∈N∗ (xα i −1) tel que P  − g  2 < ε.
Soit h la primitive de g sur [0, 1] vérifiant h(0) = P (0). On voit  xfacilement que h ∈ E . En posant
Q = h − P , on a Q(0) = 0, donc pour tout x ∈ [0, 1], Q(x) = 0 Q (t) dt, donc d’après l’inégalité
de Schwarz,
 x 1/2
 √
∀x ∈ [0, 1], |Q(x)| ≤ 2
Q (t) dt x ≤ Q 2 = P  − g  2 < ε,
0

donc h − P ∞ = Q ∞ < ε. Finalement, pour tout ε > 0, nous avons trouvé h ∈ E tel que
h − P  ∞ < ε. On en conclut P ∈ E , et finalement, E = C . 
Finalement, on a E = C pour la norme  . ∞ si et seulement si la série n∈N ∗ 1/α n diverge.

Probl
 ème 29 (Théorème de Cantor sur les séries trigonométriques). 1/ Soit
a n une série à termes complexes, convergente et de somme nulle. On pose
 
∗ ∗ ∗ sin nt 2
U0 : R → C t → 1 et ∀n ∈ N , Un : R → C t → .
nt

Pour tout t = 0, montrer que S (t) = +∞ n=0 a n U n(t) existe, puis montrer lim t→0 S (t) = 0.t=0

2/ (Théorème de Cantor-Lebesgue.) Soit (cn) n∈Z une famille de nombres complexes. On


suppose que pour tout t ∈ R, lim n→+∞ c n eint + c−n e −int = 0. Montrer que lim|n|→+∞ c n =
0. (Indication : on se ramènera à montrer que si pour tout t ∈ R, ρ n cos(nt − θn) → 0,
alors ρn → 0. Ensuite on construira, en supposant ρ n → 0, une suite décroissante (I k) de
segments non vides de R vérifiant cos(n kt − θn k ) ≥ 1/2 pour tout t ∈ Ik et pour tout k,
avec (nk ) bien choisie.)
6. PROBLÈMES 313

3/ (Théorème de Cantor.) Soit (cn )n∈Z une famille de nombres complexes telle que
N

∀x ∈ R, lim cn einx = 0.
N→+∞
n=−N

a) Montrer que la fonction


+∞
x2  cn einx + c−n e−inx
F : x → c0 +
2 n=1
(in)2
existe et est continue sur R.
b) Pour tout fonction continue G : R → R, on définit
G(x + h) + G(x − h) − 2G(x)
∀x ∈ R, DG(x) = lim
h→0
h=0
h2
lorsque cette limite existe (on rappelle — voir la question c) de l’exercice 5 page 102 —
que si pour tout x, DG(x) existe et est nul, alors G est affine.) Montrer que pour tout
x ∈ R, DF (x) existe et est nul.
c) En déduire cn = 0 pour tout n ∈ Z.

Solution. 1/ La série a n converge, donc la suite (a n ) tend vers 0. Par ailleurs, |a n Un(t)| ≤
(|an |/t2 ) 1/n2, donc S (t) existe pourtout t = 0.
Pour tout n ∈ N∗, notons s n = nk=0 ak . Une transformation d’Abel donne
N
 N−1

∀t =
 0, an Un(t) = s n (Un (t) − Un+1 (t)) + sN U N (t),
n=0 n=0
et comme (sN UN (t)) tend vers 0, on a finalement
+∞

∀t =
 0, S(t) = sn (Un(t) − Un+1(t)).
n=0
Soit ε > 0 et N ∈ N∗ tel que |sn | ≤ ε pour n ≥ N , et soit A un majorant de la suite (|s n|). La
formule précédente entraı̂ne la majoration
N−1
 +∞

∀t =
 0, |S (t)| ≤ A |U n(t) − Un+1 (t)| + ε |Un(t) − U n+1(t)|. (∗)
n=0 n=N
On remarque que
    
 (n+1)t  (n+1)t
d sin x 2
 
|Un (t) − Un+1 (t)| =  f (x) dx  ≤ |f (x)| dx, f (x) = .
 nt  nt dx x
 k x2k
La fonction x → (sin x)/x est égale à +∞ k=0 (−1) (2k+1)! , donc prolongeable en 0 en une fonction
1
de classe C ∞, donc f est prolongeable en 0 en une fonction de classe C ∞ . L’intégrale 0 |f (x)| dx
2
existe
 +∞ donc. Par ailleurs, un calcul rapide donne |f (x)| = O (1/x ) en +∞, donc l’intégrale
0 |f (x)| dx existe bien. Si on note M sa valeur, (*) entraı̂ne
N−1

∀t =
 0, |S (t)| ≤ A |Un (t) − Un+1 (t)| + εM.
n=0
N−1
Comme pour tout n, lim t→0 Un (t) = 1, la fonction t → A n=0 |Un (t) − Un+1 (t)| tend vers 0
t=0
lorsque t → 0, t = 0, ce qui entraı̂ne l’existence de α > 0 tel que |S (t)| < ε + ε = 2ε pour tout t
tel que 0 < |t| < α, d’où le résultat.
2/ On peut écrire cne int + c−n e−int sous la forme (an cos nt + b n sin nt) + i(an cos nt + b n sin nt),
où an, b n , an , b n ∈ R. Nous allons montrer que les suites (a n) et (bn ) tendent vers 0 (on aura
314 4. SUITES ET SÉRIES

de même an → 0 et b n → 0, ce qui montrera que les suites (cn) et (c−n ) tendent  vers 0).
Pour tout t ∈ R, on sait que lim n→+∞(an cos nt + bn sin nt) = 0. En posant ρn = a2n + b 2n ,
il existe pour tout n un nombre réel θn ∈ [0, 2π ] tel que an cos nt + b n sin nt = ρ n cos(nt − θ n )
pour tout t ∈ R. Ainsi, nous nous sommes ramené au problème suivant : si pour tout t ∈ R,
limn→+∞ ρn cos(nt − θ n ) = 0, il faut montrer que la suite (ρn) tend vers 0.
Supposons le contraire. Alors il existe δ > 0 et une suite strictement croissante (nk ) k∈N
d’entiers telle que ρ nk ≥ δ pour tout k . Quitte à restreindre la suite (nk ), on peut même
supposer nk+1 ≥ 6n k pour tout k.
Ensuite, on pose
  
1 π 1  π
I1 = θn1 − , θn 1 + ,
n1 3 n1 3
de sorte que pour tout t ∈ I1 , cos(n1t − θn1 ) ≥ 1/2. Lorsque t parcourt I1, n 2t − θn2 parcourt un
intervalle de longueur n2 · 2π/(3n1 ) ≥ 4π . On peut donc trouver un segment I2 ⊂ I1 de longueur
2π/(3n2 ) tel que cos(n 2t − θn 2) ≥ 1/2 pour tout t ∈ I2 . En itérant le procédé, on construit ainsi
pour tout k un segment Ik ⊂ Ik−1 de longueur 2π/(3nk ) tel que
1
∀t ∈ Ik, cos(n k t − θnk ) ≥ .
2
D’après la proposition 9 page 20, on sait qu’il existe ξ ∈ R tel que ∩k∈N I k = {ξ }. Pour tout k,
on a ρn k cos(n k ξ − θk ) ≥ δ/ 2, ce qui est absurde car limn→+∞ ρ n cos(nξ − θ n ) = 0. La suite (ρn )
converge donc vers 0.

3/ a) Pour tout x, on a limN→+∞ N −N cn e
inx
= 0, donc pour tout x, limn→+∞(cn einx +
c −ne−inx ) = 0. D’après la question précédente, les suites (cn) et (c −n ) convergent donc vers 0.
Elles sont donc bornées, et si M désigne un majorant du module de leur terme général, on a
pour tout n ∈ Z ∗ la majoration |c n /(in)2 | ≤ M/n 2 , ce qui assure l’existence et la continuité de
F.
b) Un calcul donne immédiatement, pour tout x ∈ R et pour tout h = 0
+∞
  
F (x + h) + F (x − h) − 2F (x) inx −inx
 sin(nh/2) 2
= c0 + cn e + c−ne ,
h2 n=1
(nh/2)

donc d’après la question 1/, DF (x) existe et DF (x) = 0 pour tout x ∈ R.


c) On en déduit l’existence de α, β ∈ C tels que

x2  cn
∀x ∈ R, F (x) = αx + β = c 0 + 2
e inx .
2 n∈Z∗
(in )

Ceci montre que la fonction x → αx + β − c20 x2 est 2π-périodique. Donc α = c 0 = 0 et


 cn
∀x ∈ R, β= 2
e inx.

(in )
n∈Z

Nous avons vu plus haut que la famille (cn)n∈Z est bornée, la série trigonométrique de l’égalité
précédente converge donc normalement sur R. Elle est donc égale à sa série de Fourier, et comme
c’est une constante, on en déduit cn /(in)2 = 0 pour tout n ∈ Z∗. Finalement, on a cn = 0 pour
tout n ∈ Z.
Remarque. Ce résultat a été démontré pour la première fois par Cantor en 1871. Cantor
a ensuite affaibli les hypothèses, en recherchant des ensembles dit exceptionnels, tels que
si une série trigonométrique converge vers 0 sauf peut être sur un tel ensemble, alors ses
coefficients sont nuls. Ces considérations l’amenèrent peu à peu à construire la théorie des
ensembles, en particulier la notion de puissance d’un ensemble (théorie de l’équipotence)
que l’on connaı̂t.
7. SUJETS D’ÉTUDE 315

7. Sujets d’étude
Sujet d’étude 1 (Intégrales eul ériennes : fonction gamma, fonction bêta).
Le but de ce sujet d’étude est d’étudier et de donner quelques propriétés des fonctions
gamma et bêta définies respectivement par
 +∞ 1
−t x−1
Γ(x) = e t dt (x > 0), B(x, y ) = tx−1 (1 − t)y−1 dt (x, y > 0).
0 0
1/ (Fonction gamma.) a) Montrer que Γ est de classe C∞ et convexe sur R +∗.
b) Montrer que Γ est logarithmiquement convexe (i. e. que log Γ est convexe).
c) Montrer
∀x > 0, Γ(x + 1) = xΓ(x) et ∀n ∈ N, Γ(n + 1) = n!.

d) Donner un équivalent de Γ en 0+ et tracer l’allure de son graphe.


2/ (Fonction bêta.) a) Montrer que B vérifie les équations fonctionnelles
x
∀x, y > 0, B(x, y ) = B(y, x) et B(x + 1, y) = B(x, y ).
x+y
n n
b) Pour n ∈ N∗, on pose I n(x) = 0 1 − nt tx−1 dt. Montrer que pour tout x > 0, on a
lim n→+∞ In (x) = Γ(x). En exprimant I n(x) en fonction de la fonction B, en déduire
nx n!
Γ(x) = lim .
n→+∞ x(x + 1) · · · (x + n)

c) Montrer que pour x, y > 0 fixés, B(x + n + 1, y) ∼ Γ(y )/ny lorsque n → +∞. En
déduire la formule
Γ(x)Γ(y)
∀x, y > 0, B(x, y ) = .
Γ(x + y )
d) Calculer Γ(1/2).
3/ a) Démontrer la formule de Weierstrass :
+∞ 
1 γx
 x  −x/n 
∀x > 0, = xe 1+ e ,
Γ(x) n=1
n
 N
où γ est la constante d’Euler (le produit infini +∞ n=1 signifie lim N→+∞ n=1 ).
b) Montrer la formule de duplication
 
2x−1 1 √
∀x > 0, 2 Γ(x)Γ x + = π Γ(2x).
2

c) En utilisant le développement en produit infini de la fonction sinus (voir la question


b) de l’exercice 2 page 273) montrer la formule des compléments
1 sin πx
∀x ∈ ]0, 1[ , = .
Γ(x)Γ(1 − x) π

d) Montrer la relation
+∞
Γ  (x) 1  x
∀x > 0, = −γ − + .
Γ(x) x n=1 n(x + n)
 +∞
En déduire 0
(log t)e−t dt = −γ .
316 4. SUITES ET SÉRIES

Solution. 1/ a) Remarquons tout d’abord que l’intégrale qui définit Γ est bien convergente
lorsque x > 0. Nous allons démontrer que la fonction Γ est de classe C ∞ et que
 +∞
(p)
∀p ∈ N, ∀x > 0, Γ (x) = (log t)p e−t tx−1 dt. (∗)
0

On remarque que lorsque x est dans un segment [a, b] ⊂ ]0, +∞[, l’intégrande de (*) vérifie

p −t x−1 | log t| p ta−1 si t ∈ ]0, 1]
|(log t) e t | ≤ ϕp (t), ϕp (t) =
(log t) p e −tt b−1 si t > 1

et que la fonction ϕ p est intégrable sur ]0, +∞[ puisque lorsque t → 0+ , on a ϕp (t) = o(ta/2−1 )
(car | log t| pt a/2 = o(1)) et ϕp (t) = O (e−t/2 ) lorsque t → ∞.
Soit [a, b] un segment inclus dans ]0, +∞[. Nous allons prouver par récurrence sur p ∈ N que
Γ est de classe C p sur [a, b] et que Γ(p) vérifie la relation (*) sur cet intervalle. Comme ceci sera
vrai pour tout segment inclus dans ]0, +∞[, on aura prouvé le résultat sur ]0, +∞[ tout entier.
Pour p = 0, il s’agit de montrer que Γ est continue sur [a, b]. La majoration |tx−1 e−t | ≤ ϕ0 (t)
avec ϕ0 intégrable sur ]0, +∞[ assure que l’hypothèse de domination du théorème de continuité
sous le signe intégral est bien vérifiée. Comme par ailleurs l’intégrande de Γ est continue, ceci
assure la continuité de Γ sur [a, b].
Supposons maintenant l’hypothèse de récurrence vraie au rang p et montrons là au rang
p + 1. L’intégrande de (*) est bien continûment dérivable par rapport à x et sa dérivée est égale
à fx (t) = (log t)p+1 tx−1 e−t. Grâce à la majoration |fx (t)| ≤ ϕp+1 (t) lorsque x ∈ [a, b], avec ϕp+1
intégrable, on peut appliquer le théorème de dérivation sous le signe  intégral qui nous assure
(p) 1
que Γ est de classe C sur [a, b] et que sa dérivée est égale à R+∗ f x . Ainsi, l’hypothèse de
récurrence est vraie au rang p + 1. Ceci termine la preuve que Γ est bien de classe C∞ .
Pour montrer la convexité de Γ, il suffit de montrer que Γ est positive, ce qui est immédiat
car la relation (*) montre que Γ  a son intégrande positive.
b) Posons g : x → log Γ(x). On a g  = (ΓΓ  − Γ 2 )/Γ2 , il s’agit donc de montrer que Γ 2 ≤ ΓΓ .
Pour tout x > 0, l’inégalité de Schwarz donne
 +∞    2  +∞  +∞
(x−1)/2 −t/2 (x−1)/2 −t/2 x−1 −t
t e t log t e dt ≤ t e dt · (log t)2 t x−1e−t dt,
0 0 0

ce qui est précisément l’inégalité Γ (x)2 ≤ Γ(x)Γ  (x).


c) Une intégration par parties donne
 +∞   +∞  +∞
x −t x −t
∀x > 0, Γ(x + 1) = t e dt = − t e + xtx−1 e−t dt = xΓ(x).
0 0 0

La seconde
 +∞formule s’en déduit facilement par récurrence sur n ∈ N, compte tenu du fait que
−t
Γ(1) = 0 e dt = 1.
d) Lorsque x → 0+ , xΓ(x) = Γ(x + 1) ∼ Γ(1) = 1, autrement dit Γ(x) ∼ 1/x lorsque x → 0+ .
Le comportement de Γ au voisinage de 0+ nous donne un premier renseignement sur l’allure
de son graphe. Notons que Γ(1) = Γ(2) = 1, ce qui entraı̂ne l’existence d’un point c de ]1, 2[
où Γ s’annule d’après le théorème de Rolle. Comme Γ est convexe, la fonction Γ croı̂t sur
[c, +∞[, et comme Γ(n + 1) = n! → +∞, on en déduit que Γ tend vers +∞ en +∞. De plus,
Γ(x + 1)/x = Γ(x), donc Γ(x)/x tend vers +∞ lorsque x → +∞, donc Γ admet une branche
parabolique dans la direction verticale en +∞. On en déduit l’allure du graphe de Γ (voir la
figuresous-suite
ci dessous).
2/ a) L’égalité B(x, y) = B(y, x) s’obtient en effectuant le changement de variable u = 1 − t
dans l’intégrale définissant B(x, y ).
7. SUJETS D’ÉTUDE 317

(Γ)

0 1 2 3

Figure 2. Le graphe de la fonction Γ.

Pour la seconde identité, on intègre par parties, en écrivant


 1 x    x 1
t x+y−1 (1 − t)x+y t
B(x + 1, y) = (1 − t) dt = −
0 1−t x+y 1−t 0
 1  x−1
x t dt x
+ (1 − t)x+y 2
= B(x, y ),
x+y 0 1−t (1 − t) x+y
ce qui prouve le résultat.
b) Fixons x > 0. La suite de fonctions (gn )n∈N∗ définie sur R+∗ par
 
t n x−1
gn (t) = 1 − t si 0 < t ≤ n, gn (t) = 0 si t > n
n
converge simplement vers t → e−t t x−1. De plus on a la majoration |gn(t)| ≤ e −tt x−1 (car
(1−t/n)n ≤ e−t sur [0, n], conséquence de l’inégalité log(1−u) ≤ −u qui s’obtient par l’égalité des
accroissements finis), et comme t → e−t tx−1 est intégrable
 sur R+ , le théorème de convergence
dominée nous assure de la convergence de In (x) = R+ gn vers Γ(x).
Le changement de variable u = t/n dans l’intégrale In (x) donne In (x) = n x B(x, n + 1).
Compte tenu de la seconde identité de la question précédente, on a
n! n!
∀n ∈ N∗ , B(x, n + 1) = B(n + 1, x) = B(1, x) =
(x + 1) · · · (x + n) x(x + 1) · · · (x + n)

car B(1, x) = 01(1 − t)x−1 dt = 1/x. On en déduit le résultat car on a montré que In(x) =
nx B(x, n + 1) converge vers Γ(x) lorsque n → +∞.
1
c) On a B(x + n + 1, y) = B(y, x + n + 1) = 0 ty−1 (1 − t)x+n dt, et en effectuant le changement
de variable t = u/n dans cette dernière intégrale, on s’aperçoit que
 n 
∗ 1 u x+n
∀n ∈ N , B(x + n + 1, y) = y uy−1 1 − du.
n 0 n
Fixons x, y > 0. Pour u fixé, (1 − u/n)x converge vers 1 lorsque n → +∞. Ainsi, la suite
de fonctions (hn )n∈N ∗ définie par h n(u) = u y−1(1 − u/n) x+n si 0 < u ≤ n, hn(u) = 0 si
u > n, converge simplement vers u → u y−1e−u . La majoration par une fonction intégrable
|h n(u)| ≤ uy−1 e−u permet d’appliquer le théorème de convergence dominée, ce qui assure que
la dernière intégrale converge vers Γ(y) lorsque n → +∞. On a donc B(x + n + 1, y) ∼ Γ(y )/ny
lorsque n → +∞.
Maintenant, compte tenu de la seconde identité prouvée à la question 2/a), on a
(x + y)(x + y + 1) · · · (x + y + n)
∀n ∈ N∗ , B(x, y) = B(x + n + 1, y),
x(x + 1) · · · (x + n)
318 4. SUITES ET SÉRIES

et compte tenu de l’équivalent z (z + 1) · · · (z + n) ∼ n z n!/Γ(z ) valable pour tout z > 0 fixé


(conséquence de la question précédente), on en déduit, lorsque n → +∞
nx+y n!/Γ(x + y ) Γ(y ) Γ(x)Γ(y )
B(x, y ) ∼ x y
= ,
n n!/Γ(x) n Γ(x + y )
d’où le résultat.
d) La formule précédente donne B(1/2, 1/2) = Γ(1/2)2 /Γ(1) = Γ(1/2)2 . Le changement de
variable t = sin2 u dans l’intégrale définissant B(1/2, 1/2) montre que B(1/2, 1/2) = π . Comme

Γ est positive sur R+∗ (c’est l’intégrale d’une fonction positive), on en déduit Γ(1/2) = π.
3/ a) Compte tenu de la relation log n = 1 + 12 + · · · + n1 − γ + o(1) (voir page 211), on a
n 
x
n 
x
 n
 
x(x + 1) · · · (x + n) −x log n γx o(1) −x/k
= xe 1+ = xe e e 1+ ,
n x n! k k
k=1 k=1 k=1

et ceci converge vers 1/Γ(x) d’après 2/b), d’où le résultat (remarquez que du même coup on a
la convergence du produit infini).
b) Fixons x > 0. D’après la question 2/b), on a, lorsque n → +∞,
 
1 n 2x+1/2(n!) 2 2 2n+2n2x+1/2 (n!)2
Γ(x)Γ x + ∼ =
2 x(x + 1/2) · · · (x + n)(x + n + 1/2) (2x)(2x + 1) · · · (2x + 2n + 1)
Γ(2x) 2n+2 2x+1/2 2 Γ(2x)22n+2 n1/2(n!) 2 Γ(2x)2 2n+1−2x (n!)2
∼ 2 n (n!) ∼ = .
(2n + 1)2x(2n + 1)! 2 2x (2n)!(2n) n1/2 (2n)!
Or d’après la formule de Stirling,
(n!) 2 n2n e −2n(2πn) (πn)1/2
∼ =
(2n)! (2n)2n e−2n(4πn) 1/2 2 2n
(on peut aussi montrer ce résultat avec la formule de Wallis), et on en déduit finalement
1−2x √
Γ(x)Γ(x + 1/2) ∼ Γ(2x)2 π, d’où le résultat.
c) Toujours en utilisant 2/b) on a, pour x ∈ ]0, 1[ fixé et lorsque n → +∞,
1 x(1 + x)(1 + x/2) · · · (1 + x/n) (1 − x)(1 − x/2) · · · (1 − x/n)(1 − x + n)
∼ ·
Γ(x)Γ(1 − x) nx n1−x
     n  
x(1 − x + n) 2 x2 xn x2
= (1 − x ) 1 − 2 · · · 1 − 2 ∼ x 1− 2
n 2 n k
k=1

On conclut facilement avec le développement de la fonction sinus en produit infini.


d) La formule de Weierstrass s’écrit aussi

+∞
x  x 
∀x > 0, log Γ(x) = − log x − γx + − log 1 + . (∗∗)
n=1
n n

En posant fn(x) = x/n − log(1 + x/n) pour tout n ∈ N∗ et pour tout x ≥ 0, on a


1 1 1 1 1 x
∀x ≥ 0, ∀n ∈ N∗, fn (x) = − = − = .
n n 1 + x/n n n+x n(n + x)
 
Ainsi, pour tout A > 0, on a |f n(x)| ≤ nA2 pour tout x ∈ [0, A], donc la série de fonctions fn (x)
 +∗
converge normalement sur [0, A]. Comme fn converge simplement sur R (conséquence de
l’existence de la formule (**)), on en déduit quela somme de cette série est dérivable sur [0, A],
et que sa dérivée est donnée par la somme de f n . Ainsi sur ]0, A], la dérivée de log Γ vérifie,
d’après la formule (**),
+∞

Γ  (x) 1 x
= − −γ + .
Γ(x) x n(n + x)
n=1
7. SUJETS D’ÉTUDE 319

Ceci est vrai sur ]0, A] pour tout A > 0, donc sur R+∗ . C’est en particulier vrai pour x = 1, ce
qui fournit, compte tenu du fait que Γ(1) = 1,
+∞
 1  1
+∞
1


Γ (1) = −1 − γ + = −γ − 1 + − = −γ − 1 + 1 = −γ.
n=1
n(n + 1) n=1
n n+1

Comme Γ(1) = 0+∞ (log t)e−t dt (voir 1/a)), on en déduit le résultat.

Sujet d’étude 2 (Nombres et polyn ômes de Bernoulli). On définit la fonction


+∞

zn ∗ ez − 1
f : C→C z→
 qui vérifie ∀z ∈ C , f (z) = .
n=0
(n + 1)! z
Comme f (0) = 1 = 0, on sait (voir l’exercice 9 page 262) que 1/f est développable en
série entière dans un disque {z ∈ C, |z | < r}. Il existe donc une suite (bn) telle que
 bn +∞
∗ z
∀z ∈ C , |z | < r, z
= z n.
e −1 n=0
n!
Les nombres bn s’appellent les nombres de Bernoulli. On constate, par un produit de
Cauchy, qu’il existe également une suite de polynômes (Bn ) telle que
+∞
ze zx  B n(x)

∀x ∈ C, ∀z ∈ C , |z | < r, z
= zn .
e − 1 n=0 n!
Les polynômes B n s’appellent les polynômes de Bernoulli.
1/ a) En utilisant les propriétés de la fonction (z, x) → zezx /(ez − 1), montrer

(i) ∀n ∈ N, B n (1 − x) = (−1) nB n (x) (ii) ∀n ∈ N∗, Bn = nB n−1


(iii) ∀n ∈ N∗ , Bn (x + 1) − Bn (x) = nxn−1 (iv) ∀n ≥ 2, Bn (0) = B n (1)
1
(v) ∀n ∈ N∗ , 0
Bn (x) dx = 0 (vi) ∀n ∈ N ∗, b 2n+1 = 0.

b) Exprimer les polynômes de Bernoulli en fonction des nombres de Bernoulli. Montrer


que bn ∈ Q pour tout n ∈ N, et calculer b0 , b 1, b 2, b 4 , b6.
2/ Pour tout n ∈ N ∗, on note B̃n l’application de R dans R, 1-périodique, qui coı̈ncide
avec B n sur ]0, 1[ et telle que B̃n (0) = 12 [Bn (0) + Bn (1)].
a) En procédant par récurrence, montrer que pour tout k ∈ N∗ et pour tout x ∈ R
+∞
 cos(2πnx) +∞

B̃2k (x) k+1 B̃2k−1 (x) k sin(2πnx)
= 2 · (−1) , = 2 · (−1) .
(2k )! n=1
(2nπ) 2k (2k − 1)! n=1
(2nπ) 2k−1
+∞
b) En déduire, pour tout k ∈ N∗ , la valeur de ζ (2k) = n=1 n
−2k
en fonction de b2k.
c) Donner un équivalent de b2k lorsque k → +∞.

Solution. 1/ a) (i). Soit x ∈ C. Pour tout z ∈ C, 0 < |z | < r, on a


+∞
 +∞
zez (1−x) (−z )e(−z )x Bn (1 − x) n  B n(x)
= donc z = (−z )n .
ez − 1 e −z − 1 n! n!
n=0 n=0
Deux séries entières dont les sommes coı̈ncident dans un voisinage de 0 ont les mêmes coefficients,
on en déduit (i).
320 4. SUITES ET SÉRIES

(ii). L’idée est de dériver terme à terme l’expression qui définit les polynômes de Bernoulli.
Pour montrer que l’on a bien le droit de procéder ainsi, on fixe ρ ∈ ]0, r[ et on utilise la formule
de Cauchy (voir le théorème 4 page 250) qui donne
 2π
Bn (x) zezx
∀x ∈ R, 2πρn = f (ρeiθ , x)e−niθ dθ où f (z, x) = z .
n! 0 e −1
L’intégrande est continûment dérivable par rapport à x sur R, on peut donc dériver sous le signe
intégral ce qui donne, pour tout x ∈ R,
  2π  2π
n Bn (x) ∂f iθ −niθ Bn−1 (x)
2πρ = (ρe , x)e dθ = ρeiθ f (ρe iθ , x)e−niθ dθ = 2πρn ,
n! 0 ∂x 0 (n − 1)!
d’où on déduit Bn = nB n−1 .
(iii). On procède de manière analogue à (i). Soit x ∈ C. Pour tout z ∈ C, 0 < |z | < r, on a
+∞
 +∞ n
 +∞

zez (x+1) zx zezx Bn (x + 1) n x n B n(x)
= ze + donc z = z + zn
ez − 1 ez − 1 n=0
n! n=0
n! n=0
n!
n
et on conclut en identifiant les coefficients de z de part et d’autre de cette expression.
(iv). C’est une conséquence immédiate de (iii) appliqué à x = 0.
(v). C’est une conséquence de (ii) et (iv).
(vi). L’assertion (i) entraı̂ne B 2n+1(0) = −B2n+1(1), donc d’après (iv), B2n+1 (0) = 0. Il suffit
ensuite de remarquer que Bk (0) = bk pour tout k par définition des b k et des Bk (x).
b) Un produit de Cauchy donne, pour tout x ∈ C et pour tout z ∈ C∗ , |z | < r,
+∞
 +∞  +∞  +∞  n 
 B n(x) z  bn  xn   b n−k x k
zn = z ezx = zn zn = zn
n=0
n! e − 1 n=0
n! n=0
n! n=0
(n − k )! k!
k=0
donc en identifiant les coefficients de zn de part et d’autre
n

∀n ∈ N, ∀x ∈ C, Bn (x) = Cknb n−k xk .
k=0

Comme Bn(1) = B n (0) = bn pour tout n ≥ 2, cette dernière relation entraı̂ne


n
 n n−2
1 k 1 k
∀n ≥ 2, Ckn bn−k = 0 donc bn−1 =− C n bn−k = − Cn bk .
n n
k=1 k=2 k=0
En procédant par récurrence (sachant que b0 = 1/f (0) = 1) ceci permet d’affirmer que bn ∈ Q
pour tout n ∈ N. De plus, cette relation de récurrence permet de calculer les bk . On trouve
b 0 = 1, b 1 = −1/2, b 2 = 1/6, b4 = −1/30, b6 = 1/42.
2/ a) Les B̃p sont des fonctions 1-périodiques, continues par morceaux, et égales à la demi-
somme de leur limite à gauche et à droite en leurs discontinuités, elles sont donc égales à leurs
séries de Fourier. Par commodité, nous calculons les coefficients de Fourier cn ( B̃p). Montrons
que
p!
∀p ∈ N∗, c0 (B̃ p) = 0 et ∀n ∈ Z ∗ , cn (B̃p) = . (∗)
(2πni)p
La première égalité de (*) est une conséquence de l’assertion (v) établie à la question 1/a).
Pour la seconde, nous procédons par récurrence sur p ∈ N∗ . Pour p = 1, on a B1(x) = x− 1/2
d’après la question 1/b), donc une intégration par parties fournit, pour tout n ∈ Z∗ ,
 1    1  1
1 −2πnix 1 e−2πnix 1 1
cn(B̃ 1 ) = x− e dx = x− + e−2πnix dx = .
0 2 2 −2πni 0 2πni 0 2πni
Supposons maintenant le résultat vrai au rang p − 1 ≥ 1 et montrons le au rang p. On a p ≥ 2,
donc Bp (0) = Bp(1) et comme de plus B p = pBp−1 , une intégration par parties entraı̂ne
 1  1
−2πnix 1 p p!
c n( B̃p) = Bp(x)e dx = pBp−1(x)e −2πnix dx = c n(B̃ p−1) = p
.
0 2πni 0 2πni (2πni)
7. SUJETS D’ÉTUDE 321

Ainsi, nous avons prouvé (*). Avec les relations liant les coefficients de Fourier an ( B̃p), b n(B̃ p)
aux cn (B̃ p), on en déduit a 0( B̃p ) = 0 pour tout p ∈ N∗, et pour tout k, n ∈ N ∗ ,
(−1)k+1 2(2k)! (−1)k2(2k − 1)!
a n(B̃2k ) = , bn (B̃ 2k ) = 0, an (B̃ 2k−1 ) = 0, bn ( B̃2k−1 ) = .
(2nπ) 2k (2nπ) 2k−1
On en déduit le résultat car nous avons vu plus haut que les fonctions B̃p sont égales à leur série
de Fourier.
b) En faisant x = 0 dans la première relation établie à la question précédente, on a
+∞

b 2k 1 (−1) k+1b 2k
∀k ∈ N ∗, = 2 · (−1) k+1 donc ζ (2k) = (2π)2k. (∗∗)
(2k )! n=1
(2nπ)2k 2(2k)!

En particulier, on trouve ζ (2) = π2 /6, ζ (4) = π4/90, ζ (6) = π6 /945.


c) pour tout k ∈ N∗ , on a
 n +∞
  +∞
1 dt 1 dt 1
∀n ≥ 2, ≤ donc ≤ = .
n2k n−1 t2k n=2
n 2k
1 t 2k 2k − 1

On en déduit 1 ≤ ζ (2k) ≤ 1 + 1/(2k − 1), donc ζ (2k ) → 1 lorsque k → +∞. Grâce à la relation
(**), on en déduit
 √ 2k+1/2 
2(2k)! 4 π k
b2k ∼ (−1)k+1 ∼ (−1)k+1 en utilisant la formule de Stirling .
(2π)2k (eπ )2k

Remarque. On ne connaı̂t presque rien sur la somme de lasérie 1/np lorsque p est
impair (le seul résultat connu à ce jour sur le sujet est que +∞ 3
n=1 1/n est irrationnel —
démontré par Apéry en 1978).
– Les nombres de Bernoulli jouent un rôle important et assez mystérieux dans des parties
aussi diverses des mathématiques que l’analyse, la théorie des nombres et la topologie
différentielle. Citons par exemple l’étonnant théorème de Von Staudt : si pour tout n ∈
N∗ , s n désigne la somme des inverses des nombres premiers p tels que p − 1 divise 2n,
alors sn + b2n est un entier. Le sujet d’étude qui suit propose également une intéressante
application des nombres et des polynômes de Bernoulli.

Sujet d’étude 3 (Formule d’Euler-Maclaurin). Ce sujet d’étude fait suite au


précédent, dont on reprend les notations.
a) Soient (m, n) ∈ Z 2, m < n, r ∈ N∗ , et f : [m, n] → C de classe Cr . Montrer
 n
1
f (m) + f (m + 1) + · · · + f (n) = f (t) dt + [f (m) + f (n)]
m 2
r
 bh  
(h−1) (h−1)
 (−1)r+1 n
+ f (n) − f (m) + R r avec Rr = B̃ r (t)f(r) (t) dt.
h=2
h! r! m

b) (Application.) Donner, lorsque n → +∞, un développement asymptotique à un nombre


fixé quelconque de termes de Hn = 1 + 12 + · · · + n1 .

Solution. a) On va procéder par récurrence sur r ∈ N∗ . Pour r = 1, compte tenu du fait que
B1 (x) = x − 1/2 sur ]0, 1[, une intégration par parties donne pour tout entier k , m ≤ k ≤ n − 1,
 k+1  k+1  k+1  k+1
 f (k + 1) + f (k )
B̃ 1(t)f (t) dt = B̃ 1 (t)f (t) − f (t) dt = − f (t) dt,
k k k 2 k
322 4. SUITES ET SÉRIES

puis en sommant cette relation pour k allant de m à n − 1, on obtient


 n  n
f (m) + f (n)
B̃1 (t)f (t) dt = f (m) + f (m + 1) + · · · + f (n) − − f (t) dt,
m 2 m
d’où le résultat pour r = 1.
Supposons le résultat vrai au rang r − 1 ∈ N∗ et montrons le au rang r. Soit k ∈ Z,
m ≤ k ≤ n −1. Sur ]k, k +1[, on a B̃r = r B̃r−1 ; par ailleurs, pour r ≥ 2, on a Br (0) = Br (1) = b r ,
donc par définition de B̃r , B̃ r est continue sur [k, k + 1]. En intégrant par parties, on obtient
 k+1 
1 (r−1) 1 (r−1)
 k+1 1 k+1
B̃r−1 (t)f (t) dt = B̃ r(t)f (t) − B̃ r(t)f (r) (t) dt
(r − 1)! k r! k r! k
br  (r−1)  1  k+1
= f (k + 1) − f (r−1)(k ) − B̃ r(t)f (r) (t) dt,
r! r! k
donc après sommation de ces relations pour k allant de m à n − 1, puis multiplication par (−1) r ,
(−1)r br  (r−1) 
R r−1 = f (n) − f (r−1) (m) + Rr.
r!
r
On a toujours (−1) br = b r (si r ≥ 2 est impair on a br = 0). On en déduit le résultat au rang r.
b) Soit r ∈ N∗ . On applique la formule précédente à la fonction f (t) = 1/t entre 1 et n. Comme
f (h)(t) = (−1)h h!/th+1, on a
 n    r   n
dt 1 1 bh (−1)h−1 h−1 (−1) r+1 (−1)r r!
Hn = + 1+ + − ( − 1) + B̃ r (t ) dt
1 t 2 n h nh r! 1 t r+1
h=2
 r  +∞  r
1  (−1)h bh dt 1  b 1
= log n + + − B̃r (t) r+1 + + (−1) h−1 h h + εr(n)
2 h 1 t 2n h n
h=2 h=2
  
=γr
avec
 +∞  +∞  
dt dt M 1
ε r (n) = B̃r(t) qui vérifie |εr (n)| ≤ M = r =O
n tr+1 n tr+1 rn nr
( où M = supx∈[0,1] |B r (t)| = sup x∈R |B̃r (t)|). On a donc
r−1
 (−1) h−1 b h 1  
1 1
H n = log n + γ r + + +O ,
2n h nh nr
h=2
et la constante γr est indépendante de r car la formule précédente donne
γ r = lim Hn − log n = γ.
n→+∞
CHAPITRE 5

Fonctions de plusieurs variables

L’émergence de la notion de dérivée a joué un rôle important dans la naissance


de l’analyse au cours du dix-huitième siècle. Elle s’est accompagnée presque
simultanément par la naissance de la différentielle, généralisant naturellement
ce qui était connu sur les fonctions d’une seule variable aux fonctions de
plusieurs variables.
Les énoncés n’avaient pas toujours la précision qu’on essaie de leur donner
aujourd’hui. C’est ainsi que le théorème des fonctions implicites semble avoir
été utilisé sans plus de justification. Sa première démonstration semble due à
Cauchy en 1839.
C’est dans la thèse de Banach (en 1922) que se trouve la généralisation
aux espaces qui portent aujourd’hui son nom.

1. Différentielle, dérivées partielles


1.1. Différentielle
La théorie des fonctions de la variable réelle s’est considérablement développée avec
l’introduction de la notion de dérivée. L’expression f (x + h) = f (x) + hf  (x) + o(h)
s’interprète en disant qu’au voisinage de x, f est approchée par la fonction affine f (x) +
hf  (x) : on a linéarisé f . De la même manière, il est naturel de chercher à linéariser une
fonction f : Rn → Rp, en étudiant l’existence d’une fonction linéaire ϕ : R n → Rp telle
que f (x + h) = f (x) + ϕ(h) à o(h) près. Ceci motive la définition suivante.
Définition 1. Soient E et F deux R-e.v.n, U un ouvert de E et a ∈ U . Une application
f : U → F est dite différentiable en a s’il existe ϕ ∈ Lc (E, F ) (e.v des applications
linéaires et continues de E dans F ) telle que
 
f (a + h) = f (a) + ϕ(h) + o h lorsque h → 0.
Si ϕ existe, ϕ est unique et s’appelle la différentielle de f en a. On la note dfa.
Si f est différentiable en tout point de U , on dit que f est différentiable sur U et
l’application df : U → Lc(E, F ) a → dfa est appelée application différentielle de f . Si
df est continue, on dit que f est de classe C 1 .
Remarque 1. — Une application de la variable réelle f est dérivable en a si et seule-
ment si elle est différentiable en a, et on a dfa : h → f  (a) h. Pour cette raison, on
trouve parfois la notation f  (a) pour désigner la différentielle de f en a.
— En dimension quelconque, dfa dépend a priori des normes  . E et  . F choisies
sur E et F . Cependant, en dimension finie, les normes sont toutes équivalentes et
on montre facilement que l’existence et la valeur de df a ne dépend pas des normes
choisies.
— Noter qu’une différentielle dfa doit être continue. En dimension finie, toutes les
fonctions linéaires sont continues et le problème de la continuité de la différentielle
ne se pose donc pas.
— Enfin, si f est linéaire et continue, l’égalité f (a + h) = f (a) + f (h) montre que f
est différentiable sur E et que dfa = f pour tout a ∈ E .
324 5. FONCTIONS DE PLUSIEURS VARIABLES

Exemple 1. Soit f : M n(R) 2 → M n (R) (X, Y ) → XY . Donnons nous une norme


d’algèbre  .  sur Mn (R) et considérons un point (X0 , Y0 ) ∈ Mn (R)2 . On a
∀(H, K ) ∈ M n (R)2, f (X 0 + H, Y 0 + K) = f (X0 , Y0 ) + (HY0 + X0 K) + HK. (∗)
2
Si on munit Mn(R) de la norme produit (H, K )1 = sup{H , K }, on a HK ≤
H · K ≤ (H, K )21 donc (*) entraı̂ne f (X0 + H, Y0 + K) = f (X 0 , Y0) + ϕ(H, K ) +
o((H, K)1 ) , où ϕ : Mn(R) 2 → Mn (R) (H, K) → HY0 + X0 K (attention à la non-
commutativité des matrices) est linéaire. Comme de plus ϕ est continue (on est en dimen-
sion finie), ceci montre que f est différentiable en (X0 , Y0 ) et que df(X 0,Y0) = ϕ.
Proposition 1. Une fonction différentiable en un point est continue en ce point.
Gradient.
Définition 2. Soient E un espace euclidien et f : U ⊂ E → R une application
différentiable en a ∈ U ( où U est un ouvert de E ). Alors dfa ∈ L(E, R) = E ∗ et il
existe un unique vecteur v de E tel que dfa(h) = v · h pour tout h ∈ E . Le vecteur v
s’appelle le gradient de f en a et est noté grada f .
Remarque 2. On peut également définir la notion de gradient lorsque E est un espace
hilbertien réel, car la propriété d’existence et d’unicité d’un vecteur v de E tel que dfa(h) =
v · h pour tout h ∈ E reste vraie (voir le théorème de Représentation de Riesz, question 3/
du problème 1 page 427).
Propriétés des différentielles.
Proposition 2. Soient E et F deux e.v.n et f , g deux applications de U dans F , où U
est un ouvert de E , toutes deux différentiables en a ∈ U . Alors
— f + g est différentiable en a et d(f + g )a = df a + dga ;
— pour tout λ ∈ R, λf est différentiable en a et d(λf )a = λ dfa.
 Proposition 3. Soient E , F , G des R-e.v.n, U ⊂ E et V ⊂ F deux ouverts, et deux
applications f : U ⊂ E → F , g : V ⊂ F → G vérifiant f (U ) ⊂ V . Si f est différentiable
en a ∈ U et g différentiable en f (a), alors g ◦ f : U → G est différentiable en a et on a
d(g ◦ f )a = dg f (a) ◦ dfa .
Remarque 3. — Si E = R, cette formule s’écrit aussi (g ◦ f )(a) = dg f (a) (f  (a)).
— Si f et g : U ⊂ E → R sont différentiables en a, alors le produit f g l’est aussi
et d(f g)a = df a g + f dga (appliquer la proposition précédente en remarquant que
f g est la composée des applications ϕ : U → R2 x → (f (x), g(x)) et ψ : R2 →
R (x, y ) → xy). Ce résultat reste d’ailleurs valable en remplaçant R par Mn (R),
ou plus généralement par toute algèbre normée.
1.2. Dérivées partielles
Dérivée selon un vecteur.
Définition 3. Soient E et F deux R-e.v.n, U un ouvert de E , et f : U ⊂ E → F une
application. Soit a ∈ U et v ∈ E . Si la fonction à variable réelle ϕ : t → f (a + tv) est
dérivable en t = 0, f est dite dérivable en a selon le vecteur v. On note alors
f (a + tv) − f (a)
fv(a) = ϕ (0) = lim .
t→0
t=0
t

Proposition 4. Si f est différentiable en un point a, alors f admet une dérivée selon


tout vecteur en a et on a fv(a) = dfa (v ) pour tout v ∈ E .
1. DIFFÉRENTIELLE, D ÉRIVÉES PARTIELLES 325

Remarque 4. Attention, la dérivabilité selon tout vecteur en a n’entraı̂ne pas forcément


la différentiabilité de f en a. En fait, cela n’entraı̂ne même pas la continuité en a (voir
l’exercice 1 page 329).
Dérivées partielles. Nous travaillons maintenant sur E = Rn .
Définition 4. Soit une application f : U ⊂ Rn → F , où U est un ouvert de Rn et F
un e.v.n. Soit a ∈ U et (e1, . . . , e n) la base canonique de Rn . Si pour i ∈ {1, . . . , n}, f est
dérivable en a selon ei, on dit que f admet une dérivée partielle en a d’indice i et on note
∂f
fe i (a) = (a).
∂xi

∂f
Remarque 5. — La dérivée partielle ∂x i
(a) est aussi la dérivée de l’application partielle
t → f (a1 , . . . , ai−1 , ai + t, ai+1 , . . . , an ) en t = 0.
— à plus forte raison que dans la remarque précédente, il se peut que toutes les
dérivées partielles de f existent en a sans que f soit différentiable en a, ni même
continue en a (voir cependant le théorème qui suit).
∂f
— Si f : U ⊂ Rn → R est une application différentiable en a ∈ U , alors (a) existe
∂xi
pour tout i, et on a
n n
∂f ∂f
dfa = (a) dxi , grada f = (a) e i ,
i=1
∂x i i=1
∂x i

où ( dxi) est la base duale dans (Rn )∗ de la base canonique de Rn (conséquence de
la proposition 4) — Rn est muni de son produit scalaire standard.

 Théorème 1. Soit f : U ⊂ R n → F une application, où U et un ouvert de R n et F un


e.v.n. Si toutes les dérivées partielles de f sur U existent et si elles sont continues en un
point a de U , alors f est différentiable en a et on a
 n
∂f
dfa = (a) dxi .
i=1
∂x i

Démonstration. On choisit sur Rn la norme x = ni=1 |xi |. Il s’agit de montrer que l’application
n ∂f
g : x = (x1, . . . , x n ) → f (x) − i=1 x i ∂xi
(a) vérifie g (x) − g (a) = o(x − a) au voisinage de a.
Soit ε > 0. Par hypothèse, les dérivées partielles de f sont continues en a, donc
   
 ∂g   ∂f ∂f 
∃α > 0, ∀x ∈ U, a − x < α, ∀i ∈ {1, . . . , n},    
 ∂xi (x) =  ∂xi (x) − ∂xi (a)  < ε.
Quitte à diminuer α > 0, on peut supposer que B(a, α) ⊂ U .
Soit x ∈ B(a, α). On considère les points
y0 = (a 1, . . . , an ) et ∀k ∈ {1, . . . , n}, yk = (x1 , . . . , x k , ak+1, . . . , an ),
de sorte que y0 = a et yn = x. Pour tout k ∈ {1, . . . , n}, la fonction
gk : [a k , xk] → F t → g (x1 , . . . , x k−1 , t, ak+1 , . . . , a n)
([ak , xk ] désigne l’intervalle [xk , ak ] si x k < ak ) vérifie
∂g
g k (t) = (x1 , . . . , xk−1, t, a k+1 , . . . , an ),
∂x k
donc g k(t) < ε sur [a k, xk ]. On en déduit, avec le théorème 5 page 75, que gk (xk ) − g k (ak ) ≤
ε |x k − ak |. Comme gk (ak) = g (y k−1 ) et gk (xk ) = g (y k ), ceci entraı̂ne
 n   n 
   n 
 
g (x) − g(a) =  [g (yk ) − g (yk−1 )]  ≤ g (yk ) −g (y k−1) ≤ ε |x k − ak | = ε x − a.
 
k=1 k=1 k=1
326 5. FONCTIONS DE PLUSIEURS VARIABLES

On a donc bien g (x) − g (a) = o(x − a) au voisinage de a, d’où le résultat. 

Remarque 6. Attention, la réciproque est fausse. Par exemple, l’application f : R → R


définie par f (x) = x 2 sin(1/x) si x = 0, f (0) = 0, est différentiable en 0 mais f  n’est pas
continue en 0.
Dérivées partielles d’ordre supérieur. Sous réserve d’existence, on peut définir par
récurrence sur p une dérivée partielle d’ordre p par la relation
 
∂ pf ∂ ∂ p−1 f
= .
∂xip · · · ∂xi1 ∂xip ∂xi p−1 · · · ∂xi 1
Une fonction f : U ⊂ Rn → F est dite de classe C p si toutes ses dérivées partielles
jusqu’à l’ordre p existent et sont continues sur U . Le théorème 1 assure la cohérence de
cette définition avec la définition 1 pour le cas C 1 .

 Théorème 2 (Schwarz). Soit une application f : U ⊂ R 2 → R, où U est un ouvert de


R2 , telle que f admette des dérivées partielles ∂ 2f /∂x∂y et ∂ 2f/∂y∂x sur U , continues
en un point a de U . Alors
∂2 f ∂2 f
(a) = (a).
∂x∂y ∂y∂x
Démonstration. Soient x0 , y0 les coordonnées de a. Soient h > 0 et k > 0 tels que [x0 , x0 + h] ×
[y0 , y0 + k ] ⊂ U . On pose
δ (h, k ) = f (x0 + h, y 0 + k ) − f (x 0 + h, y0 ) − f (x0, y 0 + k) + f (x 0 , y0 ).
Si on pose ϕ : x → f (x, y0 + k) − f (x, y0 ), on a δ (h, k) = ϕ(x 0 + h) − ϕ(x0 ), et la fonction
ϕ étant dérivable sur [x0, x 0 + h], le théorème des accroissements finis assure l’existence de
θ1 ∈ ]0, 1[ tel que δ (h, k ) = hϕ (x 0 + θ1 h), ce qui s’écrit encore
 
∂f ∂f
δ (h, k ) = h (x0 + θ 1h, y 0 + k ) − (x 0 + θ1 h, y0 ) .
∂x ∂x
Maintenant, l’application y → ∂f (x + θ 1h, y) étant dérivable sur [y0 , y0 + k], une nouvelle
∂x 0
application du théorème des accroissements finis donne l’existence de θ 2 ∈ ]0, 1[ tel que
∂ 2f
δ (h, k ) = hk (x0 + θ 1h, y 0 + θ 2k ). (∗)
∂y∂x
En travaillant à partir de la fonction ψ : y → f (x0 + h, y ) − f (x0 , y), on montrerait de même
l’existence de θ3, θ 4 ∈ ]0, 1[ tels que
∂ 2f
δ (h, k ) = hk (x0 + θ 3h, y 0 + θ 4k ). (∗∗)
∂x∂y
En égalant (*) et (**) et en faisant tendre h et k vers 0, on en déduit en vertu de la continuité
∂2f ∂ 2f
des dérivées partielles ∂x∂y et ∂y∂x en a, l’égalité de ces dernières au point a. 

Remarque 7. Sans la condition de continuité en a des dérivées partielles d’ordre 2, ce


résultat est faux (voir l’exercice 1 page 329 pour un contre-exemple).
Corollaire 1. Si f : U ⊂ Rn → Rm ( où U est un ouvert de Rn ) est une application
de classe C p , alors les dérivées partielles jusqu’à l’ordre p ne dépendent pas de l’ordre de
dérivation. On peut donc les écrire toutes sous la forme
∂q f
où i1 + i2 + · · · + in = q ≤ p.
∂xi11 ∂xi22 · · · ∂xinn
1. DIFFÉRENTIELLE, D ÉRIVÉES PARTIELLES 327

Matrice jacobienne, jacobien. On se donne une fonction f : U ⊂ Rn → Rm ( où U


est un ouvert de Rn ), différentiable en un point a de U , et on désigne par (e 1, . . . , en) et
(e1, . . . , e m) les bases canoniques de R n etRm .
On peut écrire f sous la forme f = m 
i=1 fi ei où pour tout i, f i : U → R est une
application différentiable en a, de sorte que
 m m
 ∂f i
∀j ∈ {1, . . . , n}, dfa(e j) = df i,a (ej ) e i = (a) e i .
i=1 i=1
∂xj
La matrice de dfa dans les bases canoniques de Rn et Rm est
 
∂fi
Ja = (a) ∈ M m,n (R).
∂xj 1≤i≤m
1≤j ≤n

On l’appelle matrice jacobienne de f en a. Lorsque m = n, Ja est une matrice carrée et


son déterminant est appelé jacobien de f en a.
Rappelons que la différentielle de la composée de deux fonctions différentiables est la
composée des différentielles ; la matrice jacobienne de la composée est donc le produit des
matrices jacobiennes. On en déduit en particulier le résultat qui suit :
 Proposition 5. Soient deux applications f : U ⊂ R n → R et ϕ : V ⊂ Rm → R n ( où
U et V sont ouverts) telles que ϕ(V ) ⊂ U . On écrit ϕ sous la forme ϕ = (ϕ1 , . . . , ϕn ) où
ϕi : V → R pour tout i. Soit a ∈ V tel que ϕ est différentiable en a et f est différentiable
en ϕ(a). Alors l’application F = f ◦ ϕ : V → R est différentiable en a et
n
 ∂f
∂F ∂ϕ
∀j ∈ {1, . . . , m}, (a) = (ϕ(a)) · i (a).
∂uj i=1
∂xi ∂uj
Conséquence : En dimension finie, la composée de deux fonctions de classe C p est de
classe C p . En particulier la somme, le produit (pour des fonctions à valeurs réelles) de
fonctions C p est Cp .
Remarque 8. — En particulier, si ϕ est une fonction d’une seule variable réelle, la
formule de la proposition précédente s’écrit
n
∂f
F  (a) = (ϕ(a)) · ϕi(a).
i=1
∂xi

— Il est important de bien maı̂triser la formule de la proposition précédente. Vous


pourrez vous entraı̂ner à montrer que si f : R2 → R est une fonction de classe
C 2 et si ϕ : R∗ × R → R2 (r, θ ) → (r cos θ, r sin θ), l’application F = f ◦ ϕ (qui
est l’expression de f en coordonnées polaires) est de classe C 2 et le laplacien de f
vérifie
∂ 2f ∂2 f ∂2 F 1 ∂F 1 ∂ 2F
∆f = + = + + .
∂x 2 ∂y 2 ∂r 2 r ∂r r2 ∂θ2

1.3. Formules de Taylor


Accroissements finis.

Théorème 3 (Inégalit é des accroissements finis). Soit f : U ⊂ E → F , où E


et F sont deux e.v.n et U un ouvert de E . Soit (a, b) ∈ U 2 tel que le segment [a, b] =
{ta + (1 − t)b, t ∈ [0, 1]} soit inclus dans U . Si f est continue sur [a, b], différentiable
en tout point de ]a, b[ et s’il existe M > 0 tel que |||df c||| ≤ M pour tout c ∈ ]a, b[, alors
f (b) − f (a) ≤ M b − a.
328 5. FONCTIONS DE PLUSIEURS VARIABLES

Démonstration. La fonction g : [0, 1] → F t → f (a + t(b − a)) est continue sur [0, 1], dérivable
sur ]0, 1[, et
∀t ∈ ]0, 1[ , g (t) = dfa+t(b−a) (b − a)
(voir la remarque 8). On a donc g (t) ≤ M b − a sur ]0, 1[, donc d’après le théorème 5 page 75,
f (b) − f (a) = g(1) − g(0) ≤ M b − a.

Conséquence :
— Si U est convexe, si f est différentiable sur U et si |||dfx||| ≤ M pour tout x ∈ U ,
alors
∀(a, b) ∈ U 2, f (b) − f (a) ≤ M b − a.
— Si U est un ouvert connexe et dfx = 0 pour tout x ∈ U , alors f est constante sur
U (en effet, U est connexe par lignes brisées — voir le théorème 5 page 42 — et
f (a) = f (b) pour tout segment [a, b] ⊂ U ).
Remarque 9. Si f prend ses valeurs dans R, si [a, b] ⊂ U , si f est continue sur [a, b] et
différentiable sur ]a, b[, alors l’égalité des accroissements finis est valable et s’écrit
∃c ∈ ]a, b[ , f (b) = f (a) + dfc(b − a).
Comme nous l’avons vu dans la remarque 2 page 74, ceci est faux dès que la dimension
de l’espace d’arrivée est > 1 et on utilise alors l’inégalité des accroissements finis.
Formules de Taylor. Notation. Soit f : U ⊂ R q → R p ( où U est un ouvert de Rq )
une application de classe C k sur U . Par analogie avec le fait que
 n!
∀(a 1 , . . . , aq ) ∈ Rq , (a1 + · · · + aq) n = ai11 · · · a iqq
i +···+i =n
i 1 ! · · · iq !
1 q

on note pour tout n ∈ {1, . . . , k}


 q [n]
 ∂f  n! ∂ nf
hi (a) = hi11 · · · hiqq i
(a).
i=1
∂xi i ! · · · i q!
i1 +···+iq =n 1 ∂xi11 · · · ∂xqq

Cette expression s’appelle puissance symbolique n-ième de qi=1 h i ∂f
∂xi
(a). C’est la dérivée
n-ième de la fonction de la variable réelle t → f (a + th) en t = 0.
A partir de cette notation, nous allons énoncer les formules de Taylor relatives aux
fonctions de plusieurs variables. On les démontre en appliquant à la fonction t → f (x +th)
(t ∈ [0, 1]) les formules de Taylor vraies pour les fonctions de la variable réelle.

Théorème 4 (Formule de Taylor-Lagrange). Soit f : U ⊂ Rn → R ( où U est un


ouvert de R n) une application de classe Cp sur U et x ∈ Rn , h = (h 1 , . . . , hn ) ∈ R n tels
que le segment [x, x + h] = {x + th, t ∈ [0, 1]} soit inclus dans U . Alors il existe θ ∈ ]0, 1[
tel que
 n   n  [2]
∂f 1  ∂f
f (x + h) = f (x) + hi (x) + hi (x) + ···
i=1
∂xi 2! i=1 ∂xi
 n  [p−1]  n [p]
1  ∂f 1  ∂f
··· + hi (x) + hi (x + θh) .
(p − 1)! i=1 ∂xi p! i=1 ∂x i
1. DIFFÉRENTIELLE, D ÉRIVÉES PARTIELLES 329

Exemple 2. Si f : R2 → R est de classe C2 , alors pour tout (h, k) ∈ R2, il existe θ ∈ ]0, 1[
tel que
∂f ∂f
f (h, k) = f (0, 0) + h (0, 0) + k (0, 0)
∂x ∂y
 
1 2 ∂ 2f ∂2f 2
2∂ f
+ h (θh, θk) + 2hk (θh, θk) + k (θh, θk) + o((h, k )2 ).
2 ∂x2 ∂x∂y ∂y2
Remarque 10. Attention, comme pour l’égalité des accroissements finis, ceci n’est vrai que
pour des fonctions à valeurs réelles. Pour des fonctions à valeurs dans Rp , on peut par
contre utiliser la formule de Taylor avec reste intégral.
Théorème 5 (Formule de Taylor avec reste intégral). Soient f : U ⊂ Rq →
Rp ( où U est un ouvert de Rq ) une application de classe Ck sur U , x ∈ Rq et h =
(h 1, . . . , h q ) ∈ R q tels que [x, x + h] ⊂ U . Alors
 q   q  [2]
 ∂f 1  ∂f
f (x + h) = f (x) + hi (x) + hi (x) + ···
i=1
∂xi 2! i=1 ∂xi
 q [k−1]   q [k]
1  ∂f 1
(1 − t) k−1  ∂f
··· + hi (x) + hi (x + th) dt.
(k − 1)! i=1 ∂xi 0 (k − 1)! i=1
∂x i

Théorème 6 (Formule de Taylor-Young). Soit f : U ⊂ Rq → R p ( où U est un


ouvert de Rq ) une application de classe C k et x ∈ U . Alors lorsque h = (h1 , . . . , hq) ∈ Rq
tend vers 0,
 q   q [2]  q [k]
 ∂f 1  ∂f 1  ∂f
f (x+h) = f (x)+ hi (x) + hi (x) +· · ·+ hi (x) +o(h k).
i=1
∂xi 2! i=1
∂x i k! i=1
∂xi

1.4. Exercices
Exercice 1. a) Montrer que la fonction f : R2 → R définie par f (x, y ) = y2/x si x = 0,
f (0, y) = y, est dérivable selon tout vecteur au point (0, 0), mais n’est pas continue en
(0, 0).
b) On considère l’application
x 2 − y2
f : R 2 → R f (x, y ) = xy si (x, y ) = (0, 0), f (0, 0) = 0.
x 2 + y2
∂2 f ∂2 f
Montrer que les dérivées partielles ∂x∂y
(0, 0) et ∂y∂x
(0, 0) existent mais ne sont pas égales
(exemple dû à Péano).

Solution. a) Soit (a, b) un vecteur de R 2 . Si a = 0, on a


t2 b 2 b2
∀t ∈ R ∗, f(ta, tb) − f (0, 0) = = t,
ta a
ce qui montre que f est dérivable en (0, 0) selon le vecteur (a, b) et que f (a,b) (0, 0) = b2/a.
Si a = 0, on a f (0, tb) = tb pour tout t ∈ R, donc f est dérivable en (0, 0) selon le vecteur
(a, b) et f(a,b) (0, 0) = b.
Cependant, f n’est pas continue en (0, 0) car pour tout n ∈ N∗ , f (1/n 3, 1/n) = n tend vers
+∞ lorsque n → +∞, alors que (1/n3 , 1/n) tend vers (0, 0).
b) Pour tout x ∈ R, on a
f (x, y) − f (x, 0) x2 − y 2
∀y =
 0, =x 2 .
y x + y2
330 5. FONCTIONS DE PLUSIEURS VARIABLES

∂f
En faisant tendre y vers 0, on en déduit si x = 0 que ∂y (x, 0) existe et vaut x ; si x = 0, on voit
de même que ∂f
∂y
(0, 0) existe et vaut 0. Finalement, on a ∂f
∂y
(x, 0) = x pour tout x ∈ R. On en
∂2 f
conclut que ∂x∂y (0, 0) existe et vaut 1.
Maintenant, on en déduit, grâce à la relation d’antisymétrie f (y, x) = −f (x, y) le fait que
∂2 f
∂y∂x (0, 0) existe et vaut −1. D’où le résultat.

Exercice 2. Soit f : R n → R une application différentiable en 0 et telle que


∀x ∈ R n , (x = 0), ∀t ∈ R+∗, f (tx) = tf (x).
Montrer que f est linéaire.

Solution. Fixons un vecteur x = 0 quelconque de Rn . La relation f (tx) = tf (x) pour tout


t > 0 montre que f (tx) tend vers 0 lorsque t → 0, et comme f est continue en 0 (car elle est
différentiable en 0), on en conclut f (0) = 0. Maintenant, la différentiabilité de f en 0 entraı̂ne,
lorsque t → 0+,
f (tx) = f (0) + df 0(tx) + o(tx) = t df 0(x) + tε(t) donc f (x) = df0 (x) + ε(t)
avec ε(t) → 0 lorsque t → 0+ . En faisant tendre t vers 0, on en déduit f (x) = df0(x). Ceci est
vrai pour tout x ∈ Rn non nul, et est vrai également pour x = 0 car nous avons montré f (0) = 0.
Finalement, f = df0 est linéaire.

Exercice 3. 1/ Soient E, F, G des e.v.n et ϕ : E × F → G une application bilinéaire. On


suppose que ϕ est continue, de sorte qu’il existe C > 0 tel que ϕ(x, y ) ≤ C x y pour
tout (x, y ) ∈ E × F . Montrer que ϕ est différentiable sur E × F et calculer sa différentielle
dϕ.
2/ (Applications.) a) Soit E un espace préhilbertien réel. Montrer la différentiabilité et
calculer la différentielle de l’application produit scalaire Φ : E2 → R (x, y ) → x · y.
b) Soit n ∈ N∗. Pour tout p ∈ N ∗ , montrer que l’application ϕp : M n (R) → Mn (R)
définie par ϕ(M ) = M p est différentiable et calculer sa différentielle dϕp .

Solution. 1/ Fixons (x, y) ∈ E × F . Pour tout (h, k ) ∈ E × F , on a


ϕ(x + h, y + k ) = ϕ(x, y + k) + ϕ(h, y + k) = ϕ(x, y) + ϕ(x, k) + ϕ(h, y ) + ϕ(h, k ),
donc si L(h, k) = ϕ(x, k) + ϕ(h, y), L est linéaire et
ϕ(x + h, y + k ) − ϕ(x, y) − L(h, k ) = ϕ(h, k ) ≤ C h k  = o((h, k ))
(en prenant par exemple (h, k ) = sup{h, k }). Donc ϕ est différentiable en (x, y) et
dϕ(x,y) = L, c’est-à-dire dϕ(x,y) : (h, k ) → ϕ(x, k) + ϕ(h, y ).
2/ a) L’application Φ est bilinéaire et continue car d’après l’inégalité de Schwarz, |Φ(x, y)| ≤
x y  pour tout (x, y) ∈ E2 . En appliquant le résultat de la question précédente, on en déduit
que Φ est différentiable sur E 2 et que dΦ(x,y) : (h, k ) → x · h + y · k pour tout (x, y) ∈ E 2 .
b) Notons Ψ l’application bilinéaire continue Mn (R) 2 → M n(R) (A, B ) → AB . D’après la
question 1/, Ψ est différentiable et dΨ(A,B ) (H, K) = HB + AK pour tout (A, B).
Ceci étant, montrons par récurrence sur p ∈ N∗ que ϕp est différentiable sur M n (R) et vérifie
p−1

∀M, H ∈ M n(R), dϕp,M (H) = M i HM (p−1)−i.
i=0
1. DIFFÉRENTIELLE, D ÉRIVÉES PARTIELLES 331

Pour p = 1, c’est évident car ϕ1 est linéaire. Supposons le résultat vrai au rang p et montrons
le au rang p + 1. On a ϕp+1 = ϕ1 ϕ p = Ψ(ϕ1 , ϕ p), donc composée d’applications différentiables,
donc ϕp+1 est différentiable et dϕ p+1,M = dΨ ϕ 1(M ),ϕ p (M )[dϕ1,M , dϕ p,M ] c’est-à-dire
dϕp+1,M (H) = dϕ 1,M (H ) ϕp (M ) + ϕ1 (M ) dϕp,M (H )
p−1
 p

= HM p + M M iHM (p−1)−i = M iHM p−i ,
i=0 i=0
d’où le résultat.

 Exercice 4 (Lemme d’Hadamard). Soit f : R n → R une fonction de classe C ∞.


a) On suppose f (0) = 0. Montrer que l’on peut écrire
n

f (x1, . . . , x n) = xi gi (x1 , . . . , xn )
i=1

où pour tout i, gi : Rn → R est une fonction de classe C∞ .


b) Si de plus df0 = 0, montrer que l’on peut écrire

f (x1, . . . , x n ) = x i x j h i,j(x 1 , . . . , x n)
1≤i≤n
1≤j ≤n

où pour tout (i, j ), hi,j : Rn → R est une fonction de classe C ∞.

Solution. a) On commence par écrire la formule de Taylor avec reste intégral à l’ordre 1, qui
s’écrit ici, pour tout x = (x1, . . . , x n) ∈ Rn ,
 1 n
 n
 1
∂f ∂f
f (x) = xi (tx) dt = xi gi(x) avec g i(x) = (tx) dt.
0 ∂x i 0 ∂xi
i=1 i=1
Il suffit ensuite de remarquer, grâce au théorème de dérivation sous le signe intégral, que les
fonctions gi ainsi définies sont de classe C ∞.
∂f
b) Si de plus df0 = 0, alors ∂x i
(0) = g i(0) = 0 pour tout i. Comme les fonctions g i sont de classe

C , on peut leur appliquer le résultat précédent qui montre que pour tout i, on peut trouver des

fonctions (hi,j )1≤j ≤n de classe C ∞ telles que g i (x) = nj=1 xj h i,j (x) pour tout x ∈ Rn . Ainsi,
 
n n 
∀x = (x 1, . . . , xn ) ∈ R n, f(x) = xi  xj hi,j (x) = xi x j hi,j (x),
i=1 j=1 1≤i≤n
1≤j ≤n

d’où le résultat.
Remarque. On aurait pu aussi démontrer le résultat de la question b) directement en
utilisant la formule de Taylor avec reste intégral à l’ordre 2.

Exercice 5 (Différentielle de l’inverse). 1/ a) Soit n ∈ N∗ .Montrer que l’appli-


cation Inv : Gn (R) → Gn (R) M → M −1 est de classe C ∞ sur G n(R).
b) En calculant les dérivées partielles de Inv au point In , calculer la différentielle DInv
de Inv au point I n .
c) En déduire la valeur de la différentielle de Inv en tout point de Gn(R).
2/ On veut généraliser le résultat précédent en dimension infinie. Soit E un espace de
Banach. On note G c(E) l’ensemble des endomorphismes inversibles u de E tels que u et
332 5. FONCTIONS DE PLUSIEURS VARIABLES

u−1 soient continus. La norme utilisée sur Lc (E ) est |||u||| = sup x=1 u(x). On rappelle
(voir l’exercice 4 page 52) que Gc (E) est un ouvert de L c (E).
a) Montrer que l’application Inv : Gc (E ) → Gc (E ) u → u−1 est différentiable au point
IdE et calculer la différentielle de Inv en ce point.
b) Montrer que Inv est différentiable en tout point de Gc(E ) et calculer sa différentielle.

Solution. 1/ a) Remarquons tout d’abord que Gn (R) est un ouvert de Mn (R) (c’est l’image
réciproque de l’ouvert R∗ par l’application déterminant, qui est continue).
Pour tout M ∈ G n(R), on a M −1 = (det M )−1 tM  où M désigne la comatrice de M . Cette
expression montre que les coefficients de M −1 sont des fractions rationnelles en les coefficients
de M , ce qui prouve que Inv est de classe C∞ .
b) On désigne par (Ei,j )1≤i,j ≤n la base canonique de Mn (R) (Ei,j est la matrice dont tous
les termes sont nuls sauf celui d’indice (i, j) qui vaut 1). En notant les matrices sous la forme
M = (mi,j )1≤i,j ≤n , calculons
∂Inv 1 
(I n ) = lim (In + tE i,j )−1 − I n .
∂mi,j t→0 t

Si i = j , on a
 
1 1 1 1 ∂Inv
[(In + tEi,i )−1 − In ] = − 1 E i,i = − Ei,i donc (In) = −E i,i.
t t t+1 t+1 ∂mi,i
Si i = j , la matrice Ei,j vérifie E2i,j = 0 donc pour tout t ∈ R, (In + tE i,j) −1 = In − tEi,j
∂Inv
(car (In − tEi,j )(In + tEi,j ) = In − t 2 E2i,j = In ), et on en déduit ∂m i,j
(I n ) = −Ei,j .
∂Inv
En résumé, nous avons montré ∂m i,j
(In ) = −Ei,j pour tout (i, j ). On en déduit que la
différentielle DInvIn de Inv au point In vérifie
 ∂Inv
∀M = (mi,j )1≤i,j ≤n ∈ Mn (R), DInv In (M ) = mi,j (In ) = −M,
i,j
∂m i,j

d’où le résultat.
c) Soit M ∈ Gn (R). Lorsque H ∈ Mn (R) tend vers 0, on a M + H ∈ Gn(R) pour H suffisam-
ment petit (car G n (R) est ouvert) et
Inv(M + H ) = (M + H ) −1 = (In + M −1 H)−1 M −1 = [In + DInv In (M−1 H ) + o(H )]M−1
= [I n − M −1 H + o(H )]M −1 = Inv(M ) − M −1 HM −1 + o(H )
On en déduit DInvM (H) = −M−1 HM −1 pour tout H ∈ Mn (R).
2/ a) Pour tout h ∈ L c (E), |||h||| < 1, on sait (voir la proposition 2 page 49)  que Idn+h est
inversible et que (Id +h)−1 = Id −h + h2 − h3 + · · · + (−1)n hn + · · · . Comme ||| +∞ n=2(−1) h n ||| ≤
+∞ n 2
n=2 |||h||| = |||h||| /(1 − |||h|||), on en déduit que lorsque h → 0, Inv(Id +h) = Id −h + o(|||h|||).
Ainsi, Inv est différentiable au point Id et DInvId (h) = −h.
b) On procède comme dans la question 1/c). Soit u ∈ Gc (E ). Comme Gc (E ) est ouvert (voir
l’exercice 4 page 52), on a u + h ∈ G c(E) lorsque h est suffisamment petit, et lorsque h → 0,
(u + h)−1 = (Id +u−1 h)−1 u −1 = (Id −u −1h + o(|||h|||)) u−1 = u −1 − u −1hu −1 + o(|||h|||).
De plus, l’application linéaire h → −u −1hu −1 est continue car pour tout h ∈ L c (E), |||u−1 hu−1 ||| ≤
|||u−1||| |||h||| |||u −1|||. En définitive, Inv est différentiable au point u et D Invu (h) = −u −1hu−1 pour
tout h ∈ Lc(E).

Exercice 6 (Différentielle du d éterminant). Soit n ∈ N∗ . Montrer que l’appli-


cation déterminant définie par det : Mn (R) → R M → det M est de classe C ∞ et
calculer sa différentielle D det.
1. DIFFÉRENTIELLE, D ÉRIVÉES PARTIELLES 333

Solution. Le déterminant d’une matrice est un polynôme en ses coefficients, on en déduit que
l’application M → det M est de classe C∞ .
Soit M ∈ Mn (R). Pour calculer D det M (différentielle de det au point M ), nous allons
calculer les dérivées partielles de det au point M . Désignons par (E i,j )1≤i,j ≤n la base canonique
de Mn(R). En notant A = (a i,j )1≤i,j ≤n les éléments de Mn (R), il s’agit donc de calculer les
∂ det
∂ai,j (M ). En désignant par C = (Ci,j )1≤i,j ≤n la comatrice de M (de sorte que Ci,j est le cofacteur
de l’élément d’indice (i, j ) de M ), la n-linéarité du déterminant entraı̂ne, pour tout (i, j) et pour
tout t ∈ R,
∂ det det(M + tE i,j) − det M
det(M + tEi,j ) = (det M ) + t Ci,j donc (M ) = lim = Ci,j .
∂a i,j t→0
t=0
t

Ainsi, si H = (h i,j ) ∈ Mn (R), on a


 ∂ det 
D det M (H) = hi,j (M ) = hi,j C i,j = tr( tCH).
i,j
∂a i,j i,j

Lorsque M est inversible, on peut obtenir une autre expression de D det M en utilisant
l’identité t C = (det M )M −1 qui entraı̂ne D det M (H) = (det M ) tr(M −1H ) pour tout H ∈
Mn (R).

Exercice 7. Soit f : R → R une application de classe C1. On définit l’application


f (x) − f (y )
F : R2 → R F (x, y ) = si x = y, F (x, x) = f  (x).
x−y
a) Montrer que F est continue.
b) Si f est de classe C2, montrer que F est de classe C 1.

Solution. a) Si x = y , on a
 y  1
1 
F (x, y) = f (u) du = f [x + t(y − x)] dt, (∗)
y−x x 0

égalité qui reste évidemment vraie pour x = y . Comme f est de classe C 1, la dernière intégrale
de (*) a son intégrande qui est continue en (x, y ) sur R2 , donc d’après le théorème de continuité
sous le signe intégral, F est continue sur R2.
b) Notre point de départ est toujours la relation (*). L’intégrande f (x + t(y − x)) est continûment
dérivable par rapport à x, on peut donc appliquer le théorème de dérivation sous le signe intégral
qui montre que F admet une dérivée partielle par rapport à x sur R 2 qui vérifie
 1  1
∂F ∂   
2
∀(x, y ) ∈ R , (x, y) = f [x + t(y − x)] dt = (1 − t)f [x + t(y − x)] dt,
∂x 0 ∂x 0

fonction de (x, y ) qui est bien continue sur R 2. De même, F admet une dérivée partielle par
rapport à y continue sur R2. On en conclut que F est de classe C 1 .
Remarque. On peut également résoudre l’exercice sans exprimer F par l’intégrale (*),
mais c’est plus difficile.

Exercice 8 (Fonctions holomorphes, fonctions harmoniques). On note D =


{z ∈ C, |z | < 1} le disque unité ouvert complexe. Par commodité, on identifie les nombres
complexes x + iy avec les couples (x, y) (avec x, y ∈ R).
1/ Soit f : D → C une application, et u, v les deux applications définies sur D , à valeurs
334 5. FONCTIONS DE PLUSIEURS VARIABLES

réelles, telles que f (x + iy ) = u(x, y ) + iv (x, y ). On dit que f est holomorphe sur D si elle
est continûment dérivable par rapport à la variable complexe z, c’est-à-dire si
(i) pour tout z0 ∈ D, (f (z 0 + h) − f (z 0))/h converge lorsque h ∈ C ∗ tend vers 0 (cette
limite est notée f  (z 0)),
(ii) l’application z → f (z) est continue sur D.
a) Montrer que f est holomorphe si et seulement si les applications u et v sont de classe
C1 sur D et vérifient les conditions de Cauchy
∂u ∂v ∂u ∂v
= , =− .
∂x ∂y ∂y ∂x

b) Si f est holomorphe et si u et v sont de classe C2 alors montrer que u et v sont


harmoniques, c’est-à-dire
∂ 2u ∂ 2 u ∂ 2 v ∂2 v
+ = 0, + = 0.
∂x2 ∂y 2 ∂x2 ∂y2

2/ On considère une fonction u : D → R de classe C2 et harmonique.


a) Montrer que ∂u/∂x est la partie réelle d’une fonction holomorphe.
b) Montrer que u est la partie réelle d’une fonction holomorphe.

Solution. 1/ a) On remarque tout d’abord que les conditions de Cauchy sont équivalentes à
l’identité ∂f ∂f
∂x + i ∂y = 0.
Supposons f holomorphe. Soit z = x + iy ∈ D (avec x, y ∈ R). Lorsqu’on fait tendre le
nombre réel s vers 0 dans l’expression
f (z + s) − f (z ) f (x + s, y) − f (x, y)
=
s s

le membre de gauche converge vers f (z), donc celui de droite converge, ce qui montre l’existence
de ∂f  ∂f
∂x (x, y) et de plus, f (z) = ∂x (x, y). Lorsque cette fois ci, on fait tendre le nombre réel t vers
0 dans l’expression
f (z + it) − f (z ) 1 f (x, y + t) − f (x, y)
=
it i t
on aboutit à l’existence de ∂y (x, y ) et à l’égalité f (z) = −i ∂f
∂f  ∂f
∂y (x, y ). Ainsi, on a montré ∂x (x, y)+
i ∂f  ∂f  ∂f 
∂y (x, y ) = 0. Par ailleurs, f étant continue, les égalités ∂x = f et ∂y = if , entraı̂nent que les
dérivées partielles de f sont bien continues, donc f (et donc u et v ) est de classe C 1 .
Réciproquement, supposons f = u + iv de classe C 1 et vérifiant ∂f ∂f
∂x + i ∂y = 0. Soit z ∈ D .
L’application f est différentiable en z = x + iy donc la formule de Taylor-Young entraı̂ne, lorsque
les nombres réels s et t tendent vers 0
∂f ∂f
f (x + s, y + t) − f (x, y) = s (x, y) + t (x, y) + o(|s + it|).
∂x ∂y
∂f
Comme ∂y
= i ∂f
∂x
ceci entraı̂ne
∂f ∂f ∂f
f (x + s, y + t) − f (x, y) = s (x, y) + it (x, y) + o(|s + it|) = (s + it) (x, y) + o(|s + it|).
∂x ∂x ∂x
Ceci montre que la limite (f (z + h) − f (z ))/h converge vers ∂f ∂x (x, y) lorsque h = s + it tend vers
0. Donc f est dérivable par rapport à la variable complexe z et de plus, f (z) = ∂f ∂x (x, y). Cette
dernière égalité entraı̂ne la continuité de f  , donc f est bien holomorphe.
b) Les conditions de Cauchy et l’indépendance des dérivées partielles à l’ordre de dérivation
entraı̂nent
       
∂ 2u ∂ ∂u ∂ ∂v ∂ 2v ∂ ∂v ∂ ∂u ∂ 2u
2
= = = = = − = − 2.
∂x ∂x ∂x ∂x ∂y ∂xy ∂y ∂x ∂y ∂y ∂y
2. EXTREMUMS RELATIFS 335

Donc u est harmonique. On montrerait de même que v est harmonique.


2/a) On va montrer que f = ∂u ∂u 1
∂x − i ∂y est holomorphe. La fonction f est bien de classe C et
comme u est harmonique, on peut écrire
∂f ∂2 u ∂ 2u ∂2 u ∂2 u ∂f
= −i 2 = +i 2 = i .
∂y ∂yx ∂y ∂xy ∂x ∂x
Ainsi, les conditions de Cauchy sont vérifiées, donc f est bien holomorphe.
b) Il faut en quelque sorte exhiber une primitive de f = ∂u ∂u
∂x − i ∂y . Pour tout z ∈ D , on va
considérer l’intégrale curviligne de f le long d’un segment joignant 0 à z (c’est possible car le
domaine D est une partie étoilée par rapport à 0), en posant
1 1
g (z ) = z f (tz ) dt = (x + iy) f (tx, ty ) dt.
0 0
Le théorème de dérivation sous le signe intégral nous assure que g est continûment dérivable par
rapport à x et par rapport à y et que
 1  1
∂g ∂f
(x, y) = f (tx, ty ) dt + (x + iy) t (tx, ty ) dt
∂x 0 0 ∂x
1  1
∂g ∂f
(x, y) = i f (tx, ty ) dt + (x + iy) t (tx, ty ) dt.
∂y 0 0 ∂y
On en déduit
 1  
∂g ∂g ∂f ∂f
(x, y) + i (x, y) = (x + iy) t (tx, ty) + i (tx, ty ) dt.
∂x ∂y 0 ∂x ∂y
∂f
et comme ∂x + i ∂f
= 0, on en déduit
∂y
∂g ∂g
∂x + i ∂y = 0. Ainsi la fonction g est holomorphe. Calculons
sa partie réelle. On a
 1  
∂u ∂u
g (z ) = (x + iy) −i (tx, ty ) dt
0 ∂x ∂y
donc  1 
∂u ∂u
(g)(z) = x (tx, ty) + y (tx, ty ) dt.
0 ∂x ∂y
L’intégrande n’est autre que la dérivée par rapport à t de l’application t → u(tx, ty ), on en déduit
 1
(g)(z) = u(tx, ty ) 0 = u(x, y ) − u(0, 0). Ainsi, la fonction h = g + u(0, 0) est holomorphe (la
propriété d’holomorphie ne change pas si on ajoute une constante à g) et sa partie réelle est bien
égale à u.
Remarque. Les fonctions holomorphes font aussi l’objet de l’exercice 13 page 265. Dans la
remarque à la fin de cet exercice, il est montré qu’une fonction holomorphe est analytique.
On en déduit qu’une fonction holomorphe est forcément de classe C ∞ .
– Lorsque le domaine D de définition contient des trous (plus précisément s’il n’est pas
simplement connexe, c’est-à-dire qu’un lacet dans D n’est pas continûment déformable en
un point tout en restant dans D), une fonction harmonique n’est pas forcément la partie
réelle d’une fonction holomorphe.

2. Extremums relatifs
Dans toute cette section, U désigne un ouvert de Rn.
Proposition 1. Si f : U ⊂ Rn → R admet un extremum relatif en un point a de U et
∂f
si f est différentiable en a, alors dfa = 0 (en d’autres termes, ∂xi
(a) = 0 pour tout i).
336 5. FONCTIONS DE PLUSIEURS VARIABLES

Un point a pour lequel dfa = 0 est appelé point critique de f . Ce résultat nous
dit qu’un extremum relatif est nécessairement un point critique (notez que U doit être
ouvert : c’est comme dans R, un extremum relatif est un point critique lorsque c’est un
point intérieur à l’ensemble de définition). La réciproque est fausse dans le cas général.
Cependant, moyennant un développement limité de f à l’ordre 2, il est possible d’obtenir
une condition suffisante d’existence d’un extremum.
 Théorème 1. Soit f : U ⊂ R n → R une fonction de classe C 2 et supposons qu’il existe
a ∈ U tel que dfa = 0, de sorte que d’après la formule de Taylor-Young, f (a + h) =
f (a) + Q(h)/2 + o(h2), où Q est la forme quadratique
 n [2]
 ∂f  ∂2 f  ∂ 2f
Q(h) = hi (a) = h2i (a) + 2 hihj (a).
i=1
∂x i i
∂x 2i i<j
∂x i ∂xj

Alors
(i) si f admet un minimum (resp. un maximum) relatif en a, Q est une forme qua-
dratique positive (resp. négative) ;
(ii) si Q est une forme quadratique définie positive (resp. définie négative), f admet
un minimum (resp. un maximum) relatif en a.
Démonstration. Si f admet un minimum relatif en a, on a, sur un voisinage de 0 pour h,
1
f (a + h) = f (a) + Q(h) + o(h 2) ≥ f (a) donc Q(h) + o(h2 ) ≥ 0.
2
Fixons h ∈ Rn . Lorsque t ∈ R tend vers 0, on peut donc écrire
Q(th) + o(th2 ) = t2(Q(h) + o(1)) ≥ 0 donc Q(h) + o(1) ≥ 0
En faisant tendre t vers 0, on en déduit Q(h) ≥ 0. Ceci est vrai pour tout h ∈ Rn , d’où (i).
Si Q est une forme quadratique définie positive, alors pour tout h ∈ R n, h = 0, Q(h) > 0.
Comme la sphère unité de R n est compacte, on en déduit α = inf h=1 Q(h) > 0. Ainsi, lorsque
h tend vers 0,
   
1 2 h 2 h h2
f (a + h) − f (a) = [Q(h) + o(h )] = Q + o(1) ≥ (α + o(1)).
2 2 h 2
Comme α + o(1) ≥ 0 sur un voisinage de h = 0, on en déduit f (a + h) ≥ f (a) sur ce voisinage,
d’où (ii). 

Remarque 1. — Si la forme quadratique Q est seulement supposée positive (et non


pas définie positive), on n’est pas assuré d’avoir un minimum relatif en a. Par
exemple, pour la fonction f : R → R x → x 3 en a = 0, Q = 0 est positive (elle
est nulle) et pourtant, f n’admet pas d’extremum en 0.
— Dire que la forme quadratique Q2 est positive
 (resp. définie positive), c’est dire que
∂ f
la matrice symétrique A = ∂xi∂xj (a) est positive (resp. définie positive), ou
i,j
encore que les valeurs propres de A sont ≥ 0 (resp. > 0). La matrice A est appelée
matrice hessienne de F au point critique a.
 
Exemple 1. (en dimension 2). Soit A = rs st ∈ M 2 (R) et supposons det A = rt − s2 > 0.
Alors les deux valeurs propres λ1 et λ2 de A (qui sont réelles car A est symétrique) ont
même signe (car λ1 λ2 = det A > 0). De plus, λ1 + λ 2 = tr A = r + t, et comme r et t ont
même signe (car det A = rt − s2 > 0 donc rt > 0), on en déduit que λ1 + λ 2 a le signe de
r. Ainsi, les deux valeurs propres λ1 et λ2 ont le signe de r.
En résumé, si det A > 0, alors A est définie positive (resp. définie négative) si et
seulement si r > 0 (resp. r < 0). Si det A < 0, on a λ1 λ2 < 0 donc A n’est ni positive, ni
négative.
2. EXTREMUMS RELATIFS 337

Maintenant, considérons une fonction f : U ⊂ R2 → R de classe C 2 et telle que


dfa = 0 pour a ∈ U . Posons
∂2 f ∂2f ∂2 f
r= (a), s= (a), t= (a).
∂x2 ∂x∂y ∂y2
Alors d’après le théorème et ce que l’on vient de voir,
— si rt − s2 > 0 et r > 0, f admet un minimum relatif en a ;
— si rt − s2 > 0 et r < 0, f admet un maximum relatif en a ;
— si rt − s2 < 0, f n’a pas un extremum en a ;
— si rt − s2 = 0, on ne peut conclure.
Extremums liés. On doit parfois maximiser (ou minimiser) une fonction f : U ⊂ Rn →
R sur un sous-ensemble de U de la forme Γ = {x ∈ U | g1(x) = · · · = gr (x) = 0}, où
les gi sont des fonctions de U dans R. Pour ce faire, le théorème suivant, démontré à la
page 347, rend de précieux services ; il est dû à Lagrange.
Théorème 2. Soient f, g1 , . . . , gr : U ⊂ R n → R des fonctions de classe C1 , où U est
un ouvert de Rn. On désigne par Γ l’ensemble {x ∈ U | g1 (x) = · · · = gr (x) = 0}.
Si f|Γ (restriction de f à Γ) admet un extremum relatif en a ∈ Γ et si les formes
linéaires dg1,a , . . . , dgr,a sont linéairement indépendantes, alors il existe des réels λ1, . . . , λr
(appelés multiplicateurs de Lagrange) tels que
dfa = λ1 dg 1,a + · · · + λr dgr,a .
Remarque 2. — La famille (dgi,a) étant libre, les multiplicateurs de Lagrange (λ i) sont
uniques.
— La condition “U ouvert” est essentielle dans le théorème des extremums liés.
2.1. Exercices
Exercice 1. Étudier les extremums relatifs puis les extremums absolus de la fonction
f : R 2 → R (x, y ) → x 4 + y 4 − 2(x − y)2 .

Solution. La fonction f est de classe C ∞ . Commençons par rechercher les extremums relatifs de
f . Pour cela, on recherche ses points critiques (x, y), qui vérifient df(x,y) = 0, c’est-à-dire
∂f ∂f
0= = 4x 3 − 4(x − y) et 0= = 4y 3 + 4(x − y).
∂x ∂y
Ce système de deux équations s’écrit aussi
 
x3 + y3 =0 (x + y)(x 2 − xy + y2) =0
3
ou encore .
x − (x − y) = 0 x3 − x + y =0
Ce système admet la solution triviale (x, y) = (0, 0). Si (x, y) = (0, 0), alors x2 − xy + y 2 =
(x − y )2 /2 + x2 /2 + y 2 /2 > 0 n’est pas nul, donc le système équivaut dans ce cas à

x+y =0 √ √   √ √ 
3
dont les solutions non nulles sont (x, y) = 2, − 2 et − 2, 2 .
x − 2x = 0
√ √ √ √
Finalement, f a trois points critiques : (0, 0), ( 2, − 2) et (− 2, 2).
— Au point (0, 0), on voit facilement que la matrice hessienne de f est négative mais non
définie négative (on a r = t = −4 et s = 4). On ne peut donc pas conclure en utilisant le
théorème 1 page 336. On s’en sort en remarquant que f (x, −x) = 2x 4 − 8x 2 = −2x 2(4 −
x2 ) < 0 pour 0 < |x| < 2. Par ailleurs f (x, x) = 2x4 > 0 pour tout x = 0. Ainsi, f ne
présente ni maximum ni minimum relatif en (0, 0) (on a affaire à un point col ).
338 5. FONCTIONS DE PLUSIEURS VARIABLES

√ √ ∂2f ∂ 2f ∂ 2f
— Au point ( 2, − 2), comme ∂x2
= 12x2 − 4, ∂x∂y = 4, = 12y 2 − 4, on a r = 20,
∂y2 √ √
s = 4 et t = 20. Donc rt − s2 > 0 et r > 0. La matrice hessienne de f en ( √2, − √ 2) est
donc définie positive, donc f admet un minimum relatif en ce point (égal à f ( 2, − 2) =
−8). √ √
— Le résultat est identique en (− 2, 2) car f vérifie la relation f (x, y) = f (−x, √ −y √).
√Ainsi,
√ les seuls extremums relatifs de f sont des minimums, atteints aux points ( 2, − 2) et
(− 2, 2). Nous allons prouver, par une technique générale, que ces minimums relatifs sont en
fait des minimums absolus (c’est bien sûr faux dans le cas général). Il suffit pour cela de prouver
que f admet bien un minimum absolu, car tout minimum absolu est un minimum relatif (et la
valeur des deux minimums relatifs est la même dans notre cas).
L’idée est de dire que lorsque |x| et |y | sont grands, 2(x − y )2 est petit par rapport à x4 + y4 .
Pour cela, on utilise la norme (x, y) = sup{|x|, |y|} et les inégalités
 2
x4 + y 4 ≥ (x, y ) 4, 2(x − y )2 ≤ 2 2 (x, y ) = 8 (x, y ) 2
qui entraı̂nent f (x, y) ≥ (x, y )4 − 8 (x, y ) 2 . Ainsi, lim (x,y)→+∞ f (x, y) = +∞. On en déduit
qu’il existe un compact K de R2 tel que f (x, y) ≥ 0 dès que (x, y) ∈ K . La fonction f est
continue sur le compact K , donc il existe (a, b) ∈ K tel que f (a, b) = inf (x,y)∈K f (x, y). Comme
√ √ √ √
f (− 2, 2) = −8, on a (− 2, 2) ∈ K , et donc f (a, b) ≤ −8. Comme f (x, y ) ≥ 0 pour
(x, y ) ∈ K , on a en fait f (x, y ) ≥ f (a, b) pour tout (x, y ) ∈ R2 . Ainsi, f admet un minimum
absolu sur R2, qui est donc un minimum relatif, donc égal à −8.
Remarque. Pour montrer que f admet bien un minimum absolu égal à −8, on aurait pu
montrer directement (en utilisant de bonnes minorations), que f (x, y ) ≥ −8 pour tout
(x, y ) ∈ R2. L’avantage de la méthode que nous avons présentée est qu’elle est générale.

Exercice 2. On note S = {x ∈ Rn | x = 1} la sphère unité de Rn. Soit f : R n → R


une fonction différentiable telle que f |S (restriction de f à S ) soit constante. Montrer qu’il
existe x0 ∈ Rn , x0  < 1, tel que dfx0 = 0.

Solution. C’est l’équivalent du théorème de Rolle en dimension n. On procède comme sur R.


La boule unité fermée B de Rn est compacte, et comme la fonction f est continue (car
différentiable), il existe x0 ∈ B tel que f (x0 ) = min x∈B f (x) = m et il existe x1 ∈ B tel que
f (x1 ) = maxx∈B f (x) = M .
Si m = M , alors f est constante sur B , donc pour tout x ∈ Rn , x < 1, dfx = 0.
Sinon, m < M , et comme f est constante sur S, l’un des points x0 , x 1 n’est pas dans S.

Supposons par exemple x 0 ∈ S. Alors x0  < 1, donc f prend sur l’ouvert B son minimum en
x0. Comme de plus f est différentiable en x 0 , on en déduit dfx0 = 0.

Exercice 3 (Principe du maximum). Soit f : Rn → R une fonction de classe C 2. On


définit le laplacien de f par
n
∂2 f
∆f = .
i=1
∂x2i
On note D la boule unité ouverte de R n et D la boule unité fermée de Rn.
a) Si ∆f (x) > 0 pour tout x ∈ D , montrer que pour tout x ∈ D , f (x) < maxy=1 f (y ).
b) Si ∆f (x) = 0 pour tout x ∈ D (on dit alors f est harmonique sur D ), montrer
∀x ∈ D, min f (y ) ≤ f (x) ≤ max f (y ).
y=1 y=1
2. EXTREMUMS RELATIFS 339

Solution. a) Raisonnons par l’absurde en supposant l’existence de x0 ∈ D tel que f (x0 ) ≥


maxy=1 f (y ). L’application f est continue sur le compact D, il existe donc x ∈ D tel que
f (x) = supy∈D f (y ). Si f (x) = f (x0 ), on choisit x = x0 , sinon f (x) > f(x0 ) ≥ sup y=1 f (y ),
donc x ∈ D. Dans tous les cas, x appartient à l’ouvert D. On en déduit (voir le théorème 1
page 336) que dfx = 0 et que la matrice hessienne
 2 
∂f
A= (x)
∂xi∂xj 1≤i,j ≤n

est négative. En particulier, la trace de A est négative (c’est la somme de ses valeurs propres),
ce qui s’écrit ∆f (x) ≤ 0. Ceci est contraire aux hypothèses, d’où le résultat.
b) Nous montrons que pour tout x ∈ D , f (x) ≤ maxy=1 f (y ), l’autre inégalité s’en déduisant
ensuite en considérant −f . n 
Soit ε > 0. Considérons la fonction g : Rn → R définie par g (x) = f (x) + ε 2
i=1 xi . On a
∆g = ∆f + 2nε, ce qui montre que ∆g > 0 sur D . D’après la question précédente, on en déduit,
pour tout x ∈ D ,
 n   n

 
2 2
g (x) < max g (y ) donc f (x) + ε xi < max f (y ) + ε max yi .
y=1 y=1 y=1
i=1 i=1

Ceci étant vrai pour tout ε > 0, on en déduit que f (x) ≤ maxy=1 f (y ). D’où le résultat.
Remarque. Le résultat se généralise aisément sous la forme suivante : si f est continue
sur un compact K et harmonique sur l’intérieur de K , alors f atteint son maximum et
son minimum sur la frontière de K .

Exercice 4 (Exemples d’applications du théor ème des extremums li és).


a) Soient n ∈ N∗ , n ≥ 2, et s > 0. Soit f : R n → R (x1, . . . , xn ) → x1 · · · xn, et

Γ = {(x1 , . . . , xn) ∈ (R + )n | ni=1 xi = s}. Étudier le maximum global de f |Γ , la restriction
de f à Γ. Retrouver ainsi l’inégalité arithmético-géométrique.
b) On se donne un entier n ≥ 2 et n réels strictement positifs λ1 , . . . , λn. On note
 n

 x 4i
n
Γ = (x1 , . . . , xn) ∈ R | =1 .
i=1 i
λ4

Si f : Rn → R (x 1, . . . , xn) → ni=1 x2i , déterminer le maximum global de f|Γ.
c) Soient a1 , . . . , an des réels > 0 tels que a1 + · · · + a n = 1 (avec n ≥ 2). Démontrer que
n
 (n − 1)n
ai(1 − a i) ≤ .
i=1
n2n

Solution. a) La fonction f étant continue sur le compact Γ, f|Γ admet un maximum global
atteint en un point a de Γ. Si on note g : R n → R (x1 , . . . , xn) → x 1 + · · · + x n − s et
γ = {x ∈ Rn | g (x) = 0}, alors a ∈ Ω = γ ∩ (R+∗ ) n ⊂ Γ (car f (x) = 0 si l’un des xi est nul, et
f (x) > 0 si x ∈ γ ). Par construction, f |Ω admet un extremum global en a, et comme Ω est un
ouvert de γ , f|γ atteint un extremum local en a. De plus, si a ∈ γ , alors dga = 0, on peut donc
appliquer le théorème des extremums liés qui entraı̂ne l’existence de λ ∈ R tel que dfa = λdga .
On a donc
∂f ∂g f (a)
∀i ∈ {1, . . . , n}, (a) = λ = λ c’est-à-dire = λ.
∂xi ∂x i ai

Or f (a) = 0, on en déduit que tous les a i sont égaux. Comme ni=1 ai = s, on a ai = s/n pour
tout i. La valeur du maximum recherché est donc (s/n) n.
340 5. FONCTIONS DE PLUSIEURS VARIABLES

Ceci s’écrit aussi


n  n n  n
1/n n
 
+ n i=1 xi i=1 xi
∀(x1 , . . . , xn ) ∈ (R ) , xi ≤ c’est-à-dire xi ≤ ,
n n
i=1 i=1
ce qui n’est autre que l’inégalité arithmético-géométrique.
b) La fonction f est continue sur le compact Γ, il existe donc a ∈ Γ tel que f|Γ atteigne son
maximum global en a. 
On note g : Rn → R (x1 , . . . , xn ) → i x4i /λ4i − 1, de sorte que Γ= {x ∈ Rn | g (x) = 0}.
Comme g est de classe C 1 et que dgx =  0 pour tout x ∈ Γ (si dg x = 4 i (x3i /λ 4i ) dxi = 0, alors
xi = 0 pour tout i donc x ∈ Γ), on peut appliquer le théorème des extremums liés qui entraı̂ne
l’existence de µ ∈ R tel que df a = µ dga . On en conclut
 
∂f ∂g 4a3 a2
∀i, 2ai = (a) = µ (a) = µ 4i donc ai 1 − 2µ i4 = 0.
∂xi ∂x i λi λi
Pour tout i, on en déduit que a i = 0 ou a 2i = λ4i /(2µ). Notons I l’ensemble des indices i pour
lesquels ai =
 0. L’ensemble I est non vide car a = 0 (car a ∈ Γ). On a
n
  a4  λ4 
a4i i i
4
=1= 4
= donc 4µ 2 = λ4i ,
λ λ 4µ 2
i=1 i i∈I i i∈I i∈I
et on en déduit
 
λ4 λ4
∀i ∈ I, a2i = i =  i donc f (a) = a 2i = λ4i .
2µ 4
i∈I λi i i∈I

Résumons : nous avons montré que f (a) est nécessairement égal à 4
i∈I λi où I est un
sous-ensemble de {1, . . . , n}. Réciproquement, il est immédiat que tout élément de cette forme
peut-être atteint (définir les ai comme précédemment si i ∈ I , ai = 0 si i ∈ I ). Comme f (a)
est maximal, ceci montre que le maximum de f|Γ est atteint lorsque I = {1, . . . , n}, donc égal à
n 4 1/2
i=1 λi .

c) Soit ϕ : Rn → R (x 1 , . . . , xn ) → ni=1 xi (1 − xi ). Il s’agit d’évaluer le maximum de ϕ|Γ ,

où Γ = {(x1 , . . . , xn ) ∈ (R+∗ )n | i xi = 1}. La fonction ϕ étant continue sur le compact
γ = {(x1 , . . . , x n) ∈ (R +)n | x
i i = 1} , il existe a ∈ γ tel que ϕ(a) = supx∈γ ϕ(x). Si l’un des
ai est nul, alors ϕ(a) = 0, ce qui contredit la maximalité de ϕ(a). On a donc a ∈ Γ.
En d’autres termes, si f désigne la restriction de ϕ à l’ouvert (R+∗ )n , nous avons montré
l’existence de a ∈ Γ tel que f (a) = ϕ(a) = supx∈Γ f (x).
Soit g la fonction (x1 , . . . , x n ) → x1 + · · · + x n − 1, de sorte que Γ = {x ∈ (R+∗ )n | g (x) = 0}.
La fonction g est de classe C 1 et dgx =  0 pour tout x ∈ Γ, on peut donc appliquer le théorème
des extremums liés qui entraı̂ne l’existence de λ ∈ R tel que dfa = λ dg a. Ceci entraı̂ne, pour
tout i,
∂f ∂g f (a)
(a) = λ c’est-à-dire (1 − 2a i ) = λ. (∗)
∂xi ∂xi ai(1 − ai )
Autrement dit, chaque ai est solution de l’équation F ( X ) = λX 2 − λX − 2f (a)X + f (a) = 0.
Comme l’un au moins des a i est inférieur à 1/n (car i ai = 1), donc inférieur à 1/2, et que
ai ∈ ]0, 1[, la dernière égalité de (*) montre que λ ≥ 0. Si λ > 0, F est une fonction polynôme
de degré 2 qui vérifie F (0) = f (a) > 0, F (1) = −f (a) < 0 et lim X→+∞ F (X ) = +∞, donc F
a ses deux racines réelles, l’une dans ]0, 1[, l’autre dans ]1, +∞[. Si λ = 0, F est affine donc a
au plus une solution dans ]0, 1[. Dans tous les cas, F a donc au plus une solution dans ]0, 1[.
Comme tous les ai sont dans ]0, 1[, et que F (ai ) = 0 pour tout i, on en déduit que les a i sont
égaux. Leur somme est égal à 1, donc ai = 1/n pour tout i.
Finalement, nous avons montré
   n  
1 1 1 1 (n − 1)n
max f (x) = f (a) = f ,..., = 1− = .
x∈Γ n n n n n 2n
i=1
3. INVERSION LOCALE, FONCTIONS IMPLICITES 341

Remarque. On aura pu répondre  à la question


 c) en appliquant l’inégalité arithmético-
géométrique aux deux produits i a i et i (1 − ai ).


Exercice 5. Soit n ∈ N∗ . On munit Mn (R) de la norme M 2 = ( i,j m2i,j )1/2 , où les
mi,j sont les coefficients de la matrice M . Montrer que le groupe des matrices orthogonales
directes SOn (R) est l’ensemble des éléments de SLn (R) = {M ∈ Mn (R) : det(M ) = 1}
de norme minimale.

Solution. On remarque déjà que M  22 = tr(t M M ). Il s’agit donc de minimiser f (M ) =


tr(t M M ) sous la contrainte g (M ) = 0, avec g (M ) = det(M ) − 1.
L’application f est une forme quadratique et g est une forme multilinéaire sur un e.v de
dimension finie, donc f et g sont continues. L’ensemble SLn (R) = g−1({0}) est un fermé de
Mn (R), donc le minimum de f sur SLn(R) est atteint en un point A de SL n(R) (en effet :
dans tout e.v de dimension finie E , la distance d de 0 à un fermé F de E est toujours atteinte,
puisqu’elle est atteinte sur le compact K des éléments x de F de norme x ≤ d +1, et x > d+1
en dehors de K). La différentielle du déterminant a déjà été calculée dans l’exercice 6 page 332,
et donne dgA (H) = tr(tCH) où C est la comatrice de A. Ainsi, dgA n’est pas nul, et le théorème
des extremums liés nous assure donc l’existence de λ ∈ R tel que les différentielles de f et g en
A vérifient dfA = λdgA .
Compte tenu de l’expression f (A + H) = tr(tAA)+ 2 tr( tAH) +tr(t HH) = f (A)+2 tr(tAH)+
H  2, on voit que df A(H) = 2 tr(tAH). Par ailleurs, on a vu que dgA (H) = tr(tAH) donc
2 tr(tAH ) = λ tr(t CH) pour toute matrice H, ce qui entraı̂ne 2A = λC (en remplaçant H par
les matrices Ei,j dont tous les coefficients sont nuls sauf le (i, j )-ième qui vaut 1, on obtient
l’égalité des (i, j )-ièmes coordonnées de 2A et λC ). Comme A est inversible et que det(A) = 1,
sa comatrice s’exprime aussi sous la forme C = tA−1 , donc on a 2 tAA = λI n . En composant par
le déterminant, on en déduit 2n = λn et comme tAA est une matrice positive, on a forcément
λ > 0 donc λ = 2. Ainsi, tAA = I n, c’est-à-dire A ∈ SOn (R). On a donc démontré que le
minimum de f sur SLn(R) était forcément atteint en un point A de SO n (R). Ce minimum
est égal à tr(tAA) = tr(In ) = n. Réciproquement, tous les éléments M de SOn (R) vérifient
f (M ) = n, d’où le résultat.

3. Inversion locale, fonctions implicites


3.1. Inversion locale
Pour une fonction f : R → R de classe C 1, on sait que si f (x) = 0 pour tout x, alors
f admet un inverse global f −1 de classe C 1 qui vérifie (f −1)[f (x)] = 1/f  (x) pour tout
x. Notre objectif est de généraliser ce résultat pour des fonctions de plusieurs variables,
ou plus généralement pour des fonctions définies sur un espace de Banach. La condition
“f (x) = 0” devient ici “dfx est inversible”, et la fonction f est alors localement inversible
autour de x (et non pas globalement comme dans le cas réel).

 Théorème 1 (Inversion locale). Soient E, F deux espaces de Banach, U un ouvert


de E et f : U → F une application de classe C1 . On suppose qu’il existe a ∈ U tel que dfa
soit un isomorphisme bicontinu de E sur F (i. e. dfa−1 existe, df a et df −1
a sont continus).
Alors il existe un voisinage ouvert V de a et un voisinage ouvert W de f (a) tels que
(i) la restriction f |V de f à V est une bijection de V sur W ;
(ii) l’application inverse g : W → V est continue ;
(iii) g est de classe C 1 et pour tout x ∈ V , dg f (x) = df −1
x .
342 5. FONCTIONS DE PLUSIEURS VARIABLES

Démonstration. Pour tout r > 0, on note Br la boule ouverte de centre 0 de rayon r, et B r la


boule fermée correspondante. La norme utilisée sur Lc(E, F ) est |||u||| = sup x=1 u(x).
Quitte à considérer la fonction x → dfa−1 [f (a + x) − f (a)], on peut supposer a = 0, f (a) = 0
et df0 = df a = IdE (et donc E = F ).
Comme f est de classe C 1, il existe r > 0 tel que B r ⊂ U et |||dfx − df0||| = |||df x − IdE ||| ≤ 1/2
pour tout x ∈ Br. Considérons maintenant x ∈ B r. On a df x = IdE −u avec |||u||| = |||idE −
df x||| ≤ 1/2 donc (voir la proposition 2 page 49) df x est un isomorphisme bicontinu qui vérifie
df −1 n
x = Id E +u + · · · + u + · · · , et de plus
+∞
 +∞
 1
|||df −1
x ||| ≤ |||u|||n ≤ = 2. (∗)
n=0 n=0
2n

(i). Nous voulons montrer que f a un inverse local. Plus précisément, nous allons prouver
que pour tout y ∈ Br/2 , il existe un unique x ∈ Br vérifiant f (x) = y.
Fixons donc y ∈ Br/2 et considérons la fonction h : Br → E x → y + x − f (x). Elle
est de classe C1 , et pour tout x ∈ Br , |||dh x||| = ||| IdE −df x||| ≤ 1/2, donc d’après l’inégalité des
accroissements finis,
2 1
∀(x, x ) ∈ B r , h(x) − h(x  ) ≤ x − x. (∗∗)
2
En particulier, pour tout x ∈ B r, x − f (x) = h(x) − h(0) ≤ x/2 donc
1 r r
∀x ∈ B r, h(x) ≤ y  + x − f (x) ≤ y + x < + = r.
2 2 2
Ainsi, h est une fonction de B r dans Br , donc dans B r . Comme de plus h vérifie (**), on
peut appliquer le théorème du point fixe qui entraı̂ne l’existence et l’unicité de x ∈ B r tel que
h(x) = x, et comme h est à valeurs dans Br , x ∈ B r . On a donc f (x) = y.
Résumons. Nous avons montré, pour tout y ∈ Br/2 , l’existence et l’unicité de x ∈ Br tel que
f (x) = y . En notant V = f −1(B r/2) ∩ B r (c’est un ouvert qui est un voisinage de 0 car f (0) = 0
et f est continue en 0), ceci s’interprète en disant que f|V : V → W = B r/2 est une bijection.
(ii). Notons g : W → V l’application inverse. Désignons par h l’application h : x → x − f (x)
(c’est l’application h précédente avec y = 0), de sorte que x = h(x) + f (x) pour tout x. Pour
montrer que g est continue, il suffit de remarquer que
1
∀x, x  ∈ B r , x − x   ≤ h(x) − h(x ) + f (x) − f (x) ≤ x − x  + f (x) − f (x )
2
donc x − x  ≤ 2f (x) − f (x). On en déduit
∀y, y ∈ W, g (y ) − g (y  ) ≤ 2f (g (y)) − f (g (y )) = 2y − y, (∗∗∗)
autrement dit g est lipschitzienne, donc continue.
(iii). Fixons x ∈ V et posons y = f (x) ∈ W . Soit w tel que y + w ∈ W . On note v =
g (y + w) − g(y ). On a v ≤ 2w  d’après (***), et
∆(w) = g (y + w) − g (y ) − dfx−1(w )
= (x + v ) − x − df −1 −1
x [f (x + v ) − f (x)] = −df x [f (x + v ) − f (x) − dfx (v)].

Or |||dfx−1 ||| ≤ 2 d’après (*), donc


∆(w ) ≤ 2f (x + v ) − f (x) − df x(v ) = 2v  ε(v )
avec limv→0 ε(v ) = 0, donc
∆(w ) ≤ 4w ε[g (y + w) − g (y )] = 4w  ε (w ).
La fonction g étant continue, on a limw→0 ε (w ) = 0, donc ∆(w ) = o(w). La fonction g est
donc différentiable en y et dgy = df−1
x .
Comme f est de classe C , que la fonction g est continue et que Inv : u → u −1 est continue
1

(Inv est même différentiable d’après l’exercice 5 page 331) l’application y → dgy = df −1
g(y )
est
1
continue comme composée d’applications continues, donc g est de classe C . 
3. INVERSION LOCALE, FONCTIONS IMPLICITES 343

Remarque 1. Ce théorème est bien intuitif : sur un voisinage de a, f (x) et f (a)+dfa (x − a)


sont “proches”, et dfa étant inversible, il est donc naturel que f soit inversible sur un
voisinage de a.
Corollaire 1. Soient E et F deux espaces de Banach, et U un ouvert de E . Soit
f : U → F une application de classe C1 telle que pour tout x ∈ U , dfx soit inversible et
bicontinu. Alors f est une application ouverte, c’est-à-dire que pour tout ouvert Ω ⊂ U ,
f (Ω) est un ouvert de F .
Démonstration. Soit Ω un ouvert de U et x ∈ Ω. D’après le théorème d’inversion locale, on peut
trouver un voisinage ouvert V x ⊂ Ω et un voisinage ouvert Wx de f (x) tels que f|V x soit une
bijection de Vx sur W x. En particulier, f (Vx ) = Wx . On en déduit que
 
  
f (Ω) = f Vx = f (Vx) = Wx
x∈Ω x∈Ω x∈Ω
est un ouvert de F . 

Corollaire 2 (Inversion globale). Soient E et F deux espaces de Banach, U un


ouvert de E , et f : U → F une fonction injective et de classe C 1. Les assertions suivantes
sont équivalentes :
(i) pour tout x ∈ U , df x est inversible et bicontinu ;
(ii) V = f (U ) est un ouvert de F et f −1 : V → U est de classe C 1 .
Démonstration. (i) =⇒ (ii). D’après le corollaire précédent, que V = f (U ) est ouvert. La fonc-
tion f est donc une bijection de l’ouvert U sur l’ouvert V . Soit x ∈ U et y = f (x) ∈ V . D’après le
théorème d’inversion locale, on peut trouver un voisinage ouvert A de x et un voisinage ouvert B
−1
de f (x) tels que f|A : A → B soit bijective et f|A soit de classe C 1 . Comme (f −1)|B = (f|A ) −1 ,
on en déduit que f −1 est de classe C1 sur un voisinage de f (x) (ici B), et ceci est vrai pour tout
x ∈ V , donc f −1 est de classe C 1 sur V .
(ii) =⇒ (i). Notons g = f −1 . On a g ◦ f = IdU , donc f et g étant de classe C 1 , dgf (x) ◦ df x =
IdE pour tout x ∈ U . De même, la relation f ◦ g = IdV entraı̂ne df x ◦ dg f (x) = Id F pour tout
x ∈ U . On en déduit que pour tout x ∈ U , dfx est inversible, d’inverse dg f (x) donc continu. 

Remarque 2. D’après le théorème de Banach (voir exercice 6 page 423), une application
linéaire continue bijective de E dans F a son inverse continu. Dans les résultats précédents,
on peut donc remplacer l’hypothèse “dfx est inversible et bicontinu” par “dfx est inversible”
(dfx est forcément continu par définition d’une différentielle).
Cas des fonctions définies sur Rn.
Définition 1. Soient U et V deux ouverts de Rn et une application f : U → V . On dit
que f est un C k-difféomorphisme (k ≥ 1), si f est bijective, de classe Ck et si f −1 est de
classe Ck.
Une autre conséquence du théorème d’inversion locale est la suivante.
Corollaire 3. Soit f : U ⊂ Rn → Rn ( où U est un ouvert de Rn ) de classe Ck (k ≥ 1).
S’il existe a ∈ U tel que dfa soit inversible (en termes équivalents, si le jacobien de f en
a n’est pas nul), alors il existe un voisinage ouvert V de a et un voisinage ouvert W de
f (a) tels que f |V soit un C k-difféomorphisme de V sur W . On a alors d(f|V−1 )f (x) = df−1
x
pour tout x ∈ V .
Démonstration. L’endomorphisme dfa est inversible, donc bicontinu (on est en dimension finie).
Le théorème d’inversion locale assure alors l’existence de deux voisinages ouverts V de a et W
−1
de f (a) tels que f|V soit un C1 difféomorphisme de V sur W , et donne d(f|V )f (x) = df x−1 pour
tout x ∈ V . Il nous reste à montrer que g = f −1 k
|V est de classe C .
344 5. FONCTIONS DE PLUSIEURS VARIABLES

En notant Jf (x) (resp. J x) la matrice jacobienne de g = f −1|V


en f (x) (resp. de f en x),
−1
l’égalité dgf (x) = dfx s’écrit aussi J 
= Jx = (det Jx ) Jx , où Jx désigne la comatrice de
f (x)
−1 −1 t 

Jx . Les dérivées partielles du premier ordre de g au point f (x) s’expriment donc comme des
fractions rationnelles en les dérivées partielles du premier ordre de f en x. Ainsi, les fonctions
∂g k−1 ∂g
∂xi ◦ f sont de classe C . On conclut par récurrence sur k que les ∂x i
sont de classe C k−1, donc
g est bien de classe C k. 
De la même manière que le corollaire 2, on peut montrer le corollaire suivant :

Corollaire 4 (Inversion globale). Soit f : U ⊂ R n → R n ( où U est un ouvert de


Rn ) une application injective et de classe C k (k ≥ 1). Alors les assertions suivantes sont
équivalentes :
(i) pour tout x ∈ U , df x est inversible ;
(ii) V = f (U ) est ouvert et f est un C k -difféomorphisme de U sur V .

Remarque 3. Il se peut que df x soit inversible pour tout x ∈ U et que f ne soit pas
injective (prendre par exemple f : R2 → R 2 (x, y ) → (e x cos y, ex sin y)).
3.2. Fonctions implicites
Nous commençons par énoncer le théorème des fonctions implicites pour une fonction
à variable dans un espace de Banach, avant de passer à la dimension finie. Nous aurons
besoin pour cela de la définition qui suit (qui généralise la notion de dérivée partielle).
Définition 2. Soient E1, . . . , E n , F des e.v.n et une application f : Ω ⊂ E = E 1 × · · · ×
En → F , où Ω est un ouvert de E . Soit a = (a1 , . . . , an ) ∈ Ω. Pour 1 ≤ i ≤ n, la fonction
fi : x → f (a1 , . . . , ai−1, x, a i+1 , . . . , an ) est définie sur un voisinage de ai dans E i. Si elle
est différentiable en ai , on dit que f admet une différentielle partielle d’indice i, et on
appelle différentielle partielle d’indice i la différentielle dfi,ai , notée ∂i f(a1 ,...,an ) .
Remarque 4. — Lorsque Ei = R pour tout i, E = Rn , les différentielles partielles
∂f
s’expriment en fonction des dérivées partielles par la relation ∂ ifa (h) = h ∂xi
(a).
— Il résulte de la définition de la différentielle que si f est différentiable en a =
(a1 , . . . , an ), alors ∂i f(a 1,...,an ) existe pour tout i et de plus
∀h ∈ E i, ∂i f(a 1 ,...,an ) (h) = dfa(0, . . . , 0, h, 0, . . . , 0),
ce qui entraı̂ne
n

∀(h 1, . . . , hn ) ∈ E1 × · · · × E n, df a(h 1, . . . , h n) = ∂i fa(h i).
i=1

Théorème 2 (Fonctions implicites). Soient E, F, G trois espaces de Banach, U un


ouvert de E et V un ouvert de F . Soit f : U × V → G (x, y ) → f (x, y ) une application
de classe C 1 . Supposons que pour (a, b) ∈ U × V , ∂2f (a,b) est un isomorphisme bicontinu
de F sur G. Alors il existe
— un voisinage ouvert U  de a (avec U  ⊂ U ),
— un voisinage ouvert W de f (a, b),
— un voisinage ouvert Ω de (a, b) (avec Ω ⊂ U × V ),
— une fonction ϕ : U  × W → V de classe C 1 ,
vérifiant
— pour tout x ∈ U  , pour tout z ∈ W , il existe une unique solution y de f (x, y ) = z
avec (x, y ) ∈ Ω, et on a y = ϕ(x, z ).
En particulier, f (x, ϕ(x, z )) = z pour tout (x, z ) ∈ U  × W .
3. INVERSION LOCALE, FONCTIONS IMPLICITES 345

Démonstration. Introduisons la fonction


F : U × V → E × G (x, y) → (x, f (x, y)).
D’après la règle de composition, F est de classe C 1 et
 
dF(a,b) (h 1, h2) = (h 1, df (a,b)(h1 , h2 )) = h1 , ∂1 f(a,b)(h 1) + ∂2 f(a,b) (h2 ) .
Ainsi, dF(a,b) est un isomorphisme
 bicontinu de E × F sur  E × G — son isomorphisme inverse
−1
est donné par (1 , 2 ) → 1 , (∂ 2 f(a,b)) (1 − ∂1 f(a,b) ( 2 )) . On peut donc appliquer le théorème
d’inversion locale, qui entraı̂ne l’existence d’un voisinage ouvert Ω de (a, b), d’un voisinage ouvert
Γ de (a, f (a, b)) tels que F soit un C1 -difféomorphisme de Ω sur Γ. Quitte à restreindre Γ, on
peut même écrire Γ = U  × W où U est un voisinage ouvert de a et W un voisinage ouvert de
f (a, b).
Comme F (x, y) = (x, f (x, y )), le difféomorphisme inverse F −1 : Γ = U  × W → Ω s’écrit
sous la forme F −1 (x, z) = (x, ϕ(x, z ) ) où ϕ est de classe C1 . On en déduit que pour tout (x, z) ∈
U × W = Γ, il existe un unique y tel que (x, y) ∈ Ω et f (x, y) = z, et de plus y = ϕ(x, z). D’où
le théorème. 

Remarque 5. — Le théorème que nous venons d’énoncer peut sembler obscur. Es-
sayons de l’éclaircir : deux points x et z étant donnés, on recherche y tel que
f (x, y ) = z . A condition de prendre y proche de b (exprimé par la condition
(x, y ) ∈ Ω), y existe et est unique, et on peut écrire y = ϕ(x, z ) où ϕ est de
classe C1 .
— On peut calculer la différentielle de ϕ en différentiant la relation f (x, ϕ(x, z )) = z.
Après calculs, on trouve
 
dϕ (x,z )(h 1, h 2 ) = ∂2 f(x,ϕ(x,z ))) −1 h1 − ∂ 1f (x,ϕ(x,z )) (h2 ) .
Remarque 6. On utilise souvent le théorème des fonctions implicites lorsque z est fixé,
menant ainsi au corollaire suivant :
Sous les hypothèses du théorème, si c = f (a, b), alors il existe un voisinage ouvert U 
de a, un voisinage ouvert Ω de (a, b) et une fonction ψ : U  → V de classe C1 vérifiant
— pour tout x ∈ U  , y = ψ (x) est l’unique solution de f (x, y) = c telle que (x, y ) ∈ Ω
(avec les notations du théorème, on a ψ (x) = ϕ(x, c)). On obtient la différentielle de ψ en
calculant la différentielle des deux membres de l’égalité f (x, ψ (x)) = c, ce qui donne
0 = df(x,ψ(x)) ◦ (Id E , dψx ) = ∂1 f(x,ψ (x)) + ∂2 f(x,ψ(x)) ◦ dψx ,
donc dψ x = −∂2 f(−1
x,ψ(x)) ◦ ∂1 f (x,ψ(x)) .

Cas des fonctions définies sur Rn.


Corollaire 5. Soient p, q ∈ N∗ , A un ouvert de Rp × Rq et une fonction
f = (f 1, . . . , f q) : A ⊂ R p × Rq → Rq (x, y) = (x1 , . . . , xq ; y1 , . . . , yq) → f (x, y )
k
de classe C (k ≥ 1). Soit (a, b) ∈ A. On appelle jacobien partiel de f en (a, b) le réel
 
D (f1, . . . , f q ) ∂fi
(a, b) = det (a, b) .
D (y1, . . . , y q ) ∂y j 1≤i≤q
1≤j ≤q

S’il est non nul en (a, b), alors il existe


— un voisinage ouvert U de a, un voisinage ouvert W de f (a, b), un voisinage ouvert
Ω de (a, b),
— une application ϕ : U  × W → Rq de classe C k,
vérifiant
— pour tout x ∈ U , pour tout z ∈ W , ϕ(x, z ) est l’unique solution y de f (x, y ) = z
telle que (x, y ) ∈ Ω.
En particulier, on a f (x, ϕ(x, z )) = z pour tout x ∈ U  et pour tout z ∈ W .
346 5. FONCTIONS DE PLUSIEURS VARIABLES

Démonstration. Quitte à restreindre A, on peut supposer A = U × V , où U est un  ouvert de


∂fi
Rp , V un ouvert de R q et (a, b) ∈ U × V . Ici, ∂2f (a,b) a pour matrice ∂y j
(a, b ) , et
1≤i,j ≤q
d’après les hypothèses, le déterminant de cette matrice est non nul, autrement dit, ∂2 f(a,b) est
un isomorphisme de Rq (bien sûr bicontinu puisque l’on est en dimension finie). On peut donc
appliquer le théorème précédent. Il reste à démontrer que ϕ est de classe Ck . . . mais on peut
reprendre mot pour mot la démonstration du théorème 2 en remplaçant C 1 par Ck , d’où le
résultat. 

Remarque 7. Là encore, on utilise souvent ce résultat lorsque z est fixé, ce qui mène au
corollaire suivant :
sous les même hypothèses, on peut trouver un voisinage ouvert U  de a, un voisinage
ouvert Ω de c = f (a, b), et une fonction ψ : U → R q de classe C k telle que pour tout
x ∈ U , y = ψ (x) est l’unique solution de f (x, y) = c vérifiant (x, y) ∈ Ω.
En termes intuitifs, ceci s’énonce en disant que q contraintes (de classe C k ) sur q varia-
bles mènent localement à une solution de classe C k en les autres variables. Ceci permet,
sous de bonnes hypothèses, de voir les sous-ensembles de Rn définis implicitement par
f (x1, . . . , x n) = 0 comme des nappes paramétrées.
On peut obtenir la matrice jacobienne (resp. les dérivées partielles) de ψ en écrivant
que la matrice jacobienne (resp. les dérivées partielles) de f (x, ψ (x)) est nulle (resp. sont
nulles).
On utilise beaucoup le corollaire énoncé dans la remarque précédente lorsque p = q = 1
ou lorsque p = 2, q = 1. Dans ces cas particuliers, ceci s’énonce comme suit.
Corollaire 6. Soit f : R2 → R ( où A est un ouvert de R2 ) une fonction de classe C k,
(k ≥ 1). Soit (a, b) ∈ A et supposons
∂f
f (a, b) = 0 et (a, b) = 0.
∂y
Alors il existe α, β > 0 tel que pour tout x ∈ ]a − α, a + α[, l’équation f (x, y ) = 0 possède
une et une seule solution y = ϕ(x) dans l’intervalle ]b − β, b + β [. La fonction ϕ est de
classe C k sur ]a − α, a + α[, et on a
∂f
 ∂x
(x, ϕ(x))
∀x ∈ ]a − α, a + α[ , ϕ (x) = − ∂f
.
∂y
(x, ϕ(x))
Démonstration. L’existence de α, β > 0 et de ϕ sont assurés par le corollaire énoncé dans la
remarque précédente. Pour calculer ϕ, on dérive la relation f (x, ϕ(x)) = 0 qui entraı̂ne
∂f ∂f
(x, ϕ(x)) + ϕ(x) (x, ϕ(x)) = 0,
∂x ∂y

d’où la valeur de ϕ (x). 
On démontrerait de même le corollaire suivant
Corollaire 7. Soit f : A ⊂ R2 × R → R ( où A est un ouvert de R3 ) une fonction de
classe C k (k ≥ 1). Soit (a, b, c) ∈ A et supposons
∂f
f (a, b, c) = 0 et (a, b, c) = 0.
∂z
Alors il existe α, β, γ > 0 tels que pour tout (x, y ) ∈ ]a − α, a + α[ × ]b − β, b + β [,
l’équation en z : f (x, y, z ) = 0 admette une unique solution z = ϕ(x, y ) dans ]c − γ, c + γ [.
La fonction ϕ est de classe Ck et
∂f ∂f
∂ϕ (x, y, ϕ(x, y)) ∂ϕ ∂y
(x, y, ϕ(x, y))
(x, y) = − ∂x
∂f
, (x, y) = − ∂f .
∂x ∂z
(x, y, ϕ(x, y)) ∂y ∂z
(x, y, ϕ(x, y))
3. INVERSION LOCALE, FONCTIONS IMPLICITES 347

Preuve du théorème des extremums liés. Munis du théorème des fonctions impli-
cites, nous sommes maintenant en mesure de démontrer le théorème des extremums liés
énoncé à la page 337. On utilise les mêmes notations que celles de l’énoncé.
Soit s = n − r . Identifions R n à R s × R r . On écrit les éléments de R n sous la forme (x, y) =
(x1 , . . . , xs ; y1, . . . , yr ). Écrivons a = (α, β), α ∈ Rs et β ∈ R r.
Commençons par faire la remarque suivante. On a nécessairement r ≤ n car les formes
linéaires dgi,a forment une famille libre et la dimension du dual de Rn est n. Par ailleurs, si
r = n, le théorème est évident car les dgi,a forment une base de (R n)∗ . On peut donc supposer
r ≤ n − 1, c’est-à-dire s ≥ 1.
Les formes linéaires (dgi,a )1≤i≤r forment une famille libre, la matrice
 ∂g1 
∂x1
(a) · · · ∂g
∂xs
1
(a) ∂g 1
∂y 1
(a) · · · ∂g 1
∂y r
(a)
 .. .. .. .. 
 . . . . 
∂gr ∂g r ∂g r ∂g r
∂x1 (a) · · · ∂xs (a) ∂y 1 (a) · · · ∂y r (a)
est donc de rang r . On peut donc en extraire une sous-matrice r × r inversible. Quitte à changer
le nom des variables, on peut supposer que
 
∂gi D (g1 , . . . , gr)
det (a) = (a) = 0.
∂y j 1≤i,j ≤r D (y1 , . . . , yr)
D’après le théorème des fonctions implicites (plus précisément la remarque 7), on peut donc
trouver un voisinage ouvert U  de α dans R s , un voisinage ouvert Ω de a = (α, β) dans Rn et
une fonction ϕ = (ϕ 1, . . . , ϕ r ) : U  → Rr de classe C 1 tels que (en notant g = (g 1, . . . , gr ))
(g (x, y) = 0 avec x ∈ U  et (x, y ) ∈ Ω) ⇐⇒ (y = ϕ(x)).
En d’autres termes, sur un voisinage de a, les éléments de Γ = {z | g (z) = 0} s’écrivent
(x, ϕ(x)). Posons h(x) = f (x, ϕ(x)). La fonction h admet donc un extremum local en x = α (car
(α, ϕ(α)) = a et (x, ϕ(x)) ∈ Γ), ce qui entraı̂ne
r
∂h ∂f ∂ϕj ∂f
∀i, 1 ≤ i ≤ s, 0= (α) = (a) + (α) (a). (∗).
∂xi ∂xi j=1
∂xi ∂yj

Par ailleurs, en écrivant les dérivées partielles par rapport aux xi de g(x, ϕ(x)) = 0, on tire
r
 ∂ϕj
∂g k ∂gk
∀k, 1 ≤ k ≤ r, ∀i, 1 ≤ i ≤ s, 0= (a) + (α) (a). (∗∗)
∂xi ∂xi ∂yj
j=1

Autrement dit, les s premiers vecteurs colonnes de la matrice


 ∂f ∂f ∂f ∂f

∂x1 (a) · · · ∂x s (a) ∂y1 (a) · · · ∂yr (a)
 ∂g1 
 ∂x1 (a) · · · ∂g ∂g1
∂x s (a) ∂y1 (a) · · ·
1 ∂g1
∂yr (a) 
M=  
.. .. .. .. 
 . . . . 
∂gr ∂gr ∂gr ∂gr
∂x1 ( a ) · · · ∂x s ( a ) ∂y1 (a) · · · ∂yr (a)
s’expriment, d’après (*) et (**), linéairement en fonction de ses r derniers vecteurs colonnes.
Donc rg M ≤ r . Or le rang des vecteurs lignes est égal au rang des vecteurs colonnes de M (car
rg t M = rg M ), donc les r + 1 vecteurs lignes de M forment une famille liée, ce qui entraı̂ne
l’existence de réels µ0 , . . . , µr non tous nuls tels que µ0 dfa + µ 1 dg1,a + · · · + µ r dgr,a = 0. Comme
la famille (dgi,a )1≤i≤r est libre, on a µ0 = 0, et en posant λi = −µi /µ0 pour 1 ≤ i ≤ r , on en

déduit dfa = ri=1 λi dg i,a. D’où le théorème.

3.3. Exercices
Exercice 1. Soit f : R 2{(0, 0)} → R 2 (x, y ) → (x2 − y 2, 2xy )
a) Montrer qu’en tout point de R2 {(0, 0)}, f est un C ∞ difféomorphisme local.
b) L’application f est-elle un C ∞ difféomorphisme global ?
348 5. FONCTIONS DE PLUSIEURS VARIABLES

Solution. a) La fonction f est clairement de classe C∞ . Soit (a, b) ∈ R2 {(0, 0)}. Le jacobien
de f en (a, b) est égal à
 
 2a 2b 
  2 2
 −2b 2a  = 4(a + b ) = 0.

Donc, d’après le théorème d’inversion locale (corollaire 3), il existe un ouvert U contenant (a, b),
un ouvert V contenant f (a, b), tel que la restriction de f à U soit un C ∞ -difféomorphisme de U
sur V . Autrement dit, f est un C ∞ -difféomorphisme local en (a, b).
b) On remarque que f (−x, −y) = f (x, y ), la fonction f n’est donc pas injective, et donc ce n’est
pas un C∞ difféomorphisme global.
En fait, en identifiant R2 et C, f s’écrit f (z) = z 2 pour tout z ∈ C∗ . On en conclut que les
ensembles Uα = {(x, y ) ∈ R 2 | x cos α + y sin α > 0} sont des ouverts maximaux sur lesquels
f est injective. Avec le corollaire 4, on en conclut que f un C ∞ -difféomorphisme de U α sur
f (Uα) = R2 {−r (cos 2α, sin 2α), r ≥ 0} (avec α ∈ R quelconque).

Exercice 2. Soit l’application f : R2 → R (x, y ) → sin y + xy 4 + x 2.


a) Montrer qu’il existe deux voisinages ouverts U et V de 0 dans R et une fonction
ϕ : U → R de classe C∞ tels que pour tout x ∈ U , ϕ(x) est l’unique solution y ∈ V de
l’équation f (x, y ) = 0.
b) Donner un développement limité à l’ordre 10 de ϕ au voisinage de 0.

∂f
Solution. a) La fonction f est de classe C ∞, elle vérifie f (0, 0) = 0 et ∂y (0, 0) = 1 = 0. On en
déduit le résultat en appliquant le théorème des fonctions implicites.
b) On a bien sûr ϕ(0) = 0 puisque f (0, 0) = 0. On pourrait calculer le développement limité de
ϕ en utilisant la relation exprimant ϕ, puis en la dérivant successivement. Cette technique est
cependant fastidieuse puisque l’on veut aller jusqu’à l’ordre 10 !
Nous utilisons une autre technique, classique (et à retenir). On a vu ϕ(0) = 0, donc
ϕ(x) = O (x) lorsque x → 0 (puisque ϕ est dérivable en 0). Maintenant, à partir de la rela-
tion f (x, ϕ(x)) = 0, l’idée est d’exprimer ϕ(x) en fonction de termes ne faisant intervenir ϕ(x)
qu’avec un ordre supérieur. Ici, on a

ϕ(x) = (ϕ(x) − sin ϕ(x)) − xϕ(x)4 − x2 = O (ϕ(x)3 ) − x 2 = −x 2 + O (x 3),

ce qui nous donne déjà le développement limité à l’ordre 2. Partant maintenant de l’information
ϕ(x) = −x2 + O (x 3), on itère le procédé, ce qui donne

ϕ(x)3
ϕ(x) = (ϕ(x) − sin ϕ(x)) − xϕ(x) 4 − x2 = + O (ϕ(x)4 ) − x2
6
(−x 2 + O (x 3 )) 3 x6
= + O (x 8 ) − x2 = −x2 − + O (x 7 ).
6 6
On obtient le résultat demandé en réinjectant encore une fois,

ϕ(x) = (ϕ(x) − sin ϕ(x)) − xϕ(x) 4 − x 2


ϕ(x)3 ϕ(x)5
= − + O (ϕ(x) 7) − xϕ(x)4 − x2
6 120
 3
x6 x4 5 x10  5  4
=− 1+ + O (x ) + 1 + O (x4 ) + O (x14 ) − x9 1 + O (x4 ) − x2
6 6 120
x6 3x 10
= −x2 − − x9 − + O (x 11).
6 40
3. INVERSION LOCALE, FONCTIONS IMPLICITES 349

Exercice 3. Soit f : Rn → Rn une fonction de classe C 1 telle que l’application ϕ =


f − IdRn est k-contractante (avec 0 < k < 1). Montrer que f est un C 1 -difféomorphisme
global de Rn sur Rn .
Solution. En vertu du corollaire 4 du théorème d’inversion locale, il suffit de montrer que
(i) pour tout x ∈ Rn, df x est inversible ;
(ii) f est une bijection de R n sur Rn .
(i). Soit x ∈ Rn . Pour tout h ∈ Rn, on a ϕ(x + h) − ϕ(x) ≤ k h, et on en déduit
facilement dϕ x(h) ≤ k h pour tout h ∈ R n. En d’autres termes, |||dϕx ||| ≤ k < 1, et donc
df x = Id +dϕx est inversible d’après la proposition 2 page 49. On peut retrouver ce résultat sans
invoquer cette dernière proposition en remarquant que si dfx (h) = 0, alors h + dϕ x (h) = 0 donc
dϕx(h) = h et comme dϕ x (h) ≤ k h avec k < 1 ceci entraı̂ne forcément h = 0. Ainsi dfx
est un endomorphisme injectif et comme on est en dimension finie, dfx est donc inversible.
(ii). Il nous reste à montrer que f est une bijection. Fixons y ∈ Rn . On a f (x) = y si et
seulement si x est point fixe de l’application ψ y : Rn → Rn x → x − f (x) + y = y − ϕ(x). Or
∀x, x ∈ Rn , ψy (x) − ψy(x ) = ϕ(x) − ϕ(x  ) ≤ k x − x ,
donc ψy est k-contractante. D’après le théorème du point fixe, on en déduit qu’il existe un unique
point x de Rn tel que ψy(x) = x, autrement dit il existe un unique point x ∈ R n tel que f (x) = y.
Ceci étant vrai pour tout y ∈ R n , on en déduit que f est une bijection de Rn sur Rn.

Exercice 4. Soit E un espace euclidien (de dimension finie). On note  ,  le produit


scalaire sur E et  .  la norme euclidienne associée. Soit f : E → E une application de
classe C 1 telle que pour tout x ∈ E , dfx est une isométrie de E (i. e. dfx(h) = h pour
tout h ∈ E ).
a) Pour tout a ∈ E , montrer l’existence d’un ouvert U a contenant a tel que f (x) −f (y ) =
x − y  pour tout (x, y) ∈ Ua2.
b) Montrer que
∀(x, y ) ∈ Ua2 , ∀(h, ) ∈ E2 , df x (h), dfy() = h, .
En déduire dfx = df y pour tout (x, y ) ∈ U a2.
c) Montrer que f est une isométrie affine de E sur E .

Solution. a) Normons L(E ) avec la norme |||u||| = supx=1 u(x) pour tout u ∈ L(E). On a
|||dfx ||| = 1 pour tout x ∈ E , donc d’après l’inégalité des accroissements finis,
∀(x, y ) ∈ E 2 , f (x) − f (y ) ≤ x − y . (∗)
Ceci étant, soit a ∈ E . Comme dfa est une isométrie, dfa est inversible. D’après le théorème
d’inversion locale, il existe un ouvert V a contenant a tel que f|Va soit un C 1 -difféomorphisme de
Va sur W a = f (V a). Notons g : W a → Va le difféomorphisme inverse. Pour tout y = f (x) ∈ Wa ,
dgy = (df x)−1 est une isométrie, donc |||dgy ||| = 1. Quitte à restreindre Va en un ouvert plus petit
Ua, on peut supposer que W a = f (U a) est convexe (prendre par exemple U a = g (B) où B est une
boule ouverte centrée en f (a) incluse dans f (Va ), de sorte que Wa = f (g (B)) = B). On peut donc
appliquer l’inégalité des accroissements finis à g sur W a , ce qui entraı̂ne g (x) − g (y ) ≤ x − y 
pour tout (x, y) ∈ Wa2. On en conclut
∀(x, y ) ∈ Ua2 , x − y  = g (f (x)) − g (f (y )) ≤ f (x) − f (y ).
Avec (*), on en déduit f (x) − f (y ) = x − y  pour tout (x, y ) ∈ Ua2 .
b) Le résultat de la question précédente s’écrit aussi
∀(x, y ) ∈ Ua2 , f (x) − f (y ), f(x) − f (y ) = x − y, x − y .
En différentiant cette égalité par rapport à x selon le vecteur h ∈ E , on obtient
dfx (h), f(x) − f (y ) + f (x) − f (y ), dfx (h) = h, x − y  + x − y, h,
350 5. FONCTIONS DE PLUSIEURS VARIABLES

et par symétrie du produit scalaire, ceci s’écrit aussi dfx(h), f(x) − f (y ) = h, x − y . Par
différentiation de cette dernière égalité par rapport à y selon un vecteur  ∈ E , on en déduit
−dfx (h), df y () = −h, . Autrement dit nous venons de montrer le premier résultat voulu.
Donnons nous maintenant x, y ∈ Ua et h ∈ E . On a
dfx (h) − dfy (h)2 = dfx (h) 2 − 2dfx (h), dfy (h) + dfy (h)2 = h 2 − 2h, h + h2 = 0,
donc dfx (h) = dfy (h). Ceci étant vrai pour tout h ∈ E , on a donc dfx = dfy .
c) D’après le résultat de la question précédente, l’ensemble Γ = {x ∈ E | df x = df 0} est un
ouvert de R n. De plus, f est de classe C 1 donc Γ est aussi un fermé de Rn. En résumé, Γ est un
ouvert et un fermé de E . Or E est connexe (car convexe), donc Γ = E . En posant u = df0 , on a
donc dfx = u pour tout x ∈ E . Ainsi la fonction x → f (x) − u(x) a sa différentielle nulle sur E
tout entier, donc c’est une fonction constante. Si on note α ∈ E sa valeur, on a f (x) = u(x) + α
pour tout x ∈ Rn . Comme u = df0 est une isométrie, on en déduit que f est une isométrie affine.

Exercice 5 (Fonctions strictement monotones). Soit E un espace euclidien (de


dimension finie). On note  ,  le produit scalaire sur E et  .  la norme euclidienne associée.
Une application f : E → E est dite strictement monotone s’il existe k > 0 tel que
∀(x, y ) ∈ E2 , f (x) − f (y ), x − y  ≥ k x − y2 . (∗)
a) Soit f : E → E une fonction de classe C1 . Montrer que f vérifie (*) si et seulement si
∀x ∈ E, ∀h ∈ E, dfx (h), h ≥ k h2 .
b) Si f : E → E est de classe C1 et si elle est strictement monotone, montrer que f est
un C 1-difféomorphisme de E sur E .
Solution. a) Condition nécessaire. Supposons que f vérifie (*). Alors pour tout x ∈ E , pour
tout h ∈ E et pour tout t ∈ R∗ ,
 
2 2 f (x + th) − f (x)
f (x + th) − f (x), th ≥ kt h ou encore , h ≥ k h2 ,
t
et en faisant tendre t vers 0 on en déduit dfx (h), h ≥ k h2 .
Condition suffisante. Supposons dfx (h), h ≥ k h2 pour tout x, h ∈ E. Soient x, h ∈ E. Consi-
dérons l’application ϕ : [0, 1] → R t → f (x + th), h. On a
∀t ∈ R, ϕ  (t) = dfx+th (h), h ≥ k h2
donc ϕ(1) − ϕ(0) ≥ k h2, c’est-à-dire f (x + h) − f (x), h ≥ k h2 , d’où (*).
b) En vertu du corollaire 4 du théorème d’inversion locale, il suffit de montrer
(i) pour tout x ∈ E , df x est inversible ;
(ii) f est une bijection de E sur E .
(i). Soit x ∈ E . On a dfx(h), h ≥ k h2 pour tout h ∈ E , donc df x(h) = 0 pour tout h = 0.
Ainsi, dfx est injective donc bijective (c’est un endomorphisme en dimension finie), d’où (i).
(ii). La relation (*) entraı̂ne l’injectivité de f . Montrons maintenant que f est surjective.
Comme E est connexe, on aura prouvé f (E ) = E si on montre que f (E ) est ouvert et fermé
dans E . On sait déjà que f (E ) est ouvert d’après (i) (voir le corollaire 1). Pour montrer que
f (E ) est fermé, nous allons montrer que f (E ) est complet. Commençons par remarquer que
d’après (*) et d’après l’inégalité de Schwarz,
1
∀x, y ∈ E, k x − y  2 ≤ x − y  f (x) − f (y ) donc x − y  ≤ f (x) − f (y). (∗∗)
k
Considérons maintenant une suite de Cauchy (f (x n)) de f (E ). D’après (**), on a x p − xq  ≤
f (xp) − f (x q )/k pour tout p, q ∈ N, donc (x n ) est une suite de Cauchy dans E, donc converge.
Si on note x sa limite, on a alors f (x) = limn→+∞ f (x n). Ainsi, la suite (f (x n )) converge vers
f (x) ∈ f (E ), donc f (E ) est complet, d’où le résultat.
4. INTÉGRALES MULTIPLES, INTÉGRALES CURVILIGNES 351

Exercice 6. Soit n ∈ N ∗. On note Un = {(λ 1 , . . . , λ n) ∈ Rn | λ1 < · · · < λ n}, et on note


Rn−1 [X] l’e.v des polynômes de R[X ] de degré ≤ n − 1. Montrer que l’application
n
ϕ : U n → Rn−1 [X] (λ1, . . . , λn ) → (X − λ i ) − Xn
i=1

est un C -difféomorphisme global de l’ouvert U n sur ϕ(Un ).
Solution. Il est clair que ϕ est injective, c’est donc une bijection de Un sur ϕ(Un ).
En vertu du corollaire 4 du théorème d’inversion locale, il suffit maintenant de prouver que
la différentielle de ϕ est inversible en tout point λ de Un , ce qui équivaut à montrer que les
∂ϕ
vecteurs ∂λ (λ) = dϕλ (ei ) sont linéairement indépendants.
i
∂ϕ 
Soit λ = (λ1 , . . . , λn ) ∈ Un . Pour tout i ∈ {1, . . . , n}, ∂λ (λ) = − j =i (X − λj ). Si maintenant
 ∂ϕ
i
on suppose i µi ∂λ i
(λ) = 0, alors
n
 
µi (X − λ j ) = 0. (∗)
i=1 j =i

Soit k ∈ {1, . . . , n}. En donnant à l’indéterminée X la valeur λk dans (*), on obtient µk j =k(λk −
λj) = 0, donc µ k = 0 car les λi sont distincts. Ceci étant vrai pour tout k, on en déduit que les
∂ϕ
vecteurs ∂λ i
(λ) sont linéairement indépendants, et ceci pour tout λ ∈ U n, d’où le résultat.

4. Intégrales multiples, intégrales curvilignes


Alors que pour l’intégrale simple nous nous étions limités aux intégrales de fonc-
tions continues par morceaux (chapitre 3), cette restriction est un handicap majeur pour
les intégrales multiples, notamment à cause des domaines d’intégration qui ne sont pas
forcément des pavés. Ce problème peut être (inélégamment) contourné pour les intégrales
doubles, via le concept de parties simples de R2 , mais ce dernier se prête mal à une
généralisation raisonnable pour les intégrales de fonctions de n variables (avec n ≥ 3).
Ne souhaitant pas nous restreindre aux intégrales doubles, nous avons pris le parti de
présenter la théorie de l’intégrale multiple de Riemann (le cadre de l’intégrale de Lebesgue
allant clairement au delà du programme des classes préparatoires), en particulier pour
définir les domaines mesurables (au sens de Riemann), puis nous nous limiterons aux
intégrales de fonctions continues sur les domaines mesurables compacts. Le cas particulier
de domaines simples dans R 2 est discuté au fil des remarques de cette section. Nous
ne donnerons aucune preuve des résultats présentés, cependant nous nous attacherons à
donner précisément la définition de l’intégrale multiple de Riemann.
4.1. Définition de l’intégrale multiple de Riemann
Définition 1. On appelle pavé de Rn tout ensemble du type P = I1 × · · · × I n où pour
tout k , Ik est un intervalle borné de R. On appelle mesure n-dimensionnelle de P le réel
mes(P ) = λ1 · · · λ n où pour tout k , λ k est la longueur de Ik (si Ik = [a, b], λk = b − a).
Pour tout sous-ensemble A de Rn, on notera χ A : Rn → R la fonction caractéristique
de A, définie par χA (x) = 1 si x ∈ A, = 0 sinon.
Définition 2. Une fonction f : R n → R est dite en escalier si on peut l’écrire comme
combinaison linéaire de fonctions caractéristiques
 de pavés de Rn . Une telle famille de
dite bien adaptée à f . Si f = i λi χP i où Pi est un pavé pour tout i, le réel
pavés est
I (f ) = i λi mes(Pi ) ne dépend pas de la famille de pavés (Pi) bien adaptée à ϕ et on
l’appelle intégrale de f .
352 5. FONCTIONS DE PLUSIEURS VARIABLES

Définition 3. Soit f : Rn → R une fonction bornée, à support compact (i. e. f est nulle
en dehors d’un compact). Soient les deux ensembles
E+ = {I (v ), f ≤ v et v est en escalier} et E − = {I (u), u ≤ f et u est en escalier}.
Les ensembles E + et E − sont non vides car f est bornée, donc I + = inf E + et I − = sup E −
existent.
 On a toujours I− ≤ I + . Si I − = I +, f est dite Riemann-intégrable, et le réel

R n f (x) dx = I = I + est appelé intégrale de f sur Rn.
Remarque 1. Lorsque n = 1 et que f est continue par morceaux, cette dernière définition
de l’intégrale est cohérente avec celle d’une fonction à variable réelle. Pour tout n ∈ N∗ ,
elle est également cohérente avec l’intégrale
  d’une fonction en escalier définie plus haut.
L’intégrale de f est parfois notée · · · Rn f (x1 , . . . , xn ) dx1 · · · dx n (avec n signes d’inté-
gration).
Ensembles mesurables, intégrale sur un ensemble mesurable.
Définition 4. Une partie bornée A de Rn est dite
 mesurable si χA est Riemann-intégrable.
On appelle alors mesure de A le réel mes(A) = Rn χA(x) dx.
Remarque 2. Si n = 2, A est dite quarrable, et mes(A) est appelé l’aire de A. Si n = 3, A
est dite cubable, et mes(A) est appelé le volume de A.
Exemple 1. — Tout pavé de R n est mesurable.
— Une partie bornée A de R2 est dite élémentaire si elle admet les deux définitions
suivantes
A = {(x, y ) ∈ R2 : a ≤ x ≤ b, ϕ1(x) ≤ y ≤ ϕ2 (x)}
A = {(x, y ) ∈ R2 : c ≤ y ≤ d, ψ 1(x) ≤ x ≤ ψ2 (x)}
où [ a, b] et [c, d] sont deux segments de R et ϕ1 , ϕ 2, ψ 1 et ψ 2 sont des fonctions
continues, vérifiant ϕ1 < ϕ2 sur ]a, b[ et ψ 1 < ψ2 sur ]c, d[. Une partie de R2 est
dite simple si elle est la réunion finie de parties élémentaires dont les intérieurs
sont deux à deux disjoints. Toute partie élémentaire, toute partie simple de R2 , est
mesurable.
— Une partie A de Rn est dite négligeable (au sens de Riemann) si A est mesurable
et si mes(A) = 0. On peut montrer qu’une partie bornée A de Rn est mesurable
si et seulement si sa frontière Fr(A) est négligeable. On peut en déduire le résultat
suivant :
Si ϕ est un C 1 -difféomorphisme de U ⊂ Rn sur ϕ(U ) ( où U est un ouvert de
R n ), si A est mesurable et si A ⊂ U , alors ϕ(A) est mesurable.
Par exemple, tout disque du plan, toute sphère de Rn est mesurable.
Nous nous limiterons désormais aux cas des fonctions continues sur les ensembles
mesurables compacts.
Définition 5. Soit A ⊂ Rn un ensemble mesurable compact et f : A → R une applica-
tion continue. Le prolongement f˜ de f à Rn (obtenu en posant f˜(x) = f (x) six ∈ A, = 0
sinon) est Riemann-intégrable, et on appelle intégrale de f le réel A f (x) dx = Rn f˜(x) dx.
Propriétés élémentaires. Les intégrales multiples possèdent les mêmes propriétés que
les intégrales simples. Nous désignons par A un ensemble mesurable compact de R n.
(i) Linéarité. L’ensemble C(A, R) des fonctions continues de A dans R est un R-e.v et
l’application C (A, R) → R f → A f (x) dx est linéaire.
  
(ii) Positivité. Si f ≥ 0, alors A f (x) dx ≥ 0 ; si f ≤ g , A f (x) dx ≤ A g (x) dx.
 
(iii) On a | A f (x) dx| ≤ A |f (x)| dx.
4. INTÉGRALES MULTIPLES, INTÉGRALES CURVILIGNES 353

(iv) Relation de Chasles. Si A et B sont des compacts mesurables, A ∪ B et A ∩ B sont


des compacts mesurables. Si de
 plus mes(A ∩ B ) = 0, toute fonction f continue sur
A ∪ B vérifie A∪B f (x) dx = A f (x) dx + B f (x) dx.
Remarque 3. On peut également définir l’intégrale multiple d’une fonction continue f à
valeurs
 dans un e.v E de dimension finie. Si (e1, . . . , e p) est une base de E , on écrit
 f =
fi ei où l es fi sont à valeurs réelles, et l’intégrale multiple de f est A f = i ( A fi )ei .
Cette définition est indépendante  de la  base deE choisie. En particulier, si f = f 1 + if2
est à valeurs complexes, alors A f = A f1 + i( A f2 ).
4.2. Théorèmes de Fubini
Nous n’énoncerons ce théorème que dans certains cas particuliers qui nous suffiront.
Théorème 1. Soient P et Q deux pavés compacts de Rp et Rq respectivement. Soit
f : P × Q → R une fonction continue. Alors
      
f (x, y ) dx dy = f (x, y ) dy dx = f (x, y ) dx dy.
P ×Q P Q Q P

Remarque 4. Par applications successives de ce théorème, on obtient facilement, si P =


[a 1, b1] × · · · × [a n, b n],
  b 1  b2   bn   
f (x1, . . . , x n) dx 1 · · · dx n = ··· f (x1 , . . . , xn) dxn · · · dx 2 dx1 ,
P a1 a2 an

(égalité qui exprime les intégrales multiples en fonctions d’intégrales simples, ce qui permet
de les calculer dans la pratique). Ce dernier terme est parfois noté
 b1  b2  bn
dx1 dx2 · · · f (x1 , . . . , xn ) dxn .
a1 a2 an

L’ordre d’intégration (sur les xi) est indifférent, et dans la pratique, on s’arrange pour
intégrer dans un ordre qui facile les intégrations.
Citons un corollaire important de ce théorème :
Corollaire 1. Soient P et Q deux pavés compacts de Rp et Rq , et soient g : P → R et
h : Q → R deux fonctions continues. Alors
    
g (x)h(y ) dx dy = g (x) dx · h(y ) dy .
P ×Q P Q

Le théorème 1 ne permet de calculer des intégrales multiples que sur des pavés. Pour
calculer des intégrales multiples sur d’autres ensembles, on utilise les deux théorèmes qui
suivent.
Sommation par piles, sommation par tranches.

Théorème 2 (Sommation par piles). Soit B un ensemble mesurable compact de Rn−1 ,


et deux applications continues ϕ1 , ϕ2 : B → R telles que ϕ1 ≤ ϕ 2. Alors l’ensemble
A = {(x, x n) ∈ B × R | ϕ1 (x) ≤ xn ≤ ϕ 2 (x)}
n
est mesurable sur R et pour toute fonction continue f : A → R, on a
   ϕ 2 (x) 
f (x, xn ) dx dxn = f (x, xn ) dxn dx.
A B ϕ1 (x)
354 5. FONCTIONS DE PLUSIEURS VARIABLES

xn xn

x n = ϕ 2(x)
b

xn

x n = ϕ 1(x) a

x B x A(xn )

Figure 1. à gauche, illustration d’une sommation par piles ; à droite, d’une


sommation par tranches.

Théorème 3 (Sommation par tranches). Soit A un ensemble mesurable compact de


Rn tel que pour tout (x, x n ) ∈ Rn−1 × R vérifiant (x, xn) ∈ A, on ait a ≤ x n ≤ b. Si pour
tout xn ∈ [a, b], l’ensemble A(xn ) = {x ∈ R n−1 | (x, xn ) ∈ A} est mesurable dans Rn−1 ,
alors pour toute fonction continue f : A → R, on a
  b  
f (x, xn ) dx dxn = f (x, x n) dx dx n.
A a A(xn )

Remarque 5. — Le résultat de ces deux théorèmes reste évidemment vrai si l’on rem-
place la variable xn par l’une des autres variables xi .
— Ainsi, on dispose de deux méthodes pour réduire le calcul d’intégrales multiples.
Selon les cas, l’une peut s’avérer plus pratique que l’autre. Si n = 2, ces résultats
sont équivalents, et peuvent s’exprimer comme suit.
Soit K un compact de R2 de la forme
K = {(x, y ) ∈ [a, b] × R | ϕ1 (x) ≤ y ≤ ϕ2 (x)},
où ϕ1 et ϕ2 sont des applications continues de [a, b] dans R. Alors K est
mesurable, et pour toute fonction continue f : K → R, on a
  b  ϕ2 (x) 
f (x, y ) dx dy = f (x, y ) dy dx.
K a ϕ 1(x)

4.3. Théorème du changement de variable


Rappel. Soit U un ouvert de Rn et ϕ = (ϕ 1, . . . , ϕ n) : U → R n (x1 , . . . , xn ) →
ϕ(x1, . . . , x n) une fonction de classe C 1 . Pour tout x ∈ U , le jacobien de ϕ en x est
 
∂ϕi D (ϕ1 , . . . , ϕn)
J (ϕ)(x) = det (x) , également noté .
∂xj 1≤i,j ≤n
D (x 1 , . . . , xn)

 Théorème 4 (du changement de variables). Soient U un ensemble mesurable com-


◦ ◦
pact de Rn, et ϕ un C 1 -difféomorphisme de U sur ϕ(U ), tel que ϕ et son jacobien J (ϕ) se
4. INTÉGRALES MULTIPLES, INTÉGRALES CURVILIGNES 355

prolongent continûment sur U . Alors V = ϕ(U ) est un compact mesurable et pour toute
fonction continue f : V → R, on a
 
 
f (v ) dv = f ϕ(u) |J (ϕ)(u)| du.
V U

Remarque 6. Le résultat du théorème s’écrit aussi sous la forme


     
 D (v 1, . . . , vn ) 
· · · f (v 1, . . . , vn ) dv1 · · · dvn = · · · F (u 1, . . . , u n)   du 1 · · · dun,
V U D (u1, . . . , un ) 
où F (u1, . . . , u n) désigne f [ϕ(u 1 , . . . , un ) ] et oùD(v1 ,...,vn)
D(u1 ,...,u n)
désigne D(ϕ1 ,...,ϕn )
D(u1 ,...,un )
.
Voyons maintenant les applications les plus courantes du théorème du changement de
variable.
Passage en coordonnées polaires dans le plan. On se place dans le plan R2, où
on désigne par (r, θ) ∈ R+ × [0, 2π ] les coordonnées polaires et (x, y) = (r cos θ, r sin θ ) les
coordonnées cartésiennes. Si un domaine compact quarrable de R 2 est représenté par D
en coordonnées cartésiennes et par ∆ en coordonnées polaires, toute fonction continue f
sur D vérifie  
f (x, y ) dx dy = f (r cos θ, r sin θ ) r dr dθ.
D ∆
En effet, il suffit d’appliquer le théorème du changement de variable et de remarquer que
   
 ∂x ∂x   
D (x, y )  ∂r ∂θ   cos θ −r sin θ 
= =  = r.
D(r, θ)  ∂y ∂y   sin θ r cos θ 
∂r ∂θ

Exemple 2. Comme conséquence de ce résultat, nous allons donner une méthode classique
+∞ 2
pour calculer l’intégrale de Gauss −∞ e−x dx. Pour tout a > 0, on note

2 2 2 2 2 2 2
Da = {(x, y ) ∈ R | x + y ≤ a }, C a = [−a, a] et I a = e−(x +y ) dx dy.
Da
En passant en coordonnées polaires puis en appliquant le corollaire du théorème de Fubini,
on a
    
−r2 −r2 2
∀a > 0, Ia = e r dr dθ = e r dr dθ = π(1 − e −a ).
[0,a]×[0,2π ] [0,a] [0,2π]
 −(x2 +y2 )
En notant Ja = Ca
e dx dy, on a (toujours d’après le théorème de Fubini),
     a 2
−x2 −y 2 −x2
Ja = e dx e dy = e dx .
[−a,a] [−a,a] −a

Or Da ⊂ C a ⊂ D√ √
2 a , et la fonction intégrée étant positive, on en déduit Ia ≤ Ja ≤ J 2 a ,
ce qui s’écrit encore
 a 2
−a2 −x2 2
∀a > 0, π(1 − e ) ≤ e dx ≤ π(1 − e−2a ).
−a
 +∞ 2 √
En faisant tendre a vers +∞, on en déduit −∞
e −x dx = π.
Passage en coordonnées cylindriques dans l’espace. Nous nous plaçons dans l’es-
pace R3, dans lequel on écrit (x, y, z) = (r cos θ, r sin θ, z) avec r ≥ 0 et θ ∈ [0, 2π ]. Avec
les notations précédentes, on a
 
f (x, y, z ) dx dy dz = f (r cos θ, r sin θ, z) r dr dθ dz.
D ∆
356 5. FONCTIONS DE PLUSIEURS VARIABLES

Passage en coordonnées sphériques dans l’espace. On écrit



 x = r cos ϕ cos θ  π π
y = r cos ϕ sin θ , r ≥ 0, θ ∈ [0, 2π ], ϕ ∈ − , .
 z = r sin ϕ 2 2

Après calcul du jacobien, on obtient


 
f (x, y, z ) dx dy dz = f (r cos ϕ cos θ, r cos ϕ sin θ, r sin ϕ) r2 cos ϕ dr dϕ dθ.
D ∆

4.4. Fonctions intégrables


Comme pour les intégrales simples, on peut définir les fonctions de plusieurs variables
intégrables. Nous nous limiterons au cas de fonctions définies sur un produit d’intervalles
de Rn . Dans toute cette sous-partie, I 1 , . . . , In désignent des intervalles quelconques de R
(non réduits à un singleton) et A la partie de Rn définie par A = I1 × · · · × In
Définition 6. Soit f : A = I1 × · · · × In → R une fonction continue positive. On
 dit que
f est intégrable s’il existe M > 0 tel que pour tout pavé compact P ⊂ A, on a P f ≤ M .
On appelle alors intégrale de f le nombre réel défini par
 
f = sup f.
A P ⊂A P

Définition 7. Une fonction continue f : A → R est dite intégrable si |f | est intégrable.


Dans ce cas, pour toute suite
 croissante de pavés compacts (Pk ) inclus dans A telle que
∪kP k = A, la limite de ( Pk f ) existe et ne dépend pas du choix de la suite (Pk ). Cette

limite s’appelle intégrale de f et est noté A f .
Remarque 7. Une moyen  équivalent
 de
 définir l’intégrale d’une fonction continue réelle
intégrable est d’écrire A f = A f + − A f − où f + = sup{f, 0} et f − = sup{−f, 0}.
– L’intégrale multiple des fonctions intégrables sur A satisfait les propriétés élémentaires
de l’intégrale. L’ensemble L1 (A, R) des fonctions continues intégrables sur A est un e.v,
1 1
et
 l’application
 L (A, R) → R f →
 A
f est
 linéaire.
 Si f, g ∈ L (A, R) et si f ≤ g , alors
1
A
f ≤ A g . Enfin, si f ∈ L (A, R) alors | A f | ≤ A |f |. Par ailleurs, si g est intégrable
et si |f | ≤ g, alors f est intégrable.
Exprimons, dans le cas particulier des intégrales doubles, la généralisation du théorème
de Fubini des fonctions intégrables.
Théorème 5. Soit I et I  deux intervalles de R et f : I × I  → R une fonction continue

sur
 I × I . Si pour tout x ∈ I l’application f (x, .) est intégrable et si l’applicationg : x →
f (x, .) est continue par morceaux et intégrable, alors f est intégrable et on a I ×I  f =
I  
I
g. De même, si pour tout y ∈ I  la fonction f (., y) est intégrable etsi h : y → I f (.y)
est continue par morceaux
 et intégrable,
  alors f est intégrable et I ×I f = I  h. En
particulier on a I ( I f (., y)) = I  ( I f (x, .)).
4.5. Intégrales curvilignes
Rappels sur les arcs paramétrés.
Définition 8. On appelle arc paramétré de R n de classe C k tout couple (I, f ) où I est
un intervalle de R et f : I → R n une application de classe Ck . L’ensemble f (I ) ⊂ Rn est
appelé support de l’arc. Un arc paramétré continu est aussi appelé chemin.
Définition 9. On dit que deux arcs paramétrés (I, f) et (J, g ) de classe C k sont C k-équi-
valents s’il existe un C k-difféomorphisme θ de J sur I tel que g = f ◦ θ.
4. INTÉGRALES MULTIPLES, INTÉGRALES CURVILIGNES 357

On définit ainsi une relation d’équivalence sur les arcs paramétrés. Chaque classe est
appelée arc géométrique de classe Ck , tout représentant de classe un paramétrage admis-
sible de l’arc géométrique. Deux paramétrages admissibles d’un même arc géométriques
ont même support, on parle donc de support d’un arc géométrique.
Remarque 8. L’application θ est soit strictement croissante, soit strictement décroissante.
Définition 10. Deux paramétrages admissibles (I, f) et (J, g ) d’un arc géométrique de
classe C k sont dit de même sens s’il existe un C k-difféomorphisme croissant de J sur I tel
que g = f ◦ θ .
On définit ainsi une relation d’équivalence sur un arc géométrique (dont il y a au plus
deux classes d’après la remarque précédente) ; les classes sont appelées arcs géométriques
orientés de Rn de classe C k .
Formes différentielles de degré 1.
Définition 11. Soit Ω un ouvert de Rn . On appelle forme différentielle de degré 1 sur
Ω toute application α de Ω sur le dual (Rn ) ∗ de Rn. 
On peut écrire la forme différentielle α sous la forme α (x) = ni=1 ai(x) dx i pour x ∈ Ω
(où les ai sont à valeurs réelles et (dx i ) est la base duale de la base canonique de Rn ).
S’il existe une fonction ϕ : Ω → R de classe C 1 telle que α = dϕ, la forme différentielle
α de degré 1 est dite exacte.
Si α est de classe C 1 (i. e. si les ai sont C 1), α est dite fermée si pour tout (i, j),
∂a i
∂xj
= ∂a j
∂xi
.
Remarque 9. Si une forme différentielle de classe C 1 est exacte, elle est fermée (conséquence
du théorème de Schwarz). La réciproque est vraie lorsque Ω est un ouvert étoilé mais est
fausse dans le cas général (voir un contre exemple à l’exemple 3).
Définition
 d’une intégrale curviligne. On se donne une forme différentielle α =
i ai (x ) dx i de degré 1, définie et continue sur un ouvert Ω de R n .
Définition 12. Soit γ = ([a, b], f ) (avec f = (f1 , . . . , fn)) un arc paramétré de R n de
classe C 1 dont le support est contenu dans Ω. On appelle intégrale curviligne de α le long
de γ le réel
  b  b 
n

α= α[f (t)] f (t) dt = ai(f (t)) f i(t) dt.
γ a a i=1

Remarque 10. Si γ = ([a, b], f ) est un chemin continu de classe C 1 par morceaux, c’est-à-
dire s’il existe une subdivision a = t0 < t1 < · · · < t p = b de [a, b] telle que f|[ti−1 ,t i] soit de
classe C1 pour tout i ∈ {1, . . . , p}, on définit de même l’intégrale curviligne de α le long
de γ par l’expression
 p−1 
α= α.
γ i=0 ([ti ,t i+1],f )

Définition 13. Soit Γ un arc géométrique de classe C 1, orienté, à support dans Ω. Les
+

intégrales curvilignes de α le long des paramétrages admissibles de Γ+ sont identiques,


 et
leur valeur commune est appelé intégrale curviligne de α le long de Γ+ , et notée Γ+ α.
Proposition 1. Soit γ = ([a, b], f ) un arc paramétré de Rn de classe C 1 à support contenu
dans Ω. Si α est une forme exacte, c’est-à-dire α = dϕ avec ϕ : Ω → R de classe C1,
alors 
α = ϕ[f (b)] − ϕ[f (a)].
γ

En particulier, si γ est un lacet ( i. e. si f (a) = f (b)), γ α = 0.
358 5. FONCTIONS DE PLUSIEURS VARIABLES

Exemple 3. Soit la forme différentielle de degré 1


x dy − y dx
α : R2 {(0, 0)} → (R2 )∗ (x, y ) → .
x2 + y 2
On vérifie facilement que l’on a affaire à une forme fermée. Elle n’est cependant pas exacte,
car si γ est le lacet ([0, 2π ], f ) où f (θ ) = (cos θ, sin θ ), on trouve γ α = 2π = 0, et on
conclut avec la proposition précédente.
Théorème de Green-Riemann. Ce théorème permet de calculer certaines intégrales
sur un compact K en fonction d’une intégrale curviligne le long de sa frontière ∂K.
Intuitivement, le compact K doit avoir sa frontière qui est une courbe orientable et C1
par morceaux. Nous rendons cette définition plus rigoureuse avec la notion de compact à
bord.
Définition 14. Un compact K ⊂ R2 est dit compact à bord si
(i) il existe une famille finie (γi) = (Ii , fi )1≤i≤p de chemins fermés, simples (i. e. injectifs)
et C 1 par morceaux, dont les supports C i sont disjoints, et telle que la frontière ∂K
de K soit la réunion des Ci ;
(ii) en tout point a = fi (t) régulier d’un chemin γ i , il existe un pavé P centré en a tel
que

— P ∂K a deux composantes connexes, l’une notée P1 constituée de points de K,
l’autre de points de R2 K ;
— pour tout b ∈ P tel que le vecteur ab  fasse un angle +π/2 avec f  (t), on a b ∈ P1 .
i
Pour
p  toute forme différentielle α de degré 1 définie sur un ouvert contenant K , la valeur
i=1 γi α est alors indépendante des paramétrages admissibles des arcs géométriques

orientés γi. On la note ∂K + α.

C3
P1 → b
C1 f 1(t)
K
a
C2 P

Figure 2. Un compact à bord K

Remarque 11. Dans la pratique, on se contente en général de la notion intuitive sui-


vante : la frontière du compact à bord K est réunion finie, disjointe de supports d’arcs
géométriques orientés, fermés, simples, C 1 par morceaux, tels qu’en parcourant cette
frontière dans le sens de l’orientation, on ait K constamment à sa gauche.
Théorème 6 (Green-Riemann). Soit K ⊂ R 2 un compact à bord et α = P dx + Q dy
une forme différentielle de degré 1, de classe C1 sur un ouvert contenant K . Alors K est
mesurable et
   
∂Q ∂P
(P dx + Q dy) = (x, y ) − (x, y ) dx dy.
∂K + K ∂x ∂y

Application. Si K est un compact à bord, son aire A  = K dx dy peut  s’expri-
mer,
 d’après le théorème de Green-Riemann, comme A = ∂K + x dy = − ∂K + y dx =
1
2 ∂K +
(x dy − y dx). En désignant
 par (r, θ) les coordonnées polaires, on a facilement
2 1 2
x dy − y dx = r dθ donc A = 2 ∂K + r dθ.
4. INTÉGRALES MULTIPLES, INTÉGRALES CURVILIGNES 359

4.6. Exercices

Exercice 1. Calculer les intégrales doubles D
f (x, y ) dx dy dans les cas suivants :
2
a) f (x, y ) = xy et D = {(x, y ) ∈ R | x ≥ 0, y ≥ 0, x + y ≤ 1} ;
b) f (x, y ) = x2 et D = {(x, y ) ∈ R2 | x2 ≤ y ≤ x} ;
c) f (x, y ) = (x2 − y 2)exy et D = {(x, y ) ∈ R 2 | x2 + y 2 ≤ 1, x + y ≥ 1, x ≥ y } ;
 
d) f (x, y ) = x cos x + y et D = {(x, y ) ∈ R2 | x ≥ y ≥ 0, x 2 + y2 ≤ π }.
2 2

Solution.
a) On peut écrire D = {(x, y) ∈ R2 | 0 ≤ x ≤ 1, 0 ≤ y ≤ 1 − x}, et en 1 D
appliquant le théorème de Fubini, on a 

  1
 1−x
  1 0 1
(1 − x) 2 1
xy dx dy = xy dy dx = x dx = .
D 0 0 0 2 24

b) On procède de la même manière. Si x2 ≤ x, on a forcément 0 ≤ x ≤ 1, 1


donc D = {(x, y) ∈ R2 | 0 ≤ x ≤ 1, x2 ≤ y ≤ x}. Le théorème de Fubini ←−D
donne ensuite
0 1
  1  x  1
1
x2 dx dy = 2
x dy dx = (x − x2 )x2 dx = .
0 x2 0 20

c) La forme de D nous invite à changer de système de coordonnées. √ On Y X


effectue une transformation
√ orthogonale, en écrivant x = (X − Y ) / 2 et 1
y = (X + Y )/ 2. Dans ce√ système de coordonnées, l’ensemble ←−D
√ √ D devient
D  = {(X, Y ) ∈ R2 | − 2/2 ≤ Y ≤ 0, 2/2 ≤ X ≤ 1 − Y 2 }. Le
jacobien de la transformation est égal à 1 (transformation orthogonale), 0 1
donc d’après le théorème du changement de variable
 
2 2 xy 2 2
I= (x − y )e dx dy = (−2XY )e(X −Y )/2 dX dY,
D D
et d’après le théorème de Fubini
0  √ 
1−Y 2
2 2
I= √ √ (−2XY ) e(X −Y )/2 dX dY
− 2/2 2/2
 0     
1 2 1 Y2
= √ −2Y exp −Y + 2Y exp − dY = 1 − 2e 1/4 + e1/2 .
− 2/2 2 4 2

d) On va évidemment passer en coordonnées polaires. En polaires, on a


D = {(r, θ) ∈ [0, π] × [0, π/4]} donc d’après le théorème du changement ←−D
de variable
0 π
   
I= x cos x 2 + y 2 dx dy = r (cos θ )(cos r )r dr dθ,
D [0,π]×[0,π/4]

et d’après le théorème de Fubini


 π  π/4
 √ 
2 2 √
I= r cos r dr cos θ dθ = (−2π ) = − 2 π.
0 0 2
360 5. FONCTIONS DE PLUSIEURS VARIABLES


Exercice 2. Calculer les intégrales triples I = D
f (x, y, z ) dx dy dz dans les cas sui-
vants.
a) f (x, y, z ) = x2 + y 2 + z 2 et D = {(x, y, z ) ∈ R3 | |x| + |y | + |z | ≤ 1} ;
b) f (x, y, z ) = |z | et D = {(x, y, z ) ∈ R3 | x2 + y 2 + z2 ≤ 1} ;
c) f (x, y, z ) = (x2 + y 2 + z 2)e z et D = {(x, y, z ) ∈ R3 | |x| ≤ 1, |y | ≤ 1, |z | ≤ 1} ;
d) f (x, y, z ) = xyz et D = {(x, y, z ) ∈ R 3 | x ≥ 0, y ≥ 0, z ≥ 0, x2 + y 2 + z2 ≤ 1}.

Solution. a) Vue la symétrie de D et de l’intégrande par rapport aux plans Oxy, Oxz et Oyz ,
on a   
I =8 (x2 +y2 +z2) dx dy dz où D  = {(x, y, z ) ∈ R3 | x ≥ 0, y ≥ 0, z ≥ 0, x+ y + z ≤ 1}.
D
De même, la symétrie de D et de x2 + y 2 + z2 par rapport aux plans x = y , x = z et y = z
entraı̂ne 
I = 24 z 2 dx dy dz.
D
z
L’intégrande ne dépend plus que de z, nous allons donc effectuer une 1 D  (z )
sommation par tranches. Si (x, y, z ) ∈ D , on a 0 ≤ z ≤ 1, et l’inter- 
 
section de D avec le plan horizontal de côte z est D (z) = {(x, y ) ∈ y
2
R | x ≥ 0, y ≥ 0, x + y ≤ 1 − z}, par conséquent 0 1

 1    1
 1  2
 x
(1 − z) 2
I = 24 z2 dx dy dz = 24 z2 dz = .
0 D (z ) 0 2 5

b) La symétrie de D et de z → |z | par rapport au plan Oxy entraı̂ne



I =2 z dx dy dz où D = {(x, y, z ) ∈ R 3 | x 2 + y 2 + z2 ≤ 1, z ≥ 0}.
D
z
L’intégrande ne dépend plus que de z , le plus simple est d’effectuer une 1 D (z )
sommation par tranches. Pour tout (x, y, z ) ∈ D , on a 0 ≤ z ≤ 1, 
et l’intersection de D  avec le plan horizontal d’abscisse
√ z est D (z) =
y
{(x, y ) ∈ R2 | x2 + y 2 ≤ 1 − z2 } (disque de rayon 1 − z2 ). On en déduit 0
 1    1
 1 1
x
π
I =2 z dx dy dz = 2 z (π (1 − z 2 )) dz = .
0 D  (z ) 0 2

c) L’ensemble D est le pavé [−1, 1] 3. On a I = J + K , où


 
2 2 z
J= (x + y )e dx dy dz et K= z 2 ez dx dy dz.
D D
On calcule J en appliquant le théorème de Fubini (voir le corollaire 1), qui entraı̂ne
    
1
2 2 z
J= (x + y ) dx dy e dz ,
[−1,1]2 −1

et en appliquant une nouvelle fois le théorème de Fubini, on trouve que le premier terme du
produit est  1  1   1  
2 2 2 2 8
(x + y ) dy dx = 2x + dx = ,
−1 −1 −1 3 3
8
donc finalement J = 3 (e − 1/e).
Pour calculer K, on effectue une sommation par tranches (l’intégrande ne dépend que de z ),
ce qui donne
 1    1  
2 z 2 z 5
K= z e dx dy dz = 4 z e dz = 4 e − .
−1 [−1,1]2 −1 6
4. INTÉGRALES MULTIPLES, INTÉGRALES CURVILIGNES 361

20 52 1
Finalement, on trouve I = J + K = e− .
3 3 e
d) La forme de D nous invite à passer en coordonnées sphériques. Dans ces coordonnées, D est
le pavé [0, 1] × [0, π/2] × [0, π/ 2], donc d’après le théorème du changement de variable,

I= (r cos θ cos ϕ)(r sin θ cos ϕ)(r sin ϕ)r 2 cos ϕ dr dθ dϕ
[0,1]×[0,π/2]×[0,π/2]

= r 5 (cos θ sin θ )(cos3 ϕ sin ϕ) dr dθ dϕ,
[0,1]×[0,π/2]×[0,π/2]
et d’après le théorème de Fubini,
 1   π/2  
π/2

11 1 1
I= r 5 dr cos θ sin θ dθ cos 3 ϕ sin ϕ dϕ = = .
0 0 0 62 4 48

Exercice 3. Pour tout n ∈ N∗ , on note Dn le compact de Rn défini par


Dn = {(x1 , . . . , xn) ∈ Rn | ∀i, xi ≥ 0 et x1 + · · · + x n ≤ 1}.
 
Pour tout (k1, . . . , k n ) ∈ Nn , calculer I n(k1 , . . . , k n) = · · · D n x k1 1 · · · xknn dx1 · · · dxn .

Solution. En notant D (xn ) = {(x1 , . . . , xn−1 ) ∈ Rn−1 | ∀i, xi ≥ 0, x1 + · · · + xn−1 ≤ 1 − xn },


une intégration par tranches fournit
1   
k n−1
In (k1 , . . . , kn ) = xknn ··· xk1 1 · · · xn−1 dx1 · · · dxn−1 dxn . (∗)
0 D(xn )
  
=K (x n)

Pour xn fixé, le changement de variable x i = t i(1−xn ) (1 ≤ i ≤ n − 1) donne, d’après le théorème


du changement de variable
 
k n−1
K (x n ) = (1 − xn )k 1+···+kn−1+n−1
··· t k11 · · · tn−1 dt 1 · · · dtn−1
Dn−1
k 1+···+kn−1+n−1
= (1 − xn ) I n−1(k 1 , . . . , kn−1 ).
On en déduit d’après (*)
 1
I n(k1, . . . , k n ) = In−1(k 1, . . . , kn−1 ) xknn (1 − xn) k1+···+k n−1+n−1 dxn
0
= I n−1(k 1, . . . , kn−1 ) J (kn , k1 + · · · + k n−1 + n − 1) (∗∗)
 1
où pour tout (p, q) ∈ N 2, J (p, q) = 0 xp (1 − x) q dx.
Calculons J (p, q). Une intégration par parties fournit, si q ≥ 1,
 1  p+1 1 1
p q x q q q
J (p, q ) = x (1−x) dx = (1 − x) + xp+1(1−x)q−1 dx = J (p+1, q−1).
0 p+1 0 p+1 0 p+1
Cette relation de récurrence permet d’affirmer
q! p! q !
J (p, q ) = J (p + q, 0) = .
(p + 1) · · · (p + q ) (p + q + 1)!
D’après (**), on a donc
kn !(k 1 + · · · + kn−1 + n − 1)!
In (k1 , . . . , k n) = I n−1(k1 , . . . , kn−1 ) ,
(k1 + · · · + kn + n)!
et comme I1(k1 ) = 1/(1 + k1 ), on obtient après une récurrence facile
k1 ! · · · k n!
In(k 1 , . . . , kn ) = .
(k1 + · · · + kn + n)!
362 5. FONCTIONS DE PLUSIEURS VARIABLES

Exercice 4. a) Calculer l’aire du compact K délimité par la boucle droite de la lemnis-


cate de Bernoulli, dont l’équation polaire est r2 = a 2 cos 2θ , −π/4 ≤ θ ≤ π/4, a > 0.
b) Calculer l’aire du compact K délimité par la boucle de la strophoı̈de droite, dont
l’équation polaire est r = −a cos 2θ
cos θ
(−π/2 < θ < π/2).

Solution.
a) Il suffit d’utiliser le théorème de Green-Riemann, plus précisément
l’application
 qui le suit à la page 358, qui exprime l’aire de K par A =
1 2 0 a
2 ∂K + r dθ, ce qui ici donne

1 π/4 2 a2
A= a cos 2θ dθ = .
2 −π/4 2

b) On utilise comme précédemment, la formule A = 12 ∂K+ r2 dθ. Le
tout est de déterminer ∂K+ . Le paramètre r s’annule lorsque cos(2θ) =
0, donc pour θ = ±π/4. Un paramétrage de la frontière de K est donc
r = −a cos(2θ )/ cos θ , −π/4 ≤ θ ≤ π/4. L’orientation est bien directe, −a 0 a
on peut donc écrire

1 π/4 2 cos2 2θ
A= a dθ
2 −π/4 cos2 θ
et comme cos(2θ ) = 2 cos2 θ − 1,
  
a2 π/4 2 1 a2  π/4 a2
A= 4 cos θ − 4 + dθ = (2θ + sin 2 θ ) − 4 θ + tan θ = (4 − π ).
2 −π/4 cos2 θ 2 −π/4 2

Exercice 5 (Calcul de l’intégrale de Fresnel). Pour tout t ≥ 0, on pose


  t
i(x 2+y 2 ) 2
F (t) = e dx dy et f (t) = eix dx.
[0,t]2 0

a) Exprimer F (t) de deux manières différentes (l’une en fonction de f (t), l’autre par
passage en coordonnées polaires).
T
b) Pour T > 0, on pose I (T ) = T1 0 F (t) dt. Montrer que I (T ) converge lorsque T → +∞
et calculer sa limite. En déduire la valeur de l’intégrale de Fresnel
 +∞
2
ϕ= e ix dx.
0

2+y 2) 2 2
Solution. a) La relation ei(x = e ix eiy entraı̂ne, par application du théorème de Fubini,
 t   t 
ix 2 iy 2
F (t) = e dx e dy = f (t)2 .
0 0
Calculons maintenant F (t) de manière différente. Tout d’abord, la symétrie du domaine et
de l’intégrande par rapport à la droite x = y permet d’affirmer

2 2
F (t) = 2 ei(x +y ) dx dy,
∆t

où ∆t = {(x, y ) ∈ R | 0 ≤ x ≤ t, 0 ≤ y ≤ x}. L’intégrande s’exprime au moyen de x2 + y2 ,


2

on est amené à passer en coordonnées polaires. En polaires, le compact ∆t est représenté par
l’ensemble   π
Kt = (r, θ ) ∈ R+ × [0, 2π ] | 0 ≤ r cos θ ≤ t et θ ∈ 0, ,
4
5. PROBLÈMES 363

et après passage en coordonnées polaires, puis d’après le théorème de Fubini


  π/4  t/ cos θ 
2 2
F (t) = 2 e ir r dr dθ = 2 e ir r dr dθ
Kt 0 0
 π/4      π/4  
1 t2 iπ t2
= exp i − 1 dθ = −i exp i dθ. (∗)
0 i cos2 θ 4 0 cos 2 θ

b) La relation (*) permet d’affirmer


    
T π/4
iπ i t2
I (T ) = − exp i dθ dt.
4 T 0 0 cos2 θ

Le théorème de Fubini nous autorise à échanger l’ordre des signes d’intégration (puisqu’on a
affaire à l’intégrale sur [0, T ] × [0, π/ 4]), donc
 π/4  T      
iπ i t2 iπ i π/4 T
I (T ) = − exp i dt dθ = − cos θ f dθ. (∗∗∗)
4 T 0 0 cos2 θ 4 T 0 cos θ

Or l’intégrale ϕ = 0+∞ eix dx converge (en effectuant le changement de variable u = x2 , on
2

 +∞
remarque que f est de même nature que 0 (e iu )u−1/2 du, qui converge — voir la remarque 6
page 153), donc la fonction t → f (t) est bornée en +∞. Avec (***), on en déduit que I (T )
converge vers iπ/4 lorsque T → +∞.
Par ailleurs,
T la première relation trouvée à la question précédente entraı̂ne, pour tout T > 0,
I (T ) = 1T 0 f 2 (t) dt. Or la fonction t → f 2(t) converge vers ϕ2 lorsque t → +∞. On en déduit
que I (T ) converge vers ϕ2 lorsque T → +∞ (si une fonction ψ converge vers α en +∞, sa
T
moyenne de Césaro 1T 0 ψ converge vers α). On en déduit finalement ϕ2 = iπ/4. On a donc
 +∞
déterminé ϕ au signe près. Or s = (ϕ) = 12 0 sin(x)x −1/2 dx ≥ 0 car
+∞
  2(n+1)π  (2n+1)π  
sin u sin u 1 1
s= un , où un = √ du = √ −√ du ≥ 0,
2nπ 2 u 2nπ 2 u u+π
n=0

iπ/4 π
on en déduit facilement ϕ = e .
2
Remarque. Un autre moyen de calculer ϕ est d’utiliser le résultat de la question 2/b) de
l’exercice 4 page 168.

5. Problèmes
Problème 1. Déterminer les fonctions f : R2 → R (x, y ) → f (x, y ), de classe C2 , telles
que
∂ 2f ∂ 2f ∂2 f
+ 2 − 3 = 0. (E )
∂x2 ∂x∂y ∂y 2
(Indication : on pourra effectuer le changement de variable u = x + y , v = αx − y , où α
est un réel à déterminer).

Solution. Fixons α quelconque et notons ϕ : R2 → R2 (x, y ) → (u, v) = (x + y, αx − y ). Si


α= −1, ϕ est bijective. Plaçons nous dans ce cas. Notons F = f ◦ ϕ −1 (la fonction F est la
fonction f dans les coordonnées u et v). On a donc f (x, y) = F ◦ ϕ(x, y) = F (x + y, αx − y ) et
F = f ◦ ϕ−1 est de classe C 2. On a
∂f ∂F ∂F ∂f ∂F ∂F
= +α et = −
∂x ∂u ∂v ∂y ∂u ∂v
364 5. FONCTIONS DE PLUSIEURS VARIABLES

donc
   
∂ 2f ∂ ∂F ∂F ∂ ∂F ∂F ∂2 F ∂2 F 2
2 ∂ F
2
= +α +α +α = + 2α + α
∂x ∂u ∂u ∂v ∂v ∂u ∂v ∂u2 ∂u∂v ∂v2
   
∂ 2f ∂ ∂f ∂ ∂f ∂2 F ∂2 F ∂2 F
= +α = 2
+ (α − 1) −α 2
∂x∂y ∂u ∂y ∂v ∂y ∂u ∂u∂v ∂v
2 2 2 2
∂ f ∂ F ∂ F ∂ F
= −2 + 2,
∂y 2 ∂u2 ∂u∂v ∂v
et on en déduit
∂2 f ∂ 2f ∂2 f   ∂2 F 2 ∂2 F
+ 2 − 3 = 2 α + 2( α − 1) + 6 + (α − 2α − 3) .
∂x2 ∂x∂y ∂y2 ∂u∂v ∂v 2
2 2
L’idée est de faire disparaı̂tre le terme en ∂∂vF2 , pour se ramener à résoudre ∂u∂v ∂ F
= 0 (ce qui
2
s’intègre facilement). On va donc choisir α tel que α − 2α − 3 = 0, c’est-à-dire α = 3 ou α = −1.
Comme on doit avoir α = −1 (pour que la transformation soit bijective), on choisit α = 3.
∂ 2F
L’équation (E ) est donc équivalente à ∂u∂v = 0, où f(x, y) = F (x + y, 3x − y ). Ceci s’écrit

∂ ∂F
 1 ∂F
aussi ∂u ∂v = 0, donc la fonction C ∂v est indépendante de u. Il existe donc une fonction
ϕ : R → R de classe C 1 telle que ∂F ∂v (u, v) = ϕ ( v) pour tout (u, v ) ∈ R 2 . Notons ψ une primitive
∂ (F −ψ )
de ϕ. On a ∂v = 0, donc la fonction C 2 F − ψ est indépendante de v , de sorte qu’il existe
θ : R → R telle que F (u, v) = ψ (v ) + θ (u) pour tout (u, v) ∈ R2. Finalement, nous avons montré
que si f vérifie (E ), alors il existe deux fonctions ψ, θ : R → R de classe C 2 telles que
∀(x, y ) ∈ R 2 , f(x, y) = F (u, v) = F (x + y, 3x − y ) = θ (x + y) + ψ(3x − y ).
Réciproquement, si f a cette forme, on vérifie facilement qu’elle satisfait l’équation (E ).

Problème 2 (Dérivations). Soit n ∈ N ∗. On note G∞ 0 l’e.v des fonctions à valeurs


réelles, définies sur un voisinage de 0 et de classe C ∞. Soit L une application de G∞ 0 dans
R vérifiant les propriétés suivantes :
(i) L est linéaire ;
(ii) pour tout f, g ∈ G∞ 0 , L(f g) = f (0)L(g ) + L(f )g (0) ;
(iii) L(1) = 0 ;
(on dit que L est une dérivation). Montrer qu’il existe ξ ∈ Rn tel que L(f ) = fξ(0) =
df0(ξ) pour tout f ∈ G ∞ 0 . (Indication : on pourra utiliser le lemme d’Hadamard — voir
l’exercice 4 page 331).

Solution. Soit f ∈ G ∞
0 . Comme f − f (0) s’annule en 0, on peut écrire, d’après le lemme
d’Hadamard
n

∀x = (x1 , . . . , xn ), f(x) = f (0) + xi ϕi(x),
i=1
où pour tout i, ϕi est définie et de classe C ∞ sur un voisinage de 0, c’est-à-dire ϕi ∈ G∞ 0 . En
notant par abus xi la fonction (x 1, . . . , xn ) → xi , la linéarité et la propriété (iii) de L entraı̂nent
n
 n

L(f ) = f (0)L(1) + L(xi ϕ i) = L(xi ϕi ).
i=1 i=1
∂f
Or, d’après (ii), L(xi ϕ i) = 0 L(ϕ i ) + L(xi )ϕi(0) = L(x i )ϕi (0), et comme ϕi (0) = ∂xi (0), on en
déduit
n
∂f
L(f ) = L(xi) (0).
i=1
∂x i

En notant ξ = (L(x 1), . . . , L(xn )), ceci s’écrit aussi L(f ) = f ξ(0) = df 0(ξ ), et ceci pour tout
f ∈ G∞
0 , d’où le résultat.
5. PROBLÈMES 365

Problème 3. Soit n ∈ N∗ et f : R → R une fonction de classe C ∞ . Donner une condition


nécéssaire et suffisante sur f pour que la fonction F : Rn → Rn X → f (X 2 )X, où X 
désigne la norme euclidienne canonique de X sur Rn , soit un C ∞ difféomorphisme de Rn
sur lui même.
Solution. Considérons la fonction g : R → R, x → xf (x2 ), de classe C ∞ .
Supposons que F soit un C ∞ difféomorphisme de Rn sur Rn . En notant e 1 = (1, 0, . . . , 0) ∈
n
R , on a F (xe 1) = g (x)e1 pour tout réel x. En dérivant par rapport à x on obtient ∂F /∂x1 (xe 1 ) =
g(x)e 1, donc g  (x) = 0. Par ailleurs, F (Rn) = Rn donc pour tout y ∈ R il existe X ∈ Rn tel que
F (X ) = f (X2 )X = ye1. On en déduit que seule la première coordonnée x 1 de X n’est pas
nulle, donc X = x1 e 1 d’où f (x21)x1 e1 = ye1 , ce qui entraı̂ne y = g (x1 ). Ainsi g est une surjection
de R sur R. De plus g = 0 sur R, on en déduit que g est un C∞ difféomorphisme de R sur R.
Supposons maintenant que g est un C ∞ difféomorphisme de R sur R et montrons que F
est un C ∞ difféomorphisme de Rn sur R n . Remarquons déjà que f ne s’annule pas sur R+,
car f (0) = g (0) = 0, et pour x > 0, g (x) = g (−x) donc xf (x2 ) = −xf (x2 ) ce qui entraı̂ne
f (x2 ) = 0. Quitte à changer f par −f , on peut même supposer f > 0 sur R+.
Montrons que F est injective. Supposons F (X ) = F (Y ) pour deux vecteurs X, Y ∈ Rn . Si
X = Y = 0 on a bien X = Y , sinon l’un des deux est non nul, par exemple X = 0. On a
f (X2 )X = f (Y 2)Y , et comme f ne s’annule pas on a Y = λX avec λ = f (X 2 )/f (Y 2 ).
On en déduit f (X 2 )X = f (λ 2 X2)λX. Ceci entraı̂ne g (X ) = g(λX ), d’où X  = λX 
par injectivité de g , donc λ = 1 car X  =  0, donc Y = X .
La fonction F est surjective. En effet, si Y ∈ R n, on peut trouver x ∈ R tel que g (x) = Y .
En notant λ = x/Y  on en déduit F (λY ) = f (x2 )(x/Y )Y = (g (x)/Y )Y = Y .
La fonction F est clairement de classe C∞ et il reste à montrer que la différentielle de F
est inversible en tout point X . La matrice jacobienne JX de F en X = (x 1, . . . , xn ) vérifie
JX = (mi,j ) 1≤i,j ≤n avec mi,j = 2x i xj f  (X 2 ) si i = j et mi,i = f (X 2 ) + 2x2i f  (X 2 ).
Autrement dit, JX = f (X 2 )In + 2f (X  2)X tX . La matrice jacobienne JX est symétrique et
on va montrer qu’elle est définie positive. Pour tout vecteur V ∈ Rn non nul, on a
t
V J X V = f (X  2) t V V + 2f (X  2) t( tXV )(tXV ) = f (X  2)V  2 + 2f  (X 2 )(tXV ) 2. (∗)
Si f  (X 2) ≥ 0, alors cette expression est > 0. Sinon f (X 2 ) < 0. Comme g ne s’annule pas,
elle garde le signe de g (0) = f (0) > 0, donc g  (x) = f (x2 ) + 2x 2f (x2) > 0. Or d’après l’inégalité
de Schwarz on a (tXV ) 2 ≤ X2 · V 2, donc d’après (*)
t
V JX V ≥ f (X 2 ) V 2 − 2|f (X 2 )| X2 · V  2 = g (X ) V 2  > 0.
Ainsi JX est définie positive, donc inversible. On a donc prouvé que F est un C∞ difféomorphisme
de Rn sur lui même.

Problème 4 (Fonctions convexes sur Rn ). On munit Rn de son produit scalaire


 ,  standard et de la norme euclidienne associée. Soit C un ouvert convexe de R n. Une
application J : C → R est dite α-convexe (avec α ≥ 0) si
α
∀x, y ∈ C, ∀δ ∈ [0, 1], J[(1 − δ )x + δy] ≤ (1 − δ )J (x) + δJ(y ) − δ (1 − δ ) x − y  2 . (∗)
2
Si J est 0-convexe, on dit que J est convexe.
1/ Montrer que toute fonction convexe J : C → R est continue, et dérivable en tout
point de C par rapport à tout vecteur.

2/ Soit J : C → R une application de classe C 2. Pour tout x ∈ C , on note
 J 2(x) le 
gradient
  ∂ J
de J en x et J (x) la matrice hessienne de J en x, définie par J (x) = ∂x i∂xj (x) .
1≤i,j ≤n
Montrer que J est α -convexe si et seulement si l’une des conditions suivantes est réalisée :
(i) pour tout (x, y ) ∈ C2, J (y ) ≥ J (x) + J  (x), y − x + α2 y − x2 ;
366 5. FONCTIONS DE PLUSIEURS VARIABLES

(ii) pour tout (x, y ) ∈ C 2, J (y ) − J  (x), y − x ≥ α y − x 2 ;


(iii) pour tout x ∈ C , pour tout w ∈ Rn , J  (x) w, w ≥ α w 2 .
3/ (Optimisation dans R n) Soit J : R n → R une fonction α-convexe (avec α > 0).
a) Montrer qu’il existe x 0 ∈ R n tel que J (x0) = inf x∈R n J (x).
b) Si (un ) est une suite minimisante (i. e. si limn→+∞ J (un ) = J (x0 )), montrer que (un)
converge vers x 0.

Solution. 1/ Soit x 0 ∈ C. Commençons par montrer que J est dérivable selon tout vecteur
en x 0. Soit un vecteur ξ ∈ R n , et l’application de la variable réelle ϕ : t → J (x0 + tξ). Cette
application est définie sur un voisinage de 0 dans R, et elle de plus convexe car J est convexe.
On sait donc, d’après la théorie des fonctions convexes de la variable réelle, que ϕ est dérivable
à droite en 0, c’est-à-dire que J est dérivable selon le vecteur ξ en x0 .
Prouvons maintenant la continuité de J en x0 (attention, ce n’est pas parce que J est
dérivable selon tout vecteur en x0 qu’elle y est continue — voir un contre exemple à l’exercice 1
page 329). Quitte à considérer la fonction J (x + x0 ) −J (x0 ), on peut supposer x0 = 0et J (0) = 0.
La norme sur Rn utilisée dans la suite de cette question sera (x 1, . . . , xn ) 1 = i |xi |. L’en-
semble C est ouvert, donc il existe ρ > 0 tel que la boule B(0, ρ) soit incluse dans C . Quitte à
considérer la fonction J (ρx/2), on peut même supposer ρ = 2, de sorte que la boule unité fermée
est incluse dans C.
Commençons par montrer que  J est majorée sur la sphère unité. Notons (ei ) la base cano-
nique de Rn et considérons u = ni=1 xi ei tel que u = 1. Pour tout i, il existe εi ∈ {−1, 1} tel
que x i = |x i | ε i. Comme u = i |xi | = 1, on a facilement (comme pour les fonctions convexes
à variables réelles)
 n  n n
  
J (u) = J |x i | εi ei ≤ |xi| J (ε i ei ) ≤ M |xi | = M, avec M = sup J (ε ei).
1≤i≤n
i=1 i=1 i=1 ε∈{−1,1}

Achevons la démonstration. Soit x ∈ C , 0 < x ≤ 1, et soit u = x/x. La fonction


ψ : [−1, 1] → R λ → J (λu) est une fonction de la variable réelle convexe sur [−1, 1], donc
ψ (λ) − ψ (0)
∀λ ∈ [−1, 1], λ =
 0, ψ(0) − ψ (−1) ≤ ≤ ψ(1) − ψ (0),
λ
ce qui s’écrit aussi
J (λ u) J (λ u)
∀λ ∈ [−1, 1], λ =
 0, −J (−u) ≤ ≤ J (u) donc − M ≤ ≤ M.
λ λ
Ainsi, |J (x)| = |J (x u)| ≤ M x. Comme J (0) = 0, on en déduit la continuité de J en 0.
2/ (J est α-convexe) ⇐⇒ (i) :
— Supposons J α-convexe. Soient (x, y ) ∈ C 2 et δ ∈ ]0, 1]. L’inégalité (*) s’écrit encore
J (x + δ(y − x)) − J (x) α
≤ J (y) − J (x) − (1 − δ) y − x 2,
δ 2
d’où (i) en faisant tendre δ vers 0.
— Inversement, supposons (i) vérifié. Écrivons (i) pour le couple (y, x + δ (y − x)) puis pour
le couple (x, x + δ (y − x)), avec δ ∈ [0, 1]
α
J (y) ≥ J (x + δ (y − x)) + J  (x + δ (y − x)), (1 − δ)(y − x) + (1 − δ) 2 y − x 2 ,
2
 α 2
J (x) ≥ J (x + δ(y − x)) + J (x + δ (y − x)), −δ (y − x) + δ y − x 2 .
2
En multipliant la première inégalité par δ , la seconde par 1 − δ , puis en additionnant, il
vient
α
δJ(y) + (1 − δ )J (x) ≥ J (x + δ (y − x)) + δ (1 − δ ) y − x2 ,
2
donc J est α-convexe.
(i) ⇐⇒ (ii).
5. PROBLÈMES 367

— Supposons (i) vraie. En écrivant l’inégalité (i) pour le couple (x, y ), pour le couple (y, x),
puis en additionnant, on obtient (ii).
— Supposons (ii) vérifiée. Si on fixe (x, y) ∈ C 2, la fonction ϕ : [0, 1] → R δ → J (x +
δ (y − x)) a pour dérivée ϕ (δ) = J (x + δ (y − x)), y − x et vérifie donc
1
∀δ ∈ ]0, 1] , ϕ (δ ) − ϕ (0) = J (x + δ (y − x)) − J (x), δ(y − x) ≥ α y − x 2δ,
δ
(inégalité qui reste évidemment vraie pour δ = 0) d’où (i) car par intégration, on obtient
 1
   α
ϕ(1) ≥ ϕ (0) + α y − x 2 δ dδ = ϕ(0) + ϕ(0) + y − x 2.
0 2
(ii) ⇐⇒ (iii).
— Supposons (ii) vérifiée. Soit w ∈ R n . Lorsque ρ est un nombre réel non nul suffisamment
proche de 0, on a d’après (ii)
  
  2 2 J (x + ρw ) − J (x)
J (x + ρw ) − J (x), ρw ≥ αρ w  ou encore , w ≥ α w  2 ,
ρ
d’où (iii) en faisant tendre ρ vers 0 (remarquer que J (x) est la matrice jacobienne de
x → J(x)).
— Supposons (iii) vérifiée. Fixons (x, y) ∈ C 2 , et considérons la fonction ψ : [0, 1] → R
définie par ψ (x) = J (x + δ (y − x)), y − x). On a
∀δ ∈ [0, 1], ψ (δ) = J (x + δ (y − x))(y − x), y − x ≥ α y − x2 ,
donc par intégration ψ (1) − ψ (0) ≥ α y − x2 , d’où (ii).
3/ a) L’inégalité d’α-convexité, appliquée avec x = 0 donne
α
∀y ∈ Rn , ∀δ ∈ [0, 1], J(δy ) ≤ (1 − δ )J (0) + δJ (y ) − δ (1 − δ ) y 2
2
ce qui s’écrit encore
J (δy) − J (0) α
∀y ∈ R n , ∀δ ∈ ]0, 1], J(y ) ≥ J (0) + + (1 − δ) y  2.
δ 2
En appliquant cette dernière inégalité aux y tels que y ≥ 2 avec δ = 1/y , on a
     
y α 1 α
J (y) ≥ J (0) + J − J (0) y  + 1− y  2 ≥ J (0) + K y + y 2 ,
y  2 y  4
où K est un minorant de J (u) − J (0) sur la sphère unité (K existe car J est continue et la
sphère unité est compacte). On en déduit limy→+∞ J (y) = +∞.
Ainsi, l’ensemble Γ = {x ∈ Rn | J (x) ≤ J (0)} est borné. Or Γ est fermé (J est continue), donc
Γ ⊂ Rn est compact. Il existe donc x0 ∈ Γ tel que J (x0 ) = inf x∈Γ J (x) (ceci car J est continue).
Par construction de Γ, on a J (x) ≥ J (x 0 ) pour tout x ∈ Rn, donc J (x 0 ) = inf x∈Rn J (x).
b) L’inégalité d’α-convexité donne, pour tout m, n ∈ N,
 
α 2 J (un) + J (u m) un + um J (un ) + J (um )
un − um ≤ −J ≤ − J (x0 ),
8 2 2 2
et on en conclut que limm,n→+∞ α8 un − u m 2 = 0. Donc (un ) est une suite de Cauchy. Comme
Rn est complet, elle converge. Soit u sa limite. Par continuité de J on a J (u) = J (x0), et d’après
l’inégalité d’α-convexité
 
α 2 J (u) + J (x0 ) u + x0
u − x0  ≤ −J ≤ 0,
8 2 2
donc u = x0 . D’où le résultat.
Remarque. Les résultats des questions 2/ et 3/ restent vrais dans un espace de Hilbert.
Par contre, le résultat de 1/ est faux en dimension infinie.
368 5. FONCTIONS DE PLUSIEURS VARIABLES

Problème 5 (Lemme de Morse). 1/ (Préliminaires.) Soit n ∈ N ∗. On note S l’e.v des


matrices symétriques de Mn (R). On fixe A ∈ S inversible, et on note F l’e.v F = {U ∈
Mn (R) | tU A = AU }.
a) Montrer que Ω = F ∩G n(R) est un ouvert de F et que l’application ϕ : Ω → S U →
t
U AU est un C∞ -difféomorphisme local en U = I n .
b) Montrer l’existence d’un voisinage ouvert V de A dans S et d’une application h : V →
Gn (R) de classe C ∞ telle que h(A) = In et telle que t h(B ) A h(B) = B pour tout B ∈ V .
c) Montrer qu’il existe une application g : V → Gn(R) de classe C ∞ telle que pour tout
B ∈ V , t g (B ) D g(B) = B, où D est une matrice diagonale dont les coefficients diagonaux
sont égaux à 1 ou à −1.
2/ (Lemme de Morse.) Soit f : Rn → R une fonction de classe C ∞ , nulle
 en 0, telle
 que
∂2 f
df0 = 0. On suppose que la matrice hessienne de f en 0 définie par H = ∂x i∂xj (0)
1≤i,j ≤n
est inversible. Montrer qu’il existe un C ∞ -difféomorphisme ϕ = (ϕ1 , . . . , ϕn) défini sur un
voisinage W de 0, et un entier r tels que
∀x ∈ W, f (x) = [ϕ1 (x)]2 + · · · + [ϕr (x)] 2 − [ϕr+1 (x)] 2 − · · · − [ϕ n(x)]2
(on utilisera le résultat de la question b) de l’exercice 4 de la page 331).

Solution. 1/ a) On sait que G n(R) est un ouvert de M n(R) (image réciproque de la fonction
déterminant, qui est continue, de l’ouvert R∗ ), donc Ω = F ∩ Gn (R) est un ouvert de F .
La fonction ϕ est de classe C ∞ car les coefficients de ϕ(U ) s’expriment comme des polynômes
en les coefficients de U .
On a bien sûr In ∈ Ω. Calculons dϕ In , la différentielle de ϕ en In . Si In + H ∈ Ω, on a
ϕ(In + H ) − ϕ(In ) = tHA + AH + tHAH = tHA + AH + o(H ),
donc dϕI n (H) = tHA + AH pour tout H ∈ F .
Montrons que dϕI n : F → S est un isomorphisme de F sur S , ce qui permettra de conclure
que ϕ est un C ∞ -difféomorphisme local en In d’après le théorème d’inversion locale.
— La différentielle dϕIn : F → S est injective. En effet, si dϕI n(H) = tHA + AH = 0, alors
comme tHA = AH (puisque H ∈ F), on a AH = 0, donc H = 0 car A est inversible.
— La différentielle dϕIn est surjective. En effet, pour tout S ∈ S la matrice H = 12 A−1 S
est bien dans F (car comme A ∈ S , on a tHA = 12 tS tA −1A = 12 S = AH ) et dϕIn (H) =
t
HA + AH = 2AH = S .
b) Il existe donc un voisinage ouvert W de I n dans Ω, un voisinage ouvert V de ϕ(I n) = A
dans S tel que ϕ|W soit un C ∞-difféomorphisme de W sur V . Si on note h : V → W son
difféomorphisme inverse, h est de classe C ∞, à valeurs dans G n (R) (car W ⊂ Gn (R)) et pour
tout B ∈ V , ϕ(h(B )) = B = th(B ) A h(B).
c) Notons (p, q) la signature de la forme quadratique X → tXAX (X ∈ Rn ). On sait que p +q = n
(car A est inversible) et que A est congrue à la matrice diagonale D dont les p premiers termes
diagonaux sont égaux à 1, les q = n − p derniers égaux à −1, autrement dit, il existe P ∈ Gn (R)
tel que A = tP DP . On définit maintenant l’application g : V → G n(R) par g(B) = P h(B).
Elle est de classe C∞ et elle vérifie
t
 
∀B ∈ V, g (B ) D g(B) = th(B ) tP DP h(B ) = th(B ) A h(B) = B.

2/ D’après la question b) de l’exercice 4 de la page 331, on peut trouver n 2 fonctions h i,j : R n →


R, de classe C ∞, telles que

∀x = (x1, . . . , x n) ∈ R n , f(x) = xixj hi,j (x).
i,j

(x) = 12 [hi,j (x)+h j,i (x)] pour tout (i, j ), les fonctions ai,j sont de classe C ∞ sur Rn ,
Si on pose a i,j
on a f (x) = i,j xi x j ai,j (x), et pour tout x, la matrice A(x) = (a i,j(x))1≤i,j ≤n est symétrique.
Résumons. Il existe une application A : Rn → M n(R), de classe C ∞, telle que
5. PROBLÈMES 369

∈ Rn , A(X)
— pour tout X  t
 est symétrique et f (X ) = XA(X )X ;
∂ 2f
— A(0) = H = ∂xi∂xj (0) est inversible.
i,j
Soit (r, n − r ) la signature de la forme quadratique X → tXHX. D’après 1/c), il existe un
voisinage V de H = A(0) dans S et une fonction g : V → Gn (R), de classe C∞ , telle que
tg (B )Dg(B ) = B pour tout B ∈ V , où D est la matrice diagonale dont les r premiers éléments

diagonaux sont égaux à 1 et les n − r derniers égaux à −1.


Comme X → A(X ) est continue, il existe un voisinage W de 0 dans Rn tel que A(X ) ∈ V
pour tout X ∈ W . On a donc A(X ) = tg [A(X )] D g[A(X )] pour tout X ∈ W , et on en déduit
∀X ∈ W, f (X) = tX A(X ) X = tϕ(X ) D ϕ(X ) où ϕ(X) = g [A(X)] X.
En notant ϕ1(X ), . . . , ϕ n (X ) les coordonnées de ϕ(X ), ceci s’écrit aussi
∀X ∈ W, f (X) = ϕ 1 (X )2 + · · · + ϕr (X )2 − ϕ r+1 (X )2 − · · · − ϕ n (X )2 .
Ce n’est pas tout-à-fait fini. On a dϕ0 = g [A(0)] ∈ Gn (R), donc quitte à restreindre W , on
peut supposer (d’après le théorème d’inversion locale) que ϕ est un C ∞-difféomorphisme de W
sur ϕ(W ). On a bien sûr ϕ(0) = g [A(0)] 0 = 0.

Problème 6 (Équation de la chaleur).


On se place dans le plan R 2 dont les éléments sont notés (x, t). Pour t ΛT
tout T > 0, on note ↓ CT
T
— RT = ]0, π[ × ]0, T [, 
— CT = ({0} × [0, T ]) ∪ ([0, π] × {0}) ∪ ({π} × [0, T ]), RT
— ΛT = ]0, π[ × {T }.
de sorte que RT , CT et Λ T soient disjoints et que leur réunion soit 0 π
x
le rectangle fermé RT .
1/ Soit ϕ : [0, π] → R une fonction de classe C1, telle que
 ϕ(0) = ϕ(π ) = 0.
a) Montrer l’existence d’une suite réelle (bn) telle que |bn | converge et telle que
+∞

∀x ∈ [0, π], ϕ(x) = bn sin(nx).
n=1

b) Soit T 0 > 0. Construire une fonction Φ continue sur RT0 telle que
∂ 2 Φ ∂Φ
(i) Φ est de classe C 2 sur RT 0 et − = 0 sur R T0 ;
∂x2 ∂t
(ii) pour tout t ∈ [0, T0], Φ(0, t) = Φ(π, t) = 0 ;
(iii) pour tout x ∈ [0, π], Φ(x, 0) = ϕ(x).
2/ On veut montrer qu’il n’existe qu’une seule fonction Φ vérifiant (i), (ii) et (iii). Soit f
une fonction à valeurs réelles, continue sur RT0 et de classe C2 sur RT0 .
2
a) Si ∂∂x f2 − ∂f∂t
> 0 sur RT 0 , montrer que f atteint son maximum sur CT 0 (on pourra
commencer par prouver le résultat sur RT et CT pour tout T ∈ ]0, T 0 [).
2
b) Si ∂∂xf2 − ∂f∂t
≥ 0 sur R T0 , montrer que f atteint son maximum sur C T0 .
2
c) Si f est nulle sur CT0 et si ∂∂xf2 − ∂f∂t
= 0 sur RT 0, montrer que f est nulle.
d) En déduire que la fonction Φ construite à la question 1/b) est la seule fonction vérifiant
(i), (ii) et (iii).

Solution. 1/ a) On pense évidemment au développement en série de Fourier. Pour cela, on


prolonge ϕ sur R de la manière suivante : sur ] − π, 0[ on pose ϕ˜(x) = −ϕ(−x), sur [0, π],
ϕ̃(x) = ϕ(x). On définit ensuite une fonction ϕ sur R par ϕ(x + 2kπ) = ϕ̃(x) pour tout k ∈ Z et
370 5. FONCTIONS DE PLUSIEURS VARIABLES

pour tout x ∈ ]−π, π ]. Ainsi construite, la fonction ϕ est 2π -périodique sur R, impaire, de classe
C 1 par morceaux et continue sur R car ϕ(0) = ϕ(π ) = 0. On en déduit que sescoefficients de
Fourier a n(f ) sont nuls, que ses coefficients de Fourier bn = b n (f ) sont tels que |bn | converge
(voir le théorème 3 page 272), et que de plus
+∞

∀x ∈ R, (x) =
ϕ b n sin nx.
n=1
(x) pour tout x ∈ [0, π].
On en déduit le résultat car ϕ(x) = ϕ
2t ∂ 2f ∂f
b) Remarquons que la fonction f : (x, t) → (sin nx)e−n vérifie ∂x2
− ∂t = 0. On pose donc
+∞
 2
Φ : R T0 → R (x, t) → b n (sin nx) e −n t .
n=1

La série est normalement convergente sur RT0 , donc Φ est bien définie et continue sur RT0 . Par
ailleurs, les conditions (ii) et (iii) sont vérifiées.
2
Il nous reste à vérifier (i). Soit p ∈ N. Pour tout a > 0, on a lim n→+∞ e −n a n p = 0, donc les
 p 2  2
séries de fonctions n b n(sin nx)e −n t et n pbn (cos nx)e−n t convergent normalement pour
(x, t) ∈ [0, π] × [a, T 0]. On en déduit facilement, grâce au théorème de dérivation des séries de
fonctions que Φ est de classe Cp sur ]0, π[ × ]a, T0[, et ceci pour tout a > 0, donc de classe C p
sur R T0 , et que les dérivées partielles de Φ s’obtiennent en dérivant formellement chacun de ses
termes. Comme ceci est vrai pour tout p ∈ N, Φ est de classe C ∞ . En particulier, Φ est de classe
C 2 et
+∞
 +∞

∂2 Φ 2 −n 2t ∂Φ 2
∀(x, t) ∈ R T0 , 2
= − n (sin nx ) e , = − n2 (sin nx)e−n t .
∂x n=1
∂t n=1
∂2Φ ∂Φ
Ainsi, ∂x2
− ∂t = 0 sur RT0, d’où (i) et le résultat.
2/ a) Montrons d’abord que pour tout T ∈ ]0, T0 [, la restriction de f à RT atteint son maximum
en un point de CT (on sait déjà que f atteint son maximum sur RT car R T est compact et f est
continue). Raisonnons par l’absurde : si ceci n’est pas vrai, le maximum est atteint en un point
(x0 , t0) de RT ou de Λ T .
— Si (x 0, t 0) ∈ RT , alors comme (x0 , t 0) est un point intérieur à R T , on a
∂f ∂f
(x0 , t 0 ) = (x 0, t0 ) = 0
∂t ∂x
et la forme quadratique
∂2 f ∂2 f 2
2 ∂ f
x2 (x0 , t 0 ) + 2xt (x 0 , t 0 ) + t (x0 , t0)
∂x 2 ∂x∂t ∂t2
2
 2

est négative, en particulier ∂∂xf2 (x0 , t0) ≤ 0. On a donc ∂∂xf2 − ∂f ∂t (x0 , t0) ≤ 0, ce qui est
contraire aux hypothèses.
— Si (x 0, t 0) = (x 0, T ) ∈ ΛT , alors
∂f
— ∂t (x0 , T ) ≥ 0 puisque t → f (x 0, t) atteint son maximum en t =T ;
∂ 2f
— ∂x2 (x 0 , T ) ≤ 0 puisque x → f (x, T ) atteint son maximum en x = x 0, point intérieur
à [ 0, π].
 2 
On a donc ∂∂xf2 − ∂f ∂t (x0 , t0 ) ≤ 0, ce qui est contraire aux hypothèses.
Nous avons donc montré le premier résultat annoncé (on ne pouvait pas procéder directement
avec T = T0 car f n’est pas supposée C 2 sur un voisinage de ΛT0 ).
Achevons notre raisonnement. Soit (T n) une suite croissante de points de ]0, T [ qui converge
vers T . Comme nous venons de le montrer, il existe pour tout n un point (xn, tn ) de CTn ⊂ CT tel
que f (xn , tn ) = sup(x,t)∈R T f (x, t). Comme CT est compact, on peut extraire de la suite (xn, t n )
n
une sous-suite qui converge vers un point (x∗ , t∗ ) de CT . Quitte à retirer des termes de la suite
et à la réindicer, on peut même supposer que (x n, t n ) converge vers (x ∗, t∗ ). Par continuité de
5. PROBLÈMES 371

f , on a f (x∗ , t∗ ) = limn→+∞ f (x n, tn ). Maintenant soit (x, t) ∈ [0, π] × [0, T [. Comme t < T , il


existe N tel que t < T N, donc
∀n ≥ N, f (xn , tn ) ≥ f (x, t).
En faisant n → ∞, on en déduit f (x∗ , t ∗ ) ≥ f (x, t). Ceci est vrai pour tout (x, t) ∈ [0, π] × [0, T [.
Par continuité de f , on a donc f (x ∗, t ∗) ≥ f (x, t) pour tout (x, t) ∈ [0, π]× [0, T ], d’où le résultat.
b) La fonction f est continue sur le compact C T0 donc il existe (x ∗ , t∗ ) ∈ CT0 tel que f (x ∗, t∗ ) =
sup (x,t)∈CT f (x, t). Pour tout n ∈ N∗ , considérons la fonction fn définie sur R T 0 par fn (x, t) =
0
f (x, t) + x2 /n. La fonction f n est de classe C2 sur R T0 et
∂ 2 fn ∂fn ∂2 f ∂f 2 2
∀(x, t) ∈ R T0, 2
(x, t) − (x, t) = (x, t) − (x, t) + ≥ > 0.
∂x ∂t ∂x2 ∂t n n
On peut donc appliquer le résultat de la question précédente à fn qui entraı̂ne l’existence d’un
point (xn, tn ) de CT 0 tel que f n(x n, tn ) = sup (x,t)∈RT fn (x, t). On a donc
0

xn2 x2n π2
f (x ∗, t∗) ≥ f (x n , t n) = fn (xn , tn) − = sup f n(x, t) − ≥ sup f (x, t) − .
n (x,t)∈RT 0
n (x,t)∈RT
n
0

Ceci étant vrai pour tout n ∈ N , on en déduit, en prenant en compte les deux termes extrêmes
des inégalités, que f (x∗, t ∗ ) ≥ sup(x,t)∈R T f (x, t). Ainsi, f atteint son maximum en (x∗, t∗ ) ∈
0
CT0 .
c) La fonction f vérifie les hypothèses de la question précédente, donc son maximum est atteint
sur CT0 . Comme f est nulle sur CT0 , on en déduit f ≤ 0. Le même raisonnement s’applique à
−f , donc f ≥ 0. On en déduit que f est la fonction nulle.
d) Supposons trouvée une fonction Φ 1 vérifiant les assertions (i), (ii) et (iii). Alors f = Φ − Φ1
satisfait les hypothèses de la question précédente, donc f = 0, donc Φ = Φ1.

Problème 7. Soit f : Rn → Rn une fonction de classe C1 telle que pour tout x ∈ Rn ,


dfx (différentielle de f en x) est inversible, et telle que lim x→+∞ f (x) = +∞.
a) Pour tout compact K de Rn , montrer que f −1(K) est un compact de R n.
b) Montrer que f est surjective (on utilisera un argument de connexité).
c) Soit y un élément de R n . Montrer que f −1({y }) est un ensemble fini. On note m =
Card [f −1 ({y })] et on note x1 , . . . , xm les éléments de f −1 ({y }).
d) Pour tout ε > 0, montrer qu’il existe un voisinage ouvert V de y tel que f −1(V ) ⊂
∪1≤i≤m B(xi, ε) (où B(x i, ε) désigne la boule ouverte de centre x i de rayon ε).
e) Montrer qu’il existe un voisinage ouvert W de y tel que
 
∀z ∈ W, Card f −1({z }) = m.
f ) En déduire que l’application R n → N z → Card [f −1 (z)] est constante.
g) Si f (0) = 0 et f (z ) = 0 pour tout z = 0, montrer que f est un C 1 -difféomorphisme de
Rn sur Rn.
Solution. a) Soit K ⊂ Rn un compact. Comme K est fermé et que f est continue (car C1 ),
f −1 (K ) est fermé. Il reste à montrer que f −1(K ) est borné pour montrer sa compacité.
Comme K est compact donc borné, il existe M > 0 tel que x ≤ M pour tout x ∈ K . Or
lim f (x) = +∞ donc ∃A > 0, ∀x ∈ Rn , x > A, f (x) > M.
x→+∞

Ainsi, si x ∈ f −1(K ), on a x ≤ A, d’où le résultat.


b) On va montrer que f (Rn) est à la fois ouvert et fermé. Comme R n est connexe, on en déduira
f (Rn ) = Rn , c’est-à-dire la surjectivité de f .
372 5. FONCTIONS DE PLUSIEURS VARIABLES

Pour tout x ∈ R n , dfx est inversible, donc f est une application ouverte (voir le corollaire 1
page 343). En particulier, f (Rn) est un ouvert de Rn .
Enfin, nous avons vu que pour tout compact K de Rn, f −1 (K ) est compact. On sait alors (ou
alors on le retrouve, voir l’exercice 1 page 31) que f est une application fermée. En particulier,
f (Rn ) est fermé.
c) Commençons par montrer que les points de f −1({y }) sont isolés. Soit x 0 ∈ f −1 ({y }), de sorte
que f (x0 ) = y. Comme dfx0 est inversible, on peut appliquer le théorème d’inversion locale qui
nous dit que f est un C 1-difféomorphisme local autour de x0 . En d’autres termes, il existe un
ouvert V contenant x0 tel que f |V soit une bijection de V sur f (V ). En particulier, f est injective
sur V , donc pour tout x = x 0 et x ∈ V , f (x) = f (x0 ) = y, c’est-à-dire V ∩ f −1({y }) = {x0 }.
Or d’après a), f −1({y }) est compact. Un compact dont tous les éléments sont des points
isolés est un ensemble fini (s’il était infini, il contiendrait un point d’accumulation, qui n’est pas
isolé), donc f−1 ({y }) est fini.
d) Raisonnons par l’absurde. Si le résultat était faux, alors
   
−1 1
∀p ∈ N, f B y, p ⊂ B(x i, ε).
2
1≤i≤m

En d’autre termes, il existe pour tout p ∈ N un élément zp vérifiant f (zp ) − y ≤ 1/2p tel que
zp ∈ B(xi , ε) pour tout i. La suite (z p ) est à valeurs dans le compact f −1 (Bf (y, 1)), on peut
donc en extraire une sous-suite convergente (z ϕ(p)), de limite z . Comme f est continue, et que
f (zp) − y ≤ 1/2p pour tout p, on a f (z ) = y . Il existe donc i, 1 ≤ i ≤ m, tel que z = xi , et
comme (zϕ(p)) converge vers z = x i , il existe P ∈ N tel que zϕ(p) ∈ B(x i, ε) pour tout p ≥ P .
Ceci est impossible par construction des z p, d’où le résultat.
e) Nous allons nous servir du résultat de la question précédente. Pour cela, il faut choisir
correctement ε > 0.
Pour tout i, dfx i est inversible, donc d’après le théorème d’inversion locale, il existe ε i > 0 tel
que f soit injective sur B(xi , εi). Si ε = inf 1≤i≤m ε i, on a donc ε > 0 et f est injective sur chaque
B(xi , ε). Quitte à diminuer ε > 0, on peut supposer que les boules B(x i , ε) sont disjointes.
D’après la question précédente, on peut trouver un voisinage ouvert V de y tel que f −1(V ) ⊂
∪1≤i≤m B(xi , ε). Posons alors W = V ∩ Γ, où Γ = ∩1≤i≤mf (B(x i, ε)). L’ensemble W est un
voisinage ouvert de y (car nous avons montré à la question b) que f est une application ouverte).
Par ailleurs,
— pour tout z ∈ W , on a z ∈ Γ donc pour tout i, 1 ≤ i ≤ m, il existe  x i ∈B(xi, ε) tel que
z = f (xi). Comme les B(x i, ε) sont disjoints, on en déduit Card f −1(z ) ≥ m.
— Pour tout z ∈ W , on a z ∈ V donc f −1({z }) ⊂ f −1(V ) ⊂ ∪1≤i≤m B(xi , ε), et comme f
−1
est injectivesur chaque B(xi, ε), on a Card f ({z }) ≤ m.
−1
On en déduit Card f ({z }) = m pour tout z ∈ W .
f ) D’après la question précédente, l’ensemble Γm = {z ∈ R n | Cardf −1 ({z }) = m} est un ouvert.
Or f est surjective et f −1 ({z }) est fini pour tout z ∈ R n , donc
 
   
Rn = Γm = Γ m0  Γm  , où m0 = Cardf −1 ({0}).
m∈N∗ m=m o
m>0

Ainsi, Rn est la réunion des deux ouverts Γ m0 et ∪ m=m0 Γm. Comme Rn est connexe, que ces
 ∅ (car 0 ∈ Γ m0 ) on en tire Γ m0 = R n. En d’autres termes,
ouverts sont disjoints et que Γm0 =
Cardf ({z }) = m 0 pour tout z ∈ Rn.
−1

g) Les hypothèses s’écrivent aussi f −1 ({0}) = {0}. Donc m0 = 1, donc d’après la question
précédente, Cardf −1 ({z }) = 1 pour tout z ∈ Rn . Autrement dit, f est bijective. On conclut
avec le corollaire 4 page 344 (inversion globale) que f est un C 1-difféomorphisme global.
CHAPITRE 6

Équations différentielles

C’est au début du dix-septième siècle, avec le calcul différentiel et intégral


découvert par Leibniz et Newton, qu’apparut la notion d’équation différen-
tielle : divers problèmes de géométrie ou de mécanique conduisaient à des
questions équivalentes à l’intégration de systèmes différentiels. A l’époque,
on disposait d’une méthode générale d’intégration qui consistait à recher-
cher le développement en série entière des fonctions inconnues recherchées, les
coefficients de ces séries se déterminant à l’aide des équations et des condi-
tions initiales. Mais les mathématiciens de la fin du dix-septième siècle ne se
contentèrent pas de ce procédé et cherchèrent à exprimer les solutions à l’aide
de “fonctions élémentaires”, ou tout au moins à l’aide de primitives portant sur
des fonctions données directement par les équations. Ainsi, au début du dix-
huitième siècle, les méthodes classiques de résolution de certaines équations
(linéaires, de Bernoulli, de Ricatti, . . .) furent découvertes.
L’importance des équations différentielles dans divers domaines (en phy-
sique par exemple) amena naturellement cette branche des mathématiques
à devenir l’une des plus importantes, notamment grâce à Euler, Lagrange,
Laplace.
Jusque là, on admettait sans discussion l’existence de solutions d’équations
différentielles, sans chercher d’ordinaire à préciser les domaines où ces solu-
tions étaient définies. C’est Cauchy qui, dans ses cours donnés à l’École Poly-
technique, à partir de 1820, aborda le problème de façon rigoureuse. Il montra,
sous de bonnes hypothèses, l’existence et l’unicité d’une solution dans un voi-
sinage d’un point donné (d’ailleurs, c’est sans doute la première fois que l’on
voit apparaı̂tre une étude de caractère local en mathématiques). En 1868, Lip-
schitz remarqua que ce résultat subsiste sous une hypothèse que l’on appelle
depuis lors “condition de Lipschitz”.

1. Généralités
1.1. Définitions
Une équation différentielle est une équation portant sur les dérivées d’une fonction.
Plus précisément :
Définition 1. Soit n ∈ N∗ et E un espace de Banach. Soit une application F : Ω ⊂
R×E n → E (où Ω est un ouvert de R×E n). On appelle solution de l’équation différentielle
d’ordre n
y (n) = F (t, y, y  , . . . , y(n−1) ) (∗)
toute application ϕ : I → E ( où I est un intervalle de R), n fois dérivable et vérifiant
(i) pour tout t ∈ I, (t, ϕ(t), . . . , ϕ(n−1)(t)) ∈ Ω ;
(ii) pour tout t ∈ I, F (t, ϕ(t), . . . , ϕ(n−1) (t)) = ϕ(n)(t).
Dans la suite de cette partie, nous utiliserons les notations de cette définition.
374 6. ÉQUATIONS DIFFÉRENTIELLES

Toute équation différentielle peut se ramener à une équation différentielle d’ordre 1.


Avec les notations de la définition précédente, si on définit les applications
Φ : I → En t → (ϕ(t), . . . , ϕ (n−1)(t))
G : Ω ⊂ R × E n → En (t, y) = (t, x0 , x1 , . . . , x n−1) → (x 1 , . . . , xn−1, F (t, y)),
il est équivalent de dire que ϕ est solution de (*) ou que Φ est solution de Φ = G(t, Φ).
On peut donc se limiter à étudier les équations différentielles du premier ordre.
Remarque 1. Nous n’avons considéré ici que les équations différentielles dites résolues, i. e.
celles qui se mettent sous la forme y(n) = F (t, y, . . . , y (n−1) ). De manière plus générale,
on peut étudier les équations différentielles non résolues d’ordre n qui sont du type
F (t, y, . . . , y (n−1), y (n)) = 0. Ces dernières peuvent également se ramener à des équations
différentielles non résolues du premier ordre G(t, Y, Y ) = 0 (en utilisant les mêmes tech-
niques).
Solution maximale.
Définition 2. Une fonction ϕ : I → E ( où I est un intervalle de R) est dite solution
maximale de l’équation différentielle y(n) = F (t, y, . . . , y(n−1)) s’il n’existe pas d’autre
solution ψ : J → E de cette équation différentielle telle que I ⊂ J , I = J et ψ = ϕ sur
I , où J est un intervalle de R.
1.2. Problème de Cauchy, théorème de Cauchy-Lipschitz
Problème de Cauchy. On se donne une équation différentielle
y (n) = F (t, y, y  , . . . , y(n−1) ) (∗)
et (t0 , x0 , . . . , x n−1 ) ∈ Ω. On appelle problème de Cauchy de (*) en (t0 , x0 , . . . , xn−1) la
recherche d’une fonction ϕ : I → E ( où I est un intervalle de R) solution de (*) et
vérifiant t0 ∈ I , ϕ(t 0) = x0 , . . . , ϕ(n−1) (t0 ) = xn−1 .
On dit qu’il y a unicité au problème de Cauchy (*) en (t0 , x0 , . . . , xn−1) s’il existe au
moins une solution à ce problème de Cauchy et si pour toutes solutions ϕ : I → E et
ψ : J → E à ce problème, les fonctions ϕ et ψ coı̈ncident sur I ∩ J .
Lorsque la fonction F vérifient certaines hypothèses, on peut assurer l’unicité au
problème de Cauchy. Rappelons que l’on peut se limiter à l’étude des équations différen-
tielles du premier ordre.
Existence et unicité locale d’une solution : théorème de Cauchy-Lipschitz.
Rappel. On dit qu’une application F d’un ouvert Ω de R × Rn est localement lipschitzienne
en la seconde variable si pour tout (t0 , x 0 ) ∈ Ω, il existe un voisinage V de (t0, x0 ) et k > 0
tels que
∀t ∈ R, ∀x, x  ∈ R n , (t, x) ∈ V, (t, x ) ∈ V, F (t, x) − F (t, x) ≤ k x − x  .

 Théorème 1 (Cauchy-Lipschitz). Soit F : Ω ⊂ R × Rn → Rn ( où Ω est un ouvert de


R × Rn ) une fonction continue et localement lipschitzienne en la seconde variable. Alors
pour tout (t0, x0 ) ∈ Ω, il existe un intervalle I voisinage de t0 dans R et une application
ϕ : I → Rn solution de y = F (t, y ) telle que ϕ(t 0) = x 0 . De plus, il y a unicité pour le
problème de Cauchy de cette équation différentielle en (t0 , x0).
Démonstration. Existence. Nous aurons besoin de la notion de cylindre de sécurité. Un point
(t0 , x0) ∈ Ω est fixé. Pour tout r > 0, on pose B r = {x ∈ Rn | x − x0  ≤ r} et pour tout
α > 0, Iα = ]t 0 − α, t 0 + α[. Soit V un voisinage compact de (t 0, x0 ) dans Ω sur lequel F est
k-lipschitzienne en la seconde variable (la compacité de V est une commodité pour que F soit
1. G ÉNÉRALIT ÉS 375

bornée sur V , mais même en dimension infinie, comme F est continue en (t0 , x0 ), on peut choisir
V telle que F soit bornée sur V ). Notons M un majorant de F sur V . On peut choisir r > 0 et
α > 0, désormais fixés, tels que Iα × B r ⊂ V et αM < r (on dit que Iα × B r est un cylindre de
sécurité pour F en (t 0, x0)).
La fonction F est continue, toute solution est donc de classe C 1. Ainsi, une application
n
 t : Iα → R est solution si et seulement si pour tout t ∈ I , (t, ϕ(t)) ∈ Ω et ϕ(t) = x 0 +
ϕ
t0 F (u, ϕ(u)) du.
Notons Γ l’ensemble des fonctions continues ψ : Iα → Rn telles que ψ (Iα ) ⊂ Br . Pour tout
ψ ∈ Γ, l’application  t
ψ : I α → Rn t → x 0 + F (u, ψ(u)) du (∗)
t0
vérifie  t 
 
∀t ∈ I α, 
 ψ (t) − x0  ≤  F (u, ψ(u)) du ≤ αM < r,

t0
donc ψ ∈ Γ. On est donc autorisé (et c’est là l’utilité du cylindre de sécurité) à définir la suite
de fonctions (ψn ) par
ψ 0 : I α → Rn t → x0 et ∀n ∈ N, ψ n+1 = ψn.
Montrons que pour tout t ∈ Iα et pour tout n ∈ N, ψn+1 (t) − ψn (t) ≤ r kn |t − t0 |n /n!. Pour
n = 0, c’est immédiat d’après (*). Pour passer du rang n − 1 au rang n, il suffit d’écrire, pour
tout t ∈ Iα ,
 t   t 
   

ψn+1 (t) − ψn (t) ≤  F (u, ψ n (u)) − F (u, ψ n−1(u)) du  ≤  k ψn(u) − ψn−1 (u) du
 
t0 t0
 t 
k n   n
≤r  |t − t 0| n−1 dt = r k |t − t 0 |n.
(n − 1)!  t0  n!

La série de fonctions (ψn+1 − ψn) converge donc normalement sur I α , par conséquent la suite
de fonctions (ψn ) converge uniformément sur Iα . Notons ψ la fonction limite. On a ψ ∈ Γ et
t
∀n ∈ N, ∀t ∈ Iα, ψn+1 (t) = x0 + F (u, ψ n(u)) du,
t0

et en faisant tendre n vers +∞ on en déduit ψ (t) = x0 + tt0 F (u, ψ(u)) du, c’est-à-dire que ψ est
solution au problème de Cauchy en (t0 , x0 ) sur l’intervalle Iα .
Unicité. Soient ϕ : I → R n et ψ : J → R n deux solutions au problème de Cauchy en (t0 , x0 ).
En notant, pour tout t ∈ I ∩ J , Mt = supu∈(t 0,t) ψ (u) − ϕ(u), une récurrence sur n donne
 t 
  |t − t0 | n n
∀t ∈ I ∩ J, ∀n ∈ N, ϕ(t) − ψ (t) ≤  F (u, ϕ(u)) − F (u, ψ(u)) du ≤
 k Mt .
t0 n!
Ceci étant vrai pour tout n ∈ N, on en déduit ϕ(t) = ψ (t) pour tout t ∈ I ∩ J . 

Remarque 2. — L’existence au problème de Cauchy reste vraie si F est seulement


supposée continue (théorème de Péano), mais il n’y a plus forcément l’unicité (voir
l’exercice 1).
— L’unicité au problème de Cauchy est riche de conséquences (voir les exercices de
cette partie).
— Si F est de classe C1, F est localement lipschitzienne en la seconde variable. C’est
souvent ce que l’on rencontre dans la pratique, et le théorème de Cauchy-Lipschitz
est alors applicable.
— Pour les équations différentielles d’ordre n (en les ramenant à celles d’ordre 1 par
la technique décrite plus haut), le théorème de Cauchy s’exprime ainsi : si y (n) =
F (t, y, y , . . . , y (n−1)) est une équation différentielle d’ordre n et si F est continue
et localement lipschitzienne par rapport à la seconde variable Y = (y0 , . . . , yn−1),
alors il y a unicité au problème de Cauchy.
376 6. ÉQUATIONS DIFFÉRENTIELLES

Terminons par le corollaire suivant portant sur les solutions maximales.


Corollaire 1. Soit F : Ω ⊂ R × Rn → Rn ( où Ω est un ouvert de R × Rn ) une fonction
continue et localement lipschitzienne en la seconde variable. Alors pour tout (t 0, x0 ) ∈ Ω,
il existe une unique solution maximale ϕ de l’équation différentielle y  = F (t, y ) prenant
la valeur x0 en t 0 ; cette solution maximale est définie sur un intervalle ouvert de R.
1.3. Exercices
Exercice 1. Montrer que pour l’équation différentielle définie sur R par

0 si y < 0
y = √
y si y ≥ 0
il n’y a pas unicité au problème de Cauchy.
Solution. Faisons une première remarque : cette équation différentielle ne faisant pas intervenir
la variable t, si une fonction f : R → R est solution, il en est de même pour toutes les fonctions
fc : R → R t → f (t − c) (c ∈ R).
La fonction ϕ : R → R définie par ϕ(x) = 0 pour x ≤ 0, et ϕ(x) = x2/4 pour x ≥ 0, est
solution comme on le vérifie facilement. Notre remarque précédente montre alors que pour tout
c ∈ R, la fonction 
0 si t ≤ c
ϕc : R → R t → 2
(t − c) /4 si t > c
est solution. Mais pour c > 0, ces fonctions coı̈ncident sur R−, et pourtant elles ne sont pas
identiques. Il n’y a donc pas unicité au problème de Cauchy (le théorème de Cauchy-Lipschitz
ne s’applique pas car il n’y a pas de caractère lipschitzien).

Exercice 2. Soit F : R3 → R une application de classe C 1 . On s’intéresse à l’équation


différentielle y  = F (t, y, y  ). On suppose que pour tout t ∈ R, F (t, 0, 0) = 0 (en d’autres
termes, la fonction nulle est solution). Montrer que toute solution non identiquement nulle
a ses zéros isolés.
Solution. C’est classique ! Soit ϕ une solution non identiquement nulle, et soit t0 un zéro éventuel
de ϕ.
On a ϕ (t 0 ) = 0. En effet, si ϕ (t0 ) = 0, d’après le théorème de Cauchy-Lipschitz (qui
s’applique car F est de classe C1 ), ϕ ayant même valeur et même dérivée que la fonction nulle
en t0, ϕ est la fonction nulle (unicité au problème de Cauchy pour les équations différentielles
du second ordre), ce qui est absurde.
On peut écrire, lorsque t tend vers t0 ,
 
ϕ(t) = ϕ(t0) + (t − t0)ϕ  (t0 ) + o(t − t0 ) = (t − t0 ) ϕ (t 0 ) + o(1) ,
et comme ϕ (t0) = 0, le terme ϕ (t 0 ) + o(1) est non nul sur un voisinage de t0 , donc sur un
voisinage de t0 , ϕ ne s’annule qu’en t0 .
Les zéros de ϕ sont donc isolés.

Exercice 3. Soit F : R2 → R une fonction de classe C 1 et f, g : R → R deux solutions de


l’équation différentielle y  = F (t, y). On suppose qu’il existe t0 ∈ R tel que f (t0) < g(t 0).
Montrer que pour tout t ∈ R, f (t) < g(t).

Solution. Raisonnons par l’absurde. S’il existe t1 ∈ R tel que f (t 1 ) ≥ g (t1 ), alors comme f et
g sont continues (elles sont même de classe C 1 ), d’après le théorème des valeurs intermédiaires,
2. ÉQUATIONS DIFF ÉRENTIELLES LINÉAIRES 377

il existe u ∈ R tel que f (u) = g (u). Mais alors, d’après le théorème de Cauchy-Lipschitz (qui
s’applique car F est de classe C 1) il y a unicité au problème de Cauchy au point u, donc f = g .
Ceci est absurde car f (t0 ) = g (t0 ), d’où le résultat.

Exercice 4. Soit F : R 2 → R une application de classe C 1 vérifiant


∃T > 0, ∀t ∈ R, F (t + T, . ) = F (t, . ). (∗)
Soit ϕ : R → R une solution de l’équation différentielle y  = F (t, y). Montrer que la suite
(ϕ(kT ))k∈N est strictement monotone ou est constante. Dans ce dernier cas, montrer que
ϕ est une solution T -périodique.

Solution. Comme ϕ est solution, la relation (*) montre que la fonction ϕ T : t → ϕ(t + T ) est
aussi une solution. Trois cas se présentent.
(i) Si ϕ(0) < ϕ(T ) = ϕ T(0), alors d’après l’exercice précédent, on a ϕ(t) < ϕ T (t) pour tout
t ∈ R. On en conclut ϕ(kT ) < ϕT (kT ) = ϕ((k + 1)T ) pour tout k ∈ N, autrement dit, la
suite (ϕ(kT )) k∈N est strictement croissante.
(ii) Si ϕ(0) > ϕ(T ), on montrerait en procédant de la même manière que la suite (ϕ(kT )) k∈N
est strictement décroissante.
(iii) Si ϕ(0) = ϕ(T ) = ϕT (0), alors d’après le théorème de Cauchy-Lipschitz (qui donne
l’unicité au problème de Cauchy), ϕ = ϕT , c’est-à-dire que ϕ est T -périodique, et en
particulier, la suite (ϕ(kT )) est constante.

2. Équations différentielles linéaires


Nous nous limiterons à l’étude des équations différentielles linéaires en dimension finie.
Dans toute cette partie, n désigne un entier naturel non nul, I un intervalle de R (non
réduit à un singleton), et K désigne le corps R ou C. On identifie les éléments de Kn et
leur écriture sous forme de matrice colonne.
2.1. Généralités
Définition 1. Soit p ∈ N∗ . Toute équation différentielle sur Kn d’ordre p du type
Y (p) = Ap−1 (t) Y (p−1) + · · · + A0 (t) Y + B (t), (L)
où Ap−1 , . . . , A0 sont des fonctions continues de I dans M n(K) et B : I → Kn une
fonction continue quelconque, est appelée équation différentielle linéaire d’ordre p.
Lorsque la fonction B est identiquement nulle sur I , l’équation différentielle linéaire
(L) est dite homogène.
Comme on l’a vu dans la partie précédente, on peut ramener toute équation différen-
tielle d’ordre p à une équation différentielle d’ordre 1. Ici, l’équation différentielle linéaire
(L) peut aussi s’écrire
      
Y 0 In 0 Y 0
d  Y    ... ..
.
..
.   Y    ... 
    +  
dt 
..
. = 0 ··· 0 In  
..
.   0 
Y (p−1) A0(t) · · · · · · Ap−1 (t) Y (p−1) B (t)
(écriture en matrices par blocs). Ainsi, nous avons ramené l’équation différentielle linéaire
(L) d’ordre p à une équation différentielle linéaire d’ordre 1 (l’espace des vecteurs de base
378 6. ÉQUATIONS DIFFÉRENTIELLES

passe de Kn à K np ). Pour cette raison, nous nous limiterons à l’étude des équations
différentielles linéaires d’ordre 1.
Remarque 1. Lorsque l’équation différentielle linéaire porte sur des fonctions à valeurs
dans Kn (avec n ≥ 2), on parle aussi de système différentiel linéaire. Si n = 1, on parle
d’équation différentielle linéaire scalaire.
Solutions des équations différentielles linéaires. Les solutions maximales des
équations différentielles linéaires ont la propriété d’être définies sur tout l’intervalle I
où les fonctions de l’équation sont définies. Plus précisément, on a le résultat suivant,
dont une preuve est donnée au corollaire 1 page 400.
Théorème 1. Soit une équation différentielle linéaire
Y  = A(t) Y + B (t), (L1)
n
où A : I → Mn(K) et B : I → K sont des fonctions continues. Alors pour tout t0 ∈ I
et pour tout X 0 ∈ K n , il existe une unique solution V de (L1 ) définie sur I tout entier,
telle que V (t0) = X 0.
Remarque 2. La version de ce théorème pour les équations différentielles linéaires d’ordre
p est la suivante : pour tout t 0 ∈ I , pour tout X0 , . . . , X p−1 ∈ Kn , il existe une unique
solution ϕ de (L) définie sur I tout entier, telle que ϕ(t0 ) = X 0, . . . , ϕ(p−1)(t0 ) = Xp−1 .

 Théorème 2. Soit A : I → M n (K) une fonction continue. L’ensemble SH des solutions


maximales de l’équation différentielle linéaire homogène
dY
= A(t) Y (H )
dt
est un s.e.v de dimension n du K-espace vectoriel C 1 (I, Kn ) (fonctions C1 de I dans Kn ).
Démonstration. La forme de (H) montre que SH est un s.e.v de C 1 (I, Kn ). Par ailleurs, pour
t0 ∈ I fixé, l’application Φ : SH → Kn V → V (t0) est linéaire. C’est même un isomorphisme
d’après le théorème précédent, d’où le résultat. 

Remarque 3. — De ce théorème, on déduit que l’ensemble des solutions de l’équation


différentielle (L) : Y  = A(t) Y + B (t) est un espace affine de dimension n. En
effet, si V0 est une solution particulière de (L), les solutions de L s’écrivent V + V0 ,
où V décrit les solutions de (H ).
— La construction effectuée plus haut pour transformer une équation différentielle
linéaire d’ordre p en une équation différentielle linéaire d’ordre 1 montre, avec ce
dernier théorème, que les solutions d’une équation différentielle linéaire homogène
sur Kn , d’ordre p, forment un K-e.v de dimension np.
Wronskien. On se donne (H ) : Y  = A(t) Y une équation différentielle linéaire ho-
mogène d’ordre 1 sur Kn , où A : I → Mn (K) est continue.
Définition 2. Soient V1 , . . . , Vn n solutions de (H ). On appelle wronskien de V1 , . . . , Vn
l’application wronskien : I → K t →  det(V1 (t), . . . , Vn (t)).
Remarque 4. Considérons une équation différentielle linéaire homogène scalaire d’ordre p
y(p) = a p−1(t) y (p−1) + · · · + a 0 (t) y (Hp)
(où les ai sont des fonctions de I dans K). Nous avons vu plus haut comment transformer
p
(Hp) en une équation différentielle (H 1 ) d’ordre1 sur K , et il résulte de la construction
v 
que v est une solution de (H p ) si et seulement si .. est solution de (H 1). On appelle
.
(p−1)
v
2. ÉQUATIONS DIFF ÉRENTIELLES LINÉAIRES 379

wronskien de p solutions v 1, . . . , v p de (Hp ) le wronskien de l’équation différentielle (H1)


vi 
..
des solutions . (1 ≤ i ≤ p) de (H1 ), en d’autres termes
(p−1)
vi
 
 v 1(t) v 2(t) ··· v p (t) 
 
 .. .. .. 
wronskien(v1 , . . . , v p)(t) =  . . . .
 (p−1) (p−1) (p−1) 
 v1 (t) v 2 (t) · · · vp (t) 

Par exemple, le wronskien de deux solutions u et v d’une équation différentielle linéaire


homogène d’ordre 2 : y  = p(t)y  + q (t)y est | uu vv | = uv  − uv.
Nous verrons dans l’exercice 7 page 388 un moyen pratique de calculer le wronskien.
Proposition 1. Soient V1 , . . . , Vn des solutions de (H ). Le rang des vecteurs V1 (t), . . .,
Vn(t) est indépendant de t ∈ I .
Démonstration. C’est une conséquence immédiate de l’isomorphisme Φ : S H → Kn V → V (t 0)
( où t0 ∈ I est fixé) — voir la preuve du théorème 2. 

Corollaire 1. Des solutions V1, . . . , V n de (H ) forment une base des solutions de (H )


si et seulement s’il existe t0 ∈ I tel que wronskien(V1 , . . . , Vn)(t0 ) = 0, et dans ce cas on
a wronskien(V 1, . . . , Vn)(t) = 0 pour tout t ∈ I .
Résolution de l’équation différentielle linéaire d’ordre 1 sur K. Il est facile de
résoudre l’équation différentielle linéaire homogène (H) : y  = a(t) y, où a : I → K est
continue. Les solutions (dont on sait d’après le théorème 2 qu’elles forment un K-e.v de
dimension 1) sont toutes proportionnelles à t → eψ(t) , où ψ est une primitive de a sur
l’intervalle I.
Pour résoudre ensuite l’équation différentielle linéaire inhomogène (L) : y = a(t) y +
b(t) , où b : I → K est continue, on applique la méthode de variation de la constante.
On recherche les solutions sous la forme t → λ(t) eψ(t) où λ : I → K est une fonction
de classe C1 encore inconnue. Par dérivation, on voit que notre fonction est solution si
et seulement si λ (t)eψ(t) = b(t) pour tout t ∈ R, ce qui équivaut à dire que λ est une
primitive de e−ψ(t) b(t) sur I (voir des calculs pratiques dans l’exercice 1). Rappelons que
la solution générale de (L) est la somme de la solution générale de (H ) et d’une solution
particulière de (L).
Résolution d’un système linéaire d’ordre 1. Dans le cas général, il n’existe pas de
méthode pratique pour résoudre un système différentiel homogène (H) : Y  = A(t) Y
lorsqu’on est en dimension n ≥ 2. Cependant, si on connaı̂t n solutions V1 , . . . , Vn de (H ),
linéairement indépendantes, on sait que la solution générale de (H) est i λi Vi où l es λi ∈
K (voir le théorème 2). Connaissant ces solutions, on peut résoudre le système différentiel
inhomogène (L) : Y  = A(t)Y + B (t) par la méthode de variation  des constantes.
n
On recherche les solutions de (L) sous la forme V : t → i=1 λ i(t) V i(t) où l es
1
λi : I → K sont des fonctions C . Par dérivation, on obtient que V est solution de (L) si
et seulement si
 n n
 n

λi (t) Vi(t) + λ i(t) V i(t) = λi (t) A(t) V i(t) + B (t),
i=1 i=1 i=1

et comme Vi = A(t) Vi , ceci s’écrit aussi


n

λi(t) Vi (t) = B (t).
i=1
380 6. ÉQUATIONS DIFFÉRENTIELLES

Les Vi étant indépendants, ceci apparaı̂t comme un système de Cramer dont les inconnues
sont les λi(t). Par résolution puis intégration, on trouve les λi (voir l’exercice 1 pour des
calculs pratiques).
Regardons en particulier comment mettre en œuvre cette méthode pour les équations
différentielles linéaires d’ordre 2 sur K.
Supposons connues deux solutions linéairement indépendantes u et v de (H ) : y  =
a(t) y + b(t) y, et recherchons la solution générale de (L) : y  = a(t) y  + b(t) y +c(t).
y
On se ramène d’abord à un système différentiel d’ordre 1 : en posant Y = y  , les
équations différentielles (H ) et (L) deviennent respectivement (H1 ) : Y  = A(t) Y et
(L1) : Y  = A(t) Y + C (t) où
   
0 1 0
A(t) = et C (t) = .
b(t) a(t) c(t)
Il s’agit donc de trouver les solutions de (L1 ). On pose W (t) = λ(t) U (t) + µ(t) V (t) , où
   
U (t) = uu((tt)) et V (t) = vv((tt)) . Comme on l’a vu plus haut, W est solution de (L1 ) si et
seulement si λ(t)U (t) + µ (t)V (t) = C (t) pour tout t ∈ I , donc t → λ(t) u(t) + µ(t) v (t)
est solution de (L) si et seulement si
 
λ u + µv = 0
,
λ u  + µ  v = c(t)
relations utiles dans les exercices (voir l’exercice 1).
Abaissement de l’ordre dans une équation différentielle linéaire sur K. On
considère une équation différentielle de la forme (L) : y (n) = an−1 (t) y (n−1)+· · ·+a1 (t) y +
a0(t) y, où les a i sont des fonctions continues de I dans K. Si l’on connaı̂t une solution ϕ
de (L), il est possible d’abaisser l’ordre de (L) en procédant comme suit : on recherche les
solutions de (L) sous la forme f = gϕ, où g est une fonction de classe Cn encore inconnue.
En remplaçant dans (L), on voit que f est solution si et seulement si
n
 n−1

Cnk g(k) ϕ(n−k) = an−1 (t) k
Cn−1 g (k)ϕ(n−1−k) + · · · + a 1 (t) (gϕ + g ϕ) + a 0 (t) gϕ,
k=0 k=0

et comme ϕ est solution de (L), tous les termes contenant g s’éliminent, ce qui entraı̂ne
n
 n−1

Cnk (k)
g ϕ (n−k)
= an−1 (t) C kn−1 g(k)ϕ (n−1−k) + · · · + a 1 (t) (g ϕ).
k=1 k=1

Autrement dit, g est solution d’une équation différentielle d’ordre n − 1 : nous venons
d’abaisser l’ordre de l’équation.
Cette technique est particulièrement sympathique dans le cas des équations du second
ordre (L 2) : y  = a(t) y  + b(t) y. Si on connaı̂t une solution ϕ, f = gϕ sera solution de
(L2) si et seulement si (2g  ϕ + g ϕ) = a(t) gϕ, équation linéaire d’ordre 1 en g  que l’on
sait intégrer, ce qui permet de connaı̂tre toutes les solutions de (L 2).

2.2. Équations différentielles linéaires à coefficients constants


Lorsque A(t) est une matrice constante A, il est possible de résoudre l’équation
différentielle Y  = AY en utilisant une exponentielle de matrice
 (voir le tome Algèbre).
Rappelons que pour A ∈ Mn(K), on note exp(A) = eA = +∞ n=0 A n
/n!.
Proposition 2. Soit A ∈ Mn (K). L’équation différentielle (H ) : Y  = A Y a ses
solutions maximales définies sur R, et la solution prenant une valeur donnée V0 ∈ Rn en
t = 0 est V : R → Rn t → etA V 0 , où V0 ∈ Kn .
2. ÉQUATIONS DIFF ÉRENTIELLES LINÉAIRES 381

Démonstration. La fonction V prend bien la valeur V0 en 0 car l’exponentielle de lat nmatrice nulle
est la matrice identité. Par ailleurs, on peut écrire V sous la forme V (t) = +∞
n=0 n! (A n V ). Par
0
les techniques usuelles sur les séries de fonctions, on montre que V est de classe C 1 sur R et que
+∞
 +∞ 
 tn−1  tn
 n n
∀t ∈ R, V (t) = (A V 0) = A (A V0 ) = A V (t).
n=1
(n − 1)! n=0
n!

Remarque 5. — En réduisant la matrice A, on peut donc avoir de précieux renseigne-


ments sur les solutions de (H ).
— Lorsque l’on travaille sur le corps K = R, on peut réduire la matrice A dans C, puis
écrire les solutions de (H ) sous la forme ϕ(t) + ϕ(t) où ϕ est une solution complexe
de (H ).
Un cas particulier intéressant est celui des équations différentielles linéaires homogènes
d’ordre p sur K, à coefficients constants :
y(p) + a 1 y (p−1) + · · · + a p y = 0. (E )
En se ramenant au cas matriciel, on peut montrer le résultat suivant (dans le cas K = C).
On considère le polynôme P = X n +a1 X n−1+· · ·+a n (appelé polynôme caractéristique

de E ), puis on le factorise sous la forme ki=1 (X − ri )mi. Les solutions de (E) sont les
applications
k

t → er it Pi(t),
i=1
où l es Pi sont des polynômes de degré < m i . Ce résultat fait l’objet de l’exercice 4 page 385.
Il généralise les résultats connus pour l’ordre 1 et l’ordre 2. Rappelons ceux du deuxième
ordre. Soit (H ) : y  + ay + by = 0 et P = X 2 + aX + b. Alors
— Si P a deux racines distinctes r 1 et r2 , les solutions de (H ) sont les fonctions
t → λ1er1t + λ2er 2 t , (λ1 , λ2 ) ∈ C2 (si on est sur R et si les ri sont complexes, il faut
conjuguer les expressions) ;
— Si P a une racine double r , les solutions sont les fonctions t → (λt + µ)ert , (λ, µ) ∈
K2.
2.3. Exercices
Exercice 1. Déterminer les solutions réelles définies sur R (sauf dans la question 2/ b)
où on demande les solutions sur ] − π/2, π/2[) des équations différentielles suivantes.
1/ (Équations linéaires du premier ordre.)
a) y  + y = sin t ; b) (1 + t 2 )y = ty + (1 + t 2) ; c) (1 − t 2 )y  + ty = 0.
2/ (Équations linéaires du second ordre.)
 π π
a) y +2y + y = te t ; b) y  +y = tan2 t − <t< ; c) t 2y  −2y = 3t2 .
2 2

Solution. 1/ a) On commence par résoudre l’équation homogène associée (H ) : y  + y = 0,


dont les solutions sont t → λe −t (λ ∈ R).
Pour résoudre (L) : y  + y = sin t, on applique la méthode de variation de la constante. On
cherche une solution sous la forme t → λ(t) e −t. Celle ci sera solution de (L) si et seulement si
λ (t) e−t = sin t, c’est-à-dire λ (t) = et sin t, ce qui par intégration équivaut à λ(t) = µ + et (sin t −
cos t)/2, où µ est une constante réelle. Finalement, les solutions sur R de (L) sont les fonctions
sin t − cos t
t → + µe −t , µ ∈ R.
2
382 6. ÉQUATIONS DIFFÉRENTIELLES

b) On procède de la même manière. Ici, l’équation homogène associée√est (H ) : (1 + t 2)y  = ty ,


t
qui équivaut à y = 1+t 2 y , dont les solutions sont les fonctions t → λ 1 + t
2 (λ ∈ R).
t
Pour résoudre (L) : y = 1+t 2 y + 1, on applique la méthode de variation de la constante.

2
La fonction t → λ(t) 1 + t est solution si et seulement si
1 
λ(t) = √ ⇐⇒ λ(t) = µ + log(t + 1 + t2 ), µ ∈ R.
1 + t2
√ √ √
Les solutions de (L) sont donc celles de la forme t → µ 1 + t2 + 1 + t2 log(t + 1 + t2 ) (µ ∈ R).
c) On ne peut diviser l’équation par (1 − t2) pour la rendre résolue que si t ∈ {−1, 1}. On a donc
affaire à une équation incomplète. Nous allons la résoudre sur chacun des intervalles ] − ∞, −1[,
] − 1, 1[ et ]1, +∞[ puis “recoller” éventuellement les solutions pour trouver une solution définie
sur R tout entier.
t
On montre facilement que les solutions de (L) : y  = t2−1 y sont

2
— sur ] − ∞, −1[, t →√λ t − 1 (λ ∈ R) ;
— sur ] − 1, 1[, t → µ √1 − t2 (µ ∈ R) ;
— sur ]1, +∞[, t → ν t2 − 1 (ν ∈ R).
Si (λ, µ, ν) = (0, 0, 0), ces solutions ne peuvent, après prolongement, donner une solution sur
R tout entier (elle ne serait pas dérivable en −1 ou en 1). La seule solution de (L) définie sur R
est donc la fonction nulle.
2/ a) L’équation homogène associée est (H) : y + 2y + y = 0. Son polynôme caractéristique
est X2 + 2X + 1 = (X + 1) 2, qui a une racine double en −1. Les solutions de (H ) sont donc
celles de la forme t → (λt + µ)e −t (λ, µ ∈ R).
Il nous reste à trouver une solution de (L) : y  + 2y + y = tet . On pourrait pour cela utiliser
la méthode de variation des constantes. Ici, la forme du second membre nous invite à faire plus
simple, et à rechercher une solution sous la forme t → (at2 + bt + c)et . En reportant dans (L) et
après identification, on trouve a = 0, b = 1/4, c = −1/4.
La solution générale de (L), somme d’une solution particulière de (L) et de la solution
générale de (H ), est donc
 
t−1
t → e t + (λt + µ)e−t , (λ, µ) ∈ R 2 .
4

b) Les solutions de l’équation homogène associée sont les fonctions t → λ cos t + µ sin t avec
(λ, µ) ∈ R2. Pour trouver une solution de (L) : y  + y = tan 2 t, on applique la méthode de
variation des constantes. Une fonction de la forme t → λ(t) cos t + µ(t) sin t sera solution de (L)
si et seulement si

λ  cos t + µ sin t = 0 sin3 t sin2 t
  2 ⇐⇒ λ  (t) = − 2 et µ (t) = ,
λ (− sin t) + µ cos t = tan t cos t cos t
et par intégration on en déduit
  
1 π t
λ(t) = α − cos t − , µ(t) = β − sin t + log tan + , (α, β) ∈ R 2 .
cos t 4 2
La solution générale de (L) est donc
 π π   
π t
− , →R t→  α cos t + β sin t − 2 + sin t log tan + , (α, β) ∈ R2 .
2 2 4 2

c) Remarquons que les fonctions t → t 2 et t → 1/t sont solutions de (H) : t2 y  − 2y = 0


(attention, ici l’équation est incomplète ; il faut résoudre sur ] − ∞, 0[ et sur ]0, +∞[, puis
recoller les solutions par prolongement pour trouver des solutions définies sur R tout entier).
Sur chaque intervalle ] − ∞, 0[ et ]0, +∞[, ces deux solutions forment une base de l’e.v des
solutions de (H ).
2. ÉQUATIONS DIFF ÉRENTIELLES LINÉAIRES 383

La méthode de variation des constantes va une fois de plus nous permettre de trouver une
solution de (L) : y − 2y/t 2 = 3. La fonction t → λ(t) t2 + µ(t)/t sera solution si et seulement si
  2
λ t + µ/t = 0 1
⇐⇒ λ = et µ = −t2 ,
2t λ  − µ /t 2 = 3 t

ce qui équivaut à λ(t) = α + log |t| et µ(t) = β − t3 /3 (α, β ∈ R), d’où les solutions de (L) :

β1 t2
t → α1 t2 + + t2 log |t| − sur ] − ∞, 0[, α 1, β 1 ∈ R
t 3
β t 2
2
t → α 2 t 2 + + t 2 log t − sur ]0, +∞[, α 2 , β2 ∈ R.
t 3
Pour que ces solutions puissent se prolonger pour donner une solution sur R tout entier, il faut
avoir β1 = β2 = 0 (pour avoir une limite finie en 0). Dans ce cas, on remarque que le raccord est
dérivable en 0, n’est pas deux fois dérivable en 0 (à cause du terme en t 2 log |t|). Il n’y a donc
pas de solution sur R.
Remarque. La méthode utilisée dans la question 2/ a) se généralise comme suit. Soit
(L) : y(n) + a 1 y (n−1) + · · · + an−1 y + a n y = eλt P (t) une équation différentielle linéaire,
où l es ai et λ sont des constantes et P un polynôme de degré q . Soit χ le polynôme
caractéristique X n + a 1 X n−1 + · · · + a n , et soit m l’ordre de multiplicité de la racine
éventuelle λ de χ (en convenant m = 0 si χ(λ) = 0). Alors l’équation (L) admet une
unique solution de la forme t → tm eλtQ(t) , où Q est un polynôme de degré q.

Exercice 2. Déterminer les solutions réelles des systèmes différentiels suivant :


 
x = x + z 
x  + x + 4y − x − 3y = 0
a) y = −y − z b)
 z y  − 3y + x + 3y = 0
= 2y + z

1 0 1 
Solution. a) Soit A la matrice 0 −1 −1 , de sorte que le système s’écrit X  = AX , avec
x 0 2 1
X = z . Pour déterminer les solutions, nous diagonalisons A. On calcule facilement son
y

polynôme caractéristique PA = −(X − 1)(X 2 + 1) = −(X − 1)(X + i)(X − i), et après calculs,
les sous-espaces propres sont
     
1 1+i 1−i
Ker(A − I ) = C  0  , Ker(A − iI ) = C  1 − i  , Ker(A + iI ) = C  1 + i  .
0 −2 −2

Remarquons que A se diagonalise dans C3 . On va donc trouver des solutions complexes, puis on
déterminera les solutions réelles en prenant les parties réelles et imaginaires.
Soit λ ∈ {1, i, −i}. Si X : R → C3 t → X (t) est solution et vérifie X (t) ∈ Ker(A − λI ) pour
tout t, on a X = AX = λ X, donc X = e λt X(0), où X (0) ∈ Ker(A − λ I). Réciproquement, on
vérifie facilement que ainsi définie, X est bien solution. Par linéarité, on en déduit donc que les
fonctions
     
1 1+i 1−i
t → α e t  0  + β eit  1 − i  + γ e −it  1 + i  , (α, β, γ ∈ C)
0 −2 −2

sont des solutions. L’espace des solutions étant de dimension 3 (voir le théorème 2), on en déduit
que ceci est la solution générale (complexe). Pour obtenir la solution réelle générale, on prend
384 6. ÉQUATIONS DIFFÉRENTIELLES

les parties réelles et imaginaires. On en conclut facilement que la solution réelle générale est
       
x(t) 1 cos t − sin t sin t + cos t
t →  y (t)  = λ et  0  + µ  cos t + sin t  + ν  sin t − cos t  , λ, µ, ν ∈ R.
z (t) 0 −2 cos t −2 sin t

b) Notons (i) la première équation du système et (ii) la seconde. En additionnant (i) et (ii), on
obtient (x + y  ) + (x  + y  ) = 0, donc x + y est solution de l’équation différentielle z  + z = 0,
donc il existe λ ∈ R tel que x + y  = λe−t. Par intégration, on en déduit l’existence de deux
constantes α, β ∈ R telles que x + y = α + βe−t. En remplaçant cette égalité dans (ii), on obtient
(iii) : y − 3y + 2y = −α − βe −t. L’équation homogène associée est (iv) : y − 3y  + 2y = 0.
Son polynôme caractéristique est (X − 1)(X − 2), les solutions de (iv) sont donc de la forme
t → γet + δe2t , avec γ, δ ∈ R. On recherche une solution particulière de (iii) sous la forme
t → a + be−t . En remplaçant et après identification, on obtient a = −β/2, b = −α/6, et on en
déduit que la solution générale de (iii) est
β α −t
− e + γe t + δe2t ,
t → − γ, δ ∈ R.
2 6
Or on a vu plus haut que x = β + αe−t − y, donc la solution générale du système est
         
x(t) 3 −t 7 t 1 2t 1
t → =β +α e +γ e +δ e , (α, β, γ, δ) ∈ R4 .
y (t) −1 −1 −1 −1

Remarque. Dans la question a), on aurait aussi pu trouver les solutions en passant par
les exponentielles de matrice. D’ailleurs, la méthode que nous avons utilisée ne s’applique
plus lorsque la matrice n’est pas diagonalisable.
Dans la question b), la technique que nous avons employée n’est pas toujours ap-
plicable : parfois, des transformations linéaires simples sur les équations du système ne
permettent pas de le réduire facilement. On peut utiliser une méthode systématique, qui
consiste a transformer le système en un système
  d’ordre 1.
 Ici, le
 système b) s’écrit aussi
X + AX  + BX = 0 avec X = xy , A = 10 −43 et A = −11 −33 , et on le transforme en
     
X 0 I X
= ,
Y −B −A Y
système que l’on sait désormais résoudre.

Exercice 3. Déterminer l’ensemble des fonctions f : ]0, +∞[ → R qui sont dérivables
et qui vérifient (E) : f (t) = f (1/t) pour tout t > 0.

Solution. Si f vérifie (E ), alors f  est dérivable sur ]0, +∞[ (car f (t) = f (1/t)), et en dérivant
(E), on voit que
   
1 1 1 1 f (t)
∀t > 0, f  (t) = − 2 f  = − 2f =− 2 ,
t t t 1/t t
autrement dit, f est solution sur ]0, +∞[ de l’équation différentielle (L) : t2 y + y = 0. Ce
type d’équation différentielle est classique et s’appelle équation d’Euler. On peut la résoudre sur
chacun des intervalles ] − ∞, 0[ et ]0, +∞[ en effectuant le changement de variable t = ±ex, nous
ramenant ainsi à une équation différentielle linéaire homogène d’ordre 2 à coefficients constants
(voir la remarque à la fin de l’exercice). Ici, nous allons utiliser une technique différente.
On veut résoudre (L) sur ]0, +∞[. Nous allons commencer par en rechercher les solutions
complexes. La forme de (L) nous invite à rechercher des solutions particulières sous la forme t →
tα , où α est une constante complexe. Ceci sera le cas si et seulement si t2(α(α − 1)t α−2 ) + tα = 0
pour tout t > 0, ce qui équivaut à α(α − 1) + 1 = 0. Ainsi, si α est racine du polynôme
P = X (X − 1) + 1 = X 2 − X + 1, l’application t → tα est solution de (L) sur ]0, +∞[. Le
2. ÉQUATIONS DIFF ÉRENTIELLES LINÉAIRES 385

√ √
polynôme P a deux racines distinctes qui sont α1 = 12 + i 23 et α2 = 12 − i 23 = α 1 , on a donc
trouvé deux solutions de (L) qui sont linéairement indépendantes. L’ensemble des solutions de
l’équation linéaire homogène (L) formant un C-espace vectoriel de dimension 2, on en déduit
que la solution générale de (L) est
t → a t α1 + b t α2, a, b ∈ C. (∗)
Finalement, nous venons de montrer que si f est une solution complexe de (E ), elle est de la
forme (*). Réciproquement, une fonction de la forme (*) est solution si et seulement si
∀t > 0, aα1 t α1 −1 + bα2 tα2 −1 = a t−α1 + b t −α 2 ,
ce qui en remplaçant α 1 et α 2 par leurs valeurs s’écrit aussi
 √   √ 
1 3 √ 1 3 √ √ √
−1/2+i 3/2
∀t > 0, a +i t +b −i t −1/2−i 3/2 = a t −1/2−i 3/2+b t −1/2+i 3/2.
2 2 2 2
Ceci sera vérifié si et seulement si
 √   √ 
1 3 1 3
a +i = b et b −i = a, ou encore a eiπ/3 = b et b e−iπ/3 = a.
2 2 2 2
Ces deux équations sont liées, et elles sont équivalentes à l’existence d’un nombre complexe c tel
que a = ce−iπ/6 et b = ceiπ/6 . Finalement, f est une solution complexe de (E ) si et seulement si
elle est de la forme
√ 
 √ √  3 π
t → c e −iπ/6 t1/2+i 3/2 + eiπ/6 t1/2−i 3/2 = c t 1/2 cos log t − , c ∈ C.
2 6
On en déduit que la solution réelle générale de (E) est la même que la précédente, où la constante
c doit être prise dans R.
Remarque. De manière générale, on appelle équation d’Euler les équations linéaires ho-
mogènes du type
tn y (n) + a1 t n−1 y (n−1) + · · · + an−1 t y  + an y = 0, (∗∗)
où l es ia sont des constantes complexes. En effectuant le changement de variable t = ε e x ,
(ε ∈ {−1, 1}), on ramène cette équation à une équation linéaire homogène à coefficients
constants d’ordre n que l’on sait résoudre.
La technique que nous avons utilisée dans l’exercice pour résoudre ce type d’équations
est de rechercher des solutions particulières sous la forme t → tα . En remplaçant dans
(**), on trouve que ceci donne une solution si et seulement si α est racine d’un certain
polynôme P de degré n. Si P a ses n racines distinctes, alors cette méthode nous donne n
solutions linéairement indépendantes, et on en déduit toutes les solutions puisque l’espace
des solutions est de dimension n. Par contre, si P a une ou plusieurs racines multiples,
ceci ne nous donne qu’un nombre < n de solutions indépendantes, et on ne peut donc
pas en déduire toutes les solutions par cette méthode. Il faut procéder comme décrit
précédemment.

Exercice 4 (Solutions d’une équation différentielle  n d’ordre n à coeffi-


i
cients constants). Pour tout polynôme complexe P = i=0 a i X et pour toute fonc-
tion f : R → C de classe C n , on note
n
P (D )(f ) = ai f(i) = a n f (n) + · · · + a1 f + a0 f
i=0

(D est l’opérateur de dérivation).


a) Pour tout polynôme P ∈ C[X ], on note SP l’espace des solutions de l’équation
386 6. ÉQUATIONS DIFFÉRENTIELLES

différentielle P (D )(y ) = 0. Si P1 , . . . , Pk ∈ C[X ] sont des polynômes premiers entre eux


deux à deux, montrer que SP1 ···Pk = SP1 ⊕ · · · ⊕ SPk .
b) Si Pn (X) = (X − α) n, déterminer la forme des solutions de l’équation différentielle
Pn(D)(y) = 0.
c) En déduire, pour tout P ∈ C[X ] (deg(P ) ≥ 1), la forme des solutions de l’équation
différentielle P (D )(y) = 0.

Solution. Nous faisons tout de suite la remarque suivante, qui nous sera utile :
∀P, Q ∈ C[X ], P Q(D)(y) = P (D)[Q(D)(y)] = Q(D)[P (D)(y)].
Remarquons aussi que pour tout polynôme P , toute solution de P (D)(y ) = 0 est de classe C ∞.
a) On regarde D comme l’endomorphisme de C ∞(R, C) (e.v des fonctions de classe C ∞ de R
dans C) qui a tout u associe u. Soit P ∈ C[X ]. Dire que u est solution de l’équation différentielle
P (D )(u) = 0, c’est dire que u ∈ Ker P (D ). Autrement dit, SP = Ker P (D).
Maintenant, si les polynômes P1 , . . . , Pk sont premiers entre eux deux à deux, on a d’après le
théorème de décomposition des noyaux (voir le tome Algèbre) Ker P1 · · · P k (D ) = Ker P1(D ) ⊕
· · · ⊕ Ker Pk (D). En d’autres termes, SP1 ···Pk = SP 1 ⊕ · · · ⊕ SPk .
b) L’intuition nous guide et nous fait pressentir que les solutions de Pn (D)(y ) = 0 sont les
fonctions t → eαt F (t) où F est un polynôme complexe de degré < n. Montrons donc ce résultat
par récurrence sur n ∈ N∗.
Pour n = 1, c’est facile car P1 (D)(y) = 0 équivaut à y = αy, dont les solutions sont les
fonctions t → λ eαt , λ ∈ R.
Supposons le résultat vrai au rang n − 1 et montrons le au rang n. L’équation P n(D)(y) = 0
s’écrit aussi Pn−1 (D)[P 1(D)(y)] = 0, ce qui équivaut à dire que P1 (D)(y) a la forme t → eαy F (t),
où F ∈ C[X ], deg(F ) < n − 1. Or l’équation y − αy = eαt F (t) s’écrit aussi (ye −αt)  = F (t), ce
qui équivaut à y = e αtG(t) où G est une primitive de F , c’est-à-dire G ∈ C[X ] et deg(G) < n.
On en déduit le résultat au rang n, donc pour tout n ∈ N∗ .

c) Soit P = λ ki=1(X − α i )n i la décomposition de P en facteurs irréductibles de C[X ]. D’après
le résultat de la question a), on a
SP = S (X −α1 )n1 ⊕ · · · ⊕ S(X −αk )n k ,

et comme d’après b), S(X −α i)ni est le s.e.v des fonctions de la forme t → e αi t Fi (t) avec F i ∈ C[X ]
et deg(Fi ) < ni , on en déduit que S P est l’ensemble des fonctions de la forme t → eα 1t F1 (t) +
· · · + e α kt F k (t) où pour tout i, F i est un polynôme complexe de degré < ni .

Exercice 5. a) Soit f : R + → C une fonction de classe C1 telle que


∃α ∈ C, lim f (t) + αf (t) = 0.
t→+∞

Si (α) > 0, montrer que limt→+∞ f (t)  = 0.


b) Pour tout polynôme complexe P = ni=0 ai X i de degré n ≥ 1 et pour toute fonction
f : R+ → C de classe Cn , on pose P (D )(f ) = an f (n) + · · · + a 1 f  + a0 f . Donner une
condition nécessaire et suffisante sur un polynôme complexe P de degré n pour que la
propriété suivante soit vraie : pour toute fonction f : R+ → C de classe C n vérifiant
lim t→+∞ [P (D)(f )](t) = 0, on a limt→+∞ f (t) = 0.

Solution. a) Soit ε > 0 et soit A > 0 tel que |f (t) + αf (t)| < ε pour tout t ≥ A. En notant
a = (α), on a
 
 d αt     
∀t ≥ A,  [e f (t)] =  eαt [f(t) + αf (t)]  = eat f  (t) + αf (t) ≤ ε e at,
 dt 
2. ÉQUATIONS DIFF ÉRENTIELLES LINÉAIRES 387

ce qui entraı̂ne par intégration


 t   t
 d αu  ε
∀t ≥ A, αt
|e f (t) − e αA
f (A)| ≤   ε e au du = (eat − e aA ),
 du [e f (u)] du ≤ a
A A

donc
ε 
∀t ≥ A, |f (t)| ≤ e a(A−t)|f (A)| + 1 − e a(A−t) .
a
Comme a > 0, on en déduit l’existence de B > A tel que |f (t)| ≤ 2ε/a pour tout t ≥ B , d’où le
résultat.
b) La relation P Q(D)(f ) = P (D)[Q(D)(f )] pour tout P, Q ∈ C[X ] va nous permettre de
montrer que la condition nécessaire est suffisante recherchée est que toutes les racines de P aient
une partie réelle strictement négative.
Condition nécessaire. Nous montrons le résultat par récurrence sur n ∈ N∗. Pour n = 1, c’est
précisément le résultat démontré dans la question
précédente. Supposons le résultat vrai au rang
n − 1 et montrons le au rang n. Écrivons P = λ ni=1(X − αi ) (λ ∈ C∗ ), et supposons que toutes
les racines αi de P vérifient (αi ) < 0. On peut écrire P = Q(X − αn) , où Q est un polynôme
de degré n − 1 dont toutes les racines ont une partie réelle < 0. Soit f : R+ → C une fonction de
classe C n telle que lim t→+∞ P (D )(f )(t) = 0. On a P (D )(f ) = Q(D)[(D − α n Id)(f )] = Q(D )(g ),
où g = f  − αn f , et comme lim t→+∞ Q(D )(g )(t) = 0, on en déduit d’après l’hypothèse de
récurrence que lim t→+∞ g (t) = 0. En d’autres termes, lim t→+∞ f  (t) − αn f (t) = 0, et d’après la
question a), on en déduit que limt→+∞ f (t) = 0.
Condition suffisante. Supposons que P ait une racine α dont la partie réelle vérifie (α) ≥ 0.
On peut écrire P = Q(X − α) , où Q ∈ C[X ]. La fonction f : t → αt e ne tend pas vers 0 en
+∞, et comme f − αf = 0, on a P (D)(f ) = Q(D)[f − αf] = 0. Ainsi, nous avons trouvé une
fonction f ne tendant pas vers 0 à l’infini telle que limt→+∞ P (D )(f ) = 0. Ceci est absurde,
donc toutes les racines de P ont nécessairement leur partie réelle < 0.

Exercice 6 (Théorème de Floquet). On considère un système différentiel sur C n


(E) : Y  = A(t) Y où A : R → Mn (C) est une fonction continue et T -périodique.
a) Montrer que (E ) admet une solution V non nulle telle que

∃λ ∈ C, ∀t ∈ R, V (t + T ) = λ V (t).

b) On considère n solutions V1, . . . , Vn de (E ) linéairement indépendantes. En notant


M (t) la matrice carrée dont les vecteurs colonnes sont V1 (t), . . . , V n(t), montrer

∃C ∈ G n (C), ∀t ∈ R, M (t + T ) = M (t) C.

Solution. a) On sait (voir le théorème 2 page 378) que l’ensemble S des solutions de (E ) est
un C-e.v de dimension n. Comme A est T -périodique, on voit que pour toute solution V de
(E), l’application VT : t → V (t + T ) est aussi une solution. On construit ainsi une application
Φ : S → S V → V T . Cette application est évidemment un endomorphisme de S . Comme S
est un C-e.v de dimension finie, Φ admet au moins une valeur propre λ ∈ C. Si V ∈ S est un
vecteur propre non nul associé, on a Φ(V ) = λV , ce qui s’écrit aussi V (t + T ) = λ V (t) pour
tout t ∈ R.
b) Comme S est un espace vectoriel de dimension n, (V 1, . . . , V n) est une base de S. Notons
B = (bi,j ) la matrice de l’endomorphisme Φ dans cette base, de sorte que
n
 n

∀i ∈ {1, . . . , n}, Vi,T = bi,j Vj ou encore ∀t ∈ R, Vi(t + T ) = bi,j Vj (t).
j=1 j=1
388 6. ÉQUATIONS DIFFÉRENTIELLES

En désignant par Vi,k la k-ième composante de Vi, on a donc


n

∀i, k ∈ {1, . . . , n}, ∀t ∈ R, Vi,k (t + T ) = bi,jVj,k (t).
j=1

Comme M (t) est la matrice dont l’élément d’indice (i, j) est Vj,i (t), ceci s’écrit aussi tM (t + T ) =
B tM (t), donc M (t + T ) = M (t) tB pour tout t ∈ R. La matrice B étant inversible (car Φ est
inversible, puisque si VT = 0, alors V = 0 donc Ker Φ = {0}), la matrice C = t B est aussi
inversible.

Exercice 7 (Calcul du wronskien). a) Soient I un intervalle de R et deux applica-


tions p, q : I → C continues. On considère deux solutions u et v de l’équation différentielle
(L) : y  + p(t) y  + q (t) y = 0. Calculer le wronskien de u et v en fonction de sa valeur en
un point a de I.
b) On veut étendre ce résultat aux systèmes linéaires. Soient I un intervalle de R et
A : I → Mn (C) une fonction continue. On considère f1 , . . . , fn n solutions sur I de
l’équation différentielle (S) : Y  = A(t) Y . Calculer le wronskien de f 1 , . . . , fn en fonction
de sa valeur en un point a de I.

Solution. a) Posons W (t) = wronskien(u, v )(t) = (uv  − u v )(t). L’idée est de rechercher une
équation différentielle vérifiée par W . Pour tout t ∈ I , on a W (t) = (uv −u  v )(t). En multipliant
par u l’égalité v + p(t) v  + q (t) v = 0 et par v l’égalité u  + p(t) u+ q (t) u = 0, puis en retranchant,
on obtient uv − u v + p(t)(uv − u v ) = 0. Autrement dit, W  + p(t)W = 0. On en déduit en
résolvant cette équation différentielle linéaire d’ordre 1 que
 t 
∀t ∈ I, W (t) = W (a) exp − p(u) du .
a

b) On pose W (t) = wronskien(f1 , . . . , fn )(t) = det(f1, . . . , fn )(t). Nous allons comme plus haut
rechercher une équation différentielle satisfaite par W .
La formule de dérivation d’un déterminant (qui peut s’obtenir facilement à partir de son
développement en fonction de ses coefficients — voir le tome Algèbre) entraı̂ne
n

∀t ∈ I, W (t) = det(f1 , . . . , fi−1 , f i, f i+1, . . . , fn )(t)
i=1
n

= det(f1 (t), . . . , fi−1 (t), A(t)f i(t), fi+1(t), . . . , fn(t)).
i=1

Fixons un élément quelconque t de I . L’application Φt : (Cn )n → C définie par


n

Φt (v1 , . . . , vn ) = det(v 1, . . . , vi−1 , A(t)vi , vi+1 , . . . , vn )
i=1

est une forme n-linéaire antisymétrique, donc alternée, donc proportionnelle à l’application
déterminant (voir le tome Algèbre). On peut donc trouver λ ∈ R tel que Φt (v1 , . . . , vn) =
λ det(v1, . . . , vn ) pour tous vecteurs v 1, . . . , v n ∈ Cn . Or, en désignant par (e1 , . . . , en ) la base
canonique de Cn , on a facilement
n

Φt (e 1, . . . , e n) = det(e1 , . . . , e i−1 , A(t)e i, e i+1 , . . . , en) = tr A(t),
i=1

donc tr A(t) = λ det(e1, . . . , e n ) = λ. On connaı̂t donc Φt , et


∀t ∈ I, W (t) = Φ t(f 1, . . . , fn ) = tr A(t) · det(f1, . . . , f n)(t) = tr A(t) · W (t).
2. ÉQUATIONS DIFF ÉRENTIELLES LINÉAIRES 389

On en déduit finalement
 t 
∀t ∈ I, W (t) = W (a) exp tr A(u) du .
a

Remarque. Ce résultat peut être utile, surtout le cas particulier de a) car beaucoup
d’exercices portent sur les équations linéaires d’ordre 2.
En transformant les équations différentielles linéaires sur C d’ordre n en un système
différentiel sur Cn, un corollaire du résultat de la question b) est que le wronskien W de n
solutions d’une équation différentielle sur C du type y (n) + a1 (t) y (n−1) + · · · + an (t) y = 0
vérifie    t
∀t ∈ I, W (t) = W (a) exp − a1(u) du .
a

Exercice 8. Soit f ∈ L(Cn ). Soient λ1 , . . . , λk les valeurs propres distinctes de f . Pour


tout i, on note Ni le sous-espace caractéristique
 de f associé à la valeur propre λ i et on
note αi = dim N i (de sorte que Pf = (−1)n ki=1 (X − λi)αi ). Pour tout s.e.v N de C n, pour
tout α ∈ N∗ et pour tout λ ∈ C, on note S N,α,λ l’e.v des fonctions R → C t → e λt P (t),
où P est un polynôme à coefficients dans N de degré < α.
Soit S l’e.v des solutions de l’équation différentielle X  = f (X). Montrer
S ⊂ S N1 ,α 1 ,λ1 ⊕ · · · ⊕ S Nk,α k,λ k.
Quand peut-on dire que cette inclusion est une égalité ?

Solution. Commençons par donner quelques rappels d’algèbre (voir le tome Algèbre). On a
Cn = N1 ⊕ · · · ⊕ Nk, et pour tout i, N i est stable par f ce qui fait de fi = f |Ni un endomorphisme
de Ni . Pour tout i, on peut aussi écrire fi sous la forme f i = λi IdN i +n i, où ni est nilpotent (et
plus précisément nαi i = 0).
Soit u une solution de X = f (X). Pour tout i, notons πi la projection sur Ni parallèlement
à ⊕j =i Nj , puis ui = π i(u), de sorte que u = u 1 + · · · + uk et pour tout i, ui = f i (ui). Nous allons
montrer que pour tout i, ui ∈ S Ni ,αi ,λi . Posons vi = e−λ i tui . On a
vi = −λi vi + e −λi t u i = −λivi + e−λi t fi(u i ) = −λivi + f i (vi) = n i (vi ),
(r)
et une récurrence immédiate donne ensuite vi = nri (v i) (on peut bien dériver autant de fois
que l’on veut puisque la relation vi = n i(v i) entraı̂ne le fait que vi est de classe C ∞ ). Or n αi i = 0,
(α )
on a donc v i i = 0. Autrement dit, vi = P est un polynôme de degré < α i , à coefficients dans
Ni , et comme ui = e λi tvi , on en déduit ui ∈ SNi ,α i,λ i .
On a donc montré S ⊂ SN 1,α 1,λ 1 + · · · + SNk ,α k ,λ k . De plus, il est immédiat que les SN i,αi ,λi
sont en somme directe, d’où le premier résultat demandé.
Comme il y a inclusion, il y aura égalité si et seulement si les dimensions des sous-espaces
 
correspondants sont égales, c’est-à-dire dim S = ki=1 dim SNi ,αi ,λ i . Or dim S = n = k i=1 α i et
2
pour
 2 tout i , dim S Ni ,α i,λi = α i dim Ni = αi . Il y aura donc l’égalité si et seulement si i αi =
α
i i , ce qui équivaut à dire α i = 1 pour tout i , ou encore que les valeurs propres de f sont
toutes distinctes.
Remarque. Ce résultat reste évidemment vrai, par isomorphisme, pour les systèmes
linéaires du type X  = AX, où A ∈ Mn(C). Dans la pratique, pour résoudre (S) : X  =
AX, on ne cherche pas directement  les sous-espaces N i. On écrit qu’une solution V s’écrit
nécessairement sous la forme V = ki=1 eλ i t Pi (t) , où Pi est un polynôme de degré < αi
à valeurs dans Cn , puis on trouve la forme des coefficients des Pi par identification en
remplaçant dans (S ).
390 6. ÉQUATIONS DIFFÉRENTIELLES

3. Équations différentielles non linéaires


Le but de cette partie est de donner des méthodes pratiques de résolution de certains
types d’équations différentielles non linéaires.
Les méthodes qui suivent donnent en général des solutions sur des intervalles distincts
qui, mis bout à bout, peuvent donner des solutions sur des intervalles plus grands. Autre-
ment dit, ce qui suit ne donne pas forcément les solutions maximales. Par ailleurs, c’est
surtout des recettes qui sont mises en avant, et lors d’une résolution pratique, il faudra
prendre garde à les rendre rigoureuses (c’est d’ailleurs parfois un problème difficile).

3.1. Équations incomplètes


On appelle ainsi les équations différentielles dans lesquelles x ou y ne figure pas.
Équations de la forme F (x, y ) = 0. On note Γ = {(X, Y ) ∈ R2 | F (X, Y ) = 0}.
Supposons connu un paramétrage de Γ de la forme t → (ϕ(t), ψ(t)), où ϕ est un C 1-
difféomorphisme et où ψ est continue. L’équation F (x, y) = 0 s’écrit aussi

x = ϕ(t)
donc dy = ψ (t) dx = ψ (t)ϕ  (t) dt,
y = ψ (t)
et le graphe des solutions peut être paramétré par

x = ϕ  (t t) .
y = t0 ψ (u) ϕ (u) du + y0

Équations de la forme F (y, y ) = 0. Comme précédemment, on suppose connu un


paramétrage de Γ = {(X, Y ) ∈ R2 | F (X, Y ) = 0} de la forme t → (ϕ(t), ψ(t)), où ϕ est
un C 1-difféomorphisme et ψ est continue. Pour résoudre F (y, y ) = 0, on écrit
 
y = ϕ(t) dy = ϕ (t) dt ϕ  (t)
donc d’où dx = dt,
y  = ψ (t) dy = ψ (t) dx ψ (t)
puis on intègre, ce qui donne un paramétrage en t du graphe des solutions.
Remarque 1. Si ψ s’annule en un point t0, la fonction t → ϕ(t0 ) est solution. Elle peut se
raccorder à des solutions dont la dérivée s’annule en t 0.
Équations de la forme F (y, y , y) = 0. On cherche des solutions x → ϕ(x) telles que
ϕ ne s’annule pas, de sorte que ϕ soit un C 2 -difféomorphisme.
On cherche ensuite à paramétrer le graphe des solutions avec la variable y = ϕ(x). On
pose p = y  et on regarde p comme une fonction de y . Comme
dp dy dp dp
y = p et y  = = = y ,
dx dx dy dy
la fonction p dépendante de y vérifie l’équation différentielle F (y, p, p dp
dy
) = 0, équation
du premier ordre en p. Si on sait la résoudre, on écrit dx = dy/p(y ) puis par intégration,
on en déduit x en fonction de y.
Exemple 1. Pour résoudre l’équation (E) : y + (yy )3 = 0, on pose p = y , et on regarde
p comme une fonction de y . L’équation (E ) devient
dp dp
p + (yp)3 = 0 donc = −y 3dy,
dy p2
ce qui par intégration fournit 1/p = y 4 /4 + α, (α ∈ R) ce qui entraı̂ne
 4 
dy 1 y
p= = 4 ou encore + α dy = dx,
dx y /4 + α 4
3. ÉQUATIONS DIFF ÉRENTIELLES NON LIN ÉAIRES 391

donc après intégration, y 5/20 + αy + β = x (α, β ∈ R).


Remarque 2. — S’il existe y 0 tel que F (y0, 0, 0) = 0, alors la fonction constante y = y0
est solution, et peut se raccorder avec les solutions trouvées précédemment.
— La technique utilisée peut être généralisée aux équations incomplètes d’ordre n du
type F (y, y , . . . , y(n)) = 0, pour abaisser d’une unité l’ordre de l’équation.

3.2. Équations homogènes


On appelle ainsi les équations différentielles qui peuvent s’exprimer en fonction de y
et y/x.
Équations homogènes résolues : y = f (y/x). On pose t = y/x, donc y = tx et
f (t) dx = dy = x dt + t dx. On se ramène donc à [f (t) − t] dx = x dt, que l’on sait intégrer.
Remarque 3. S’il existe t 0 tel que f (t0 ) = t0 , la fonction y = t 0 x est solution.
Équations homogènes non résolues : F (y/x, y) = 0. On suppose connu un para-
métrage t → (u(t), v(t)) de Γ = {(U, V ) | F (U, V ) = 0}, où u est de classe C1 et v continue.
On écrit y/x = u(t) et y  = v (t), donc
dy = v (t) dx = u (t)x dt + u(t) dx d’où [v (t) − u(t)] dx = xu (t) dt.
dx u (t)
— Sur un intervalle où u − v ne s’annule pas, on écrit = dt, puis on
x v (t) − u(t)
intègre ;
— s’il existe t0 tel que u(t0) = v (t 0 ), alors x → u(t0 ) x est solution.

3.3. Étude d’équations particulières


Équations de Bernoulli. Il s’agit des équations différentielles du type
y = a(t) y + b(t) y α, α ∈ R{0, 1},
où a et b sont deux fonctions continues d’un intervalle J de R dans R (si α = 0 ou α = 1,
on a affaire à une équation linéaire, que l’on sait résoudre).
On cherche les solutions qui ne s’annulent pas (si α n’est pas entier, on cherche les
solutions > 0). L’équation peut s’écrire
y 1 1
α
= a(t) α−1 + b(t) ou encore z  = a(t) z + b(t), z = y1−α .
y y 1−α
On s’est ainsi ramené à une équation linéaire d’ordre 1, que l’on sait résoudre.
Remarque 4. Si α > 0, la fonction nulle est solution. Si de plus α ≥ 1, le théorème de
Cauchy Lipschitz s’applique et montre qu’aucune autre solution ne s’annule ; si 0 < α < 1,
on peut faire des raccords de classe C1 avec la solution nulle.
Équations de Ricatti. Il s’agit des équations différentielles du type
y = a(t) y2 + b(t) y + c(t), (R)
où a, b et c sont des fonctions continues d’un intervalle de R dans R.
Si on en connaı̂t une solution particulière ϕ0 , on sait résoudre (R) : on pose y = ϕ 0 + z ,
et en remplaçant dans (R), on obtient
z  = [2a(t) ϕ 0(t) + b(t)] z + a(t) z 2 , (R)
(qui est une équation de Bernoulli).
Le théorème de Cauchy Lipschitz montre qu’en dehors de la solution nulle, aucune
solution de (R ) ne s’annule. On peut donc poser z = 1/u, et on se ramène à une équation
différentielle du premier ordre.
392 6. ÉQUATIONS DIFFÉRENTIELLES

Remarque 5. Ici, il n’y a pas de raccords de solutions à faire.


Équation de Lagrange. Il s’agit des équations différentielles du type
y = a(y ) t + b(y ),
où a et b sont des fonctions de classe C 1 sur un intervalle de R.
On cherche d’abord les solutions affines. On voit facilement qu’elles sont de la forme
t → m t + b(m), avec a(m) = m.
On cherche ensuite les solutions de classe C 2 telle que y ne s’annule pas. On recherche
à paramétrer le graphe d’une telle solution avec la variable admissible p = y. En dérivant
par rapport à t l’égalité y = a(p)t + b(p), on obtient
dy dp dp dt
p= = a(p) t + a(p) + b  (p) donc [p − a(p)] = a (p)t + b (p).
dt dt dt dp
L’annulation de y étant équivalente au fait que p = a(p) (on s’en rend compte en dérivant
l’équation initiale), cette dernière équation différentielle est linéaire résolue du premier
ordre tant que y n’est pas nulle. On sait la résoudre, ce qui fournit la fonction p → t(p),
et on connaı̂t alors y (p) grâce à l’équation initiale. Finalement, un paramétrage du graphe
des solutions C 2 dont la dérivée seconde ne s’annule pas est p → (t(p), a(p)t(p) + b(p)).
Il ne reste plus qu’à effectuer des raccords de classe C 1 entre ces dernières solutions et
les solutions affines.
Équation de Clairaut. Il s’agit des équations différentielles du type
y = y t + b(y ),
où b est une fonction de classe C 1 sur un intervalle de R. Il s’agit d’un cas particulier
dégénéré d’équation de Lagrange pour lequel a(m) = m pour tout m.
Les solutions affines sont de la forme t → mt + b(m), m ∈ I .
L’étude effectuée pour les équations de Lagrange montrent que les solutions C 2 dont
la dérivée seconde ne s’annule pas peuvent se paramétrer sous la forme p → (t(p), y(t(p)))
où t(p) = −b (p) et y (t(p)) = −pb (p) + b(p).
L’arc ainsi paramétré possède la propriété suivante : les tangentes à cet arc sont
aussi solution de l’équation différentielle (autrement dit, l’arc est l’enveloppe des solutions
affines).
Toutes les solutions s’obtiennent par raccordement des solutions des deux types.
3.4. Exercices
Exercice 1. Déterminer
 les solutions maximales de l’équation différentielle suivante
(E) : xy  − y = x 2 + y2 .

Solution. On a affaire à une équation homogène du premier ordre. Soit y une solution maximale
de (E ) sur un intervalle I coupant R+∗ . Alors x → y (−x) est aussi solution, on va donc se limiter
dans un premier temps à étudier y sur J = I ∩ R+∗. √
Lorsque x ∈ J , on pose t = y/x, et l’équation (E ) devient (E ) : t = 1x 1 + t2 . On a donc
dt dx
√ = ,
1 + t2 x
ce qui par intégration donne argsh t = log(x/λ), avec λ > 0. Donc
  x  1  x λ

x2 − λ2
∀x ∈ J, t(x) = sh log = − donc y (x) = x t(x) = .
λ 2 λ x 2λ
La fonction x → (x2 − λ2 )/(2λ) est aussi solution sur R, et comme la fonction y est une solution
maximale, on en déduit J = R+∗ , puis K = I ∩ R−∗ = ∅ (mais attention, a priori, cela ne veut
3. ÉQUATIONS DIFF ÉRENTIELLES NON LIN ÉAIRES 393

pas dire que ce soit la seule solution maximale égale à y sur R+∗ ). Par symétrie, on a
x 2 − µ2
∃µ ∈ R, ∀x ∈ K, y(x) = .

Comme précédemment, on en déduit que K = R−∗ . Ainsi, I est un intervalle vérifiant I ∩ R+∗ =
R+∗ et I ∩ R−∗ = R −∗ , donc I = R. Comme y est continue, les limites à gauche et à droite de y
en 0 coı̈ncident, ce qui entraı̂ne −µ/2 = −λ/2, donc µ = λ.
Finalement, nous avons montré que toutes les solutions maximales de (E) sont définies sur
R et qu’elles sont de la forme
x 2 − λ2
y : x → .

Exercice 2. Déterminer les solutions maximales à valeurs réelles de l’équation différen-


tielle (R) : y  + y + y 2 + 1 = 0.
Solution. Il s’agit d’une équation de Ricatti. Cherchons en une solution particulière. La fonction
constante t → α est solution si et seulement si α2 + α + 1 = 0, c’est-à-dire α = j ou α = j2
(j = e2iπ/3 ). Ceci nous amène à commencer à rechercher les solutions complexes de (R).
En posant z = y − j et en remplaçant dans (R), on obtient z  + (2j + 1)z + z 2 = 0. On a
affaire à une équation résolue. Si z s’annule en un point, alors z aussi et d’après le théorème de
Cauchy-Lipschitz, z doit être la fonction nulle. Si z n’est pas la fonction nulle, on en conclut que
z ne s’annule pas, et on peut donc écrire l’équation sous la forme z  /z2√+ (2j + 1)/z + 1 = 0, ou
encore, en posant u = √ 1/z , −u + (2j + 1)u + 1 = 0. Comme 2j + 1√= i 3, cette équation s’écrit
aussi (L) : −u + i 3 u + 1 = 0. La fonction constante t → i/ 3 est solution, et l’équation
√ √
homogène associée est v = i 3 v , dont les solutions sont v (t) = λ ei 3t, λ ∈ C. Finalement,
nous avons montré que si z n’est pas la fonction nulle, alors
1 √ √ i 1
= u = v + i 3 = λei 3t + √ , donc y = z + j = √ √ + j.
z 3 λei 3t + i/ 3
Mais nous voulions les solutions réelles de (E ). Pour cela, nous recherchons les λ ∈ C tels
que la partie imaginaire de la fonction précédente soit nulle pour tout t. Ceci se produit si et
seulement si, pour tout t,
√   √ √
3 1 (λe i 3t + i/ 3)
− = −(j) =  √ √ =− √ √ ,
2 λei 3t + i/ 3 |λei 3t + i/ 3| 2

et en notant I = (λe i 3t
), ceci s’écrit aussi
  √
2 1 2I 3 1
|λ| + + √ =I+ √ ,
3 3 2 3
2 iθ

ce qui équivaut à |λ| = 1/3. En posant λ = e / 3, (θ ∈ R), on voit que les solutions de (R)
sont de la forme √ √ √
3 1 3 cos( 3 t + θ)
t → j + √ = + √ .
ei( 3t+θ) + i 2 2 1 + sin( 3t + θ )

Les solutions maximales sont définies sur des intervalles ouverts de longueur 2π/ 3.

Exercice 3. Déterminer les solutions de l’équation différentielle (C) : y = ty  − y  2 /4.

Solution. Il s’agit d’une équation de Clairaut. On utilise le paramètre p = y . Ainsi, (C ) s’écrit


y = tp − p2 /4, donc dy = p dt = p dt + t dp − p dp/2, c’est-à-dire (t − p/2)dp = 0.
— Lorsque dp = 0, on trouve les solutions affines de (C), qui sont de la forme t → mt − m 2 /4
(m ∈ R).
394 6. ÉQUATIONS DIFFÉRENTIELLES

— Lorsque t − p/2 = 0, on trouve la solution t → t2 .


On a ainsi trouvé les solutions de classe C2 de (C ). On obtient maintenant toutes les solutions
de classe C1 par raccordement entre les solutions des deux types.

4. Quelques compléments
Dans cette section sont présentés les compléments de cours suivant :
— quelques résultats sur les équations différentielles linéaires d’ordre 2 ;
— le lemme de Gronwall ;
— la dépendance des solutions d’une équation différentielles par rapport aux condi-
tions initiales ;
— le principe de majoration a priori, résultat qui donne des renseignements sur le
comportement des solutions maximales aux extrémités de leurs intervalles de défi-
nition.
4.1. Équations différentielles linéaires du second ordre
Les équations différentielles linéaires du second ordre sont les plus simples des équations
différentielles linéaires que l’on ne sache pas en général intégrer. Elles interviennent en
outre dans de nombreux problèmes de mécanique et de physique. Il parait donc normal
de les étudier plus particulièrement.
On désigne par K l’un des corps R ou C.
On s’intéresse à l’équation différentielle linéaire homogène
y + p(t) y  + q (t) y = 0 (L)
où p et q sont deux fonctions continues de I dans K, où I est un intervalle ouvert de R,
et où on étudie les solutions à valeurs dans K.
Rappel. Les équations différentielles du type (L) sont un cas particulier d’équations
différentielles linéaires homogènes, pour lesquels on a les résultats suivant (voir la partie 2.1
de ce chapitre) :
— toute solution maximale de (L) est définie sur I tout entier ;
— l’ensemble S des solutions maximales de (L) est un K-e.v de dimension 2. De plus,
pour tout t0 ∈ I, l’application S → K 2 f → (f (t0), f (t0 )) est un isomorphisme ;
— tous les éléments f ∈ S sont de classe C∞ ;
Wronskien. Le wronskien d’un couple de solutions maximales (u, v ) de (L), défini par
wronskien(u, v) = uv − u v vérifie
  t 
∀t0 ∈ I, ∀t ∈ I, wronskien(u, v )(t) = wronskien(u, v )(t0 ) exp − p(s) ds
t0

(voir l’exercice 7 page 388). En particulier, on retrouve le résultat du corollaire 1 page 379.
Par ailleurs, si p est identiquement nulle, le wronskien de (u, v ) est constant.
Astuces de calcul. On peut toujours ramener (L) au cas où p = 0 en procédant comme
suit. Posons y = z · x, où z et x sont deux fonctions de classe C2 de I dans K encore
inconnues. En remplaçant dans (L) on obtient :
zx  + (2z  + pz )x + (z + pz + qz )x = 0.
Il suffit donc de choisir z tel que 2z  + pz = 0, et le tour est joué (on doit supposer p de
classe C1 pour que z soit C 2 ).
Si p = 0, (L) s’écrit y  + qy = 0, et par produit par y on a y  y + qyy  = 0, ou encore
d
dt
(y 2 ) + q (t) dt
d
(y 2) = 0. Cette expression peut parfois rendre des services.
4. QUELQUES COMPL ÉMENTS 395

Rappelons enfin que la connaissance de deux solutions indépendantes de (L) permet


de résoudre (L ) : y  + p(t) y + q (t) y = r (t) par la méthode de variation des constantes
(voir page 380). Si on ne connaı̂t qu’une seule solution particulière de (L), on peut en
trouver une autre indépendante grâce à la méthode d’abaissement de l’ordre exposée à la
page 380.
Exercices.

 +
 +∞différentielle (L) : y + q(t) y = 0, où q : R → R
Exercice 1. On considère l’équation
est une fonction continue telle que 0 |q (t)| dt converge.
a) Soit y une solution bornée de (L). Étudier le comportement de y en +∞.
b) Montrer que (L) admet des solutions non bornées.
 +∞  +∞
Solution. a) Comme y est bornée et que 0 |q (t)| dt converge, on voit que 0 q (t)y (t) dt =

− 0+∞ y(t) dt converge. On en conclut que y  converge 
 x  en +∞x. Notons α = lim t→+∞ y (t). Si

α = 0, y (t) ∼ α en +∞, donc lorsque x → +∞, 0 y (t) dt ∼ 0 α dt = α x, donc y (x) ∼ αx, ce
qui absurde car y est bornée. On en déduit α = 0, donc y  tend vers 0 en +∞.
b) Raisonnons par l’absurde en supposant que (L) n’admette que des solutions bornées. Soit
(u, v) une base des solutions de (L). On a vu dans la question précédente que u et v  tendent
vers 0 en +∞, donc wronskien(u, v) = uv  − u v tend vers 0 en +∞. Or la forme de l’équation
différentielle (L) montre que le wronskien de u et v est constant (voir plus haut la sous-partie
sur le wronskien), et comme sa limite est nulle, il est identiquement nul. Ceci est absurde car
comme (u, v) est une base des solutions de (L), on a wronskien(u, v )(t) = 0 pour tout t.

Exercice 2. On considère l’équation différentielle (L) : y  + q (t) y = 0, où q : R → R


est une fonction continue et strictement négative sur R.
a) Montrer que la seule solution réelle de (L) bornée sur R est la fonction nulle.
b) Montrer qu’une solution non nulle s’annule au plus une fois sur R.

Solution. a) Soit f une solution de (L) sur R, et posons z = f 2. On a


2 2
z  = 2f f  + f  = −2q (t) f2 + f  ≥ 0,
la fonction z est donc convexe. Deux cas se présentent.
— Si z est constante, f est constante donc nulle.
— Sinon, il existe t 0 ∈ R tel que z (t0 ) = 0. Comme z est convexe, sa courbe représentative
est au dessus de sa tangente en t0 , ce qui s’écrit z (t) ≥ z (t0)+z  (t0) (t − t0 ). On en déduit,
selon le signe de z (t0 ), que z tend vers +∞ en −∞ ou en +∞. Donc z n’est pas bornée
sur R, donc y n’est pas bornée sur R.
b) Avec les notations précédentes, si y s’annule en deux points t1 et t2 (t 1 < t2 ), alors z = y2
aussi. Comme z est convexe et positive, ceci entraı̂ne z (t) = 0 sur [t1 , t 2 ], donc y (t) = 0 sur
[t1 , t2 ], donc d’après le théorème de Cauchy-Lipschitz, y est nulle sur R tout entier. Ainsi, une
solution y qui s’annule en deux points est identiquement nulle.

 Exercice 3 (Théorèmes d’oscillation et de comparaison). 1/ On considère


l’équation différentielle (L) : y  + p(t) y + q (t) y = 0, où p et q sont deux fonctions réelles
continues sur un intervalle I de R.
a) Montrer qu’une solution non nulle f de (L) n’a qu’un nombre fini de zéros dans tout
segment de I .
b) Soient f, g deux fonctions qui forment une base des solutions de (L). Soient t1 et t2
396 6. ÉQUATIONS DIFFÉRENTIELLES

(t1 < t2 ) deux zéros consécutifs de f . Montrer qu’il existe un unique point t 0 ∈ ]t1 , t 2[ tel
que g (t0) = 0.
2/ a) Soient I un intervalle de R, et r, s : I → R deux fonctions continues telles que r ≤ s
sur I . Soit x une solution non nulle de (L1 ) : x  + r (t) x = 0, y une solution non nulle de
(L2) : y  + s(t) y = 0. Soient t1 et t2 (t 1 < t2 ) deux zéros consécutifs de x. Montrer que
si x et y ne sont pas proportionnelles sur ]t1 , t2 [, il existe t0 ∈ ]t1 , t 2[ tel que y (t0 ) = 0.
b) Si q est une fonction continue sur I qui vérifie q (t) ≤ µ2 pour tout t (avec µ >
0), montrer que deux zéros consécutifs t 1 et t2 (t1 < t2 ) d’une solution non nulle de
(L) : y + q (t) y = 0 vérifient t 2 − t1 ≥ π/µ.
c) Si q est une fonction continue sur I qui vérifie q (t) ≥ λ2 pour tout t (avec λ > 0),
montrer que toute solution de (L) : y  + q (t) y = 0 s’annule au moins une fois dans tout
intervalle fermé de longueur π/λ.

Solution. 1/ a) On sait déjà (voir l’exercice 2 page 376), que les zéros de f sont isolés.
Supposons que f ait une infinité de zéros dans un segment [a, b] ⊂ I . Comme [a, b] est
compact, il existe un élément x ∈ [a, b] qui est un point d’accumulation des zéros de f . Par
continuité de f , on a f (x) = 0, autrement dit x est un zéro de f . Ceci est absurde car x est un
point d’accumulation des zéros de f et les zéros de f sont isolés.
b) La fonction f ne s’annule pas sur ]t1 , t2 [. Quitte à changer f en −f , on peut donc supposer
f > 0 sur ]t 1, t2 [. Comme f (t 1) = 0, on a
f (t) − f (t1) f (t)
f (t1) = lim = lim ≥ 0,
t→t1
t>t1
t − t1 t→
t>t
t1 t − t1
1

et comme f (t 1 ) = 0 (sinon f serait la fonction nulle d’après le théorème de Cauchy-Lipschitz),


on a f (t 1) > 0. On montrerait de même f  (t2) < 0.
Ceci étant, posons W = f g − f g = wronskien(f, g). Comme (f, g) est une base de solutions
de (L), W ne s’annule pas, donc garde un signe constant sur [t 1, t2]. Quitte à changer g en −g ,
on peut même supposer W > 0 sur [t1, t 2 ]. On a W (t 1 ) = −f (t1 )g (t 1 ) > 0, donc g (t 1) < 0, et
de même, à partir de W (t2 ) = −f (t2 )g (t 2 ) > 0, on obtient g (t2 ) > 0. Comme g est continue, le
théorème des valeurs intermédiaires nous assure donc l’existence de t0 ∈ ]t 1 , t2 [ tel que g (t 0 ) = 0.
Il nous reste à montrer que g ne s’annule qu’une fois dans ]t1 , t2[. Supposons que outre le
point t0 , g s’annule en un autre point t0 de ]t1 , t2[. En appliquant à f ce que l’on vient de montrer
pour g, on en déduit que f s’annule entre t 0 et t0 , ce qui absurde car t1 et t2 sont deux zéros
consécutifs de f . D’où l’unicité de t0 ∈ ]t1 , t2 [ tel que g (t0 ) = 0.
2/ a) Supposons que y ne s’annule pas sur ]t1 , t2 [. Par hypothèse, x ne s’annule pas sur ]t1 , t 2[,
et quitte à changer x en −x et y en −y , on peut supposer que x et y prennent des valeurs > 0
sur ]t1, t 2 [. On généralise maintenant la méthode utilisée dans la question précédente. On pose
W (t) = xy−x  y. Comme précédemment, on a x (t1 ) > 0 et x (t2 ) < 0. Comme x(t1) = x(t 2 ) = 0,
on a donc W (t1) ≤ 0 et W (t 2) ≥ 0.
Par ailleurs, W  = xy  − x  y = x(−s(t)y ) − y (−r(t)x) = xy(r (t) − s(t)) ≤ 0 sur [t1 , t2 ], donc
W décroı̂t de W (t1 ) ≤ 0 à W (t2 ) ≥ 0. Autrement dit, W est nulle sur [t 1, t2 ]. On en conclut que
x /x = y /y sur ]t1, t 2[, donc x et y sont proportionnelles sur ]t1 , t2 [. D’où le résultat.
b) Soit f une solution non nulle de (L) et t 1, t2 deux zéros consécutifs de f . Supposons t2 − t1 <
π/µ. Alors il existe ϕ ∈ R telle que la fonction g : t → sin(µt + ϕ), ne s’annule pas sur [t1 , t 2].
Une telle fonction ne peut pas être proportionnelle à f car f (t 1) = f (t2 ) = 0. Comme g est
solution de l’équation différentielle y + µ2 y = 0, le résultat de la question précédente nous dit
que g s’annule au moins une fois sur ]t1 , t 2[, ce qui est absurde. On a donc bien t2 − t 1 ≥ π/µ.
c) Soit [t1, t2 ] un segment de longueur π/λ. On peut trouver ϕ ∈ R tel que la fonction g : t →
sin(λt + ϕ) s’annule aux points t1 et t2 . La fonction g est solution de l’équation différentielle
y + λ 2 y = 0. Si f et g sont proportionnelles, alors f s’annule en t 1 et t 2 . Sinon d’après 2/a),
f doit s’annuler au moins une fois sur ]t1 , t 2[. Ainsi f s’annule toujours au moins une fois sur le
segment [t1, t2 ].
4. QUELQUES COMPL ÉMENTS 397

Exercice 4. On considère l’équation différentielle (L) : y  + q (t) y = 0, où q : [a, b] → R


est une fonction continue. On suppose que (L) possède une solution y nulle en a et b et
> 0 sur ]a, b[. Montrer
 b
4
|q (t)| dt > .
a b−a

Solution. La fonction y est continue sur [a, b], donc il existe c ∈ [a, b] tel que y (c) = supt∈[a,b] y (t).
Comme de plus y (a) = y (b) = 0 et que y > 0 sur ]a, b[, on a c ∈ ]a, b[. On écrit maintenant, pour
tout α, β tels que a < α < β < b,
 b  b     b
y (t) 
|q (t)| dt =   dt > 1 |y  (t)| dt ≥
1
|y  (β ) − y (α)|.
 y (t)  y (c) y (c)
a a a
Or, d’après le théorème des accroissements finis, on peut trouver α ∈ ]a, c[ et β ∈ ]c, b[ tels
que
y (c) − y (a) y (b) − y (c)
= y (α) et = y (β ).
c−a c−b
On en conclut
 b  
1 1  y (c) − y (a) y (b) − y (c) 
|q (t)| dt > |y(β ) − y  (α)| =  − = 1 + 1
y (c) y (c)  c−a b−c  c−a b−c
a

(car y (a) = y (b) = 0). Une rapide étude de la fonction c → c−1 a + b−1
c
montre qu’elle atteint son
minimum pour c = (a + b)/2, point en laquelle elle vaut 4/(b − a). Donc finalement
b
1 1 4
|q (t)| dt > + ≥ ,
a c−a b−c b−a
d’où le résultat.

4.2. Lemme de Gronwall et applications


On a parfois affaire à des inégalités portant sur une fonction et sa dérivée. Ce type de
problème peut être résolu grâce au résultat qui suit.
 Théorème 1 (Lemme de Gronwall). Soient ϕ, ψ et y trois fonctions continues sur
un segment [a, b], à valeurs positives et vérifiant l’inégalité
 t
∀t ∈ [a, b], y(t) ≤ ϕ(t) + ψ (s)y (s) ds. (∗)
a
Alors   
t t
∀t ∈ [a, b], y(t) ≤ ϕ(t) + ϕ(s)ψ (s) exp ψ (u) du ds.
a s
t
Démonstration. Posons F (t) = a ψ (s)y (s) ds. En multipliant les deux membres de (*) par ψ (t),
on obtient F  (t) − ψ (t)F (t) ≤ ϕ(t)ψ (t), ce qui s’écrit aussi
  t    t 
G (t) ≤ ϕ(t)ψ (t) exp − ψ (s) ds avec G(t) = F (t) exp − ψ (s) ds .
a a
Comme G(a) = F (a) = 0, on en déduit, par intégration
 t  s 
G(t) ≤ ϕ(s)ψ (s) exp − ψ (u) du ds.
a a
 
t
Or, d’après (*), y (t) ≤ ϕ(t) + G(t) exp a ψ (s) ds , d’où le résultat en utilisant l’inégalité ci
dessus. 
398 6. ÉQUATIONS DIFFÉRENTIELLES

Ce théorème n’est pas au programme des classes de mathématiques spéciales. Il n’est


pas nécessaire de connaı̂tre le résultat, mais il faut en revanche savoir le retrouver fa-
cilement, et se souvenir que le lemme de Gronwall exprime qu’à partir d’une inégalité
intégrale portant sur y , on peut en déduire une inégalité sur y.
On utilise parfois le lemme de Gronwall dans le cas particulier suivant.
Corollaire 1. Soient ψ et y : [a, b] → R + deux fonctions continues et vérifiant
t
∃c ≥ 0, ∀t ∈ [a, b], y(t) ≤ c + ψ (s)y (s) ds.
a
Alors  
t
∀t ∈ [a, b], y(t) ≤ c exp ψ (s) ds .
a

Démonstration. Il s’agit du lemme de Gronwall dans le cas particulier où ϕ est la fonction
constante égale à c, on a donc pour tout t ∈ [a, b]
 t  t    t  s=t  t 
y (t) ≤ c + cψ(s) exp ψ (u) du ds = c − c exp ψ (u) du = c exp ψ (s) ds .
a s s s=a a

Entraı̂nez vous à démontrer ce corollaire directement, sans utiliser le lemme de Gron-
wall général.
Citons enfin une application intéressante du lemme de Gronwall, utile dans les majo-
rations d’erreurs sur les solutions d’équations différentielles.
Corollaire 2. Soit y : [a, b] → Rn une fonction de classe C 1 vérifiant
∃α > 0, ∃β ≥ 0, ∀t ∈ [a, b], y (t) ≤ β + αy (t).
Alors
β  α(t−a) 
∀t ∈ [a, b], y (t) ≤ y (a)eα(t−a) + e −1 .
α
Démonstration. Il suffit d’écrire, pour tout t ∈ [a, b],
 t t

y (t) ≤ y (a) + y (t) − y (a) ≤ y (a) + y (t) dt ≤ y (a) + β (t − a) + α y (s) ds,
a a
puis on applique le lemme de Gronwall et on conclut en intégrant par parties. 

Exercices.

Exercice 1. Soit q : R+ → R une fonction de classe C1 , strictement positive et croissante.


Montrer que toutes les solutions de l’équation différentielle (L) : y + q (t) y = 0 sont
bornées sur R+ .
Solution. On va chercher à utiliser le lemme de Gronwall. Soit y une solution de (L) sur R+.
On a 2yy  + 2q (t)yy  = 0, donc par intégration
 t
 
∀t ∈ R ,+ 2 2
y (t) − y (0) + 2 q (s)y (s)y (s) ds = 0,
0
et en intégrant par parties, on obtient
 t
∀t ∈ R +, y (t)2 + q (t)y (t)2 = K + q (s)y (s)2 ds, K = y (0)2 + q (0)y (0)2 .
0
On a donc  t
q (s)
∀t ∈ R +, q(t)y (t) 2 ≤ K + q (s)y 2 (s) ds.
0 q (s)
4. QUELQUES COMPL ÉMENTS 399

En appliquant le premier corollaire du lemme de Gronwall à la fonction qy 2, on en déduit


 t  
q (s) q (t)
∀t ∈ R +, q(t)y (t) 2 ≤ K exp ds = K ,
0 q (s ) q(0)
donc y (t)2 ≤ K/q (0) pour tout t ∈ R+ , d’où le résultat.

Exercice 2. Soit F : R × Rn → Rn (t, x) → F (t, x) une fonction de classe C 1 et


globalement lipschitzienne en x, i.e
∃C > 0, ∀t ∈ R, ∀x, y ∈ R n, F (t, x) − F (t, y) ≤ C x − y .
On pose G = F + f , où f : R × R est une fonction de classe C 1 qui vérifie f (t, x) ≤ ε
n

pour tout (t, x) ∈ R × Rn , où ε > 0 est fixé.


On suppose que les deux problèmes différentiels
   
x = F (t, x) y = G(t, y )
et
x(0) = x 0 y (0) = y 0
admettent respectivement une solution x et y , définies sur un même intervalle [0, T ] de
R. Montrer
ε  Ct 
∀t ∈ [0, T ], x(t) − y (t) ≤ x0 − y 0  eCt + e −1 .
C

Solution. Posons u = x − y . Pour tout t ∈ [0, T ], on a


u(t) = x(t) − y  (t) = F (t, x(t)) − G(t, y (t)) = F (t, x(t)) − F (t, y (t)) − f (t, y (t))
≤ F (t, x(t)) − F (t, y (t)) + f (t, y (t)) ≤ C u(t) + ε,
et il suffit ensuite d’appliquer le second corollaire du lemme de Gronwall.
Remarque. Ce résultat entraı̂ne qu’une petite variation de la vitesse initiale d’un point
matériel et une petite variation du champ de forces auquel est soumis ce point entraı̂ne
une petite variation de sa trajectoire.

4.3. Principe de majoration a priori


Cette partie est consacrée à un résultat appelé principe de majoration a priori (ou
théorème de sortie de tout compact), portant sur le comportement d’une solution maximale
d’une équation différentielle aux extrémités de son intervalle de définition. Ce résultat est
hors-programme ; il en existe une version faible (voir l’exercice 1 page 401) qui doit être
connue et qui suffit à montrer de nombreux résultats portant sur les solutions maximales.
Préliminaires. Soit F : Ω ⊂ R × Rn → Rn (où Ω est un ouvert de R × Rn) une applica-
tion continue, localement lipschitzienne en la seconde variable. Le théorème de Cauchy-
Lipschitz dit que pour tout (t0 , x0 ) ∈ Ω, il existe un intervalle réel I centré en t0 et une
solution ϕ de y  = F (t, y) vérifiant ϕ(t0 ) = x0 .
Autrement dit, le problème de Cauchy a toujours une solution, définie sur un intervalle
centré en t0 . Nous allons voir qu’on peut trouver un voisinage de (t0 , x 0 ) dans Ω tel que
toutes les solutions du problème de Cauchy pour (t, x) dans ce voisinage soient définies
sur un même intervalle I.
Lemme 1. Soit (t0 , x0) ∈ Ω. Il existe un voisinage V de x0 dans Ω et un intervalle ouvert
I centré en t0 tel que pour tout (t 1 , x) ∈ I × V , il existe une solution ϕ de y  = F (t, y )
définie sur I et vérifiant ϕ(t1 ) = x.
400 6. ÉQUATIONS DIFFÉRENTIELLES

Démonstration. Commençons par un rappel. Pour tout t ∈ R et pour tout α > 0, on pose Iα(t) =
]t − α, t + α[. Pour tout x ∈ Rn et pour tout r > 0, on pose Br (x) = {y ∈ Rn | x − y  < r}.
Nous avons vu, au cours de la démonstration du théorème de Cauchy-Lipschitz page 374 que si
U = Iα (t) × Br (x) est un cylindre de sécurité pour F en (t, x), (i. e. F est lipschitzienne en la
seconde variable sur U et il existe un majorant M > 0 de F sur U tel que αM < r), alors il
existe une solution ϕ de y = F (t, x) définie sur I α (t) et vérifiant ϕ(t) = x.
Soit U ⊂ Ω un voisinage compact de (t0 , x0 ) tel que F soit lipschitzienne en la seconde
variable sur U . Notons M un majorant de F sur U , et choisissons α > 0 et r > 0 tels que
I α(t0 ) × Br (x0) ⊂ U et αM < r (ainsi I α (t0 ) × Br (x 0 ) est un cylindre de sécurité pour F en
(t0 , x0)).
Posons I = Iα/3 (t0) et V = Br/3 (x0 ). Soit (t 1 , x) ∈ I × V . Alors I2α/3 (t 1) × B 2r/3(x) est
un cylindre de sécurité pour F en (t 1, x) (en effet, I2α/3 (t 1) ⊂ I α (t0 ), B 2r/3(x) ⊂ Br (x0 ), et
(2α/3)M < 2r/3 < r), donc il existe une solution ϕ de y = F (t, y) définie sur I 2α/3(t 1 ) et
vérifiant ϕ(t1 ) = x. Comme I ⊂ I 2α/3 (t1 ), on en déduit le résultat. 
Nous sommes maintenant en mesure de démontrer le théorème suivant.

Théorème 1 (Principe de majoration a priori). Soient ]a, b[ un intervalle ouvert


de R, O un ouvert de R n, et F : ]a, b[ × O → R n une fonction continue et localement
lipschitzienne en la seconde variable. Soit ϕ : ]α, β[ → Rn une solution maximale de
y  = F (t, y ).
Alors si β < b (resp. si a < α), pour tout compact K ⊂ O , il existe un voisinage V de
β (resp. de α) dans ]α, β [ tel que ϕ(t) ∈ K pour tout t ∈ V .
Démonstration. Montrons le résultat au voisinage de β (le problème au voisinage de α de traite
de la même manière). Raisonnons par l’absurde et supposons l’existence d’un compact K ⊂ O
et d’une suite (βn) de ]α, β[ convergeant vers β tels que ϕ(β n) ∈ K pour tout n.
Comme K est compact, on peut extraire de (ϕ( βn )) une sous-suite convergente, encore notée
(ϕ(βn)) (par commodité). Soit x0 la limite de (ϕ(β n)). On a x 0 ∈ O car x0 ∈ K ⊂ O.
D’après le lemme précédent, comme (β, x0 ) ∈ ]a, b[×O, il existe un voisinage ]β − ε, β + ε[×V
de (β, x0) dans ]a, b[ × O vérifiant la propriété suivante : pour tout t 1 ∈ ]β − ε, β + ε[, pour tout
x ∈ V , il existe une solution ψ de y = F (t, y) définie sur ]β − ε, β + ε[ et vérifiant ψ (t 1) = x.
Choisissons n ∈ N tel que βn ∈ ]β − ε, β + ε[ et ϕ(βn ) ∈ V . Notons ψ la solution définie sur
]β − ε, β + ε[ de y = F (t, y) telle que ψ(β n ) = ϕ(βn ). L’unicité au problème de Cauchy donnée
par le théorème de Cauchy Lipschitz montre que ψ = ϕ sur ]β − ε, β [. La fonction définie sur
]α, β + ε[ par
t → ϕ(t) si t ∈ ]α, β[ , t → ψ (t) si t ∈ [β, β + ε[
est donc solution de y = F (t, y ) sur ]α, β + ε[. Ceci est absurde, car ϕ est une solution maximale
sur ]α, β[. D’où le résultat. 

Remarque 1. — Dans le cas O = Rn, ce résultat s’écrit : si β < b, alors lim t→β ϕ(t) =
+∞.
— Une fonction localement lipschitzienne F : Rn → R n y → F (y ) peut être vue
comme une fonction de R × R n → Rn (en posant F (t, y) = F (y )). Le théorème
précédent s’énonce alors comme suit : si ϕ : ]a, b[ → Rn est une solution maximale
de y = F (y ), et si b < +∞, alors lim t→b ϕ(t) = +∞.
— Il existe une version faible du principe de majoration a priori dont la preuve est
plus simple et ne repose pas sur le lemme précédent (voir l’exercice 1).
Grâce au principe de majoration a priori, nous pouvons maintenant démontrer le
théorème 1 page 378 (la version faible — exercice 1 — permet aussi de le démontrer).
Corollaire 1. Soit (L) : X = A(t) X + B (t) une équation différentielle linéaire sur
Rn , où A : ]a, b[ → Mn (R) et B : ]a, b[ → Rn sont continues.
Alors les solutions maximales de (L) sont définies sur ]a, b[.
4. QUELQUES COMPL ÉMENTS 401

Démonstration. On définit l’application F : ]a, b[ × Rn → Rn (t, X ) → A(t) X + B (t). Elle est


continue sur ]a, b[ × R n et localement lipschitzienne en X , car F (t, X ) − F (t, Y ) = A(t) (X − Y )
et A est continue.
Soit X : ]α, β[ → Rn une solution maximale de (L). Supposons β < b, et t0 ∈ ]a, b[ étant
fixé, notons M = supt∈[t0 ,β] A(t) et N = supt∈[t0 ,β] B (t) (on a muni Mn (R) de la norme
d’algèbre A = supX =1 AX ). On a
∀t ∈ [t0 , β[, X  (t) ≤ M X (t) + N,
donc d’après le lemme de Gronwall (voir le corollaire 2 page 398), X est bornée au voisinage
de β −. Ceci contredit le principe de majoration a priori, donc β = b. On montrerait de même
α = a. 

Exercices.

 Exercice 1 (Une version faible du principe de majoration a priori). On consi-


dère F : ]a, b[ × Rn → Rn (avec −∞ ≤ a < b ≤ +∞) une fonction continue et localement
lipschitzienne en la seconde variable.
Soit ϕ : ]α, β[ → Rn une solution maximale de y  = F (t, y). Si β < b, montrer que
ϕ n’est pas bornée au voisinage de β (sans utiliser, bien sûr, le principe de majoration a
priori).

Solution. Raisonnons par l’absurde : supposons β < b et ϕ bornée au voisinage de β. Soit


t0 ∈ ]α, β [ et soit M > 0 tel que ϕ(t) ≤ M pour tout t ∈ [t0 , β[.
Comme F est continue sur le compact [t0, β] × Bf (0, M), il existe K > 0 tel que F (t, x) ≤ K
pour tout (t, x) ∈ [t0 , β] × Bf (0, M), et on a ϕ(t) = F (t, ϕ(t)) ≤ K pour tout t ∈ [t0 , β[.
β
Comme de plus ϕ est continue, on en déduit que l’intégrale t 0 ϕ  (t) dt converge absolument,
donc converge, donc ϕ converge en β . Notons x0 = limt→β ϕ(t). Prolongeons ϕ par continuité
en β en posant ϕ(β) = x0 . Comme ϕ (t) = F (t, ϕ(t)), on voit que ϕ converge en β, donc le
prolongement de ϕ est dérivable en β et ϕ (β) = F (β, ϕ(β)).
Ainsi prolongée, ϕ est solution de l’équation différentielle y  = F (t, y ) sur ]α, β]. Or d’après
le théorème de Cauchy-Lipschitz, il existe une fonction ψ définie sur un intervalle ouvert Iβ
centré en β, solution de y = F (t, y), et vérifiant ψ (β ) = x0 = ϕ(β ). Comme ϕ et ψ coı̈ncident
en x 0, l’unicité au problème de Cauchy garantie par le théorème de Cauchy-Lipschitz entraı̂ne
que ϕ = ψ sur ]α, β] ∩ Iβ . Donc la fonction définie sur ]α, β] ∪ Iβ par
t → ϕ(t) si t ∈ ]α, β], t → ψ (t) si t ∈ Iβ ]α, β ],

est solution de y = F (t, y) sur un intervalle contenant strictement ]α, β[. Ceci est absurde car
ϕ est une solution maximale. D’où le résultat.
Remarque. Ce résultat est plus faible que le principe de majoration a priori énoncé à la
page 400, car le fait que ϕ ne soit pas bornée n’entraı̂ne pas forcément ϕ(t) → +∞.
Cette version faible permet aussi de montrer le corollaire 1. En fait, elle suffit à montrer
beaucoup de conséquences du principe de majoration a priori.

Exercice 2. Soit F : R n → Rn une fonction de classe C 1 et bornée sur Rn . Montrer que


le champ de vecteurs F est complet, c’est-à-dire que toute solution maximale de l’équation
différentielle X = F (X ) est définie sur R tout entier.

Solution. Soit ϕ : ]a, b[ → Rn une solution maximale de X  = f (X). Supposons b < +∞. Alors
si t0 ∈ ]a, b[, en notant M un majorant de F sur Rn , on a d’après l’inégalité des accroissements
finis
∀t ∈ [t0 , b[, ϕ(t) ≤ ϕ(t 0) + ϕ(t) − ϕ(t 0) ≤ ϕ(t 0 ) + M (t − t0 ) ≤ M (b − t0).
402 6. ÉQUATIONS DIFFÉRENTIELLES

Ainsi, ϕ est bornée au voisinage de b. En appliquant le principe de majoration a priori (ou le


résultat de l’exercice précédent), on conclut à une absurdité. Donc b = +∞. On montrerait de
même a = −∞.

Exercice 3. Soit F : Rn → Rn une application de classe C1 . On considère le système


différentiel (E) : X  = F (X ). En munissant R n de la métrique euclidienne standard, on
suppose de plus que pour tout X ∈ Rn tel que X  = 1, F (X ) · X < 0.
Soit X0 ∈ R n vérifiant X0 ≤ 1. Montrer qu’il existe une solution ϕ de (E ) telle
que ϕ(0) = X 0 et définie sur un intervalle de la forme ] − α, +∞[ avec α > 0, vérifiant
ϕ(t) < 1 pour tout t > 0.

Solution. Le théorème de Cauchy-Lipschitz nous assure l’existence d’une solution maximale ϕ


de (E ) définie sur un intervalle de la forme ] − α, β[ (α, β > 0) et vérifiant ϕ(0) = X0.
Montrons
∃η ∈ ]0, β[ , ∀t ∈ ]0, η[ , ϕ(t) < 1. ( ∗)

Si X0  < 1 c’est évident par continuité de ϕ en 0. Sinon, X0  = 1, donc dtd [ϕ(t) · ϕ(t)]t=0 =
2ϕ(0) · ϕ(0) = 2F (X0 ) · X0 < 0, c’est-à-dire ddt(ϕ(t)2 )t=0 < 0, d’où (*).
Montrons maintenant que ϕ(t) < 1 pour tout t ∈ ]0, β[. Notons Γ = {t0 ∈ ]0, β[ | ∀t ∈
]0, t0 [ , ϕ(t) < 1}. D’après (*), Γ est non vide, donc γ = sup Γ existe. Supposons γ < β. Alors
d
on a ϕ(γ ) = 1 par continuité de ϕ, et dt [ϕ(t) · ϕ(t)]t=γ = 2ϕ  (γ ) · ϕ(γ) = 2F (ϕ(γ )) · ϕ(γ ) < 0,
d
c’est-à-dire dt (ϕ(t)2 )t=γ < 0. On en déduit qu’il existe δ ∈ ]0, γ[ tel que ϕ(δ ) > ϕ(γ ) = 1.
Ceci est absurde par définition de γ . On a donc γ = β .
Terminons. La fonction ϕ est bornée au voisinage de β, donc d’après le principe de majoration
a priori (ou le résultat de l’exercice 1), on a β = +∞. D’où le résultat.

4.4. Dépendance des solutions vis-à-vis des conditions initiales


Soit F : Ω ⊂ R × Rn → Rn (où Ω est un ouvert de R × Rn) une application continue
et localement lipschitzienne en la seconde variable. Le théorème de Cauchy-Lipschitz dit
que pour tout (t0 , X0) ∈ Ω, il existe une solution maximale ϕt0 ,X 0 de X  = F (t, X ) définie
sur un intervalle ouvert I t0 ,X0 contenant t0 et telle que ϕt 0 ,X0 (t 0) = X0 . En notant

W = {(t0, X 0 )} × It 0,X 0 ⊂ Ω × R,
(t0,X 0 )∈Ω

on peut ainsi définir une application Φ de W dans Rn par Φ(t0, X 0; t) = ϕ t 0,X 0(t). Ainsi,
Φ(t0 , X0 ; . ) est la solution maximale de X  = F (t, X ) passant en X0 au temps t 0.
Un problème nouveau (et intéressant) se pose alors : celui de l’étude de la dépendance
de Φ par rapport à ses paramètres. Ce changement de point de vue s’avère d’ailleurs très
fécond dans la théorie des équations différentielles.
Dans le cas général, on peut montrer que W est ouvert (cela découle essentiellement
du lemme 1 page 399) et que Φ est continue sur W . La démonstration est assez technique.
Nous allons démontrer ces résultats dans certains cas particuliers.
Systèmes différentiels linéaires. On considère le système différentiel homogène sur
Rn (H) : X  = A(t) X où A : I → Mn(R) est continue (et I un intervalle ouvert de R).
Soit (e1 , . . . , en ) la base canonique de R n . Fixons t0 ∈ I . On sait que pour tout i, il
existe une unique solution Xi de (H ) définie sur I et telle que Xi (t0) = ei. Pour tout
4. QUELQUES COMPL ÉMENTS 403

t ∈ I , on note Mt 0 (t) la matrice dont les vecteurs colonnes sont les X i (t), de sorte que
d
Mt = A(t) Mt 0 et Mt 0 (t0 ) = In .
dt 0
Soit X0 ∈ Rn . La fonction ϕ : t → Mt 0 (t) X 0 est donc solution de (H ) et vérifie ϕ(t 0) =
X0. Avec les notations précédentes, on a donc, pour tout t ∈ I , l’égalité ϕ t 0,X0 (t) =
Φ(t0 , X0 ; t) = Mt0 (t)X0 . La matrice Mt 0 (t) est souvent notée R(t, t0 ).
Définition 1. L’application I 2 → Mn (R) (t, t ) → R(t, t ) est appelée la résolvante de
(H). Elle vérifie les propriétés suivantes.
— Pour tout (t0, X0) ∈ I × R n , la solution ϕt0 ,X0 de (H ) vérifiant ϕ t 0,X0 (t 0) = X0
s’écrit ϕt0 ,X0 (t) = R(t, t0 ) X 0 = Φ(t 0, X0; t) ;
— par construction, on a
∀(t, t, t ) ∈ I 3, R(t, t ) ◦ R(t  , t ) = R(t, t) et ∀t ∈ I, R(t, t) = I n.
d
— pour tout t0 ∈ I fixé, on a dt
R(t, t0 ) = A(t) ◦ R(t, t 0).
Proposition 1. Pour tout (t, t) ∈ I 2 , on a R(t, t ) ∈ Gn (R) et R(t, t)−1 = R(t, t).
Démonstration. Il suffit de remarquer que R (t, t) ◦ R(t , t) = R(t, t) = I n . 

Proposition 2. La résolvante R est de classe C 1 sur I2 .


Démonstration. Fixons t0 ∈ I . Par construction, l’application t → R(t, t0 ) est de classe C 1 sur
I. Maintenant, comme R (t, t) = R(t, t0 ) ◦ R(t 0 , t) = R(t, t 0 ) ◦ R(t  , t0 )−1 , on en conclut que R
est de classe C 1 sur I 2 . 

Proposition 3. L’application Φ : (t0 , X 0; t) → Φ(t0 , X 0; t) = ϕ t0 ,X0 (t) est continue sur


I × Rn × I.
Démonstration. Il suffit d’écrire Φ(t0 , X 0 ; t) = R (t, t0 ) X 0 et d’utiliser la proposition précédente.

Remarque 1. — Les colonnes de la matrice R(t, t0) sont des solutions de (H ), donc
det R(t, t0) est un wronskien. En particulier, d’après le résultat de la question b)
de l’exercice 7 page 388, on a
 t   t 
det R(t, t 0) = [det R(t 0, t0 )] exp tr A(s) ds = exp tr A(s) ds .
t0 t0

— Connaissant R(t, t0), la méthode de variation des constantes pour résoudre le


système différentiel (L) : X  = A(t) X + b(t) en terme de résolvante s’exprime
comme suit : on cherche les solutions ϕ de (L) sous la forme ϕ(t) = R(t, t 0)ψ (t),
où ψ : I → Rn est une fonction C 1 encore inconnue. La fonction ϕ est solution de
(L) si et seulement si
 
dϕ dψ
∀t ∈ I, (t) = A(t) ◦ R(t, t0 )ψ (t) + R(t, t0 ) (t) = A(t) ◦ R(t, t0 )ψ (t) + b(t),
dt dt
ce qui équivaut à dψ
dt
(t) = R(t, t 0 )−1 b(t) = R(t0 , t)b(t). La solution ϕ de (L) vérifiant
ϕ(t0 ) = X0 , notée ϕ t0 ,X0, est donc donnée par
  t 
ϕ t0 ,X0 (t) = R(t, t0 ) X 0 + R(t 0 , s)b(s) ds .
t0
On en déduit la proposition qui suit.
Proposition 4. Soit le système différentiel (L) : X  = A(t) X + b(t), où A : I → Mn (R)
et b : I → Rn sont continues. Avec les notations précédentes, la fonction Φ : (t0 , X 0; t) →
ϕt0,X 0 (t) est continue sur I × R n × I.
404 6. ÉQUATIONS DIFFÉRENTIELLES

Cas des équations différentielles de la forme X  = F (X ).


Proposition 5. Soit F : R n → Rn une fonction localement lipschitzienne. Conformé-
ment aux notations précédentes, on note t → Φ(t0 , X0 ; t) la solution maximale ϕ de
X = F (X ) telle que ϕ(t0) = X 0.
Soit (t0, X 0) ∈ R × Rn , et notons ]a, b[ l’intervalle de définition de la solution maximale
t → Φ(t0 , X0; t). Alors pour tout α ∈ R, la solution maximale t → Φ(t 0 + α, X0 , t) est
définie sur ]a + α, b + α[ et on a
∀t ∈ ]a + α, b + α[ , Φ(t0 + α, X0 ; t) = Φ(t 0 , X0 ; t − α).
Démonstration. Posons ϕ : ]a, b[→ R n t → Φ(t 0, X0 ; t) et ψ : ]a + α, b + α[→ Rn t → ϕ(t − α).
On vérifie facilement que ψ  = F (ψ ), et comme ψ (t 0 + α) = ϕ(t0 ) = X0 , on a ψ (t) = Φ(t0 +
α, X 0; t) pour tout t ∈ ]a + α, b + α[.
Or ψ est une solution maximale (sinon ϕ ne serait pas une solution maximale), donc t →
Φ(t0 + α, X0; t) est définie sur ]a + α, b + α[ et
∀t ∈ ]a + α, b + α[ , Φ(t0 + α, X0 ; t) = ψ (t) = ϕ(t − α) = Φ(t0 , X0 ; t − α).

Dans l’exercice qui suit, on montre la continuité de Φ dans un autre cas particulier.
Exercices.
Exercice 1. Soit F : R × Rn → Rn une application de classe C 1 et globalement lipschit-
zienne en la seconde variable, i. e.
∃L > 0, ∀X, Y ∈ R n , ∀t ∈ R, F (t, X ) − F (t, Y ) ≤ L X − Y .
a) Montrer que les solutions maximales de X  = F (t, X ) sont définies sur R.
b) Pour tout t0 ∈ R, pour tout X0 ∈ Rn , on note t → Φ(t0 , X0 ; t) la solution sur R de
X = F (t, X ) qui prend la valeur X0 au point t0 . Montrer que l’application Φ ainsi définie
est continue sur R × Rn × R.
Solution. a) Soit ϕ une solution maximale de X  = F (t, X ) définie sur ]a, b[. Supposons par
exemple b < +∞. Soit c ∈ ]a, b[. Alors pour tout t ∈ [c, b[,
ϕ (t) = F (t, ϕ(t)) ≤ F (t, ϕ(t)) − F (t, 0) + F (t, 0) ≤ L ϕ(t) + M,
où M est un majorant de t → F (t, 0) sur le compact [c, b]. On en conclut, avec le corollaire 2
du lemme de Gronwall (page 398), que ϕ est bornée au voisinage de b. Ceci est absurde d’après
le principe de majoration a priori (ou le résultat de l’exercice 1 page 401). On a donc b = +∞.
On montrerait de même a = −∞.
b) On procède en plusieurs étapes.
(i) Fixons d’abord t0 ∈ R n . Pour tout X ∈ Rn , on note ϕX : R → R t → ϕ(t 0, X; t). On a
∀t, ∈ R, ∀X, Y ∈ R n , ϕX (t) − ϕY (t) = F (t, ϕX (t)) − F (t, ϕ Y (t)) ≤ L ϕX (t) − ϕ Y (t),
donc d’après le corollaire 2 du lemme de Gronwall,
∀t ∈ R, ϕX (t) − ϕ Y (t) ≤ ϕX (t 0) − ϕ Y (t0 ) e L |t−t0 |,
ce qui s’écrit aussi
∀t ∈ R, ∀X, Y ∈ R n , Φ(t0 , X; t) − Φ(t 0, Y ; t) ≤ X − Y  e L|t−t0|. (∗)
En écrivant maintenant
Φ(t 0, X; t) − Φ(t 0 , Y ; t ) ≤ Φ(t 0 , X; t) − Φ(t0 , X; t) + Φ(t 0 , X; t  ) − Φ(t0 , Y, t )
 −t |
≤ Φ(t0 , X; t) − Φ(t0 , X; t) + X − Y  e L|t 0
,
on en conclut que pour tout t0 ∈ R fixé, l’application (X, t) → Φ(t0 , X; t) est continue.
4. QUELQUES COMPL ÉMENTS 405

(ii) Montrons maintenant, pour t1 ∈ R fixé, la continuité de (t, X ) → Φ(t, X ; t 1 ). Soit


(t0 , X 0) ∈ R × R n . Pour tout (t, X) ∈ R × Rn, on a
Φ(t, X ; t1 ) = Φ(t0 , Φ(t, X ; t0 ), t1)
(en effet, pour t, X, t 0 fixés, ces deux fonctions de t 1 sont des solutions de X = F (t, X) qui
coı̈ncident en t1 = t0 , elles sont donc égales d’après le théorème de Cauchy-Lipschitz). On en
conclut avec (*) que
Φ(t, X ; t1 ) − Φ(t0 , X0 ; t1) ≤ Φ(t, X ; t0 ) − X 0  eL|t1−t 0| .
Or X0 = Φ(t0 , X0 ; t0 ) = Φ(t, Φ(t0 , X0 ; t); t0 ), donc toujours d’après (*)
Φ(t, X ; t 0 ) − X0  ≤ X − Φ(t0 , X 0 ; t) eL|t−t 0|.
On en déduit
Φ(t, X ; t 1) − Φ(t 0, X 0; t1 ) ≤ X − Φ(t 0, X 0; t) e L(|t−t0 |+|t1−t0 |).
Ceci entraı̂ne que lorsque (t, X ) → (t 0, X 0), on a Φ(t, X ; t1 ) → Φ(t 0, X 0; t 1), d’où la continuité
voulue.
(iii) Il suffit maintenant d’écrire Φ(s, X; t) = Φ(t0 , Φ(s, X ; t0); t) (pour t0 fixé), pour conclure
que Φ : (s, X ; t) → Φ(s, X ; t) est continue.

Exercice 2. Soit f : R2 → R une fonction de classe C1, globalement lipschitzienne en


la seconde variable, telle que
∀t ∈ R +, f (t, 1) > 0, f (t, −1) < 0.
Montrer qu’il existe une fonction ϕ : R → R solution de (E) : y = f (t, y) vérifiant
|ϕ(t)| < 1 pour tout t ≥ 0 (on pourra utiliser le résultat de l’exercice précédent).

Solution. D’après l’exercice précédent, les solutions maximales de (E) sont définies sur R. De
plus, en notant t → Φ(a, t) la solution maximale de (E ) prenant la valeur a en t = 0, la fonction
Φ : R2 → R ainsi définie est continue.
Remarquons maintenant le fait suivant. Si ϕ : R → R est solution de (E ) et s’il existe
t0 ≥ 0 tel que ϕ(t0 ) > 1, alors ϕ(t) > 1 pour tout t ≥ t0 . En effet, supposons qu’il existe t1 > t0
tel que ϕ(t1 ) ≤ 1. Alors il existe u ∈ ]t0 , t1] tel que ϕ(u) = 1 d’après le théorème des valeurs
intermédiaires. Autrement dit, le fermé ϕ −1 ({1}) a une intersection non vide avec ]t0 , t1 ]. Notons
t2 = inf ϕ −1({1}) ∩ [t0 , t1 ]. On a ϕ(t2 ) = 1 et t2 = t 0 (car ϕ(t0 ) > 1), et ϕ(t) > 1 pour tout
t ∈ ]t0, t2 [, donc ϕ (t2 ) ≤ 0, ce qui est absurde puisque ϕ(t2 ) = f (t 2 , ϕ(t 2)) = f (t 2, 1) > 0. On
montrerait de même que s’il existe t 0 ≥ 0 tel que ϕ(t0 ) < −1, alors ϕ(t) < −1 pour tout t ≥ t0 .
Ceci étant, notons
A+ = {y ∈ [−1, 1] | ∃t ∈ R + , Φ(y, t) > 1} et A− = {y ∈ [−1, 1] | ∃t ∈ R +, Φ(y, t) < −1}.
Comme Φ est continue, on voit facilement que A+ et A− sont des ouverts de [−1, 1]. Par ailleurs,
A+ = ∅. En effet, si ϕ : R → R est la solution de (E) vérifiant ϕ(1) = 1, on ne peut pas avoir
ϕ(0) > 1 (sinon ϕ(t) > 1 sur R + d’après le principe énoncé plus haut), et de même, on ne peut
pas avoir ϕ(0) < −1. Donc ϕ(0) ∈ [−1, 1], et comme ϕ(1) = 1 et ϕ(1) = f (1, 1) > 0, on a
ϕ(t) > 1 sur un voisinage à droite de 1, d’où y = ϕ(0) ∈ A+. De même, A − est non vide.
On a A+ ∩ A− = ∅. En effet, raisonnons par l’absurde en supposant que A + ∩ A− contienne
un élément y . Notons ϕ : t → Φ(y, t ), et soient t1 , t2 > 0 tels que ϕ(t1) > 1 et ϕ(t 2) < −1. On a
ϕ(t1) > 1 donc ϕ(t) > 1 pour tout t > t 1 d’après le principe énoncé plus haut, d’où t2 < t1 . On
montrerait de même t 1 < t 2, ce qui est absurde. Donc A+ ∩ A − = ∅.
Comme [−1, 1] est connexe et que A+ et A − sont deux ouverts disjoints et non vides de
[−1, 1], on ne peut pas avoir A + ∪ A− = [−1, 1]. Il existe donc y 0 ∈ [−1, 1] tel que y0 ∈ A + ∪ A−.
On voit alors que la fonction ϕ = Φ(y0 , . ) vérifie |ϕ(t)| ≤ 1 pour tout t ∈ R+. Si |ϕ(t 0 )| = 1 pour
un t0 ≥ 0, par exemple ϕ(t 0) = 1, le fait que ϕ (t0) = f (t 0 , 1) > 0 montre que sur un voisinage à
406 6. ÉQUATIONS DIFFÉRENTIELLES

droite de t0, ϕ(t) > 1, donc y 0 ∈ A+ ce qui est absurde. Finalement, ϕ est une solution de (E)
sur R qui vérifie |ϕ(t)| < 1 pour tout t ∈ R+ .

Exercice 3. Soit f : R2 → R une fonction de classe C 1 et globalement lipschitzienne en


la seconde variable. On suppose
∃T > 0, ∀(t, y ) ∈ R 2, f (t + T, y) = f (t, y).

On suppose également que (E) : y = f (t, y) admet une solution définie sur R et bornée
sur R. Montrer que (E) admet une solution définie sur R et T -périodique (on pourra
utiliser le résultat de l’exercice 1).
Solution. Les hypothèses de l’énoncé de l’exercice 1 sont satisfaites, donc les solutions maximales
de (E) sont définies sur R. De plus, si on note t → Φ(y, t) la solution maximale de (E) prenant
la valeur y en t = 0, la fonction Φ : R2 → R ainsi définie est continue sur R2 .
On sait qu’il existe une solution ϕ de (E) bornée sur R. Par ailleurs, l’exercice 4 page 377
montre que la suite (ϕ(kT ))k∈N est monotone. De plus ϕ est bornée, donc cette suite est bornée,
donc elle converge. Notons  sa limite.
Remarquons maintenant que pour tout k ∈ N, on a Φ(ϕ(kT ), t) = ϕ(kT + t) pour tout t ∈ R
(en effet, ces fonctions de t sont solutions de (E) et prennent la même valeur en 0). Fixons alors
t ∈ R. Pour tout k ∈ N, on a
Φ(ϕ(kT ), t + T ) = ϕ[(k + 1)T + t] = Φ(ϕ[(k + 1)T ], t),
donc en faisant tendre k vers +∞, on obtient, Φ étant continue Φ(, t + T ) = Φ(, t). Ceci est vrai
indépendamment de t ∈ R. Finalement, la fonction ψ = Φ(, . ) est une solution T -périodique
de (E) sur R.

Exercice 4. Si M = (mi,j ) ∈ M n (R), on note M ≥ 0 si pour tout (i, j), mi,j ≥ 0. Si


Y = (y1, . . . , y n ) ∈ Rn , on note Y ≥ 0 si y i ≥ 0 pour tout i, et on note Y > 0 si y i > 0
pour tout i.
Soit A : R+ → M n (R) une fonction continue vérifiant A(t) ≥ 0 pour tout t ∈ R +.
a) Soit Y : R+ → R n une solution de Y  = A(t) Y vérifiant Y (0) > 0. Montrer que pour
tout t ∈ R+ , Y (t) > 0.
b) Montrer qu’il existe une solution Y de (L) : Y  = −A(t) Y , non nulle, telle que
Y (t) ≥ 0 pour tout t ∈ R + .

Solution. a) Notons (yi) les composantes de Y et (ai,j ) celles de A et supposons le résultat faux.
Alors c = inf{t ∈ R+ | ∃i, yi (t) = 0} existe. Comme Y est continue,  il existe i tel que yi(c) = 0,
et par ailleurs, Y (t) ≥ 0 pour tout t ∈ [0, c]. On a donc yi (t) = nj=1 ai,j (t) y j (t) ≥ 0 pour tout
t ∈ [0, c], donc yi est croissante sur [0, c], donc yi (c) ≥ yi (0) > 0, ce qui est absurde.
b) Donnons l’idée. Lorsque p est grand, et si X vérifie X = A(t) X avec X (0) ≥ 0, alors
Y : t → X (p − t) est solution de (L). Comme X (t) ≥ 0 sur [0, p], on a Y (t) ≥ 0 sur [0, p].
Ensuite, notre but sera de faire tendre p vers +∞ et d’en tirer parti.
Soit R(t, s) la résolvante de (L) (voir la définition 1 page 403), de sorte que pour tout s ∈ R+
fixé, dR
dt (t, s) = −A(t) R (t, s) et R (s, s) = In .
Notons E le vecteur de Rn dont toutes les composantes sont égales à 1, et pour tout p ∈ N,
notons Cp = R(0, p) E. Pour tout p ∈ N, nous notons Y p(t) = R(t, 0) C p = R(t, p) E. On
a Yp (t) ≥ 0 pour tout t ∈ [0, p]. En effet. Considérons l’application ϕ : [0, p] → Rn t →
R(p − t, t) E = Yp (p − t). On a
dϕ dR
∀t ∈ [0, p], (t) = − (p − t, p) E = A(p − t) R(p − t, p) E = A(p − t) ϕ(t),
dt dt
5. PROBLÈMES 407

et comme ϕ(0) = E > 0, on en déduit d’après la question précédente que ϕ(t) ≥ 0 pour tout
t ∈ [0, p], c’est-à-dire Yp (t) ≥ 0 pour tout t ∈ [0, p].
Pour tout p ∈ N, on note ensuite Ap = C p/Cp . La suite (Ap ) est à valeurs dans la sphère
unité de Rn qui est compacte, on peut donc en extraire une sous-suite convergente (A ϕ(p) ).
Notons A sa limite. Posons Y (t) = R(t, 0) A = limp→+∞ R(t, 0) Aϕ(p) . Alors Y est solution de
(L) et on a
1
∀p ∈ N, R(t, 0) A ϕ(p) = Y (t),
C ϕ(p)  ϕ(p)
donc R(t, 0) Aϕ(p) ≥ 0 pour ϕ(p) > t. En passant à la limite lorsque p → +∞ (avec t fixé), on
en déduit Y (t) ≥ 0, et ceci pour tout t ∈ R+ , d’où le résultat.

5. Problèmes
Problème 1. Résoudre sur R+∗ l’équation différentielle (E) : y log t + 2ty2 − y/t = 0.
Étant donné y0 ∈ R, étudier l’existence de solutions y de (E ) définies sur R+∗ telles que
y (1) = y0.

Solution. L’équation différentielle (E) est résolue sur chacun des intervalles ]0, 1[ ou ]1, +∞[.
Nous nous plaçons donc sur l’un de ces intervalles.
La forme de (E) nous invite à poser y = z log t. En remplaçant dans (E), on obtient fa-
cilement (E ) : z  + 2tz 2 = 0. Il s’agit d’une équation de Ricatti. Le théorème de Cauchy-
Lipschitz nous dit qu’en dehors de la solution nulle, aucune solution z de (E ) ne s’annule.
Ainsi, si z n’est pas la solution nulle, (E  ) peut s’écrire z /z2 = −2t, donc par intégration, on a
1/z = −t2 + c, où c est une constante réelle. Autrement dit, les solutions de (E ) sont celles de
la forme z (t) = (t2 − c) −1 ou la fonction nulle.
Ainsi, si y est une solution de (E ) sur R+∗ , en notant I1 = ]0, 1[ et I 2 = ]1, +∞[, on a pour
tout k ∈ {1, 2}
log t
∃c k ∈ R, ∀t ∈ Ik , y(t) = 2 , (∗)
t − ck
ou bien y est identiquement nulle sur Ik .
Si y est du type (*) sur Ik, on doit nécessairement avoir c 1 ≥ 1 ou c1 ≤ 0 (pour k = 1), et
c 2 ≤ 1 (pour k = 2).
(i) De plus, si c 1 = 1 (resp. c2 = 1), on a y(1) = 0 et la dérivée à gauche (resp. à droite) de
y en 1 est yg(1) = (1 − c 1) −1 (resp. yd(1) = (1 − c 2 )−1 ).
(ii) Si c 1 = 1 (resp. c2 = 1), on a facilement y(1) = 1/2.
Traitons maintenant plusieurs cas selon la forme de y sur I1 .
— Si y est identiquement nulle sur I 1 , alors nécessairement y est identiquement nulle sur I2
(d’après (i) et (ii)), donc y est identiquement nulle sur R+∗ .
— Si y est du type (*) avec c 1 = 1, alors nécessairement (d’après (i) et (ii)), y est du type
(*) sur I2 avec c 2 = 1 et l’égalité des dérivées à gauche et à droite de f en 1 donne
c1 = c2 , et vues les conditions imposées à c 1 et c 2 , on a nécessairement c1 = c2 ≤ 0.
— Si y est du type (*) avec c 1 = 1, alors y est nécessairement du type (*) sur I2 avec c2 = 1.
Dans ce cas, on a y (t) = log t/(t2 − 1) pour tout t =  1, y (1) = 1/2, et on vérifie facilement
que y est dérivable en 1.
Finalement, les solutions sur R+∗ de (E) sont
— la fonction nulle ;
— les fonctions de la forme t → log t/(t2 − c), c ≤ 0 ;
— la fonction t → log t/(t2 − 1) si t =  1, 1 → 1/2.
Si y 0 ∈ R est donné, on en déduit qu’il n’y a de solution y sur R+∗ vérifiant y (1) = y 0 si et
seulement si y0 ∈ {0, 1/2}. Si y 0 = 0, il y a une infinité de solutions ; si y0 = 1/2, il n’y a qu’une
seule solution.
408 6. ÉQUATIONS DIFFÉRENTIELLES

Problème 2. Soit f : R → R une fonction continue. On suppose que l’équation


différentielle (E) : y  = f (y) admet une solution ϕ définie sur R et bornée sur R. Montrer
qu’il existe t0 ∈ R tel que f (t0 ) = 0.

Solution. Raisonnons par l’absurde, en supposant f (t) = 0 pour tout t ∈ R. Comme f est
continue, f garde alors un signe constant sur R, par exemple f (t) > 0 pour tout t ∈ R.
Ainsi, pour tout t ∈ R, ϕ (t) = f (ϕ(t)) > 0. Donc ϕ est strictement croissante. De plus
ϕ est bornée, elle converge donc en +∞. Soit  sa limite. En faisant tendre t vers +∞ dans
ϕ (t) = f (ϕ(t)), on en déduit, par continuité de f , que ϕ  converge vers  = f () > 0 en +∞. La
fonction ϕ est continue et vérifie ϕ (t) ∼ , donc par intégration des équivalents, on en déduit
que lorsque t → +∞, ϕ(t) − ϕ(0) ∼  t. Ceci est impossible car  = 0 et ϕ est bornée sur R.
D’où le résultat.

Problème 3 (M éthode d’Euler-Cauchy pour l’intégration numérique d’une


équation différentielle). Soient T > 0, I = [0, T ], et f : I × Rm → Rm une fonction
de classe C1 . On suppose
∃L > 0, ∀t ∈ I, ∀x, y ∈ R m, f (t, x) − f (t, y) ≤ L x − y .
Soit y : I → Rm une solution de l’équation différentielle (E) : y  = f (t, y). On se
propose d’approcher numériquement y par la méthode dite d’Euler-Cauchy. Soit N ∈ N∗ ,
h = T /N et pour tout n ∈ {0, 1, . . . , N }, tn = nh. On définit y0 , y1 , . . . , yN par récurrence
par
y0 = y(0) et yn+1 = y n + h f (tn , yn ).
Pour tout n ∈ {0, 1, . . . , N }, on pose en = y (tn ) − yn . On se propose de majorer en.
Comme f est de classe C1 , y est de classe C 2, et on pose M = sup{y (t), t ∈ I }.
a) On pose εn = [y (tn+1) −y (t n )]−h f (tn , y(tn)). Montrer que pour tout n ∈ {0, 1, . . . , N },
εn ≤ M h2 /2.
b) En déduire
M h Lt n
∀n ∈ {0, 1, . . . , N }, e n ≤ e .
2L

Solution. a) On peut aussi écrire


 t n+1

εn = [y (tn+1) − y (tn )] − hy (t n ) = [y(t) − y  (tn)] dt,
tn
 tn+1
et comme y (t) − y (tn ) ≤ M (t − tn ), on en déduit εn  ≤ tn M (t − tn) dt = M h 2/2.
b) On commence par écrire
en+1 − e n = y (tn+1 ) − y (tn ) − (yn+1 − yn ) = εn + h[f (tn , y(t n )) − f (tn, yn )],
donc
e n+1 − en  ≤ ε n  + hL en  d’où e n+1 ≤ (1 + hL)e n  + εn .
Maintenant, avec les inégalités 1 +hL ≤ ehL et ε n ≤ M h 2 /2, on en déduit en+1 ≤ e hLe n +
M h2/2, et une récurrence sur n donne
  h2
en ≤ e nhL e0  + 1 + ehL + · · · + e(n−1)hL M .
2
Comme e0 = 0, ceci s’écrit aussi
  h2 e nhL − 1 h2 enhL h2 Mh
e n  ≤ 1 + ehL + · · · + e(n−1)hL M = hL M ≤ M = eLtn ,
2 e −1 2 hL 2 2L
et c’est terminé.
5. PROBLÈMES 409

Problème 4. Soit f : R → R une fonction continue, impaire, strictement positive sur


R+∗ . On considère l’équation différentielle (E) : x + f (x) = 0 (équation d’un point
matériel soumis à un champ de forces contraire à sa position).
a) Montrer que les solutions maximales de (E ) sont définies sur R.
b) Soit x une solution de (E ) vérifiant les conditions initiales x(0) = x 0 > 0, x(0) = 0.
Montrer que x est une fonction périodique et exprimer sa période.
Solution. a) Soit x une solution maximale de (E ), et soit ]a, b[ son intervalle de définition. On a
x x + x f (x) = 0, donc par intégration, on obtient l’existence d’une constante K ∈ R telle que
 u
x (t)2
∀t ∈ ]a, b[ , + F (x(t)) = K, où F (u) = f (t) dt. (∗)
2 0
(l’interprétation physique de ce résultat est la conservation de l’énergie cinétique ajoutée à
l’énergie potentielle). La fonction F , primitive d’une fonction impaire, est paire. Par ailleurs,
F  (t) = f (t) ≥ 0 sur R+, et comme F (0) = 0, F est positive sur R +, donc sur R.
Donc x (t)2 /2 = K − F (x(t)) ≤ K est majorée, donc x  est bornée. Si b < +∞, on en
déduit,
 d’après
 l’inégalité des accroissements finis, que x est bornée au voisinage de b. Le vecteur
V = xx est donc borné au voisinage de b. Par ailleurs, V est solution de l’équation différentielle
   −y 
du premier ordre X + g (X) = 0, où g xy = f (x) , le principe de majoration a priori (voir le
théorème 1 page 400 ou l’exercice 1 page 401) entraı̂ne que V n’est pas borné au voisinage de b,
ce qui est absurde. On a donc b = +∞. On montrerait de même que a = −∞.
b) En faisant t = 0 dans (*) on obtient K = F (x0), donc
x (t)2
∀t ∈ R, = F (x 0) − F (x(t)). (∗∗)
2
On a x(0) = −f (x 0 ) < 0, et comme x (0) = 0, il existe ε > 0 tel que x(t) < 0 sur ]0, ε[. Ainsi,
l’ensemble Γ = {t > 0 | ∀u ∈ ]0, t[ , x (u) < 0} est non vide. Notons t0 = sup Γ. On a x (t) < 0
pour tout t ∈ ]0, t0 [, et d’après (**), on a F (x(t)) < F (x 0) pour tout t ∈ ]0, t 0[. Comme F est
croissante sur R+ et paire, on en déduit |x(t)| < x0 pour tout t ∈ ]0, t0 [.
Maintenant, l’équation (**) entraı̂ne
 x0
x (t) du
∀t ∈ ]0, t0 [ ,  = −1 donc t =  (∗∗∗)
2(F (x 0 ) − F (x(t))) x(t) 2( F (x 0 ) − F (u))

(l’intégrale
 est bienconvergente car au voisinage de x− 0 , F (x 0) − F (u) ∼ (x 0 − u)f (x0 ) donc

F (x 0 ) − F (u) ∼ f (x0) x0 − u). Par ailleurs, l’intégrale
 x0
du
T0 = 
−x0 2(F (x0 ) − F (u))
converge (la fonction F est paire), et comme son intégrande est positive et que |x(t)| < x 0 pour
tout t ∈ ]0, t0[, (***) entraı̂ne t < T 0 pour tout t ∈ ]0, t0[, donc t 0 ≤ T 0. Ainsi, t0 est fini. Par
continuité de x on a x (t0 ) = 0, donc F (x(t 0 )) = F (x0 ) d’après (**), et comme F est paire,
strictement croissante sur R+ , ceci entraı̂ne |x(t0 )| = x0 . De plus, x est strictement décroissante
sur ]0, t0[, donc x(t0 ) < x0 , donc la seule possibilité est x(t0 ) = −x0. En faisant maintenant
tendre t vers t 0 par valeurs inférieures dans (***), on en déduit t 0 = T0 .
Considérons maintenant la fonction y : t → −x(t + T0). On a y (0) = −x(t 0 ) = x0, y (0) =

−x (t0) = 0, et comme f est impaire, y est solution de (E). D’après le théorème de Cauchy-
Lipschitz, il y a unicité au problème de Cauchy, et comme x et y ont les mêmes conditions
initiales en 0, on a x = y sur R. En d’autres termes, x(t) = −x(t + T 0) pour tout t ∈ R. On en
déduit x(t) = −x(t + T0) = x(t + 2T 0). Donc x est 2T0 -périodique. La fonction x n’admet pas
de période plus petite, car nous avons montré x(t) < x(0) sur ]0, T0 ] et sur ]T0 , 2T 0[, |x(t)| =
|x(t − T0 )| < x(0), donc finalement, x(t) =  x(0) sur ]0, 2T 0[.
Remarque. On traite de la même manière l’équation du penduleLθ +g sin θ = 0. Partant
θ
d’une vitesse nulle en θ = θ0, le pendule a sa période égale à 2 L/g −θ00 √ dθ .
2(cos θ−cos θ0)
410 6. ÉQUATIONS DIFFÉRENTIELLES

Problème 5. Déterminer les solutions à valeurs réelles définies sur R, deux fois dérivables,
de l’équation différentielle (E) : yy y  = 0.

Solution. Soit ϕ : R → R une fonction deux fois dérivable vérifiant ϕϕϕ = 0. On va montrer
(on s’en doutait) que ϕ = 0, c’est-à-dire que ϕ est une fonction affine.
Pour cela, raisonnons par l’absurde et supposons l’existence de t0 ∈ R tel que ϕ (t 0) = 0.
Quitte à considérer la fonction t → ϕ(t + t0 ), on peut même supposer t 0 = 0, de sorte que
ϕ (0) = 0.
Dans un premier temps, nous montrons l’existence d’un α > 0 vérifiant ϕ (t) = 0 pour tout
t tel que 0 < |t| < α. Si ϕ (0) = 0, c’est évident par continuité de ϕ . Sinon, ϕ (0) = 0, et il suffit
d’écrire ϕ (t) = ϕ(0) + tϕ  (0) + o(t) = t[ϕ (0) + o(1)] pour conclure.
Ensuite, nous montrons l’existence de β > 0 vérifiant ϕ(t) = 0 pour tout t tel que 0 < |t| < β.
Si ϕ(0) = 0, c’est immédiat par continuité de ϕ. Sinon, ϕ(0) = 0. Si ϕ (0) = 0, il suffit d’écrire
2
ϕ(t) = t[ϕ(0) + o(1)], et si ϕ (0) = 0, on écrit ϕ(t) = t2 [ϕ(0) + o(1)] pour conclure.
En posant maintenant γ = inf{α, β }, on a donc ϕ(t)ϕ (t) = 0 pour tout t tel que 0 < |t| < γ.
Comme ϕ est solution de (E), on en déduit ϕ(t) = 0 pour tout t vérifiant 0 < |t| < γ.
Pour montrer ϕ (0) = 0, on pourrait maintenant conclure avec le théorème de Darboux. On
peut s’en tirer autrement, en procédant comme suit. Comme ϕ = 0 sur ]0, γ[, ϕ est constante
sur ]0, γ[. De même, ϕ est constante sur ] − γ, 0[. Par continuité de ϕ  en 0, ϕ  est donc constante
sur ] − γ, γ[, donc ϕ (0) = 0. Ceci est contradictoire, d’où le résultat.

Problème 6. Soit f : R n → R n une fonction de classe C1 telle que f (0) = 0. On suppose


que pour tout x ∈ R n, la matrice jacobienne de f en x, notée f  (x), est symétrique définie
négative.
Montrer que si Y : R → R n est une solution de l’équation différentielle Y  = f (Y ),
alors limt→+∞ Y (t) = 0.

Solution. Soit X ∈ Rn , X = 0. En posant V : t → t Xf (tX), on a V  (t) = tXf  (tX )X < 0


pour tout t ∈ R (car f  (tX ) est définie négative). Ainsi V est strictement décroissante. Comme
V (0) = 0, on en déduit V (1) = t Xf (X ) < 0. En résumé, nous avons montré
∀X ∈ Rn , X = 0, t
Xf (X ) < 0. (∗)
t 2
Ceci étant, posons α : t → Y (t)Y (t) = Y (t) . On a
∀u ∈ R, α  (u) = t Y  (u)Y (u) + t Y (u)Y (u) = 2 tY (u)Y (u) = 2t Y (u) f (Y (u)), (∗∗)

donc d’après (*), α (u) ≤ 0 pour tout u ∈ R. Ainsi, la fonction α est décroissante. Comme elle
est positive, elle admet donc une limite  ∈ R+ en +∞. Il s’agit de montrer  = 0.
Supposons  > 0. Sur R+ , on a  ≤ α(t) = Y (t)2 ≤ α(0), autrement dit, pour t ≥ 0, Y (t)
prend ses valeurs dans le compact K = {X ∈ R n |  ≤ X 2 ≤ α(0)}. D’après (*) et comme
 > 0, on a t Xf (X ) < 0 pour tout X ∈ K , et la compacité de K entraı̂ne γ = sup{t Xf (X ), X ∈
K } < 0. Ainsi, d’après (**), on a α (t) ≤ 2γ < 0 pour tout t ≥ 0, donc α(t) − α(0) ≤ −2γt, ce
qui entraı̂ne limt→+∞ α(t) = −∞. Ceci est impossible puisque α est positive. Donc  = 0, d’où
le résultat.

Problème 7 (R ésolution d’équations matricielles grâce aux équations dif-


férentielles). 1/ Soit A ∈ M n(C) une matrice dont les valeurs propres λ1 , . . . , λn
vérifient (λi) < 0 pour tout i. Si  .  désigne une norme d’algèbre sur Mn (C), montrer
qu’il existe α > 0 et K > 0 tel que etA ≤ Ke−αt pour tout t ≥ 0.
2/ a) Soient A et B ∈ Mn (R). Exprimer la solution Y : R → Mn (R) de l’équation
différentielle (E) : Y  = AY + Y B, telle que Y (0) = C, où C ∈ Mn(R) est une matrice
5. PROBLÈMES 411

donnée.
b) On suppose que pour toute valeur propre λ de A ou de B , (λ) < 0. Montrer que
pour toute matrice C ∈ Mn (R), il existe une unique matrice X ∈ Mn (R) telle que
AX + XB = C , puis exprimer X en fonction de A, B et C .
3/ Soit A ∈ Mn (R). Montrer que l’équation matricielle t AS + SA = −C admet une
solution S ∈ Mn (R) symétrique positive pour toute matrice symétrique positive C si
et seulement si pour toute valeur propre λ de A, on a (λ) < 0. (Indication. Pour la
condition nécessaire, on pourra étudier les variations de la fonction u → t (e uAX )S (e uAX),
où X ∈ Cn ).

Solution. 1/ On sait que l’on peut écrire A = D + N , où D est une matrice diagonalisable
dont les valeurs propres sont λ 1, . . . , λn et N une matrice nilpotente vérifiant DN = N D (voir
le tome Algèbre sur la décomposition de Dunford).
Soit P ∈ Gn (C) telle que D = P −1D 1 P où D1 est la matrice diagonale dont les coefficients
diagonaux sont λ1 , . . . , λn . Pour tout t ∈ R, e tD1 est la matrice diagonale dont les coefficients
diagonaux sont les etλi . En désignant par  . ∞ la norme (ai,j) ∞ = supi,j |ai,j | sur M n(C),
on a etD1 ∞ = supi |etλ i| = e −ct, où c = − supi (λ i ) > 0. Comme toutes les normes sont
équivalentes en dimension finie, on en déduit l’existence de K1 > 0 tel que e tD1  ≤ K 1 e−ct
pour tout t ∈ R. Comme etD = P −1e tD1 P pour tout t ∈ R, on en déduit

∀t ∈ R, etD  ≤ K2 e −ct avec K2 = P −1  · K1 · P .


Maintenant, comme N est nilpotente, on a
t n−1
∀t ∈ R, e tN = I n + tN + · · · + Nn−1 ,
(n − 1)!
et on en déduit que etN  = o(t n ) lorsque t → +∞. Maintenant, D et N commutent, donc
etA = etD+tN = etD etN pour tout t ∈ R+, donc e tA ≤ etD  · e tN  = o(e−ct tn ) lorsque
t → +∞. Comme tn = o(ect/2 ), on a e−ctt n = o(e−ct/2) lorsque t → +∞, donc finalement
etA = o(e −αt ) lorsque t → +∞ avec α = c/2 > 0. D’où le résultat.
2/ a) L’équation différentielle correspondante est linéaire à coefficients constant, donc on sait
déjà qu’une telle solution X existe et est unique. Or on vérifie facilement que Y : t → e tACe tB
convient, c’est donc la solution de (E ) vérifiant Y (0) = C .
b) Existence. En désignant par Y la fonction de R dans Mn (R) trouvée précédemment, on a
Y  = AY + Y B donc par intégration entre 0 et t, on obtient
 t   t 
Y (t) − C = A Y (s) ds + Y (s) ds B. (∗).
0 0

Par ailleurs, les hypothèses sur les valeurs propres des matrices A et B permettent d’affirmer,
grâce au résultat de la question 1/, l’existence de α > 0 et M > 0 telles que etA  ≤ M e−αt et
etB  ≤ M e−αt pour tout t > 0. La forme de Y (t) entraı̂ne donc

∀t ≥ 0, Y (t) ≤ e tA  · C · etB  ≤ Ke −2αt avec K = M 2 C .


 +∞
On en déduit que l’intégrale 0 Y (s) ds converge absolument donc converge, et que Y (t) tend
vers 0 lorsque t → +∞. En faisant t → +∞ dans (*), on en déduit
 +∞  +∞
C = AX + XB avec X = − Y (s) ds = − etA C etB dt.
0 0

Unicité. Pour montrer que X ainsi défini est unique, il suffit de montrer que l’application
Φ : Mn (R) → M n(R) X → AX + XB est injective. Nous venons de montrer dans la partie
existence que Φ est surjective. Comme Φ est un endomorphisme en dimension finie, ceci entraı̂ne
l’injectivité de Φ, d’où le résultat.
412 6. ÉQUATIONS DIFFÉRENTIELLES

3/ Condition nécessaire. Par hypothèse, il existe une matrice symétrique positive S telle que
tAS + SA = −I . Soit X ∈ C n un vecteur non nul, et considérons l’application
n

VX : R → Cn u → t(euA X ) S (euA X ).
On a
∀u ∈ R, VX (u) = t(AeuA X ) S (euA X) + t(e uAX ) S (AeuA X)
= tX t euA (tAS + SA)e uA X = −t (euA X )(euA X ) < 0
(le terme t (e uA X )(euA X) est le carré de la norme hermitienne de euA X qui est non nul, car
X = 0 et e uA est inversible — toute exponentielle de matrice est inversible, l’inverse de e M étant
e−M ).
Ceci étant, soit λ une valeur propre de A et X ∈ Cn un vecteur propre non nul associé. Pour
tout n ∈ N, An X = λn X donc e uAX = e λuX, et en passant au conjugué, e uAX = eλu X, donc
∀u ∈ R, V X (u) = t (eλu X ) S (eλ u X ) = e(λ+λ)u tXSX.
On en déduit VX (0) = (λ + λ)V X(0) = 2(λ)VX (0). Or on a vu plus haut que VX (0) < 0 et
comme S est positive, on a VX (0) = t XSX ≥ 0. Donc nécessairement (λ) < 0, et ceci pour
toute valeur propre λ de A.
Condition suffisante. Supposons que (λ) < 0 pour toute valeur propre λ de A. Comme tA a
les mêmes
 +∞ valeurs propres A, la résultat de la question 1/b) s’applique et montre que la matrice
t t
S = 0 eu A C euA du vérifie tAS + SA = −C . Pour tout u, la matrice S (u) = euA CeuA est
congrue à la matrice C , donc symétrique positive. Comme S est l’intégrale de S (u) sur R +, on
en déduit que S est symétrique. Par ailleurs, pour tout X ∈ Rn, on a
 +∞
t t
XSX = XS(u)X du ≥ 0
0
car XS(u)X ≥ 0 pour tout u. Finalement, S est symétrique positive et vérifie tAS + SA = −C ,
t

d’où le résultat.

Problème 8. Soit A : R → M n(R) une application continue telle que pour tout t ∈ R, la
matrice symétrique B (t) = t A(t) + A(t) est négative. On considère le système différentiel
(L) : Y  = A(t) Y (Y ∈ Rn ).
a) Si Y 1 et Y2 sont deux solutions de (L) sur R, montrer que t Y1 (t)Y2 (t) converge lorsque
t → +∞.
b) Montrer que (L) admet une solution non identiquement nulle Y qui tend vers 0 en
+∞ si et seulement si  t
lim tr(A(s)) ds = −∞.
t→+∞ 0

(Indication. On pourra utiliser le résultat de l’exercice 7 page 388).

Solution. a) Commençons par remarquer que si Y est une solution de (L), alors la fonction
V : t → Y (t)2 = t Y (t) Y (t) vérifie
∀t ∈ R, V (t) = t Y  (t)Y (t) + tY (t)Y  (t) = t Y (t) B (t) Y (t) ≤ 0,
donc V est décroissante, et comme elle est positive, V (t) = Y (t) 2 converge nécessairement
lorsque t → +∞.
Maintenant, si Y1 et Y2 sont deux solutions de (L), on a t Y1 Y2 = 12 (Y1+Y 2 2 −Y1  2 −Y22 ),
donc d’après ce que l’on vient de voir, tY1 Y 2 converge en +∞.
b) On sait que l’ensemble S des solutions de (L) est un espace vectoriel de dimension n. Soit
(Y1 , . . . , Yn) une base de S . Pour tout t ∈ R, on note R(t) la matrice de M n (R) dont les vecteurs
colonnes sont les Yi (t), et on note M (t) = t R(t)R(t). Le coefficient d’indice (i, j ) de M (t)
5. PROBLÈMES 413

est t Yi(t)Y j (t), donc d’après la question précédente, M (t) converge lorsque t → +∞. Notons
M = limt→+∞ M (t).
Comme (Y1 , . . . , Yn) est une base de solutions de S , toute solution de (L) peut s’écrire sous la
forme t → R(t) X0 , où X0 ∈ R n est un vecteur fixé. Comme R (t)X0 2 = t (R(t)X0 )(R(t)X0 ) =
t
X 0M (t)X 0, on voit donc que l’existence d’une solution non nulle de (L) qui tend vers 0 en
+∞ équivaut à l’existence d’un vecteur X0 = 0 tel que t X 0M (t)X0 tend vers 0 en +∞. Comme
M (t) converge vers M , ceci équivaut aussi à l’existence de X0 = 0 tel que t X 0 M X0 = 0.
Comme M est positive (limite de matrices positives), ceci équivaut à det M = 0, et comme
l’application déterminant est continue, ceci équivaut aussi à limt→+∞ det M (t) = 0. Comme
M (t) = t R(t)R(t), ceci s’écrit aussi limt→+∞ det R (t) = 0. On conclut facilement puisque d’après
la question b) de l’exercice 7 page 388,
 t 
∀t ∈ R, det R (t) = wronskien(Y 1, . . . , Yn )(t) = det R(0) · exp tr A(u) du .
0


Problème 9. Soit y : R + → R une solution de l’équation
 +∞différentielle (L) : y + q(t)y =
+
0, où q : R → R est une fonction continue telle que 0 t|q (t)| dt converge.
Montrer que y  converge en +∞.
Solution. La formule de Taylor avec reste intégral donne l’existence de deux constantes réelles
a et b telles que
t  t

∀t ≥ 1, y(t) = at + b + (t − s)y (s) ds = at + b − (t − s)q (s)y (s) ds.
1 1
Ceci entraı̂ne    t  
 y (t)  |b|  y (s) 
∀t ≥ 1,     ds,
 t  ≤ |a| + t + s |q (s)| 
s 
1
donc d’après le lemme de Gronwall (voir le théorème 1 page 397),
  t  t 
y (t)  |b|
∀t ≥ 1,  
 t  ≤ |a| + t + (|a|s + |b|) |q (s)| exp u |q (u)| du ds.
1 s
 +∞
Comme 0 t|q (t)| dt converge, on en déduit facilement que |y (t)/t| est bornée sur [1, +∞[.
En d’autres termes, y (t) = O (t) au voisinage de +∞. Comme
 t  t
∀t ∈ R +, y(t) = y  (0) + y (s) ds = y  (0) − q (s)y (s) ds,
0 0

on en déduit que y (t) converge lorsque t → +∞.

Problème 10. On s’intéresse aux solutions sur R+∗, à valeurs réelles, de l’équation
différentielle  
2   2 1
t y + (a + 1)t y + t + y = 0, (L)
4
où a ≥ 1.
a) Montrer que sur R +∗ , (L) admet au moins une solution de la forme t → tαϕ(t) où
α ∈ R et où ϕ est la somme d’une série entière de rayon infini avec ϕ(0) = 0.
b) Donner le comportement asymptotique en 0+ de la solution générale sur R+∗ de (L).
(Indication. Distinguer deux cas, selon que a2 − 1 soit le carré d’un entier ou non.)
2
c) (Application.) Montrer que l’équation différentielle (E) : t4 y  − 23 t3 y + (1 + t4 ) y = 0
admet une unique solution f sur R+∗ qui vérifie f (t) ∼ t1/6 lorsque t → +∞, et donner
un développement asymptotique à trois termes de f en +∞.
414 6. ÉQUATIONS DIFFÉRENTIELLES

Solution. a)  Nous recherchons une solution f de (L) la forme tα ϕ(t) , où ϕ(t) est la somme d’une
série entière un tn dont le rayon de convergence est infini. Supposons qu’une telle solution f
existe ; on aura
+∞
α
∀t > 0, f(t) = t ϕ(t) = un tn+α. (∗)
n=0
La fonction f est de classe C ∞ sur R+∗, et compte tenu de l’expression de la dérivée d’une
fonction définie par une série entière, on a, pour tout t > 0,
+∞
 +∞
 +∞

f (t) = αtα−1 ϕ(t) + tα ϕ (t) = αt α−1 un tn + tα nun tn−1 = (α + n)un tn+α−1 .
n=0 n=0 n=0

En d’autres
 termes, on peut dériver terme à terme l’expression (*). De même, on montrerait
f  (t) = +∞
n=0 (α + n)(α + n − 1) tn+α−2
. Maintenant, f vérifie l’équation différentielle (L), donc
+∞
 +∞
   +∞

n+α n+α 2 1 n+α
∀t > 0, (α+n)(α+ n−1)un t +(a+1) (α+n)u n t + t + un t = 0.
n=0 n=0
4 n=0
Après division par tα, nous sommes en présence d’une série entière en t dont la somme est nulle
pour tout t > 0. On en déduit que ses coefficients sont nuls (voir la conséquence du corollaire 1
page 249), ce qui s’écrit
 
1
α(α − 1) + (a + 1)α + u0 = 0
4
 
1
(α + 1)α + (a + 1)(α + 1) + u1 = 0
4
 
1
(α + 2)(α + 1) + (a + 1)(α + 2) + u2 + u 0 = 0
4
......
 
1
(α + n)(α + n − 1) + (a + 1)(α + n) + u n + un−2 = 0.
4
Autrement dit, si P est le polynôme P = X (X − 1) + (a + 1)X + 1/4 = X 2 + aX + 1/4, on a
P (α ) u 0 = 0,
P (α + 1) u 1 = 0 et ∀n ≥ 2, P (α + n) un + u n−2 = 0. (∗∗)
√ √
Les racines de P sont α1 = − a2 + 12 a2 − 1 et α2 = − a2 − 12 a2 − 1.
Maintenant, choisissons α = α1 , de sorte que P (α) = 0 et P (α + n) = 0 pour tout n ∈ N∗ .
Partant de u 0 = 1, l’unique suite (u n) vérifiant (**) est donnée par
1
∀n ∈ N, u2n =  n et u2n+1 = 0.
k=1 (α + 2k)
P
 
La série entière u n t n = u2n t2n a un rayon de convergence infini car u2n/u 2n−2 = 1/P (α +
2n) donc limn→+∞ u 2n/u2n−2 = 0 (voir la règle de D’Alembert, page 248). De plus, elle vérifie
par construction la relation de récurrence (**), donc si ϕ désigne sa somme, t → tα ϕ(t) vérifie
l’équation différentielle (L) et ϕ(0) = u0 = 1 = 0.
b) Si a2 − 1 n’est pas le carré d’un entier, alors les deux racines √ α1 et α2 du polynôme P
exhibé plus haut ne diffèrent pas d’un entier (car α1 − α 2 = a2 − 1), et l’opération effectuée
plus haut en remplaçant α par α1 peut être reprise telle quelle en remplaçant α par α2 (car
P (α2) = 0 et P (α 2 + n) = 0 pour tout n ∈ N ∗). Ainsi, nous avons prouvé l’existence de
deux séries entières dont les sommes ϕ et ψ sont non nulles en 0 et telles que t → t α1 ϕ(t) et
t → tα2 ψ (t) sont solutions de (L). Ces deux solutions sont linéairement indépendantes, car si
λ tα1 ϕ(t) + µ tα 2 ψ (t) = 0 pour tout t > 0, alors λ t α 1−α 2ϕ(t) + µ ψ(t) = 0, et en faisant tendre
t vers 0+ , on obtient µψ (0) = µ = 0 (car α2 < α1 ), puis on conclut facilement λ = 0. On sait
par ailleurs que l’ensemble des solutions de (L) sur R +∗ forme un e.v de dimension 2, et on en
conclut que la solution générale de (L) est t → λ tα1 ϕ(t) + µ tα2 ψ (t). Nous connaissons α1 , α2 ,
5. PROBLÈMES 415

et nous savons donner des relations de récurrence permettant de calculer les coefficients de ϕ et
ψ. Ceci nous permet d’obtenir un développement asymptotique de la solution générale de (L).
Traitons maintenant le cas où a2 − 1 = m 2 , avec m ∈ N. Soit Φ : t → tα ϕ(t) la solution
de (L) trouvée à la question précédente. Munis de cette solution de (L), nous allons déterminer
la solution générale de (L) grâce à la méthode d’abaissement de l’ordre décrite à la page 380 :
on cherche la solution générale f de (L) sous la forme f (t) = g (t) Φ(t) , où g est une fonction
de classe C 2 encore inconnue (la fonction Φ est > 0 sur R+∗ donc ne s’y annule pas, car par
construction, tous les coefficients de ϕ sont strictement positifs on nuls). En remplaçant dans
(L), on obtient
 
1
0 = t2 (g Φ + 2g Φ + g Φ) + (a + 1)t (g  Φ + gΦ ) + t2 + (g Φ)
4
   
1  
= g t2 Φ + (a + 1)t Φ + t2 + Φ + t t g Φ + g  (2tΦ + (a + 1)Φ)
4
et comme Φ est solution de (L), on voit que f = g Φ sera solution de (L) si et seulement si
g est solution de l’équation différentielle (L ) : t y  Φ + y(2tΦ + (a + 1)Φ) = 0. Les solutions
de (L) sont les fonctions de la forme t → λ exp(−F (t)), où λ est une constante réelle et F
une primitive de 2Φ /Φ + (a + 1)/t sur R+∗ . On peut prendre pour F la fonction définie par
F (t) = 2 log Φ + (a + 1) log t, donc finalement, la solution générale de (L) est
1 −(a+1) t−2α1 −a−1 t−m−1
t → λ t = λ = λ , λ ∈ R. (∗∗∗)
Φ(t)2 ϕ(t)2 ϕ(t)2
La fonction t → 1/ϕ(t)2 est de classe C ∞ sur R+. On considère son développement limité en 0
jusqu’à un ordre p ≥ m :
1
= a0 + a1 t + a2 t2 + · · · + ap t p + o(tp )
ϕ(t) 2
(on connaı̂t les coefficients de ϕ, on peut donc calculer les coefficients du développement limité de
1/ϕ2), de sorte qu’un développement asymptotique de (***) en 0 + à la constante multiplicative
λ près est
a0 a a a
+ m1 + · · · + m−1 + m + am+1 + · · · + ap tp−m−1 + o(tp−m−1).
tm+1 t t2 t
 
Comme g est solution de (L ), on en déduit par intégration qu’un développement asymptotique
de g est, à une constante multiplicative près,
a0 1 am−1 a p p−m
c− m
−··· − + am log t + am+1 t + · · · + t + o(tp−m ),
m t t p−m
où c est une constante réelle. Maintenant, il ne reste plus qu’à écrire que la solution générale
de (L) est le produit de g par Φ, pour obtenir un développement asymptotique de la solution
générale de (L). (Le point remarquable ici est la présence d’un logarithme dans le développement
asymptotique).
c) Pour étudier le comportement asymptotique en +∞, on va effectuer le changement de variable
u = 1/t pour se ramener en 0. Posons donc z (t) = y (1/t), de sorte que y (t) = z (1/t). Comme
     
 1  1  1  1 2  1
y (t) = − 2 z et y (t) = 4 z + 3z ,
t t t t t t
on voit que y est solution de (E ) sur R+∗ si et seulement si
       
1 8 1 t2 1
∀t > 0, 0 = z  + t z + 1+ z .
t 3 t 4 t
En posant u = 1/t, ceci est équivalent à dire que z est solution de l’équation différentielle
(E) : u2 z  + 83 u z + (u 2 + 14 ) z = 0. On s’est ramené à une équation différentielle du type
précédent avec a = 5/3, et il s’agit de montrer qu’il existe une unique solution g de (E ) sur R+∗
telle que g (u) ∼ u−1/6 au voisinage de 0.
416 6. ÉQUATIONS DIFFÉRENTIELLES

En reprenant les notations précédentes, le polynôme P est ici P = X (X − 1) + 83 X + 14 =


X2 + 53 + 14 , dont les racines sont α1 = −32 et α 2 = − 16 . Ces deux racines ne diffèrent pas d’un
entier, nous sommes donc dans les conditions du premier cas traité à la question précédente.
Nous avions montré qu’une base de solutions de (E ) est constitué des fonctions u → u α1 ϕ(u)
et u → uα2 ψ (u) , où ϕ et ψ sont les sommes de deux séries entières de rayon de convergence
infini, non nulles en 0, dont les coefficients sont déterminés gr âce à la relation de récurrence (**).
Vues les valeurs α1 = −3/2 et α2 = −1/6, on voit qu’il existe une unique  solution g de (E  )
+∞
équivalente à u−1/6 en 0+ , qui est la fonction u → u−1/6 ψ (t) , où ψ (u) = n=0 u n un avec
1
u0 = 1, ∀n ∈ N ∗, u2n = n et u 2n−1 = 0.
k=1 P (α2 + k )
De menus calculs nous donnent u0 = 1, u2 = −3/20 et u4 = 9/1280, donc g(u) = u −1/6(1 −
3 2 9 4 6
20 u + 1280 u + O (u )). Comme f (t) = g(1/t), on en conclut finalement que lorsque t → +∞,
3 −11/6 9 −23/6
f (t) = t1/6 − t + t + O (t−35/6 ).
20 1280
Remarque. L’équation P (α) = 0 de l’exercice s’appelle équation indicielle de l’équation
différentielle (L).
Cet exercice est un cas particulier du théorème de Fuchs portant sur les points singu-
liers réguliers d’une équation différentielle linéaire. Dans le cas des équations différentielles
du type
t n p n(t) y (n) + t n−1 pn−1 (t) y (n−1) + · · · + t p1(t) y  + p0 (t) y = 0, (E )
(0 est singulier pour (E) car le terme dominant s’annule en 0) où les pi(t) sont des
polynômes en t et pn (0) = 0, ce dernier s’exprime comme suit : si l’équation indicielle de
(E), définie par
α(α − 1) · · · (α − n + 1) p n (0) + · · · + α(α − 1) p2 (0) + α p1 (0) + p0 (0) = 0,
est telle que deux de ses racines α1 , . . . , αn ne diffèrent jamais d’un entier, alors une
base des solutions de (E ) est constituée de fonctions de la forme t → tα i ϕi(t) , où ϕi
est la somme d’une série entière dont le rayon de convergence est non nul, et dont les
coefficients peuvent être déterminés par une relation de récurrence obtenue en remplaçant
formellement cette solution dans (E ). Lorsque certains des αi diffèrent d’un entier, c’est
plus délicat, et on voit apparaı̂tre des comportements en log t en voisinage de 0+ .
Les équations différentielles linéaires à coefficients polynomiaux sont appelées équa-
tions holonomes. Parmi ce type d’équations, on peut trouver des solutions avec d’autres
β
types de singularités, qui font apparaı̂tre des comportements en (log t)k tα e c t au voisinage
de l’origine.
ANNEXE A

Théorème de Baire et applications

Cette annexe présente, sous forme d’une série d’exercices, le théorème de Baire, suivi
de plusieurs applications, notamment le théorème de Banach-Steinhaus. Ces notions ne
sont pas au programme des classes de mathématiques spéciales, mais elles constituent
une fort jolie théorie tout à fait accessible. Certains résultats sont surprenants, comme
par exemple
— l’ensemble des points de continuité d’une fonction dérivée est dense (exemple his-
torique dû à Baire lui-même, voir l’exercice 2 page 419) ;
— l’ensemble des fonctions continues nulle part dérivables est dense dans l’ensemble
des fonctions continues. Cette “plaie lamentable” dont se détournait Hermite est
plus étendue que l’on ne pourrait le penser (voir l’exercice 4 page 421) ;
— un endomorphisme bijectif et continu sur un espace de Banach a son inverse continu
(théorème de Banach, voir l’exercice 6 page 423).

Le théorème de Baire

Exercice 0 (Théorème de Baire).

Définition 1. On dit qu’un espace métrique (E, d) est un espace de Baire si toute
intersection dénombrable d’ouverts denses dans E est dense dans E , autrement dit si

(i) pour toute suite d’ouverts (On )n∈N telle que ∀n ∈ N, On = E, ∩n∈N On = E.

1/ a) Montrer que cette définition est équivalente à la suivante :


(ii) Toute réunion dénombrable de fermés d’intérieurs vides de E
est d’intérieur vide dans E.

Définition 2. Soient (E, d) un espace de Baire et A une partie de E.


— On dit que A est un résiduel si elle contient une intersection dénombrable d’ouverts
denses dans E.
— On dit que A est maigre si elle est contenue dans une réunion dénombrable de
fermés de E d’intérieurs vides.

b) Montrer qu’une partie A d’un espace de Baire E est maigre si et seulement si E A


est un résiduel.
2/ (Théorème de Baire.) Montrer que tout espace métrique complet est un espace de
Baire.
3/ (Un lemme utile dans les applications.) Soit (E, d) un espace de Baire et (Fn)n∈N une

suite de fermés de E telle que ∪n∈N Fn = E. Montrer que l’ouvert ∪ n∈N Fn est dense dans
E.
418 A. THÉORÈME DE BAIRE ET APPLICATIONS

Solution. 1/ a) (i) =⇒ (ii). Soit (Fn ) n∈N une suite de fermés d’intérieurs vides. Pour tout n,

l’ouvert On = E F n est dense dans E car E F n = EF n = E. Ainsi, d’après (i),
 
  
On = (E Fn ) = E Fn
n∈N n∈N n∈N

est dense dans E , c’est-à-dire : ∪n∈NFn est d’intérieur vide.


L’implication (ii) =⇒ (i) se traite de la même manière en considérant les fermés Fn =
E On .
b) Soit (F n )n∈N une suite de fermés d’intérieurs vides. Pour tout n ∈ N, posons O n = E F n ,
ouvert dense dans E . On a
   
   
(A ⊂ F n ) ⇐⇒ (E A) ⊃ E Fn = (E Fn ) = On,
n∈N n∈N n∈N n∈N

et on en déduit facilement que A est maigre si et seulement si E A est un résiduel. (Au passage,
remarquons par définition d’un espace de Baire, qu’un ensemble maigre est d’intérieur vide et
qu’un résiduel est dense dans E .)
2/ Soit (E, d) un espace métrique complet et (On ) n∈N une suite d’ouverts denses dans E. Pour
montrer que ∩n∈N On est dense dans E, il faut montrer
 

∀V ouvert non vide de E, V ∩ O n = ∅.
n∈N

Donnons nous donc un ouvert non vide V de E . Par récurrence, on va construire une suite (Bn )
de boules fermées de E telles que
(I) ∀n ∈ N, Bn est une boule fermée de rayon non nul et inférieur à 1/2n .

(II) B0 ⊂ O0 ∩ V et ∀n ∈ N, Bn+1 ⊂ O n+1 ∩ B n.
L’ouvert O0 est dense dans E donc O 0 ∩ V =  ∅. Or O 0 ∩ V est ouvert, il existe donc une boule
ouverte B(x0 , r) ⊂ O 0 ∩ V . Si B 0 est la boule fermée de centre x0 et de rayon inf{r/2, 1}, on a
donc B0 ⊂ O0 ∩ V .
Supposons les boules B0, . . . , Bn construites et vérifiant (I) et (II). L’ouvert O n+1 est dense

dans E , donc On+1 ∩ Bn est un ouvert non vide. Il existe donc une boule ouverte B(x, r ) incluse

dans On+1 ∩ Bn . Si B n+1 désigne le boule fermée de centre x et de rayon inf {r/2, 1/2 n+1} on a

donc Bn+1 ⊂ O n+1 ∩ B n. Ainsi, B n+1 vérifie (I) et (II).
Par construction, (Bn )n∈N est une suite décroissante de fermés non vides de E dont le
diamètre tend vers 0. De plus E est complet, il existe donc x ∈ E tel que ∩n∈NB n = {x} (voir
la proposition 9 de la page 20). Or B0 ⊂ V , donc x ∈ V . D’après (II), on a aussi B n ⊂ On pour
tout n, donc x ∈ On pour tout n. Ainsi, x ∈ ∩ n∈NOn . Finalement, nous avons prouvé que V et
∩n∈N On ont au moins un point commun, d’où le théorème.

3/ Soit G le fermé E(∪n Fn ). Il s’agit de montrer que G est d’intérieur vide.

   ◦
Pour tout n ∈ N, le fermé G ∩ Fn est d’intérieur vide car G ∩ F n ⊂ G ∩ Fn = ∅, donc (E, d)
étant un espace de Baire,
 
 
(G ∩ Fn ) = G ∩ Fn = G ∩ E = G
n∈N n∈N

est d’intérieur vide.


Remarque. On peut également définir un espace de Baire pour un espace topologique
général, avec les mêmes définitions. On peut montrer qu’un espace topologique compact
est un espace de Baire.
APPLICATIONS 419

Applications
Les exercices qui suivent sont indépendants les uns des autres, mais il est nécessaire
d’avoir fait l’exercice 0 pour les traiter.
Exercice 1 (Un e.v.n à base dénombrable n’est pas complet). Prouver, en
utilisant le théorème de Baire, qu’un espace vectoriel normé E admettant une base
dénombrable n’est pas complet.

Solution. Soit (en ) n∈N une base de E . Pour tout entier naturel n, on pose F n = Vect(e0 , . . . , e n).
Le s.e.v Fn est un fermé (car s.e.v de dimension finie), d’intérieur vide car si une boule ouverte
B(x, r) est incluse dans Fn (avec r > 0), alors x ∈ Fn donc B(0, r) = B(x, r) − {x} est inclus
dans Fn , et Fn étant invariant par homothétie, E ⊂ Fn ce qui est absurde.
Supposons maintenant E complet. D’après le théorème de Baire, ∪n∈N F n est d’intérieur vide
dans E, ce qui est absurde car ∪ n∈NFn = E. D’où le résultat.
Remarque. Ce même résultat est prouvé sans utiliser le théorème de Baire à l’exercice 8
de la page 56.

Exercice 2 (Une fonction dériv ée est continue sur un ensemble dense).
1/ Soient (E, d) et (F, δ ) deux espaces métriques. On suppose que (E, d) est complet. On
considère une suite (fn ) d’applications continues de E dans F , convergeant simplement
vers une application f de E dans F .
a) Pour tout ε > 0, pour tout n ∈ N, on pose
Fn,ε = {x ∈ E | ∀p ≥ n, δ(fn(x), f p (x)) ≤ ε}.

Montrer que Ωε = ∪ n∈N Fn,ε est un ouvert dense dans E et que
∀x0 ∈ Ω ε , ∃V voisinage de x0 , ∀x ∈ V, δ(f (x0 ), f (x)) ≤ 3ε.

b) En déduire que l’ensemble des points de continuité de f est un résiduel.


2/ (Application.) Soit f : R → R une application dérivable sur R. Que dire de l’ensemble
des points de continuité de la fonction dérivée f  ?

Solution. 1/ a) Fixons ε > 0 et n ∈ N. Pour p ≥ n, l’ensemble Gp = {x ∈ E | δ (f n (x), fp (x)) ≤


ε} est fermé (car fn et fp sont continues) donc Fn,ε = ∩ p≥nG p est fermé.
Par hypothèse, la suite (fn ) converge simplement, donc ∪n∈N F n,ε = E, ce qui entraı̂ne (voir
la partie 3/ de l’exercice 0) que

Ωε = ∪ n∈NF n,ε
est un ouvert dense dans l’espace complet E .

Ceci étant, soit x0 ∈ Ωε et soit n ∈ N tel que x0 ∈ F n,ε. Comme fn est continue, il existe un

voisinage V de x0 inclus dans F n,ε tel que
∀x ∈ V, δ(f n(x0 ), fn(x)) ≤ ε.
Or V est inclus dans Fn,ε donc
∀x ∈ V, ∀p ≥ n, δ (fn (x), fp(x)) ≤ ε,
donc en faisant tendre n vers l’infini (pour x et n fixés) on obtient δ (fn (x), f(x)) ≤ ε pour tout
x ∈ V . Finalement,
∀x ∈ V, δ(f (x), f (x0 )) ≤ δ (f (x), fn (x)) + δ (fn(x), f(x)) + δ (f n(x 0), f(x 0 )) ≤ ε + ε + ε = 3ε.
420 A. THÉORÈME DE BAIRE ET APPLICATIONS

b) Posons R = ∩ n∈N∗ Ω1/n et montrons que f est continue en tout point de R. Soit x0 ∈ R et
ε > 0. Fixons n ∈ N∗ tel que 1/n ≤ ε/3. Comme x 0 ∈ Ω1/n , d’après le résultat de la question
précédente, il existe un voisinage V de x0 tel que
  3
∀x ∈ V, δ f (x), f(x0 ) ≤ ≤ ε.
n
Ceci suffit à prouver que f est continue en x0 .
L’ensemble des points de continuité de f contient donc R. C’est donc un résiduel, en parti-
culier dense dans E d’après le théorème de Baire.
2/ Pour tout n ∈ N∗ , on considère la fonction
 
f x + n1 − f (x)
fn : R → R x → 1 .
n
La suite (fn ) est une suite de fonctions continues qui converge simplement vers f  sur R. On en
déduit d’après 1/b) que l’ensemble des points de continuité de f  est un résiduel, en particulier
dense dans R puisque R est complet.
Remarque. On sait qu’il existe des fonctions dérivées discontinues sur un ensemble dense
(voir l’exercice 9 page 244).

Exercice 3. a) On considère une application f : R+ → R continue et on suppose que


pour tout x > 0, la suite (f (nx))n∈N converge vers 0. En utilisant le théorème de Baire,
montrer que lim x→+∞ f (x) = 0.
b) Soit Ω ⊂ ]0, +∞[ un ouvert non borné. Montrer
∃x 0 > 0, ∀N ∈ N, ∃n ≥ N, nx 0 ∈ Ω.
Retrouver grâce à ce dernier résultat le résultat du a).

Solution. a) Soit ε > 0. On introduit, pour tout n ∈ N∗ , l’ensemble


Fn = {x ≥ 0 | ∀p ∈ N, p ≥ n, |f (px)| ≤ ε}.
Comme f est continue, Fn est un fermé de R+ , donc de R. L’hypothèse vérifiée par f entraı̂ne

]0, +∞[ ⊂ ∪n∈N∗ Fn , donc d’après le théorème de Baire, il existe n 0 ∈ N∗ tel que F n0 = ∅. Soient
α, β > 0 tels que ]α, β [ ⊂ Fn0 , de sorte que
∀x ∈ ]α, β [, ∀p ≥ n 0 , |f (px)| ≤ ε. (∗)
 
L’équivalence (p + 1)α < pβ ⇐⇒ (p > β −αα ) montre que
α 
∀N ∈ N, N > , ]pα, pβ[ = ]N α, +∞[.
β−α
p≥N

Ceci est en particulier vrai pour un entier N fixé supérieur à n0 et à α/(β − α). Donc d’après
(*), on a |f (x)| ≤ ε pour tout x ≥ N α, d’où le résultat.
b) Fixons n ∈ N∗ et posons Ωn = {x > 0 | ∃p ≥ n, px ∈ Ω}. L’ensemble Ω n est ouvert. Il
est même dense dans R+ . En effet, considérons un intervalle ouvert ]α, β [ inclus dans R+ . En
procédant comme dans la question précédente, on montre que

∃N ≥ n, ]pα, pβ[ = ]N α, +∞[ .
p≥N
 
Comme Ω n’est pas borné, on en déduit ∪p≥N ]pα, pβ [ ∩ Ω =  ∅, donc ]α, β [ ∩ Ω n = ∅.
Ainsi Ωn est un ouvert dense dans R + pour tout n ∈ N∗ . Comme R+ est complet, on en
déduit d’après le théorème de Baire que ∩n∈N∗ Ωn est dense dans R+ . Ce dernier contient donc
au moins un élément x0 > 0, et il est clair qu’un tel réel x0 répond à la question.
APPLICATIONS 421

Résolvons maintenant la question a) à partir de ce dernier résultat. Raisonnons par l’absurde.


Si a) est faux,
∃ε > 0, ∀A > 0, ∃x > A, |f (x)| > ε.
Autrement dit, l’ouvert Ω = {x > 0, |f (x)| > ε} est non borné. Donc
∃x0 > 0, ∀N ∈ N, ∃n ≥ N, nx0 ∈ Ω,
ce qui s’exprime en disant que la suite (f (nx0))n∈N ∗ ne tend pas vers 0. Ceci est absurde, d’où
le résultat.

Exercice 4 (Les fonctions continues nulle part dérivables sont denses).


On note C le R-e.v des fonctions continues de I = [0, 1] dans R, muni de la norme de la
convergence uniforme f ∞ = supt∈I |f (t)|. L’e.v C est un espace de Banach (car fermé de
B(I, R), ensemble des fonctions bornées de I dans R, complet d’après l’exercice 1 page 21).
Pour ε > 0 et n ∈ N, on considère l’ensemble
   
 f (y ) − f (x) 
Uε,n = f ∈ C | ∀x ∈ I, ∃y ∈ I, 0 < |y − x| < ε,   >n .
y−x 

a) Montrer que Uε,n est un ouvert de C.


b) Montrer que Uε,n est dense dans C (indication : pour f ∈ C, pour δ > 0, on considérera
la fonction x → f (x) + δ sin(N x) avec N bien choisi).
c) En déduire que l’ensemble des fonctions de C nulle part dérivables est un résiduel dans
C.
Solution. a) Nous allons montrer que le complémentaire Fε,n de U ε,n dans C est fermé. Pour
cela, on commence par remarquer que
Fε,n = {f ∈ C | ∃x ∈ I, ∀y ∈ I, |y − x| < ε |f (y ) − f (x)| ≤ n|y − x|} .
On considère ensuite une suite (f p ) p∈N de F ε,n qui converge vers f ∈ C. Il s’agit de montrer que
f ∈ Fε,n.
Pour tout p ∈ N, on a fp ∈ F ε,n donc
∃xp ∈ I, ∀y ∈ I, |y − x p | < ε, |f p (y ) − fp (x p)| ≤ n|y − xp |.
La suite (xp ) prend ses valeurs dans le compact I, on peut donc en extraire une sous-suite
sous suite
convergente (xϕ(p) ), dont nous notons x la limite. Soit y ∈ I tel que 0 < |y − x| < ε. Il existe
P ∈ N tel que 0 < |y − xϕ(p) | < ε pour tout p ≥ P . Ainsi,
∀p ≥ P, |fϕ(p) (y ) − f ϕ(p)(xϕ(p))| ≤ n|y − x ϕ(p)|. (∗)
Le caractère
 continu
 de f conjugué au fait que (fp) tende vers f au sens de  .  ∞ montre
que fϕ(p) (xϕ(p) ) tend vers f (x). En faisant tendre p vers l’infini dans (*), on en déduit que
|f (y ) − f (x)| ≤ n|y − x|. Ceci étant vrai pour tout y ∈ I tel que |y − x| < ε, on en déduit que
f ∈ Fε,n et le résultat.
b) Soit f ∈ C et δ > 0. Il s’agit de trouver g ∈ Uε,n tel que f − g ∞ ≤ δ. On va chercher g sous
la forme f (x) + δ sin(N x).
L’uniforme continuité de f sur le compact I est assurée par le théorème de Heine, de sorte
que
δ
∃α ∈ ]0, ε[, ∀(x, y ) ∈ I 2, |x − y| < α, |f (x) − f (y )| < .
4
4π δN
On choisit maintenant N > 2π tel que < α et > n. Posons g (x) = f (x) + δ sin(N x).
N 8π
Soit x ∈ I . Il est clair que
∃y ∈ I, 2π ≤ |N x − N y| ≤ 4π et | sin(N x) − sin(N y)| ≥ 1.
422 A. THÉORÈME DE BAIRE ET APPLICATIONS

On a en particulier
 
2π 4π  f (y ) − f (x) δ N δN
≤ |x − y | ≤ donc |x − y | < α donc   < ·
N N  y−x  4 2π = 8π .
De plus,  
δ sin(N y) − δ sin(N x)  δ δN
 ≥ ≥ ,
 y−x  | y − x| 4π
donc  
 g (y ) − g (x)  δN δN δN
 
 y − x  > 4π − 8π = 8π > n, avec 0 < |y − x| < α < ε.
Donc g ∈ Uε,n , et comme f − g ∞ = δ, on en déduit le résultat.
c) Posons R = ∩n∈N∗ U1/n,n . Soit f ∈ R. Pour tout n, f ∈ U 1/n,n donc si on se donne x ∈ I ,
 
1  f (x) − f (xn )

∀n ∈ N , ∃xn ∈ I, 0 < |x − xn | < et   > n.

n x − xn
 
 f (x n) 
Donc (x n) tend vers x et lim n→+∞  f (xx)−−x n  = +∞, ce qui montre que f n’est pas dérivable
en x. Ceci étant vrai pour tout x ∈ I , f est nulle part dérivable.
Ainsi, tout élément de R est nulle part dérivable. Comme R est une intersection dénombrable
d’ouverts denses dans le complet C, l’ensemble des éléments de C nulle part dérivables est un
résiduel, en particulier dense dans C d’après le théorème de Baire.
Remarque. Une construction explicite d’une application continue nulle part dérivable fait
l’objet de l’exercice 9 page 86.

Exercice 5. Soit ϕ : R → R une application de classe C ∞ vérifiant


∀x ∈ R, ∃n ∈ N, ϕ(n)(x) = 0.
On veut montrer que ϕ est une fonction polynôme sur R.

On pose Fn = {x ∈ R | ϕ (n)(x) = 0} pour tout n ∈ N puis Ω = ∪ n∈NF n et F = RΩ.
a) Soit (xp ) p∈N une suite de points distincts de R tendant vers x ∈ R telle qu’il existe
n0 ∈ N vérifiant ϕ(n 0 )(xp) = 0 pour tout p. Montrer que ϕ(n) (x) = 0 pour tout n ≥ n0 .
b) Montrer que sur toute composante connexe de Ω, ϕ est polynomiale.
c) Montrer que F n’a aucun point isolé.
d) En supposant F = ∅, obtenir une absurdité. Conclure.

Solution. a) Quitte à prendre une sous-suite de (xp ), on peut supposer cette suite strictement
monotone, par exemple strictement croissante. Pour tout p, on a ϕ (n0 )(x p ) = ϕ(n 0 )(x p+1 ) donc
d’après le théorème de Rolle, il existe y p ∈ ]x p , x p+1[ tel que ϕ (n0 +1)(yp ). Ainsi construite, la
suite (yp) est une suite de points distincts tendant vers x et annulant ϕ (n 0+1) . En itérant le
procédé, on peut ainsi construire pour tout n ≥ n 0 une suite de points distincts de R tendant
vers x et annulant ϕ(n) . Par continuité de ϕ(n) , on en déduit ϕ(n) (x) = 0 pour tout n ≥ n0 .
b) Soit ]a, b[ une composante connexe de l’ouvert Ω et soit [c, d] un segment inclus dans ]a, b[.

Soit x0 ∈ ]c, d[. Il existe n ∈ N tel que x 0 ∈ F n donc il existe α > 0 tel que ϕ (n) s’annule sur
]x 0 − α, x 0 + α[. On peut donc trouver un polynôme P tel que ϕ = P sur ]x 0 − α, x0 + α[. Ainsi,
l’ensemble
Γ = {t ∈ ]x 0, d] | ∀x ∈ [x0, t], ϕ(x) = P (x)}
est non vide, donc β = sup Γ existe bien.
Supposons β < d. Comme β ∈ ]c, d[ ⊂ Ω, il existe un voisinage V = ]β − η, β + η [ de β et
un polynôme Q tels que ϕ = Q sur V (même raisonnement que précédemment). Donc P = Q
sur l’ouvert V ∩ ]x 0, β[=
 ∅, donc P et Q sont identiques, donc β + η ∈ Γ. Ceci est absurde, donc
APPLICATIONS 423

β = d, c’est-à-dire ϕ = P sur [x0 , d]. On montrerait de même que ϕ = P sur [c, x0 ]. Ainsi, ϕ = P
sur tout segment de la composante connexe ]a, b[, donc ϕ = P sur ]a, b[ tout entier.
c) Raisonnons par l’absurde et supposons que F admette un point isolé x0 , de sorte qu’il existe
ε > 0 tel que ]x0 − ε, x0 + ε[ ∩ F = {x 0 }. On a ]x0 − ε, x 0[ ⊂ Ω et d’après la question b), il
existe un polynôme P tel que ϕ = P sur ]x0 − ε, x0 [. De même, il existe un polynôme Q tel que
ϕ = Q sur ]x0 , x0 + ε[. Par continuité des dérivées successives de ϕ et de P, Q, on a ∀n ∈ N,
P (n) (x0 ) = ϕ(n)(x0 ) = Q(n) (x 0 ). La formule de Taylor pour les polynômes en x = x0 montre
alors que P et Q sont identiques, donc ϕ = P sur ]x0 − ε, x0 + ε[. Si n = deg(P ), on a donc

]x 0 − ε, x0 + ε[ ⊂ F n+1 ⊂ Ω. Ceci est absurde car x0 ∈ Ω, d’où le résultat.
d) Supposons le fermé F non vide. Par hypothèse, ∪n∈NFn = R, donc F = ∪ n∈N (F ∩ Fn ). Pour
tout n, F ∩ Fn est fermé dans F et F est complet (car fermé de R). D’après le théorème de
Baire, il existe donc n0 ∈ N tel que l’intérieur de F ∩ F n0 (pour la topologie induite par F ) soit
non vide, c’est-à-dire
∃n 0 ∈ N, ∃a, b ∈ R, ]a, b[ ∩ F =
 ∅ et ]a, b[ ∩ F ⊂ F n0 . (∗)
Soit x ∈ ]a, b[ ∩ F . D’après c), on peut trouver une suite de points distincts (xp )p∈N de ]a, b[ ∩ F
tendant vers x. D’après (*), on a ϕ(n0 )(xp ) = 0 pour tout p, donc d’après a), ϕ(n) (x) = 0 pour
tout n ≥ n0 .
Si maintenant x ∈ ]a, b[ ∩ Ω, alors comme ]a, b[ ∩ F = ∅, la composante connexe Ωx de Ω
contenant x possède au moins une extrémité x 0 dans ]a, b[. D’après b), il existe un polynôme
P tel que ϕ = P sur Ωx . Or x0 ∈ F donc pour tout n ≥ n0 , ϕ (n) (x0 ) = P (n) (x 0) = 0, ce qui
montre que deg(P ) < n0 . Donc ϕ(n 0 )(x) = P (n 0)(x) = 0.

En résumé, on a prouvé ϕ(n 0)(x) = 0 pour tout x ∈ ]a, b[. Donc ]a, b[ ⊂ F n 0, ce qui est
absurde car ]a, b[ ∩ F = ∅. Donc F = ∅, donc Ω = R, et d’après b), ϕ est polynomiale sur R
tout entier.

Exercice 6 (Théorème de l’application ouverte ; th éorème de Banach).


Soient E et F deux espaces de Banach.
1/ Théorème de l’application ouverte. Soit T : E → F une application linéaire continue
et surjective. Pour tout r > 0, on note B(r) = {x ∈ E | x < r}.
a) En utilisant le théorème de Baire, montrer que pour tout r > 0, T [B(r )] est un voisinage
de 0 dans F .
b) Soit r > 0. Pour tout n ∈ N, on pose Vn = B(r/2n ). Montrer que T (V1 ) ⊂ T (V0)
(indication : si y1 ∈ T (V1 ), construire deux suites (yn)  de F et (xn ) de E telles que
yn ∈ T (V n), x n ∈ Vn et yn − y n+1 = T (x n) puis considérer x n)
c) En déduire que T est une application ouverte (i. e. pour tout ouvert Ω de E , T (Ω) est
un ouvert de F ).
2/ Théorème de Banach. Si T : E → F est une application linéaire continue et bijective,
montrer que T −1 est continue.

Solution. 1/ a) Pour tout n ∈ N ∗, Fn = T [B(nr)] est un fermé de F . Comme T est surjective,



∪n∈N Fn = F , donc d’après le théorème de Baire, il existe n ∈ N ∗ tel que Fn = ∅, i. e. il existe
une boule ouverte B(x, ρ) de centre x de rayon ρ > 0 incluse dans Fn .
On a alors B(0, ρ) ⊂ F2n. En effet, donnons nous y ∈ B(x, ρ) ⊂ F n. Il existe deux suites (x p)
et (yp ) de B(nr) telles que
x = lim T (xp ) et y = lim T (y p).
p→+∞ p→+∞

Donc y − x = limp→∞ T (y p − x p ), et comme pour tout p, yp − xp ∈ B(2nr), on en déduit


y − x ∈ F2n . Ceci est vrai pour tout y ∈ B(x, ρ) donc B(0, ρ) ⊂ F2n .
424 A. THÉORÈME DE BAIRE ET APPLICATIONS

Finalement, on a
 ρ 1 1
B 0, = B(0, ρ) ⊂ F2n = T [B(r)],
2n 2n 2n
d’où le résultat.
b) Soit y1 ∈ T (V1 ). Comme T (V2 ) est un voisinage de 0 d’après la question précédente, on a
T (V1 ) ⊂ T (V1 ) + T (V2) donc ∃x1 ∈ V1 , ∃y 2 ∈ T (V2 ), y1 − y 2 = T (x1).
En itérant le procédé, on construit ainsi deux suites (yn ) de F et (xn ) de E telles que
∀n ∈ N, y n ∈ T (V n ), xn ∈ Vn
et y n − y n+1 = T (xn).

 Pour tout n, xn ∈ Vn donc xn  < r/2n ,
donc xn  converge, donc E étant complet,
x n converge. On note x = ∞n=1 xn . En écrivant
N  N N
  

∀N ∈ N , T xn = T (xn ) = (y n − yn+1 ) = y 1 − yN+1
n=1 n=1 n=1

et en faisant tendre N vers +∞, on obtient T (x) = y1 (ceci parce que T est continue et que
yN → 0 lorsque N → +∞, ce dernier fait étant aussi une conséquence de la continuité de T ).
Comme pour tout n, xn  < r/2 n, on a x < r. Finalement, y1 ∈ T (V0 ).
c) En combinant les résultats des deux questions précédentes, on s’aperçoit que T [B(r )] est un
voisinage de 0 pour tout r > 0. Donc si U est un voisinage de 0, T (U ) est un voisinage de 0.
Considérons maintenant un ouvert Ω de E et x ∈ Ω. L’ensemble Ω − x est un voisinage de
0, donc T (Ω − x) est un voisinage de 0, donc T (Ω) = T (Ω − x) + T (x) est un voisinage de T (x),
et ceci pour tout x ∈ Ω, donc T (Ω) est ouvert.
2/ Soit U = T −1. D’après le théorème de l’application ouverte, pour tout ouvert Ω de E,
U−1 (Ω) = T (Ω) est un ouvert de E. L’image réciproque par U de tout ouvert est un ouvert, on
en conclut que U = T −1 est continue.

Exercice 7 (Théorème de Banach-Steinhaus et conséquences).


Soit E un espace de Banach, F un e.v.n. On note L c(E, F ) l’e.v des applications linéaires
continues de E dans F , muni de la norme |||f ||| = supx=1 f (x).
1/ Théorème de Banach-Steinhaus. Soit H ⊂ L c(E, F ). Montrer que ou bien (|||f |||) f ∈H
est borné, ou bien il existe x ∈ E tel que supf ∈H f (x) = +∞.
2/ (Une première conséquence.) Soit (fn) une suite d’applications linéaires continues de
E dans F , qui converge simplement vers une fonction f : E → F . Montrer que f est une
application linéaire et continue.
3/ (Une autre conséquence.) Soit E1 un espace de Banach, E2 un e.v.n. Soit B : E1 ×E2 →
F une application bilinéaire dont les applications partielles sont continues, c’est-à-dire
— pour tout x ∈ E1 , l’application B (x, ·) : E2 → F y → B (x, y ) est continue ;
— pour tout y ∈ E2, l’application B(·, y) : E1 → F x → B (x, y ) est continue.
Montrer que B est continue sur E1 × E 2.

Solution. 1/ Pour tout k ∈ N, on pose


Ωk = {x ∈ E | sup f (x) > k}.
f ∈H

L’ensemble Ωk est ouvert. En effet, si x0 ∈ Ωk , il existe f ∈ H tel que f (x0 ) > k. Comme f
est continue, il existe ρ > 0 tel que f (x) > k pour x − x0  < ρ. Donc la boule B (x 0 , ρ) est
contenue dans Ωk .
Si chaque Ωk est dense dans E , alors E étant complet, le théorème de Baire assure que
∩k∈N Ωk est dense dans E, en particulier non vide. Si on choisit x ∈ ∩ k∈NΩk , on a alors
sup f ∈H f (x) = +∞.
APPLICATIONS 425

Sinon, il existe un entier k tel que Ωk ne soit pas dense dans E. En d’autres termes,
∃x0 ∈ E, ∃ρ > 0, B(x0 , ρ) ∩ Ωk = ∅,
de sorte que pour tout x ∈ B(x0 , ρ), supf ∈H f (x) ≤ k. On en déduit
∀x ∈ B(0, ρ), ∀f ∈ H, f (x) = f (x + x0 ) − f (x0 ) ≤ f (x + x0) + f (x 0) ≤ 2k.
Par continuité de chaque f ∈ H , cette inégalité reste vraie sur la boule fermée Bf (0, ρ). Ainsi,
1 2k 2k
∀f ∈ H, ∀x ∈ E, x = 1, f (x) = f (ρx) ≤ donc ∀f ∈ H, |||f ||| ≤ ,
ρ ρ ρ
d’où le résultat.
2/ La fonction f , limite simple de fonctions linéaires, est clairement linéaire.
Il reste à montrer la continuité de f . Appliquons le théorème de Banach-Steinhaus avec H =
{fn , n ∈ N}. La suite (fn ) converge simplement, donc pour tout x ∈ E , sup n∈N fn (x) < +∞.
D’après la question précédente, ceci entraı̂ne l’existence de M > 0 tel que |||f n||| ≤ M pour tout
n ∈ N. Si x = 1, on a donc fn (x) ≤ M pour tout n, donc f (x) ≤ M . Ceci est vrai pour
tout vecteur normé x, donc f est continue.
3/ Considérons l’ensemble
H = {B (·, y), y  = 1} ⊂ Lc (E1 , F ).
D’après les hypothèses, pour tout x ∈ E 1, l’application y → B (x, y) est continue. Ceci entraı̂ne
sup y=1 B(x, y) < +∞, autrement dit on a sup f ∈H f (x) < +∞ pour tout x ∈ E 1. L’e.v.n
E1 est complet, donc d’après le théorème de Banach-Steinhaus,
∃M > 0, ∀y ∈ E2 , y  = 1, |||B (·, y)||| ≤ M.
Autrement dit, pour tout (x, y) ∈ E 1 × E2 vérifiant x = y  = 1, on a B(x, y) ≤ M . Donc
B est continue sur E1 × E 2 (c’est classique à partir de cette dernière inégalité !)

Exercice 8 (Existence de fonctions continues différentes de leur série de


Fourier). On note C2π l’ensemble des fonctions de R dans C qui sont 2π-périodiques et
continues. Pour tout f ∈ C 2π, on note
 π
1
∀p ∈ Z, cp (f ) = f (t)e −ipt dt
2π −π
(coefficients de Fourier de f ). Pour tout n ∈ N∗ , on considère l’application
n

n : C2π → C f → cp (f ).
p=−n

On muni C 2π de la norme f  ∞ = sup−π≤t≤π |f (t)|.


a) Pour tout n ∈ N ∗, montrer que n est une forme linéaire continue et calculer sa norme
|||n ||| = sup f ∞ =1 |n (f )|.
b) Montrer que limn→+∞ |||n ||| = +∞. Conclure avec le théorème de Banach-Steinhaus.

Solution. a) L’application n est clairement une forme linéaire. La relation classique


n  π
ipt sin[(2n + 1)t/2] 1 sin[(2n + 1)t/2]
e = entraı̂ne ∀f ∈ C2π,  n(f ) = f (t) dt,
p=−n
sin(t/2) 2π −π sin(t/2)

donc si f ∞ = 1, on a
  
1 π  sin[(2n + 1)t/2] 
|n (f )| ≤   dt. (∗)
2π  sin(t/2) 
−π
426 A. THÉORÈME DE BAIRE ET APPLICATIONS

Pour tout ε > 0, on définit la fonction de C 2π


Dn (t) sin[(2n + 1)t/2]
fε : R → C t → , où D n(t) = .
|Dn(t)| + ε sin(t/2)
On a fε ∞ ≤ 1 et on montre facilement que
  
1 π  sin[(2n + 1)t/2] 
lim |n (fε )| =  dt.
ε→0
ε>0
2π −π  sin(t/2)
Avec (*), on en déduit   
1 π  sin[(2n + 1)t/2] 
|||n ||| =   dt.
2π  sin(t/2) 
−π

b) L’inégalité | sin(t/2)| ≤ |t/2| pour tout nombre réel t entraı̂ne


     
∗ 1 π sin[(2n + 1)t/2]  2 (2n+1)π/2  sin u 
∀n ∈ N , |||n||| ≥  dt = π  u  du.
π 0  t/2 0
 +∞  nπ 1
 nπ
Comme l’intégrale 0 | sin u/u| du diverge (car (n−1)π | sin u/u| du ≥ nπ (n−1)π | sin u| du =
2
nπ ), on en déduit limn→+∞ |||n ||| = +∞.
Maintenant, C2π est complet (car fermé de B (R, C), e.v des fonctions bornées de R dans C
qui est complet — voir l’exercice 1 page 21), et on peut donc appliquer le théorème de Banach-
Steinhaus qui entraı̂ne l’existence de f ∈ C2π tel que supn∈N | n(f )| = +∞. Autrement dit, la
série de Fourier de f en 0 diverge. La fonction f est donc différente de sa série de Fourier. En
conclusion, il existe des fonctions continues différentes de leur série de Fourier.
Remarque. Un exemple explicite d’une fonction continue 2π -périodique dont la série de
Fourier diverge en 0 fait l’objet de l’exercice 4 page 275.

Autres applications
Le lecteur amusé par les applications précédentes du théorème de Baire pourra s’exer-
cer sur les suivantes :
— il n’existe pas de partition dénombrable de [0, 1] en fermés ;
— il n’existe pas d’application f : R → R continue en tout point de Q et discontinue
en tout point de RQ (par contre, il existe des fonctions continues en tout point
de RQ et discontinues en tout point de Q, voir le problème 11 page 114) ;
— si E 1 et E2 sont deux espaces métriques complets et si une fonction f définie sur
E1 × E 2 a toutes ses applications partielles continues, alors f est continue sur un
ensemble dense.
Remarque sur le théorème de Banach-Steinhaus
Il existe une version plus forte de ce théorème, valable sur des espaces vectoriels appelés
espaces de Fréchet. Ce ne sont pas des espaces normés, mais des espaces métriques dont
la distance n’est pas issue d’une norme.
ANNEXE B

Espaces de Hilbert

Cette annexe présente, sous forme d’un problème, les résultats généraux relatifs aux
espaces de Hilbert. Ce problème est suivi de deux exercices indépendants :
— le premier porte sur la topologie faible dans un espace de Hilbert,
— le second sur les opérateurs compacts et leur théorie spectrale dans les espaces de
Hilbert.
Cet appendice suppose connue la théorie des espaces préhilbertiens.

1. Résultats généraux sur les espaces de Hilbert


Problème 1. Nous n’étudierons ici que les espaces de Hilbert sur R (les propriétés des
espaces de Hilbert sur C sont analogues et se montrent de la même manière).
Rappel. Un espace vectoriel est dit préhilbertien s’il est muni d’un produit scalaire. S’il
est complet pour la norme issue du produit scalaire, on dit que c’est un espace de Hilbert
(ou hilbertien).
Les e.v de dimension finie sont très maniables, mais beaucoup de leurs propriétés
intéressantes ne sont plus vraies en dimension infinie. Les propriétés topologiques des
espaces de Hilbert en font des espaces de dimension infinie très souples, comme nous
allons le voir.
— Dans tout le problème, H désigne un espace de Hilbert de dimension infinie.
— Le produit
 scalaire de deux éléments x, y ∈ H est noté x, y , la norme associée est
x = x, x.
1/ Théorème de projection sur un convexe fermé. Soit C ⊂ H un convexe fermé.
a) Soit x ∈ H . Montrer qu’il existe un unique élément y ∈ C tel que x − y  = d(x, C) =
inf z ∈C x − z .
L’élément y s’appelle la projection orthogonale de x sur C , et on note y = xC .
b) Si x ∈ H , montrer que xC est caractérisé par la propriété suivante :
∀z ∈ C, z − xC , x − xC  ≤ 0.
2/ Orthogonal d’un sous-espace. On rappelle que l’orthogonal d’une partie A de H
est
A⊥ = {y ∈ H | ∀x ∈ A, x, y  = 0}.

a) Si A ⊂ H , montrer que A ⊥ est un s.e.v fermé de H .


b) Soit F un s.e.v fermé de H . Montrer :
(i) F ⊕ F⊥ = H ;
(ii) si x ∈ H , xF = p F (x) où pF est la projection orthogonale sur F ;
(iii) F = F ⊥⊥ .
c) Soit F un s.e.v quelconque de H . Montrer que F = F ⊥⊥ .
d) Montrer qu’un s.e.v F de H est dense si et seulement si F ⊥ = {0}.
3/ Théorème de représentation de Riesz. On note H  le dual topologique de H, i. e.
l’e.v des formes linéaires de H continues.
428 B. ESPACES DE HILBERT

a) Pour tout a ∈ H , on note ϕa la forme linéaire H → R x → a, x. Montrer que


l’application H → H a → ϕa est un isomorphisme.
b) (Application : adjoint d’un endomorphisme). Soit u ∈ Lc (H) un endomorphisme
continu de H . Montrer que
∃!v ∈ L c (E ), ∀(x, y ) ∈ H2 , u(x), y = x, v (y ).
L’endomorphisme v est appelé adjoint de u et est noté u∗ . Montrer que |||u∗ ||| = |||u||| ( où
|||u||| = supx=1 u(x)).
4/ Bases hilbertiennes.
a) Soit (en ) n∈N une suite de vecteurs de H de norme 1, deux à deux orthogonaux. On
note E= Vect(en ) n∈N. Soit v ∈ H . Pour tout n ∈ N, on note λn = v, e n . Montrer que la
série λ n en converge, que sa somme w est la projection orthogonale de v sur E et que
+∞

2
w  = λ 2n ≤ v 2 (inégalité de Bessel).
n=0

b) On dit que H est séparable s’il existe une suite (e n)n∈N de vecteurs de H de norme 1,
deux à deux orthogonaux, telle que Vect(en) n∈N soit dense dans H (on parle alors de base
hilbertienne). 
Montrer que si H est séparable, tout élément v de H s’écrit v = +∞
n=0 λn e n où pour
2 ∞ 2
tout n, λ n = v, en , et que v  = n=0 λn .

Solution. 1/ a) Posons δ = d(x, C ) = inf z ∈C x − z . Il existe une suite (yn ) de C telle que
limn→+∞ x − yn  = δ. En utilisant le fait que H est un espace de Hilbert, nous allons montrer
que ceci entraı̂ne la convergence de (yn ). Comme H est complet, il suffit de montrer que (yn )
est de Cauchy.
Comme la norme  .  est issue d’un produit scalaire, elle vérifie l’identité du parallélogramme,
donc
 
∀p, q ∈ N, (x − yp ) + (x − yq)2 + yp − y q 2 = 2 x − yp  2 + x − yq  2 . (∗)
y +y
Or C est convexe, donc (yp + y q )/2 ∈ C pour tout p, q ∈ N, donc x − p 2 q  ≥ δ, ou encore
(x − y p) + (x − y q)2 ≥ 4δ 2. Avec (*), on en déduit
 
∀p, q ∈ N, yp − yq 2 ≤ 2 (x − yp 2 − δ 2 ) + (x − y q 2 − δ2) ,
et comme x − yn tend vers δ , on voit que (yn ) est bien une suite de Cauchy.
Soit y la limite de (yn ). Comme C est fermé, on a y ∈ C et x − y  = limn→+∞ x − yn  =
δ = d(x, C ).
Il nous reste à montrer que y est unique. Supposons x − z  = δ avec z ∈ C, et définissons
une suite (yn ) de C par yn = y si n est pair, y n = z si n est impair. Cette suite vérifie x − yn  = δ
pour tout n, en particulier x − yn  tend vers δ , donc d’après ce que nous avons prouvé plus
haut, (yn ) converge. Ceci entraı̂ne z = y, d’où l’unicité.
b) Supposons d’abord que pour y ∈ C , on ait
∀z ∈ C, z − y, x − y ≤ 0.
Alors pour tout z ∈ C ,
z −x2 = (z −y )−(x− y )2 = z − y  2 +x−y  2 −2z − y, x − y ≥ z −y 2 +x−y 2 ≥ x −y 2 ,
donc z − x ≥ y − x pour tout z ∈ C . De plus y ∈ C , donc y − x = d(x, C), et d’après la
question précédente, y = xC.
Montrons maintenant que la propriété est vérifiée pour y = xC . Pour tout z ∈ C , on a
x − z  2 ≥ x − xC 2, et en développant x − z 2 = (x − xC ) − (z − xC ) 2, on obtient donc
∀z ∈ C, −2x − xC , z − x C  + z − xC 2 ≥ 0. (∗∗)
1. RÉSULTATS G ÉNÉRAUX SUR LES ESPACES DE HILBERT 429

Il s’agit de se débarrasser du terme en z − xC  2. L’idée est que lorsque z est proche de x C , le


terme z − x C 2 est petit devant l’autre ; on va donc appliquer (∗∗) a un point z qui tend vers
xC. Fixons z0 ∈ C . Comme C est convexe, on a z = λ z0 + (1 − λ)xC ∈ C pour tout λ ∈ [0, 1],
donc en appliquant (**) à z , on tire

∀λ ∈ [0, 1], −2λx − x C , z0 − xC  + λ2 z 0 − x C  2 ≥ 0,

donc
∀λ ∈ ]0, 1], −2x − xC , z0 − x C  + λz 0 − x C 2 ≥ 0,
et en faisant tendre λ vers 0 dans cette dernière inégalité, on obtient −2x − xC , z0 − x C  ≥ 0,
et ceci pour tout z0 ∈ C, d’où le résultat.
2/ a) Le fait que A⊥ soit un s.e.v de H est immédiat. Pour montrer que A⊥ est fermé, on
considère une suite (xn ) de A ⊥ qui converge vers un point x de H. Pour tout y ∈ A, on a
xn, y = 0, et par continuité du produit scalaire (qui provient de l’inégalité de Schwarz), on en
déduit, en faisant n → +∞, que x, y = 0. Ceci est vrai pour tout y ∈ A, donc x ∈ A ⊥ .
b) Montrons (i). Il est clair que F ∩ F ⊥ = {0}, car si x, x = 0, on a x = 0. Montrons maintenant
F ⊕ F ⊥ = H. Soit x ∈ H . Comme F , s.e.v fermé de H, est un convexe fermé de H , on peut
appliquer les résultats de 1/. Donc il existe un unique élément x F ∈ F tel que x − xF  = d(x, F ),
et on a
∀z ∈ F, z − x F , x − xF  ≤ 0.
Comme F est un s.e.v, il est symétrique par rapport à x F ∈ F , et cette dernière inégalité est
donc une égalité. On en tire

∀z ∈ F, z, x − xF  = z − x F , x − xF  − 0 − xF , x − xF  = 0,

donc x − xF ∈ F ⊥. En conclusion, on a x = x F + (x − x F ) avec x F ∈ F et x − xF ∈ F ⊥, donc


x ∈ F ⊕ F ⊥ , et ceci étant vrai pour tout x ∈ H , on a bien H = F ⊕ F ⊥. Du même coup, nous
avons montré que x F = p F (x), qui est l’assertion (ii).
Il nous reste à montrer (iii). Il est clair que F ⊂ F ⊥⊥ , car si x ∈ F , on a x, y  = 0 pour tout
y ∈ F ⊥ , donc x ∈ F ⊥⊥ . Montrons l’inclusion réciproque. Soit x ∈ F ⊥⊥ . Comme F ⊕ F ⊥ = H, il
existe un unique couple (x1 , x2) ∈ F × F ⊥ tel que x = x 1 + x2. Or x est orthogonal à F ⊥, donc
x2 , x = 0 = x2 , x 1 + x 2, x2  = x 2 2, donc x 2 = 0, et finalement x = x 1 ∈ F . On a donc
F ⊥⊥ ⊂ F , d’où (iii).
c) L’ensemble F est un s.e.v fermé de H , donc F = (F ) ⊥⊥. Or

F ⊂F donc (F )⊥ ⊂ F ⊥ donc F ⊥⊥ ⊂ (F ) ⊥⊥ = F . (∗∗∗)

Par ailleurs, F ⊂ F ⊥⊥, et comme un orthogonal est fermé, F ⊂ F ⊥⊥ . Avec (***), on en déduit
F = F ⊥⊥ .
d) Comme F = F ⊥⊥ et que H = F ⊥ ⊕ F ⊥⊥ (car F ⊥ est un s.e.v fermé), on a H = F⊥ + F ,
donc F = H si et seulement si F ⊥ = {0}.
3/ a) Remarquons tout d’abord que ϕa est bien une forme linéaire, et elle est continue d’après
l’inégalité de Schwarz.
L’application H → H a → ϕ a est clairement linéaire. Elle est injective car si ϕa = 0, alors
0 = ϕa (a) = a, a = a2, donc a = 0.
Il nous reste à montrer (c’est plus délicat) qu’elle est surjective. Soit L ∈ H . Si L = 0,
L = ϕ0 et c’est terminé. Sinon, L = 0, et Ker L = L −1 ({0}) est un s.e.v de H , fermé car L est
continue. Donc d’après 2/a), (Ker L) ⊕ (Ker L) ⊥ = H. Si L était nulle sur (Ker L)⊥, alors L
serait nulle sur H ce qui est absurde par hypothèse. Donc il existe a ∈ (Ker L) ⊥ tel que L(a) = 0.
Montrons que (Ker L)⊥ = Vect(a). Soit v ∈ (Ker L)⊥ . On pose
 
L(v ) L(v ) L(v )
w=v− a qui vérifie L(w ) = L(v ) − L a = L(v ) − L(a) = 0,
L(a) L(a) L(a)
430 B. ESPACES DE HILBERT

donc w ∈ Ker L. Par ailleurs, w est combinaison linéaire de deux éléments de (Ker L)⊥, donc
w ∈ (Ker L)⊥ . Finalement, w ∈ (Ker L) ∩ (Ker L)⊥ = {0} donc w = 0, c’est-à-dire v = L (v)
L(a)
a∈
Vect(a). On a donc bien (Ker L) ⊥ = Vect(a).
Ceci étant, posons b = L(a)
a 2
a. Nous allons montrer ϕ b = L.
L(a)
— Pour tout x ∈ Ker L, on a ϕ b(x) = a2
a, x = 0 car a ∈ (Ker L)⊥.
— Si x ∈ (Ker L)⊥, alors il existe λ ∈ R tel que x = λa, donc
L(a)
ϕ b(x) = λ ϕb (a) = λ a, a = λL(a) = L(λa) = L(x).
a 2
Finalement, nous avons montré que L et ϕb coı̈ncident sur les deux espaces supplémentaires
Ker L et (Ker L) ⊥. Donc L = ϕ b , d’où la surjectivité désirée.
b) Soit y ∈ H . L’application x → u(x), y est une forme linéaire continue, donc d’après 3/a),
∃!ay ∈ H, ∀x ∈ H, u(x), y = ay , x.
L’unicité de ay entraı̂ne que y → a y est linéaire. Notons v l’endomorphisme de H défini par
v (y ) = ay , de sorte que
∀x, y ∈ H, u(x), y = x, v(y ).
L’unicité de ay entraı̂ne celle de v. Il nous reste à montrer que v est continu. Pour tout y ∈ H ,
on a
v (y )2 = v (y ), v(y ) = u[v (y)], y ≤ u[v (y )] · y ≤ |||u||| · v (y ) · y ,
donc v (y ) ≤ |||u||| · y. Ceci étant vrai pour tout y ∈ H , on en déduit que v est continu et que
|||v ||| ≤ |||u|||.
Nous avons donc prouvé l’existence et l’unicité de u∗ , et on a |||u∗ ||| ≤ |||u|||. De même,
|||u ||| ≤ |||u∗|||. L’unicité de l’adjoint entraı̂ne u ∗∗ = u, donc |||u||| ≤ |||u∗ |||, et finalement, on a
∗∗

|||u∗ ||| = |||u|||.


4/ a) Pour tout n, notons En = Vect(e k ) 0≤k≤n,s.e.v de dimension finie de H, donc  fermé. La
projection orthogonale de v sur En est pEn (v) = nk=0 λk e k (en effet, si p E n(v) = nk=0 µk ek , on
a v − pEn (v ) ∈ En⊥, donc pour tout k, 0 ≤ k ≤ n, 0 = v − pEn (v ), ek = v, e k  − µk = λk − µ k ).
n
En particulier, p En (v ) 2 = k=0 λk
2
. Comme p E n (v )2 ≤ v 2 2
q (car v 2 = p
2
Eq n (v )2 +
2 2
v − pE n (v ) ), on en déduit que la série λ n converge. Comme  n=p λ n e n = n=p λn , la

série λn en est de Cauchy, donc elle converge car H est complet. Notons w sa somme.
Pour tout n ∈ N, on a v − w, en  = v, e n − w, en  = λn − λ n = 0, donc v − w ∈ E ⊥ . Or
w ∈ E , donc w = pE (v). 
Comme w = lim n→+∞ p En(v ), on a w 2 = limn→+∞ pEn (v )2 = +∞ 2
n=0 λn , et w  =
2

pE (v )2 ≤ v2.


b) Il suffit de remarquer que dans ce cas, E = H et w = PH (v) = v .
Remarque. Les résultats des questions de 1/ (resp. de 2/b)) restent vrais dans un espace
préhilbertien si C est supposé complet (resp. si F est complet, en particulier s’il est de
dimension finie). Les démonstrations sont les mêmes.
S’il existe une famille libre dénombrable de vecteurs (en )n∈N telle que Vect(e n)n∈N
soit dense dans H , alors H est séparable (appliquer le procédé d’orthonormalisation de
Schmidt). La plupart des espaces de Hilbert dans lesquels on travaille sont séparables.

2. Quelques propriétés des espaces de Hilbert


Les deux exercices qui suivent ont pour but de présenter des propriétés intéressantes
des espaces de Hilbert. Ils sont indépendants. Il est nécessaire, pour les traiter, d’avoir
pris connaissance des résultats du problème précédent.
2. QUELQUES PROPRI ÉTÉS DES ESPACES DE HILBERT 431

Exercice 1 (Topologie faible dans un espace de Hilbert). Soit H un espace de


Hilbert dont le produit scalaire est noté  , . Soit (x n) une suite de points de H ,
— on dit que (x n ) converge fortement vers x ∈ H si lim x − xn  = 0 (c’est la
n→+∞
convergence usuelle) ;
— on dit que (xn ) converge faiblement vers x ∈ H si pour tout y ∈ H , lim x n, y =
n→+∞
x, y .
1/ a) Si (xn) converge faiblement vers x, montrer que x est la seule limite faible possible
de (xn ). Si de plus il existe M > 0 tel que xn  ≤ M pour tout n ∈ N, montrer que
x ≤ M .
b) Si (xn ) converge fortement vers x, montrer que (xn ) converge faiblement vers x. La
réciproque est-elle vraie ?
c) Si (xn ) converge faiblement vers x et si limn→+∞ xn = x, montrer que (x n ) converge
fortement vers x.
2/ Compacité faible de la boule unité fermée. Soit (xn ) une suite bornée de H .
On veut montrer qu’il existe une sous-suite de (xn ) qui converge faiblement.
a) Montrer qu’il existe une sous-suite (yn ) = (xϕ(n)) de (xn ) telle que pour tout k ∈ N, la
suite (yn , xk )n∈N soit convergente.
b) Si E est l’adhérence du s.e.v Vect(x n )n∈N , montrer
∃!u ∈ E, ∀v ∈ E, lim yn, v = u, v .
n→+∞

c) En déduire que (yn ) converge faiblement vers u.

Solution. 1/ a) Soit x ∈ H tel que pour tout y ∈ H , limn→+∞ x n, y = x , y. Alors pour tout
y ∈ H , x, y = limn→+∞ xn , y = x , y, donc x − x , y = 0 pour tout y ∈ H. En particulier
x − x, x − x  = x − x 2 = 0, donc x = x .
Si xn  ≤ M pour tout n, alors pour tout n, |x n , x| ≤ x n ·x ≤ M x. Comme (xn, x)
converge vers x, x = x2 , on en déduit x2 ≤ M x donc x ≤ M .
b) C’est évident car pour tout y ∈ H , |xn , y − x, y| = |xn − x, y| ≤ xn − x · y.
Par contre, la réciproque est fausse. Par exemple, dans un espace de Hilbert séparable dont
(e
 n∈N
n ) est une base hilbertienne (par exemple H =  2 , espace des suites réelles (xn ) telles que
x n < +∞), la suite (en )n∈N converge faiblement vers 0 car si y ∈ H , on a y = +∞
2
n=0 y, en  en

et y, e n 2 converge, donc lim n→+∞ y, e n = 0, Bien sûr, il est clair que (en ) ne converge pas
fortement (si c’était le cas, sa limite forte serait sa limite faible donc 0, ce qui est impossible
puisque en  = 1 pour tout n).
c) Il suffit d’écrire xn − x 2 = x2 + xn 2 − 2x, xn.
2/ a) Comme (xn ) est bornée, la suite réelle (xn , x1 )n∈N∗ est bornée. On peut donc extraire
une sous-suite (xϕ 1(n) ) de (xn) telle que la suite (xϕ1(n) , x1 ) n∈N∗ converge.
De même, la suite réelle (xϕ1 (n), x2 )n∈N ∗ est bornée, on peut donc extraire une sous-suite
(xϕ1 ◦ϕ2(n) (n))n∈N∗ de (x ϕ1 (n)) telle que la suite (x ϕ 1◦ϕ 2(n), x2 )n∈N ∗ converge.
En procédant par récurrence, on construit ainsi pour tout k une sous-suite (xϕ1◦···◦ϕ k (n) )n∈N∗
telle que la suite (xϕ1 ◦···◦ϕ k (n) , xk ) n∈N∗ converge.
On définit alors la suite (yn ) par y n = x ϕ1 ◦···◦ϕn (n) pour tout n (procédé d’extraction diago-
nal). Pour n ≥ k , la suite (yn ) est une sous-suite de (xϕ 1◦···◦ϕk (n) )n∈N∗ , donc la suite (yn , xk )n∈N∗
converge, et ceci pour tout k ∈ N ∗ .
b) Pour tout k , la suite (yn , xk )n∈N ∗ converge. Il est donc clair que pour tout v ∈ F =
Vect(x k )k∈N∗ , la suite (yn , v)n∈N ∗ converge.
Montrons que cette propriété reste vraie si v ∈ E = F . Soit v ∈ E . Soit ε > 0 et w ∈ F tel
que v − w  < ε. La suite (yn , w) n∈N∗ converge. Elle est donc de Cauchy, ce qui entraı̂ne
∃N ∈ N, ∀p, q ≥ N, |yp, w − y q , w| < ε.
432 B. ESPACES DE HILBERT

On en déduit
∀p, q ≥ N, |yp , v − yq , v| ≤ |yp , v − w| +|yp, w − yq , w| + |yq , v − w| ≤ M ε + ε + M ε,
où M est un majorant de (xn) (donc de (yn )). Donc (yn , v) n∈N∗ est une suite de Cauchy,
donc elle converge car H est complet.
Pour tout v ∈ E , on note (v) la limite de la suite (yn , v)n∈N∗ . Il est clair que l’application
 : E → R ainsi définie est linéaire. Elle est continue car (yn) est bornée. Le s.e.v E, fermé
dans un complet, est complet. Muni du produit scalaire sur H, c’est donc un espace de Hilbert,
donc d’après le théorème de représentation de Riesz, il existe un unique vecteur u de E tel que
(v ) = u, v pour tout v ∈ E . Ceci s’écrit aussi
∃!u ∈ E, ∀v ∈ E, lim yn , v = u, v .
n→+∞

c) Comme E est un s.e.v fermé de H , on sait que E ⊕ E ⊥ = H. Soit v ∈ H , et soit (v1 , v2 ) ∈ E ×


E⊥ tel que v = v 1 + v2 . Pour tout n, on a yn ∈ E, donc y n, v = yn , v1 . Or lim n→+∞ yn , v1  =
u, v 1 car v 1 ∈ E , et comme u ∈ E, on a u, v 1 = u, v . On en déduit limn→+∞ yn , v = u, v ,
et ceci pour tout v ∈ H , d’où le résultat.
Remarque. A l’aide du théorème de Banach-Steinhaus (voir l’exercice 7 page 424 dans
l’annexe A), on montre facilement que si (xn ) converge faiblement, alors (xn) est bornée
(considérer les formes linéaires fn : H → R x → xn , x).
La convergence faible permet de définir la topologie faible sur H (un ensemble est
faiblement fermé si toute limite faible de points de cet ensemble reste dans cet ensemble).
Si H est séparable, la topologie faible sur la boule unité de H est métrisable (i. e. il existe
une distance d sur H telle que la topologie induite par d est la topologie faible — prendre
+∞ 1
pour d la fonction d(u, v) = n=0 2 n |en, u − v|, où (e n) est une base hilbertienne).
On comprend alors mieux le terme de compacité faible, car dans un espace métrique, la
propriété de Bolzano Weierstrass est équivalente à la propriété de Borel-Lebesgue.

Exercice 2 (Opérateurs compacts dans un espace de Hilbert).


Soit E un R-espace de Banach. On note Lc (E) l’e.v des endomorphismes continus de
E , muni de la norme |||f ||| = supx=1 f (x).
Un endomorphisme (on dit aussi opérateur) f sur E est dit compact si f (B ) est compact
( où B désigne la boule unité fermée de E ). On note Lcomp (E) l’ensemble des opérateurs
compacts de E.
I. Généralités.
1/ a) Montrer Lcomp (E ) ⊂ Lc (E).
b) Soit f ∈ Lc (E). Montrer que les trois assertions suivantes sont équivalentes :
(i) f est un opérateur compact ;
(ii) pour tout ε > 0, on peut recouvrir f (B ) par un nombre fini de boules ouvertes de
rayon ε ;
(iii) pour toute suite (xn ) bornée de E , on peut extraire de la suite [f (xn )] une sous-
suite convergente.
(pour (ii), on pourra utiliser le résultat de l’exercice 2 page 32).
2/ a) Montrer que Lcomp (E ) est un s.e.v fermé de Lc (E).
b) Si f ∈ Lc (E) est de rang fini (i. e. si f (E) est de dimension finie), montrer que
f ∈ Lcomp (E).
II. Opérateurs compacts dans un espace de Hilbert. Dorénavant, E désigne un
R-espace de Hilbert.
1/ a) Si f ∈ L comp (E), montrer que f est limite d’opérateurs continus de rang fini.
2. QUELQUES PROPRI ÉTÉS DES ESPACES DE HILBERT 433

b) Si f ∈ Lcomp (E), montrer que son adjoint f ∗ est un opérateur compact.


c) Si (en ) n∈N est une suite de vecteurs de norme 1, deux-à-deux orthogonaux, et si f ∈
Lcomp (E), montrer limn→+∞ f (e n) = 0.
2/ Valeurs propres d’un opérateur compact dans un Hilbert. Soit f ∈ Lcomp (E). On note
Λ(f ) = {λ ∈ R | ∃x = 0, f (x) = λ x}
(ensemble des valeurs propres de f ).
a) Si λ ∈ Λ(f ) et λ = 0, montrer que le sous-espace propre Ker(f − λ Id) correspondant
est de dimension finie.
b) Montrer que 0 est le seul point d’accumulation possible de Λ(f ). En déduire que Λ(f )
est au plus dénombrable.
III. Opérateurs compacts autoadjoints. Soit f ∈ Lcomp(E) autoadjoint (i. e ∀x, y ∈
E , f (x), y = x, f (y )).
1/ a) Montrer qu’il existe une suite (x n) de vecteurs de E, tous de norme 1, telle que
lim n→+∞ |f (x n ), x n | = |||f |||.
b) Si f = 0, en déduire que f a au moins une valeur propre non nulle.
2/ a) Montrer que deux sous-espaces propres de f pour des valeurs propres distinctes
sont orthogonaux.
b) Soit G le s.e.v engendré (algébriquement) par les vecteurs propres de f associés à des
valeurs propres non nulles. En notant F = G, montrer que F ⊥ = Ker f .
c) On suppose que Λ(f ) n’est pas fini. Montrer qu’il existe une famille orthonormale
dénombrable (en ) n∈N de vecteurs propres, associés à des valeurs propres non nulles, telle
que
+∞

∀x ∈ E, ∃y ∈ Ker f, x= x, e n e n + y.
n=0
Que dire si Λ(f ) est fini ?

Solution. I. 1/ a) Soit f ∈ Lcomp (E ). L’ensemble f (B ) est compact, donc borné, donc f (B )


est borné, donc f est continue, d’où le résultat.
b) Montrons d’abord que les assertions (i) et (ii) sont équivalentes.
— (i) =⇒ (ii) est clair car f (B ) étant compact, on peut toujours, pour tout ε > 0, recouvrir
f (B ) (donc f (B )) par un nombre fini de boules ouvertes de rayon ε.
— (ii) =⇒ (i). Soit ε > 0. On peut recouvrir f (B) par un nombre fini de boules de rayon
ε/2 : f (B ) ⊂ ∪1≤i≤n B(xi , ε/2). Ainsi, f (B ) ⊂ ∪1≤i≤n B(xi , ε), et comme ceci est possible
pour tout ε > 0, f (B ) est précompact (voir l’exercice 2 page 32). De plus f (B), fermé
dans le complet E, est complet. On en déduit que f (B ) est compact.
Montrons maintenant l’équivalence entre les assertions (i) et (iii).
— (i) =⇒ (iii). Soit (xn ) une suite de E, bornée. Soit M > 0 tel que xn  ≤ M pour
tout n. Pour tout n, xn /M ∈ B , donc la suite (f (xn/M)) est à valeurs dans le compact
f (B ). On peut donc en extraire une sous-suite convergente (f (xϕ(n)/M)) n∈N, donc la
suite (f (xϕ(n) ))n∈N est une sous-suite convergente de (f (xn )).
— (iii) =⇒ (i). Soit (y n ) une suite de f (B ). Il s’agit de montrer que l’on peut en extraire
une sous-suite convergente. Pour tout n ∈ N∗, il existe xn ∈ B tel que f (xn )−y n  < 1/n.
D’après (iii), on peut extraire de (f (xn )) une sous-suite convergente [f (xϕ(n))] n∈N, et il
est alors clair que la sous-suite
sous suite (y ϕ(n)) est convergente.
2/ a) Montrons que Lcomp (E ) est un s.e.v de Lc(E).
Si f ∈ Lcomp(E ), et si λ ∈ R, on a (λf )(B) = λ f (B), donc λf ∈ L comp(E).
Soient f, g ∈ Lcomp (E). Soit (x n ) une suite bornée de E . On peut extraire de [f (xn)]n∈N
une sous-suite convergente [f (xϕ(n) )]n∈N . De même, comme (x ϕ(n)) est bornée, on peut extraire
434 B. ESPACES DE HILBERT

de [g (xϕ(n) )]n∈N une sous-suite convergente [g (xϕ◦ψ(n) )]n∈N. Les deux suites [f (x ϕ◦ψ(n))] n∈N et
[g (x ϕ◦ψ(n))] n∈N sont donc convergentes, donc la suite [(f + g)(xϕ◦ψ(n) )] n∈N est convergente.
D’après l’équivalence (iii) ⇐⇒ (i) de la question précédente, on en déduit que f + g est
un opérateur compact.
Montrons maintenant que Lcomp (E) est un fermé de Lc(E). Soit (fn ) une suite de Lcomp (E )
qui converge vers f ∈ Lc (E). Soit ε > 0. Choisissons n ∈ N tel que |||f n − f ||| < ε/2. Comme
fn est un opérateur compact, d’après 1/b)(ii), on peut recouvrir f n(B) par un nombre fini de
boules ouvertes de rayon ε/2. Alors f (B ) est recouvert par les boules de même centre et de
rayon ε, donc f est compact d’après 1/b)(ii).
b) Soit f ∈ Lc (E) de rang fini. Comme f est continue, f (B ) est bornée. Par ailleurs, f (E ) est
fermé (c’est un sous-espace de dimension finie), donc f (B ) ⊂ f (E ). Finalement, f (B ) est un
fermé borné de f (E ). Comme f (E ) est de dimension finie, f (B ) est donc un compact de f (E ),
donc de E puisque f (E ) est fermé.
II. 1/ a) Soit ε > 0. D’après I.1/b)(ii), il existe un nombre fini de boules de rayon ε recouvrant
f (B ) : f (B ) ⊂ ∪1≤i≤n B(yi, ε). Soit F = Vect(y 1, . . . , yn) et p F le projecteur orthogonal sur F .
On a |||f − pF ◦ F ||| < ε. En effet, si x ∈ B , on sait qu’il existe i tel que f (x) − y i  < ε.
De plus, on sait que f (x) − pF ◦ f (x) = infy∈F f (x) − y , et comme yi ∈ F , on en déduit
f (x) − pF ◦ f (x) ≤ f (x) − y i  < ε.
En résumé, pour tout ε > 0, nous avons trouvé un opérateur g continu de rang fini (ici
g = p F ◦ f qui est bien de rang fini car F est de dimension finie), tel que |||g − f ||| < ε. D’où le
résultat.
b) Remarquons déjà que si f ∈ Lc (E) est de rang fini, alors f ∗ aussi. En effet. La relation
x, f ∗ (y ) = f (x), y pour tout x, y ∈ E entraı̂ne f∗ (E ) ⊂ (Ker f )⊥. Par ailleurs, la restriction
de f à (Ker f ) ⊥ est un isomorphisme de (Ker f )⊥ sur f (E), donc (Ker f ) ⊥ est de dimension
finie. Donc f ∗(E) est de dimension finie.
Ceci étant, soit f ∈ Lcomp (E). D’après la question précédente, il existe une suite (fn) d’opéra-
teurs continus de rang fini qui converge vers f . Or pour tout n, |||fn∗ −f ∗ ||| = |||(fn −f )∗ ||| = |||fn−f |||,
donc (f ∗n ) converge vers f ∗ . Or tous les fn∗ sont continus et de rang fini, donc compacts (voir
I.2/b)), et comme Lcomp(E ) est fermé, on en déduit f ∗ ∈ Lcomp (E).
c) Supposons le contraire. Alors il existe α > 0 et une sous-suite de (e n), encore notée (en ), telle
que f (en ) ≥ α pour tout n. Comme f est compact, il existe une sous-suite (eϕ(n) ) de (en ) telle
que (f (eϕ(n) )) converge. Notons x sa limite. On a x ≥ α , et
∀y ∈ E, x, y  = lim f (eϕ(n)), y = lim eϕ(n) , f ∗ (y ),
n→+∞ n→+∞

et le dernier membre est nul en vertu de l’inégalité de Bessel (voir le problème 1, question 4/a)).
On en déduit x, y  = 0 pour tout y ∈ E , donc x = 0. Ceci est absurde car x ≥ α > 0.
2/ a) Supposons que Ker(f − λ Id) soit de dimension infinie. Alors on peut trouver une suite
orthonormale (en) de vecteurs de Ker(f − λ Id) (en partant d’un vecteur e 0 de norme 1, construire
les en par récurrence). Pour tout n ∈ N, on a f (en ) = λ en, donc f (e n) = λ = 0. Ceci est
absurde d’après le résultat de la question précédente.
b) Soit (λn ) une suite de valeurs propres distinctes de f , convergeant vers une valeur λ. Il
s’agit de montrer que λ = 0. Pour tout n ∈ N, on note xn un vecteur propre associé à la valeur
propre λn . Le procédé d’orthonormalisation de Schmidt permet de construire une suite orthonor-
male (y n )n∈N qui vérifie Vect(y0, . . . , yn ) = Vect(x0, . . . , x n) pour tout n. D’après II.1/c), on a
limn→+∞ f (yn) = 0. Pour tout n, on a (f − λ n Id)(yn ) ∈ Vect(x0 , . . . , x n−1) = Vect(y0 , . . . , yn−1 ).
Donc λn yn et (f − λ n Id)(yn) sont orthogonaux, et comme f (y n ) = λn yn + (f − λ n Id)(y n),
on en déduit f (y n) ≥ |λ n| y n  = |λn |. Comme (f (yn)) tend vers 0, on en déduit que (λ n )
tend vers 0. Ainsi, 0 est le seul point d’accumulation possible de Λ(f ). D’après le théorème de
Weierstrass, on en déduit que pour tout n, Λ(f ) ∩ [1/n, n] est fini, donc
  
1

Λ(f ) ∩ R+∗ = Λ(f ) ∩ ,n

n
n∈N
2. QUELQUES PROPRI ÉTÉS DES ESPACES DE HILBERT 435

est au plus dénombrable. On montrerait de même que R−∗ ∩ Λ(f ) est au plus dénombrable.
Donc Λ(f ) est au plus dénombrable.
III. 1/ a) Montrons déjà
sup |f (x), y| = |||f |||. (∗)
x=y=1
Si x = y  = 1, on a |f (x), y| ≤ f (x) y  ≤ |||f |||.
Par définition de la norme intrinsèque |||f |||, il existe une suite (xn) de vecteurs de norme 1
qui vérifie limn→+∞ f (xn ) = |||f |||. En posant yn = f (xn )/f (xn ) pour tout n, on a yn  = 1
et f (xn ), yn  = f (x n), donc limn→+∞ f (x n ), yn  = |||f |||, d’où (*).
Ceci étant, posons M = sup x=1 |f (x), x| et montrons M = |||f |||. Tout d’abord, il est clair
que pour tout vecteur x de norme 1, |f (x), x| ≤ f (x) x = f (x) ≤ |||f |||, donc M ≤ |||f |||.
Montrons l’inégalité réciproque. Si x = y  = 1, le caractère autoadjoint de f permet
d’écrire, pour ε ∈ {−1, 1}, f (x + εy ), x + εy  = f (x), x + f (y ), y + 2εf (x), y donc
4 f (x), y = f (x + y ), x + y  − f (x − y ), x − y 
ce qui entraı̂ne
4 |f (x), y| ≤ |f (x + y ), x + y| + |f (x − y ), x − y|.
Or, par normalisation on a |f (x + y ), x + y | ≤ M x + y 2 et |f (x − y ), x − y| ≤ M x − y 2 ,
donc finalement
   
4 |f (x), y| ≤ M x + y 2 + x − y 2 = 2M x 2 + y 2 = 4M.
Finalement, nous avons montré |f (x), y| ≤ M dès que x = y  = 1. Avec (*), on en déduit
|||f ||| ≤ M .
Nous venons de montrer |||f ||| = supx=1 |f (x), x|, on conclut facilement qu’il existe une
suite (xn ) de vecteurs normés telle que f (x n), xn  converge vers |||f |||.
b) Il est clair que l’on peut trouver une sous-suite de la suite (x n) construite précédemment,
encore notée (xn ), et α ∈ {−1, 1} tels que lim n→+∞ f (xn ), xn  = α |||f |||.
Comme f est compact, on peut en extraire de (xn ) une sous-suite, encore notée (x n), telle
que (f (xn )) converge. Soit y sa limite. Posons λ = α |||f |||. Pour tout n, on a
f (xn) − λ x n 2 = f (xn ) 2 + λ2 x n2 − 2λx n, f(xn) ≤ 2λ 2 − 2λf (xn), x n ,
donc limn→+∞ f (xn ) − λ xn = 0. Comme y − λ xn = (y − f (x n)) + (f (xn ) − λ xn), on en déduit
que lim n→+∞ y − λ x n = 0. De plus λ = α |||f |||  = 0, on a donc limn→+∞ xn = y/λ, et comme
xn = 1 pour tout n, on a y = 0.
Le fait que limn→+∞ f (x n) − λ x n = 0 permet alors d’affirmer f (y) = λ y, donc λ est valeur
propre non nulle de f .
2/ a) C’est comme en dimension finie. Si f (x) = λx et f (y) = µy avec λ = µ, alors
λx, y = f (x), y = x, f (y ) = µx, y ,
donc x, y = 0.
b) Montrons que G⊥ ⊂ Ker f . L’ensemble G⊥ est stable par f . En effet, considérons x ∈ G⊥.
Pour tout vecteur propre y de f associé à une valeur propre λ =  0, on a f (x), y = x, f (y ) =
λx, y = 0, donc par linéarité f (x), y pour tout y ∈ G, c’est-à-dire x ∈ G⊥ .
La restriction g de f à G ⊥ est donc un endomorphisme (qui reste bien sûr compact et
autoadjoint) de G⊥ , qui est un espace de Hilbert (car fermé dans E ). Comme G ∩ G⊥ = {0}, g
n’a aucune valeur propre non nulle. Donc d’après III.1/b), g = 0, autrement dit, f (x) = 0 pour
tout x ∈ G⊥. Donc G⊥ ⊂ Ker f .
Montrons maintenant l’inclusion réciproque. Soit x ∈ Ker f , soit y un vecteur propre de f
associé à une valeur propre λ = 0. Alors
0 = f (x), y = x, f (y ) = λ x, y ,
donc x, y = 0, et par linéarité, x ∈ G⊥ .
Donc Ker f = G ⊥. Or G⊥ = (G)⊥ = F ⊥ , d’où le résultat.
436 B. ESPACES DE HILBERT

c) Si Λ(f ) n’est pas fini, d’après II.2/b), Λ(f ) est dénombrable. Notons (λn) n∈N les éléments
non nuls de Λ(f ). Pour tout n, le sous-espace propre E λn = Ker(f − λ n Id) est de dimension
finie, et on note (en,1 , . . . , e n,pn ) une base orthonormale de Eλn . Alors (e n) = (en,i ) n∈N est une
1≤i≤p n
base orthonormale de G (d’après III.2/a)). Tout élément de F = G s’écrit donc sous la forme

x = +∞ 0 x, e n  en et comme F ⊕ F ⊥ = E, on en déduit le résultat d’après III.2/b).
Si Λ(f ) est fini, en notant Λ(f ){0} = {λ1 , . . . , λ p}, G = Eλ 1 ⊕ · · · ⊕ Eλp est de dimension
finie, donc F = G, donc
Eλ1 ⊕ · · · ⊕ Eλp ⊕ Ker f = F ⊕ F ⊥ = E,
la somme directe étant orthogonale.
Remarque. Rappelons que la boule unité d’un e.v.n n’est compacte qu’en dimension finie
(théorème de Riesz, voir l’exercice 9 page 56). L’opérateur IdE n’est donc compact qu’en
dimension finie.
ANNEXE C

Théorème des nombres premiers

Cette annexe propose une preuve du théorème des nombres premiers, qui exprime que
π (x), le nombre de nombres premiers inférieurs à x, vérifie
x
π (x) ∼ , x → +∞ π (x) = Card {p premier, p ≤ x}.
log x
Ce célèbre résultat, conjecturé à la fin du dix-huitième siècle, résista aux mathématiciens
jusqu’à la fin du dix-neuvième siècle. 
La preuve que nous proposons est classique et utilise la fonction ζ (s) = +∞ n=1 1/n
s

dans le domaine complexe. Nous utilisons une démonstration qui n’emploie pas la théorie
des fonctions analytiques (qui est le cadre standard de cette preuve) et qui nous est donc
accessible à partir du programme des classes préparatoires.
La preuve fait l’objet du problème 3. Elle est précédée de deux problèmes qui four-
nissent les ingrédients nécessaires à la démonstration : on démontre d’abord l’absence de
zéro de ζ (s) sur la ligne (s) = 1, puis on montre la formule de Perron, qui nous permet
d’exploiter l’expression de ζ (s) en fonction des nombres premiers.
La dernière partie de cette annexe présente une histoire de la preuve du théorème des
nombres premiers.

1. Préliminaires
Problème 1 (Propriét és de la fonction ζ (s)). 1/ (La fonction ζ (s)) On considère
la fonction Zêta de Riemann, définie pour la variable complexe s par la série
+∞
 1
ζ (s) = , s ∈ D = {s ∈ C | (s) > 1}
n=1
ns

a) Montrer que cette série est bien définie pour s ∈ D et que ζ (s) est continue sur D.
Montrer de plus que pour s = σ + it ∈ D (σ, t ∈ R), on a |ζ (s)| ≤ σ/(σ − 1).
b) Pour tout N ∈ N∗ , montrer que
N  +∞
1 1/2 − {x} N 1−s N −s
∀s ∈ D, ζ(s) − = s dx + − , (∗)
n=1
ns N xs+1 s−1 2
où {x} = x − [x] désigne la partie fractionnaire de x.
c) Montrer que ζ (s) est prolongeable en une fonction continue sur D {1}, où D = {s ∈
C | (s) ≥ 1}, et qu’il existe une fonction η (s) continue sur D tout entier tel que
1
∀s ∈ D{1}, ζ(s) = η (s) + .
s−1

d) Pour une fonction s → f (s) de la variable complexe s = σ + iτ (σ, τ ∈ R), on désigne


par f (s) la dérivée en x = σ de la fonction x → f (x + iτ ) (nous n’aurons pas besoin du
cadre, généralement utilisé, de la dérivée par rapport à la variable complexe s de f (s)).
Montrer que ζ (s) existe et est continue sur D{1} et que la fonction η (s) est même
438 C. THÉOR ÈME DES NOMBRES PREMIERS

définie et continue sur D tout entier.


2/ a) En désignant par pk le k -ième nombre premier, montrer l’identité d’Euler
 
 1 
+∞ ∞ n
∀s ∈ D, ζ(s) = on rappelle que = lim .
k=1
1 − p−s
k k=1
n→+∞
k=1

b) En déduire que pour tout s ∈ D , on a ζ (s) = 0.


c) On considère la série définie pour s ∈ D par
+∞
 
Λ(n) log p si ∃k ∈ N∗ : n = p k avec p premier
Z (s) = , Λ( n ) =
ns 0 sinon.
n=1

(Λ(n) s’appelle la fonction de Mangoldt). Montrer l’identité ζ (s)Z (s) = −ζ (s) pour tout
s ∈ D.
3/ a) Montrer l’existence de M0 > 0 et de M1 > 0 tels que
∀σ ∈ [1, 2], ∀t ∈ R, |t| ≥ 2, |ζ (σ + it)| ≤ M0 log |t|, |ζ  (σ + it)| ≤ M1 log 2 |t|
b) Montrer que si p > 1,
+∞
 cos(nτ log p)
∀s ∈ D, s = σ + iτ, log |1 − p−s | = − . (∗∗)
n=1
np nσ
Après avoir montré 3 + 4 cos θ + cos 2θ ≥ 0 pour θ ∈ R, démontrer ensuite l’inégalité
∀σ > 1, ∀τ ∈ R, |ζ (σ )| 3|ζ (σ + iτ )| 4|ζ (σ + 2iτ )| ≥ 1.
c) En déduire que ζ (s) ne s’annule pas sur D {1}. Montrer ensuite l’existence de m0 > 0
tel que
m0
∀σ ∈ [1, 2], ∀t ∈ R, |t| ≥ 2, |ζ (σ + it)| ≥
log7 |t|
(considérer séparément les cas σ ≥ σt = 1 + ε/ log 9|t| et 1 ≤ σ < σ t. Pour ce dernier,
σ
utiliser 3/b) et l’inégalité |ζ (σ + it)| ≥ |ζ (σt + it)| − σ t |ζ (x + it)| dx).
d) Montrer que la fonction F (s) = ζ (s) − Z (s) est prolongeable en une fonction continue
sur D tout entier, et qu’il existe M F > 0 et A > 0 tels que
∀σ, t : 1 ≤ σ ≤ 2, |t| ≥ A, |F (σ + it)| ≤ MF log 9 |t|.

Solution. 1/ a) Pour σ > 1, notons Dσ = {z ∈ C | (z ) ≥ σ}. Pour s ∈ Dσ avec σ > 1,


on a |1/ns| ≤ 1/nσ , donc la série converge normalement sur Dσ . Ainsi, ζ (s) est bien définie et
continue sur Dσ , et ceci est vrai pour tout σ > 1 donc ζ (s) est bien définie et continue sur D.
L’inégalité |ζ (σ + it)| ≤ σ/ (σ − 1) s’obtient en écrivant
+∞
 1   n dt
+∞  +∞
dt 1 σ
|ζ (σ + it)| ≤ 1 + σ
≤1+ σ
=1+ σ
=1+ = .
n=2
n n=2 n−1
t 1 t σ − 1 σ −1

b) Une intégration par parties donne l’égalité, pour n ∈ N∗ et s ∈ D


 n+1    n+1  n+1
1/2 − x + n 1/2 − x + n n+1 dx 1 1 dx
s s+1
dx = − s
− s
= s
+ s
− .
n x x n n x 2( n + 1) 2n n xs
Par sommation pour n allant de N à +∞ on obtient
 +∞ +∞ 
   +∞ N
1/2 − {x} 1 1 dx 1 1 N 1−s
s s+1
dx = s
+ s
− s
= ζ (s ) − s
+ s
− .
N x 2n 2(n + 1) N x n=1
n 2N s−1
n=N
1. PRÉLIMINAIRES 439

c) L’identité précédente utilisée avec N = 1 donne


 +∞
1 1 1/2 − {x}
∀s ∈ D, η(s) = ζ (s) − = +s dx. (∗∗∗)
s−1 2 1 x s+1
L’intégrande de la dernière intégrale est continue par rapport à s sur D et elle est majorée en
valeur absolue sur D par x → 1/(2x 2 ), intégrable sur [1, +∞[. Donc l’intégrale est une fonction
continue de s sur D et η (s) est bien prolongeable en une fonction continue sur D, d’où le résultat.
d) Il suffit de montrer que η(s) existe et est continue sur D . Soit s = σ + it ∈ D . Dans l’intégrale
de l’expression (***) de η (s), l’intégrande (1/2 − {x})/x σ+1+it est continûment dérivable par
rapport à σ, sa dérivée est −(1/2 − {x}) log x/(xσ+1+it) qui est majorée en valeur absolue par
log x/x2 lorsque s = σ + it ∈ D . Cette dernière fonction est intégrable sur [1, +∞[, l’hypothèse
de domination est donc vérifiée. On en déduit que η (s) existe et est continue sur D.
+∞ −ns
2/ a) On remarque déjà que pour tout k et pour tout s ∈ D , (1 − p−s k )
−1 =
n=0 pk .
Fixons m, M ∈ N∗ . Pour tout s = σ + it ∈ D , l’unicité de la décomposition d’un entier en
facteurs premiers implique
 
m M
   1
 1  1
ik s
= i1 im s
= s
,
i =0 (pk ) (p1 · · · pm ) n
k=1 k 0≤i1 ,...,im ≤M n∈F m,M

où Fm,M désigne l’ensemble des entiers n ∈ de la forme n = pi11 · · · pimm avec 0 ≤ ik ≤ M .
N∗
Maintenant, donnons nous N ∈ N∗ . Soit pm0 le plus grand nombre premier pi et M 0 la plus
grande des puissances ik apparaissant dans toutes les décompositions en facteurs premiers des
N premiers entiers 1, . . . , N. Considérons m ≥ m 0 et M ≥ M0 . Tous les entiers compris entre 1
et N sont dans Fm,M donc
    
 m M   
   1    1   1
ζ (s) −  
 =  ≤ .
 ik s   s  σ
 k=1 i k=0
(pk )   n∈N∗ ,n∈F m,M  n>N n
n

Cette expression est valable pour tout M ≥ M 0 . En faisant tendre M vers +∞, on en déduit
    

 m
 1    m
 +∞
 1   1
 
ζ (s) −  = ζ (s ) − ≤ .
 1 − p−s
k
  
(pik ) s  nσ
k=1 k=1 ik =0 k n>N
 σ
Comme on peut choisir N pour rendre n>N 1/n arbitrairement petit pour σ ≥ σ 0 > 1 et que
l’inégalité précédente vaut pour tout m ≥ m0 , on en déduit que le produit infini de l’identité
d’Euler converge uniformément vers ζ (s) sur Dσ0 dès que σ 0 > 1. Ceci étant vrai pour tout
σ0 > 1 on en déduit que l’identité d’Euler vaut sur D tout entier.
b) Fixons s = σ + it ∈ D. Remarquons d’abord que 1 − p −s
k est non nul pour tout k et que

1 − p−σ −s −σ
k ≤ |1 − p k | ≤ 1 + pk donc log(1 − p −σ −s −σ
k ) ≤ log |1 − p k | ≤ log(1 + pk ).

On en déduit, lorsque k → ∞, l’estimation log |1 − p−s k | = O(p−σ


k ) = O (k
−σ
), donc la série
 +∞
log |1 − p k | converge. Si on note  sa limite, ceci signifie que k=1 1/|1 − p−s
−s
k | = exp(−) est
non nul. On en déduit que ζ (s) = 0 d’après l’identité d’Euler.

c) Pour σ > 1 fixé et s ∈ Dσ , la majoration |Λ(n)/ns | ≤ log n/n σ montre que la série Λ(n)/n s
converge normalement sur ce domaine. On peut même écrire
+∞
 +∞ +∞ +∞
Λ(n)   log pk  p−s log pk
k
Z (s) = = = ,
n=1
ns j=1
k=1
(pj s
k ) 1 − p−s
k k=1

et cette dernière série converge normalement vers Z (s) sur Dσ .


On considère maintenant pour tout n ∈ N∗ , le produit fini
n
 1
Pn (s) =
k=1
1 − p−s
k
440 C. THÉOR ÈME DES NOMBRES PREMIERS

dont la dérivée vérifie


 
n −s n
   1  (−pj log pj )  p −s
j log pj
Pn(s) =  −s  −s 2 = −Pn (s ) .
j=1 1≤k≤n
1 − pk (1 − pj ) j=1
1 − p −s
j
k=j


Nous avons vu plus haut que pour σ > 1 fixé, la série j p−s −s
j log p j /(1 − pj ) converge normale-
ment sur Dσ . Comme de plus P n (s) converge uniformément vers ζ (s) sur D σ (voir 2/a)) et que
|ζ (s)| ≤ σ/(1 − σ) est bornée sur ce domaine, on en déduit que P n(s) converge uniformément
sur Dσ , donc forcément vers ζ (s). En faisant tendre n vers l’infini dans la dernière identité on
obtient
 p−s
+∞
j log p j
ζ  (s) = −ζ (s) = −ζ (s)Z (s).
j=1
1 − p−s
j

3/a) Commençons par montrer l’estimation sur ζ (σ + it), pour 1 ≤ σ ≤ 2 et |t| ≥ 2. L’identité
(*) entraı̂ne
N 

     1−s   −s 
1  +∞     N 
|ζ (s)| ≤    1  dx +  N  +  
 n s  + |s |  2xs+1   s − 1  2 
n=1 N
N   
 1 +∞
1  1  1
≤ + |s | 
dx +   + ≤ (1 + log N ) + |s| + 1.
n=1
n N 2x 2 s−1 2 2N

En choisissant N = [|t|] +1, on en déduit |ζ (σ + it)| ≤ 2 +log(|t| +1)+2|t|/2N ≤ 4+ log |t|. Ainsi,
on a montré ζ (σ + it) = O(log |t|) uniformément pour 1 ≤ σ ≤ 2 et |t| ≥ 2, d’où la première
estimation.
On s’y prend de manière similaire pour l’estimation de ζ(s). On commence par dériver
l’identité (*) (nous avions montré que nous pouvions bien dériver sous le signe intégral en 1/d)),
ce qui donne, pour tout N ∈ N ∗
N  +∞
 log n 1/2 − {x} N 1−s log N N 1−s N−s log N
ζ (s) = − + (1 − s log x ) dx − − + .
n=1
ns N x s+1 s−1 (s − 1)2 2

Lorsque s = σ + it avec 1 ≤ σ ≤ 2 et |t| ≥ 2, l’expression précédente donne la majoration

N  +∞
log n 1 + |s| log x log N 1 log N
|ζ  (s)| ≤ + 2
dx + + +
n=2
n N 2x 2 4 2N
 N 
log x 1 |s| +∞ log x 1
≤ dx + + 2
dx + log N + .
1 x 2N 2 N x 4
 
Or 1N log x/x dx = 12 log 2 N et N+∞ log x/x2 dx = (1 + log N )/N , donc finalement

log2 N 1 |s| 1 + log N 1


|ζ(s)| ≤ + + + log N + .
2 2N 2 N 2
En choisissant N = [|t|] + 1, on voit que ceci entraı̂ne |ζ (s)| = O(log 2 |t|) uniformément pour
1 ≤ σ ≤ 2.
b) La première identité est tout simplement une conséquence de la définition du logarithme
complexe, mais cette notion n’est pas aux programmes des classes  préparatoires. On s’en tire
autrement. Fixons θ ∈ R et considérons la série entière f θ(x) = +∞ n
n=1 cos(nθ )x /n, dont le rayon
de convergence est R = 1. Sa dérivée se calcule explicitement
+∞
  
eiθ cos θ − x
∀x ∈ ] − 1, 1[, fθ(x) = cos(nθ )x n−1 =  = .
1 − xeiθ 1 − 2x cos θ + x2
n=1
1. PRÉLIMINAIRES 441

Ainsi, comme fθ (0) = 0, on a


 x  x
 cos θ − t 1
fθ (x) = f θ (t) dt = 2
dt = − log(1 − 2x cos θ + x2 ) = − log |1 − xeiθ |.
0 0 1 − 2t cos θ + t 2

On en déduit l’identité (**) en choisissant x = p−σ et θ = τ log p.


Comme 3 + 4 cos θ + cos 2θ = 2(1 + cos θ )2 , la première expression est toujours positive pour
θ ∈ R. Ceci entraı̂ne, lorsque p est un nombre premier et σ > 1, que l’expression
+∞
 3 + 4 cos(nτ log p) + cos(2nτ log p)
3 log |1 − p−σ | + 4 log |1 − p −σ−iτ | + log |1 − p−σ−2iτ | = −
n=1
np nσ

est toujours négative, donc |1 − p−σ |3 · |1 − p−σ−iτ |4 · |1 − p −σ−2iτ | ≤ 1. Compte tenu de l’identité
d’Euler (question 2/a)) appliquée à ζ (σ), ζ (σ + iτ ) et ζ (σ + 2iτ ), ceci entraı̂ne bien le résultat
attendu.
c) On a déjà démontré que ζ (s) ne s’annule pas sur D , donc il s’agit ici de montrer que ζ (s) ne
s’annule pas pour (s) = 1. Raisonnons par l’absurde et supposons ζ (1 + iτ ) = 0, avec τ = 0.
La dérivabilité en σ = 1 de l’application [1, +∞[ → C, σ → ζ (σ + iτ ), établi en 1/d), entraı̂ne
ζ (σ + iτ ) = O (σ − 1) lorsque σ → 1+. Par ailleurs, ζ (σ) = O (1/(σ − 1)) lorsque σ → 1 +. Comme
de plus ζ (σ + 2iτ ) = O(1), tout ceci entraı̂ne

|ζ (σ )| 3|ζ (σ + iτ )|4 |ζ (σ + 2iτ )| = O((σ − 1)−3 ) × O((σ − 1)4) × O (1) = O (σ − 1), σ → 1 +,

donc cette expression peut être rendue arbitrairement proche de 0, ce qui est incompatible avec
l’inégalité établie à la question précédente. Donc ζ (1 + iτ ) ne s’annule jamais.
Donnons maintenant une minoration de |ζ (σ +it)|. Soit ε > 0. L’inégalité établie à la question
précédente entraı̂ne, lorsque σt = 1 + ε/ log 9 |t| ≤ σ ≤ 2 et |t| ≥ 2
  1/4
 1  3/4 1/4
2M0 log1/4 |t| log7 |t|
  ≤ |ζ (σ )| 3/4|ζ (σ + 2it)| 1/4 ≤ σ (M0 log |2t|) ≤ = ,
 ζ (σ + it)  (σ − 1)3/4 (ε log−9 |t|)3/4 Aε 3/4
1/4
avec A = 1/(2M0 ). Lorsque 1 ≤ σ < σ t et |t| ≥ 2, on écrit
 σt
Aε 3/4 ε 3/4(A − ε 1/4M1 )
|ζ (σ + it)| ≥ |ζ (σt + it)| − |ζ (x + it)| dx ≥ 7
− (σt − σ )M1 log2 |t| = .
σ log |t| log7 |t|

En choisissant ε > 0 suffisamment petit, on peut supposer que m 0 = ε 3/4(A − ε 1/4M 1) est
strictement positif. Ainsi choisi, m0 vérifie |ζ (σ + it)| > m 0 / log7 |t| pour 1 ≤ σ ≤ 2 et |t| ≥ 2.
d) Il est immédiat que F , comme ζ et ζ , est prolongeable en une fonction continue sur D{1}.
Lorsque s ∈ D {1} on a, vu que Z (s) = −ζ  (s)/ζ (s),

1 η (s) − (s−11)2 η (s) + (s − 1)η  (s)


F (s) = η (s) + + 1
= η (s) + ,
s−1 η (s) + s−1 1 + (s − 1)η(s)

et le dernier terme converge lorsque s → 1 (vers 2η (1)) donc F est bien prolongeable en une
fonction continue sur D tout entier.
La majoration de |F (s)| est immédiate à partir des majorations démontrées précédemment
pour |ζ (s)| et |ζ  (s)| et de la minoration obtenue pour |ζ (s)|.
Remarque. L’expression (***) permet de définir ζ (s) pour (s) > 0 (sauf en s = 1).
Dans le cadre des fonctions analytiques (ce qui est le cas pour ζ (s)), ce prolongement
est unique. On peut même poursuivre le procédé (l’identité (***) est issue de la formule
d’Euler-Maclaurin au premier ordre, que l’on peut utiliser à un ordre supérieur) pour
prolonger la fonction ζ (s) sur C{1} tout entier.
442 C. THÉOR ÈME DES NOMBRES PREMIERS

Problème 2 (Formule de Perron). a) En utilisant le résultat de la question c) du


problème 17 page 190, montrer

 T σ+it  1 (x > 1)
1 x
∀x > 0, ∀σ > 0, P σ(x) = lim dt = 1/2 (x = 1)
2π T →+∞ −T σ + it  0 (0 < x < 1)

b) Montrer que
 +∞ 
1 x1+σ+it (x − 1) (x > 1)
∀x > 0, ∀σ > 0, Qσ (x) = dt =
2π −∞ (σ + it)(1 + σ + it) 0 (0 < x ≤ 1)

c) Soit (an)n≥1 une suite de nombres


+∞ complexes vérifiant, pour tout ε > 0, an = O (nε ). On
s
considère la fonction G(s) = n=1 a n/n de la variable complexe s (série de Dirichlet).
Montrer que G est définie et continue sur D = {z ∈ C | (z ) > 1} et que
 x  +∞ 1+σ+it 
1 x G(σ + it)
∀σ > 1, ϕ(y ) dy = dt, où ϕ(y ) = an .
0 2π −∞ (σ + it)(1 + σ + it) 1≤n≤y

Solution. a) En coupant l’intégrale en deux pour se ramener sur l’intervalle [0, T ], on trouve
 T σ+it  T  σ+it   T
x x xσ−it σ 2σ cos(t log x) + 2t sin(t log x)
dt = + dt = x dt
−T σ + it 0 σ + it σ − it 0 σ 2 + t2
 σT  σT 
cos(σu log x) u sin(σu log x)
= 2x σ du + du
0 1 + u2 0 1 + u2
où on a effectué le changement de variable t = σu. Lorsque T → +∞, les deux dernières
intégrales convergent respectivement vers les intégrales C (σ log x) et S (σ log x) de la question c)
du problème 17 page 190. On en déduit que Pσ (x) existe bien et que

Pσ (x) = (C (σ log x) + S (σ log x)).
π
Les valeurs de C (v ) et S (v ) sont connues avec la question c) du problème 17 page 190. Elles
entraı̂nent C (v ) + S (v ) = πe −v si v > 0, C (0) + S (0) = π/2 et C (v) + S (v ) = 0 si v < 0.
σ
On en déduit que si x > 1 alors σ log x > 0 donc Pσ (x) = xπ (πe−σ log x ) = 1, si x = 1,
Pσ (x) = π1 (π/2) = 1/2, et si 0 < x < 1 alors Pσ (x) = 0.
b) L’idée est de montrer que Qσ (x) = Pσ (x). L’hypothèse de domination n’étant pas vérifiée
pour l’intégrande dérivée, on procède autrement. On considère pour tout n ∈ N∗
 n
1 x1+σ+it
fn (x) = dt.
2π −n (1 + σ + it)(σ + it)
L’intégrande est continûment dérivable par rapport à x donc fn (x) est dérivable et
 n σ+it
 1 x
fn (x) = dt.
2π −n σ + it
On intègre par parties en supposant x > 0 et x = 1 ce qui donne
 in  n 
 xσ x x−in ixit
f n (x) = − + 2
dt .
2iπ log x σ + in σ − in −n (σ + it)
σ  +∞ it
Comme Pσ (x) = limn→∞ f n(x) on en déduit P σ (x) = 2iπxlog x −∞ (σix +it) 2 dt, et donc
   −n  +∞ 
 xσ  2 1 1 2xσ

|fn(x) − Pσ (x)| ≤   + dt + dt = .
2π log x  n −∞ t
2
n t2 πn| log x|
Cette inégalité entraı̂ne la convergence uniforme de (fn ) vers Pσ sur tout segment de la forme
[a, b] avec 0 < a < b < 1 ou 1 < a < b. On en déduit que Qσ , limite simple de (f n ), est dérivable
sur tout segment de cette forme, donc sur ]0, 1[ ∪ ]1, +∞[ et que Qσ = Pσ sur cet intervalle.
2. PREUVE DU TH ÉOR ÈME DES NOMBRES PREMIERS 443

Maintenant on a Qσ (x) = O (x1+σ ) donc limx→0,x>0 Qσ (x) = 0 et comme Qσ (x) = P σ(x) = 0
pour 0 < x < 1, on en déduit Q σ (x) = 0 pour 0 < x < 1. Comme Qσ est continue sur R+ (son
intégrande est continue et majorée en valeur absolue par une fonction intégrable lorsque x est
dans tout segment inclus dans ]0, +∞[), on a aussi Qσ (1) = 0. Comme de plus Qσ (x) = P σ(x) = 1
pour x > 1, le théorème des accroissements finis entraı̂ne Qσ (x) = x − 1 pour x > 1.
c) Soit δ > 0. Par hypothèse, il existe M > 0 tel que |a n | ≤ M nδ/2 pour tout n ∈ N ∗ , donc pour
s 1+δ/2
tout
 s ∈s D1+δ = {z ∈ C | (z ) ≥ 1 + δ} on a |an /n | ≤ M/n . Ainsi, la série de fonctions
a n/n converge normalement sur D 1+δ, elle est donc continue (et bornée) sur ce domaine.
Ceci étant vrai pour tout δ > 0 on en déduit que G(s) est bien définie et continue sur D.
Montrons la formule de Perron. Soit σ > 1. Comme an/n s converge normalement sur Dσ ,
en particulier sur (s) = σ , on peut inverser les signes de sommation ce qui donne
 +∞ 1+σ+it +∞  +∞  x
1 x G(σ + it) 1  +∞ x1+σ+it an /n σ+it 
dt = dt = na n Qσ .
2π −∞ (σ + it)(1 + σ + it) 2π −∞ (σ + it)(1 + σ + it) n
n=1 n=1
Compte tenu de la valeur de Qσ (x) obtenue à la question précédente, ceci entraı̂ne
 +∞ 1+σ+it 
1 x G(σ + it)
dt = (x − n)an . (∗)
2π −∞ (σ + it)(1 + σ + it)
1≤n≤x

Par ailleurs, pour 0 ≤ y ≤ x on peut écrire ϕ(y) = 1≤n≤x an χ n(y) où χn(y) = 0 si y < n,
χn(y ) = 1 pour y ≥ n. Ceci montre que
x   x   x 
ϕ(y ) dy = anχ n(y ) dy = an dy = (x − n)an .
0 1≤n≤x 0 1≤n≤x n 1≤n≤x

Grâce à l’égalité (*) on en déduit la formule de Perron.

2. Preuve du Théorème des nombres premiers


Nous sommes maintenant armés pour démontrer le théorème des nombres premiers.
Dans le problèmesous-suite
ci dessous, on utilisera les résultats établis dans les deux problèmes
précédents.
Problème 3 (Théor ème +∞des nombress premiers). 1/a) On considère la fonction
F (s) = Z (s) − ζ (s) = n=1 (Λ(n) − 1)/n , que nous avons définie dans le problème 1
page 437. Nous avons vu que F , définie et continue sur D , est prolongeable en une fonction
continue sur D tout entier. Montrer l’identité
 x  
x2 +∞ F (1 + it)x it
∀x > 1, (ψ (y ) − [y]) dy = dt, où ψ (y ) = Λ(n).
0 2π −∞ (1 + it)(2 + it) 1≤n≤y
x
b) En déduire 0 ψ (y ) ∼ x 2/2 lorsque x → ∞.
c) Montrer que ψ (x) ∼ x lorsque x → ∞.
2/a) Pour alléger les notations, lorsque la lettre p apparaı̂tra dans un indice ceci signifiera
forcément que p est un nombre premier. Montrer que
  log x 
ψ (x) = log p,
p≤x
log p

puis montrer que π (x) = p≤x 1 (nombre de nombres premiers inférieurs à x) vérifie
ψ (x) ≤ π (x) log x ≤ 2ψ (x).
b) Démontrer que π (x) = ψ (x)/ log x + O (x/ log2 x). En déduire le théorème des nombres
premiers : π (x) ∼ x/ log x.
444 C. THÉOR ÈME DES NOMBRES PREMIERS

Solution. 1/a) Nous sommes dans les hypothèses de la formule de Perron établie au problème
précédent, on en déduit que pour tout σ > 1 on a
 x x   +∞
1 F (σ + it)x1+σ+it
(ψ (y ) − [y]) dy = (Λ(n) − 1) dy = dt. (∗)
0 0 2π −∞ (σ + it)(1 + σ + it)
1≤n≤y

La borne sur |F (σ + it)| pour |t| suffisamment grand, établie à la fin du problème 1, entraı̂ne
qu’il existe M > 0 tel que |F (σ + it) ≤ M log9 (2 + |t|) pour 1 ≤ σ ≤ 2 et pour tout t ∈ R. On
en déduit que pour tout x > 1 on a
 
 F (σ + it)x1+σ+it  9 3
∀σ ∈ [1, 2], ∀t ∈ R,   ≤ M log (2 + |t|)x .
(σ + it)(1 + σ + it)  1+t 2

La dernière fonction étant intégrable sur R, l’hypothèse de domination est bien vérifiée donc
l’intégrale de droite dans (*) est bien une fonction continue de σ ∈ [1, 2], en particulier en σ = 1.
L’égalité (*) a donc lieu également lorsque σ = 1, d’où le résultat.
b) Posons G(t) = F (1 + it) × (1 + it) −1 × (2 + it)−1, fonction continue et intégrable sur R. La
formule établie à la question précédente s’écrit
 x  +∞
2 1
(ψ (y ) − [y]) dy = x FG (log x), F G (α) = G(t)e iαt dt.
0 2π −∞

D’après le lemme de Riemann-Lebesgue (voir l’exercice 6 page 157), on a limα→+∞ F G (α) = 0,
x x
donc
 x ceci entraı̂
 x ne 0 (ψ (y ) − [y])dy = o(x2 ). Comme y − 1 < [y ] ≤ y on a 0 (y − 1) dy ≤
0
[y ] dy ≤
 x 0 y dy d’où on déduit 0x[y ] dy = x 2 /2 + o(x2 ), et finalement 0x ψ (y ) dy = 0x (ψ (y ) −
[y]) dy + 0 [y ] dy = x /2 + o(x2 ).
2

c) Soit ε > 0 (ε < 1/2). De ce qui précède, on déduit, la fonction ψ étant croissante
 x  x(1+ε)
x2 x 2 (1 − ε)2 2 x2 (1 + ε)2 x2
− + o(x ) ≤ ψ (y ) dy ≤ εxψ (x) ≤ ψ (y ) dy ≤ − + o(x 2)
2 2 x(1−ε) x 2 2
d’où, en divisant par εx on tire (1 − ε/2)x + o(x) ≤ ψ(x) ≤ (1 + ε/2)x + o(x). Comme on
peut avoir les petits o(x) plus petits que ε2 x en valeur absolue pour x suffisamment grand, ceci
entraı̂ne (1 − ε)x ≤ ψ (x) ≤ (1 + ε)x pour x assez grand.
2/a) On peut écrire
 
 
ψ (x) =  log p  ,
p≤x k≥1,p k ≤x

et comme le nombre d’entiers k ≥ 1 tels que pk ≤ x est égal à [log x/ log p], on en déduit la
formule voulue pour ψ (x).
Pour tout nombre réel u ≥ 1, l’inégalité [u] ≤ u < [u] + 1 ≤ 2[u] entraı̂ne [log x/ log p] log p ≤
log x ≤ 2[log x/ log p] donc ψ (x) ≤ π (x) log x ≤ 2ψ (x).
b) On a déjà montré 0 ≤ π (x) log x − ψ (x), majorons maintenant cette quantité. On a
 
log x
    
log x
    
x
π (x) log x−ψ (x) = log x − log p = log x − log p + log
p≤x
log p √ log p √ p
p≤ x x<p≤x

car
 lorsq ue x < p ≤ x on a 1 ≤ log x/ log p < 2 donc [log x/ log p] = 1. Maintenant l’inégalité
log x
log p
≥ log x
log p
− 1 et le fait que π (n) − π(n − 1) = 1 si n est premier, = 0 sinon, entraı̂ne
   x
π (x) log x − ψ (x) ≤ log p + (π (n) − π (n − 1)) log ,
√ √ n
p≤ x x<n≤x

et grâce à une transformation d’Abel sur la dernière somme, on obtient


  
√ √ x x x
π (x) log x − ψ (x) ≤ x log x + π (n) log − log + π (x) log .
√ n n + 1 [ x]
x<n≤x−1
3. HISTOIRE DU TH ÉORÈME DES NOMBRES PREMIERS 445

Pour x ≥ 1 on a log(x/[x]) < log(2), et on sait que π(x) ≤ 2ψ (x)/ log(x) = O(x/ log x). Par
 n+1
ailleurs log xn − log n+1
x
= log n+1
n = n dt/t, donc tout ceci entraı̂ne
  n+1    x  
dt x π (t) x
π (x) log x − ψ (x) ≤ π (n) +O ≤ dt + O .
√ n t log x 1 t log x
x<n≤x−1
x x
Comme π (x) = O (x/ log x), on a 1 π (t)/t dt = O ( 1 dt/ log t) = O (x/ log x) (voir le théorème 3
page 163), donc on a finalement démontré 0 ≤ π (x) log x − ψ(x) ≤ O (x/ log x). Ainsi, on a bien
π (x) = ψ (x)/ log(x) + O (x/ log2 x). On en déduit facilement le théorème des nombres premiers
car ψ (x) = x + o(x) donc π (x) = x/ log x + o(x/ log x).

3. Histoire du Théorème des nombres premiers


L’histoire du théorème des nombres premiers est exemplaire, à bien des égards, du
développement des mathématiques.
Les premières considérations relatives à l’ensemble des nombres premiers se trouvent
dans les livres d’arithmétique d’Euclide, où il est montré qu’il y a une infinité de nombres
premiers. Il faut attendre vingt et un siècle pour qu’un nouveau résultat tangible sur
ce sujet apparaisse : en 1737, Euler découvre le développement de la fonction zêta (aux
nombres entiers) en produit infini faisant intervenir les nombres premiers, et il en déduit
que la somme des inverses des nombres premiers diverge :
1 1 1 1 1
+ + + + + · · · = +∞.
2 3 5 7 11
Ceci entraı̂ne que l’ensemble des nombres premiers est infini (voir également le problème 22
page 302). Euler venait ainsi de démarrer la théorie analytique des nombres. Il remarque
également que les nombres premiers sont infiniment moins nombreux que les entiers, ce
qu’on exprime sous la forme π (x) = o(x). En 1808, Legendre observe empiriquement
l’estimation plus précise
x
π (x) ∼ ,
log x
puis Gauss, à la même époque, à partir de données statistiques, remarque que l’approxi-
mation x
dt
π (x) ≈ Li(x), Li(x) =
1 log t
est meilleure. L’approximation de Gauss n’est pas incompatible avec l’estimation de
Legendre, et est effectivement plus précise : par exemple, pour x = 1010 on sait que
π (x) = 455 052 511, et x/ log(x)  455 743 003 et Li(x)  455 055 613.
Jusque là, les estimations proposées restent empiriques. En 1852, Tchébychev démontre
que si π (x)/(x/ log(x)) converge alors sa limite est forcément 1. Il n’obtient donc pas
l’équivalent π (x) ∼ x/ log x mais il démontre que pour x suffisamment grand, on a
x x
0, 921 < π(x) < 1, 105 .
log x log x
Une nouvelle étape est franchie en 1859, lorsque Riemann a l’idée géniale de s’intéresser
à la fonction ζ (s) dans le domaine complexe. Il ne parvient pas à démontrer le théorème des
nombres premiers mais remarque que la distribution des nombres premiers est étroitement
liée aux zéros de ζ (s) et conjecture l’hypothèse de Riemann, qui exprime que les seuls zéros
de ζ (s) dans la bande 0 ≤ (s) ≤ 1 sont sur la droite (s) = 1/2 (pour rappel, on peut
prolonger ζ (s) au delà de (s) > 1, en utilisant par exemple la formule (***) de la
solution du problème 1). Cette conjecture n’est toujours pas démontrée ou infirmée et
reste aujourd’hui un des problèmes ouverts les plus célèbres des mathématiques. On sait
446 C. THÉOR ÈME DES NOMBRES PREMIERS

depuis 1901 que si cette hypothèse est vraie, alors l’approximation de π (x) par Li(x) est
très précise et vérifie
π (x) = Li(x) + O (x1/2 log x).
Ainsi, l’hypothèse de Riemann est reliée à la régularité de la répartition des nombres
premiers.
Dans la continuité des travaux de Riemann, en 1896, Hadamard et La Vallée-Poussin
démontrent indépendamment et presque en même temps le théorème des nombres pre-
miers. Leur preuve (de même nature que celle proposée dans cette annexe) repose es-
sentiellement sur la démonstration que ζ (s) ne s’annule pas sur la droite (s) = 1.
Cette preuve du théorème des nombres premiers accélère le développement de la théorie
analytique des nombres dont elle est issue. On a longtemps pensé que le théorème des
nombres premiers était profondément lié aux méthodes de l’analyse complexe et qu’il
était impossible de le démontrer en dehors de ce cadre. Ces considérations furent rendues
caduques lorsqu’en 1949, Erdös et Selberg produisirent la première preuve élémentaire
(au sens ou elle n’utilise pas la théorie de l’analyse complexe) du théorème des nombres
premiers. Bien qu’élémentaire, la preuve est très difficile. Pour ses travaux, Selberg a
obtenu la médaille Fields en 1950 (la médaille Fields a été crée en 1936 après une pro-
position d’un mathématicien canadien John Fields, pour pallier l’absence de prix No-
bel de mathématiques ; elle est décernée tous les quatre ans. Depuis 2003, le prix Abel
récompense également les mathématiciens et est décerné annuellement).
L’hypothèse de Riemann. L’hypothèse de Riemann, qui exprime que les seuls zéros de
ζ (s) de la bande 0 ≤ (s) ≤ 1 sont sur la droite (s) = 1/2 a fait l’objet de nombreuses
études. La première avancée fut celle obtenue par Hadamard et La Vallée-Poussin, qui
montrèrent que ζ (s) ne s’annule pas sur (s) = 1. En 1900, Hilbert inclut l’hypothèse
de Riemann dans sa célèbre liste de 23 problèmes non résolus. En 1914, Hardy montra
qu’il y a une infinité de zéros de ζ (s) sur la droite critique (s) = 1/2. Selberg prouva en
1932 qu’il y a une proportion non nulle de zéros sur la droite critique. Le meilleur résultat
est celui de Conrey qui en 1989 prouva qu’au moins 40,219% des zéros sont sur la droite
critique.
De très nombreux mathématiciens, et même parfois les plus éminents d’entre eux,
ont proposé une preuve de l’hypothèse de Riemann. Jusqu’à présent (2019), toutes se
sont avérées fausses, ou pas dignes d’intérêt pour être étudiées par la communauté des
mathématiciens. En 2000, le Clay Mathematics Institute proposa un million de dollars
pour qui démontrerait ou infirmerait l’hypothèse de Riemann.
On a également cherché à calculer numériquement les zéros de ζ (s) pour vérifier qu’ils
sont bien sur la droite critique. On sait aujourd’hui que les 10 13 premiers zéros sont bien
sur la droite critique, mais ceci n’apporte rien à la preuve. Il suffirait d’en trouver un en
dehors de cette droite pour démontrer qu’elle est fausse !
Index des notations

Les numéros en fin de ligne indiquent les pages où apparaissent les notations corres-
pondantes.

x norme du vecteur x, 7 


xα norme définie par (x1, . . . , xn ) = ( i |xi | α)1/α, 8, 98
d(x, y ) distance du point x au point y dans un espace métrique, 8
δ (A) diamètre de la partie A, 8
d(A, B ) distance entre les parties A et B , 8
inf x∈A f (x) borne inférieure de f (x) pour x ∈ A, 8
sup x∈A f (x) borne supérieure de f (x) pour x ∈ A, 8
AB l’ensemble A privé de B (= {x ∈ A | x ∈ B }), 8
B(x, ρ) boule ouverte de centre x et de rayon ρ, 9
Bf (x, ρ) boule fermée de centre x et de rayon ρ, 9
S(x, ρ) sphère de centre x et de rayon ρ, 9
A adhérence de A, 10

A intérieur de A, 10
Fr(A), ∂A frontière de A, 11
g◦f composée des applications f et g : x → g (f (x)), 11
f −1 si f est bijective, application réciproque de f , 12
f −1 (Y ) image réciproque par f de l’ensemble Y (= {x | f (x) ∈ Y }), 14
limxx→∈Aa f (x) =  limite de f en a selon A, 15
f (x−), f (x+) limite à gauche, à droite, d’une fonction f au point x, 15
IdE application identité de E , 22
C(X, K) algèbre des applications continues de X dans K, 38
Ker ϕ noyau de l’application linéaire ϕ, 38
A complémentaire de l’ensemble A, 43
L(E, F ) e.v des applications linéaires de E dans F , 48
L c(E, F ) e.v des applications linéaires continues de E dans F , 48
|||f ||| norme intrinsèque de f ∈ Lc(E, F ), 48
L c(E) e.v des endomorphismes continus de E , 48
E dual topologique de E , 49
Gc (E) algèbre des endomorphismes inversibles et bicontinus de E , 50
exp(u) exponentielle de u ∈ Lc (E), 50
[A, B] segment joignant les points A et B , 51
Conv(A) enveloppe convexe de l’ensemble A, 51
448 INDEX DES NOTATIONS

Card I cardinal de l’ensemble fini I , 54


inf Γ, sup Γ borne inférieure, supérieure, de Γ ⊂ R, 54
Vect(ei )i∈I sous-suite
sous espace vectoriel engendré par les vecteurs ei , i ∈ I , 56
f (a) dérivée de la fonction f au point a, 71
fg(a), f d (a) dérivée à gauche, à droite, de la fonction f au point a, 71
f (n) dérivée n-ième de f , 72
C n(I, E) ensemble des fonctions de classe C n de I dans E , 72
Ckn coefficient binomial, égal à k!(nn!−k )! , 72
log x logarithme népérien de x > 0, 73
det M déterminant de la matrice carrée M , 85
f (x) = O (g (x)) f est un “grand o” de g , 87
f (x) = o(g (x)) f est un “petit o” de g , 87
f (x) ∼ g (x) f et g sont équivalentes, 87
C m([a, b], E) e.v des fonctions continues par morceaux de [a, b] dans E , 98
b
a f (t) dt intégrale de la fonction f entre a et b, 124
f (t) dt l’une des primitives de f , 136
L 1c(I, E) e.v des fonctions continues et intégrables sur I , 150
2
L c(I, E)
 +∞ e.v des fonctions continues et de carrés intégrables sur I , 150
a
f (t) dt intégrale généralisée de f sur ]a, +∞[, 151
Γ(x) fonction gamma, 162, 315
(z ), (z) partie réelle, partie imaginaire du nombre complexe z , 169, 386
(u
n ) suite de terme général u n, 199
+∞ un série de terme général
 un, 208
n=0 u n somme de la série un , 208
Hn nombres harmoniques : Hn = 1 + 12 + · · · + n1 , 211
 
n∈Z c n e
inx
désigne limN→+∞ N n=−N cn e
inx
(convention de Cauchy), 267
a n(f ), b n (f ), cn (f ) coefficients de Fourier d’une fonction f , 269
D notation du programme définie à la page 269
f (x−), f (x+) limite à gauche, à droite, d’une fonction f au point x, 269
ζ (s) fonction zêta, 302
f g convolée de f et g , 304
dfa différentielle de f au point a, 323
Mn (K) e.v des matrices n × n à coefficients dans K, 324
grada f gradient de f au point a, 324
fv(a) dérivée de f selon le vecteur v au point a, 324
∂f
∂xi
dérivée partielle de f d’indice i, 325
dx1, . . . , dx n base duale de la base canonique de Rn , 325
∆f laplacien de f , 327, 338
 [n]
q ∂f
h
i=1 i ∂xi (a) puissance symbolique n-ième, 328
Gn (K) groupe des matrices inversibles de Mn (K), 331
tr M trace de la matrice carrée M , 333
∂ if différentielle partielle de f d’indice i, 344
x, y  produit scalaire de x et y , 349
χ
A fonction caractéristique de A, 351
∂K +
α intégrale curviligne de α sur le bord orienté ∂K + , 358
wronskien(V1, . . . , V n) wronskien de n solutions d’une équation différentielle, 378
Index

abaissement de l’ordre (dans une équation Bieberbach, théorème de – (cas réel), 301
différentielle linéaire), 380 Bioche (règle de –), 139
Abel Bolzano-Weirstrass (théorème de –, propriété
règle d’–, 152, 215 de –), 28
théorème d’–, 263 Borel (théorème de réalisation de –), 295
transformation d’–, 215 Borel-Lebesgue (propriété de –), 27
accroissements finis bornée (partie –), 8
généralisés (théorème des –), 74 boule (ouverte, fermée), 9
inégalité des –, 76, 327 Brouwer (théorème de –), 52
théorème des –, 74
accumulation (point d’–), 11 Cantor
adhérence (d’une partie), 10 ensemble triadique de –, 63
adjacentes (suites –), 200 th. de – sur les séries trigonométriques, 312
algèbre normée, 14 Cantor-Lebesgue
analytique (fonction –), 266 escalier de –, 66
application théorème de –, 312
– ouverte (théorème de l’–), 423 caractéristique (fonction –), 351
application partielle, 14 Carathéodory (théorème de –), 54
arc, 42 Cauchy
arc géométrique, 357 critère de –
arc paramétré, 356 – pour les fonctions, 21
arithmético-géométrique – pour les suites, 20
inégalité –, 97 formule de –, 250
arithmético-géométrique problème de –, 374
moyenne –, 188 suite de –, 20
arithmético-géométrique Cauchy-Lipschitz (théorème de –), 374
suite –, 201 Césaro (moyenne de –), 203
chaleur (équation de la –), 369
Baire (espace de –), 417 changement de variable (th. du –), 127, 150, 354
Banach chemin, 42, 356
espace de –, 20, 21 Clairaut (équation de –), 392
théorème de –, 423 classe C n (fonction de –), 72
Banach-Steinhaus (théorème de –), 424 compact (espace –), 27
base hilbertienne, 428 compact (opérateur –), 432
Bernoulli compact à bord, 358
équations de –, 391 complet (espace métrique –), 20
nombres, polynômes de –, 319 complété (d’un espace métrique), 25
Bernstein concave (fonction –), 95
polynômes d’approximation de –, 242 connexe
Bernstein S., théorème de – – par arcs, 42
– sur les séries de Fourier, 278 – par lignes brisées, 42
– sur les séries entières, 261 composante –, 41
Bertrand espace – , 39
intégrales de –, 149 continue
séries de –, 212 – à gauche. à droite (fonction –), 16
Bessel (inégalité de –), 270, 428 application uniformément –, 12, 31
bêta (fonction –), 315 fonction –, 11
449
450 Index

continue par morceaux (fonction –), 98, 147 distances


continuité sous le signe intégral (théorème de – équivalentes, 12
–), 161 – topologiquement équivalentes, 12
contractante (application –), 21 – uniformément équivalentes, 13
convention de Cauchy, 267 dual topologique, 49
convergence
– absolue, 209 échelle de comparaison, 88
– normale, 232 enchaı̂né (espace bien –), 45
– simple, 231 enveloppe convexe, 51
– uniforme, 231 équation caractéristique
convergence absolue – d’une récurrence linéaire, 202
– des intégrales généralisées, 151 équation différentielle, 373
convergence dominée (théorème de –), 151, 194, équation différentielle
196 – linéaire, 377
convexe – non résolue, 374
ensemble –, 51 équation indicielle, 416
fonction –, 95, 365 équivalent (d’une fonction en un point), 88
fonction logarithmiquement –, 103 escalier (fonction en –), 98, 123, 351
convolution (produit de –), 304 espace métrique, 8
critère de Cauchy espace vectoriel normé (ou e.v.n), 7
– uniforme, 232 étoilée (partie –), 51
critique (point –), 336 Euler
cylindre de sécurité, 374 équation d’–, 384
constante d’–, 211
d’Alembert, théorème de –, 185 identité d’–, 302, 438
Darboux Euler-Cauchy (méthode d’–), 408
théorème de –, 47, 80 eulériennes (intégrales –), 315
dense (partie –), 10 Euler-Maclaurin (formule d’–), 321
dérivable (fonction –), 71 exponentielle
dérivable (fonction continue, jamais –), 86, 421 – complexe, 254
dérivation par rapport à la var. complexe, 249 – d’un endomorphisme continu, 50
dérivations, 364 extraite (suite –), 19, 199
dérivation sous le signe intégral (th. de –), 161 extremums
déri vée – liés, 337
– à points de discontinuités denses, 244 – relatifs, 335
– partielle, 325 faible (topologie – dans un Hilbert), 431
points de continuité d’une –, 419 Fejér (théorème de –), 306
dérivée selon Schwarz, 108 fermée (partie –), 9
développement asymptotique, 88 fermée (application –), 31
développement limité, 89 Floquet (théorème de –), 387
diamètre (d’une partie), 8 fonction entière, 248
difféomorphisme, 73, 343 fonctions strictement monotones (de R n), 350
différentielle, 323 forme différentielle de degré 1, 357
différentielle formule de la moyenne, 127, 128
– de l’inverse, 331 formule de Stirling, 219
– du déterminant, 332 Fourier
différentielle partielle, 344 coefficients de –, 268
Dini (théorèmes de –), 238 série de –, 269
Dirac (séquence de –), 304 Fréchet (espaces de –), 426
Dirichlet Fresnel (intégrale de –), 362
intégrale de –, 178 frontière (d’une partie), 11
noyau de –, 271 Fubini (théorème de –), 353
série de –, 442 Fuchs (théorème de –), 416
discontinuité de première espèce, 16, 99
disque de convergence, 247 gamma (fonction –), 162, 315
distance, 8 Gauss
distance intégrale de –, 167, 355
– discrète, 8 Gauss
– produit, 13 constante de –, 189
Index 451

gradient, 324 laplacien, 327, 338


Green-Riemann (théorème de –), 358 Leibniz (formule de –), 72
Gronwall (lemme de –), 397 L’Hospital (règle de –), 75
groupe topologique, 59 ligne brisée, polygonale, 42
limite
Hadamard (lemme d’–), 331 – d’une application en un point, 15
Hadamard (produit de – de deux séries – d’une suite, 19
entières), 254 Liouville (théorème de –), 259
Hardy (notation de –), 87 lipschitzienne (application –), 13
Hardy-Littlewood (th. taubérien de –), 308 logarithme intégral, 173
harmonique (fonction –), 333, 338 logarithmiquement convexe, 103
harmoniques (nombres –), 211
Hausdorff (distance de –), 61 Maclaurin (formule de –), 75
Heine (théorème de –), 31 maigre (partie –), 417
Helly (théorème de –), 245 Mangoldt, fonction de –, 438
hessienne (matrice –), 336 maximum (principe du –), 338
Hilbert (espaces de –), 427 mesurable (ensemble –, au sens de Riemann),
Hölder (inégalité de –), 97 352
holomorphe (fonction –), 249, 266, 333 mesure (d’un ensemble mesurable), 352
holonomes (équations –), 416 Minkowsky (inégalité de –), 97
homéomorphisme, 12, 31 morceaux
homogènes (équations –), 391 fonction C k par –, 269
homogènes (fonctions positivement –), 115 fonction continue par –, 98, 147
homographique (récurrence –), 201 Morse (lemme de –), 368
hypothèse de Riemann, 445 moyennes, 117
Müntz (théorème de –), 310
incomplètes (équations –), 390
induite (topologie –), 11 négligeable (partie –), 63
inégalité norme, 7
– de réarrangement, 229 norme
intégrable – de la convergence en moyenne, 126, 150
fonction –, 147 – de la convergence en moyenne quadratique,
fonction positive –, 147 126, 150
fonction Riemann–, 125 – de la convergence uniforme, 8, 126, 232
intégrale norme d’algèbre, 14
– absolument convergente, 151 normes équivalentes, 12
– convergente, 151 noyau de Dirichlet, 271
– curviligne, 357
– d’une fonction continue par morceaux, 124 oscillation d’une fonction, 62
– dépendant d’un paramètre, 161 ouverte (partie –), 9
– généralisée, ou impropre, 151 ouverte (application –), 12
– multiple, 351
intégration par parties, 127 Parseval (égalité de –), 250
intérieur (d’une partie), 10 partie élémentaire, 352
inverse d’une série entière, 262 partie principale, 88
inversion globale, 343, 344 partie simple, 352
inversion locale, 341, 343 pavé (de Rn ), 351
irrationalité de π 2 , 187 Péano (courbe de –), 112
isolé (point –), 11 Perron (formule de –), 442
phase stationnaire (méthode de la –), 173
jacobien, 327 Picard, théorème de –, 259
jacobienne (matrice –), 327 Pisot (nombres de –), 286
jacobien partiel, 345 point fixe, 21, 23
Poisson (formule sommatoire de –), 284
lacunaire (zéros d’un polynôme –), 105 polynôme trigonométrique, 267
Lagrange précompact, 29, 32
équations de –, 392 première formule de la moyenne, 127
multiplicateurs de –, 337 presque-périodiques (fonctions –), 120
Landau (notation de –), 87 primitive, 127
Laplace (méthode de –), 164 primitives (calcul de –), 136
452 Index

principe de majoration a priori, 399 somme partielle (d’une série), 208


principe (des tiroirs), 287 sous-groupes additifs de R, 205
problème de Cauchy, 374 sous-suite, 19, 199
procédé d’extraction diagonal, 32, 431 sphère, 9
produit de Cauchy Steinhaus (Banach–, théorème de –), 424
– de deux séries, 216 Stirling (formule de –), 166, 219
– de deux séries entières, 248 subdivision, 123
prolongement par continuité, 16 subdivision
pseudo-dérivée, 108 – pointée, 128
puissance symbolique n-ième, 328 pas ou module d’une –, 123
suite, 199
quadratique (convergence en moyenne –), 269 suite
– à récurrence homographique, 201
Raab-Duhamel (règle de –), 213
– à récurrence linéaire, 202
rayon de convergence, 247 – arithmético-géométrique, 201
réarrangement, inégalité de –, 229 – arithmétique, 201
récurrence linéaire, 202
– géométrique, 201
récurrente (suite –), 200
– récurrente, 200
réglée (fonction –), 98
support compact (fonction à –), 181
résiduel, 417
support (d’un arc paramétré), 356
résidus (théorème des –), 192
résolvante (d’un système différentiel), 403 taubérien (théorème –), 264, 308
reste (d’une série), 209 Taylor (formule de –)
Ricatti (équations de –), 391 avec reste intégral, 77
Riemann-Lebesgue (lemme de –), 157 Taylor (formules de –), 328
Riesz (théorème de –), 56 Taylor-Lagrange
Riesz (théorème de représentation de –), 427 formule de –, 75
Rolle (théorème de –), 73 inégalité de –, 76
Runge (phénomène de –), 240 Taylor-Young (formule de –), 77
théorème
Sarkowski, théorème de –, 111 – des nombres premiers, 437
Schwarz – des valeurs intermédiaires, 31, 41
inégalité de –, 126, 150 théorèmes d’oscillation et de comparaison, 395
théorème de –, 326 thêta (fonction –), 285
seconde formule de la moyenne, 128 Tietze-Urysohn (th. de prolongement de –), 66
segment, 42, 51 tiroirs (principe des –), 287
semi-convergentes (intégrales –), 151, 152 topologie (d’un espace métrique), 9
séparable (espace de Hilbert –), 428 topologique (espace –), 10
séparé (espace topologique –), 10 transformation d’Abel, 215
série
– arithmétique, 209 unité (boule, sphère), 9
– commutativement convergente, 216
– de Dirichlet, 442 valeur d’adhérence (d’une suite), 19
– de Fourier, 269 variation bornée (fonctions à –), 118
– entière, 247 variation de la constante (méthode de –), 379
– géométrique, 209 Viète (François –), 207
– numérique, 208 voisinage (d’un point), 9
– trigonométrique, 267 Von Staudt (théorème de –), 321
série entière d’un endomorphisme continu, 50 Wallis (intégrales de –), 130
Simpson (formule de –), 81 Weierstrass
solution maximale (d’une équation fonction de –, 114
différentielle), 374 théorème de –, 235, 242, 304
sommable théorème de Bolzano–, 28
famille –, 217 wronskien, 378, 388
fonction –, 147
fonction positive –, 147 Young (inégalité de –), 134
sommation
– par piles, 353 zéros des solutions d’une éq. diff., 376, 395
– par tranches, 354 zêta (fonction – de Riemann), 302, 437
sommes de Riemann, 128
X. GOURDON
Cette troisième édition rassemble dans un même volume des
rappels de cours complets, des compléments de cours, ainsi que
308 exercices et problèmes corrigés, classiques ou originaux, le tout
portant sur le programme d’analyse de mathématiques spéciales
MP*. Il complète le tome Algèbre de la même collection.
L’accent est porté sur la relation cours-exercice, indispensable
pour parvenir à une compréhension globale des concepts. Tous les
thèmes classiques sont présentés, expliqués, exploités et fournissent
ainsi un bagage mathématique solide pour affronter les concours
scientifiques. Tout au long de l’ouvrage, de multiples remarques et
renvois ponctuent les résultats et permettent à l’étudiant de trou-
ver des points de repère.

ANALYSE
Cette nouvelle édition contient des exercices et problèmes supplé -
mentaires dans l’esprit de l’édition précédente.
C’est l’outil idéal de l’élève des classes préparatoires scientifiques
pour la révision des concours.
Il pourra également intéresser les candidats à l’agrégation.

les maths en tête


Illustration de couverture : L’École d’Athènes, Raphaël, Vatican

-:HSMDOA=UX]Z[V:

Vous aimerez peut-être aussi